EMRCS and pas test MCQs - These MCQs prepared from only success-MRCS part A whatsApp group - September 2018

1. A 36-year-old man with end-stage renal disease who is undergoing haemodialysis has normocytic normochromic anaemia. Which of the following is the most appropriate therapy? A. Erythropoietin B. Ferrous sulphate C. Folate D. Vitamin B6 E. Vitamin B12

ANSWER IS A Erythropoietin, or EPO, is a glycoprotein hormone that is a cytokine for erythrocyte (red blood cells) precursors in the bone marrow. Also called haematopoietin or haemopoietin, it is produced by the kidney and is the hormone regulating red blood cell production.

Erythropoietin is available as a therapeutic agent produced by recombinant DNA technology in mammalian cell culture. It is used in treating anaemia resulting from chronic renal failure or from cancer chemotherapy. Its use is also believed to be common as a doping agent in endurance sports such as bicycle racing, triathlons and marathon running.

2. A blood sample taken from the umbilical artery of a newborn was subjected to electrophoresis to detect antibodies (immunoglobulins). Which of the following antibodies will have the highest percentage in a newborn? A. Ig A B. Ig D C. Ig E D. Ig G E. Ig M

ANSWER IS D IgG is a monomeric immunoglobulin, built of two heavy chains γ and two light chains. Each molecule has two antigen-binding sites.

This is the most abundant immunoglobulin and is approximately equally distributed in blood and in tissue liquids, constituting 75% of serum immunoglobulins in humans. This is the only isotype that can pass through the placenta, thereby providing protection to the newborn in its first weeks of life before its own immune system has developed.

It can bind to many kinds of pathogens, for example viruses, bacteria and fungi, and protects the body against them by complement activation (classic pathway),

Prepared by Dr: Mohammed Musa Brema Idress – My best wishes Page 1 opsonisation or phagocytosis and neutralization of their toxins. There are four subclasses: → IgG1 (66%), IgG2 (23%), IgG3 (7%) and IgG4 (4%): - IgG1, IgG3 and IgG4 cross the placenta easily - IgG3 is the most effective complement activator, followed by IgG1 and then IgG2 - IgG4 does not activate complement - IgG1 and IgG3 bind with high affinity to Fc receptors on phagocytic cells - IgG4 has intermediate affinity and IgG2 affinity is extremely

3. A 58-year-old male patient needed a blood transfusion after repair of an abdominal aortic . His blood was sent to the laboratory. The technician, while checking for this patient’s blood group, said that the patient’s blood agglutinates with antisera anti-A and anti-D, while the patient’s serum agglutinates cells of blood group B. What is the blood group of this patient? A. A positive B. B positive C. A negative D. B negative E. O positive

ANSWER IS A According to the ABO blood typing system there are four different kinds of blood types: A, B, AB or O. ❖ Blood groups A - If you belong to the blood group A, you have A antigens on the surface of your red blood cells and B antibodies in your blood plasma. ❖ Blood group B - If you belong to the blood group B, you have B antigens on the surface of your red blood cells and A antibodies in your blood plasma. ❖ Blood group AB - If you belong to the blood group AB, you have both A and B antigens on the surface of your red blood cells and no A or B antibodies at all in your blood plasma. ❖ Blood group O - If you belong to the blood group O, you have neither A nor B antigens on the surface of your red blood cells but you have both A and B antibodies in your blood plasma.

Many people also have a so-called Rh factor on the red blood cell’s surface. This is also an antigen and those who have it are called Rh+. Those who have not are called Rh–. A person with Rh– blood does not have Rh antibodies naturally in the blood plasma (as one can have A or B antibodies, for instance) but they can develop Rh antibodies in the blood plasma if they receive blood from a person with Rh+ blood, whose Rh antigens can trigger the production of Rh antibodies.

A person with Rh+ blood can receive blood from a person with Rh– blood without any problems. So, in this vignette the patient’s blood group is A positive as he has antigen A, antibody B and Rh antigens.

4. In humans there are five types of antibody: IgA, IgD, IgE, IgG and IgM. Which of the following statements regarding IgM is CORRECT?

Prepared by Dr: Mohammed Musa Brema Idress – My best wishes Page 2

A. It binds to allergens B. It functions mainly as an antigen receptor on B cells C. It is the largest immunoglobulin molecule D. It is the most abundant immunoglobulin E. It is a tetramer of four

ANSWER IS C IgM forms polymers where multiple immunoglobulins are covalently linked together with disulphide bonds, normally as a pentamer or occasionally as a hexamer. It has a large molecular mass of approximately 900 kDa (in its pentamer form).

The J chain is attached to most pentamers, while hexamers do not possess the J chain due to space constraints in the complex. Because each monomer has two antigen binding sites, an IgM has 10 of them; however, it cannot bind 10 antigens at the same time because they hinder each other.

Because it is a large molecule, it cannot diffuse well and is found in the interstitium only in very low quantities. IgM is primarily found in serum; however, because of the J chain, it is also important as a secretory immunoglobulin.

Due to its polymeric nature, IgM possesses high avidity and is particularly effective at complement activation. It is sometimes called a ‘natural antibody’, but it is likely that the antibodies arise due to sensitization in the very young to antigens that are naturally occurring in nature. For example anti-A and anti-B IgM antibodies can be formed in early life as a result of exposure to anti-A- and anti-B-like substances that are present on bacteria or perhaps also on plant materials.

In germ-line cells, the gene segment encoding the µ constant region of the heavy chain is positioned first among other constant-region gene segments. For this reason, IgM is the first immunoglobulin expressed by mature B cells.

IgM is also by far the physically largest antibody in the circulation. IgM antibodies are mainly responsible for the clumping (agglutination) of red blood cells if the recipient of a blood transfusion receives blood that is not compatible with his/her blood type. IgM antibodies appear early in the course of an infection and usually do not reappear after further exposure. IgM antibodies do not pass across the human placenta. These two biological properties of IgM make it useful in the diagnosis of infectious diseases. Demonstrating IgM antibodies in a patient’s serum indicates recent infection or, in serum from a neonate, indicates intrauterine infection such as congenital rubella.

5. The lack of normal factor VIII causes haemophilia A, an inherited bleeding disorder. Factor VIII is synthesized predominantly in: A. Hepatocytes B. Histiocytes C. Kupffer cells D. Platelets

Prepared by Dr: Mohammed Musa Brema Idress – My best wishes Page 3

E. Vascular endothelium

ANSWER IS E Factor VIII (FVIII) is an essential clotting factor. The lack of normal FVIII causes haemophilia A, an inherited bleeding disorder. The gene for Factor VIII is located on the X chromosome (Xq28). FVIII is a glycoprotein pro-cofactor. Factor VIII is synthesized predominantly in the vascular endothelium and is not affected by liver disease. In fact, levels usually are elevated in such instances. It is also synthesized and released into the bloodstream by the liver.

In the circulating blood, it is mainly bound to von Willebrand factor (vWF, also known as factor VIII-related antigen) to form a stable complex. Upon activation by thrombin or factor Xa, it dissociates from the complex to interact with factor IXa in the coagulation cascade. It is a co-factor to factor IXa in the activation of factor X, which, in turn, with its co-factor factor Va, activates more thrombin. Thrombin cleaves fibrinogen into fibrin, which polymerises and cross links (using factor XIII) into a blood clot. No longer protected by vWF, activated FVIII is proteolytically inactivated in the process (most prominently by activated protein C and factor IXa) and quickly cleared from the bloodstream.

FVIII concentrated from donated blood plasma or alternatively recombinant FVIII can be given to haemophiliacs to restore haemostasis. So, FVIII is also known as antihaemophilic factor. The transfer of a plasma by-product into the bloodstream of a patient with haemophilia often led to the transmission of diseases such as HIV and hepatitis before purification methods were improved. In the early 1990s, pharmaceutical companies began to produce recombinant synthesized factor products, which now prevent nearly all forms of disease transmission during replacement

6. A 45-year-old woman, with a past history of easy bruising and heavy menstrual periods, was admitted for elective cholecystectomy and was diagnosed with von Willebrand’s disease on routine preoperative investigations. von Willebrand’s disease is: A. Autosomal dominant B. Characterized by decreased bleeding time C. Characterized by decreased factor VII D. Characterized by decreased platelets E. X-linked

ANSWER IS A Von Willebrand’s disease (vWD) is the most common hereditary coagulation abnormality described in humans, although it can also be acquired as a result of other medical conditions. It arises from a qualitative or quantitative deficiency of von Willebrand factor (vWF), a multimeric protein that is required for platelet . It is known to affect humans and, in veterinary medicine, dogs.

Prepared by Dr: Mohammed Musa Brema Idress – My best wishes Page 4

There are three types of hereditary vWD, but other factors such as ABO blood group may also play a part in the cause of the condition. The various types of vWD present with varying degrees of bleeding tendency. Severe internal or joint bleeding is rare (only in type 3 vWD); bruising, nosebleeds, heavy menstrual periods (in women) and blood loss during childbirth (rare) may occur. Death may occur The vWF gene is located on chromosome 12 (12p13.2). It has 52 exons spanning 178 kbp. Types 1 and 2 are inherited as autosomal dominant traits and type 3 is inherited as autosomal recessive.

Occasionally type 2 also inherits recessively. In humans, the incidence of vWD is roughly about 1 in 100 individuals. Because most forms are rather mild, they are detected more often in women, whose bleeding tendency shows during menstruation. The actual abnormality (which does not necessarily lead to disease) occurs in 0.9–3% of the population. It may be more severe or apparent in people with blood group O. Acquired vWD can occur in patients with autoantibodies. In this case the function of vWF is not inhibited but the vWF– antibody complex is rapidly cleared from the circulation. A form of vWD occurs in patients with aortic valve stenosis, leading to gastrointestinal bleeding (Heyde’s syndrome). This form of acquired vWD may be more prevalent than is presently thought.

Acquired vWF has also been described in the following disorders: Wilms’ tumour, hypothyroidism and mesenchymal dysplasias. Patients with vWD normally require no regular treatment, although they are always at increased risk for bleeding. Prophylactic treatment is sometimes given for patients with vWD who are scheduled for surgery. They can be treated with human-derived medium purity factor VIII concentrates. Mild cases of vWD can be trialed on desmopressin (1-desamino-8-D- arginine vasopressin, DDAVP) (antihaemophilic factor, more commonly known as humate-P), which works by raising the patient’s own plasma levels of vWF by inducing release of vWF stored in the Weibel–Palade bodies in the endothelial cells

7. A 68-year-old woman complaining of easy fatigability and shortness of breath after abdominal aortic aneurysm repair was diagnosed with iron-deficiency anaemia and prescribed an oral iron preparation. Which of the following statements about iron metabolism is CORRECT? A. Ferritin is a plasma protein that transports iron in the blood B. Haemosiderin is a product of haemoglobin degradation C. Iron is more efficiently absorbed in the ferrous state (Fe2+) than in the ferric state (Fe3+) D. Most iron in the body is stored as haemosiderin E. The gastrointestinal rate of iron absorption is extremely high

ANSWER IS The absorption of non-haem iron in any food is strongly affected by the composition of the meals. Iron is more efficiently absorbed in the ferrous state (Fe2+) than in the ferric state (Fe3+) and commercial iron preparations often contain vitamin C to prevent oxidation of Fe2+ to Fe3+. Still, only 3–6% of the ingested daily iron is actually absorbed in the upper .

Prepared by Dr: Mohammed Musa Brema Idress – My best wishes Page 5

Seventy per cent of the total body iron is used for haemoglobin and myoglobin; the remainder is stored as readily exchangeable ferritin and some is stored in less easily mobilized haemosiderin. When old red blood cells are destroyed by the tissue macrophage system, haem is separated from globin and degraded to biliverdin. Iron in the plasma is bound to the iron transporting protein transferrin. Transferrin level (total iron-binding capacity) and saturation are clinically important indicators of iron deficiency anaemia.

8. A 45-year-old man on warfarin for a mechanical mitral valve was admitted in the Accident and Emergency Department with persistent bleeding following dental extraction. He was told that his coagulation was deranged. Which of the following statements about blood coagulation is CORRECT? A. Absence of Ca2+ promotes blood coagulation B. Disseminated intravascular coagulation (DIC) results in depletion of fibrin split products C. Patients with haemophilia A usually have a normal bleeding time D. von Willebrand factor suppresses platelet adhesion E. von Willebrand factor suppresses blood coagulation

ANSWER IS C Prolonged bleeding time is characteristic of platelet disorders, e.g., thrombocytopaenia. Patients with haemophilia A or B (i.e. absence of factor VIII or IX, respectively) have a prolonged partial thromboplastin time (PTT), but do not have a prolonged bleeding time. Ca2+ is a necessary co-factor for blood coagulation, and chelation of Ca2+ ions by citrate inhibits coagulation.

Von Willebrand factor is part of the factor VIII complex and also promotes platelet adherence to the vascular subendothelium. Patients who lack this factor (von Willebrand’s disease) have both a prolonged PTT and a prolonged bleeding time. Disseminated intravascular coagulation results in depletion of coagulation factors and accumulation of fibrin split products.

9. Nerve gas (organophosphate) is a weapon of chemical warfare that kills by causing respiratory and cardiovascular failure. The expected effect of organophosphate poisoning on the heart would be: A. Decrease the force of myocardial contractions by potentiating the vagal tone to the ventricular muscle B. Decrease the rate of rhythmicity of the sinoatrial (SA) node by inducing hyperpolarisation C. Depolarize cells of the SA node by closing potassium channels under the control of the muscarinic acetylcholine receptor D. Increase the rate of rhythmicity of the SA node by increasing the upward drift in membrane potential caused by sodium leakage E. Increase conductivity at the atrioventricular (AV) junction by inducing depolarization

Prepared by Dr: Mohammed Musa Brema Idress – My best wishes Page 6

ANNSWER IS B The toxic effects of nerve gas derive from its ability to inhibit the enzyme cholinesterase. The inhibition of this naturally occurring degradative enzyme engenders a massive accumulation of acetylcholine evoking an overstimulation of the acetylcholine receptors throughout the body. In the heart, specifically, acetylcholine released by the vagal nerve stimulates muscarinic receptors in the cells of the sinoatrial (SA) node.

This results in the opening of potassium channels and hyperpolarisation of the SA node. It therefore takes longer for sodium leakage to cause the membrane potentials of these cells to reach the threshold required for an action potential. The rate of rhythmicity is so decreased. A similar hyperpolarisation of the fibers at the atrioventricular (AV) junction decreases conduction velocity of atria impulses to the ventricle. The force of ventricular contractions is not affected by the vagus nerve.

10. The resting membrane potential of a neuronal cell body is –60 mV. Opening chloride channels in the neuronal membrane will most likely cause: A. Depolarization to about –30 mV B. Depolarization to about +30 mV C. Hyperpolarisation to about –70 mV D. Initiation of an action potential E. No change in membrane potential

ANSWER IS Increasing the membrane’s conductance to chloride will result in chloride influx and the membrane potential approaching the value dictated by the chloride equilibrium potential (calculated from the Nernst equation), which is about –70 mV for neurons.

A value of –30 mV is near the Nernst potential for Cl− ions in smooth muscle cells, but not in neurons; +30 mV is near the Nernst potential for Na+ ions. The membrane potential would remain unchanged only if the cell resting membrane potential is already at the Nearest potential of the ion channels that were opened. Action potentials occur if the cell membrane is depolarised above threshold.

11. Chloride ions are associated with changes in neuronal membrane potential. Which of the following statements most accurately describes the response of a neuron to a decrease in the conductance of the cell membrane to chloride ions? A. The cell will depolarize if its membrane potential is positive with respect to the equilibrium potential for chloride ions B. The cell will hyperpolarize if its membrane potential is positive with respect to the equilibrium potential for chloride ions C. The cell will hyperpolarize if the external chloride concentration is greater than the internal chloride concentration D. The cell will hyperpolarize if the external chloride concentration is less than the internal chloride concentration E. No change in membrane potential will occur if the external and internal chloride ion concentrations are equal

Prepared by Dr: Mohammed Musa Brema Idress – My best wishes Page 7

ANSWER IS A Although electrogenic pumps may contribute to the membrane potential of certain cells, the major determinants of membrane potential are the external and internal concentrations of permanent ions and their relative permeabilities in the membrane. Decreasing the conductance causes the membrane potential to move away from the equilibrium potential for that ion. So, a decrease in the conductance of a membrane to chloride ions causes the cells to depolarize – that is, become more positive – if the membrane potential is positive with respect to the chloride equilibrium potential.

Conversely, increasing the conductance for an ion causes the membrane potential to approach the equilibrium potential for that ion. External and internal ion chloride concentrations are needed to calculate the Nernst potential for this ion, but a simple comparison of these two values does not allow predictions about the change in membrane potential.

12. The physiological dead space:- A. Is exactly equal to the anatomic dead space in normal people B. Increases during exercise C. Is measured by single breath nitrogen analysis D. Is higher in elderly than young adults E. Is the same as the physiological shunt

ANSWER IS D

13. A high alveolo-arterial PO2 difference most likely results from: A. Low fractional concentration of O2 in the inspired air B. Hypoventilation C. Arteriovenous shunt D. Pulmonary edema E. Polycythemia

ANSWER IS D

14. At 33 feet under water the ambient pressure is: A. Two atmospheres B. One and a half atmosphere C. 33 pounds per square inch D. Same as the pressure in a tunnel 33 feet below sea level E. Half the pressure 33 feet above sea level

ANSWER IS D

15. Which of the following combinations are typical findings in compensated respiratory alkalosis:- A. Low PaCO2, low bicarbonate level and normal pH B. Low PaCO2, normal bicarbonate level and high pH

Prepared by Dr: Mohammed Musa Brema Idress – My best wishes Page 8

C. Low PaCO2, low bicarbonate level and high pH D. Low PaCO2, normal bicarbonate level and normal pH E. Low PaCO2, high bicarbonate level and high pH

ANSWER IS D

16. The partial pressure of oxygen in the alveoli is expected to be least affected by: A. Fractional concentration of O2 in inspired air B. Respiratory quotient C. PCO2 in alveoli D. Hemoglobin concentration E. Barometric pressure

ANSWER IS C

17. A veno-arterial shunt is likely to:- A. Decrease the cardiac output B. Increase the pulse pressure C. Increase venous return D. Decrease PaO2 E. Increase the peripheral vascular resistance

ANSWER IS D

18. The arterio-venous difference of O2 is highest across:- A. Liver B. Kidney C. Cardiac muscle D. Skin E. Brain

ANSWER IS A

19. In a patient, hypoventilation is most likely if: A. PaO2 is low B. pH of arterial blood is low C. PaCO2 is high D. O2 in arterial blood is low E. PCO2 of expired air is high

ANSWER IS C

20. A decrease in the recoil force of the lung is likely to:- A. Decrease total lung capacity B. Increase vital capacity C. Increase functional residual capacity D. Decrease the residual volume

Prepared by Dr: Mohammed Musa Brema Idress – My best wishes Page 9

E. Decrease compliance of the lung

ANSWER IS E

21. Central chemoreceptors differ from chemoreceptors in that they:- A. Respond to changes in PaCO2 B. Respond to changes in pH C. Do not respond to changes in PO2 D. Become more sensitive to CO2 with chronic exposure E. They are less sensitive to changes in PaCO2 than pH

ANSWER IS C

22. Surfactant:- A. Helps to equalize pressure within interconnected alveoli B. Reduces surface tension more effectively as alveolus becomes smaller C. Increases lung compliance D. Increases the slope of the pressure-volume curve E. All of the above

ANSWER IS C

23. One month of an exposure to an inspired CO2 of 5% would result in:- A. Increased ventilatory sensitivity to CO2 B. Reduced ventilatory sensitivity to hypoxia C. Increased plasma bicarbonate D. Alkalosis in arterial plasma E. Increased alveolar PO2

ANSWER IS ……….

24. In a normal adult man, breathing 100% oxygen:- A. Is likely to double O2 in arterial blood B. Is likely to reduce alveolar PCO2 C. Is likely to raise PO2 in venous blood to about 200 mmHg D. Is likely to raise arterial PO2 to about 660 mmHg E. Increases the affinity of hemoglobin to oxygen

ANSWER IS E

25. At the end of maximum inspiration:- A. Intra-alveolar pressure is zero B. Intra-pleural pressure is zero C. Recoil force of the chest is greater than the recoil force of the lung D. Surface tension is lowest E. Average alveolar PO2 is lower than at the end of maximum

Prepared by Dr: Mohammed Musa Brema Idress – My best wishes Page 10

ANSWER IS C

26. On ascent to a high altitude:- A. The concentration of oxygen in the air decreases B. The number of oxygen molecules per liter of air decreases C. Alveolar PCO2 is kept constant despite the high ventilation D. Density of air increases E. The PCO2 in air is the same as at sea level

ANSWER IS ………

27. Compliance of the lung:- A. Is higher in small children than in adults B. Is about 500 ml/cm H2O C. Is normally twice the compliance of the chest D. Is higher in elderly than young adults E. Is decreased in emphysematous changes

ANSWER IS D

28. Dennervation of carotid and aortic bodies leads to all the following except:- A. Decreased sensitivity of ventilation to change in PCO2 B. Decreased sensitivity of ventilation to change in pH C. Complete absence of response of ventilation to change in PO2 D. Absence response of ventilation to exercise E. Decreased sensitivity of ventilation to stagnant hypoxia

ANSWER IS A

29. Minute ventilation is:- A. The volume of air moved in or out of the alveoli per minute B. The volume of air moved in or out of the lung per breath C. Alveolar ventilation plus dead space ventilation D. Invariably increases if the respiratory rate increases E. Is measured using Bohr's equation (CO2 analysis in expired air)

ANSWER IS A

30. The ventilatory response to hypoxia:- A. Is independent of arterial PCO2 B. Is mediated by the central chemoreceptors C. Decreases gradually with time D. Is largely mediated by the aortic bodies in humans E. None of the above

ANSWER IS D

Prepared by Dr: Mohammed Musa Brema Idress – My best wishes Page 11

31. With respect to oxygen and carbon dioxide transport in the blood:- A. For the same partial pressure a unit of blood carries the same amount of O2 and CO2 B. High PCO2 favors O2 binding to hemoglobin C. Metabolic acidosis reduces PCO2 D. Oxygenation of hemoglobin increases the affinity of hemoglobin to CO2 E. Per unit volume of arterial blood there is more O2 than CO2

ANSWER IS D

32. PCO2 in arterial blood:- A. Invariably increases with hypoxia B. Stimulates ventilation mainly through peripheral chemoreceptors C. Is the major controller of ventilation D. Increases early in exercise E. Increases on ascent to high altitude

ANSWER IS C Because PCO2 cause increased Hydrogen ions in the brain which stimulates the repiratory center

33. The functional residual capacity:- A. Is the volume of air in the lung at the end of normal inspiration B. Increases with age C. Is measured by simple spirometry D. Is about 3 liters in an average adult male E. Is about 1 liter in average adult male

ANSWER IS B

34. At the end of inspiration at sea level the PO2 in the anatomic dead space is approximately:- A. 150 mmHg B. 100 mmHg C. 160 mmHg D. 95 mmHg E. Above a hundred but less than 150 mmHg

ANSWER IS E

35. Which is true concerning control of ventilation:- A. Ventilation is more sensitive to changes in PO2 than changes in PCO2 in arterial blood B. Carbon dioxide stimulates ventilation only through the central chemoreceptors C. At high altitude the primary stimulus to ventilation is high PCO2 D. The response of ventilation to a sustained rise in PCO2 increases with time

Prepared by Dr: Mohammed Musa Brema Idress – My best wishes Page 12

E. Cutting the vagus nerves decreases the depth of breathing

ANSWER IS D

36. Which of the following is likely to decrease airway resistance? A. Beta-adrenergic blockers B. Muscarinic cholinergic agonists C. Breathing at higher lung volumes D. Leukotrienes E. Histamine

ANSWER IS C

37. Compared to normal arterial blood, normal mixed venous blood has: A. Higher PO2, lower PCO2 and higher pH B. Lower PO2, higher PCO2 and higher pH C. Lower PO2, higher PCO2 and lower pH D. Lower PO2, lower PCO2 and higher pH E. Higher PO2, lower PCO2 and lower pH

ANSWER IS C

38. Deficiency of surfactant is likely to result in: A. Increased compliance B. Decreased work of breathing C. Reduced surface tension of fluid in the alveoli D. Decreased compliance E. Asthma

ANSWER IS D

39. For a normal Hb-O2 dissociation curve, the most correct relationship is: A. PO2 is 40 mmHg, percent saturation 50 B. PO2 is 95 mmHg, percent saturation 97 C. PO2 is 30 mmHg, percent saturation 50 D. PO2 is 60 mmHg, percent saturation 60 E. PO2 is 50 mmHg, percent saturation 50

ANSWER IS B

40. In a normal subject, blood at the end of the pulmonary capillary:- A. Has a PO2 similar to blood in the left ventricle B. Has a PO2 similar to that in the left atrium C. Has a PO2 higher than blood from the aorta D. Has a percent saturation of 95 E. Contains 20 mL of CO2 per 100 mL

Prepared by Dr: Mohammed Musa Brema Idress – My best wishes Page 13

ANSWER IS C

41. PCO2 in arterial blood:- A. Invariably increases with hypoxia B. Stimulates ventilation mainly through peripheral chemoreceptors C. Is lower than normal in compensated metabolic acidosis D. Increases early in exercise E. Increases on ascent to high altitude

ANSWER IS C

42. In which of the following conditions is the percent saturation of hemoglobin in arterial blood likely to be normal:- A. Venoarterial shunts B. Pulmonary edema C. Carbon monoxide poisoning D. Anemia E. Methemoglobinemia

ANSWER IS D

43. At which of the following points is the intrapleural pressure closest to zero:- A. End of normal expiration B. End of normal inspiration C. End of maximum expiration D. End of maximum inspiration E. Mid-normal expiration

ANSWER IS C

44. Regarding the vital capacity, all of the following are correct except:- A. It is higher in young adults than small children B. It is higher in males than females C. It is normal in pure restrictive lung disease D. It can be measured by simple spirometry E. It is related to body size

ANSWER IS C

45. Immediate acclimatization to high altitude is by:- A. Polycythemia B. Hyperventilation C. Increased cardiac output D. Shift of hemoglobin dissociation curve to the left E. Increase formation of fetal hemoglobin

ANSWER IS B

Prepared by Dr: Mohammed Musa Brema Idress – My best wishes Page 14

46. At the end of maximal expiration the volume of air in the lungs is:- A. FRC - RV B. IRV + RV C. FRC - TV D. TLC - VC E. VC – ERV - IRV

ANSWER IS D

47. At the end of expiration at sea level the PO2 in the anatomic dead space is approximately:- A. 150 mmHg B. 100 mmHg C. 160 mmHg D. 40 mmHg E. 130 mmHg

ANSWER IS C

48. Which is true regarding lung and chest wall mechanics:- A. At maximum expiration the lungs have no tendency to collapse but the chest has a great tendency to expand B. At end of normal inspiration the recoil tendency of the lung is greater than that of the chest C. During inspiration the recoil tendencies of both the lungs and chest increases D. At the end of normal expiration the recoil tendency of the chest is slightly greater than that of the lung E. The combined compliance is greater than the individual compliances of the lung and chest

ANSWER IS C

49. Which is true concerning the diffusing capacity of the lungs? A. It increases whenever the partial pressure gradient increases B. It increases as the alveolar capillary surface area rises C. It increases as the thickness of the respiratory membrane area rises D. It is usually measured using carbon dioxide E. It is usually measured using oxygen

ANSWER IS A

50. A person whose anatomic dead space is 100 mL breathes 12 times per minute with a tidal volume of 400 mL. His pulmonary ventilation is:- A. 1.2 liters B. 2.4 liters C. 3.6 liters

Prepared by Dr: Mohammed Musa Brema Idress – My best wishes Page 15

D. 4.8 liters E. 6.0 liters

ANSWER IS D

51. While repaying the O2 dept:- A. The [H+] is increasing B. PCO2 is decreasing C. Muscle blood flow is increasing D. Respiratory rate is increasing E. Minute volume is decreasing

ANSWER IS D

52. An average adult male has a vital capacity of: A. 3 liters B. 4 liters C. 5 liters D. 6 liters E. 7 liters

ANSWER IS C

53. The residual volume can be calculated by subtracting the expiratory reserve volume from: A. Vital Capacity B. Inspiratory capacity C. Functional residual capacity D. Total lung capacity E. Alveolar ventilation

ANSWER IS C

54. The volume of air that describes the lung's ability to eliminate carbon dioxide is: A. Tidal volume B. Pulmonary ventilation C. Vital capacity D. Alveolar ventilation E. Functional residual capacity

ANSWER IS C

55. A decrease in the recoil force of the lung is expected to: A. Increase vital capacity B. Decrease residual volume C. Decrease compliance of the lung

Prepared by Dr: Mohammed Musa Brema Idress – My best wishes Page 16

D. Increase functional residual capacity E. Decrease total lung capacity

ANSWER IS C

56. Deficiency of surfactant may lead to: A. Increased work of breathing B. Increased compliance of the lung C. Decreased recoil force of the lung D. Bronchial asthma E. An increase in functional residual capacity

ANSWER IS A

57. The most common cause of hypokalaemia is? Single best answer question – choose ONE true option only A. B. Severe diarrhoea C. Nephrotic syndrome D. Dietary deficiency E. Diuretic treatment

ANSWER IS E The most common cause of hypokalaemia is diuretic treatment with thiazides or loop diuretics, which causes increased renal excretion of potassium. Prolonged vomiting and severe diarrhoea can also induce marked hypokalaemia but are less common.

58. Which of the following is not associated with a metabolic acidosis? Single best answer question – choose ONE true option only A. A fall in serum bicarbonate B. Ketosis C. Hypokalaemia D. Hypovolaemic shock E. Hyperventilation

ANSWER IS C Serum potassium levels are intimately linked with serum H+ levels via the sodium potassium ATPase. This cell membrane pump principally exchanges intracellular sodium ions with extracellular potassium ions in order to maintain the cell membrane potential. However, potassium ions compete with hydrogen ions in the exchange pump and therefore in the presence of hypokalaemia, more hydrogen ions will move into the intracellular compartment via this pump.

Conversely, in the presence of hyperkalaemia, less hydrogen ions will move out of the extracellular compartment which results in a metabolic acidosis.

Prepared by Dr: Mohammed Musa Brema Idress – My best wishes Page 17

59. Which the following conditions would result in a metabolic alkalosis? Single best answer question – choose ONE true option only A. Acute shortness of breath B. Gastric outlet obstruction C. Addison’s disease D. A patient taking potassium sparing diuretics E. Salicylate overdose

ANSWER IS B Repeated vomiting in gastric outlet obstruction, results in the loss of hydrochloric acid and a rise in plasma bicarbonate. Bicarbonate levels rise partly because the gastric outlet obstruction results in a decrease in pancreatic exocrine bicarbonate secretion which is the normal response to the presence of acidic gastric contents in the . A secondary hyperaldosteronism will also occur as a response to dehydration resulting in renal potassium loss in preference for sodium reabsorption. With the loss of chloride in hydrochloric acid, the net result is a hypokalaemic, hypochloraemic metabolic alkalosis.

60. Which category of hypoxia results from tissue exposure to metabolic inhibitors such as cyanide? Single best answer question – choose ONE true option only A. Stagnant hypoxia B. Anaemic hypoxia C. Hypoxic hypoxia D. Histotoxic hypoxia E. None of the above

ANSWER IS D Hypoxic challenges may be categorized into several classes according to the underlying cause. Stagnant hypoxia results from reduced perfusion of tissues producing a fall in local pO2 when there is inadequate oxygen delivery and extraction to meet tissue demands. Anaemic hypoxia occurs with substantial reductions in blood haemoglobin concentration.

In this case, although values of PaO2 may be in the normal range, blood oxygen content, and hence oxygen delivery, are reduced resulting in local tissue hypoxia. Similarly, reducing the affinity of haemoglobin for oxygen also results in a fall in blood oxygen content and may induce tissue hypoxia. Hypoxic hypoxia results from a fall in PaO2 secondary to a reduction in inspired pO2 (such as occurs at altitude). Histotoxic hypoxia occurs in the presence of metabolic poisons, such as cyanide, that act intracellularly to inhibit the mitochondrial respiratory electron transport chain.

61. Which of the following is NOT a defining feature of the systemic inflammatory response syndrome (SIRS)? Single best answer question – choose ONE true option only A. Temperature >37.5 Co B. Heart rate >90/min C. Respiratory rate >20/min

Prepared by Dr: Mohammed Musa Brema Idress – My best wishes Page 18

D. PaCO2 <32 mmHg (4.3 kPa) E. White blood cell count of >12 x109/l

ANSWER IS A SIRS is the syndrome arising from the body’s inflammatory reaction to a damaging insult such as infection, trauma, burns or . SIRS is recognized by the presence of the following clinical criteria:- • Temperature > 38 oC or <36 oC • Heart rate > 90/min • Respiratory rate > 20/min or PaCO2 < 32 mmHg (4.3 kPa) • White blood cell count of >12 x109/l, or <4 x109/l, or the presence of >10 % immature

62. Arterial blood gas analysis on a patient shows a plasma pH of 7.35, a PaCO2 of 6.5 and a plasma HCO3- concentration of 40 mmol/l. This is suggestive of which acid/base state? Single best answer question – choose ONE true option only A. Acute respiratory acidosis B. Compensated respiratory acidosis C. Acute metabolic acidosis D. Compensated metabolic acidosis E. Acute metabolic alkalosis

ANSWER IS B Reference ranges: pH plasma 7.36 – 7.44 PaCO2 4.7 – 6.0 kPa (35 – 54 mmHg) PaO2 >10.6 kPa [HCO3-] 20 – 29 mmol/l Procedure for interpreting arterial blood gas results:- 1. Does the pH indicate acidosis or alkalosis? In this case the plasma pH suggests acidosis. 2. Is the PaCO2 elevated or depressed compared with the normal range? In this case the PaCO2 is elevated, suggesting a respiratory acidosis. 3. Is the [HCO3-] high or low compared with the normal range? In this case [HCO3-] is elevated (suggesting compensated respiratory acidosis), rather than being depressed (which would be the case for acute respiratory acidosis).

63. What is the main method by which intracellular pH is regulated? Single best answer question – choose ONE true option only A. The bicarbonate buffer system B. The phosphate buffer system C. Cytoplasmic proteins D. Carbonic anhydrase E. The globin component of haemoglobin

Prepared by Dr: Mohammed Musa Brema Idress – My best wishes Page 19

ANSWER IS C Cytoplasmic proteins provide the main contribution to pH buffering of the intracellular compartment. In the interstitial (i.e. extracellular and extravascular) compartment, the bicarbonate system is the main mechanism of pH buffering.

In the intravascular (plasma) compartment, pH buffering mechanisms include:- • The bicarbonate buffer system: CO2 + H2O « H2CO3 « H+ + HCO3- catalyzed by the enzyme carbonic anhydrase • The phosphate buffer system: HPO42- + H+ « H2PO4- • Plasma proteins • The globin component of haemoglobin

64. Which of the following statements regarding the flow of air through the airways of the lung is correct? Single best answer question – choose ONE true option only A. Flow rate is proportional to the length of the airway B. Flow rate is proportional to the cube of the radius of the airway C. Flow rate is proportional to the viscosity of the gas passing along the airway D. Flow rate is inversely proportional to the pressure gradient along the airway E. None of the above

ANSWER IS E This question tests knowledge and physiological application of Poiseuille’s Law which states that for a rigid, wide bore tube:- Fluid flow rate = pr4 (DP) 8hL where: r = radius of the tube, DP is the pressure gradient along the tube, h is the viscosity of the fluid running through the tube and L is the length of the tube. Therefore, the flow rate is proportional to the fourth power of the radius and the pressure gradient along the tube, but is inversely proportional to the viscosity of the fluid and the length of the tube.

65. In a person breathing normally at rest with an environmental temperature of 25°C, the partial pressure of: A. CO2 in alveolar air is about twice that in room air. B. Water vapour in alveolar air is less than half the alveolar PCO2 level. C. Water vapour in alveolar air is greater than that in room air even at 100 per cent humidity. D. O2 in expired air is greater than in alveolar air. E. CO2 in mixed venous blood is greater than in alveolar .

ANSWER A. False: Room air PCO2 (0.2 mmHg; 0.03 kPa) is negligible compared with alveolar air PCO2 (40 mmHg; 5.3 kPa). B. False: Alveolar H2O vapour pressure at 37oC is 47 mmHg (6.3 kPa).

Prepared by Dr: Mohammed Musa Brema Idress – My best wishes Page 20

C. True: Alveolar air is saturated with water vapour at 37˚C. At 25˚C saturated vapour pressure is 24 mmHg (3.2 kPa), about half that at 37˚C. D. True: Expired air is alveolar air plus dead space air. E. True: This is necessary for CO2 excretion by diffusion.

66. A 24-year-old, unconscious man is admitted to A&E. No history is available. The results of arterial blood gas analysis are: [H+] 80 nmol/l (pH 7.1), p(CO2) 7.0 kPa, p(O2) 8.2 kPa, [HCO3–] 17.1 mmol/l. These results indicate which one of the following acid–base disturbances? Single best answer question – choose ONE true option only A. Metabolic acidosis with respiratory compensation B. Mixed metabolic and respiratory acidosis C. Respiratory acidosis D. Respiratory acidosis with metabolic alkalosis E. Uncompensated metabolic acidosis

ANSWER IS B The high hydrogen-ion concentration (low pH) indicates acidosis. The elevated P(CO2) indicates a respiratory component; in compensated metabolic acidosis, P(CO2) is reduced; in an uncompensated metabolic acidosis (a very unusual situation, since the respiratory response to a metabolic acidosis is usually a rapid one), it would be normal. The hydrogen-ion concentration is too low to be accounted for by a respiratory acidosis alone: there must therefore be a metabolic acidosis in addition (as the low bicarbonate concentration also indicates).

67. As blood passes through systemic capillaries ( True and False): A. pH rises. B. HCO3 ions pass from red cells to plasma. C. Cl ion concentration in red cells falls. D. Its oxygen dissociation curve shifts to the right. E. Its ability to deliver oxygen to the tissues is enhanced.

ANSWER A. False: It falls, due largely to CO2 uptake. B. True: Incoming CO2 is converted to HCO3 (carbonic anhydrase in RBCs catalyses the reaction CO2H2O, HHCO3; HCO3 migrates out as its concentration rises. C. False: It rises as Cl moves in to replace departing HCO3 in the ‘chloride shift’. D. True: Due largely to the rise in blood PCO2. E. True: Due to the shift of the oxygen dissociation curve.

68. For The respiratory centre: A. Is in the hypothalamus. B. Sends impulses to inspiratory muscles during quiet breathing. C. Sends impulses to expiratory muscles during quiet breathing.. D. Is involved in the swallowing reflex. E. Is involved in the vomiting reflex.

Prepared by Dr: Mohammed Musa Brema Idress – My best wishes Page 21

ANSWER A. False: It is in the medulla oblongata. B. True: Causing expansion of the thorax cage. C. False: Expiration at rest is passive. D. True: It is inhibited during swallowing so preventing aspiration of food. E. True: Expiratory, including abdominal muscle contraction with the oesophagus relaxed and the glottis closed helps expel gastric contents in vomiting.

69. An over enthusiastic medical student decides to ask you questions about ECGs. Rather than admitting your dwindling knowledge on this topic, you bravely attempt to answer her questions! One question is what segment of the ECG represents ventricular repolarization? A. QRS complex B. Q-T interval C. P wave D. T wave E. S-T segment

ANSWER IS D The T wave represents ventricular repolarization. The common sense approach to remembering this, is to acknowledge that ventricular repolarization is the last phase of cardiac contraction and should correspond the last part of the ECG. The normal ECG :- P wave - Represents the wave of depolarization that spreads from the SA node throughout the atria - Lasts 0.08 to 0.1 seconds (80-100 ms) - The isoelectric period after the P wave represents the time in which the impulse is traveling within the AV node P-R interval - Time from the onset of the P wave to the beginning of the QRS complex - Ranges from 0.12 to 0.20 seconds in duration - Represents the time between the onset of atrial depolarization and the onset of ventricular depolarization QRS complex - Represents ventricular depolarization - Duration of the QRS complex is normally 0.06 to 0.1 seconds ST segment - Isoelectric period following the QRS - Represents period which the entire ventricle is depolarized and roughly corresponds to the plateau phase of the ventricular action potential T wave - Represents ventricular repolarization and is longer in duration than depolarization

Prepared by Dr: Mohammed Musa Brema Idress – My best wishes Page 22

- Small positive U wave may follow the T wave which represents the last remnants of ventricular repolarization. Q-T interval - Represents the time for both ventricular depolarization and repolarization to occurs, and therefore roughly estimates the duration of an average ventricular action potential. - Interval ranges from 0.2 to 0.4 seconds depending upon heart rate. - At high heart rates, ventricular action potentials shorten in duration, which decreases the Q-T interval. Therefore the Q-T interval is expressed as a "corrected Q-T (QTc)" by taking the Q-T interval and dividing it by the square root of the R-R interval (interval between ventricular depolarizations). This allows an assessment of the Q-T interval that is independent of heart rate. - Normal corrected Q-Tc interval is less than 0.44 seconds.

70. Which of the following is part of the reflex response to an increase in arterial pressure? A. Decreased firing of carotid sinus baroreceptors B. Increased sympathetic activity to the ventricles C. Increased parasympathetic activity to the SA Node D. Increased parasympathetic activity to the arterioles of skeletal muscles & skin E. Increased parasympathetic stimulation to the ventricles

ANSWER IS C

71. Correct sequences of steps in short-term compensation for hemorrhage include: A. Decreased arterial pressure Increased baroreceptor firing rate B. Increased formation of Angiotensin II Increased renin released by kidneys C. Decreased excretion of Na+ and water Increased aldosteron formation D. Decreased firing of baroreceptors Increased sympathetic activity E. Decreased atrial volume Increased volume receptor firing rate

ANSWER IS D

72. Venous Return: A. Is increased on standing B. Decreases by deep inspiration C. Is decreased by venoconstriction D. When increased, activates Bainbridge reflex E. When increased, increases end-systolic volume

ANSWER IS D

73. The nerve supply to the heart is as follows: A. SA Node is innervated by sympathetic only B. Atrial muscle is innervated by both sympathetic & parasympathetic C. Right vagus supplies SA node & AV node

Prepared by Dr: Mohammed Musa Brema Idress – My best wishes Page 23

D. Left vagus supplies SA node E. Parasympathetic supplies ventricular muscle

ANSWER IS B

74. Which of the following is not a characteristic of cardiac muscle? A. It's a syncytium of muscle fibers B. There are intercalated discs C. Gap junctions cause spread of depolarization D. Has myosin and actin muscles E. There are paracellular spaces

ANSWER IS D

75. The pacemaker prepotential:- A. Is due to a slow decrease in K+ influx B. Is a slow increase in Resting Membrane Potential C. Maintained by opening of long acting Ca2+ channels D. Is augmented by opening of transient Ca2+ channels E. Occurs only in the SA node

ANSWER IS E

76. Parasympathetic stimulation results in:- A. Decreases K+ efflux in cardiac muscle B. Increases Ca2+ influx in SA node C. Bradycardia D. Increases Na+ influx E. Increasing slope of prepotential

ANSWER IS C

77. Which of the following is true of the electrical activity of cardiac muscle? A. Increased extracellular K+ causes depolarization B. Repolarization is due to Na+ current C. Extracellular Na+ affects the pacemaker potential D. Plateu of action potential is due to Ca2+ influx E. Initial repolarization is due to delayed K+ efflux

ANSWER IS D

78. The conductive system of the heart does not include: A. Internodal pathways B. Bundle of His & its branches C. Purkinje system of fibers D. Interventricular septum E. AV node

Prepared by Dr: Mohammed Musa Brema Idress – My best wishes Page 24

ANSWER IS D

79. Which of the following is not true of the ECG? A. Needs six pairs of electrodes B. Is useful in detection of arrhythmias C. It is recorded from limb leads D. Its amplitude indicates the mass of cardiac muscle E. The PR interval indicates strength of contraction

ANSWER IS D

80. In a normal ECG: A. The P wave indicates the condition of the conductive system B. The P wave represents depolarization of atrial myocardium C. QRS complex is mainly negative due to spread of depolarization down D. QRS is due to depolarization in the ventricular septum E. The T wave is a positive wave in all chest leads

ANSWER IS B

81. Which of the following is true of the cardiac cycle? A. Ventricular diastole follows atrial systole B. Ventricular systole causes an immediate rise in aortic pressure C. The beginning of systole causes closure of AV valves D. Closure of semi-lunar valves gives rise to first heart sound E. Isovolumetric relaxation occurs at the end of diastole

ANSWER IS C

82. In the atrial pressure wave: A. The maximum pressure is about 12 mmHg B. Pressure remains constant while AV valves are closed C. The C wave is due to bulging of AV valve during ventricular filling D. The V wave is due to filling of the ventricles E. The A wave is due to atrial systole

ANSWER IS E

83. Protodiastole: A. Is characterized by a rapid rise in aortic pressure B. Occurs at the onset of diastole C. Is a period in the cardiac cycle with constant blood pressure D. Is the last one-third of diastole E. Is a period of slow ejection from the ventricles

ANSWER IS A

Prepared by Dr: Mohammed Musa Brema Idress – My best wishes Page 25

84. Which of the following statements is true of the heart sounds? A. The first sound is of high pitch B. The first sound is of longer duration than the second C. They are produced by abnormal valves D. The second sound is heard best at the apex E. The second sound is due to the closure of AV valves

ANSWER IS B

85. Pressure in the pulmonary artery is: A. Higher than that in the right atrium B. About 25 mmHg during systole C. About 80 mmHg during diastole D. Is lower than that in the right ventricle E. Is regulated by parasympathetic innervation

ANSWER IS D

86. The following is not correct about baroreceptors: A. Found in the wall of carotid sinuses B. Found in the aortic arch C. Monitor pressure in the arterial system D. Send impulses through the glosso-pharyngeal nerve E. Normally fires during late systole

ANSWER IS E

87. Chemoreceptors are: A. Found in aortic and carotid sinuses B. Stimulated best by hypercapnoea C. Send impulses to vasomotor center through vagus nerve D. Have a high blood flow rate E. Are not sensitive to hydrogen ion concentration

ANSWER IS E

88. At rest the LV end-systolic volume is: A. 10 mL B. 30 mL C. 50 mL D. 120 mL E. 140 mL

ANSWER IS C

89. The best site to measure mixed venous PO2 is: A. Superior Vena Cava

Prepared by Dr: Mohammed Musa Brema Idress – My best wishes Page 26

B. Right Atrium C. Pulmonary Artery D. Pulmonary Vein E. Left Ventricle

ANSWER IS C

90. The atrial contraction component of ventricular filling is: A. 5% B. 10% C. 30% D. 50% E. 80%

ANSWER IS C

91. The highest oxygen extraction is found in the: A. Resting skeletal muscles B. Heart C. Kidney D. Brain E. Skin

ANSWER IS D

92. The lowest intrinsic discharge activity resides in the: A. SA Node B. AV Node C. Bundle of His D. Bundle of branches E. Purkinje fibers

ANSWER IS B

93. With a mixed venous oxygen content of 110 ml/L and an arterial oxygen content of 150 ml/L an oxygen uptake of 280 ml/min, the cardiac output is: A. 5 liters/ min B. 6 liters/ min C. 7 liters/ min D. 8 liters/ min E. 9 liters/ min

ANSWER IS C

94. The pulmonary valve closes when the pressure in the right ventricle is about: A. 0 mmHg B. 15 mmHg

Prepared by Dr: Mohammed Musa Brema Idress – My best wishes Page 27

C. 30 mmHg D. 50 mmHg E. 120 mmHg

ANSWER IS B

95. Which of the following results in increased stroke volume? A. An increase in end-diastolic volume B. An increased after-load C. Parasympathetic stimulation D. An increase in the heart rate E. Venodilation

ANSWER IS A

96. During the cardiac cycle, closure of the aortic valve occurs at: A. The end of isovolumetric contraction B. The beginning of rapid ejection phase C. The beginning of isometric relaxation D. The end of systole E. The end of rapid filling phase

ANSWER IS C

97. SA node is the pacemaker of the heart because: A. Location in the right atrium B. Neural control C. Natural leakiness to Cl- D. Natural leakiness to K+ E. Fastest rate of discharge

ANSWER IS E

98. The volume of blood is greatest in: A. Systemic capillaries B. Veins C. Arteries D. The spleen E. The heart

ANSWER IS B

99. The velocity of blood flow is the slowest in: A. Capillaries B. Pulmonary vein C. Small arteries D. Inferior Vena Cava

Prepared by Dr: Mohammed Musa Brema Idress – My best wishes Page 28

E. Arterioles

ANSWER IS A

100. Stroke Volume: A. Is the volume of blood pumped by the heart per minute B. Decreases by increased end-diastolic volume C. Is decreased by increased after load D. From the left ventricle is more than from the right ventricle E. Multiplied by heart rate gives cardiac index

ANSWER IS C

101. In the ECG, the T wave denotes: A. Atrial contraction B. Atrial depolarization C. Ventricular repolarization D. SA node depolarization E. Ventricular contraction

ANSWER IS C

102. Cardiac Output (in liters per minute) divided by the heart rate (in beats per minute) equals to: A. Cardiac Index B. Cardiac Efficiency C. Mean Arterial Pressure D. Stroke Volume E. Blood Velocity

ANSWER IS D

103. The segment of the vascular bed responsible for local regulation of blood flow in most tissues is: A. Distributing arteries B. Large veins C. Capillaries D. Venules E. Arterioles

ANSWER IS E

104. First heart sound occurs at the beginning of: A. Isometric relaxation B. Isotonic relaxation C. Isovolumetric contraction D. Isovolumetric relaxation

Prepared by Dr: Mohammed Musa Brema Idress – My best wishes Page 29

E. Atrial contraction

ANSWER IS C

105. Absolute refractory period of the heart: A. Corresponds to the duration of relaxation B. Lasts till half of cardiac contraction C. Shorter than refractory period in skeletal muscle D. Lasts till cardiac contraction E. The heart can be stimulated by very strong stimulus

ANSWER IS D

106. You are called to see a 56-year-old man 2 h after a cardiac catheterization. He is actively bleeding from his catheter site and his dressings and bedclothes are soaked with blood. Which of the following statements is true? Single best answer - choose ONE true option only A. Grade I shock applies with up to a 20% loss of circulating blood volume B. Loss of 2 liters of blood is consistent with normal systolic blood pressure C. The pulse can remain normal in patients with grade I shock D. Anuria is pathognomonic of grade III shock E. Grade IV shock is seen with a 30% loss of circulating blood volume

ANSWER IS C Grade I shock: Loss of up to 15% (750ml) of blood volume; blood pressure is normal but there may be a slight tachycardia Grade II shock: 15–30% (750 ml – 1.5 l) blood-volume loss, systolic blood pressure is usually normal but a tachycardia is present Grade III shock: 30–40% (1.5–2 liters) loss, hypotension, tachycardia and fall in urine output seen Grade IV shock: > 40% (> 2 l) blood-volume loss, anuria and severe shock observed

107. A 60-year-old obese smoker has been admitted to hospital with chest pain due to unstable angina. A nitrate infusion is started to relieve his chest pain. Which blood vessels are most sensitive to the vasodilatatory effect of nitrates? Single best answer question – choose ONE true option only A. Large arteries B. Coronary arteries C. Capillaries D. Large veins E. Pulmonary arteries

ANSWER IS D The antianginal and haemodynamic effects are mediated predominantly by vasodilatation of the venous system, leading to a fall in left ventricular preload and cardiac work.

Prepared by Dr: Mohammed Musa Brema Idress – My best wishes Page 30

108. You are called to ICU to see a 65-year-old patient who requires controlled mechanical ventilation after major non-cardiac surgery but is becoming hypoxaemic when the FiO2 is reduced from 0.4 to 0.3. Which of the following statements is true? Single best answer question – choose ONE true option only A. Simple indices of circulatory status – such as urine output, blood pressure and CVP – correlate well with outcome from high-risk surgery B. Survivors after major surgery decrease their cardiac index and oxygen delivery in the perioperative period below baseline normal values C. Measurement of mixed venous oxygen saturation (SVO2) requires a pulmonary venous (PV) catheter to sample pulmonary capillary blood D. Cardiac index and oxygen delivery correlate poorly with outcome from high- risk surgery E. Pre- or perioperative beta-blockade can improve survival after major non- cardiac surgery in patients with pre-existing cardiac disease

ANSWER IS E Simple indices of circulatory status – such as urine output, blood pressure and CVP – correlate poorly with outcome from high-risk surgery. Survivors after major surgery increase their cardiac index and oxygen delivery in the perioperative period above baseline normal values. Measurement of mixed venous oxygen saturation (SVO2) requires a pulmonary artery (PA) catheter to sample pulmonary capillary blood. Cardiac index and oxygen delivery correlate well with outcome from high-risk surgery.

Two recent multicentre trials have confirmed the advantage of using highly selective pre- or perioperative beta-blockade to improve survival after major non-cardiac surgery in patients with pre-existing cardiac disease, e.g. previous heart failure, moderate hypertension and myocardial infarction (MI). The regime is started 48–72 h preoperatively and continued for 14–28 days post-surgery.

109. Which of the following best describes the physiological principles that determine movement of fluid across a capillary? Single best answer question – choose ONE true option only A. Fluid movement is determined by the difference in plasma and tissue oncotic pressure B. Fluid movement is determined by capillary permeability C. Fluid movement is determined by the difference in hydrostatic pressure between the capillary and the tissue D. Fluid movement is determined by the difference in capillary hydrostatic pressure + tissue oncotic pressure AND tissue hydrostatic pressure + capillary oncotic pressure E. Fluid movement is determined by the difference in capillary hydrostatic pressure + capillary oncotic pressure AND tissue hydrostatic pressure + tissue oncotic pressure

Prepared by Dr: Mohammed Musa Brema Idress – My best wishes Page 31

ANSWER IS D Fluid will move across a capillary if there is sufficient hydrostatic and oncotic pressure difference across the capillary wall. Oncotic pressure holds fluid within that compartment and so a high capillary oncotic pressure will hold fluid within that capillary. Albumin is the most important determent of intravascular oncotic pressure.

110. Within normal physiological limits, which of the following factors does not influence cardiac stroke volume? Single best answer question – choose ONE true option only A. Preload B. After load C. Heart rate D. Cardiac sympathetic nerve activity E. Myocardial contractility

ANSWER IS B Increases in preload (up to a limit) increase stroke volume by Starling’s Law of the heart. Starling’s Law also indicates that increases in after load (up to a limit), whilst causing ventricular stretch and an increase in end-diastolic volume, do not increase stroke volume but instead maintain it.

Increases in myocardial contractility increase the force of contraction during systole and therefore increase stroke volume. If preload does not become limiting, increases in heart rate increase myocardial contractility, and therefore stroke volume, via the Bowditch effect (a rate-related phenomenon thought to be due to accumulation of intracellular calcium in the cardiomyocytes). Cardiac sympathetic nerve activity increases stroke volume by increasing cardiac contractility and heart rate.

111. Which of the following is a function of atrial natruretic peptide (ANP)? Single best answer question – choose ONE true option only A. Increases renin secretion B. Decreases aldosteron secretion C. Promotes the effects of antidiuretic hormone (ADH) D. Causes renal vasoconstriction E. Promotes the feeling of thirst

ANSWER IS B ANP is released from atrial muscle cells when the atria are stretched due to increased circulating blood volume. Therefore ANP works to reduce blood volume by inhibiting the release of renin, aldosteron and ADH resulting in increased sodium and water excretion. It promotes renal vasodilatation; blocks Na reabsorption.

112. A 65-year-old diabetic woman is feeling generally unwell 4 days following an elective anterior resection for a Dukes’ A colorectal carcinoma. On examination, the patient is a febrile, tachycardic and hypotensive with clammy skin. Her

Prepared by Dr: Mohammed Musa Brema Idress – My best wishes Page 32 abdomen is mildly tender with a clean dry wound and scanty but audible bowel sounds. Her urine output has been poor for the last 6 hours. The ECG shows a new left-bundle branch block. What type of shock is most likely? A. Cardiogenic shock B. Hypovolaemic shock C. Neurogenic shock D. Septic shock

ANSWER IS A This patient has an important risk factor for ischaemic heart disease – diabetes mellitus – which is associated with silent myocardial infarction. The patient feels generally unwell with signs of shock and left-bundle branch block on the ECG. It is quite possible that the patient may have had a myocardial infarction and now presents with cardiogenic shock. The presence of bowel sounds and a mildly tender abdomen suggest that an intra-abdominal problem is unlikely.

113. A patient receives too many infusions after an operation resulting in a 20% increase in his blood volume. What is the physiological process that is most likely to correct this abnormality? Single best answer question – choose ONE true option only A. Reduced activity of arterial pressure sensors B. Increased activity of renal sympathetic nerves C. Aldosteron release D. Atrial natriuretic peptide (ANP) release E. Venous dilatation

ANSWER IS D The atria contain granulated cells that release peptides, atrial natriuretic peptide (ANP), in response to stretch. This natriuretic agent also relaxes the peripheral vasculature and thereby opposes the actions of the sympathetic and renin– angiotensin systems.

114. Phase 0 of the cardiac action potential relates to a: Single best answer question – choose ONE true option only A. Rapid efflux of calcium B. Rapid influx of calcium C. Rapid influx of potassium D. Rapid influx of sodium E. None of the above

ANSWER IS D Phase 0 rapid sodium influx Phase 1 efflux of potassium Phase 2 slow influx of calcium Phase 3 efflux of potassium Phase 4 sodium/calcium efflux, potassium influx

Prepared by Dr: Mohammed Musa Brema Idress – My best wishes Page 33

115. Which one of the following is MOST likely to increase during exercise? Single best answer question – choose ONE true option only A. Peripheral vascular resistance B. Pulmonary vascular resistance C. Stroke volume D. Diastolic pressure E. Venous compliance

ANSWER IS C During exercise, increased oxygen consumption and increased venous return to the heart result in an increase in cardiac output and an increase in blood flow to both skeletal muscle and coronary circulation, when oxygen utilization is greatest. The increase in cardiac output is due to an increase in both heart rate and stroke volume. Systemic arterial pressure also increases in response to the increase in cardiac output. However, the fall in total peripheral resistance, which is caused by dilatation of the blood vessels within the exercising muscles, results in a decrease in diastolic blood pressure.

The pulmonary vessels undergo passive dilatation as more blood flows into the pulmonary circulation. As a result, pulmonary vascular resistance decreases. The decrease in venous compliance, caused by sympathetic stimulation, helps to maintain ventricular filling during diastole.

116. A 56-year-old woman sustains a myocardial infarction. ST elevation and Q waves are present in leads V4–V6, I and AVL. Which of the following aspects of the heart is most likely to have been involved in the infarct? Single best answer question – choose ONE true option only A. Anterior B. Anterolateral C. Anteroseptal D. Inferior E. Lateral

ANSWER IS B This combination suggests an anterolateral infarct. Purely anterior infarcts tend to involve the chest leads only (typically V2–V5), anteroseptal V1–V3, lateral infarcts chest leads only (I, II, AVL) and inferior infarcts II, III and AVF.

117. Cardiac output is decreased: Single best answer question – choose ONE true option only A. During stimulation of sympathetic nerves to the heart B. On cutting the vagus nerves to the heart C. By increasing the end diastolic volume of the heart D. As a result of decreased pressure within the carotid sinus E. Upon assuming the upright position

Prepared by Dr: Mohammed Musa Brema Idress – My best wishes Page 34

ANSWER IS E Stimulation of the sympathetic nervous system results in a rise in heart rate and stroke volume and therefore cardiac output increases. Cutting the vagus nerves to the heart, results in an increase in heart rate, because of the abolition of vagal tone and therefore cardiac output increases.

If the end diastolic volume of the heart (pre-load) is increased, under normal physiological circumstances, cardiac output is increased by the Frank-Starling mechanism. The exception is in the failing heart where the Law of Laplace becomes more important and cardiac output actually falls.

Arterial blood pressure is homeostatically regulated through the action of baroreceptors, principally located in the carotid sinus and the wall of the aortic arch. If the carotid sinus pressure is reduced, the baroreceptors become inactive and lose their inhibitory effect on the vasomotor centre in the brainstem. The result is activation of the sympathetic nervous system. This produces a rise in heart rate, stroke volume, mean systemic filling pressure and venous return, leading to an increase in cardiac output and return of the mean arterial blood pressure to its original value.

Cardiac output falls when one stands up due to the pooling of blood on the venous side of the circulation, which has a large capacitance. Stepping out of a hot bath exacerbates this pooling effect because superficial cutaneous veins dilate in response to heat, increasing their capacitance even further.

Under normal circumstances, activation of the baroreceptor reflex compensates to some degree, preventing syncope. However, in the elderly, or in patients on anti- hypertensive, inadequate compensation from the baroreceptor reflex may result in a vasovogal syncope, or othostatic hypotension.

118. Flow through a vessel or lumen is: Single best answer question – choose ONE true option only A. Is inversely proportional to the pressure head of flow B. Is inversely proportional to the radius C. Is directly proportional to the length of the tube D. Is directly proportional to the viscosity of blood passing through it E. Is directly proportional to the fourth power of radius

ANSWER IS E The Hagen-Poiseuille law states that the flow through a vessel is: 1. Directly proportional to the pressure head of flow 2. Directly proportional to the fourth power of radius 3. Inversely proportional to the viscosity 4. Inversely proportional to the length of the tube The radius of the tube is therefore the most important determinant of flow through a blood vessel. Thus, doubling the radius of the tube will lead to a 16-fold increase in flow at a constant pressure gradient. The implications of this are several fold.

Prepared by Dr: Mohammed Musa Brema Idress – My best wishes Page 35

First, owing to the fourth power effect on resistance and flow, active changes in radius constitute an extremely powerful mechanism for regulating both the local blood flow to a tissue and central arterial pressure. The arterioles are the main resistance vessels of the circulation and their radius can be actively controlled by the tension of smooth muscle within its wall.

Second, in terms of intravenous fluid replacement in hospital, flow is greater through a peripheral cannula than through central lines. The reason is that peripheral lines are short and wide (and therefore of lower resistance and higher flow) compared to central lines, which are long and possess a narrow lumen. A peripheral line is therefore preferential to a central line when urgent fluid resuscitation, or blood, is required.

119. Carbon dioxide is principally transported in the blood in the form of: Single best answer question – choose ONE true option only A. CO2 physically dissolved in solution B. Carboxyhaemoglobin C. Bicarbonate D. Carbaminohaemoglobin E. Carbonic anhydrase

ANSWER IS C Carbon dioxide is transported in the blood in the form of: - Bicarbonate accounts for about 80-90% of the total CO2 in the blood - As carbamino compounds (5-10%) - Physically dissolved in solution (only 5%) Carbon dioxide is carried on the haemoglobin molecule as carbamino-haemoglobin; carboxyhaemoglobin is the combination of haemoglobin with carbon monoxide. Erythrocytes contain the enzyme carbonic anhydrase that catalyses the reaction CO2 + H2O = H+ + HCO3- and requires zinc as a co-factor. This plays an important role in carbon dioxide transport and in the buffering of pH.

120. The plateau phase of the cardiac action potential is due to: Single best answer question – choose ONE true option only A. Magnesium influx B. Potassium influx C. Calcium influx D. Chloride efflux E. Sodium influx

ANSWER IS C The most important source of activator calcium in cardiac muscle remains its release from the sarcoplasmic reticulum. Calcium however also enters from the extracellular space during the plateau phase of the action potential. This calcium entry provides the stimulus that induces calcium release from the sarcoplasmic reticulum (calcium induced calcium release).

Prepared by Dr: Mohammed Musa Brema Idress – My best wishes Page 36

The result is that tension generated in cardiac, but not in skeletal, muscle is profoundly influenced both by extracellular calcium levels and factors that affect the magnitude of the inward calcium current. This is of practical value in two key clinical situations; in heart failure where digoxin is utilized to increase cardiac contractility (by increasing the intracellular calcium concentration) and in hyperkalaemia where calcium gluconate is used to stabilize the myocardium.

The plateau phase of the action potential in cardiac muscle (principally due to calcium influx) maintains the membrane at a depolarised potential for as long as 500ms.

The result is that the cell membrane is refractory throughout most of the mechanical response, largely due to the inactivation of fast sodium channels. This prevents tetany upon repetitive stimulation which would be detrimental to cardiac output. Furthermore, the prolonged refractory period in cardiac muscle allows the impulse that originates in the sino-atrial node to propagate throughout the entire myocardium just once, thereby preventing re-entry arrhythmias.

121. The ejection fraction is defined as: Single best answer question – choose ONE true option only A. The ratio of the end diastolic volume to stroke volume B. The ratio of stroke volume to end diastolic volume C. End diastolic volume minus end systolic volume D. End systolic volume divided by stroke volume E. The ratio of stroke volume to end systolic volume

ANSWER IS B During diastole, filling of the ventricles normally increases the volume of each ventricle to about 120mls. This volume is known as the end diastolic volume. Then, as the ventricles empty in systole, the volume decreases about 70mls, which is known as the stroke volume. The remaining volume in each ventricle, about 50mls, is known as the end systolic volume and acts as a reserve which can be utilized to increase stroke volume in exercise.

The fraction of end diastolic volume that is ejected is called the ejection fraction – usually equal to about 60%. The ejection fraction is often used clinically as an indirect index of contractility. It is a particularly useful in assessing the state of the myocardium prior to aortic aneurysm repair where cross-clamping of the aorta places particular stress on the myocardium.

122. The cardiovascular effects of raised intracranial pressure include: Single best answer question – choose ONE true option only A. Decreased blood pressure, decreased heart rate, decreased cerebral perfusion pressure B. Decreased blood pressure, increased heart rate, decreased cerebral perfusion pressure

Prepared by Dr: Mohammed Musa Brema Idress – My best wishes Page 37

C. Increased blood pressure, increased heart rate, decreased cerebral perfusion pressure D. Increased blood pressure, decreased heart rate, decreased cerebral perfusion pressure E. Decreased blood pressure, increased heart rate HR, increased cerebral perfusion pressure

ANSWER IS D The important relationship between the cerebral perfusion, mean arterial blood pressure and intracranial pressure is as follows: CPP = MABP – ICP, where CPP = cerebral perfusion pressure MABP = mean arterial blood pressure ICP = intracranial pressure

It stems from the fact that the adult brain is enclosed in a rigid, incompressible box, with the result that the volume inside it must remain constant (Monroe-Kelly doctrine). A rise in intracranial pressure therefore decreases cerebral perfusion pressure (and hence cerebral blood flow).

In raised intracranial pressure, as the brainstem becomes compressed, local neuronal activity causes a rise in sympathetic vasomotor drive and thus a rise in blood pressure. This is known as the Cushing’s reflex. This elevated blood pressure evokes a bradycardia via the baroreceptor reflex. The Cushing’s reflex helps to maintain cerebral blood flow and protect the vital centers of the brain from loss of nutrition if the intracranial pressure rises high enough to compress the cerebral arteries.

123. In the respiratory system, physiological shunt? Single best answer question – choose ONE true option only A. Is greater than the anatomical shunt B. Is not present in healthy adult C. Affects arterial carbon dioxide more than arterial oxygen tension D. Has the same effect on respiratory gas exchange as does physiological dead space E. Is abolished when the subject breathes pure oxygen

ANSWER IS A The physiological shunt is the sum of the anatomical shunt (blood passing from the right ventricle to the systemic circulation via normal anatomical pathways, e.g. the bronchial vessels, without passing through the pulmonary alveolar capillaries), and the element of pulmonary alveolar capillary blood that has passed through non or poorly aerated alveoli. Therefore physiological shunt is always at least as great as or greater than the anatomical shunt.

There is always a normal anatomical shunt even in the young healthy adult.

Prepared by Dr: Mohammed Musa Brema Idress – My best wishes Page 38

The difference in carbon dioxide tension between arterial and mixed venous blood is a little less than 1 kPa, and therefore even a 50% shunt only increases arterial carbon dioxide tension by about 0.5 kPa. A 50% shunt would reduce arterial oxygen tension from 13.5kPa to below 9 kPa.

The physiological dead space results primarily in a failure to remove carbon dioxide from alveolar gas, i.e. a rise in arterial carbon dioxide tension if ventilation not increased. The breathing of pure oxygen cannot eliminate the anatomical right to left portion of the physiological shunt.

124. There are three ways by which carbon dioxide is transported in the blood. About 70% of the carbon dioxide is transported to the lungs: Single best answer question – choose ONE true option only. A. In the form of bicarbonate ions B. In the form of carbonic acid C. In the form of carbaminohaemoglobin D. In chemical combination with albumin E. In the dissolved state in the water of the plasma and cells.

ANSWER IS A Dissolved carbon dioxide reacts with water inside red blood cells to form carbonic acid. This reaction is catalyzed by a protein enzyme in the red cells called carbonic anhydrase. Most of the carbonic acid immediately dissociates into bicarbonate ions and hydrogen ions, the hydrogen ions in turn combining with haemoglobin.

Approximately 23% of the carbon dioxide produced in the tissues combines directly with haemoglobin to form carbaminohaemoglobin and an additional 7% is transported in the dissolved state in the water of the plasma and cells.

125. A 69- year-old women with a previous myocardial infarct is admitted to HDU with shortness of breath, peripheral oedema and signs of chest sepsis. A line is inserted to measure her central venous pressure. Which of the following is correct regarding CVP? Single best answer – select one answer only. A. CVP can be estimated by examining the external jugular vein B. Indicates left ventricular filling pressure in left ventricular failure C. Correlates well with cardiac after load D. Is most useful when taken as a one-off measurement E. Is low in hypervolemia.

ANSWER IS A The central venous pressure (CVP) is an important concept in clinical cardiology because it is a major determinant of the filling pressure and therefore the preload of the right ventricle (and not the cardiac after load). The CVP can be measured invasively by using a manometer after central venous cannulation. The CVP can be estimated by examining the height of the pulsation in the external jugular vein.

Prepared by Dr: Mohammed Musa Brema Idress – My best wishes Page 39

The height of the pulsation should be compared with the angle of Louis as the patient sits at 45° to the horizontal. Zero is taken at the level of the right atrium. This is somewhat arbitrary and so one-off measurements are not very useful unless there is a profoundly low CVP indicating profound hypovolemia. More useful is to measure the changes in CVP occurring during and after the administration of a bolus of 200 ml of colloid intravenously. If the patient is hypovolemic, there will be only a very small transient change in CVP.

In normovolemia there will be a rise in CVP, which will be sustained for 10 min or so. In hypervolemia the rise in CVP will be sustained for longer. A high CVP indicates myocardial failure. CVP indicates the right and left ventricular filling pressure only when these are equal. If there is ventricular failure and the filling pressure of the two ventricles is not equivalent then CVP is a measure only of the right atrial pressure.

126. A 79-year-lady presents to the pre-admission clinic with untreated hypertension. After discussion with Cardiology you start her on lisinopril, which acts to block angiotensin secretin enzyme and lowers aldosteron. Of the following, which one reflects a true action of aldosteron? Single best answer - select one answer only. A. Decreases excretion of ammonium B. Decreases secretion of hydrogen C. Decreases secretion of potassium D. Increases excretion of magnesium E. Increases secretion of potassium

ANSWER IS E The primary effects of aldosteron on the renal system are to increase reabsorption of sodium and increase secretion of potassium and hydrogen ions. Aldosteron acts by increasing the number and activity of the active transport systems in the distal renal tubule to sodium and potassium. Owing to the enhanced reabsorption of sodium, water reabsorption is also increased. Aldosteron increases excretion of both ammonium and magnesium.

127. Increased venous return to the heart is most likely to be caused by Single best answer question – choose ONE true option only. A. Deep inspiration B. Forced expiration C. Hypovolaemia D. Positive pressure ventilation E. Tension pneumothorax.

ANSWER IS A Blood returns to the heart from the lower limbs via the action of the calf muscle pumps, valves in the veins of the leg, and the effect of negative intra-thoracic pressure generated during inspiration. Anything causing the intra thoracic pressure to become less negative will decrease the venous return to the right atrium. Tension pneumothorax, positive pressure ventilation and forced expiration all cause this

Prepared by Dr: Mohammed Musa Brema Idress – My best wishes Page 40 effect, and therefore reduce the venous return. Although hypovolaemia may cause vasoconstriction in an attempt to increase venous return, it is unlikely to increase above normal levels.

128. You attend a trauma call in the emergency department. The patient is a 32- year-old male who has been stabbed in the left side of the chest, just below the fifth rib. You suspect that the patient has cardiac tamponade. Which of the following signs would be consistent with the diagnosis of cardiac tamponade? Select one answer only. A. A ‘globular’ cardiac outline on chest X-ray (CXR) B. Charcot’s triad C. Collapsed neck veins D. Hypertension E. Prominent heart sounds.

ANSWER IS A The classical symptom triad of cardiac tamponade (Beck’s triad) includes muffled heart sounds, distended neck veins and hypotension, and is seen in the majority of patients. Pulsus paradoxus is defined as a fall in systolic blood pressure of over 10 mmHg on inspiration and this only occurs in 1 in 10 cases of tamponade. In cardiac tamponade, the cardiac outline on CXR is classically globular. Charcot’s triad refers to fever, jaundice and , and is suggestive of cholangitis.

129. A 35-year-old maintenance engineer falls from a flagpole and is admitted with a head injury and is profoundly hypotensive. Which of the following regarding neurogenic shock is correct? Single best answer – select one answer only. A. Causes decreased cardiac pre-load B. Is characterized by tachycardia C. Is associated with cool peripheries D. Leads to a narrowed pulse pressure E. Can be caused by isolated intracranial injuries.

ANSWER IS A Neurogenic shock follows spinal transaction, usually a high cervical spine injury or brainstem injury with loss of sympathetic outflow below the level of injury and consequent vasodilatation. The rapid increase in size of the vascular bed, including venous capacitance vessels (venous pooling of blood), leads to a reduction in the venous return and cardiac output (due to less blood being returned to the heart – decreased pre-load). Loss of sympathetic tone also leads to hypotension. The classic picture of neurogenic shock is hypotension without tachycardia or cutaneous vasoconstriction (causing warm pink peripheries).

The other signs and symptoms may include tachypnoea, diaphragmatic breathing and quadriplegia. A narrowed pulse pressure is not seen in neurogenic shock. Isolated intracranial injuries do not cause neurogenic shock or any other forms of shock. The presence of shock in a patient with a head injury necessitates a search for

Prepared by Dr: Mohammed Musa Brema Idress – My best wishes Page 41 another cause of shock. Patients sustaining a spinal injury would have also suffered a concurrent thoracic, abdominal or pelvic trauma.

The loss of sympathetic tone compounds the physiological effects of hypovolemia and hypovolemia compounds the physiological effects of sympathetic denervation. Therefore, patients with diagnosed or suspected neurogenic shock should be treated initially for hypovolemia. The inability to restore organ perfusion with adequate fluid resuscitation suggests either continuing haemorrhage or a neurogenic shock.

130. You insert a subclavian central line for one of you patients, but forget to close it following insertion. Which recognized complication of central venous line insertion is most likely to occur as a result? Select one answer only. A. Arterial air embolism B. Chylothorax C. Haemorrhage D. Pneumothorax E. Venous air embolism

ANSWER IS E Central venous line insertion may cause trauma to adjacent tissues with consequent haemorrhage and pneumothorax. If it is left open then air may enter the blood, causing venous air embolism. Systemic arterial air embolism may occur if the cannula is mistakenly placed in the carotid artery.

131. The electrocardiogram (ECG) of a patient shows a prolonged QT interval. A normal QT interval is usually about: Single best answer question – choose ONE true option only. A. 0.10 s B. 0.20 s C. 0.30 s D. 0.40 s E. 0.50 s.

ANSWER IS D The QT interval is measured from the beginning of the QRS complex to the end of the T-wave. A normal QT interval is usually about 0.40 s. The QT interval, as well as, the corrected QT interval is important in the diagnosis of long-QT syndrome and short-QT syndrome. The QT interval varies based on the heart rate and various correction factors have been developed to correct the QT interval for the heart rate.

132. Which of the following is true about renal blood flow? A. The kidneys receive 12-15% of cardiac output B. Is kept constant within a small range by sympathetic vasomotor tone C. Measured by renal clearance of inulin D. The dilution method is used for its measurement E. Goes mainly to the renal cortex

Prepared by Dr: Mohammed Musa Brema Idress – My best wishes Page 42

ANSWER IS E

133. The characteristics of the Loop of Henle include: A. Descending limb is not permeable to water B. Urine becomes more dilute as it reaches the tip of the loop C. Ascending limb is permeable to water D. In thin ascending limb urea is actively transported to interstitial fluid E. Na is reabsorbed by Na-K ATPase in ascending limb

ANSWER IS E

134. The following is true about the Vasta Recta: A. Descending vasta recta reabsorbs water from the interstitial fluid B. The descending capillaries loose Na and Urea C. Plasma becomes hyperosmotic as it descends in vasta recta D. Ascending capillaries loose water to interstitial fluid E. They are responsible for the countercurrent multiplier mechanism

ANSWER IS C

135. A bout Bicarbonate Reabsorption: A. Represents a high capacity low gradient system B. Filtered bicarbonate/ day= 400 – 500 mmol/day C. More HCO3 is absorbed in distal than proximal tubules D. PCT apical membrane is permeable to HCO3- E. H2CO3 is PCT lumen is catalyzed at brush border by carbonic anhydrase to give CO2 and HCO3-

ANSWER IS D

136. All of the following are true of ammonium production except: A. It is mainly a function of PCT B. The source is mainly glutamine C. It represents the main mechanism in renal excretion of H+ D. It is a low capacity high gradient system E. It is increased when the urine pH is low

ANSWER IS A

137. The functions of the renal proximal convoluted tubes include: A. Reabsorption of all bicarbonate filtered at the glomerulus B. Production of renin C. Reaborption of 80% of filtered bicarbonate D. Reabsorption of bicarbonate resulting in fixed acid secretion E. Hydrogen ion secretion by facilitated diffusion

ANSWER IS C

Prepared by Dr: Mohammed Musa Brema Idress – My best wishes Page 43

138. Titrable Acidity (TA): A. Can be measured by titration against sodium hydroxide to pH 7.4 B. Is a measure of H+ mainly buffered by ammonium ions in the distal tubules C. Is the same as the anion gap D. Creatinine contributes to significant amounts to TA E. Ketoacids in diabetic acidosis do not increase TA

ANSWER IS E

139. For those substances that are actively reabsorbed or secreted, the highest limit of the amount that can be transported per unit time by the kidney tubules: A. Depends on the rate at which the transport mechanism operates B. Is directly related to the plasma concentration of the substance C. Depends on the tubular transport maximum D. Is dependent upon tubular load E. Depends on the GFR

ANSWER IS C

140. If the clearance of Substance X is greater than the clearance on inulin, it is most likely that substance X is: A. Bound to tubular proteins B. Bound to plasma proteins C. Secreted D. Reabsorbed E. Neither secreted nor reabsorbed

ANSWER IS C

141. Renin is produced and secreted from granules located in the: A. Macula densa B. Mesangial cells C. Intercalated cells D. Afferent arteriole E. Interstitial cells

ANSWER IS A

142. The tonicity of urine as it enters the renal collecting duct is: A. Isotonic B. Hypotonic or isotonic, but never hypertonic C. Hypotonic D. Hypertonic E. Hypertonic or isotonic, but never hypotonic

ANSWER IS A

Prepared by Dr: Mohammed Musa Brema Idress – My best wishes Page 44

143. On assuming the upright posture, one would expect: A. Stimulation of the baroreceptor reflex B. Increased pulmonary artery pressure C. A decrease in pulse rate D. Venodilation E. An increase in renal blood flow

ANSWER IS E

144. In a healthy adult man the glomerular filteration rate is expected to be: A. 225 ml/min B. 325 ml/min C. 125 ml/min D. 25 ml/min E. 180 ml/min

ANSWER IS C

145. A substance that is used to measure renal plasma flow should be: A. Bound to plasma proteins B. Easily reabsorbed C. Highly secreted D. Metabolized by the body E. Stored only in the kidney

ANSWER IS C

146. The following factors when increased, they will increase the glomerular filteration rate except: A. Renal blood flow B. Afferent arteriolar diameter C. Efferent arteriolar diameter D. Permeability of glomerular membrane E. Capillary surface area

ANSWER IS C

147. Renal blood flow is: A. Increased when renal sympathetic nerves are stimulated B. Autoregulated when mean arterial blood pressure is between 60-180 mmHg C. Measured by urea clearance D. About 2 liters in a normal man E. Decreased when the fluid flow at the site of the macula densa is decreased

ANSWER IS B

Prepared by Dr: Mohammed Musa Brema Idress – My best wishes Page 45

148. The following hormones can be formed by the kidney except: A. Erythropoietin B. Angiotensin II C. Prostaglandins D. ADH E. Calcitrol

ANSWER IS D

149. Sodium reabsorption in the proximal tubules from the luminal surface is marked by all of the following except: A. Sodium glucose co-transporters B. Sodium-hydrogen exchanger C. Na-K pump D. Solvent drag E. Sodium amino acid transporters

ANSWER IS C

150. Glomerular filteration rate would be increased by: A. A decrease in afferent arteriolar pressure B. A decrease in renal blood flow C. A decrease in efferent arteriolar resistance D. Decrease in the concentration of plasma proteins E. Obstruction of ureters

ANSWER IS D

151. A 23 year old is stabbed in the groin and develops hypovolaemic shock. What is the most likely finding on analysis of his urine? A. Decreased specific gravity B. Increased specific gravity C. Increased urinary glucose D. Increased urinary protein E. Increased red blood cells in the urine

ANSWER IS B Hypovolaemic shock is likely to compromise renal blood flow especially if blood pressure falls below the range at which the kidney is able to autoregulate its blood flow. The result of this will be an increase of the specific gravity as water retention occurs in an attempt to maintain circulating volume.

Renal Physiology :- Overview - Each nephron is supplied with blood from an afferent arteriole that opens onto the glomerular capillary bed. Blood then flows to an efferent arteriole, supplying

Prepared by Dr: Mohammed Musa Brema Idress – My best wishes Page 46

the peritubular capillaries and medullary vasa recta. The kidney receives up to 25% of resting cardiac output.

Control of blood flow - The kidney is able to autoregulate its blood flow between systolic pressures of 80- 180mmHg so there is little variation in renal blood flow. This is achieved by myogenic control of arteriolar tone, both sympathetic input and hormonal signals (e.g. renin) are responsible.

Glomerular structure and function - Blood inside the glomerulus has considerable hydrostatic pressure. The basement membrane has pores that will allow free diffusion of smaller solutes, larger negatively charged molecules such as albumin are unable to cross. The glomerular filtration rate (GFR) is equal to the concentration of a solute in the urine, times the volume of urine produced per minute, divided by the plasma concentration (assuming that the solute is freely diffused e.g. inulin). In clinical practice creatinine is used because it is subjected to very little proximal tubular secretion. Although subject to variability, the typical GFR is 125ml per minute. Glomerular filtration rate = Total volume of plasma per unit time leaving the capillaries and entering the bowman's capsule. Renal clearance = volume plasma from which a substance is removed per minute by the kidneys

Substances used to measure GFR have the following features: 1) Inert 2) Free filtration from the plasma at the glomerulus (not protein bound) 3) Not absorbed or secreted at the tubules 4) Plasma concentration constant during urine collection Examples: inulin, creatinine

GFR = urine concentration (mmol/l) x urine volume (ml/min) ------plasma concentration (mmol/l)

The clearance of a substance is dependent not only on its diffusivity across the basement membrane but also subsequent tubular secretion and / or reabsorption. So glucose which is freely filtered across the basement membrane is usually reabsorbed from tubules giving a clearance of zero.

Tubular function - Reabsorption and secretion of substances occurs in the tubules. In the proximal tubule substrates such as glucose, amino acids and phosphate are co transported with sodium across the semi permeable membrane. Up to two thirds of filtered water is reabsorbed in the proximal tubules. This will lead to increase in urea concentration in the distal tubule allowing for its increased diffusion. Substances to be secreted into the tubules are taken up from the peritubular blood by tubular cells. Solutes such as para-aminohippuric acid are cleared with a single passage through the kidneys and this is why it is used to measure renal plasma flow. Ions such as

Prepared by Dr: Mohammed Musa Brema Idress – My best wishes Page 47 calcium and phosphate will have a tubular reabsorption that is influenced by plasma PTH levels. Potassium may be both secreted and re-absorbed and is co-exchanged with sodium.

Loop of Henle - Approximately 60 liters of water containing 9000mmol sodium enters the descending limb of the loop of Henle in 24 hours. Loops from the juxtamedullary nephrons run deep into the medulla. The osmolarity of fluid changes and is greatest at the tip of the papilla. The thin ascending limb is impermeable to water, but highly permeable to sodium and chloride ions. This loss means that at the beginning of the thick ascending limb the fluid is hypo osmotic compared with adjacent interstitial fluid. In the thick ascending limb the reabsorption of sodium and chloride ions occurs by both facilitated and passive diffusion pathways. The loops of Henle are co-located with vasa recta, these will have similar solute compositions to the surrounding extracellular fluid so preventing the diffusion and subsequent removal of this hypertonic fluid. The energy dependent reabsorption of sodium and chloride in the thick ascending limb helps to maintain this osmotic gradient.

152. A 28 year old man is shot in the abdomen and haemorrhages. Which of the following substances will produce vasoconstriction in response to this process? A. Renin B. Angiotensin I C. Angiotensin II D. Aldosteron E. None of the above

ANSWER IS C Renin does not cause vasoconstriction. Angiotensin I is biologically inactive. Aldosteron will increase blood pressure but does not have direct vasospastic effects.

153. Which of the following additional abnormal serum biochemistry test results is most suggestive of a chronic component to his renal failure? Single best answer - choose ONE true option only A. Hyperkalaemia B. Hyperuricaemia C. Hypocalcaemia D. Hyponatraemia E. Low serum bicarbonate concentration

ANSWER IS C Hyperkalaemia and hyperuricaemia (due to decreased excretion), hyponatraemia (due mainly to continued water intake with decreased ability to excrete it) and metabolic acidosis occur in both acute and chronic renal failure. The presence of hypocalcaemia in renal failure suggests that this is, at least in part, of longstanding, and is due to decreased renal synthesis of calcitriol (1,25-dihydroxycholecalciferol).

Prepared by Dr: Mohammed Musa Brema Idress – My best wishes Page 48

154. A patient has chronic renal impairment, with a plasma creatinine of 225µmol/l and the glomerular filtration rate (GFR) of 25ml/min. Concerning renal function: Single best answer question – choose ONE true option only A. The GFR is the main factor determining the rate of urine production B. GFR can be measured by para-aminohippuric acid (PAH) C. The normal GFR is 50mls/min D. Inulin clearance can be used to estimate GFR E. A normal plasma creatinine implies normal renal function

ANSWER IS D In the normal adult human the GFR (or normal renal clearance) averages 125ml/min, or 180 liters/day. The entire plasma volume (about 3 liters) can therefore be filtered and processed by the kidney approximately 60 times each day. The rate of urine production in humans is dominated by tubular function and not by GFR. The GFR remains relatively constant through autoregulation.

After 35 years of age, GFR falls at about 1ml/min/year. By the age of 80, GFR has fallen to about 50% of its youthful level. GFR can decrease by as much as 50% before plasma creatinine rises beyond the normal range. Consequently, a normal creatinine does not necessarily imply normal renal function, although a raised creatinine does usually indicate impaired renal function.

A substance used to measure the GFR must be freely filtered at the glomerulus, not be secreted by the tubules, not be reabsorbed, not be metabolized or synthesized in the body, not alter the renal function/GFR, be non-toxic and soluble in plasma. Such a substance is the poly fructose molecule, inulin. However, it is too cumbersome to use in routine clinical practice. Instead, GFR is more commonly quantified by measuring the 24-hour urinary creatinine excretion. Para-aminohippuric acid is used to measure renal blood flow and not GFR.

155. Aldosteron is secreted from the: Single best answer question – choose ONE true option only A. Liver B. Zona glomerulosa of the adrenal cortex C. Juxtaglomerular apparatus D. Adrenal medulla E. Zona fasciculata of the adrenal cortex

ANSWER IS B The adrenal gland comprises an outer cortex and an inner medulla, which represent two developmentally and functionally independent endocrine glands within the same anatomical structure. The adrenal medulla secretes adrenaline (70%) and noradrenaline (30%). The adrenal cortex consists of 3 layers, or zones. The layers from the surface inwards may be remembered by the mnemonic GFR: G = Zona glomerulosa (secretes aldosteron) F = Zona fasciculata (secretes cortisol and sex steroids)

Prepared by Dr: Mohammed Musa Brema Idress – My best wishes Page 49

R = Zona reticularis (secretes cortisol and sex steroids)

Aldosteron is a steroid hormone that facilitates the reabsorption of sodium and water and the excretion of potassium and hydrogen ions from the distal convoluted tubule and collecting ducts. Conn’s syndrome is characterized by increased aldosteron secretion from the adrenal glands.

156. Vasopressin (ADH) Single best answer question – choose ONE true option only A. Is synthesized in the posterior pituitary gland B. Deficiency leads to a risk of water intoxication C. Excessive secretion usually results in diabetes insipidus D. Increased plasma osmolarity is the primary physiological stimulus E. Acts on the proximal convoluted tubules of the kidney

ANSWER IS D Vasopressin is synthesized in the supraoptic nucleus of the hypothalamus and transported to the posterior pituitary via the axons. Excessive secretion is associated with the risk of impaired water excretion. Diabetes insipidus results from deficient secretion or action of this hormone leading to thirst and polyuria. It acts mainly on the distal convoluted tubules and the collecting ducts of the kidney.

157. The following factors stimulate renin release Single best answer question – choose ONE true option only A. Is decreased by isoflurane in general anaesthesia B. Propranolol C. Increase in plasma K+ concentration D. Angiotensin II E. Salt depletion

ANSWER IS E A decrease in blood pressure and salt depletion stimulates renin release. B-blockers and increased K+ levels inhibit renin release. Angiotensin II inhibits renin release through a negative feedback mechanism.

158. Which of the following hormones is secreted by the kidney in response to sympathetic nervous stimulation? Single best answer question – choose ONE true option only A. Aldosteron B. Angiotensin I C. Angiotensin II D. Erythropoetin E. Renin

ANSWER IS E Renin is produced by the juxtaglomerular apparatus of the kidney in response to hypovolaemia, via 3 mechanisms:

Prepared by Dr: Mohammed Musa Brema Idress – My best wishes Page 50

1) Increased catecholamine levels secondary to sympathetic stimulation from arterial receptors 2) Direct effect of hyponatraemia on the juxtaglomerular apparatus 3) Reduction of renal perfusion pressure via afferent arteriolar baroreceptors Renin acts to cleave angiotensin I from angiotensinogen produced in the liver.

Angiotensin converting enzyme is present in many tissues, especially the lungs, and converts angiotensin I to angiotensin II. Angiotensin II is a powerful vasoconstrictor, causing vasoconstriction of renal arteries, as well as a positive inotropic effect on the heart. It also causes release of ADH and adrenaline.

Along with aldosteron, whose release is also stimulated, Angiotensin II conserves Na+ and H2O in the gut. Aldosteron acts to conserve Na+ and H2O in the distal renal tubule and collecting ducts. These mechanisms combine to restore the plasma volume in hypovolaemia.

Erythropoetin is released by the kidney in response to hypoxia and high levels of the products of red cell breakdown, and increases the rate of red cell production.

159. Aldosteron causes the following effect Single best answer question – choose ONE true option only A. Na+, H2O loss and K+ conservation in the distal tubule B. Na+, H2O conservation and K+ loss in the distal tubule C. H+ loss in the distal tubule D. Na+, H2O loss and K+ conservation in the ascending limb of the loop of Henle E. Na+, H2O conservation and K+ loss in the ascending limb of the loop of Henle

ANSWER IS B Aldosteron is a steroid hormone produced by the zona glomerulosa of the adrenal cortex. Its release is stimulated mainly by the rennin/ angiotensin system. ACTH also causes aldosteron production, as do hyponatraemia, hyperkalaemia and hypovolaemia.

It regulates Na+/ K+ and water balance, as well as playing a role acid base balance. At the distal tubule, it acts to increase the Na+/K+ permeability of the luminal surface of the cells, and causes resorption of Na+ and H2O in exchange for K+, which is excreted into the urine. It also acts in the collecting ducts to secrete H+ ions, thereby regulating plasma HCO3- levels.

160. ADH (Vasopressin) release in response to dehydration causes Single best answer question – choose ONE true option only A. Decreased permeability of the collecting ducts to water B. Decreased urine osmolality C. Increased Na+ resorption in the ascending limb of the loop of Henle D. Increased Na+ resorption in the descending limb of the loop of Henle E. Increased permeability of the collecting ducts to water

Prepared by Dr: Mohammed Musa Brema Idress – My best wishes Page 51

ANSWER IS E ADH is released by the posterior pituitary in response to dehydration, from stimulation of osmoreceptors adjacent to the supraoptic nucleus, as well as volume receptors in the aorta atria and great veins. Water absorption in the collecting ducts is independent of sodium concentration, and is under the control of ADH, which causes increased permeability of the ducts. Increased ADH levels will increase the osmolality of the urine via this method.

In the descending limb of the loop of Henle, sodium and water are passively resorbed. The ascending limb is impermeable to water, with active sodium resorption, producing a concentration gradient in the renal medulla, which is essential for the maintenance of water balance.

161. Which of the following hormones is synthesized in the hypothalamus and secreted from the posterior pituitary? Single best answer question – choose ONE true option only A. Anti diuretic hormone (ADH) B. Adrenocorticotrophic hormone (ACTH) C. Corticotrophin releasing hormone (CRH) D. Thyrotrophin releasing hormone (TRH) E. Thyroid stimulating hormone (TSH)

ANSWER IS A Vasopressin (ADH) and oxytocin are synthesized in the hypothalamic nuclei and pass down axons to the posterior pituitary where they are secreted into the blood stream.

In contrast, the trophic hormones such as CRH and TRH are secreted by the hypothalamus in response to neural stimuli, and drain into the hypothalamo– hypophyseal portal vessels to the anterior pituitary. There is then resultant stimulation of ACTH and TSH secretion. The other hormones produced by a similar mechanism by the anterior pituitary are growth hormone (GH), prolactin (PRL), luteinizing hormone (LH) and follicle stimulating hormone (FSH).

162. The most common cause of hypokalaemia is? Single best answer question – choose ONE true option only A. Vomiting B. Severe diarrhoea C. Nephrotic syndrome D. Dietary deficiency E. Diuretic treatment

ANSWER IS E The most common cause of hypokalaemia is diuretic treatment with thiazides or loop diuretics, which causes increased renal excretion of potassium. Prolonged vomiting and severe diarrhoea can also induce marked hypokalaemia but are less common.

Prepared by Dr: Mohammed Musa Brema Idress – My best wishes Page 52

163. A 24-year-old, unconscious man is admitted to A&E. No history is available. The results of arterial blood gas analysis are: [H+] 80 nmol/l (pH 7.1), p(CO2) 7.0 kPa, p(O2) 8.2 kPa, [HCO3–] 17.1 mmol/l. These results indicate which one of the following acid–base disturbances? Single best answer question – choose ONE true option only A. Metabolic acidosis with respiratory compensation B. Mixed metabolic and respiratory acidosis C. Respiratory acidosis D. Respiratory acidosis with metabolic alkalosis E. Uncompensated metabolic acidosis

ANSWER IS B The high hydrogen-ion concentration (low pH) indicates acidosis. The elevated p(CO2) indicates a respiratory component; in compensated metabolic acidosis, p(CO2) is reduced; in an uncompensated metabolic acidosis (a very unusual situation, since the respiratory response to a metabolic acidosis is usually a rapid one), it would be normal. The hydrogen-ion concentration is too low to be accounted for by a respiratory acidosis alone: there must therefore be a metabolic acidosis in addition (as the low bicarbonate concentration also indicates).

164. A woman, aged 55, presents with features consistent with Cushing’s syndrome. She is taking no medication. Her basal cortisol and plasma ACTH levels are significantly raised. She has failed the dexamethasone suppression test. What is the most likely diagnosis? Single best answer question – choose ONE true option only A. Adrenal tumour B. Carney’s syndrome C. Cushing’s disease D. Depression E. Ectopic ACTH-secreting tumour

ANSWER IS C A raised adrenocorticotrophic hormone (ACTH) level with a raised cortisol implies the problem is caused by excess ACTH production, otherwise negative feedback would suppress ACTH. A low ACTH level would therefore be expected in patients with an adrenal tumour.

Carney’s syndrome comprises atrial myxoma and freckles with high cortisol levels independent of ACTH.

Pituitary tumours producing ACTH and adrenal stimulation are the cause of Cushing’s disease. While ectopic ACTH-secreting tumours are associated with significantly raised ACTH and cortisol levels, it’s unusual to develop classic cushinoid features. Failure of the dexamethasone suppression test can occur in patients with depression, but cushingoid features are not expected.

Prepared by Dr: Mohammed Musa Brema Idress – My best wishes Page 53

You are reviewing a man with significant early arterial disease. The brachial artery response post occlusion is severely diminished and confirms a diagnosis of endothelial dysfunction.

165. The successful production of which compound is best linked to vascular smooth muscle cell relaxation? Single best answer question – choose ONE true option only A. Angiotensin II B. Nitrous oxide C. Endothelin I D. Nitric oxide E. Serotonin

ANSWER IS D Nitric oxide (NO) is a rapidly diffusible gas with a half-life of only a few seconds. It is formed in endothelial cells via the metabolism of arginine and catalyzed by nitric oxide synthase (NOS).

NO has immunological and inflammatory functions, and also anti-atherosclerotic, antithrombotic effects. It also contributes to smooth muscle cell relaxation and the control of blood pressure.

Impaired brachial artery relaxation after a period of occlusion is a sign of endothelial dysfunction and associated with increased cardiovascular risk. This, at least in part, is due to endothelial damage and reduced NO production; in turn, reduced NO production brings about an increased susceptibility to further endothelial damage. Angiotensin II, endothelin I and serotonin are all vasoconstrictive agents

166. A patient has the following urea and electrolytes results: Sodium 140 mmol/l Potassium 4 mmol/l Chloride 105 mmol/l Bicarbonate 20mmol/l Calculate the anion gap. Single best answer - choose ONE true option only A. 19 meq/l B. 5 meq/l C. 10 meq/l D. 30 meq/l E. 0 meq/l

ANSWER IS A Anion gap = ([Na+] + [K+]) - ([Cl–] + [HCO3–]) (all units mmol/l). Normal range is 8–16 meq/l

167. In estimating the physiological clearance of 10 ml of an intravenous substance which has been administered at 10 mg/ml, the plasma concentration at equilibration is 15 mg/liter, the urine concentration is 150 mg/liter and the subject

Prepared by Dr: Mohammed Musa Brema Idress – My best wishes Page 54 produces 1440 ml of urine during a 24h collection. What is the clearance of the substance? Single best answer question – choose ONE true option only A. 1 ml/min B. 10 ml/min C. 0.1 ml/min D. 100 ml/min E. Cannot say from the information given

ANSWER IS B Clearance is calculated using the formula (U × V)/P where U = urine concentration in mg/ml, V = urine production in ml/min, P = plasma concentration in mg/ml. The bolus size of the substance is irrelevant to the clearance.

168. A 25-year-old man is admitted to hospital with persistent vomiting. He is clinically dehydrated and hypotensive. His serum sodium concentration is 124 mmol/l, potassium 4.9 mmol/l, urea 9.8 mmol/l, creatinine 96 mol/l. Urine sodium concentration in a specimen passed on admission is 62 mmol/l. Which of the following is the most likely cause of the hyponatraemia? Single best answer question – choose ONE true option only A. Adrenal failure B. Cerebral salt wasting C. Gastrointestinal fluid loss D. Low sodium intake E. Syndrome of inappropriate antidiuresis (SIAD)

ANSWER IS A Natriuresis in a dehydrated, hyponatraemic patient suggests that there is uncontrolled renal loss of sodium, such as occurs in adrenal failure. Cerebral salt wasting can also cause dehydration and hyponatraemia due to excessive natriuresis, but typically occurs following a head injury or brain surgery.

Hyponatraemia and dehydration due to gastrointestinal fluid loss or sodium deficiency due to a low intake should lead to renal conservation of sodium. Although SIAD is an important cause of hyponatraemia and sodium excretion may be high, the hyponatraemia is due to water excess and patients are not dehydrated.

169. Some 24 hours after sustaining severe skeletal and soft tissue trauma in a road traffic accident, a 19-year-old man becomes oliguric. Which of the following would most suggest that this is pre-renal in origin, rather than being due to acute tubular necrosis? Single best answer question – choose ONE true option only A. Glycosuria B. Hyperkalaemia C. Hyperuricaemia D. Urinary osmolality less than 250 mmol/kg E. Urinary sodium concentration less than 10 mmol/l

Prepared by Dr: Mohammed Musa Brema Idress – My best wishes Page 55

ANSWER IS E Trauma can cause transient glycosuria because of a stress-induced rise in blood glucose concentration. Tissue breakdown and decreased urine production can cause both hyperkalaemia and hyperuricaemia. A urinary sodium concentration less than 10 mmol/l suggests that renal tubular function is intact; whereas if oliguria is pre- renal, it would be expected that the urine would be concentrated as a result of hypovolaemic stimulation of vasopressin (antidiuretic hormone) secretion.

170. Which of the following statements concerning hyperacute rejection after renal transplantation is correct? Single best answer - choose ONE true option only A. Occurs at least three days after surgery B. Is successfully treated with cyclosporine A C. Is caused by cyclosporine A D. Is mediated by preformed circulating antibodies E. Is largely a B-cell-mediated response

ANSWER IS D Hyperacute rejection of a donor kidney may only take a few minutes once the organ has been vascularized. Preformed circulating antibodies react with MHC class 1 antigens on the transplanted kidney. A reaction ensues involving complement molecules, an influx of polymorphs and the aggregation of platelets. Blood vessels supplying the organ become obstructed causing ischaemia.

Cyclosporine A has no role in the treatment of hyperacute reactions: the only treatment is removal of the organ. However, circulating antibodies mean the next organ must be MHC 1-matched. Matching for MHC 2 is well known to prolong graft survival. The use of cyclosporine has revolutionized donor organ survival. It modifies T-cell responses, and is itself nephrotoxic.

171. In a patient with chronic hyponatraemia (sodium concentration 112 mmol/l), which of the following findings would most suggest a diagnosis of the syndrome of inappropriate [secretion of] antidiuretic hormone (SIADH)? Single best answer question - choose ONE true option only A. Normal cortisol response to ACTH B. Plasma albumin concentration 28 g/l C. Plasma osmolality 248 mosmol/kg D. Urinary osmolality 350 mosmol/kg E. Urinary sodium concentration < 20 mmol/l

ANSWER IS D The most significant finding is that the urine is concentrated relative to plasma. This is inappropriate, in that a low plasma osmolality should suppress ADH secretion and lead to the formation of a maximally dilute urine. The low plasma osmolality reflects the hyponatraemia, since sodium is the principal determinant of extracellular fluid osmolality. A low urine sodium excretion would suggest extrarenal sodium depletion as a cause of hyponatraemia. Addison’s disease, another cause of chronic hyponatraemia, is excluded by the cortisol response to ACTH, as it should be before

Prepared by Dr: Mohammed Musa Brema Idress – My best wishes Page 56

SIADH is diagnosed. The low plasma albumin concentration could just reflect dilution of the plasma by the excess water, but it could also suggest chronic liver disease, another cause of chronic hyponatraemia.

172. A 45-year-old woman with type-2 diabetes is making an apparently good recovery 7 days after a partial resection of the following trauma sustained in a stabbing incident. She is receiving parenteral nutrition with additional ‘normal’ saline and, because of a history of deep vein thrombosis some 10 years previously, is on prophylactic heparin. Before her admission she was well, with no ongoing medical problems and taking no regular medication. Serum electrolyte results are as follows: sodium 129 mmol/l, potassium 6.5 mmol/l, bicarbonate 24 mmol/l, urea 8.5 mmol/l, creatinine 120 µmol/l, glucose 10.2 mmol/l. Her potassium concentration has risen over the past 3 days. The potassium content of the parenteral feed has been reduced from 60 to 20 mmol/24 h during this period. Urine output is appropriate to her fluid input. Her red cell, white cell and platelet counts are all normal. What is the most likely cause of the hyperkalaemia? Single best answer question - choose ONE true option only A. Heparin treatment B. Overprovision of potassium in the parenteral feed C. Primary adrenal failure (Addison’s disease) D. Pseudohyperkalaemia E. Renal impairment

ANSWER IS A Approximately 20 mmol/24 h is the minimum obligatory potassium output, while the typical potassium requirements for patients on parenteral feeding are 40–80 mmol/24 h. Pseudohyperkalaemia is hyperkalaemia occurring as a result of a loss of potassium from white cells and platelets during clotting, usually seen in patients with high white cell or platelet counts.

Typically, the plasma potassium concentration is significantly lower than the serum potassium concentration in this condition. The elevated urea may be due to an excessive provision of amino acids, but neither it nor the creatinine level suggest sufficient renal impairment to cause such a severe hyperkalaemia.

Incipient adrenal failure could have been made overt by the stress of surgery, but this is uncommon. The heparin is more likely to be responsible: heparin inhibits aldosteron secretion by the adrenal cortex, leading to impaired renal potassium excretion, particularly in patients with diabetes or those who are acidotic.

173. Which of the following can result in a hypokalaemia? Single best answer question – choose ONE true option only A. Salbutamol B. Potassium sparing diuretics C. Renal failure D. ACE inhibitors

Prepared by Dr: Mohammed Musa Brema Idress – My best wishes Page 57

E. A metabolic acidosis

ANSWER IS A The sodium potassium ATPase pump exchanges intracellular sodium ions for extracellular potassium ions and is driven by the conversion of ATP to ADP. It is located on cell membranes and is the mechanism that ensures that sodium remains largely extracellular and potassium intracellular. It can also be driven by B sympathetic receptors. Therefore a b-agonist like salbutamol can reduce serum potassium levels and is used as a second line agent along with insulin in the emergency treatment of hyperkalaemia.

174. Which of the following is not associated with a metabolic acidosis? Single best answer question – choose ONE true option only A. A fall in serum bicarbonate B. Ketosis C. Hypokalaemia D. Hypovolaemic shock E. Hyperventilation

ANSWER IS C Serum potassium levels are intimately linked with serum H+ levels via the sodium potassium ATPase. This cell membrane pump principally exchanges intracellular sodium ions with extracellular potassium ions in order to maintain the cell membrane potential. However, potassium ions compete with hydrogen ions in the exchange pump and therefore in the presence of hypokalaemia, more hydrogen ions will move into the intracellular compartment via this pump. Conversely, in the presence of hyperkalaemia, less hydrogen ions will move out of the extracellular compartment which result in a metabolic acidosis.

175. Which of the following factors does NOT influence net filtration of water across a capillary wall? Single best answer question – choose ONE true option only A. The hydrostatic pressure gradient across the vessel wall B. The colloid osmotic (oncotic) pressure gradient across the vessel wall C. The thickness of the layer of smooth muscle cells in the vessel wall D. The surface area of the vessel wall E. The hydraulic conductance of the vessel wall

ANSWER IS C By definition, capillaries are only one endothelial cell thick and do not contain smooth muscle. The other factors all contribute to determining the net movement of water across a capillary wall. These factors are related by the Starling equation:- Net filtration of water = LpS [(Pc – Pi) – s(pc – pi)] Where: Lp → is hydraulic conductance (filtration rate per unit of pressure across the membrane). S → is the surface area of the vessel wall; Pc → is the hydrostatic pressure in the capillary

Prepared by Dr: Mohammed Musa Brema Idress – My best wishes Page 58

Pi → is the hydrostatic pressure in the interstitial space s → is the reflection coefficient pc → is the osmotic (oncotic) pressure in the capillary pi → is the osmotic (oncotic) pressure in the interstitial space In basic terms, this equation indicates that the net movement of water out of capillaries into the surrounding interstitial space is governed by the properties of the capillary wall itself and the difference between hydrostatic and osmotic driving forces acting across the capillary wall.

176. Which of the following features are NOT present in a patient with septic shock? Single best answer question – choose ONE true option only A. Increased systemic vascular resistance B. The systemic inflammatory response syndrome (SIRS) C. Hypotension D. Tachycardia E. Organ dysfunction

ANSWER IS A Reduced systemic vascular resistance (indicated by warm, vasodilated peripheries) and increased cardiac output (tachycardia) are present in septic shock. Some important and distinct definitions need to be known in relation to sepsis and septic shock:- - Bacteraemia → the presence of viable bacteria in the circulation. - Systemic inflammatory response syndrome (SIRS) → the bodies excessive inflammatory reaction to critical illness such as infection or trauma. Clinical criteria for SIRS include temperature >38 oC or <36 oC, heart rate >90/min, respiratory rate >20/min, and a white blood cell count of >12 x109/l, or <4 x109/l. - Sepsis → a known/presumed source of infection + the presence of 2 or more of the SIRS criteria. - Septic shock → the presence of sepsis, with resulting hypotension, leading to inadequate tissue perfusion and organ dysfunction.

177. Arterial blood gas analysis on a patient shows a plasma pH of 7.35, a PaCO2 of 6.5 and a plasma HCO3- concentration of 40 mmol/l. This is suggestive of which acid/base state? Single best answer question – choose ONE true option only A. Acute respiratory acidosis B. Compensated respiratory acidosis C. Acute metabolic acidosis D. Compensated metabolic acidosis E. Acute metabolic alkalosis

ANSWER IS B Reference ranges: pH plasma 7.36 – 7.44 PaCO2 4.7 – 6.0 kPa (35 – 54 mmHg) PaO2 >10.6 kPa [HCO3-] 20 – 29 mmol/l

Prepared by Dr: Mohammed Musa Brema Idress – My best wishes Page 59

Procedure for interpreting arterial blood gas results:- 1) Does the pH indicate acidosis or alkalosis? In this case the plasma pH suggests acidosis. 2) Is the PaCO2 elevated or depressed compared with the normal range? In this case the PaCO2 is elevated, suggesting a respiratory acidosis. 3) Is the [HCO3-] high or low compared with the normal range? In this case [HCO3-] is elevated (suggesting compensated respiratory acidosis), rather than being depressed (which would be the case for acute respiratory acidosis).

178. What is the main method by which intracellular pH is regulated? Single best answer question – choose ONE true option only A. The bicarbonate buffer system B. The phosphate buffer system C. Cytoplasmic proteins D. Carbonic anhydrase E. The globin component of haemoglobin

ANSWER IS C Cytoplasmic proteins provide the main contribution to pH buffering of the intracellular compartment. In the interstitial (i.e. extracellular and extravascular) compartment, the bicarbonate system is the main mechanism of pH buffering.

In the intravascular (plasma) compartment, pH buffering mechanisms include:- 1) The bicarbonate buffer system: CO2 + H2O « H2CO3 « H+ + HCO3- catalyzed by the enzyme carbonic anhydrase 2) The phosphate buffer system: HPO42- + H+ « H2PO4- 3) Plasma proteins 4) The globin component of haemoglobin

179. A patient is found to have hyponatraemia. Which condition should be excluded by subsequent investigations? Single best answer question – choose ONE true option only A. Diabetes insipidus B. Syndrome of inappropriate antidiuretic hormone secretion (SIADH) C. Diabetes mellitus D. Conn’s syndrome E. Cushing’s syndrome

ANSWER IS B SIADH causes excess water retention over sodium retention by promoting water reabsorption in the collecting ducts of the kidneys. This results in hyponatraemia. Diabetes insipidus and diabetes mellitus may both cause hypernatraemia by resulting in excess water loss over Na loss. In contrast, Conn’s syndrome and Cushing syndrome cause hypernatraemia by promoting excess sodium retention over water retention.

Prepared by Dr: Mohammed Musa Brema Idress – My best wishes Page 60

180. Which one of the following statements about renin secretion is true? Single best answer question – choose ONE true option only A. Renin is secreted by the epithelial cells of the renal glomerulus B. Reduced delivery of NaCl to the macula densa cells of nephrons increases renin secretion C. A rise in pressure in the renal afferent arteriole increases renin secretion D. Renin secretion is reduced by increased activity in the renal sympathetic nerves E. Renin secretion is reduced by inhibition of angiotensin-converting enzyme

ANSWER IS B Renin is an enzyme involved in activating the angiotensin-aldosteron system. It is produced and secreted by modified smooth muscle cells of the afferent arterioles of the kidney. Renin secretion is stimulated by a local fall in blood pressure in the afferent arterioles, by reduced delivery of filtered NaCl to the macula densa cells of the nephrons (‘tubulo-glomerular feedback’) and by increased activity in the renal sympathetic nerves.

Renin secretion is increased by inhibition of angiotensin-converting enzyme since the resulting reduction in angiotensin II and aldosteron levels reduces the negative feedback effect on renin secretion.

181. Which components of the nephron are most important with regulation of extracellular fluid osmolality? Single best answer question – choose ONE true option only A. Proximal convoluted tubule and distal convoluted tubule B. Glomerulus and distal convoluted tubule C. Loop of Henle and collecting ducts D. Glomerulus and proximal convoluted tubule E. Glomerulus and loop of Henle

ANSWER IS C Each component of the nephron is associated with particular predominant functions. The glomerulus is involved with passive filtration of the plasma and formation of tubular filtrate.

The proximal convoluted tubule is mainly involved with conservation of filtered solutes and water as well as secretion of certain waste products. The distal convoluted tubule plays a role in regulating preferential reabsorption of Na+ ions at the expense of K+ and H+ ions, under the control of aldosteron. It is the loop of Henle and collecting ducts which play the most important role in regulating extracellular fluid osmolality.

The loop of Henle creates the large medullary interstitial osmotic driving force for the reabsorption of water through the walls of the collecting ducts whose permeability is regulated by antidiuretic hormone (arginine vasopressin).

Prepared by Dr: Mohammed Musa Brema Idress – My best wishes Page 61

182. A 41-year-old woman presents with renal colic. Biochemistry results reveal a plasma calcium of 3.3mmols/L with an albumin of 45g/L. Her medical history includes dyspepsia and depression. What is the likely underlying diagnosis? Single best answer question – choose ONE true option only A. Actinomycosis infection B. Ectopic parathyroid hormone C. Hyperparathyroidism D. Hypoparathyroidism E. Vitamin D deficiency

ANSWER IS C Parathyroid hormone (PTH) and vitamin D are the major factors regulating serum calcium levels and bone turnover. PTH stimulates osteoclastic bone resorption, increases synthesis of 1,25 (OH)2 D3, increases intestinal/renal calcium absorption, and decreases phosphate absorption.

The symptoms of hypercalcaemia may be remembered by the mnemonic “stones, groans, and psychic overtones” i.e. associations with calculi formation, general aches/pains, and psychiatric problems. The most likely cause of hypercalcaemia in a fit, young patient is primary hyperparathyroidism (always consider malignancy). - Hyperparathyroidism may be primary, secondary, or tertiary:- A) Primary hyperparathyroidism is almost always due to a single functioning parathyroid adenoma. B) Secondary hyperparathyroidism is a normal physiological response and is seen following prolonged states of hypocalcaemia i.e. as one may see in renal failure. C) Tertiary hyperparathyroidism develops in cases of established secondary hyperparathyroidism where there is substantial parathyroid hyperplasia.

183. In a patient with small bowel ischaemia, what metabolic picture would most likely be seen on blood gas analysis? Single best answer question – choose ONE true option only A. Compensated metabolic acidosis B. Metabolic acidosis and increased anion gap C. Metabolic acidosis and normal anion gap D. Metabolic alkalosis E. Respiratory acidosis

ANSWER IS B This patient has had a significant operation during which infarcted bowel has been resected. The most likely abnormality is a metabolic acidosis secondary to mesenteric ischaemia and hypovolaemia resulting in anaerobic metabolism and accumulation of lactic acid. By definition, this patient will have a low arterial pH and a low bicarbonate concentration. An increased anion gap will also be seen. The anion gap may be calculated by:- [Na+] + [K+] – [CL-] – [HCO3-] - The normal anion gap is 8-16mmol/L.

Prepared by Dr: Mohammed Musa Brema Idress – My best wishes Page 62

The anion gap is a useful tool in differentiating between an acidosis due to the accumulation of organic acids e.g. lactic acid (as in this patient) and acidosis that are secondary to the loss of base or ingestion of acid where there will be a normal anion gap.

184. Concerning Glomerular Filtration, which of the following is true of the proximal convoluted tubule? Single best answer question – choose ONE true option only A. Reabsorbs water by sodium secretion B. Reabsorbs phosphate C. Increases the volume of reabsorbed fluid under aldosteron stimulation D. Contains renin-secreting cells E. Receives most of its blood supply from the vasa recta

ANSWER IS B The proximal convoluted tubule actively reabsorbs sodium. This sets up an osmotic gradient and water is drawn out of the tubule. It is the site of both phosphate and calcium reabsorption under control of parathyroid hormone. Aldosteron acts on the distal convoluted tubules. Renin is secreted by the cells of the juxtaglomerular apparatus in the distal convoluted tubules.

185. Which of the following is NOT a characteristic of the loop of Henle? Single best answer question – choose ONE true option only A. Is under the control of aldosteron B. Is permeable to water and electrolytes along its distal limb C. Proximal limb absorption is isotonic D. Generates high osmolality in the renal medulla E. Is impermeable to water along its distal limb

ANSWER IS E The loop of Henle’s main function is to produce a high medullary osmolality which is the driving force for water reabsorption from the collecting ducts. In the loop of Henle there is a concentration and reduction in volume of filtrate as sodium and chloride pass into the descending limb and water is osmotically moved out. In the ascending limb there is active reabsorption of sodium chloride from the filtrate producing a low osmolality filtrate. Aldosteron acts on the distal convoluted tubules and collecting ducts.

186. Which of the following is the site of renin production? Single best answer question – choose ONE true option only A. Collecting ducts B. Proximal convoluted tubule C. Loop of Henle D. Juxtaglomerular apparatus E. Liver

Prepared by Dr: Mohammed Musa Brema Idress – My best wishes Page 63

ANSWER IS D The juxtaglomerular apparatus is formed of specialized juxtaglomerular cells in the wall of afferent arterioles and macula densa of the distal convoluted ducts. Renin secretion is stimulated by reduced renal perfusion. Angiotensinogen is produced by the liver and is catalyzed by renin to form angiotensin I. This is in turn catalyzed by angiotensin converting enzyme (ACE) to produce angiotensin.

Angiotensin has several functions which aim to increase blood pressure and restore renal perfusion. It causes vasoconstriction, stimulates the adrenal cortex to produce aldosteron which promotes renal reabsorption of sodium and water from the distal convoluted tubules and collecting ducts.

187. What is the site of action of antidiuretic hormone (ADH) in a nephron? Single best answer question – choose ONE true option only A. Proximal convoluted tubule B. Ascending limb of loop of Henle C. Descending limb of loop of Henle D. Distal convoluted tubule E. Collecting duct

ANSWER IS E ADH is produced by the posterior pituitary gland in response to reduced extracellular osmolality, blood volume and blood pressure. It promotes reabsorption of water from the collecting ducts, resulting in reduced osmolality and expanded blood volume.

188. Which of the following is a function of atrial natruretic peptide (ANP)? Single best answer question – choose ONE true option only A. Increases renin secretion B. Decreases aldosteron secretion C. Promotes the effects of antidiuretic hormone (ADH) D. Causes renal vasoconstriction E. Promotes the feeling of thirst

ANSWER IS B ANP is released from atrial muscle cells when the atria are stretched due to increased circulating blood volume. Therefore ANP works to reduce blood volume by inhibiting the release of renin, aldosteron and ADH resulting in increased sodium and water excretion. It promotes renal vasodilatation

189. The glomerular filtration rate is increased by? Single best answer – choose ONE true option only A. Increased plasma colloid osmotic pressure B. Constriction of the glomerular afferent arterioles C. Constriction of the glomerular efferent arterioles D. Saline depletion E. Respiratory alkalosis

Prepared by Dr: Mohammed Musa Brema Idress – My best wishes Page 64

ANSWER IS C Constriction of the glomerular efferent arterioles increases the hydrostatic pressure within the glomerulus and hence the filtration pressure.

190. A 77-year-old man presents with a history of vomiting undigested food. Routine biochemistry shows a serum bicarbonate concentration of 38 mmol/l. Which of the following findings would most suggest that he had a chronic metabolic alkalosis? Single best answer question – choose ONE true option only A. Alkaline urine B. Base excess 18 mmol/l C. Elevated arterial p(CO2) D. Hypokalaemia E. Hypomagnesaemia

ANSWER IS C The base excess provides no additional information: it is directly related to the high bicarbonate concentration. In prolonged metabolic alkalosis, the urine may become acidic, reflecting increased proximal bicarbonate resorption (a consequence of hypochloraemia). Gastric secretions contain about 10 mmol/l potassium and, although potassium depletion is likely to become more severe the longer vomiting occurs, hypokalaemia can develop at any time. However, the development of hypercapnoea as compensation for metabolic alkalosis tends to take some time.

Although alkalosis inhibits respiration, the tendency for p (CO2) to increase acts as a respiratory stimulant, though with time, the sensitivity of the respiratory centre to carbon dioxide may decline so that significant hypoventilation does occur. Hypomagnesaemia is frequently found in patients with potassium depletion.

191. A 23 year old is stabbed in the groin and develops hypovolaemic shock. What is the most likely finding on analysis of his urine? A) Decreased specific gravity B) Increased specific gravity C) Increased urinary glucose D) Increased urinary protein E) Increased red blood cells in the urine

ANSWER IS B Hypovolaemic shock is likely to compromise renal blood flow especially if blood pressure falls below the range at which the kidney is able to autoregulate its blood flow. The result of this will be an increase of the specific gravity as water retention occurs in an attempt to maintain circulating volume.

192. There is decreased secretion of which one of the following hormones in response to major surgery: A) Insulin B) Cortisol C) Renin

Prepared by Dr: Mohammed Musa Brema Idress – My best wishes Page 65

D) Anti diuretic hormone E) Prolactin

ANSWER IS A Endocrine parameters reduced in stress response: 1) Insulin 2) Testosterone 3) Oestrogen - Insulin is often released in decreased quantities following surgery. - Stress response: Endocrine and metabolic changes 1) Surgery precipitates hormonal and metabolic changes causing the stress response 2) Stress response is associated with: substrate mobilization, muscle protein loss, sodium and water retention, suppression of anabolic hormone secretion, activation of the sympathetic nervous system, immunological and haematological changes. 3) The hypothalamic-pituitary axis and the sympathetic nervous systems are activated and there is a failure of the normal feedback mechanisms of control of hormone secretion. Sympathetic nervous system - Stimulates catecholamine release - Causes tachycardia and hypertension Pituitary gland - ACTH and growth hormone (GH) is stimulated by hypothalamic releasing factors, corticotrophin releasing factor (CRF) and somatotrophin (or growth hormone releasing factor) - Perioperative increased prolactin secretion occurs by release of inhibitory control - Secretion of thyroid stimulating hormone (TSH), luteinizing hormone (LH) and follicle stimulating hormone (FSH) does not change significantly - ACTH stimulates cortisol production within a few minutes of the start of surgery. More ACTH is produced than needed to produce a maximum adrenocortical response. Cortisol - Significant increases within 4-6h of surgery (>1000 nmol litre-1) - The usual negative feedback mechanism fails and concentrations of ACTH and cortisol remain persistently increased - The magnitude and duration of the increase correlate with the severity of stress and the response is not abolished by the administration of corticosteroids. - The metabolic effects of cortisol are enhanced: Skeletal muscle protein breakdown to provide gluconeogenic precursors and amino acids for protein synthesis in the liver stimulation of lipolysis 'Anti-insulin effect' Mineralocorticoid effects Anti-inflammatory effects Growth hormone - Increased secretion after surgery has a minor role - Most important for preventing muscle protein breakdown and promote tissue repair by insulin growth factors

Prepared by Dr: Mohammed Musa Brema Idress – My best wishes Page 66

Alpha Endorphin - Increased Antidiuretic hormone - An important vasopressor and enhances haemostasis - Renin is released causing the conversion of angiotensin I to angiotensin II, which causes the secretion of aldosteron from the adrenal cortex. This increases sodium reabsorption at the distal convoluted tubule Insulin - Release inhibited by stress - Occurs via the inhibition of the alpha cells in the pancreas by the α2-adrenergic inhibitory effects of catecholamines - Insulin resistance by target cells occurs later - The perioperative period is characterized by a state of functional insulin deficiency Thyroxine (T4) and tri-iodothyronine (T3) - Circulating concentrations are inversely correlated with sympathetic activity and after surgery there is a reduction in thyroid hormone production, which normalizes over a few days.

Metabolic effect of endocrine response:- Carbohydrate metabolism - Hyperglycaemia is a main feature of the metabolic response to surgery - Due to increased increase in glucose production and a reduction in glucose utilization - Catecholamines and cortisol promote glycogenolysis and gluconeogenesis - Initial failure of insulin secretion followed by insulin resistance affects the normal responses - The proportion of the hyperglycaemic response reflects the severity of surgery - Hyperglycaemia impairs wound healing and increase infection rates Protein metabolism - Initially there is inhibition of protein anabolism, followed later, if the stress response is severe, by enhanced catabolism - The amount of protein degradation is influenced by the type of surgery and also by the nutritional status of the patient - Mainly skeletal muscle protein is affected - The amino acids released from acute phase proteins (fibrinogen, C reactive protein, complement proteins, a2-macroglobulin, amyloid A and ceruloplasmin) and are used for gluconeogenesis - Nutritional support has little effect on preventing catabolism Lipid metabolism - Increased catecholamine, cortisol and glucagon secretion, and insulin deficiency, promotes lipolysis and ketone body production. Salt and water metabolism - ADH causes water retention, concentrated urine, and potassium loss and may continue for 3 to 5 days after surgery - Renin causes sodium and water retention

Prepared by Dr: Mohammed Musa Brema Idress – My best wishes Page 67

Cytokines - Glycoproteins - Interleukins (IL) 1 to 17, interferons, and tumour necrosis factor - Synthesized by activated macrophages, fibroblasts, endothelial and glial cells in response to tissue injury from surgery or trauma - IL-6 main cytokine associated with surgery. Peak 12 to 24 h after surgery and increase by the degree of tissue damage other effects of cytokines include fever, granulocytosis, haemostasis, tissue damage limitation and promotion of healing.

Modifying the response:- 1. Opioids suppress hypothalamic and pituitary hormone secretion 2. At high doses the hormonal response to pelvic and abdominal surgery is abolished. However, such doses prolong recovery and increase the need for postoperative ventilatory support 3. Spinal anaesthesia can reduce the glucose, ACTH, cortisol, GH and epinephrine changes, although cytokine responses are unaltered 4. Cytokine release is reduced in less invasive surgery 5. Nutrition prevents the adverse effects of the stress response. Enteral feeding improves recovery 6. Growth hormone and anabolic steroids may improve outcome 7. Normothermia decreases the metabolic response.

193. Which of the following statements related to the coagulation cascade is true? A. The intrinsic pathway is the main pathway in coagulation B. Heparin inhibits the activation of Factor 8 C. The activation of factor 8 is the point when the intrinsic and the extrinsic pathways meet D. Tissue factor released by damaged tissue initiates the extrinsic pathway E. Thrombin converts plasminogen to plasmin

ANSWER IS D The extrinsic pathway is the main path of coagulation. Heparin inhibits the activation of factors 2, 9, 10, 11. The activation of factor 10 is when both pathways meet. Thrombin converts fibrinogen to fibrin. During fibrinolysis plasminogen is converted to plasmin to break down fibrin.

Coagulation cascade - Two pathways lead to fibrin formation Intrinsic pathway (components already present in the blood) - Minor role in clotting - Subendothelial damage e.g. collagen - Formation of the primary complex on collagen by high-molecular-weight kininogen (HMWK), prekallikrein, and Factor 12 - Prekallikrein is converted to kallikrein and Factor 12 becomes activated - Factor 12 activates Factor 11 - Factor 11 activates Factor 9, which with its co-factor Factor 8a form the tenase complex which activates Factor 10

Prepared by Dr: Mohammed Musa Brema Idress – My best wishes Page 68

Extrinsic pathway (needs tissue factor released by damaged tissue) - Tissue damage - Factor 7 binds to Tissue factor - This complex activates Factor 9 - Activated Factor 9 works with Factor 8 to activate Factor 10 Common pathway - Activated Factor 10 causes the conversion of prothrombin to thrombin - Thrombin hydrolyses fibrinogen peptide bonds to form fibrin and also activates factor 8 to form links between fibrin molecules Fibrinolysis Plasminogen is converted to plasmin to facilitate clot resorption Intrinsic pathway Increased APTT Factors 8, 9, 11, 12 Extrinsic pathway Increased PT Factor 7 Common pathway Increased APTT & PT Factors 2, 5, 10 Vitamin K dependent Factors 2, 7, 9, 10 (1972)

194. Which of the following is not secreted by the parietal cells? A) Hydrochloric acid B) Mucus C) Magnesium D) Intrinsic factor E) Calcium

ANSWER IS B Chief of Pepsi cola = Chief cells secrete pepsinogen Parietal cells: secrete HCl, Ca, Na, Mg and intrinsic factor Surface mucosal cells: secrete mucus and bicarbonate

Gastric secretions A working knowledge of gastric secretions is important for surgery because peptic ulcers are common, surgeons frequently prescribe anti secretory drugs and because there are still patients around who will have undergone acid lowering procedures (Vagotomy) in the past.

Gastric acid - Is produced by the parietal cells in the stomach - pH of gastric acid is around 2 with acidity being maintained by the H+/K+ ATPase pump. As part of the process bicarbonate ions will be secreted into the surrounding vessels. - Sodium and chloride ions are actively secreted from the parietal cell into the canaliculus. This sets up a negative potential across the membrane and as a result sodium and potassium ions diffuse across into the canaliculus. - Carbonic anhydrase forms carbonic acid which dissociates and the hydrogen ions formed by dissociation leave the cell via the H+/K+ antiporter pump. At the same time sodium ions are actively absorbed. This leaves hydrogen and chloride ions in the canaliculus these mix and are secreted into the lumen of the oxyntic gland. Phases of gastric acid secretion there are 3 phases of gastric secretion: 1) Cephalic phase (smell / taste of food)

Prepared by Dr: Mohammed Musa Brema Idress – My best wishes Page 69

- 30% acid produced - Vagal cholinergic stimulation causing secretion of HCL and gastrin release from G cells 2) Gastric phase (distension of stomach ) - 60% acid produced - Stomach distension/low H+/peptides causes Gastrin release 3) Intestinal phase (food in duodenum) - 10% acid produced - High acidity/distension/hypertonic solutions in the duodenum inhibits gastric acid secretion via enterogastrones (CCK, secretin) and neural reflexes. Regulation of gastric acid production Factors increasing production include: 1. Vagal nerve stimulation 2. Gastrin release 3. Histamine release (indirectly following gastrin release) from enterchromaffin like cells Factors decreasing production include: 1. Somatostatin (inhibits histamine release) 2. Cholecystokinin 3. Secretin - Increases secretion of bicarbonate-rich fluid from pancreas and hepatic duct cells, decreases gastric acid secretion, trophic effect on pancreatic acinar cells - VIP Small intestine, pancreas - Neural Stimulates secretion by pancreas and intestines, inhibits acid and pepsinogen secretion - Somatostatin D cells in the pancreas and stomach - Fat, bile salts and glucose in the intestinal lumen. Decreases acid and pepsin secretion, decreases gastrin secretion, decreases pancreatic enzyme secretion, decreases insulin and glucagon secretion inhibits trophic effects of gastrin, stimulates gastric mucous production

195. A 45 year old male is diagnosed with carcinoma of the head of the pancreas. He reports that his stool sticks to the commode and will not flush away. Loss of which of the following enzymes is most likely to be responsible for this problem? A) Lipase B) Amylase C) Trypsin D) Elastase E) None of the above

ANSWER IS A Loss of lipase is one of the key features in the development of steatorrhoea which typically consists of pale and offensive stools that are difficult to flush away.

196. A 34 year old man receives morphine following an appendicectomy. He develops constipation as a result. Which of the following best accounts for this process? A) Stimulation of DOPA receptors

Prepared by Dr: Mohammed Musa Brema Idress – My best wishes Page 70

B) Inhibition of DOPA receptors C) Stimulation of µ receptors D) Stimulation of serotonin release E) Inhibition of serotonin release

ANSWER IS C 4 Types of opioid receptor: 1) δ (located in CNS)- Accounts for analgesic and antidepressant effects 2) k (mainly CNS)- analgesic and dissociative effects 3) µ (central and peripheral) - causes analgesia, miosis, decreased gut motility 4) Nociceptin receptor (CNS)- Affect of appetite and tolerance to µ agonists. Constipation is a common side effect of morphine treatment and stimulation of µ receptors accounts for this process.

197. Which of the following is not well absorbed following a gastrectomy? A) Vitamin c B) Zinc C) Vitamin B12 D) Copper E) Molybdenum

ANSWER IS C Vitamin B12. The others are unaffected Post gastrectomy syndrome: Rapid emptying food from stomach into the duodenum: diarrhoea, abdominal pain, hypoglycaemia Complications: Vitamin B12 and iron malabsorption, osteoporosis Treatment: High protein, low carbohydrate diet. Replace B12/Fe/Ca

198. Which vitamin is involved in the formation of collagen? A) Vitamin A B) Vitamin B C) Vitamin C D) Vitamin D E) Vitamin E

ANSWER IS C Vitamin C is needed for the hydroxylation of proline during collagen synthesis.

199. A 56 year old man has long standing chronic pancreatitis and develops pancreatic insufficiency. Which of the following will be absorbed normally? A) Fat B) Protein C) Folic acid D) Vitamin B12 E) None of the above

Prepared by Dr: Mohammed Musa Brema Idress – My best wishes Page 71

ANSWER IS C Pancreatic lipase is required for of fat, Proteases facilitate protein and B12 absorption. Folate digestion is independent of the pancreas.

200. A 56 year old male presents to the acute surgical take with severe abdominal pain. He is normally fit and well. He has no malignancy. The biochemistry laboratory contacts the ward urgently, his corrected calcium result is 3.6 mmol/l. What is the medication of choice to treat this abnormality? A) IV Pamidronate B) Oral Alendronate C) Dexamethasone D) Calcitonin E) IV Zoledronate

ANSWER IS A IV Pamidronate is the drug of choice as it most effective and has long lasting effects. Calcitonin would need to be given with another agent, to ensure that the hypercalcaemia is treated once its short term effects wear off. IV zoledronate is preferred in scenarios associated with malignancy.

201. The oxygen-haemoglobin dissociation curve is shifted to the right in which of the following scenarios? A) Hypothermia B) Respiratory alkalosis C) Low altitude D) Decreased 2,3-DPG in transfused red cells E) Chronic iron deficiency anaemia

ANSWER IS E Mnemonic to remember causes of right shift of the oxygen dissociation curve: CADET face RIGHT - C O2, A cidosis , 2,3-DPG, Exercise and Temperature The curve is shifted to the right when there is an increased oxygen requirement by the tissue. This includes: 1) Increased temperature 2) Acidosis 3) Increased DPG DPG is found in erythrocytes and is increased during glycolysis. It binds to the Hb molecule, thereby releasing oxygen to tissues. DPG is increased in conditions associated with poor oxygen delivery to tissues, such as anaemia and high altitude.

202. Which of the following physiological changes do not occur following tracheostomy? A) Alveolar ventilation is increased. B) Anatomical dead space is reduced by 50%. C) Work of breathing is increased. D) Proportion of ciliated epithelial cells in the trachea may decrease.

Prepared by Dr: Mohammed Musa Brema Idress – My best wishes Page 72

E) Splinting of the larynx may lead to swallowing difficulties.

ANSWER IS C Work of breathing is decreased which is one reasons it is popular option for weaning ventilated patients. Humidified air in this setting helps to reduce the viscosity of mucous that forms.

203. Which of the following does not stimulate insulin release? A) Gastrin B) Atenolol C) Protein D) Secretin E) Vagal cholinergic action

ANSWER IS B Beta blockers inhibit the release of insulin. Stimulation of insulin release: 1. Glucose 2. Amino acid 3. Vagal cholinergic 4. Secretin/Gastrin/CCK 5. Fatty acids 6. Beta adrenergic drugs

204. A 63 year old female is referred to the surgical clinic with an iron deficiency anaemia. Her past medical history includes a left hemi colectomy but no other co- morbidities. At what site is most dietary iron absorbed? A) Stomach B) Duodenum C) Proximal ileum D) Distal ileum E) Colon

ANSWER IS B Iron is best absorbed from the proximal small bowel (duodenum and jejunum) in the Fe 2+ state. Iron is transported across the small bowel mucosa by a divalent membrane transporter protein (hence the improved absorption of Fe 2+). The intestinal cells typically store the bound iron as ferritin. Cells requiring iron will typically then absorb the complex as needed

205. Which of the following drugs increases the rate of gastric emptying in the vagotomised stomach? A) Ondansetron B) Metoclopramide C) Cyclizine D) Erythromycin E) Chloramphenicol

Prepared by Dr: Mohammed Musa Brema Idress – My best wishes Page 73

ANSWER IS D Vagotomy seriously compromises gastric emptying which is why either a pyloroplasty or is routinely performed at the same time.

Chloramphenicol has no effect on gastric emptying. Ondansetron slows gastric emptying slightly. Metoclopramide increases the rate of gastric emptying but its effects are mediated via the vagus nerve.

206. Which of the following haemodynamic changes is not seen in hypovolaemic shock? A) Decreased cardiac output B) Increased heart rate C) Reduced left ventricle filling pressures D) Reduced blood pressure E) Reduced systemic vascular resistance

ANSWER IS E Cardiogenic Shock: e.g. MI, valve abnormality Increased SVR (vasoconstriction in response to low BP) increased HR (sympathetic response) decreased cardiac output decreased blood pressure

Hypovolaemic shock: blood volume depletion e.g. haemorrhage, vomiting, diarrhoea, dehydration, third-space losses during major operations. Increased SVR increased HR decreased cardiac output decreased blood pressure

Septic shock: occurs when the peripheral vascular dilatation causes a fall in SVR similar response may occur in anaphylactic shock, neurogenic shock. Reduced SVR increased HR normal/increased cardiac output decreased blood pressure. SVR will typically increase

Shock - Shock occurs when there is insufficient tissue perfusion. - The pathophysiology of shock is an important surgical topic and may be divided into the following aetiological groups: 1- Septic 2- Haemorrhagic 3- Neurogenic 4- Cardiogenic 5- Anaphylactic Septic shock: is a major problem and those patients with severe sepsis have a mortality rate in excess of 40%. In those who are admitted to intensive care mortality ranges from 6% with no organ failure to 65% in those with 4 organ failure. Sepsis: is defined as an infection that triggers a particular Systemic Inflammatory Response Syndrome (SIRS). This is characterized by body temperature outside 36 oC - 38 o C, HR >90 beats/min, respiratory rate >20/min, WBC count >12,000/mm3 or < 4,000/mm3.

Prepared by Dr: Mohammed Musa Brema Idress – My best wishes Page 74

Patients with infections and two or more elements of SIRS meet the diagnostic criteria for sepsis. Those with organ failure have severe sepsis and those with refractory hypotension -septic shock.

During the septic process there is marked activation of the immune system with extensive cytokine release. This may be coupled with or triggered by systemic circulation of bacterial toxins. These all cause endothelial cell damage and neutrophil adhesion. The overall hallmarks are thus those of excessive inflammation, coagulation and fibrinolytic suppression.

The surviving sepsis campaign highlights the following key areas for attention: 1) Prompt administration of antibiotics to cover all likely pathogens coupled with a rigorous search for the source of infection. 2) Haemodynamic stabilization. Many patients are hypovolaemic and require aggressive fluid administration. Aim for CVP 8-12 cm H2O, MAP > 65mmHg. 3) Modulation of the septic response. This includes manoeuvres to counteract the changes and includes measures such as tight glycaemic control, use of activated protein C and sometimes intravenous steroids.

In surgical patients, the main groups with septic shock include those with anastomotic leaks, abscesses and extensive superficial infections such as necrotizing fasciitis. When performing surgery the aim should be to undertake the minimum necessary to restore physiology. These patients do not fare well with prolonged surgery. Definitive surgery can be more safely undertaken when physiology is restored and clotting in particular has been normalized.

Haemorrhagic shock: the average adult blood volume comprises 7% of body weight. Thus in the 70 Kg adult this will equate to 5 liters. This changes in children (8-9% body weight) and is slightly lower in the elderly.

Decreasing blood pressure during haemorrhagic shock causes organ hypoperfusion and relative myocardial ischaemia. The cardiac index gives a numerical value for tissue oxygen delivery and is given by the equation: Cardiac index= 13.4 - [Hb] - SaO2 + 0.03 PaO2. Where Hb is haemoglobin concentration in blood and SaO2 the saturation and PaO2 the partial pressure of oxygen. Detailed knowledge of this equation is required for the MRCS Viva but not for part A, although you should understand the principle.

In patients suffering from trauma the most likely cause of shock is haemorrhage. However, the following may also be the cause or occur concomitantly: 1- Tension pneumothorax 2- Spinal cord injury 3- Myocardial contusion 4- Cardiac tamponade When assessing trauma patients it is worth remembering that in order to generate a palpable femoral pulse an arterial pressure of > 65mmHg is required.

Prepared by Dr: Mohammed Musa Brema Idress – My best wishes Page 75

Once bleeding is controlled and circulating volume normalized the levels of transfusion should be to maintain a Hb of 7-8 in those with no risk factors for tissue hypoxia and Hb 10 for those who have such risk factors.

Neurogenic shock: this occurs most often following a spinal cord transection, usually at a high level. There is resultant interruption of the autonomic nervous system. The result is either decreased sympathetic tone or increased parasympathetic tone, the effect of which is a decrease in peripheral vascular resistance mediated by marked vasodilation.

This results in decreased preload and thus decreased cardiac output (Starlings law). There is decreased peripheral tissue perfusion and shock is thus produced. In contrast with many other types of shock peripheral vasoconstrictors are used to return vascular tone to normal.

Cardiogenic shock: in medical patients the main cause is ischaemic heart disease. In the traumatic setting direct myocardial trauma or contusion is more likely. Evidence of ECG changes and overlying sternal fractures or contusions should raise the suspicion of injury. Treatment is largely supportive and transthoracic echocardiography should be used to determine evidence of pericardial fluid or direct myocardial injury. The measurement of troponin levels in trauma patients may be undertaken but they are less useful in delineating the extent of myocardial trauma than following MI.

When cardiac injury is of a blunt nature and is associated with cardiogenic shock the right side of the heart is the most likely site of injury with chamber and or valve rupture. These patients require surgery to repair these defects and will require cardiopulmonary bypass to achieve this. Some may require intra aortic balloon pump as a bridge to surgery.

Anaphylactic shock: anaphylaxis may be defined as a severe, life-threatening, generalized or systemic hypersensitivity reaction.

Anaphylaxis is one of the few times when you would not have time to look up the dose of a medication. The Resuscitation Council guidelines on anaphylaxis have recently been updated. Adrenaline is by far the most important drug in anaphylaxis and should be given as soon as possible. The recommended doses for adrenaline, hydrocortisone and chlorphenamine are as follows: Adrenaline Hydrocortisone Chlorphenamine < 6 months → 150 mcg (0.15ml 1 in 1,000) 25 mg 250 mcg/kg 6 months - 6 years → 150 mcg (0.15ml 1 in 1,000) 50 mg 2.5 mg 6-12 years → 300 mcg (0.3ml 1 in 1,000) 100 mg 5 mg Adult and child 12 years→ 500 mcg (0.5ml 1 in 1,000) 200 mg 10 mg - Adrenaline can be repeated every 5 minutes if necessary. The best site for IM injection is the anterolateral aspect of the middle third of the thigh. Common identified causes of anaphylaxis 1) Food (e.g. Nuts) - the most common cause in children

Prepared by Dr: Mohammed Musa Brema Idress – My best wishes Page 76

2) Drugs 3) Venom (e.g. Wasp sting)

207. Secretions from which of the following will contain the highest levels of potassium? A) Rectum B) Small bowel C) Gallbladder D) Pancreas E) Stomach

ANSWER IS A The rectum has the potential to generate secretions rich in potassium. This is the rationale behind administration of resins for hyperkalaemia and the development of hypokalaemia in patients with villous adenoma of the rectum. - Potassium secretion -GI tract - Potassium secretions → salivary glands variable may be up to 60mmol/L, stomach 10 mmol/L, bile 5 mmol/L, pancreas 4-5 mmol/L, small bowel 10 mmol/L, rectum 30 mmol/L. ❖ The above table provides average figures only and the exact composition varies depending upon the existence of disease, serum aldosteron levels and serum pH. - A key point to remember for the exam is that gastric potassium secretions are low. Hypokalaemia may occur in vomiting, usually as a result of renal wasting of potassium, not because of potassium loss in vomit.

208. In a 70 Kg male, what proportion of total body fluid will be contributed by plasma? A) 50% B) 5% C) 35% D) 65% E) 25%

ANSWER IS B - 70 Kg male = 42 L water (60% of total body weight) Fluid compartment physiology: Body fluid compartments comprise intracellular and extracellular compartments. The latter includes interstitial fluid, plasma and transcellular fluid. Typical figures are based on the 70 Kg male. Body fluid volumes Compartment Volume in liters Percentage of total volume Intracellular 28 L 60-65% Extracellular 14 L 35-40% Plasma 3 L 8% Interstitial 10 L 24% Transcellular 1 L 3%

209. A 23 year old man is undergoing an inguinal hernia repair under local anaesthesia. The surgeon encounters a bleeding site which he manages with diathermy. About a minute or so later the patient complains that he is able to feel the burning pain of the heat at the operative site. Which of the following nerve fibers is responsible for the transmission of this signal?

Prepared by Dr: Mohammed Musa Brema Idress – My best wishes Page 77

A) A α fibers B) A β fibers C) B fibers D) C fibers E) None of the above

ANSWER IS D Slow transmission of mechanothermal stimuli is transmitted via C fibers. A α fibers transmit information relating to motor proprioception, A β fibers transmit touch and pressure and B fibers are autonomic fibers.

210. What is the approximate volume of pancreatic secretions in a 24 hour period? A) 100ml B) 200ml C) 500ml D) 1500ml E) 3000ml

ANSWER IS D Typically the pancreas secretes up to 1.5L per day.

211. A 34 year old lady has just undergone a parathyroidectomy for primary hyperparathyroidism. The operation is difficult and all 4 glands were explored. The wound was clean and dry at the conclusion of the procedure and a suction drain inserted. On the ward she becomes irritable and develops respiratory stridor. On examination her neck is soft and the drain empty. Which of the following treatments should be tried initially? A) Administration of intravenous calcium gluconate B) Administration of intravenous lorazepam C) Removal of the skin closure on the ward D) Direct laryngoscopy E) Administration of calcichew D3 orally

ANSWER IS A Exploration of the parathyroid glands may result in impairment of the blood supply. Serum PTH levels. Can fall quickly and features of hypocalcaemia may ensue, these include neuromuscular irritability and laryngospasm. Prompt administration of intravenous calcium gluconate can be lifesaving. The absence of any neck swelling and no blood in the drain would go against a contained haematoma in the neck (which should be managed by removal of skin closure).

212. A 23 year old man presents with blunt abdominal trauma and a splenic bleed is suspected. He is commenced on an infusion of tranexamic acid. Which of the following best describes its mechanism of action? A) Inhibition of plasmin B) Inhibition of thrombin C) Inhibition of factor II

Prepared by Dr: Mohammed Musa Brema Idress – My best wishes Page 78

D) Inhibition of factor Xa E) Activation of factor VIII

ANSWER IS A Tranexamic acid inhibits plasmin and this prevents fibrin degradation. Tranexamic acid - Tranexamic acid is a synthetic derivative of lysine. Its primary mode of action is as a fibrinolytic that competitively inhibits the conversion of plasminogen to plasmin. Plasmin degrades fibrin and therefore rendering plasmin inactive slows this process. The role of tranexamic acid in trauma was investigated in the CRASH 2 trial and has been shown to be of benefit in bleeding trauma when administered in the first 3 hours.

213. A 34 year old male donates a unit of blood. It is stored at 4 cob After 72 hours which of the following clotting factors will be most affected? A) Factor V B) Factor II C) Factor VII D) Factor IX E) Factor XI

ANSWER IS A Factors V and VIII are sensitive to temperature which is the reason why FFP is frozen soon after collection.

214. Which of the following is not an intravenous colloid? A) Gelofusine B) Dextran 40 C) Human albumin solution D) Hydroxyethyl starch E) Bicarbonate 8.4%

ANSWER IS E Bicarbonate is a crystalloid

215. A 25 year old man undergoes an appendicetomy for appendicitis. The appendix is submitted for histopathological evaluation. Which of the following is most likely to be identified microscopically? A) Macrophages B) Neutrophils C) Fibroblasts D) Lymphocytes E) Stem cells

ANSWER IS B Neutrophil polymorphs are the cell type most commonly encountered in acute inflammation

Prepared by Dr: Mohammed Musa Brema Idress – My best wishes Page 79

216. A 48 year old women suffers blunt trauma to the head and develops respiratory compromise. As a result she develops hypercapnia. Which of the following effects is most likely to ensue? A) Cerebral vasoconstriction B) Cerebral vasodilation C) Cerebral blood flow will remain unchanged D) Shunting of blood to peripheral tissues will occur in preference to CNS perfusion E) None of the above

ANSWER IS B Hypercapnia will tend to produce cerebral vasodilation. This is of considerable importance in patients with cranial trauma as it may increase intracranial pressure

217. A surgeon is considering using lignocaine as local anaesthesia for a minor procedure. Which of the following best accounts for its actions? A) Blockade of neuronal acetylcholine receptors B) Blockade of neuronal nicotinic receptors C) Blockade of neuronal sodium channels D) Blockade of neuronal potassium channels E) Blockade of neuronal calcium channels

ANSWER IS C Lignocaine blocks sodium channels. They will typically be activated first, hence the pain some patients experience on administration.

218. A 22 year old man suffers a blunt head injury. He is drowsy and has a GCS of 7 on admission. Which of the following is the major determinant of cerebral blood flow in this situation? A) Systemic blood pressure B) Mean arterial pressure C) Intra cranial pressure D) Hypoxaemia E) Acidosis

ANSWER IS C Hypoxaemia and acidosis may both affect cerebral blood flow. However, in the traumatic situation increases in intracranial pressure are far more likely to occur especially when GCS is low. This will adversely affect cerebral blood flow.

219. A 43 year old man has recurrent episodes of dyspepsia and treatment is commenced with oral antacids. Which of the hormones listed below is released in response to increased serum gastrin levels and decreases intra gastric pH? A) Cholecystokinin B) Histamine C) Somatostatin D) Insulin

Prepared by Dr: Mohammed Musa Brema Idress – My best wishes Page 80

E) Vasoactive intestinal peptide

ANSWER IS B Histamine is released from enterochromaffin cells in the stomach mucosa which stimulates acid secretion. It is usually released in response to increased serum gastrin levels. Histamine blockers (e.g. cimetidine) were extremely popular treatments until the advent of proton pump inhibitors.

220. Which main group of receptors does dobutamine bind to? A) α-1 B) α-2 C) ß-1 D) ß-2 E) D-1

ANSWER IS C Dobutamine is asympathomimetic with both alpha- and beta-agonist properties; it displays a considerable selectivity for beta1-cardiac receptors.

Inotropes and cardiovascular receptors Inotrope cardiovascular receptor action 1) Adrenaline →α-1, α-2, β-1, β-2 2) Noradrenaline →α-1,( α-2), (β-1), (β-2) 3) Dobutamine →β-1, (β 2) 4) Dopamine →(α-1), (α-2), (β-1), D-1,D-2 minor receptor effects in brackets Effects of receptor binding α-1, α-2 vasoconstriction β-1 increased cardiac contractility and HR β-2 vasodilatation D-1 renal and spleen vasodilatation D-2 inhibits release of noradrenaline

221. Which of the following is responsible for the release and synthesis of calcitonin? A) Parathyroid glands B) Anterior pituitary C) Thyroid gland D) Posterior pituitary E) Adrenal glands

ANSWER IS C Calcitonin has the opposite effect of PTH and is release from the thyroid gland.

222. What is the half life of insulin in the circulation of a normal healthy adult? A) Less than 30 minutes B) Between 1 and 2 hours C) Between 2 and 3 hours D) Between 4 and 5 hours E) Over 6 hours

Prepared by Dr: Mohammed Musa Brema Idress – My best wishes Page 81

ANSWER IS A Insulin is degraded by enzymes in the circulation. It typically has a half life of less than 30 minutes. Abnormalities of the clearance of insulin may occur in type 2 diabetes.

223. Which of the following statements about blood clotting is untrue? A) Platelet adhesion to disrupted endothelium is dependent upon von Willebrand factor B) Protein C is a vitamin K dependent substance C) The bleeding time provides an assessment of platelet function D) The prothrombin time tests the extrinsic system E) Administration of aprotinin during liver transplantation surgery prolongs survival

ANSWER IS E Although aprotinin reduces fibrinolysis and thus bleeding, it is associated with increased risk of death and was withdrawn in 2007. Protein C is dependent upon vitamin K and this may paradoxically increase the risk of thrombosis during the early phases of warfarin treatment.

224. Which of the following cell types is least likely to be found in a wound 1 week following injury? A) Macrophages B) Fibroblasts C) Myofibroblasts D) Endothelial cells E) Neutrophils

ANSWER IS C Myofibroblasts are differentiated fibroblasts, in which the cytoskeleton contains actin filaments. These cell types facilitate wound contracture and are the hallmark of a mature wound.

225. The blood - brain barrier is not highly permeable to which of the following? A) Carbon dioxide B) Barbituates C) Glucose D) Oxygen E) Hydrogen ions

ANSWER IS E The blood brain barrier is relatively impermeable to highly dissociated compounds.

226. A 43 year old presents to the urology clinic complaining of impotence. Which of the following will occur in response to increased penile parasympathetic stimulation? A) Detumescence

Prepared by Dr: Mohammed Musa Brema Idress – My best wishes Page 82

B) Ejaculation C) Erection D) Vasospasm of the penile branches of the pudendal artery E) Contraction of the smooth muscle in the epididymis and vas deferens

ANSWER IS C Parasympathetic stimulation causes erection. Sympathetic stimulation will produce ejaculation, detumescence and vasospasm of the pudendal artery. It will also cause contraction of the smooth muscle in the epididymis and vas to convey the ejaculate.

227. In class II haemorrhagic shock in a 70Kg male, one would not expect to find? A) Blood loss greater than 750ml B) Tachycardia C) Normal blood pressure D) Urine output less than 20ml E) Anxiety

ANSWER IS D Urine output in class II shock (assuming 70Kg adult) is typically between 20 and 30ml

228. Which of the following best accounts for the action of PTH in increasing serum calcium levels? A) Activation of vitamin D to increase absorption of calcium from the small intestine. B) Direct stimulation of osteoclasts to absorb bone with release of calcium. C) Stimulation of phosphate absorption at the distal convoluted tubule of the kidney. D) Decreased porosity of the vessels at Bowman’s capsule to calcium. E) Vasospasm of the afferent renal arteriole thereby reducing GFR and calcium urinary loss.

ANSWER IS A PTH increases the activity of 1-α-hydroxylase enzyme, which converts 25hydroxycholecalciferol to 1, 25-dihydroxycholecalciferol, the active form of vitamin D. Osteoclasts do not have a PTH receptor and effects are mediated via osteoblasts.

229. Which of the following drugs does not cause syndrome of inappropriate anti diuretic hormone release? A) Haloperidol B) Carbamazepine C) Amitriptylline D) Cyclophosphamide E) Methotrexate

ANSWER IS E Drugs causing SIADH: ABCD

Prepared by Dr: Mohammed Musa Brema Idress – My best wishes Page 83

A nalgesics: opioids, NSAIDs, B arbiturates, C yclophosphamide/ Chlorpromazine/ Carbamazepine, D iuretic (thiazides)

230. Which of the following changes are not typically seen in established dehydration? A) Rising haematocrit B) Urinary sodium <20mmol/ liter C) Metabolic acidosis D) Decreased serum urea to creatinine ratio E) Hypernatraemia

ANSWER IS D Diagnosing dehydration can be complicated, laboratory features include: 1) Hypernatraemia 2) Rising haematocrit 3) Metabolic acidosis 4) Rising lactate 5) Increased serum urea to creatinine ratio 6) Urinary sodium <20 mmol/liter 7) Urine osmolality approaching 1200mosmol/kg

231. Which one of the following is least associated with thrombocytopenia? A) Heparin therapy B) Rheumatoid arthritis C) Infectious mononucleosis D) Liver disease E) Pregnancy

ANSWER IS B Rheumatoid arthritis, unlike systemic lupus erythematous, is generally associated with a thrombocytosis. In some cases of Felty's syndrome thrombocytopaenia may be seen secondary to hypersplenism. This however represents a small percentage of patients with rheumatoid arthritis.

232. Which of the following will increase the volume of pancreatic exocrine secretions? A) Octreotide B) Cholecystokinin C) Aldosteron D) Adrenaline E) None of the above

ANSWER IS B Cholecystokinin will often increase the volume of pancreatic secretions.

233. Where is the majority of iron found in the body? A) Bone

Prepared by Dr: Mohammed Musa Brema Idress – My best wishes Page 84

B) Haemoglobin C) Ferritin and haemosiderin D) Myoglobin E) Plasma iron

ANSWER IS B Approximately 70% of body iron is found bound to haemoglobin

234. A 44 year old man receives a large volume transfusion of whole blood. The whole blood is two weeks old. Which of the following best describes its handling of oxygen? A) It will have a low affinity for oxygen B) Its affinity for oxygen is unchanged C) It will more readily release oxygen in metabolically active tissues than fresh blood D) The release of oxygen in metabolically active tissues will be the same as fresh blood E) It will have an increased affinity for oxygen

ANSWER IS E Stored blood has less 2, 3 DPG and therefore has a higher affinity for oxygen, this reduces its ability to release it at metabolizing tissues.

235. Approximately what proportion of salivary secretions is provided by the submandibular glands? A) 10% B) 70% C) 40% D) 90% E) 20%

ANSWER IS B Although they are small, the submandibular glands provide the bulk of salivary secretions and contribute 70%, the sublingual glands provide 5% and the remainder from the parotid.

236. Which is not a cause of hyperuricaemia? A) Severe psoriasis B) Lesch-Nyhan syndrome C) Hyperthyroidism D) Diabetic ketoacidosis E) Alcohol

ANSWER IS C Mnemonic of the drugs causing hyperuricaemia as a result of reduced excretion of urate = 'Can't leap' C iclosporin, A lcohol, N icotinic acid, T hiazides

Prepared by Dr: Mohammed Musa Brema Idress – My best wishes Page 85

L oop diuretics, E thambutol, A spirin, P yrazinamide

237. Which of the following statements relating to low molecular weight heparins is false? A) They act via inhibition of Factor Xa B) Large doses may be used prior to commencing cardiopulmonary bypass C) They have a highly predictable pharmacokinetic profile D) They are derivatives of unfractionated heparin E) They have a molecular mass in the range of 3000-10000Da

ANSWER IS B As they are not easily reversed they are unsuitable for this purpose.

238. A 43 year old lady presents with urinary incontinence. At which of the following locations is Onufs nucleus likely to be found? A) Medulla oblongata B) Anterior horn of L5 nerve roots C) Micturition centre in the Pons D) Anterior horn of S2 nerve roots E) None of the above

ANSWER IS D Onufs nucleus is located in the anterior horn of S2 and is the origin of neurons to the external urethral sphincter.

239. Which opioid receptor does morphine attach to? A) mu B) alpha C) sigma D) beta E) kappa

ANSWER IS A Pethidine and other conventional opioids attach to this receptor. Opioids - Combine to specific opiate receptors in the CNS (periaqueductal grey matter, limbic system, substantia gelatinosa) - Morphine attaches to mu1 receptors

240. Which of the following inhibits the secretion of insulin? A) Adrenaline B) Glucagon C) Gastrin D) Arginine E) Vagal cholinergic activity

ANSWER IS A

Prepared by Dr: Mohammed Musa Brema Idress – My best wishes Page 86

241. The stimulation of nerve endings in the Golgi tendon organs leads directly to: A) Contraction of extrafusal muscle fibers B) Contraction of intrafusal muscle fibers C) Increased gamma-efferent discharge D) Increased activity in group II afferent fibers E) Reflex inhibition of motor neurons.

ANSWER IS E The stimulation of receptors in the Golgi tendon organs leads to the inverse stretch reflex. This reflex is responsible for the relaxation that is observed when a muscle is subjected to a strong stretch.

Impulses from the organs travel in type Ib fibers to the spinal cord, where they activate inhibitory interneurones. These in turn suppress the activity of motor neurons and therefore lead to relaxation of the extrafusal muscle fibers attached to the tendons.

The state of contraction of intrafusal fibers, the gamma-efferent discharge rate and the activity in group II afferent fibers control the stretch reflex, which, distinct from the inverse stretch reflex, is mediated by the Golgi tendon organs.

242. Which of the following is least likely to cause a prolonged prothrombin time? A) Cholestatic jaundice B) Disseminated intravascular coagulation C) Prolonged antibiotic treatment D) Liver disease E) Acquired factor 12 deficiency

ANSWER IS E Vitamin K deficiency results from cholestatic jaundice and prolonged antibiotic therapy. Acquired factor 12 deficiency causes prolonged APTT.

243. Which statement about peristalsis is true? A) Longitudinal smooth muscle propels the food bolus through the oesophagus B) Secondary peristalsis occurs when there is no food bolus in the oesophagus C) Food transfer from the oesophagus to the stomach is 4 seconds D) Circular smooth muscle is not involved in peristalsis E) Peristalsis only occurs in the oesophagus

ANSWER IS A Peristalsis - Circular smooth muscle contracts behind the food bolus and longitudinal smooth muscle propels the food through the oesophagus - Primary peristalsis spontaneously moves the food from the oesophagus into the stomach (9 seconds) - Secondary peristalsis occurs when food, which doesn't enter the stomach, stimulates stretch receptors to cause peristalsis

Prepared by Dr: Mohammed Musa Brema Idress – My best wishes Page 87

- In the small intestine each peristalsis waves slows to a few seconds and causes mixture of chyme - In the colon three main types of peristaltic activity are recognized

244. A 73 year old female is referred to the surgical clinic with an iron deficiency anaemia. As part of the diagnostic work up the doctor requests a serum ferritin level. Which of the conditions listed is most likely to lead to a falsely elevated result? A) Locally perforated sigmoid colonic adenocarcinoma B) Colonic angiodysplasia C) Dieulafoy lesion of the stomach D) Transitional cell carcinoma of the bladder E) Endometrial adenocarcinoma

ANSWER IS A A locally perforated colonic tumour will typically cause an intense inflammatory response and if peritonitis is not present clinically then at the very least a localized abscess. This inflammatory process is the most likely (from the list) to falsely raise the serum ferritin level. Angiodysplasia and dieulafoy lesions are mucosal arteriovenous malformations and unlikely to result in considerable inflammatory activity.

245. Which of the following is not a cause of hypercalcaemia? A) Thiazides B) Antacids C) Coeliac disease D) Sarcoidosis E) Zolinger-Ellison syndrome

ANSWER IS C Mnemonic for the causes of hypercalcaemia: CHIMPANZEES 1) C alcium supplementation 2) H yperparathyroidism 3) I atrogentic (Drugs: Thiazides) 4) M ilk Alkali syndrome 5) P aget disease of the bone 6) A cromegaly and Addison's Disease 7) N eoplasia 8) Z olinger-Ellison Syndrome (MEN Type I) 9) E xcessive Vitamin D 10) E xcessive Vitamin A 11) S arcoidosis Patients with coeliac disease tend to develop hypocalcaemia due to malabsorption of calcium by the bowel.

246. Which of the following surgical procedures will have the greatest long term impact on patient’s calcium metabolism?

Prepared by Dr: Mohammed Musa Brema Idress – My best wishes Page 88

A) Distal gastrectomy B) Cholecystectomy C) Extensive small bowel resection D) Subtotal colectomy E) Gastric banding for obesity

ANSWER IS C Calcium is mainly absorbed from the small bowel and this will have a direct long term impact on calcium metabolism and increase the risk of osteoporosis. Gastric banding and distal gastrectomy may affect a patients dietary choices but any potential deleterious nutritional intake may be counteracted by administration of calcium supplements orally.

Only 10% of calcium is absorbed from the colon so that a sub total colectomy will only have a negligible effect.

247. A 52-year-old woman with a history of gastrectomy reports lethargy and a sore tongue. Blood tests are reported as follows: Hb 10.7 g/dl MCV 121 fl Plt 177 * 10^9/l WBC 5.4 * 10^9/l What is the most likely cause? A) Vitamin B12 deficiency B) Vitamin C deficiency C) Iron deficiency anaemia D) Anaemia of chronic disease E) Vitamin E deficiency

ANSWER IS A A history of gastrectomy and a macrocytic anaemia should indicate a diagnosis of B12 deficiency.

Vitamin B12 deficiency Vitamin B12 is mainly used in the body for red blood cell development and also maintenance of the nervous system. It is absorbed after binding to intrinsic factor (secreted from parietal cells in the stomach) and is actively absorbed in the terminal ileum. A small amount of vitamin B12 is passively absorbed without being bound to intrinsic factor. Causes of vitamin B12 deficiency 1. Pernicious anaemia 2. Post gastrectomy 3. Poor diet 4. Disorders of terminal ileum (site of absorption): Cohn’s, blind-loop etc

Features of vitamin B12 deficiency 1. Macrocytic anaemia

Prepared by Dr: Mohammed Musa Brema Idress – My best wishes Page 89

2. Sore tongue and mouth 3. Neurological symptoms: e.g. Ataxia 4. Neuropsychiatric symptoms: e.g. Mood disturbances

Management 1) If no neurological involvement 1 mg of IM hydroxocobalamin 3 times each week for 2 weeks, then once every 3 months 2) If a patient is also deficient in folic acid then it is important to treat the B12 deficiency first to avoid precipitating subacute combined degeneration of the cord

248. A 43 year old lady is diagnosed with primary hyperparathyroidism. Her serum PTH levels are elevated. An endocrine surgeon performs a parathyroidectomy. How long will it take for the serum PTH levels to fall if the functioning adenoma has been successfully removed? A) 6 hours B) 24 hours C) 2 hours D) 1 hour E) 10 minutes

ANSWER IS E PTH has a very short half life usually less than 10 minutes. Therefore a demonstrable drop in serum PTH should be identified within 10 minutes of removing the adenoma. This is useful clinically since it is possible to check the serum PTH intraoperatively prior to skin closure and explore the other glands if levels fail to fall

249. Which of the following statements relating to abnormal coagulation is false? A) Warfarin affects the synthesis of factor 2,7,9,10 B) The prothrombin time is prolonged in Haemophilia A C) Cholestatic jaundice can cause vitamin K deficiency D) Disseminated intravascular coagulation is associated with thrombocytopenia E) Massive transfusion is associated with reduced levels of factor 5 and 8

ANSWER IS B In haemophilia A the APTT is prolonged and there is reduced levels of factor 8:C. The bleeding time and PT are normal. Cholestatic jaundice prevents the absorption of the fat soluble vitamin K. Massive transfusion (>10u blood or equivalent to the blood volume of a person) puts the patient at risk of thrombocytopaenia, factor 5 and 8 deficiency.

250. A 34 year old man presents with a peptic ulcer. Which of the following is responsible for the release of gastric acid? A) Chief cells B) Parietal cells C) Brunner’s Glands D) G Cells

Prepared by Dr: Mohammed Musa Brema Idress – My best wishes Page 90

E) None of the above

ANSWER IS B Parietal cells are responsible for the release of gastric acid. Brunner’s glands are found in the duodenum.

251. Which of the following does not lead to relaxation of the lower oesophageal sphincter? A) Metoclopramide B) Botulinum toxin type A C) Nicotine D) Alcohol E) Theophylline

ANSWER IS A Metoclopramide acts directly on the smooth muscle of the LOS to cause it to contract. Theophylline is a phosphodiesterase inhibitor (mimics action of prostaglandin E1) which causes relaxation of the LOS.

252. Which of the following is not classically seen in coning resulting from raised intra cranial pressure? A) Coma B) Hypotension C) Unreactive midsized pupils D) Cheyne Stokes style respiratory efforts E) Bradycardia

ANSWER IS B Cushing’s triad 1) Widening of the pulse pressure 2) Respiratory changes 3) Bradycardia Due to raised ICP systemic hypertension is usually seen. Compression of the respiratory centre will typically result in Cheyne Stokes style respiration.

Coning - The cranial vault is a confined cavity apart from infants with a non fused fontanelle. - Rises in ICP may be accommodated by shifts of CSF. - Once the CSF shifting has reached its capacity ICP will start to rise briskly. - The brain autoregulates its blood supply, as ICP rises the systemic circulation will display changes to try and meet the perfusion needs of the brain. Usually this will involve hypertension. - As CSF rises further, the brain will be compressed, cranial nerve palsies may be seen and compression of essential centers in the brain stem will occur. When the cardiac centre is involved bradycardia will often develop

Prepared by Dr: Mohammed Musa Brema Idress – My best wishes Page 91

253. Which of the following is not released from the islets of Langerhans? A) Pancreatic polypeptide B) Glucagon C) Secretin D) Somatostatin E) Insulin

ANSWER IS C Secretin is released from mucosal cells in the duodenum and jejunum.

254. Which of the following statements is true of glucagon? A) Produced in response to hyperglycaemia B) Released by beta cells C) Inhibits gluconeogenesis D) Produced in response to an increase of amino acids E) Composed of 2 alpha polypeptide chains linked by hydrogen bonds

ANSWER IS D Glucagon is a protein comprised of a single polypeptide chain. Produced by alpha cells of pancreatic islets of Langerhans in response to hypoglycaemia and amino acids. It increases plasma glucose and ketones.

255. A 28 year old man undergoes a completion right hemicolectomy for treatment of a 5cm appendiceal carcinoid. As part of his follow up he is due to undergo 24 hour urine collection for 5-HIAA. Which of the following causes an elevated 5-HIAA in a 24-hour urine collection? A) Naproxen B) Oranges C) Flucloxacillin D) Amiodarone E) Beef

ANSWER IS A It is important to be aware of what can falsely elevate 5-HIAA to avoid diagnosing carcinoid syndrome incorrectly. These include: A. Food: spinach, cheese, wine, caffeine, tomatoes B. Drugs: Naproxen, Monoamine oxidase inhibitors Recent surgery. 256. Intra cranial pressure is governed by the principles of the Monroe-Kellie doctrine. To which of the following does this concept not apply? A) A 2 month old child B) A 2 year old child C) A 5 year old child D) A 10 year old child E) An adult

ANSWER IS A - The Monroe-Kelly Doctrine assumes that the cranial cavity is a rigid box.

Prepared by Dr: Mohammed Musa Brema Idress – My best wishes Page 92

- In children with non-fused fontanells this is not the case.

Applied neurophysiology - Pressure within the cranium is governed by the Monroe-Kelly doctrine. This considers the skull as a closed box. Increases in mass can be accommodated by loss of CSF. Once a critical point is reached (usually 100- 120ml of CSF lost) there can be no further compensation and ICP rises sharply. The next step is that pressure will begin to equate with MAP and neuronal death will occur. Herniation will also accompany this process.

- The CNS can autoregulate its own blood supply. Vasoconstriction and dilatation of the cerebral blood vessels is the primary method by which this occurs. Extremes of blood pressure can exceed this capacity resulting in risk of stroke. Other metabolic factors such as hypercapnia will also cause vasodilation, which is of importance in ventilating head injured patients. - The brain can only metabolize glucose, when glucose levels fall, consciousness will be impaired.

257. Which of the following features does not put a patient at risk of re-feeding syndrome? A) BMI < 16 kg/m2 B) Alcohol abuse C) Thyrotoxicosis D) Chemotherapy E) Diuretics

ANSWER IS C Diuretics increase the risk of re-feeding syndrome through a process of increasing the risk of depletion of key electrolytes.

258. A 25-year-old man who has been morbidly obese for the past five years is reviewed in the surgical bariatric clinic. In this patient, release of which of the following hormones would increase appetite? A) Leptin B) Thyroxine C) Adiponectin D) Ghrelin E) Serotonin

ANSWER IS D Obesity hormones A) Leptin decreases appetite B) Ghrelin increases appetite Whilst thyroxine can increase appetite it does not fit with the clinical picture being described.

Prepared by Dr: Mohammed Musa Brema Idress – My best wishes Page 93

259. Which of the following bony complications is not linked to excess glucocorticoids? A) A vascular necrosis B) Vertebral body collapse C) Increased susceptibility to osteomyelitis from strep viridans D) Decreased absorption of calcium from the gut E) Growth retardation in children

ANSWER IS C This infection is not typical of steroid excess, although general increased susceptibilty to infections is.

260. A 54-year-old woman is admitted to the Surgical Admissions Unit with abdominal pain. Blood tests taken on admission show the following: Magnesium 0.40 mmol/l (normal value 0.7-1.0 mmol/l). Which one of the following factors is most likely to be responsible for this result? A) Excessive resuscitation with intravenous saline B) Digoxin therapy C) Diarrhoea D) Hypothermia E) Rhabdomyolysis

ANSWER IS C Hypomagnasaemia Cause of low magnesium 1) Diuretics 2) Total parenteral nutrition 3) Diarrhoea 4) Alcohol 5) Hypokalaemia, hypocalcaemia Features 1) Paraesthesia 2) Tetany 3) Seizures 4) Arrhythmias 5) Decreased PTH secretion --> hypocalcaemia 6) ECG features similar to those of hypokalaemia 7) Exacerbates digoxin toxicity

261. A 43 year old man has a nasogastric tube inserted. The nurse takes a small aspirate of the fluid from the stomach and tests the pH of the aspirate. What is the normal intragastric pH? A) 0.5 B) 2 C) 4 D) 5 E) 6

Prepared by Dr: Mohammed Musa Brema Idress – My best wishes Page 94

ANSWER IS B The intragastric pH is usually 2. Administration of proton pump inhibitors can result in almost complete abolition of acidity

262. A 73 year old female is referred to the surgical clinic with an iron deficiency anaemia. As part of the diagnostic work up the doctor requests a serum ferritin level. Which of the conditions listed is most likely to lead to a falsely elevated result? A) Locally perforated sigmoid colonic adenocarcinoma B) Colonic angiodysplasia C) Dieulafoy lesion of the stomach D) Transitional cell carcinoma of the bladder E) Endometrial adenocarcinoma

ANSWER IS A A locally perforated colonic tumour will typically cause an intense inflammatory response and if peritonitis is not present clinically then at the very least a localized abscess. This inflammatory process is the most likely (from the list) to falsely raise the serum ferritin level. Angiodysplasia and dieulafoy lesions are mucosal arteriovenous malformations and unlikely to result in considerable inflammatory activity.

263. Which of the following is least likely to cause a prolonged prothrombin time? A) Cholestatic jaundice B) Disseminated intravascular coagulation C) Prolonged antibiotic treatment D) Liver disease E) Acquired factor 12 deficiency

ANSWER IS E

264. Which statement about peristalsis is true? A) Longitudinal smooth muscle propels the food bolus through the oesophagus B) Secondary peristalsis occurs when there is no food bolus in the oesophagus C) Food transfer from the oesophagus to the stomach is 4 seconds D) Circular smooth muscle is not involved in peristalsis E) Peristalsis only occurs in the oesophagus

ANSWER IS A

265. You review a 39-year-old sportsman who complains of knee pain. Arthroscopy reveals damage to the cartilage. Which of the following stems best describes a property of hyaline cartilage? Single best answer question – choose ONE true option only A) It has a blood supply from small arterioles B) It is rich in type 1 collagen C) Chondrocytes secrete collagen only

Prepared by Dr: Mohammed Musa Brema Idress – My best wishes Page 95

D) It is a vascular E) Pressure from normal joint loading accelerates damage to cartilage

ANSWER IS D Hyaline cartilage forms the articular surface and is a vascular, relying on diffusion from synovial fluid for nutrients. It is rich in type II collagen and forms a meshwork containing proteoglycan molecules that retain water.

Intermittent pressure from joint loading is essential to maintain normal cartilage function. Chondrocytes secrete proteoglycans and collagen and are embedded in the cartilage. They migrate to the joint surface along with the matrix that they produce.

266. A 54-year-old woman has undergone some blood tests as part of an employment health screen. She reports she is in good health and, being very health conscious, takes regular vitamin and mineral supplements. She is taking bendrofluazide 2.5 mg for hypertension and her blood pressure is 132/82 mmHg. The only abnormality is a serum calcium concentration of 2.94 mmol/l. Which of the following is the most likely cause? Single best answer question – choose ONE true option only A) Diuretic treatment B) High dietary calcium intake C) High dietary vitamin D intake D) Occult malignancy E) Primary hyperparathyroidism

ANSWER IS E Thiazides can cause hypercalcaemia but it is usually only mild. Vitamin D itself is physiologically inactive and, whereas 1-hydroxylated derivatives can be a cause of hypercalcaemia, vitamin D – which has to be metabolized to activate it – is less commonly so.

Intestinal absorption of calcium is subject to tight control, and a high intake does not cause hypercalcaemia. The two most common causes of hypercalcaemia are primary hyperparathyroidism and malignancy. In an asymptomatic individual, primary hyperparathyroidism is the more likely cause.

267. A 24-year-old woman undergoes resection of the terminal ileum with fashioning of an ileostomy for Cohn’s disease. Some 2 weeks after surgery, she is making a good recovery, and is eating a high-energy, low-residue diet, but has a high ileostomy volume, necessitating intravenous fluid replacement. Her serum calcium concentration is 1.82 mmol/l, phosphate 1.28 mmol/l, alkaline phosphatase 82 U/l (normal < 150), albumin 30 g/l, creatinine 80 mol/l. Prior to surgery, her serum calcium concentration was 2.18 mmol/l, albumin 36 g/l. What is the most likely cause of her hypocalcaemia? Single best answer question – choose ONE true option only A) Formation of insoluble calcium salts in the intestine B) Hypoalbuminaemia

Prepared by Dr: Mohammed Musa Brema Idress – My best wishes Page 96

C) Hypomagnesaemia D) Malabsorption of calcium E) Malabsorption of vitamin D

ANSWER IS C Impaired fat absorption can lead to the formation of insoluble calcium salts in the gut. Fat and calcium are absorbed in the proximal small intestine, so, too, is vitamin D. Although bile salts are absorbed distally, and impaired absorption can lead to a secondary decrease in proximal fat absorption, this is unlikely to be responsible for hypocalcaemia developing so quickly.

The normal alkaline phosphatase level also militates against vitamin D deficiency. Hypocalcaemia would normally be expected to stimulate parathyroid hormone secretion and cause the plasma phosphate concentration to fall (PTH is phosphaturic).

Patients with ileostomies can lose large amounts of magnesium through their stomas; hypomagnesaemia impairs PTH secretion and can cause hypocalcaemia that is resistant to an increased provision of calcium.

268. An overweight 32-year-old woman presents with a short history of painless jaundice. There is no previous history of illness and, apart from the jaundice, she has no signs of chronic liver disease. Initial investigations reveal a haemoglobin of 12.7 g/dl, MCV 105 fl, serum bilirubin 162 mol/l, AST 145 U/l, alkaline phosphatase 224 U/l, gamma-glutamyltransferase 200 U/l. Which of the following is the most likely diagnosis? Single best answer question – choose ONE true option only A) Alcoholic liver disease B) Autoimmune chronic hepatitis C) Carcinoma of the head of the pancreas D) Cholecystitis E) Hepatitis A infection

ANSWER IS A Jaundice with an elevation of both AST and alkaline phosphatase suggests mixed hepatocellular damage and cholestatic liver disease, typical of acute alcoholic hepatitis on a background of chronic liver disease (and is not excluded by the lack of physical signs). The high gamma-glutamyltransferase lends support to this (although it may be increased in liver disease of any cause).

Macrocytosis is typical of chronic excessive alcohol intake and is not a feature of the other conditions; although were it not present, autoimmune liver disease would need to be considered.

In hepatitis A, AST is typically higher than alkaline phosphatase, while the reverse is true of pancreatic carcinoma. Chronic cholecystitis can cause jaundice but it would be unusual for there to be no history of acute episodes.

Prepared by Dr: Mohammed Musa Brema Idress – My best wishes Page 97

269. A 75-year-old woman is being followed by her GP for suspected developing primary hypothyroidism. Which of the following biochemical changes would you most expect to occur first? Single best answer question – choose ONE true option only A) Fall in serum free thyroxine B) Fall in serum thyroxine-binding globulin C) Fall in serum free triiodothyronine D) Fall in serum total triiodothyronine E) Increase in serum TSH

ANSWER IS E Hypothyroidism develops gradually, often over many months or even years. In the early stages, free thyroxine concentrations are maintained in the normal range by the increased secretion of TSH.

Patients with a slightly elevated TSH and low–normal thyroxine are said to have ‘compensated’ or ‘borderline’ hypothyroidism. In some individuals, it appears that this state can be maintained without progression to frank hypothyroidism. Triiodothyronine concentrations tend to fall later than thyroxine concentrations in hypothyroidism; the concentration of thyroxine-binding globulin does not change significantly.

270. A 52-year-old woman undergoes investigation for jaundice. She first noticed this symptom 2 months ago, but for 4 months prior to that, she had been experiencing generalized pruritus. The results of liver function tests are as follows: serum bilirubin 325 mol/l, aspartate aminotransaminase 55 U/l (15–42), alkaline phosphatase 436 U/l (80–150), gamma-glutamyltransferase 82 U/l (11–51), albumin 36 g/l, total protein 82 g/l. Which of the following is the most likely diagnosis? Single best answer question – choose ONE true option only A) Alcoholic cirrhosis B) Carcinoma of the head of the pancreas C) Cholangiocarcinoma D) Primary biliary cirrhosis E) Primary sclerosing cholangitis

ANSWER IS D The high alkaline phosphatase concentration suggests cholestatic jaundice. Alcoholic cirrhosis is common but is unlikely (though not excluded) by the only slightly elevated -glutamyltransferase.

Cholangiocarcinoma is a rare tumour. Carcinoma of the head of the pancreas frequently presents in this manner (though weight loss is often present also), but the slight elevation in total protein with low–normal albumin suggests a high globulin concentration, which suggests autoimmune liver disease.

Prepared by Dr: Mohammed Musa Brema Idress – My best wishes Page 98

Primary sclerosing cholangitis is a possibility but is commoner in men than women (3:1), and in 75% of cases is associated with inflammatory bowel disease. Primary biliary cirrhosis is more common in women.

271. A 21-year-old male medical student who has been feeling non-specifically unwell for several days is noticed to have slightly icteric sclerae by his girlfriend and has liver function tests performed. The results of these are normal apart from a serum bilirubin concentration of 44 mol/l (3–17). His urine does not contain bilirubin. Which of the following is the most likely diagnosis? Single best answer question – choose ONE true option only A) Dubin–Johnson syndrome B) Gilbert’s syndrome C) Hereditary spherocytosis D) Infectious mononucleosis E) Rotor syndrome

ANSWER IS B Dubin–Johnson, Rotor and Gilbert’s syndromes are all inherited disorders of bilirubin metabolism. However, in the first two, there is a defect in the secretion of bilirubin from the liver and the bilirubin that accumulates in the plasma is conjugated, water- soluble and thus is excreted in the urine.

Infectious mononucleosis can cause hepatitis and jaundice but an elevated transaminase activity would be expected. Hereditary spherocytosis is a chronic haemolytic disorder due to a defect in the red cell membrane (most frequently in spectrin, a structural protein). It can present with a wide range of severity, from jaundice at birth to asymptomatic anaemia or jaundice in adults, but is much less common (approximately 1:5000 in Northern Europeans) than Gilbert’s syndrome (approximately 1:20).

272. A 20-year-old man presents with mild jaundice following a flu-like illness. Following review by a gastroenterologist, he has been told that a diagnosis of Gilbert’s syndrome is probable. Which laboratory test is most likely to confirm this diagnosis? Single best answer question – choose ONE true option only A) Absence of bilirubin in the urine B) Decreased serum haptoglobin concentration C) Elevated serum aspartate aminotransferase (transaminase, AST) activity D) Increased reticulocyte count E) Increased urinary urobilinogen excretion

ANSWER IS A In Gilbert’s syndrome, the excess bilirubin is unconjugated, and does not appear in the urine. The same is true for jaundice secondary to haemolysis. However, in haemolytic jaundice, urinary urobilinogen is increased (increased production of bilirubin, and hence of urobilinogen), the reticulocyte count may be elevated and serum haptoglobin concentration decreased. Haemolysis may also cause a slight increase in serum aminotransferase (transaminase) activity.

Prepared by Dr: Mohammed Musa Brema Idress – My best wishes Page 99

273. A 42-year-old man is put on a proton-pump inhibitor to suppress symptoms of oesophagitis. The cell and membrane biology of the gastric acid pump has which of the following features? Single best answer question – choose ONE true option only A) Histamine-stimulated acid production is independent of the proton pump B) The proton is exchanged with magnesium ions C) Acetylcholine-stimulated acid production is independent of the proton pump D) The proton pump spans the apical membrane of the gastric parietal cell E) The proton pump spans the basolateral membrane of the gastric parietal cell

ANSWER IS D The H+–K+-ATPase is embedded in the apical membrane. The channel is susceptible to agents that bind to cysteine residues (particularly 813 and 822). The proton pump is the final common pathway of histamine and acetylcholine-stimulated production, and particularly explains the enhanced efficacy of proton-pump inhibitors in comparison to H2 antagonists or acetylcholine antagonists (e.g. pirenzepine).

274. High titers of antithyroid microsomal and antithyroglobulin antibodies would suggest which of the following diagnoses in a patient presenting with a complaint of tiredness? Single best answer - choose ONE true option only A) Hashimoto’s thyroiditis B) Reidel’s thyroiditis C) Graves disease D) Hypoparathyroidism E) Idiopathic hypothyroidism

ANSWER IS A This finding in Hashimoto’s thyroiditis is characteristic, but lower titers can occur in Reidel’s thyroiditis and Graves disease. High titers of these antibodies in euthyroid individuals indicate the possibility of future thyroid failure, but this may be many years away; hence the need for thyroid function tests every 1–2 years in such individuals.

275. Which of the following cells secretes intrinsic factor? Single best answer question – choose ONE true option only A) Goblet cells B) Kupffer cells C) Peptic cells D) Chief cells E) Parietal cells

ANSWER IS E Goblet cells are mucus-secreting cells, widely distributed throughout epithelial surfaces, but especially dense in the gastrointestinal and respiratory tracts.

Kupffer cells have phagocytic properties and are found in the liver. They participate in the removal of ageing erythrocytes and other particulate debris.

Prepared by Dr: Mohammed Musa Brema Idress – My best wishes Page 100

The gastric mucosa contains many cell subtypes, including acid-secreting cells (also known as parietal or oxyntic cells), pepsin secreting cells (also known as peptic, chief or zymogene cells) and G-cells (gastrin-secreting cells). Peptic cells synthesize and secrete the proteolytic enzyme, pepsin.

Parietal cells actively secrete hydrochloric acid into the gastric lumen, accounting for the acidic environment encountered in the stomach. However parietal cells are also involved in the secretion of the glycoprotein, intrinsic factor.

Intrinsic factor plays a pivotal role in the absorption of vitamin B12 from the terminal ileum. Autoimmune damage to parietal cells leads to a lack of intrinsic factor and hydrochloric acid, leading to vitamin B12 deficiency and achlorhydria. This is known as pernicious anaemia. Pernicious anaemia is associated with a 3-fold increase in gastric cancer risk.

276. Splenectomy increases susceptibility to which of the following organisms? Single best answer question – choose ONE true option only A) Streptococcus pyogenes B) Schistosoma haematobium C) Bacteroides fragilis D) Neisseria meningitidis E) Staphylococcus aureus

ANSWER IS D The spleen plays an important role in the removal of dead and dying erythrocytes and in the defence against microbes. Removal of the spleen (splenectomy) leaves the host susceptible to a wide array of pathogens, but especially to encapsulated organisms.

Certain bacteria have evolved ways of evading the human immune system. One way is through the production of a ‘slimy’ capsule on the outside of the bacterial cell wall. Such a capsule resists phagocytosis and ingestion by macrophages and neutrophils. This allows them not only to escape direct destruction by phagocytes, but also to avoid stimulating T-cell responses through the presentation of bacterial peptides by macrophages. The only way that such organisms can be defeated is by making them more ‘palatable’ by coating their capsular polysaccharide surfaces in opsonising antibody.

The production of antibody against capsular polysaccharide primarily occurs through T-cell independent mechanisms. The spleen plays a central role in both the initiation of the antibody response and the phagocytosis of opsonised encapsulated bacteria from the bloodstream.

This helps to explain why the asplenic individuals are most susceptible to infection from encapsulated organisms, notably Streptococcus pneumoniae (pneumococcus), Neisseria meningitidis (meningococcus) and Haemophilus influenzae.

Prepared by Dr: Mohammed Musa Brema Idress – My best wishes Page 101

The risk of acquiring such infections is reduced by immunizing individuals against such organisms and by placing patients on prophylactic penicillin, in most cases for the rest of their lives. In addition, asplenic individuals should be advised to wear a Medic Alert bracelet to warn other health care professionals of their condition.

277. Which one of the following hormones is secreted by the anterior pituitary? Single best answer question – choose ONE true option only A) Testosterone B) Oxytocin C) TSH D) CRH E) ADH

ANSWER IS C The pituitary gland (hypophysis) is the conductor of the endocrine orchestra. It is divided into both an anterior part and posterior part. The anterior pituitary (adenohypophysis or pars distalis) secretes 6 hormones namely: 1- FSH/LH: Reproduction 2- ACTH: Stress response 3- TSH: Basal metabolic rate 4- GH: Growth 5- Prolactin: Lactation The posterior pituitary (neurohypophysis or pars nervosa) secretes only 2 hormones: 1- ADH (vasopressin): Osmotic regulation 2- Oxytocin: Milk ejection and labour Testosterone is produced from Leydig cells in the testis and from the adrenal glands. CRH is produced by the median eminence of the hypothalamus.

278. Which of the following cells is cytotoxic? Single best answer question – choose ONE true option only A) CD4 T-cells B) CD8 T-cells C) B cells D) TH1 cells E) TH2 cells

ANSWER IS B Lymphocytes can be divided into two main subtypes – T cells and B cells. - B cells (or plasma cells) secrete antibodies. - T cells can be divided into two further subtypes – CD4 T-cells and CD8 T-cells. CD4 (helper) T-cells can recognize antigen only in the context of MHC Class II, whereas CD8 (cytotoxic) T-cells recognize cell-bound antigens only in association with Class I MHC. This is known as MHC restriction.

CD4 and CD8 T-cells perform distinct but somewhat overlapping functions. The CD4 helper T-cell can be viewed as a master regulator. By secreting cytokines (soluble factors that mediate communication between cells), CD4 helper T-cells influence the

Prepared by Dr: Mohammed Musa Brema Idress – My best wishes Page 102 function of virtually all other cells of the immune system including other T-cells, B- cells, macrophages and natural killer cells.

The central role of CD4 cells is tragically illustrated by the HIV virus which cripples the immune system by selective destruction of this T-cell subset. In recent years two functionally different populations of CD4 helper T-cells have been recognized – TH1 cells and TH2 cells, each characterized by the cytokines that they produce. In general, TH1 cells facilitate cell-mediated immunity, whereas TH2 cells promote humoral-mediated immunity.

CD8 cytotoxic T-cells mediate their functions primarily by acting as cytotoxic cells (i.e. they are T-cells that kill other cells). They are important in the host defence against cytosolic pathogens. Two principal mechanisms of cytotoxicity have been discovered – perforin-granzyme-dependent killing and Fas-Fas ligand dependent killing.

279. Normal Cerebrospinal fluid (CSF): Single best answer question – choose ONE true option only A) Is produced within arachnoid granulations B) Has an identical composition to that of plasma C) Has a protein content that is 0.5% that of plasma D) Has a higher potassium content than that of plasma E) pH is heavily buffered

ANSWER IS C Most of the CSF is produced by the choroid plexus, which is situated in the lateral, 3rd and 4th ventricles. CSF is absorbed directly into the cerebral venous sinuses through the arachnoid villi, or granulations, by a process known as mass or bulk flow.

The composition of CSF is different to plasma. Of importance to mention are the concentrations of K+, Ca2+, bicarbonate and protein which are lower in CSF than in plasma. This is to prevent high concentrations of these electrolytes inadvertently exciting neurons present within the brain substance.

The potassium content of the CSF in this respect is particularly important. Further buffering of the K+ content of CSF takes place through astrocytes.

Likewise, the low protein content of the CSF (the CSF protein content is 0.5% that of plasma) is deliberate to prevent some proteins and amino acids acting as “false neurotransmitters”.

The CSF is more acidic than plasma because pH of the CSF plays a critical role in the regulation of pulmonary ventilation and cerebral blood flow. Another reason why the CSF protein is kept deliberately low is to prevent proteins buffering pH. The result is that the pH of the CSF accurately reflects carbon dioxide levels of the blood. In this way changes in pH act as a powerful regulator of the respiratory system (through the action of pH on central chemoreceptors) and on cerebral blood flow.

Prepared by Dr: Mohammed Musa Brema Idress – My best wishes Page 103

280. Gastric acid secretion is stimulated by: Single best answer question – choose ONE true option only A) Somatostatin B) Gastrin C) Secretin D) The glossopharyngeal nerve E) Cholecystokinin

ANSWER IS B Gastric acid is stimulated by 3 factors: 1) Acetylcholine: From parasympathetic neurons of the vagus nerve that innervate parietal cells directly. 2) Gastrin: produced by pyloric G-cells. 3) Histamine: Produced by mast cells. This stimulates the parietal cells directly and also potentiates parietal cell stimulation by gastrin and neuronal stimulation. H2 blockers such as ranitidine are therefore an effective way of reducing acid secretion. Gastric acid is inhibited by 3 factors: 1) Somatostatin 2) Secretin 3) Cholecystokinin There are 3 classic phases of gastric acid secretion: 1) Cephalic (preparatory) phase [significant]: Results in the production of gastric acid before food actually enters the stomach. Triggered by the sight, smell, thought and taste of food acting via the vagus nerve. 2) Gastric phase [most significant]: Initiated by the presence of food in the stomach, particularly protein rich food. 3) Intestinal phase [least significant]: The presence of amino acids and food in the duodenum stimulate acid production.

281. Bile salt reuptake principally occurs in the: Single best answer question – choose ONE true option only A) Duodenum B) Jejenum C) Ileum D) Colon E) Caecum

ANSWER IS C - 90-95% of the bile salts are absorbed from the small intestine and then excreted again from the liver; most are absorbed from the terminal ileum. This is known as the enterohepatic circulation. The entire pool recycles twice per meal and approximately 6-8x per day. Disruption of the enterohepatic circulation, either by terminal ileal resection or through a diseased terminal ileum (e.g. Cohn’s disease), results in decreased fat absorption and cholesterol gallstone formation. The latter is believed to result

Prepared by Dr: Mohammed Musa Brema Idress – My best wishes Page 104 because bile salts normally make cholesterol more water-soluble through the formation of cholesterol micelles.

Loss of reuptake also results in the presence of bile salts in colonic contents, which alters colonic bacterial growth and stool consistency.

282. Which of the following gastrointestinal fluids is richest in potassium? Single best answer question – choose ONE true option only A) Salivary B) Pancreatic C) Gastric D) Bile E) Small bowel

ANSWER IS A In man about 1-1.5 liters of saliva are secreted each day. Secretion is an active process. The two-stage hypothesis of salivation states that a primary secretion is first formed by secretory end-pieces (that resembles an ultrafiltrate of plasma), which is then modified as it flows along the duct system.

Na+ and Cl- are absorbed and K+ and HCO3- are secreted as saliva flows along the ductal system. In addition, the ducts have a low water permeability.

The final saliva is hypotonic with respect to plasma and contains a higher potassium concentration than any other gastrointestinal secretion of the body. Any abnormal state in which saliva is lost to the exterior of the body for long periods can lead to a serious depletion of potassium, leading in occasional circumstances to serious hypokalaemia and paralysis.

283. The action potential of skeletal muscle? Single best answer question – choose ONE true option only A) Has a prolonged plateau phase B) Spread inwards to all parts of the muscle via the T tubes C) Causes immediate uptake of Ca into the sarcoplasmic reticulum D) Is longer than the action potential of cardiac muscle E) Is not essential for contraction

ANSWER IS B The action potential of the skeletal muscle spreads out from the motor end plate, through the T- tube system this causes mobilization of Ca2+ from the sarcoplasmic reticulum to the cytoplasm and this action potential is essential for contraction. The action potential of cardiac muscle is longer than that of the skeletal muscle and has plateau phase.

284. The rate at which a liquid meal leaves the stomach is? Single best answer question – choose ONE true option only A) Greater in the upright than in the supine position

Prepared by Dr: Mohammed Musa Brema Idress – My best wishes Page 105

B) Proportional to the volume of stomach content C) Greater if the meal contains fat D) Slower if the meal is 5% glucose than if it is 50% glucose E) Slower if vagotomy and drainage procedure (such as gastroenterostomy or pyloroplasty) has been performed

ANSWER IS B Gastric emptying accelerates on lying down. The rate of gastric emptying at any moment is proportional to the volume present in the stomach at that moment

When the fat reaches the duodenum it stimulates mixed hormonal and vagal mechanisms that slow the rate of stomach emptying. An isotonic meal will empty at maximal rate but osmotically stronger or weaker solutions will empty more slowly.

Vagotomy may temporarily slow gastric emptying, but its long term effect is to increase the rate of gastric emptying or leave it un changed so if a drainage procedure is accompanied by vagotomy there will be a tendency towards accelerating gastric emptying.

285. Absorption of calcium from the digestive tract? Single best answer question – choose ONE true option only A) Takes place mostly in the proximal jejunum B) Is prevented by the presence of small amounts of phytic acid in the diet , even when an excess calcium is ingested C) Is facilitated by the presence of fat in food D) Can be reversed (calcium is secreted into bowel lumen) when plasma calcium concentration is raised by a calcium infusion E) Is about as rapid as that of sodium

ANSWER IS A - Phytic acid produces insoluble calcium phytate, when all phytic acid has been precipitated the excess calcium is absorbed. - Fatty acids form insoluble calcium salts (soaps). - The shift of calcium ions across the intestinal mucosa is virtually one way. - Sodium is absorbed at a speed fifty times that for calcium absorption.

286. Considering the ABO and rhesus (Rh) systems: Single best answer question – choose ONE true option only A) If the patient blood group is AB, his serum will have anti A and anti B antibodies B) Naturally occurring anti A and anti B antibodies are usually IgG C) Red blood cells are the only carrier of the antigen A,B and H D) The presence of the D antigen makes the subject rhesus positive E) Rhesus antibodies are naturally occurring antibodies

Prepared by Dr: Mohammed Musa Brema Idress – My best wishes Page 106

ANSWER IS D In AB blood group, the patient will have A & B antigen but not anti-A and anti-B antibodies. These antibodies are usually Ig M and are present on most of body cells. Rhesus antibodies do not occur naturally and are formed as immune antibodies upon exposure to antigen.

287. The cerebral blood flow (CBF): Single best answer question – choose ONE true option only A) Accounts for about 15% of the cardiac output B) Is decreased by hypercapnia C) Is decreased by hypoxia D) Is mainly controlled by sympathetic and parasympathetic activity E) Is decreased by isoflurane in general anaesthesia

ANSWER IS A - Cerebral blood flow accounts for about 15% of the cardiac output. Hypoxia and hypercapnia increase the CBF. An autonomic mechanism seems to be unimportant. - Isoflurane increases CBF.

288. Which of the following hormones exerts the least effect on growth? Single best answer question – choose ONE true option only A) Growth hormone B) Testosterone C) T4 D) Insulin E) Vasopressin

ANSWER IS E Growth is stimulated by growth hormone which stimulates IGF-I secretion, androgens and estrogens initially stimulate the growth, they then ultimately terminate the growth by causing the epiphysis to fuse to the long bones.

Thyroid hormones have a permissive effect to the action of growth hormone possibly via somatomedines. Insulin promotes growth as it has an anabolic effect on protein metabolism. Vasopressin has no effect on growth.

289. Nociception (pain): Single best answer question – choose ONE true option only A) Is transmitted faster through C fibers than through A delta fibers B) Pain impulse received in the dorsal horn can be modulated by other descending spinal inputs C) Opioids act on µ receptors in the peripheral nerves D) Side effects of opioids can be reversed by neostigmine E) Glycine is excitatory pain neurotransmitter

Prepared by Dr: Mohammed Musa Brema Idress – My best wishes Page 107

ANSWER IS B Pain impulse received by dorsal horn can be modulated by other ascending and descending spinal inputs (Gate Theory). Pain is transmitted faster in myelinated A delta fibers, opioids act on µ and ? opioid receptors in the central nervous system and their effects can be reversed by naloxone. Glycine is an inhibitory neurotransmitter.

290. Hypothyroidism due to disease of the thyroid gland is associated with increased plasma level of? Single best answer question – choose ONE true option only A) Cholesterol B) Albumin C) RT3 D) Iodide E) Thyroid binding globulin (TBG)

ANSWER IS A Thyroid hormone lowers circulating cholesterol level. The plasma cholesterol level drops before the metabolic rate rises.

291. The symptoms of dumping syndrome in patients with intestinal short circuits such as anastomosis of the jejunum to the stomach are caused by? Single best answer question – choose ONE true option only A) Increased blood pressure B) Increase secretion of glucagons C) Increased secretion of CCK D) Hypoglycemia E) Hyperglycemia

ANSWER IS D There are two types of dumping syndromes resulting from rapid gastric emptying; 1) Early dumping which manifests by dizziness, sweating and palpitations within 5-45 minutes after eating. 2) Late dumping occurs 2-4 hours after eating and is due to rebound hypoglycemia, small meals and glucose help to improve symptoms

292. Pepsin is secreted from which cells of the stomach? Single best answer question – choose ONE true option only A) Parietal cells B) Chief cells C) G cells D) Mucous cells E) IX

ANSWER IS B There are many types of cells located within the gastric glands; 1) Parietal cells secrete ……. HCl and intrinsic factor

Prepared by Dr: Mohammed Musa Brema Idress – My best wishes Page 108

2) Chief cells secrete ……. pepsinogen, the precursor of pepsin 3) G cells secrete …….. the hormone gastrin 4) Mucous cells secrete ……. mucous

293. Vitamin K is involved in carboxylation of glutamic acid residues of the following clotting factors EXCEPT for? Single best answer question – choose ONE correct option only A) XI B) Prothrombin C) VII D) IX E) X

ANSWER IS A Vitamin K is essential for activation of clotting factors II (prothrombin), VII, IX, X.

294. Concerning the salivary glands: Single best answer question – choose ONE true option only A) They secrete around 150 ml of saliva per day B) They secrete saliva with a pH of 4-5 C) They secrete saliva which is hypertonic D) They are supplied by the parasympathetic nervous system E) They secrete saliva containing trypsinogen

ANSWER IS D Saliva is secreted from the acini, and transported via the salivary ducts to the oral cavity. The secretion from the sublingual gland is predominately mucous, the parotid serous and the submandibular mixed.

The pH of saliva varies from 7-8, and around 1.5L is produced per day. As well as a- amylase, saliva contains lipase and glycoproteins to lubricate food and protect the oral mucosa. Lysozyme, IgA and lactoferrin act as bacteriostatic agents, and proteins protect the tooth enamel.

The saliva is isotonic when it is excreted from the acini; Na+ and Cl- are exchanged for K+ and HC03- in the ducts, and the saliva becomes hypotonic by the time it reaches the mouth.

295. During digestion of a fatty meal, which hormone causes contraction of the gall bladder and relaxation of the sphincter of Oddi? Single best answer question – choose ONE true option only A) Cholecystokinin B) Gastrin C) Insulin D) Secretin E) Somatostatin

Prepared by Dr: Mohammed Musa Brema Idress – My best wishes Page 109

ANSWER IS A Cholecystokinin secretion from the duodenal and jejunal mucosa is stimulated by the presence of fatty acids, amino acids and peptides in the lumen of the duodenum and jejunum. As well as causing contraction of the gall bladder and relaxation of the sphincter of Oddi, it stimulates release of pancreatic enzymes, and increases the secretin mediated secretion of HC03- by pancreatic duct cells. Its release is inhibited by somatostatin.

296. Parathyroid adenoma will be most likely to cause: Single best answer question – choose ONE true option only A) Decreased osteoclastic activity B) Decreased urinary phosphate excretion C) Hypocalcaemia D) Increased osteoblastic activity E) Increased osteoclastic activity

ANSWER IS E The parathyroid glands produce parathyroid hormone (PTH) in response to serum calcium levels via a negative feedback mechanism. High levels of serum Ca2+ inhibit PTH secretion, and low levels stimulate PTH secretion. The response to Ca2+ levels is very rapid, so effects are seen very quickly after removal of the glands. PTH affects calcium levels by its action on the bone, kidney and gut.

In bone, increased osteoclastic activity causes calcium levels to rise. This is due firstly to acid secretion onto the bone surface, and secondly to proteases dissolving the matrix.

In the kidney, PTH controls the hydroxylation of 25,hydroxy cholecalciferol D to 1,25 hydroxy cholecalciferol. This has the indirect effect of increasing calcium uptake in the gut. In the proximal tubule, PTH increases the urinary excretion of phosphate, which in turn increases the ionization of calcium. There is also an increase in Ca2+ reabsorption in the distal tubule. Bicarbonate resorption is inhibited in the kidney, causing a hyperchloraemic acidosis which increase calcium ionization and resorption from bone.

PTH excess therefore causes hypercalcaemia, hypophosphataemia and hyperchloraemia, as well as raised urinary phosphate.

297. What is the half life of free triiodothyronine (T3) in the blood? Single best answer question – choose ONE true option only A) 1 minute B) 1 hour C) 1 day D) 1 week E) 1 month

Prepared by Dr: Mohammed Musa Brema Idress – My best wishes Page 110

ANSWER IS C Most of the T3 and thyroxine (T4) are carried in plasma bound to thyroxine binding globulin, and are inactive in this state. Only 1% of T3 and 0.05% of T4 is free. T3is the active hormone, and is formed from the intracellular deiodination of T4 by type 2 deiodinase. The half life of T4 is 1 week, and of T3 1 day, suggesting that T4 acts as a source of T3, rather than an active hormone in its own right.

298. Which of the following metabolic effects is most likely to be caused by thyroid hormone? Single best answer question – choose ONE true option only A) Decreased glycogenolysis in the liver B) Increased glucose absorption in the gut C) Decreased lipolysis D) Decreased expression of ß adrenergic receptors E) Decreased oxygen uptake in the mitochondria

ANSWER IS B Thyroid hormone has widespread metabolic effects. - Increased glycogenolysis in the liver, increased glucose absorption in the gut and increased insulin breakdown all tend to increase blood glucose. The glycogenolytic effects of catecholamines are also potentiated. These effects can make the diagnosis and management of diabetes in thyrotoxicosis difficult.

- There is an overall lipolytic effect, with decreased serum cholesterol seen in thyrotoxicosis, and an increase in hypothyroidism. There is an increased expression of b-adrenergic receptors in many tissues including skeletal and cardiac muscle. There is a positive inotropic effect with increased cardiac output and heart rate.

- A raised metabolic rate and increased heat production are due to increased oxygen uptake and ATP production in the mitochondria. There are also effects on bone, with an overall breakdown of bone, sometimes leading to hypercalcaemia. Increased serum 2,3 DPG leads to a right shift of the haemoglobin dissociation curve. are also essential for fetal development, with deficiency leading to cretinism. The fetus produces its own hormone from 18 weeks of gestation.

299. Which of the following systemic effects are most likely to be caused by a space occupying lesion in the brain? Single best answer question – choose ONE true option only A) Bradycardia B) Hypotension C) Tachycardia D) Tachypnoea E) Venous ulceration

ANSWER IS A The cranium is a fixed volume containing blood, CSF and brain tissue in equilibrium.

Prepared by Dr: Mohammed Musa Brema Idress – My best wishes Page 111

Increases in one component can be compensated by a decrease in the other components without increasing intracranial pressure (the Monroe-Kellie doctrine). Beyond a certain point, this compensation is insufficient, and raised intracranial pressure results (greater than 10-15mmHg).

The effects of raised intracranial pressure are hydrocephalus, cerebral ischaemia (due to decreased cerebral perfusion pressure) and systemic effects. The systemic effects include hypertension, bradycardia, slowed respiration and gastric ulceration (Cushing’s ulcer). These are thought to be due to autonomic dysregulation resulting from hypothalamic compression.

300. Insulin levels are not increased by which of the following? Single best answer question – choose ONE correct option only A) Amino acids B) B-adrenergic stimulation C) Hypokalaemia D) Increased plasma glucose E) Vagal nerve stimulation

ANSWER IS C Glucose is the most important stimulus to insulin, causing its release from secretory granules in the b-cells of the pancreas, and release of newly synthesized insulin. As well as glucose, insulin release is stimulated by amino acids, some fatty acids, and the gut hormones secretin and cholecystokinin. b-adrenergic and vagal stimulation also increase insulin levels.

One of the effects of insulin other than decreasing blood glucose is to cause an intracellular shift of K+. This reduces serum levels of K+, but not total body K+. Hypokalaemia acts to inhibit insulin release.

301. A 34 year old man presents to the surgical clinic 8 months following a laparotomy for a ruptured spleen. He complains of a nodule in the centre of his laparotomy wound. This is explored surgically and a stitch granuloma is found and excised. From which of the following cell types do granulomata arise? A) Polymorpho nucleocytes B) Plasma cells C) Reed- Sternberg cells D) Platelets E) Macrophages

ANSWER IS E Granulomas are organized collections of macrophages

302. A 72 year old man has just undergone an emergency repair for a ruptured abdominal aortic aneurysm. Pre operatively he was taking aspirin, clopidogrel and warfarin. Intra operatively he received 5000 units of unfractionated heparin prior to application of the aortic cross clamp. His blood results on admission to the

Prepared by Dr: Mohammed Musa Brema Idress – My best wishes Page 112 critical care unit are as follows: Full blood count Hb 8 g/dl Platelets 40 * 109/l WBC 7.1 * 109/l • His fibrin degradation products are measured and found to be markedly elevated. Which of the following accounts for these results? A) Anastomotic leak B) Disseminated intravascular coagulation C) Heparin induced thrombocytopenia D) Adverse effect of warfarin E) Adverse effects of antiplatelet agents

ANSWER IS B The combination of low platelet counts and raised FDP in this setting make DIC the most likely diagnosis.

Disseminated intravascular coagulation Diagnosis - Under homeostatic conditions, coagulation and fibrinolysis are coupled. The activation of the coagulation cascade yields thrombin that converts fibrinogen to fibrin; the stable fibrin clot being the final product of hemostasis. The fibrinolytic system breaks down fibrinogen and fibrin.

- Activation of the fibrinolytic system generates plasmin (in the presence of thrombin), which is responsible for the lysis of fibrin clots. The breakdown of fibrinogen and fibrin results in polypeptides (fibrin degradation products). In a state of homeostasis, the presence of plasmin is critical, as it is the central proteolytic enzyme of coagulation and is also necessary for fibrinolysis.

In DIC, the processes of coagulation and fibrinolysis are dysregulated, and the result is widespread clotting with resultant bleeding. Regardless of the triggering event of DIC, once initiated, the pathophysiology of DIC is similar in all conditions. One critical mediator of DIC is the release of a transmembrane glycoprotein (tissue factor =TF).

TF is present on the surface of many cell types (including endothelial cells, macrophages, and monocytes) and is not normally in contact with the general circulation, but is exposed to the circulation after vascular damage. For example, TF is released in response to exposure to cytokines (particularly interleukin 1), tumor necrosis factor, and endotoxin. This plays a major role in the development of DIC in septic conditions. TF is also abundant in tissues of the lungs, brain, and placenta. This helps to explain Why DIC readily develops in patients with extensive trauma. Upon activation, TF binds with coagulation factors that then triggers the extrinsic pathway (via Factor VII) which subsequently triggers the intrinsic pathway (XII to XI to IX) of coagulation.

303. A 53 year old man from Hong Kong presents with symptoms of fatigue, weight loss and recurrent epistaxis. Clinical examination reveals left sided cervical lymphadenopathy and oropharyngeal examination reveals an ulcerated mass in the nasopharynx. Which of the following viral agents is most commonly implicated in the development of this condition?

Prepared by Dr: Mohammed Musa Brema Idress – My best wishes Page 113

A) Cytomegalovirus B) Epstein Barr virus C) Coxsackie virus D) Herpes simplex virus E) None of the above

ANSWER IS B The clinical scenario is most typical for nasopharyngeal carcinoma. An association with previous Epstein Barr Virus is well established. Infection with the other viruses listed is not a recognized risk factor for the development of the condition.

304. An 18 year old male presents with lethargy, night sweats and on examination is found to have left supraclavicular lymphadenopathy. A surgical registrar performs a left supraclavicular lymph node . The pathologist identifies Reed- Sternberg cells on the subsequent histology sections, what is the most likely diagnosis? A) Metastatic gastric cancer B) Hodgkin’s lymphoma C) Non Hodgkin’s lymphoma D) Tuberculosis E) None of the above

ANSWER IS B Reed-Sternberg cells are characteristic histological cell type found in Hodgkin’s disease. Lymphadenopathy - Lymphadenopathy in the neck, axillae, groins and abdomen - Need to note: solitary/multiple, defined/indistinct, hard/rubbery/soft, tender/painless Causes of lymphadenopathy - Mnemonic: Hodgkin’s disease 1) Haematological: Hodgkin’s lymphoma, NHL, Leukaemia 2) Oncological: metastases 3) Dermatopathic lymphadenitis 4) Gaucher's disease 5) Kawasaki disease 6) Infections: TB, glandular fever, Syphilis 7) Niemann Pick disease 8) Serum sickness 9) Drug reaction (phenytoin) 10) Immunological (SLE) 11) Sarcoidosis 12) Endocrinological (Hyperthyroidism) 13) Angioimmunoplastic lymphadenopathy 14) S L E 15) Eosinophilic granulomatosis

Prepared by Dr: Mohammed Musa Brema Idress – My best wishes Page 114

305. Which of the following lesions is least likely to occur in the presence of severe atrophic gastritis? A) Duodenal ulcer B) Gastric cancer C) Gastric polyp D) Iron deficiency anaemia E) Pernicious anaemia

ANSWER IS A Due the absence of acid a duodenal ulcer is unlikely to occur.

306. A 28 year old man develops an acute paronychia and subsequent spreading sepsis. The tissue exudate has higher protein content than normal tissue because? A) Breakdown of tissue cells release protein B) Capillary walls are more permeable C) Increased blood flow transports more protein into the area D) Intracapillary pressure is raised E) Plasma cells release gamma globulin

ANSWER IS B The increased permeability allows the exudation of plasma proteins.

307. An example of a tissue or organ composed of permenant parenchymal cells is: A) Liver B) Bone Marrow C) Small Intestinal Mucosa D) Heart E) Renal Tubules

ANSWER IS D

308. The type of necrosis that occurs in peripancreatic tissue in acute Pancreatitis is: A. Liquefaction B. Fat C. Coagulation D. Gummatous E. Fibrinoid

ANSWER IS B

309. Change of Columnar Epithelium of the Bronchi into Mature Squamous Epithelium is called: A. Metaplasia B. Dysplasia C. Hyperplasia D. Neoplasia

Prepared by Dr: Mohammed Musa Brema Idress – My best wishes Page 115

E. Hypertrophy

ANSWER IS A

310. Metastatic Calcification: A. Is due to hypercalcemia B. Is due to hypocalcemia C. Occurs in Necrotic Tissue D. Occurs at sites of Chronic Inflammation E. Is due to malignancy

ANSWER IS A

311. Which of the following is a feature of Irreversible Cell injury? A. Glycogen stores are depleted B. Cytoplasmic sodium increases C. Nuclei undergo karyorrhexis D. Intracellular pH diminishes E. Blebs form on cell membranes

ANSWER IS C

312. Which of the following processes that occurs in the breast allows a mother to feed the infant: A. Stromal hypertrophy B. Epithelial dysplasia C. Steatocyte atrophy D. Ductul epithelial metaplasia E. Lobular hyperplasia

ANSWER IS E

313. An Amputated lower limb from a diabetic patient showing black discoloration of the skin and soft tissues with areas of yellowish exudates is characterized as: A. Neoplasia B. Gangrenous Necrosis C. Coagulopathy D. Hemosiderosis E. Gas gangrene

ANSWER IS B

314. Focal fat necrosis, with flecks of chalky tan-white material seen in the omentum is most often associated with the following: A. Gangrenous appendicitis B. Chronic salpingitis C. Acute pancreatitis

Prepared by Dr: Mohammed Musa Brema Idress – My best wishes Page 116

D. Hepatitis E. Acute gastritis

ANSWER IS C

315. In a 60 year old male, Gangrene of toes is most likely associated with: A. Diabetes Mellitus B. Heart Failure C. Blunt force trauma D. AIDS E. Type III hypersensitivity reaction

ANSWER IS A

316. The presence of Columnar Epithelium with Goblet cells in the Lower Oesophagus is most consistent with: A. Dysplasia B. Hyperplasia C. Carcinoma D. Ischaemia E. Metaplasia

ANSWER IS E

317. Which of the following processes explain the appearance of Calcium deposition in tuberculous lymph nodes? A. Dystrophic calcification B. Apoptosis C. Hypercalcaemia D. Metastatic calcification E. Neoplastic change

ANSWER IS A

318. Lipofuscin deposition is most likely to result from: A. Nuclear pyknosis B. Myocardial fiber hypertrophy C. Coagulative necrosis D. Autophagocytosis E. Anaerobic glycolysis

ANSWER IS D

319. Which of the following cellular changes is most likely to present irreversible cellular injury? A. Epithelial dysplasia B. Cytoplasmic fatty metamorphosis

Prepared by Dr: Mohammed Musa Brema Idress – My best wishes Page 117

C. Nuclear pyknosis D. Atrophy E. Anaerobic Glycolysis

ANSWER IS C

320. Which of the following is an anti-oxidant? A. Glutathione peroxidase B. Catalase C. Hydrogen peroxide D. NADPH oxidase E. Myeloperoxidase

ANSWER IS A

321. Which of the following is the most likely pathologic alteration following occlusion of the left middle cerebral artery by a sterile thrombus: A. Cerebral softening from liquefactive necrosis B. Pale infarction with coagulative necrosis C. Predominantly the loss of glial cells D. Recovery of damaged neurons if the vascular supply is re-established E. Wet gangrene with secondary bacterial infection

ANSWER IS A

322. The action of putrefactive bacteria on necrotic tissue results in: A. Coagulation B. Infarction C. Gangrene D. Embolism E. Caseation

ANSWER IS C

323. Which of the following type of necrosis is most commonly associated with ischaemic injury? A. Coagulation Necrosis B. Liquefaction Necrosis C. Caseous Necrosis D. Fat Necrosis E. Gangrenous Necrosis

ANSWER IS A

324. Hepatocytes are an example of: A. Permenant cells B. Stable cells

Prepared by Dr: Mohammed Musa Brema Idress – My best wishes Page 118

C. Labile cells D. Metaplasia E. Mesenchymal cells

ANSWER IS B

325. A 23 year old man presents to the surgical clinic with an inguinal hernia. On examination he has a small direct hernia. However, you also notice that he has pigmented spots around his mouth, on his palms and soles. In his history he underwent a reduction of an intussusception aged 12 years. Which of the following lesions is most likely to be identified if a were performed? A. Hamartomas B. Tubulovillous adenoma C. Colorectal cancer D. Crohns disease E. Hyperplastic polyps

ANSWER IS A He is most likely to have Peutz-Jeghers syndrome which is associated with Hamartomas. Peutz-Jeghers syndrome - Peutz-Jeghers syndrome is an autosomal dominant condition characterized by numerous benign hamartomatous polyps in the gastrointestinal tract. It is also associated with pigmented freckles on the lips, face, palms and soles. Around 50% of patients will have died from a gastrointestinal tract cancer by the age of 60 years. Genetics - Autosomal dominant - Responsible gene encodes serine threonine kinase LKB1 or STK11 Features - Hamartomatous polyps in GI tract (mainly small bowel) - Pigmented lesions on lips, oral mucosa, face, palms and soles - Intestinal obstruction e.g. intussusception (which may lead to diagnosis) - Gastrointestinal bleeding Management - Conservative unless complications develop

326. A 56 year old surgeon has been successfully operating for many years. Over the past few weeks she has begun to notice that her hands are becoming blistering and weepy. A latex allergy is diagnosed. Which of the following pathological processes accounts for this scenario? A. Type 1 hypersensitivity reaction B. Type 2 hypersensitivity reaction C. Type 4 hypersensitivity reaction D. Type 3 hypersensitivity reaction E. None of the above

Prepared by Dr: Mohammed Musa Brema Idress – My best wishes Page 119

ANSWER IS C - Hypersensitivity reactions: ACID 1) Type 1 --Anaphylactic 2) Type 2 --Cytotoxic 3) Type 3 --Immune complex 4) Type 4 --Delayed hypersensitivity

327. Calcium may play a role in cell injury by: A. Causing ATP depletion B. Activating phospholipases C. Inducing autophagocytosis D. Regulating pyknosis E. Reducing intracellular pH

ANSWER IS B

328. An area of keratinizing squamous epithelium lining a major bronchus is an example of: A. Heterotopia B. Metaplasia C. Dysplasia D. Atrophy E. Neoplasia

ANSWER IS B

329. Each of the following is an example of hyperplasia except: A. Enlargement of one kidney after surgical removal of the other kidney B. Changes in the thyroid gland in response to increased demand for thyroid hormones C. Changes in breast tissue during pregnancy D. Enlargement of lymph nodes during a viral infection E. Enlargement of skeletal muscles after weight training

ANSWER IS E

330. A 43 year old man presents with haemoptysis and is diagnosed as having tuberculosis. Which of the cell types listed below will usually internalize the tubercule bacullis? A. Fibroblast B. Neutrophil C. Erythrocyte D. Macrophage E. Eosinophil

ANSWER IS D M. Tuberculosis will reside in macrophages where it will often survive

Prepared by Dr: Mohammed Musa Brema Idress – My best wishes Page 120

331. Which of these tumour markers is most helpful in identifying an individual with hepatocellular carcinoma? A. Serum AFP B. Serum CA19-9 C. CEA D. Beta HCG E. CA125

ANSWER IS A Hepatocellular carcinoma is commonly diagnosed with imaging and an elevated alpha fetoprotein. Biopsy may seed the tumour and should be avoided. Up to 80% of hepatocellular carcinoma arise in cirrhotic livers

332. Which of the following can undergo apoptosis? A. Cells infected with virus B. Cells with DNA damage C. Increased oxidant within the cell D. None of the above E. All of the above

ANSWER IS E

333. A 64 year old man presents to the clinic with right upper quadrant discomfort. He has never attended the hospital previously and is usually well. He has just retired from full time employment as a machinist in a PVC factory. CT scanning shows a large irregular tumour in the right lobe of his liver. Which of the following lesions is the most likely? A. Liposarcoma B. Angiosarcoma C. Hamartoma D. Hyatid liver disease E. Benign angioma

ANSWER IS B Angiosarcoma of the liver is a rare tumour. However, it is linked to working with vinyl chloride, as in this case. Although modern factories minimize the exposure to this agent, this has not always been the case

334. Digestion of tissue with soap formation and calcification is characteristic of: A. Coagulation Necrosis B. Caseous Necrosis C. Fibrinoid Necrosis D. Liquefaction Necrosis E. Enzymatic Fat Necrosis

ANSWER IS E

Prepared by Dr: Mohammed Musa Brema Idress – My best wishes Page 121

335. Enzymes involved in the inactivation of free radicals include: A. Superoxide dismutase B. Vitamin E C. Both A an B D. Neither A nor B E. Alph-antitrypsin

ANSWER IS A

336. The best example of Dystrophic Calcification is seen in a: A. 55 year old woman with metastases from breast carcinoma & hypercalcemia B. Healing granuloma in a 41 year old man with pulmonary tuberculosis C. Gangrenous lower extremity in a 50 year old woman with diabetes mellitus D. 62 year old woman with a recent cerebral infarction E. Abscess of the left fallopian tube in a 19 year old woman with Neisseria Gonnorrheae Infection

ANSWER IS B

337. The presence of differentiated columnar epithelium with goblet cells in lower esophagus is consistent with: A. Dysplasia B. Hyperplasia C. Carcinoma D. Ischemia E. Metaplasia

ANSWER IS E

338. Which of the following will show hypertrophy? A. The uterine myometrium in pregnancy B. The female breast at puberty C. The liver following partial resection D. The ovary following menopause E. The cervix with chronic inflammation

ANSWER IS A

339. A 32 year old man is involved in a house fire and sustains extensive partial thickness burns to his torso and thigh. Two weeks post operatively he develops oedema of both lower legs. The most likely cause of this is: A. Iliofemoral deep vein thrombosis B. Venous obstruction due to scarring C. Hypoalbuminaemia D. Excessive administration of intravenous fluids E. None of the above

Prepared by Dr: Mohammed Musa Brema Idress – My best wishes Page 122

ANSWER IS C Loss of plasma proteins is the most common cause of oedema developing in this time frame.

340. Which of the following is not characteristic of a granuloma? A. Altered macrophages B. Fused macrophages C. Epithelioid cells D. Mixture of chronic inflammatory cells E. Polymorphnuclear leucocytes, cellular debris and fibrin

ANSWER IS E These are typical components of an abscess cavity. Polymorphonuclear leucocytes may be found in a granuloma if there is a focus of suppuration

341. Which of the following is most susceptible to liquefaction necrosis following ischemic injury? A. Pancreas B. Liver C. Spleen D. Brain E. Intestine

ANSWER IS D

342. Hypertrophy is most closely associated with: A. Hypercalcemia B. Necrosis C. Chronic Irritation D. Diminished Blood Supply E. Increased Work Load

ANSWER IS E

343. The light brown perinucleur pigment seen in an old man muscle fiber is: A. Hemosidrin B. Lipofuscin C. Glycogen D. Melanin E. Calcium

ANSWER IS B

344. Gangrene of the big toe in a 60 year old female is most likely associated with: A. Diabetes Mellitus B. Temporal Arteritis C. AIDS

Prepared by Dr: Mohammed Musa Brema Idress – My best wishes Page 123

D. Type III Hypersensitivity E. Carcinoma of the Bronchus

ANSWER IS A

345. Two days after myocardial infarction, histology of the heart will show: A. Fibroblasts and Collagen B. Granulation Tissue C. Necrotic Muscle & Neutrophils D. Granulamatous Inflammation E. Aneurysmal Dilation

ANSWER IS C

346. Cigarette Smoking does not cause: A. Squamous Metaplasia B. Smooth Muscle Hyperplasia C. Defective Ciliary Action D. Damage of Airway E. Inhibition of Alveolar Leukocytes & Macrophages

ANSWER IS B

347. Abnormal pigmentation is seen in the following except: A. Conn's Syndrome B. Pernicious Anemia C. Tattooing D. Addison's Disease E. Pregnancy

ANSWER IS B

348. Which of the following is most likely associated with Caseation Necrosis: A. Diabetic Gangrene B. Gas Gangrene C. Myocardial Infarction D. Inpissated Pus E. Splenic Infarction

ANSWER IS D

349. Which of the following pathologic changes would be considered irreversible? A. Fatty change in liver cells B. Hydrophic vacuolization or renal tubular epithelial cells C. Karyoloysis in myocardial cells D. Glycogen deposition in hepatocyte nuclei E. Hyperplasia of the breast during pregnancy

Prepared by Dr: Mohammed Musa Brema Idress – My best wishes Page 124

ANSWER IS C

350. A common manifestation of sublethal cell injury seen in organs such as the heart and liver is: A. Glycogen accumulation B. Fatty change C. Calcium deposition D. Apoptosis E. Deposition of Amyloid

ANSWER IS C

351. A 53 year old lady has undergone a bilateral breast augmentation procedure many years previously. The implants are tense and uncomfortable and are removed. During their removal the surgeon encounters a dense membrane surrounding the implants; it has a coarse granular appearance. The tissue is sent for histology and it demonstrates with the presence of calcification. The underlying process responsible for these changes is: A. Hyperplasia B. Dysplasia C. Metastatic calcification D. Dystrophic calcification E. Necrosis

ANSWER IS D Breast implants often become surrounded by a pseudocapsule and this may secondarily then be subjected to a process of dystrophic calcification. - Pathological calcification - Dystrophic calcification: deposition of calcium deposits in tissues that have undergone, degeneration, damage or disease in the presence of normal serum calcium levels Metastatic calcification Deposition of calcium deposits in tissues that are otherwise normal in the presence of increased serum calcium levels

352. Abnormal pigmentation is seen in the following except: A. Conn's Syndrome B. Pernicious Anemia C. Tattooing D. Addison's Disease E. Pregnancy

ANSWER IS A????

353. From which of the following cell types do giant cells most commonly originate? A. Neutrophils B. Myofibroblasts C. Fibroblasts

Prepared by Dr: Mohammed Musa Brema Idress – My best wishes Page 125

D. Macrophages E. Goblet cells

ANSWER IS D Although many cell types may give rise to giant cells, macrophages remain the most common.

Giant cells - A giant cell is a mass formed by the union of several distinct types of cells - They are most commonly comprised of macrophages - They are different to granulomas although causative agents may overlap

354. A 54-year-old woman who has been diagnosed with early-stage breast cancer undergoes surgery for a lumpectomy to remove a small tumor detected by mammography. The pathology report confirms the early stage of the cancer and further comments on the fact that there is significant desmoplasia in the surrounding tissue. The term desmoplasia refers to: (A) An irregular accumulation of blood vessels. (B) Maturation and spatial arrangement of cells. (C) Metastatic involvement of surrounding tissue. (D) Normal tissue misplaced within another organ. (E) Proliferation of non-neoplastic fibrous connective tissue.

ANSWER IS E Desmoplasia refers to proliferation of non-neoplastic fibrous connective tissue within a tumor and is quite common in cases of breast cancer. An irregular accumulation of blood vessels is known as a hemangioma. An area of tissue misplaced within another organ is known as a choristoma.

355. A 24-year-old woman who had previously been uneventfully transfused receives a blood transfusion during surgery and shortly thereafter develops itching, generalized urticaria, laryngeal edema, and dyspnea with wheezing respiration. She has a past history of recurrent upper respiratory tract infections and frequent episodes of . Laboratory studies are most likely to reveal decreased concentrations of which of the following immunoglobulins? A. IgA B. IgD C. IgE D. IgG E. IgM

ANSWER IS A Isolated IgA deficiency is most often asymptomatic but can be characterized by anaphylactic reactions to transfused blood. It is critical to notify the transfusion service about a patient with possible IgA deficiency prior to giving blood products so that products can be washed to remove any immunoglobulins prior to administration.

Prepared by Dr: Mohammed Musa Brema Idress – My best wishes Page 126

This disorder can also be associated with frequent episodes of diarrhea and recurrent infections, especially those involving mucosal surfaces. This inherited B-cell defect is due to inability of IgA B cells to mature into IgA-producing plasma cells. Interestingly, the defect leading to systemic anaphylaxis involves both IgA and IgE antibody formation.

Patients lacking IgA can develop IgE antibodies against the IgA antibodies present in transfused blood. This sensitization can result in susceptibility to anaphylaxis on subsequent transfusion.

356. An HIV-positive intravenous drug user is suspected of having active tuberculosis, and a tuberculin (Mantoux) intradermal skin test is performed. After 48 hours, 10 cm of induration is observed. Which of the following are involved in this form of hypersensitivity reaction? A. B cells and antibodies B. Basophils and IgE C. Immune complexes and complement D. Plasma cells and IgM E. T cells and macrophages

ANSWER IS E The tuberculin test is a classic example of delayed hypersensitivity, a form of cell- mediated hypersensitivity involving CD4+T cells and macrophages. Native CD4+T cells are converted to TH1 cells that secrete cytokines, especially interferon-γ, which is a central mediator of delayed hypersensitivity. Among the many actions of interferon-γ, the most important is the activation of macrophages.

357. A 20 year old African lady undergoes an open appendicectomy. She is reviewed for an unrelated problem 8 months later. On abdominal inspection the wound site is covered by shiny dark protuberant scar tissue that projects beyond the limits of the skin incision. Which of the following is the most likely underlying process? A. Hypertrophic scar B. Keloid scar C. Marjolins ulcer D. Repeated episodes of wound sepsis E. Mycosis fungoides

ANSWER IS B Keloid scars extend beyond the limits of the incision. Mycosis fungoides is a cutaneous T cell lymphoma.

Wound healing Surgical wounds are either incisional or excisional and either clean, clean contaminated or dirty. Although the stages of wound healing are broadly similar their contributions will vary according to the wound type.

Prepared by Dr: Mohammed Musa Brema Idress – My best wishes Page 127

The main stages of wound healing include: Haemostasis • Vasospasm in adjacent vessels, platelet plug formation and generation of fibrin rich clot. A) Inflammation - Neutrophils migrate into wound (function impaired in diabetes). - Growth factors released, including basic fibroblast growth factor and vascular endothelial growth factor. - Fibroblasts replicate within the adjacent matrix and migrate into wound. - Macrophages and fibroblasts couple matrix regeneration and clot substitution. B) Regeneration - Platelet derived growth factor and transformation growth factors stimulate fibroblasts and epithelial cells. - Fibroblasts produce a collagen network. - Angiogenesis occurs and wound resembles granulation tissue. C) Remodeling - Longest phase of the healing process and may last up to one year (or longer). - During this phase fibroblasts become differentiated (myofibroblasts) and these facilitate wound contraction. - Collagen fibers are remodeled. - Microvessels regress leaving a pale scar.

The above description represents an idealized scenario. A number of diseases may distort this process. It is obvious that one of the key events is the establishing well vascularized tissue. At a local level angiogenesis occurs, but if arterial inflow and venous return are compromised then healing may be impaired, or simply nor occur at all. The results of vascular compromise are all too evidence in those with peripheral vascular disease or those poorly constructed bowel anastomoses.

Conditions such as jaundice will impair fibroblast synthetic function and overall immunity with a detrimental effect in most parts of healing.

Problems with scars: Hypertrophic scars Excessive amounts of collagen within a scar. Nodules may be present histologically containing randomly arranged fibrils within and parallel fibers on the surface. The tissue itself is confined to the extent of the wound itself and is usually the result of a full thickness dermal injury. They may go on to develop contractures.

Keloid scars Excessive amounts of collagen within a scar. Typically a keloid scar will pass beyond the boundaries of the original injury. They do not contain nodules and may occur following even trivial injury. They do not regress over time and may recur following removal.

358. The pathogenicity of the tubercle bacillus is due to which of the following? A. Necrosis caused by expanding granulomas B. Ability to multiply within fibroblasts

Prepared by Dr: Mohammed Musa Brema Idress – My best wishes Page 128

C. Delayed hypersensitivity reaction against bacteria D. Effect of antibody response E. Direct toxic effect on host cells

ANSWER IS C Mycobacteria stimulate a specific T cell response of cell mediated immunity. This is effective in reducing the infection; the delayed hypersensitivity also damages tissues. Necrosis occurs in TB but is usually within the granuloma.

359. A 15 year old boy undergoes an emergency splenectomy for trauma. He makes a full recovery and is discharged home. Eight weeks post operatively the general practitioner performs a full blood count with a blood film. Which of the following is most likely to be present? A. Myofibroblasts B. Howell-Jolly bodies C. Multinucleate giant cells D. Reed Sternberg Cells E. None of the above

ANSWER IS B - Post splenectomy blood film features: Howell- Jolly bodies Pappenheimer bodies. Target cells Irregular contracted erythrocytes As the filtration function is the spleen is no longer present Howell-Jolly bodies are found. Post splenectomy blood film changes: The loss of splenic tissue results in the inability to readily remove immature or abnormal red blood cells from the circulation. The red cell count does not alter significantly. However, cytoplasmic inclusions may be seen e.g. Howell-Jolly bodies. In the first few days after splenectomy target cells, siderocytes and reticulocytes will appear in the circulation. Immediately following splenectomy a granulocytosis (mainly composed of neutrophils) is seen, this is replaced by a lymphocytosis and monocytosis over the following weeks. The platelet count is usually increased and this may be persistent, oral antiplatelet agents may be needed in some patients.

360. Phagocytosis is enhanced by: A. C5a B. Bradykinin C. Lysozyme D. C3b E. Serotonin

ANSWER IS D

361. A 43 year old women is identified as being a carrier of a BRCA 1 mutation. Apart from breast cancer, which of the following malignancies is she at greatest risk of developing? A. Colonic cancer

Prepared by Dr: Mohammed Musa Brema Idress – My best wishes Page 129

B. Ovarian cancer C. Follicular carcinoma of the thyroid D. Pituitary adenoma E. Phaeochromocytoma

ANSWER IS B - BRCA 1 mutation patients are 55% more likely to get ovarian cancer. Those with BRCA 2 are 25% more likely. The risk of developing other malignancies is slightly increased but not to the same extent, and not enough to justify screening. - C3b and c5a are complement factors. Both enhance phagocytosis - C4a,2a,3b opsonization of bacteria and facilitate phagocytosis by macrophages - Chemotactic for neutrophils, increase vascular permeability, release of histamine from mast cells

362. A 53 year old man is due to undergo a splenectomy as a treatment for refractory haemolytic anaemia. The underlying pathological basis for haemolytic anaemia is thought to be a Type 2 hypersensitivity response. Which of the following mechanisms best describes this process? A. Deposition of immune complexes B. Cell mediated immune response C. IgE mediated response D. Formation of autoantibodies against cell surface antigens E. None of the above

ANSWER IS D Mnemonic for the reactions and the mediators involved = ACID EGG-T Type 1 → Anaphylactic Type 2 → Cytotoxic Type 3 → Immune complex Type 4 → Delayed type

EGG T (mediators) = IgE IgG IgG T cells - Type 2 hypersensitivity reactions (which includes haemolytic anaemia) are associated with formation of antibody against cell surface antigens. - Hypersensitivity reactions - The Gell and Coombs classification divides hypersensitivity reactions into 4 types

363. 73 year old man presents with haemoptysis and is suspected of suffering from lung cancer. On examination he has an enlarged supraclavicular lymph node. Which of the following features is most likely to be present on histological examination? A. Increased mitoses B. Apoptosis C. Barr Bodies D. Multinucleate giant cells E. Granuloma

Prepared by Dr: Mohammed Musa Brema Idress – My best wishes Page 130

ANSWER IS A Increased mitoses are commonly seen in association with malignant transformation of cells. Apoptosis is not a common feature of metastatic cancer. Barr Bodies are formed during X chromosome inactivation in female somatic cells.

Histopathology of malignancy 1) Abnormal tissue architecture 2) Coarse chromatin 3) Invasion of basement membrane* 4) Abnormal mitoses 5) Angiogenesis 6) De-differentiation 7) Areas of necrosis 8) Nuclear pleomorphism *= Those features that distinguish invasive malignancy from in situ disease

364. Which of the following pathological explanations best describes the initial pathological processes occurring in an abdominal aortic aneurysm in an otherwise well 65 year old, hypertensive male? A) Loss of elastic fibers from the adventitia B) Loss of collagen from the adventitia C) Loss of collagen from the media D) Loss of elastic fibers from the media E) Decreased matrix metalloproteinases in the adventitia

ANSWER IS D In established aneurysmal disease there is dilation of all layers of the arterial wall and loss of both elastin and collagen. The primary event is loss of elastic fibers with subsequent degradation of collagen fibers.

Pathology of abdominal aortic aneurysm:- Abdominal aortic occur primarily as a result of the failure of elastic proteins within the extracellular matrix. Aneurysms typically represent dilation of all layers of the arterial wall. Most aneurysms are caused by degenerative disease.

After the age of 50 years the normal diameter of the infrarenal aorta is 1.5cm in females and 1.7cm in males. Diameters of 3cm and greater, are considered aneurysmal.

The pathophysiology involved in the development of aneurysms is complex and the primary event is loss of the intima with loss of elastic fibers from the media. This process is associated with, and potentiated by, increased proteolytic activity and lymphocytic infiltration.

Major risk factors for the development of aneurysms include smoking and hypertension. Rare but important causes include syphilis and connective tissues diseases such as Ehlers Danlos type 1 and Marfans syndrome.

Prepared by Dr: Mohammed Musa Brema Idress – My best wishes Page 131

365. A 20-year-old woman presents with malar rash, arthralgias, low-grade fever, and high titer antibodies to double-stranded DNA and to the Sm (Smith) antigen. Which of the following forms of hypersensitivity is the primary mechanism of the abnormalities found in this disorder? A) Type I (immediate or anaphylactic) hypersensitivity B) Type II (antibody-mediated or cytotoxic) hypersensitivity C) Type III (immune complex-mediated disorders) hypersensitivity D) Type IV (cell-mediated) hypersensitivity

ANSWER IS C The diagnosis is systemic lupus erythematosus, and the most characteristic lesions are mediated by immune complex deposition (type III hypersensitivity). In this form of hypersensitivity, antibody combines with antigen, resulting in antigen-antibody complexes. Insoluble aggregates of immune complex are deposited in vessel walls, serosal surfaces, and other extravascular sites, and complement is bound.

The antigen-antibody-complement complexes are highly chemotactic for neutrophils, which release lysosomal enzymes and other mediators of tissue damage (prostaglandins, kinins, and free radicals)

366. Within minutes of a bee sting, a 23-year-old woman develops generalized pruritus and hyperemia of the skin, followed shortly by swelling of the face and eyelids, dyspnea, and laryngeal edema. This reaction is mediated by A) Antigen-antibody complexes. B) Cytotoxic T cells. C) IgA antibodies. D) IgE antibodies. E) IgG antibodies.

ANSWER IS D The clinical description is characteristic of systemic anaphylaxis, an IgE-mediated type I hypersensitivity reaction. In type I hypersensitivity, reaction of antigen with preformed IgE antibodies fixed by Fc receptors to the surface of basophils or tissue mast cells results in cytolysis and degranulation of these cells, with release of histamine and other mediators.

367. A 28 year old lady has a malignant melanoma removed from her calf. Which of the following pathological criteria carries the greatest prognostic weighting? A) Vascular invasion B) Abnormal mitoses C) Breslow thickness D) Perineural invasion E) Lymphocytic infiltrates

ANSWER IS C The Breslow thickness has considerable prognostic importance.

Prepared by Dr: Mohammed Musa Brema Idress – My best wishes Page 132

Lymphocytic infiltrates may be associated with an improved prognosis, but do not carry nearly the same weight as increased thickness.

368. A 34 year old lady undergoes an elective cholecystectomy for attacks of recurrent cholecystitis due to gallstones. Microscopic assessment of the gallbladder is most likely to show which of the following? A) Dysplasia of the fundus B) Widespread necrosis C) Ashoff-Rokitansky sinuses D) Metaplasia of the fundus E) None of the above

ANSWER IS C Aschoff-Rokitansky sinuses are the result of hyperplasia and herniation of epithelial cells through the fibromuscular layer of the gallbladder wall. They may be macroscopic or microscopic.

Ashoff Rokitansky sinuses may be identified in cases of chronic cholecystitis and gallstones. Although gallstones may predispose to the development of gallbladder cancer the actual incidence of dysplasia and metaplastic change is rare. In the elective setting described above necrosis would be rare.

369. An 18 year old rock climber falls onto his left arm and sustains a large haematoma of the left upper arm. Unfortunately the wound associated with the injury is neglected and it becomes infected. Which of these changes is least likely to occur? A) Axillary lymphadenopathy B) Leucopenia C) Tenderness D) Mild pyrexia E) Local formation of yellow pus

ANSWER IS B Leucopenia would be unusual and should prompt a search for another cause. Acute inflammation Inflammation is the reaction of the tissue elements to injury. Vascular changes occur, resulting in the generation of a protein rich exudate. So long as the injury does not totally destroy the existing tissue architecture, the episode may resolve with restoration of original tissue architecture.

Vascular changes - Vasodilation occurs and persists throughout the inflammatory phase. - Inflammatory cells exit the circulation at the site of injury. - The equilibrium that balances Starlings forces within capillary beds is disrupted and a protein rich exudate will form as the vessel walls also become more permeable to proteins.

Prepared by Dr: Mohammed Musa Brema Idress – My best wishes Page 133

- The high fibrinogen content of the fluid may form a fibrin clot. This has several important immunomodulatory functions.

Sequelae 1) Resolution - Typically occurs with minimal initial injury - Stimulus removed and normal tissue architecture results 2) Organization - Delayed removed of exudate - Tissues undergo organization and usually fibrosis 3) Suppuration - Typically formation of an abscess or an empyema - Sequestration of large quantities of dead neutrophils 4) Progression to chronic inflammation - Coupled inflammatory and reparative activities - Usually occurs when initial infection or suppuration has been inadequately managed Causes 1- Microbacterial infections e.g. Viruses, exotoxins or endotoxins released by bacteria 2- Chemical agents 3- Physical agents e.g. Trauma 4- Hypersensitivity reactions 5- Tissue necrosis

370. A keen surgical trainee is about to embark on her first hemi arthroplasty for a fractured neck of femur. In the anaesthetic room the patient is given 1.2g intravenous co-amoxiclav. There is a possible history of penicillin allergy but the patient is demented and the history is not checked. The patient then develops severe respiratory compromise and haemodynamic collapse. Which pathological process accounts for this event? A. Binding of the drug to circulating IgG class antibodies B. Recognition of the drug by IgE receptors on mast cells C. Drug initiated formation of hapten-protein complexes D. Binding of the drug to circulating IgM class antibodies E. None of the above

ANSWER IS B Anaphylactic shock: Antigen recognized by IgE molecules on the surface of mast cells resulting in rapid degranulation with release of histamine and other inflammatory cytokines. This is a case of anaphylactic shock. In anaphylaxis the mast cells degranulate.

Anaphylactic shock • Suspect if there has been exposure to an allergen Management - Remove allergen - ABCD - Drugs:

Prepared by Dr: Mohammed Musa Brema Idress – My best wishes Page 134

Adrenaline 1:1000, 0.5ml intramuscular (not IV). Repeat after 5mins if no response. Then Chlorpheniramine 10mg IV Then Hydrocortisone 100-200mg IV

371. Which of the following is not associated with thrombosis? A. Endothelial cell damage B. Use of tourniquets in surgery C. Formation of platelet aggregates D. Thrombocytopenia E. Carcinoma of the stomach

ANSWER IS D All the other options either act directly to promote thrombosis e.g. endothelial cell damage or via changes in consistency or flow of blood.

372. Which of the following cellular types or features is not seen in sarcoidosis? A. Reed Sternberg Cells B. T lymphocytes C. Macrophages D. Asteroid bodies E. B lymphocytes

ANSWER IS A - Reed Sternberg cells are seen in Hodgkin’s disease. All of the other cell types are seen in sarcoid. - That Carcinoma of Stomach and Pancreas accompanies migratory superficial thrombophlebitis

373. A 20 year old man develops acute appendicitis, his appendix is removed and he makes a full recovery. Which of the following pathological processes is least likely to be present in the acutely inflamed tissues? A. Altered Starlings forces. B. Sequestration of neurophils C. Formation of fluid exudate D. Formation of granulomas E. None of the above

ANSWER IS D Neutrophil polymorphs=Acute inflammation. Granuloma = Chronic inflammation. Acute inflammation: - 3 phases: 1) Changes in blood vessel and flow: flush, flare, wheal 2) Fluid exudates (rich in protein i.e. Ig, coagulation factors) produced via increased vascular permeability 3) Cellular exudates mainly containing neutrophil polymorphs pass into extravascular space.

Prepared by Dr: Mohammed Musa Brema Idress – My best wishes Page 135

Neutrophils are then transported to tissues via: A) Margination of neutrophils to the peripheral plasmatic of the vessel rather than the central axial stream B) Pavementing: Adhesion of neutrophils to endothelial cells in venules at site of acute inflammation C) Emigration: neutrophils pass between endothelial cells into the tissue Acute inflammation - Inflammation is the reaction of the tissue elements to injury. Vascular changes occur, resulting in the generation of a protein rich exudate. So long as the injury does not totally destroy the existing tissue architecture, the episode may resolve with restoration of original tissue architecture. Vascular changes 1- Vasodilation occurs and persists throughout the inflammatory phase. 2- Inflammatory cells exit the circulation at the site of injury. 3- The equilibrium that balances Starlings forces within capillary beds is disrupted and a protein rich exudate will form as the vessel walls also become more permeable to proteins. 4- The high fibrinogen content of the fluid may form a fibrin clot. This has several important immunomodulatory functions. Sequelae Resolution - Typically occurs with minimal initial injury - Stimulus removed and normal tissue architecture results Organization - Delayed removed of exudate - Tissues undergo organization and usually fibrosis Suppuration - Typically formation of an abscess or an empyema - Sequestration of large quantities of dead neutrophils Progression to chronic inflammation - Coupled inflammatory and reparative activities - Usually occurs when initial infection or suppuration has been inadequately manage Causes 1- Microbacterial infections e.g. Viruses, exotoxins or endotoxins released by bacteria 2- Chemical agents 3- Physical agents e.g. Trauma 4- Hypersensitivity reactions 5- Tissue necrosis

374. Causes of primary chronic inflammation do not include which of the following? A. Sarcoidosis B. Tuberculosis C. Ulcerative colitis D. Prostheses

Prepared by Dr: Mohammed Musa Brema Idress – My best wishes Page 136

E. Chronic cholecystitis

ANSWER IS E Chronic cholecystitis is caused by recurrent episodes of acute inflammation. Prosthetic implants may be the site of primary chronic inflammation. A common example clinically is breast implants which may become encapsulated. The subsequent fibrosis then results in distortion and may be painful

375. Causes of granulomatous disease do not include: A. Amiodarone B. Allopurinol C. Sulphonamides D. Beryllium E. Wegener's granulomatosis

ANSWER IS A Allopurinol and sulphonamides cause hepatic granulomas.

376. A 59 year old lady is referred from the NHS breast screening program. A recent mammogram is reported as showing linear, branching microcalcification with coarse granules. Which disease process is the most likely underlying cause of these appearances? A. Invasive lobular cancer B. Lobular carcinoma in situ C. Cribriform type ductal carcinoma in situ D. Comedo type ductal carcinoma in situ E. Fibroadenosis

ANSWER IS D Comedo type DCIS is usually associated with microcalcifications. Cribriform lesions are usually multifocal but less likely to form microcalcifications. Lobular cancers and in situ lesions rarely form microcalcifications and are difficult to detect using mammography.

Breast cancer - In situ disease Breast cancer that has yet to invade the basement membrane is referred to as in situ disease. Both ductal and lobular in situ variants are recognized.

Ductal carcinoma in situ Sub types include; comedo, cribriform, micropapillary and solid A) Comdeo DCIS is most likely to form microcalcifications B) Cribriform and micropapillary are most likely to be multifocal - Most lesions are mixed (composed of multiple subtypes) - High nuclear grade DCIS is associated with more malignant characteristics (loss of p53, increase erbB2 expression) - Local excision of low nuclear grade DCIS will usually produce satisfactory outcomes.

Prepared by Dr: Mohammed Musa Brema Idress – My best wishes Page 137

- Multifocal lesions, large and high nuclear grade lesions will usually require mastectomy

Lobular carcinoma in situ - Much rarer than DCIS - Does not form microcalcifications - Usually single growth pattern - When an invasive component is found it is less likely to be associated with axillary nodal metastasis than with DCIS - Low grade LCIS is usually treated by monitoring rather than excision.

377. A 29-year-old man hospitalized for acquired immunodeficiency syndrome (AIDS) is found to have pulmonary tuberculosis. Which type of necrosis is found in the granulomatous lesions (clusters of modified macrophages) characteristic of this increasingly frequent complication of AIDS? A. Caseous B. Coagulative C. Enzymatic D. Fibrinoid E. Liquefactive

ANSWER IS A

378. A 16-year-old girl undergoes radiologic imaging of her abdomen and is found to have only one kidney. She had been entirely unaware of this problem. Which of the following terms is most descriptive of this finding? A. Agenesis B. Atrophy C. Hyperplasia D. Hypoplasia E. Metaplasia

ANSWER IS A The patient has renal agenesis, absence of the kidney due to failure of organ development. The congenital lack of one kidney differs from atrophy, in which a decrease in the size of an organ results from a decrease in the mass of preexisting cells. Unilateral renal agenesis is usually a harmless malformation, and the opposite kidney is often enlarged due to compensatory hypertrophy. Bilateral renal agenesis is incompatible with life and is of special interest since it can lead to the Potter progression

379. Which of the following immunoglobin (Ig) isotypes has the highest concentration in serum? Single best answer question - choose ONE true option only A. IgA B. IgD C. IgE

Prepared by Dr: Mohammed Musa Brema Idress – My best wishes Page 138

D. IgG E. IgM

ANSWER IS D IgG is the major serum immunoglobin (Ig) while IgA is the predominant Ig in secretions and so protects mucous membranes.

380. Immunity conferred by immune responses involving plasma cells is described as? Single best answer question - choose ONE true option only A. Acquired immunity B. Cellular immunity C. Humoral immunity D. Innate immunity E. Passive immunity

ANSWER IS C Humoral immunity refers to antibody-mediated responses. Plasma cells are terminally differentiated B lymphocytes which secrete antibody.

381. Cyclosporine acts by? Single best answer question - choose ONE true option only A. Inhibiting interleukin (IL)-2 receptor expression B. Decreasing IL-2 release C. Increasing IL-10 production D. Decreasing CD4 expression E. Decreasing expression of the T cell receptor

ANSWER IS B Cyclosporine works by inhibiting release of interleukin (IL)-2 from T cells, thereby inhibiting their proliferation. IL-10 is a natural anti-inflammatory cytokine but no successful analogues are available in clinical practice.

382. Which of the following is associated with hyperacute allograft rejection? Single best answer question - choose ONE true option only A. Cytotoxic T cells B. Decay-accelerating factor C. Major histocompatibility complex (MHC) class II D. Natural killer (NK) cells E. Pre-existing humoral antibodies

ANSWER IS E Hyperacute graft rejection occurs within minutes of transplantation and is a consequence of pre-existing humoral antibodies. These are usually due to pre- sensitization to major histocompatibility complex (MHC) class I or to blood group incompatibility.

Prepared by Dr: Mohammed Musa Brema Idress – My best wishes Page 139

383. An allograft is? Single best answer question - choose ONE true option only A. Tissue from one individual transplanted to another B. Tissue which is identically matched for human leukocyte-associated antigen (HLA) Tissue from an animal transplanted to a human C. Tissue transplanted from one site to another in a single individual D. Artificial tissue implanted into an individual

ANSWER IS A An allograft is an organ graft between individuals of the same species. In practice these are matched as closely as possible for human leukocyte-associated antigen (HLA), but are rarely identical.

An autologous transplant is when the organ e.g. skin is moved from one part of the body to another. A xenograft is an organ transplant from another species e.g. pig to another e.g. human.

384. Graft versus host disease occurs when? Single best answer question - choose ONE true option only A. Immunocompetent cells from the recipient react with the donor tissue B. Immune cells which are carried within a transplanted organ react with donor C. Tissue The transplant recipient has already been sensitized to the donor tissue D. Immunocompetent stem cells are transplanted to an individual E. Natural killer cells are present in the donor tissue transplant

ANSWER IS D Graft versus host disease occurs when immunocompetent cells in the transplanted organ e.g. bone marrow attack the recipient. In practice this is prevented by irradiation of the recipient bone marrow and by T cell depletion of the transplanted bone marrow.

385. Skin grafts between individuals rarely reject even when they are not fully human leukocyte-associated antigen (HLA) matched because? Single best answer question - choose ONE true option only A. The skin has a poor blood supply B. Local immunosuppressive factors are produced by the transplant C. Human leukocyte-associated antigen (HLA) expression is low in the skin D. Immune cells are unable to penetrate the superficial layers of skin E. ‘Burn’ tissue is relatively immunosuppressed

ANSWER IS E Certain tissues appear to do well after transplantation even after modest human leukocyte-associated antigen (HLA) matching. Liver transplants are thought to express less class I major histocompatibility complex (MHC), and so HLA matching appears not to correlate with outcome. Burnt skin becomes relatively immunosuppressed and therefore fails to reject transplants. The cornea is relatively

Prepared by Dr: Mohammed Musa Brema Idress – My best wishes Page 140 a vascular thus protecting it from rejection. But burns is not the only indication for grafting

386. 25-year-old woman with lupus has double-stranded DNA (dsDNA) antibodies and a grade 3 glomerulonephritis. Which of the following most accurately describes the immunopathological process? Single best answer question - choose ONE true option only A. Activation of the alternative complement pathway B. Type II hypersensitivity reaction C. Type I hypersensitivity reaction D. Activation of the classical complement pathway E. A complement deficiency

ANSWER IS D Activation of the classical complement pathway occurs in systemic lupus erythematosus (SLE) due to the large number of double-stranded DNA (dsDNA) immune complexes that form and are able to fix complement. These are deposited in the kidneys and other organs, where they attract other components of the immune system and tissue damage ensues. Complement consumption is common in active disease in these cases. It could also be described as a type III hypersensitivity reaction.

387. A 65-year-old man suffered a massive myocardial infarction that was complicated by shock and prolonged hypotension. On arrival in the emergency department he was found to have focal neurological signs in addition to features consistent with low-output cardiac failure. Despite the best efforts of the medical team he died the next day. At autopsy, the most likely change you would expect to see in a brain biopsy would be: A. Acute haemorrhagic change B. Coagulative necrosis C. Granulomatous change D. Lacunar infarct E. Liquefactive necrosis

ANSWER IS E Liquefactive necrosis is characteristic of focal bacterial or, occasionally, fungal infections, because microbes stimulate the accumulation of inflammatory cells. Hypoxic death of cells within the central nervous system often evokes liquefactive necrosis, though the reasons for this are obscure. Whatever the pathogenesis, liquefaction completely digests the dead cells. The end result is transformation of the tissue into a viscous liquid mass. If the process was initiated by acute inflammation the material is frequently creamy yellow in colour because of the presence of dead white cells and this is called ‘pus’.

388. A 45-year-old woman with a chronic infective lesion on her leg underwent a full-thickness biopsy of the lesion. During histological examination of this lesion a rim of multinuclear giant cells is seen. The central region is most likely to show:

Prepared by Dr: Mohammed Musa Brema Idress – My best wishes Page 141

A. Caseous necrosis B. Eosinophilic necrosis C. Fibrinous necrosis D. Foam cells E. Pyogenic necrosis

ANSWER IS A Caseous necrosis, a distinctive form of coagulative necrosis, is encountered most often in foci of tuberculous infection. In this disease the characteristic lesion is called a ‘granuloma’ or a ‘tubercle’ and is classically characterized by the presence of central caseous necrosis.

The term ‘caseous’ is derived from the cheesy white gross appearance of the area of necrosis. On microscopic examination, the necrotic focus appears as amorphous granular debris, seemingly composed of fragmented, coagulated cells and amorphous granular debris enclosed within a distinctive inflammatory border known as a ‘granulomatous reaction’. Unlike coagulative necrosis, the tissue architecture is completely obliterated.

In a granuloma, macrophages are transformed into epithelium-like cells surrounded by a collar of mononuclear leukocytes, principally lymphocytes and occasionally plasma cells. In the usual haematoxylin and eosin-stained tissue sections, the epithelioid cells have a pale pink granular cytoplasm with indistinct cell boundaries, and often appear to merge into one another. The nucleus is less dense than that of a lymphocyte, is oval or elongated in shape, and can show folding of the nuclear membrane. Older granulomas develop an enclosing rim of fibroblasts and connective tissue.

Epithelioid cells often fuse to form giant cells in the periphery or sometimes in the centre of granulomas. These giant cells can attain diameters of 40–50 µm. They have a large mass of cytoplasm containing 20 or more small nuclei arranged either peripherally (Langhans-type giant cell) or haphazardly (foreign body-type giant cell). There is no known functional difference between these two types of giant cells. The chronic infective lesion in this scenario is most probably a tuberculous granuloma.

389. A skin biopsy from an anorexic 16-year-old girl showed cellular atrophy. During atrophy: A. The cell disappears B. Cellular organelles swell C. Cell size decreases D. Cell size increases E. Protein synthesis increases

ANSWER IS C Atrophy is the shrinkage in the size of the cell due to loss of cell substance. It represents a form of adaptive response and can result in cell death. Involvement of a

Prepared by Dr: Mohammed Musa Brema Idress – My best wishes Page 142 sufficient number of cells causes the entire tissue or organ to diminish in size or become atrophic. Atrophy can be physiological or pathological.

Physiological atrophy is commonly seen during early development and is exemplified by atrophy of the notochord and the thyroglossal duct during foetal development. Another common example of physiological atrophy is the decrease in size of the uterus shortly after childbirth. Pathological atrophy can be local or generalized, depending on the underlying cause.

The common causes of atrophy are the following: 1) Ageing (senile atrophy) - Ageing is associated with cell loss, typically seen in tissues containing permanent cells, particularly the brain and heart. 2) Decreased workload (atrophy of disuse) - This is best seen when a broken limb is immobilized in a plaster cast or when a patient is restricted to complete bed rest resulting in rapid skeletal muscle. The initial rapid decrease in cell size is reversible once activity is resumed. With more prolonged disuse, however, skeletal muscle fibers decrease in number as well as in size, and this atrophy can be accompanied by increased bone resorption, leading to osteoporosis of disuse. 3) Diminished blood supply - A decrease in blood supply (ischaemia) to a tissue occurring as a result of occlusive arterial disease results in atrophy of tissue because of progressive cell loss. An example of this is seen in late adult life as the brain undergoes progressive atrophy due to reduced blood supply secondary to atherosclerosis. 4) Inadequate nutrition - In profound protein-calorie malnutrition (marasmus) skeletal muscle is used as a source of energy after other reserves such as adipose stores have been depleted. This results in marked muscle wasting (cachexia). Cachexia is also seen in patients with chronic inflammatory diseases or cancer. In the former, chronic overproduction of the inflammatory cytokine, tumour necrosis factor (TNF) is thought to be responsible for appetite suppression and muscle atrophy. 5) Loss of endocrine stimulation - Many endocrine glands, the breast, and the reproductive organs are dependent on endocrine stimulation for normal metabolism and function. The loss of oestrogen stimulation after the menopause results in physiological atrophy of the endometrium, vaginal epithelium, and breast. 6) Loss of innervation (denervation atrophy). - Normal function of skeletal muscle depends on its nerve supply. Damage to the nerves leads to rapid atrophy of the muscle fibers supplied by those nerves. 7) Pressure. - Compression of tissues for any length of time can lead to atrophy. An enlarging benign tumour can cause atrophy in the surrounding compressed tissues. Atrophy in this setting is probably the result of ischaemic changes caused by compromise of the blood supply to the surrounding tissues by the expanding mass. Atrophy is brought about by a reduction in the structural components of the cell with concomitant diminished function. For instance in atrophic muscle

Prepared by Dr: Mohammed Musa Brema Idress – My best wishes Page 143

the cells contain fewer mitochondria and myofilaments and a reduced amount of endoplasmic reticulum. The fundamental cellular changes associated with atrophy are identical in all of the above settings and represent an adaptive response to ensure survival.

390. A 35-year-old man is a habitual smoker. If a biopsy is taken from the respiratory tract in this man, the epithelium of respiratory tract is most likely to show: A. Mucous hyperplasia B. Smooth-muscle hyperplasia C. Squamous cell anaplasia D. Squamous cell hypertrophy E. Stratified squamous metaplasia

ANSWER IS E Metaplasia is a reversible change in which one adult cell type (epithelial or mesenchymal) is replaced by another adult cell type. It can represent an adaptive substitution of cells that are sensitive to stress by cell types better able to withstand the adverse environment.

The most common type of epithelial metaplasia is columnar to squamous, as occurs in the respiratory tract in response to chronic irritation. In the habitual cigarette smoker, the normal ciliated columnar epithelial cells of the trachea and bronchi are often replaced focally or widely by stratified squamous epithelial cells.

The more rugged stratified squamous epithelium is able to survive under circumstances in which the more fragile, specialized columnar epithelium would have succumbed. Although the metaplastic squamous cells in the respiratory tract are capable of surviving this environment, an important protective mechanism — mucus secretion — is lost.

Epithelial metaplasia is therefore a two-edged sword and, in most circumstances, represents an undesirable change. Moreover, the influences that predispose to metaplasia, if persistent, can induce malignant transformation in metaplastic epithelium. A common form of cancer in the respiratory tract is therefore composed of squamous cells, which arise in areas of metaplasia of normal columnar epithelium into squamous epithelium.

391. Hypertrophy can be physiological or pathological and is caused by increased functional demand or by specific hormonal stimulation. Hypertrophy can be best described as: A. Abnormal deposition in a cell B. Change in cell morphology C. Decrease in cell size D. Increase in cell number E. Increase in cell size and in its organelles

Prepared by Dr: Mohammed Musa Brema Idress – My best wishes Page 144

ANSWER IS E Hypertrophy is caused by an increase in the size of cells, resulting in an increase in the size of the organ. The hypertrophied organ has no new cells, just larger cells. The increased size of the cells is due not to cellular swelling but to the synthesis of more structural components. Cells capable of division can respond to stress by undergoing both hyperplasia and hypertrophy, whereas in non dividing cells (e.g. myocardial fibers), hypertrophy occurs. The nuclei in hypertrophied cells can have a higher DNA content than normal cells, probably because the cells arrest in the cell cycle without undergoing mitosis.

Hypertrophy can be physiological or pathological and is caused by increased functional demand or by specific hormonal stimulation. The striated muscle cells in both the heart and the skeletal muscles are capable of tremendous hypertrophy, perhaps because they cannot adapt adequately to increased metabolic demands by mitotic division and production of more cells to share the work.

The most common stimulus for hypertrophy of muscle is increased workload. For example, the bulging muscles of bodybuilders engaged in ‘pumping iron result from an increase in size of the individual muscle fibers in response to increased demand. The workload is thus shared by a greater mass of cellular components, and each muscle fiber is spared excess work and so escapes injury.

The enlarged muscle cell achieves a new equilibrium, permitting it to function at a higher level of activity. In the heart, the stimulus for hypertrophy is usually chronic haemodynamic overload, resulting from either hypertension or faulty valves. Synthesis of more proteins and filaments occurs, achieving a balance between the demand and the cell’s functional capacity. The greater number of myofilaments per cell permits an increased workload with a level of metabolic activity per unit volume of cell not different from that borne by the normal cell.

The massive physiological growth of the uterus that occurs during pregnancy is a good example of a hormone-induced increase in the size of an organ that results from both hypertrophy and hyperplasia. The cellular hypertrophy is stimulated by oestrogenic hormones acting on smooth-muscle oestrogen receptors, eventually resulting in increased synthesis of smooth-muscle proteins and an increase in cell size. Similarly, prolactin and oestrogen cause hypertrophy of the breasts during lactation. These are examples of physiological hypertrophy induced by hormonal stimulation.

392. Apoptotic cells usually exhibit distinctive morphological features. Which one of the following morphological features is usually seen in pure apoptosis? A. Cellular swelling B. Chromatin condensation C. Early disruption of the plasma membrane D. Nuclear stabilization E. Phagocytosis of apoptotic bodies by neutrophils

Prepared by Dr: Mohammed Musa Brema Idress – My best wishes Page 145

ANSWER IS B Apoptosis is a pathway of cell death that is induced by a tightly regulated intracellular programme in which cells destined to die activate enzymes that degrade the cells’ own nuclear DNA and nuclear and cytoplasmic proteins. The cell’s plasma membrane remains intact, but its structure is altered in such a way that the apoptotic cell becomes an avid target for phagocytosis.

The dead cell is rapidly cleared, before its contents have leaked out, and therefore cell death by this pathway does not elicit an inflammatory reaction in the host. Apoptosis is therefore fundamentally different from necrosis, which is characterized by loss of membrane integrity, enzymatic digestion of cells, and frequently a host reaction. However, apoptosis and necrosis sometimes coexist, and they can share some common features and mechanisms.

The following morphological features, some best seen with the electron microscope, characterize cells undergoing apoptosis: 1) Cell shrinkage - The cell is smaller in size; the cytoplasm is dense; and the organelles, although relatively normal, are more tightly packed. 2) Chromatin condensation - This is the most characteristic feature of apoptosis. The chromatin aggregates peripherally, under the nuclear membrane, into dense masses of various shapes and sizes. The nucleus itself can break up, producing two or more fragments. 3) Formation of cytoplasmic blebs and apoptotic bodies - The apoptotic cell first shows extensive surface blebbing, then undergoes fragmentation into membrane-bound apoptotic bodies composed of cytoplasm and tightly packed organelles, with or without nuclear fragments. 4) Phagocytosis of apoptotic cells or cell bodies - Usually by macrophages. The apoptotic bodies are rapidly degraded within lysosomes, and the adjacent healthy cells migrate or proliferate to replace the space occupied by the now deleted apoptotic cell. Plasma membranes are thought to remain intact during apoptosis, until the last stages, when they become permeable to solutes that are normally retained. This classic description is accurate with respect to apoptosis during physiological conditions such as embryogenesis and deletion of immune cells. However, forms of cell death with features of necrosis as well as of apoptosis are not uncommon in reaction to injurious stimuli. Under such conditions, the severity, rather than the specificity, of the stimulus determines the form in which death is expressed. If necrotic features are predominant, early plasma membrane damage occurs, and cell swelling, rather than shrinkage, is seen.

393. A histopathologist reports fat necrosis after examining a slide. Fat necrosis might be found in which one of the following situations? A. Brain injury B. Muscle injury C. Trauma to the abdomen D. Trauma to the breast

Prepared by Dr: Mohammed Musa Brema Idress – My best wishes Page 146

E. Trauma to the bowel

ANSWER IS D Fat necrosis can be seen after trauma to the breast. It can present as a painless palpable mass, skin thickening or retraction, a mammographic density, or mammographic calcifications. The majority of women will give a history of trauma or prior surgery. The major clinical significance of the condition is its possible confusion with breast carcinoma when there is a palpable mass or mammographic calcifications.

Grossly, the lesion can consist of haemorrhage in the early stages and, later, central liquefactive necrosis of fat. Later still, it can appear as an ill-defined nodule of grey- white, firm tissue containing small foci of chalky-white or haemorrhagic debris. The central focus of necrotic fat cells is initially surrounded by macrophages and an intense neutrophilic infiltration. Then, during the next few days, progressive fibroblastic proliferation, increased vascularization, and lymphocytic and histiocytic infiltration wall off the focus. Subsequently, foreign-body giant cells, calcifications, and haemosiderin make their appearance. Eventually, the focus is replaced by scar tissue or is encysted and walled off by collagenous tissue.

394. A healthy 26-year-old man fractured his right tibia in a road traffic accident. His right leg was immobilized in a plaster cast. The cast was removed from his leg after 8 weeks of immobilization. Which of the following changes is most likely to have taken place in his gastrocnemius muscle after this time? A. Decrease in the number of muscle fibers B. Decrease in the number of nerve fibers C. Increase in the number of fast fibers D. Increase in the mitochondrial content E. Increase in the number of satellite cells

ANSWER IS A Shrinkage in the size of the cell by loss of cell substance is known as ‘atrophy’. It represents a form of adaptive response and can culminate in cell death. When a sufficient number of cells are involved, the entire tissue or organ diminishes in size, or becomes atrophic. When a broken limb is immobilized in a plaster cast or when a patient is restricted to complete bed rest, skeletal muscle atrophy rapidly ensues. The initial rapid decrease in cell size is reversible once activity is resumed. With more prolonged disuse, however, skeletal muscle fibers decrease in number as well as in size; this atrophy can be accompanied by increased bone resorption, leading to osteoporosis of disuse.

395. A 21-year-old man sustained a severe soft-tissue injury following a road traffic accident. Which of the following metabolic effects is most likely to follow this injury? A. Decreased aldosteron secretion B. Inhibition of gluconeogenesis C. Mobilization of fat stores

Prepared by Dr: Mohammed Musa Brema Idress – My best wishes Page 147

D. Protein anabolism E. Respiratory alkalosis

ANSWER IS C The metabolic response to injury is an important part of the stress reaction in that it improves the individual organism’s chances of surviving under adverse circumstances or when injured.

The metabolic response to trauma includes: 1) Acid–base disturbance — usually a metabolic alkalosis or acidosis 2) Early reduced urine output and increased urine osmolality 3) Early reduction in metabolic rate 4) Gluconeogenesis via amino acid breakdown 5) Hyponatraemia due to impaired sodium pump action 6) Hypoxia and coagulopathy 7) Immunosuppression 8) Increased extracellular fluid volume volume and hypovolaemia 9) Increased vascular permeability and oedema 10) Late diuresis and increased sodium loss 11) Late increased metabolism, negative nitrogen balance and weight loss 12) Mobilization of fat stores, lipolysis and ketosis 13) Pyrexia in the absence of infection 14) Reduced ‘free’ water clearance 15) Reduced serum albumin.

396. A 32-year-old man, working in a power plant, was exposed to radioactive material. He is most likely to suffer radiation injury due to: A. Decreased intracellular Na+ B. Decreased intracellular Ca2+ C. Free radical formation D. Increased adenosine triphosphate (ATP) production E. Inhibition of protein synthesis

ANSWER IS C Cells generate energy by reducing molecular oxygen to water. During this process small amounts of partially reduced reactive oxygen forms are produced as an unavoidable byproduct of mitochondrial respiration. Some of these forms are free radicals that can damage lipids, proteins and nucleic acids. They are referred to as ‘reactive oxygen species’. Cells have defence systems for preventing injury caused by these products.

An imbalance between free radical generating and free radical-scavenging systems results in oxidative stress, a condition that has been associated with the cell injury seen in many pathological conditions. Free radical-mediated damage contributes to such varied processes as chemical and radiation injury, ischaemia-reperfusion injury (induced by restoration of blood flow in ischaemic tissue), cellular ageing and microbial killing by phagocytes.

Prepared by Dr: Mohammed Musa Brema Idress – My best wishes Page 148

397. A 15-year-old girl with haemophilia A has had episodes of pain in her knees for the past 6 years. Over time, there has been an increase in size of her knee joints, with deformity. Laboratory studies show decreased levels of coagulation factor VIII activity. Which of the following is most likely to be seen within the joint space following episodes of pain? A. Anthracotic pigment B. Cholesterol crystals C. Lipofuscin D. Neutrophils E. Russell bodies

ANSWER IS B In haemophiliac joints the lipid from the red cell membranes is broken down and cholesterol crystals form. Accumulation of lipofuscin is not a feature of haemorrhage. Russell bodies are intracellular accumulations of immunoglobulins in plasma cells. Neutrophils suggest acute inflammation. Anthracotic pigment is an exogenous carbon pigment from dusts in the air that accumulate in lung.

398. A 42-year-old woman has complained of mild, burning, substernal or epigastric pain following meals for the past 3 years. Upper gastrointestinal endoscopy is performed and are taken of an erythematous area of the lower oesophageal mucosa 3 cm above the gastro-oesophageal junction. There is no mass lesion, no ulceration, and no haemorrhage is noted. The biopsies demonstrate the presence of columnar epithelium with goblet cells. Which of the following mucosal alterations is most likely to be represented by these findings? A. Carcinoma B. Dysplasia C. Hyperplasia D. Ischaemia E. Metaplasia

ANSWER IS E Metaplasia is the substitution of one tissue type normally found at a site for another. The epithelium undergoes metaplasia in response to ongoing inflammation from reflux of gastric contents. It is common in the lower oesophagus with gastro- oesophageal reflux disease. The growth of the epithelial cells must become disordered in order to be dysplastic.

Hyperplasia can occur with inflammation, as the number of cells increases, but hyperplasia does not explain the presence of the columnar cells. Carcinoma is characterized by cellular atypia with hyperchromatism and pleomorphism. Goblet cells would not be seen. Ischaemia would be unusual at this site and would be marked by coagulative necrosis.

399. A 62-year-old diabetic and hypertensive man suffered a stroke which affected his speech and movement in the right arm and leg. A cerebral angiogram revealed an occlusion of his left middle cerebral artery. Months later, a computed

Prepared by Dr: Mohammed Musa Brema Idress – My best wishes Page 149 tomographic (CT) scan shows a large, 5-cm cystic area in his left parietal lobe cortex. This CT finding most likely demonstrates a lesion that is the consequence of resolution of which of the following events? A. Apoptosis B. Atrophy C. Caseous necrosis D. Coagulative necrosis E. Liquefactive necrosis

ANSWER IS E The brain undergoes liquefactive necrosis with infarction. As it resolves, a cystic area forms in the region of infarction. Atrophy would be a more generalized process, whereas a single cystic area in the brain suggests a remote infarction.

Coagulative necrosis is more typical of parenchymal organs such as the kidney or spleen, which do not have as high a lipid content as the brain.

Caseous necrosis is more typical of granulomatous inflammation with Mycobacterium tuberculosis. Apoptosis is single-cell necrosis that does not result in a grossly visible cystic area

400. A 25-year-old woman breastfed her first baby for almost 1 year with no difficulties and no complications. Which of the following cellular processes that occurred in the breast during pregnancy allowed her to nurse the infant for this period of time? A. Ductal epithelial metaplasia B. Epithelial dysplasia C. Lobular hyperplasia D. Stromal hypertrophy E. Steatocyte atrophy

ANSWER IS C There is an increase in lobules under hormonal influence to provide for lactation. Lobular hyperplasia would allow this woman to nurse the infant. The stroma of the breast consists of connective tissue, which provides structural support but does not have cells that produce milk. Dysplasia in epithelia is a premalignant change and not a normal physiological event. With poor nutrition and weight loss, steatocytes can decrease in size. Metaplasia is the exchange of normal epithelium for another type in response to chronic irritation. Metaplasia is not a normal physiological process.

401. An 80-year-old woman was found dead in her room in a nursing home one morning. An autopsy was carried out and her death was reported as being secondary to old age. At autopsy, her heart was small (250 g) and dark-brown in colour on section. Microscopically, there was light-brown perinuclear pigment seen after haematoxylin and eosin (H&E) staining of the cardiac muscle fibers. Which of the following substances is most likely to be increased in the myocardial fibers to produce this cardiac appearance?

Prepared by Dr: Mohammed Musa Brema Idress – My best wishes Page 150

A. Calcium, following necrosis B. Cholesterol, as a consequence of atherosclerosis C. Glycogen, resulting from a storage disease D. Haemosiderin, resulting from iron overload E. Lipochrome, from ‘wear and tear’

ANSWER IS E Lipochrome is a very common finding in older people, but it has little effect on cardiac function. Lipochrome is also known as ‘lipofuscin’. Haemosiderosis is not a complication of ageing. Glycogen storage diseases are inherited conditions that appear early in life. Glycogen does not appear pigmented in haematoxylin and eosin- stained tissue sections. Cholesterol accumulates in atheromatous plaques in the arteries, not in the myocardium. Calcium deposits appear as irregular, dark-blue areas and are not associated with ageing of the myocardium.

402. A histopathologist examining a slide used a series of immunohistochemical stains to identify different cellular components. One particular stain identified the presence of intermediate filaments within cells. This cytokeratin stain is most likely to be useful for which of the following diagnostic purposes? A. Confirmation of a history of chronic alcoholism B. Confirmation that a is a carcinoma C. Determination of the contractile properties of cells D. Visualization of cytoskeletal alterations with impending cell death E. Assessment of the degree of metaplasia or dysplasia

ANSWER IS B Malignant tumours of diverse origin often resemble one another because they are poorly differentiated. These tumours are often quite difficult to distinguish on the basis of routine haematoxylin and eosin- (H&E-) stained tissue sections. For example, certain anaplastic carcinomas, malignant lymphomas, melanomas and sarcomas can look quite similar, but they must be accurately identified because their treatment and prognosis are different. Antibodies against intermediate filaments have proved to be of value in such cases because tumour cells often contain intermediate filaments characteristic of their cell of origin. For example, the presence of cytokeratins, detected immunohistochemically, points to an epithelial origin (carcinoma), whereas desmin is specific for of muscle cell origin. Cytoskeletal alterations occur with ischaemia, but are not a useful marker for such an event. Many cell types contain intermediate filaments. Mallory’s alcoholic hyaline can be observed by H&E staining. However, it is not entirely specific for alcoholism. Metaplasia and dysplasia are assessed using their light-microscopic appearances after H&E staining.

403. A 73-year-old woman with long-standing hypertension and aortic stenosis died suddenly one morning. An autopsy was performed on her body. At autopsy, her heart weighed 540 g. The size of her heart is most likely to be the result of which of the following processes involving the myocardial fibers? A. Fatty degeneration

Prepared by Dr: Mohammed Musa Brema Idress – My best wishes Page 151

B. Fatty infiltration C. Hyperplasia D. Hypertrophy E. Oedema

ANSWER IS D In this patient the pressure load of hypertension led to myocardial fiber hypertrophy and a heart twice normal size. Fat in the heart does not increase in response to the increase in workload from hypertension. Myocardial fibers do not undergo hyperplasia. Fatty degeneration of the myocardium is typically the result of a toxic or hypoxic injury. Myocardial oedema is not a characteristic feature of myocardial injury or increased workload. However, heart failure could lead to peripheral oedema.

404. A 28-year-old woman developed a darker skin complexion after returning from a holiday trip to Goa. Her skin did not show warmth, erythema or tenderness. Her skin tone faded to its original appearance within 1 month. Which of the following substances contributes most to the biochemical process leading to these skin changes? A. Glycogen B. Haem C. Homogentisic acid D. Lipofuscin E. Tyrosine

ANSWER IS E The tanning process in skin is stimulated by ultraviolet light exposure. Melanocytes have the enzyme tyrosinase to oxidize tyrosine to dihydroxyphenylalanine in the pathway for melanin production. Haem as part of haemosiderin from breakdown of red blood cells can impart a brownish colour, but this is typically local from trauma or more global as part of an iron storage disease such as haemochromatosis. Lipochrome (lipofuscin) is a ‘wear and tear’ pigment that imparts a golden-brown appearance to granules in cells (such as myocytes or hepatocytes), but this is not a feature of skin. Homogentisic acid can be part of the process of the rare disease alkaptonuria, in which a black pigment is deposited in connective tissues. Glycogen in sufficient quantity is starch-like and imparts a paler colour to organs in which it is stored in excess. It does not involve the skin.

405. Metaplasia is a reversible change in which one adult cell type (epithelial or mesenchymal) is replaced by another adult cell type. In which of the following situations is the process of epithelial metaplasia most likely to have occurred? A. Acute myocardial infarction B. Lactation following pregnancy C. Tanning of the skin following sunlight exposure D. Urinary obstruction due to an enlarged prostate E. Vitamin A deficiency

Prepared by Dr: Mohammed Musa Brema Idress – My best wishes Page 152

ANSWER IS E Vitamin A is necessary to maintain epithelia, and squamous metaplasia of the respiratory tract can occur if there is a deficiency. Tanning of the skin is a physiological event resulting from the accumulation of melanin pigment — there is no change of one cell type to another involved. Lactation following pregnancy is a form of physiological hyperplasia of breast lobules that occurs as a result of hormonal influences.

An acute myocardial infarction will lead to cardiac muscle fiber necrosis, which will heal with the production of fibrous scar tissue, but this is not a reversible metaplasia. The enlarged prostate represents primarily glandular hyperplasia.

406. A histopathologist reviewing a slide noticed a disease process which has led to scattered loss of individual cells, with the microscopic appearance of karyorrhexis and cell fragmentation. The overall tissue structure, however, has remained intact. This process is most typical of which of the following diseases? A. Barbiturate overdose B. Brown atrophy of the heart C. Chronic alcoholic liver disease D. Renal transplant rejection E. Viral hepatitis

ANSWER IS E Viral infection leads to individual hepatocyte necrosis, which is characterized by the microscopic appearances of karyorrhexis and cell fragmentation. Brown atrophy of the heart results when there is marked lipofuscin deposition in the myocardium. Tissue destruction with transplant rejection is more widespread. Single cell necrosis is not evident in chronic alcoholic liver disease. Barbiturate overdose causes hypertrophy of smooth endoplasmic reticulum, not individual cell necrosis.

407. A 55-year-old woman with chronic atrial fibrillation suddenly developed an acute abdomen and was rushed to the emergency department. At emergency most of the small-bowel loops were dusky to purple-red in colour. Her mesenteric veins were patent. The most probable underlying pathological process is: A. Coagulative necrosis B. Dry gangrene C. Gas gangrene D. Liquefactive gangrene E. Wet gangrene

ANSWER IS E Small-intestinal infarction following sudden and total occlusion of mesenteric arterial blood flow can involve only a short segment, but more often involves a substantial portion.

Prepared by Dr: Mohammed Musa Brema Idress – My best wishes Page 153

The splenic flexure of the colon is at greatest risk of ischaemic injury because it is the watershed between the distribution of the superior and inferior mesenteric arteries, but any portion of the colon can be affected. With mesenteric venous occlusion, anterograde and retrograde propagation of thrombus may lead to extensive involvement of the splanchnic bed.

Regardless of whether the arterial or venous side is occluded, the infarction appears haemorrhagic because of blood reflow into the damaged area. In the early stages, the infracted bowel appears intensely congested and dusky to purple-red, with foci of subserosal and submucosal ecchymotic discoloration. With time, the wall becomes oedematous, thickened, rubbery, and haemorrhagic.

The lumen commonly contains sanguinous mucus or frank blood. In arterial occlusions the demarcation from normal bowel is usually sharply defined, but in venous occlusions the area of dusky cyanosis fades gradually into the adjacent normal bowel, having no clear-cut definition between viable and non-viable bowel.

Histologically, there is obvious oedema, interstitial haemorrhage, and sloughing necrosis of the mucosa. Normal features of the mural musculature, particularly cellular nuclei, become indistinct. Within 1–4 days, intestinal bacteria produce outright wet gangrene and sometimes perforation of the bowel. There may be little inflammatory response.

408. Accumulation of which of the following substances indicates ageing at a cellular level in a biopsy taken from an elderly person? A. Beta carotene B. Bilirubin C. Haemosiderin D. Lipofuscin E. Melanin

Answer IS D

409. A 48 year old women presents with recurrent loin pain and fevers. Investigation reveals a staghorn calculus of the left kidney. Infection with which of the following organisms is most likely? A. Staphylococcus saprophyticus B. Proteus mirabilis C. Klebsiella D. E-Coli E. Staphylococcus epidermidis

ANSWER IS B Infection with Proteus mirabilis accounts for 90% of all proteus infections. It has a urease producing enzyme. This will tend to favor urinary alkalinization which is a relative per-requisite for the formation of staghorn calculi.

Prepared by Dr: Mohammed Musa Brema Idress – My best wishes Page 154

410. Causes of granulomatous disease do not include: A. Amiodarone B. Allopurinol C. Sulphonamides D. Beryllium E. Wegener's granulomatosis

ANSWER IS A Allopurinol and sulphonamides cause hepatic granulomas.

411. A 59 year old lady is referred from the NHS breast screening program. A recent mammogram is reported as showing linear, branching microcalcification with coarse granules. Which disease process is the most likely underlying cause of these appearances? A. Invasive lobular cancer B. Lobular carcinoma in situ C. Cribriform type ductal carcinoma in situ D. Comedo type ductal carcinoma in situ E. Fibroadenosis

ANSWER IS D Comedo type DCIS is usually associated with microcalcifications. Cribriform lesions are usually multifocal but less likely to form microcalcifications. Lobular cancers and in situ lesions rarely form microcalcifications and are difficult to detect using mammography.

412. In patients with an where is the most likely site of obstruction? A. The first part of the duodenum B. The second part of the duodenum C. The fourth part of the duodenum D. The third part of the duodenum E. The duodeno-jejunal flexure

ANSWER IS B The pancreas develops from two foregut outgrowths (ventral and dorsal). During rotation the ventral bud and adjacent gallbladder and bile duct lie together and fuse. When the pancreas fails to rotate normally it can compress the duodenum with development of obstruction.

Usually occurring as a result of associated duodenal malformation. The second part of the duodenum is the commonest site.

413. A 32 year old woman presents with an episode of haemoptysis and is found to have metastatic tumour present within the parenchyma of the lungs. This is biopsied and subsequent histology shows clear cells. What is the most likely primary site?

Prepared by Dr: Mohammed Musa Brema Idress – My best wishes Page 155

A. Kidney B. Breast C. Liver D. Adrenal E. Bone

ANSWER IS A Clear cell tumours are a sub type of renal cell cancer it is associated with specific genetic changes localized to chromosome 3

414. A laceration of the wrist produces a median nerve transection. The wound is clean and seen immediately after injury. Collateral soft tissue damage is absent. The patient asks what the prognosis is. You indicate that the nerve should regrow at approximately: A. 0.1 mm per day B. 1 mm per day C. 5 mm per day D. 1 cm per day E. None of the above

ANSWER IS B Transaction of a peripheral nerve results in hemorrhage and in retraction of the several nerve ends. Almost immediately, degeneration of the axon distal to the injury begins. Degeneration also occurs in the proximal fragment back to the first node of Ranvier.

Phagocytosis of the degenerated axonal fragments leaves neurilemmal sheath with empty cylindrical spaces where the axons were. Several days following the injury, axons from the proximal fragment begin to regrow. If they make contact with the distal neurilemmal sheath, regrowth occurs at about the rate of 1 mm/day. However, if associated trauma, fracture, infection, or separation of neurilemmal sheath ends precludes contact between axons, growth is haphazard and a traumatic neuroma is formed. When neural transaction is associated with widespread soft tissue damage and hemorrhage (with increased probability of infection), many surgeons choose to delay reapproximation of the severed nerve end for 3 to 4 weeks.

Nerve injury There are 3 types of nerve injury: Neuropraxia - Nerve intact but electrical conduction is affected - Full recovery - Autonomic function preserved - Wallerian degeneration does not occur Axonotmesis - Axon is damaged and the myelin sheath is preserved. The connective tissue framework is not affected.

Prepared by Dr: Mohammed Musa Brema Idress – My best wishes Page 156

- Wallerian degeneration occurs. Neurotmesis - Disruption of the axon, myelin sheath and surrounding connective tissue. - Wallerian degeneration occurs.

Wallerian Degeneration - Axonal degeneration distal to the site of injury. - Typically begins 24-36 hours following injury. - Axons are excitable prior to degeneration occurring. - Myelin sheath degenerates and is phagocytosed by tissue macrophages. Nerve repair - Neuronal repair may only occur physiologically where nerves are in direct contact. Where a large defect is present the process of nerve regeneration is hampered and may not occur at all or result in the formation of a neuroma. Where nerve regrowth occurs it typically occurs at a rate of 1mm per day

415. Which of the following statements relating to gastric cancer is untrue? A. It is associated with chronic helicobacter pylori infection B. 5% of gastric malignancies are due to lymphoma C. In the Lauren classification the diffuse type of adenocarcinoma typically presents as a large exophytic growth in the antrum D. Smoking is a risk factor E. It is associated with acanthosis nigricans

ANSWER IS C The Lauren classification describes a diffuse type of adenocarcinoma (Linitis plastica type lesion) and an intestinal type. The diffuse type is often deeply infiltrative and may be difficult to detect on endoscopy. Barium meal appearances can be characteristic.

416. Which of the following statements relating to Gardner’s syndrome variant of familial adenomatous polyposis coli is false? A. It is an autosomal dominant condition B. Patients may develop retroperitoneal desmoid tumours C. The vast majority of the polyps are benign and thus the risk of colorectal cancer is small D. Patients are at increased risk of thyroid cancer E. It is characterized by a mutation in the APC gene

ANSWER IS C - The multiple polyps increase the risk of malignancy and most patients should undergo a colectomy - Gardner’s: Osteomata fibroma sebaceous cyst and polyposis - 100% risk of malignancy in duodenum n colorectal

417. A 60-year-old man presents with lower urinary tract symptoms and is offered a PSA test. Which one of the following could interfere with the PSA level?

Prepared by Dr: Mohammed Musa Brema Idress – My best wishes Page 157

A. Vigorous exercise in the past 48 hours B. Poorly controlled diabetes mellitus C. Drinking more than 4 units of alcohol in the past 48 hours D. Smoking E. Recent cholecystectomy

ANSWER IS A PSA testing Prostate specific antigen (PSA) is a serine protease enzyme produced by normal and malignant prostate epithelial cells. It has become an important tumour marker but much controversy still exists regarding its usefulness as a screening tool.

The NHS Prostate Cancer Risk Management Programme (PCRMP) has published updated guidelines in 2009 on how to handle requests for PSA testing in asymptomatic men.

A recent European trial (ERSPC) showed a statistically significant reduction in the rate of death prostate cancer by 20% in men aged 55 to 69 years but this was associated with a high risk of over-diagnosis and over-treatment. Having reviewed this and other data the National Screening Committee have decided not to introduce a prostate cancer screening programme yet but rather allow men to make an informed choice.

Age-adjusted upper limits for PSA were recommended by the PCRMP*: {Age} {PSA level (ng/ml)} 50-59 years 3.0 60-69 years 4.0 > 70 years 5.0 PSA levels may also be raised by**: 1. Benign prostatic hyperplasia (BPH) 2. Prostatitis and urinary tract infection (NICE recommend to postpone the PSA test for at least 1 month after treatment) 3. Ejaculation (ideally not in the previous 48 hours) 4. Vigorous exercise (ideally not in the previous 48 hours) 5. 6. Instrumentation of the urinary tract Poor specificity and sensitivity - Around 33% of men with a PSA of 4-10 ng/ml will be found to have prostate cancer. With a PSA of 10-20 ng/ml this rises to 60% of men - Around 20% with prostate cancer have a normal PSA - Various methods are used to try and add greater meaning to a PSA level including age-adjusted upper limits and monitoring change in PSA level with time (PSA velocity or PSA doubling time) *aide memoire for upper PSA limit: (age - 20) / 10 **whether digital rectal examination actually causes a rise in PSA levels is a matter of debate

418. Which of the symptoms below is least typical of pancreatic cancer? A. Painless jaundice B. Hyperamylasaemia

Prepared by Dr: Mohammed Musa Brema Idress – My best wishes Page 158

C. Hyperglycaemia D. Weight loss E. Classical Courvoisier syndrome

ANSWER IS B Raised serum amylase is relatively uncommon. The typical Courvoisier syndrome typically occurs in 20% and hyperglycaemia occurs in 15-20%. Pancreatic cancer Adenocarcinoma Risk factors: - Smoking, diabetes, Adenoma, Familial adenomatous polyposis - Mainly occur in the head of the pancreas (70%) - Spread locally and metastasizes to the liver - Carcinoma of the pancreas should be differentiated from other periampullary tumours with better prognosis Clinical features 1) Weight loss 2) Painless jaundice 3) Epigastric discomfort (pain usually due to invasion of the coeliac plexus is a late feature) 4) Pancreatitis 5) Trousseau's sign: migratory superficial thrombophlebitis Investigations - USS: May miss small lesions - CT Scanning (pancreatic protocol). If unresectable on CT then no further staging needed. - PET/CT for those with operable disease on CT alone - ERCP/ MRI for bile duct assessment. - Staging laparoscopy to exclude peritoneal disease.

Management 1) Head of pancreas: Whipple's resection (SE dumping and ulcers). Newer techniques include pylorus preservation and SMA/ SMV resection. 2) Carcinoma body and tail: poor prognosis, distal pancreatectomy if operable. 3) Usually adjuvant chemotherapy for resectable disease 4) ERCP and stent for jaundice and palliation. 5) Surgical bypass may be needed for duodenal obstruction.

419. A 53 year old man presents with dyspepsia. An upper GI endoscopy is performed and Helicobacter pylori is identified. A duodenal ulcer is present in the first part of the duodenum. Duodenal biopsies are taken and demonstrate epithelium that resembles cells of the gastric antrum. Which of the following is the most likely explanation for this process? A. Hyperplasia of the crypts of Lieberkhun B. Duodenal metaplasia C. Duodenal dysplasia D. Duodenal carcinoma

Prepared by Dr: Mohammed Musa Brema Idress – My best wishes Page 159

E. Hypertrophy of Brunner’s glands

ANSWER IS B The process involved is metaplasia. During metaplasia there is no direct carcinogenesis, however the persistent presence of precipitants of metaplasia will lead to malignant changes in cells. Metaplastic changes in the duodenal cap are frequently seen in association with H-Pylori induced ulcers. It typically resolves after ulcer healing and eradication therapy.

Metaplasia Definition: reversible change of differentiated cells to another cell type. - May represent an adaptive substitution of cells that are sensitive to stress by cell types better able to withstand the adverse environment. - Can be a normal physiological response (ossification of cartilage to form bone) - Most common epithelial metaplasia occurs with transformation of columnar cells to squamous cells (smoking causes ciliated columnar cells to be replaced by squamous epithelial cells; Schistosomiasis). - Metaplasia from squamous to columnar cells occurs in Barrett oesophagus. - If the metaplastic stimulus is removed, the cells will return to their original pattern of differentiation. However, if the stimulus is not removed then progression to dysplasia may occur. - Not considered directly carcinogenic, however the factors which predispose to metaplasia, if persistent may induce malignant transformation. - The pathogenesis involves a reprogramming of stem cells that are known to exist in normal tissues, or of undifferentiated mesenchymal cells present in connective tissue. In a metaplastic change, these precursor cells differentiate along a new pathway.

420. A 22 year old lady presents with symptoms and signs of hyperthyroidism. Her diagnostic work up results in a diagnosis of Graves’s disease. Which of the following best describes the pathophysiology of the condition? A. Formation of IgG antibodies to the TSH receptors on the thyroid gland B. Formation of IgG antibodies to the TRH receptors on the anterior pituitary C. Formation of IgM antibodies to the TSH receptors on the thyroid gland D. Formation of IgA antibodies to the TSH receptors on the thyroid gland E. Formation of IgM antibodies to the TRH receptors on the anterior pituitary

ANSWER IS A - Usually IgG antibodies are formed against the TSH receptors on the thyroid gland. This is why the TSH level is often very low in Grave’s disease.

Thyroid hormones Hormones of the thyroid gland Triiodothyronine T3 Major hormone active in target cells Thyroxin T4 Most prevalent form in plasma, less biologically active than T3 Calcitonin Lowers plasma calcium

Synthesis and secretion of thyroid hormones

Prepared by Dr: Mohammed Musa Brema Idress – My best wishes Page 160

1- Thyroid actively concentrates iodide to twenty five times the plasma concentration. 2- Iodide is oxidized by peroxidase in the follicular cells to atomic iodine which then iodinates tyrosine residues contained in thyroglobulin. 3- Iodinated tyrosine residues in thyroglobulin undergo coupling to either T3 or T4. 4- Process is stimulated by TSH, which stimulates secretion of thyroid hormones. 5- The normal thyroid has approximately 3 month reserves of thyroid hormones. LATS and Graves disease In Graves disease patients develop IgG antibodies to the TSH receptors on the thyroid gland. This results in chronic and long term stimulation of the gland with release of thyroid hormones. The typically situation is raised thyroid hormones and low TSH. Thyroid receptor autoantibodies should be checked in individuals presenting with hyperthyroidism as they are present in up to 85% cases.

421. A 45 year old man has widespread metastatic adenocarcinoma of the colon. Which of the following tumour markers is most likely to be elevated? A. CA19-9 B. Carcinoembryonic antigen C. Alpha Fetoprotein D. CA 125 E. Beta BCG

ANSWER IS B Screening for colonic cancer using CEA is not justified. Carcinoembryonic antigen is elevated in colonic cancer, typically in relation to disease extent with highest serum levels noted in metastatic disease. It is falsely elevated in a number of non-malignant disease states such as cirrhosis and colitis and for this reason it has no role in monitoring colitis for colonic cancer.

Colorectal cancer screening and diagnosis: Overview - Most cancers develop from adenomatous polyps. Screening for colorectal cancer has been shown to reduce mortality by 16% - The NHS now has a national screening programme offering screening every 2 years to all men and women aged 60 to 69 years. Patients aged over 70 years may request screening - Eligible patients are sent faecal occult blood (FOB) tests through the post - Patients with abnormal results are offered a colonoscopy At colonoscopy, approximately: - 5 out of 10 patients will have a normal exam - 4 out of 10 patients will be found to have polyps which may be removed due to their premalignant potential - 1 out of 10 patients will be found to have cancer

Prepared by Dr: Mohammed Musa Brema Idress – My best wishes Page 161

Diagnosis - Essentially the following patients need referral: - Altered bowel habit for more than six weeks - New onset of rectal bleeding - Symptoms of tenesmus Colonoscopy is the gold standard; provided it is complete and good mucosal visualization is achieved. Other options include double contrast barium enema and CT colonography. Staging - Once a malignant diagnosis is made patients with colonic cancer will be staged using chest / abdomen and pelvic CT. Patients with rectal cancer will also undergo evaluation of the mesorectum with pelvic MRI scanning. For examination purposes the Dukes and TNM systems are preferred. Tumour markers Carcinoembryonic antigen (CEA) is the main tumour marker in colorectal cancer. Not all tumours secrete this, and it may be raised in conditions such as IBD. However, absolute levels do correlate (roughly) with disease burden and whilst this marker may not be used extensively in follow up; it can be useful for investigation of patients with cancer of unknown primary.

422. Which of the following does not cause hypercalcaemia? A. Thiazides B. DiGeorge syndrome C. Vitamin A D. Rhabdomylosis E. Sarcoidosis

ANSWER IS B VITAMINS TRAP V - Vitamins A & D I - Immobilization T - Thyrotoxicosis A - Addison's disease M - Milk-alkali syndrome I - Inflammatory disorders N - Neoplastic diseases S - Sarcoidosis T - Thiazides and other drugs R - Rhabdomyolysis A - AIDS P - Paget's disease, Parenteral nutrition, Parathyroid disease. - The parathyroid glands don't form as a result of this syndrome.

Hypercalcaemia Main causes 1) Malignancy (most common cause in hospital in-patients) 2) Primary hyperparathyroidism (commonest cause in non hospitalized patients) Less common 3) Sarcoidosis (extrarenal synthesis of calcitriol )

Prepared by Dr: Mohammed Musa Brema Idress – My best wishes Page 162

4) Thiazides, lithium 5) Immobilization 6) Paget’s disease 7) Vitamin A/D toxicity 8) Thyrotoxicosis 9) MEN 10) Milk alkali syndrome Clinical features - Stones, bones, abdominal moans, and psychic groans High serum calcium levels result in decreased neuronal excitability. Therefore sluggish reflexes, muscle weakness and constipation may occur.

423. A 25 year old man is found to have carcinoid syndrome. Which of the following hormones is released by carcinoids? A. Serotonin B. Dopamine C. Noradrenaline D. Adrenaline E. Aldosteron

ANSWER IS A Rule of thirds: - 1/3 multiple 1/3 small bowel 1/3 metastasize 1/3 second tumour Carcinoids secrete serotonin. Carcinoid syndrome will only occur in the presence of liver metastasis as the hormone released from primary lesions will typically be metabolized by the liver.

Carcinoid syndrome: - Carcinoid tumours secrete serotonin - Originate in neuroendocrine cells mainly in the intestine (midgut-distal ileum/appendix) - Can occur in the rectum, bronchi - Hormonal symptoms mainly occur when disease spreads outside the bowel Clinical features - Onset: years - Flushing face - Palpitations - Tricuspid stenosis causing dyspnoea - Asthma - Severe diarrhoea (secretory, persists despite fasting) Investigation - 5-HIAA in a 24-hour urine collection - Scintigraphy - CT scan Treatment - Octreotide - Surgical removal

424. Which one of the following genes protects against neoplasms? A. sis B. p53 C. ras D. myc E. src

Prepared by Dr: Mohammed Musa Brema Idress – My best wishes Page 163

ANSWER IS B P53 is a tumour suppressor gene and located on chromosome 17. It plays an important role in causing cells that are undergoing neoplastic changes to enter an apoptotic pathway.

Oncogenes - Oncogenes are cancer promoting genes that are derived from normal genes (proto-oncogenes). Protooncogenes play an important physiological role in cellular growth. They are implicated in the development of up to 20% of human cancers. Proto-oncogenes may become oncogenes via the following processes: A. Mutation (point mutation) B. Chromosomal translocation C. Increased protein expression - Only one mutated copy of the gene is needed for cancer to occur - a dominant effect Classification of oncogenes 1. Growth factors e.g. Sis 2. Transcription factors e.g. Myc 3. Receptor tyrosine kinase e.g. RET 4. Cytoplasmic tyrosine kinase e.g. Src 5. Regulatory GTPases e.g. Ras Tumour suppressor genes - Tumour suppressor genes restrict or repress cellular proliferation in normal cells. Their inactivation through mutation or germ line incorporation is implicated in renal, colonic, breast, bladder and many other cancers. One of the best known tumour suppressor genes is p53. p53 gene offers protection by causing apoptosis of damaged cells. Other well known genes include BRCA 1 and 2.

425. A 55 year old man with a long history of achalasia is successfully treated by a Heller’s Cardiomyotomy. Several years later he develops an oesophageal malignancy. Which of the following lesions is most likely to be present? A. Adenocarcinoma B. Gastrointestinal stromal tumour C. Leiomyosarcoma D. Rhabdomyosarcoma E. Squamous cell carcinoma

ANSWER IS E Achalasia is a rare condition. However, even once treated there is an increased risk of malignancy. When it does occur it is most likely to be of squamous cell type.

Oesophageal cancer - Incidence is increasing

Prepared by Dr: Mohammed Musa Brema Idress – My best wishes Page 164

- In most cases in the Western world this increase is accounted for by a rise in the number of cases of adenocarcinoma. In the UK adenocarcinomas account for 65% of cases. - Barrett’s oesophagus is a major risk factor for most cases of oesophageal adenocarcinoma. - In other regions of the world squamous cancer is more common and is linked to smoking, alcohol intake, diets rich in nitrosamines and achalasia. - Surveillance of Barrett’s is important as it imparts a 30 fold increase in cancer risk and if invasive malignancy is diagnosed early then survival may approach 85% at 5 years.

Diagnosis - Upper GI endoscopy is the first line test - Contrast swallow may be of benefit in classifying benign motility disorders but has no place in the assessment of tumours - Staging is initially undertaken with CT scanning of the chest, abdomen and pelvis. If overt metastatic disease is identified using this modality then further complex imaging is unnecessary - If CT does not show metastatic disease, then local stage may be more accurately assessed by use of . - Staging laparoscopy is performed to detect occult peritoneal disease. PET CT is performed in those with negative laparoscopy. Thoracoscopy is not routinely performed.

Treatment - Operable disease is best managed by surgical resection. The most standard procedure is an Ivor- Lewis type oesophagectomy. This procedure involves the mobilization of the stomach and division of the oesophageal hiatus. The abdomen is closed and a right sided thoracotomy performed. The stomach is brought into the chest and the oesophagus mobilized further. An intrathoracic oesophagogastric anastomosis is constructed.

- Alternative surgical strategies include a transhiatal resection (for distal lesions), a left thoraco-abdominal resection (difficult access due to thoracic aorta) and a total oesophagectomy (McKeown) with a cervical oesophagogastric anastomosis. The biggest surgical challenge is that of anastomotic leak, with an intrathoracic anastomosis this will result in mediastinitis. With high mortality.

- The McKeown technique has an intrinsically lower systemic insult in the event of anastmotic leakage.

- In addition to surgical resection many patients will be treated with adjuvant chemotherapy.

426. Which of the following genes is not implicated in the adenoma-carcinoma sequence in colorectal cancer? A. src

Prepared by Dr: Mohammed Musa Brema Idress – My best wishes Page 165

B. c-myc C. APC D. p53 E. K-ras

ANSWER IS A - Other genes involved are: MCC DCC c-yes bcl-2

Colorectal cancer - Annually about 150,000 new cases are diagnosed and 50,000 deaths from the disease - About 75% will have sporadic disease and 25% will have a family history - Colorectal tumours comprise a spectrum of disease ranging from adenomas through to polyp cancers and frank malignancy. - - Polyps may be categorized into: neoplastic polyps, adenomatous polyps and non neoplastic polyps. - - The majority of adenomas are polypoidal lesions, although flat lesions do occur and may prove to be dysplastic. - - Non-neoplastic polyps include hyperplastic, juvenile, hamartomatous, inflammatory, and lymphoid polyps, which have not generally been thought of as precursors of cancer. - Three characteristics of adenomas that correlate with malignant potential have been characterized. These include increased size, villous architecture and dysplasia. For this reason most polyps identified at colonoscopy should be removed.

- The transformation from polyp to cancer is described by the adenoma - carcinoma sequence and its principles should be appreciated. Essentially genetic changes accompany the transition from adenoma to carcinoma; key changes include APC, c-myc, K RAS mutations and p53 deletions.

427. A 63 year old lady is suspected as having sarcoidosis. She is sent to the general surgeons and a lymph node biopsy is performed. Which histological feature is most likely to be identified in a lymph node if sarcoid is present? A. Psammoma bodies B. Extensive necrosis C. Dense eosinophillic infiltrates D. Asteroid bodies E. None of the above

ANSWER IS D Asteroid bodies are often found in the granulomas of individuals with sarcoid. Unlike the granulomata associated with tuberculosis the granulomas of sarcoid are rarely associated with extensive necrosis.

Prepared by Dr: Mohammed Musa Brema Idress – My best wishes Page 166

428. Brown tumours of bone are associated with which of the following? A. Hyperthyroidism B. Hypothyroidism C. Hyperparathyroidism D. Hypoparathyroidism E. Osteopetrosis

ANSWER IS C Brown tumors are tumors of bone that arise in settings of excess osteoclast activity, such as hyperparathyroidism, and consist of fibrous tissue, woven bone and supporting vasculature, but no matrix. They are radiolucent on x-ray.

The osteoclasts consume the trabecular bone that osteoblasts lay down and this front of reparative bone deposition followed by additional resorption can expand beyond the usual shape of the bone, involving the periosteum thus causing bone pain. They appear brown because haemosiderin is deposited at the site.

Primary hyperparathyroidism - In exams primary hyperparathyroidism is stereotypically seen in elderly females with an unquenchable thirst and an inappropriately normal or raised parathyroid hormone level. It is most commonly due to a solitary adenoma Causes of primary hyperparathyroidism 1) 80%: solitary adenoma 2) 15%: hyperplasia 3) 4%: multiple adenoma 4) 1%: carcinoma Features 1) Bones, stones, abdominal groans and psychic moans' 2) Polydipsia, polyuria 3) Peptic ulceration/constipation/pancreatitis 4) Bone pain/fracture 5) Renal stones 6) Depression 7) Hypertension Associations - Hypertension - Multiple endocrine neoplasia: MEN I and II Investigations 1) Raised calcium, low phosphate 2) PTH may be raised or normal 3) Technetium-MIBI subtraction scan Treatment - Parathyroidectomy, if imaging suggests target gland then a focused approach may be used

429. A 73 year old man is recovering following an emergency Hartman’s procedure performed for an obstructing sigmoid cancer. The pathology report shows a

Prepared by Dr: Mohammed Musa Brema Idress – My best wishes Page 167 moderately differentiated adenocarcinoma that invades the muscularis propria, 3 of 15 lymph nodes are involved with metastatic disease. What is the correct stage for this? A. Astler Coller Stage B2 B. Dukes stage A C. Dukes stage B D. Dukes stage C E. Dukes stage D

ANSWER IS D - The involvement of lymph nodes makes this Dukes C. In the Astler Coller system the B and C subsets are split to B1 and B2 and C1 and C2. - Where C2 denotes involvement of the nodes in conjunction with penetration of the muscularis propria.

Duke’s classification Gives the extent of spread of colorectal cancer Dukes:- 1. A Tumour confined to the mucosa (90%) 2. Dukes B Tumour invading bowel wall (70%) 3. Dukes C Lymph node metastases (45%) 4. Dukes D Distant metastases (6%)(20% if resectable)

430. A 55 year old man presents with a soft, fluctuant lesion overlying his right scapula. The surgeon suspects the lesion may be a lipoma. Which of the following, if present, may be indicative of an alternative diagnosis? A. Located in superficial tissues B. Size greater than 5cm C. Presence of multiple similar lesions at other anatomical sites D. Increased mobility of the lesion E. Lobulated appearance during surgical excision

ANSWER IS B

431. A 55 year old man with dyspepsia undergoes an upper GI endoscopy. An irregular erythematous area is seen to protrude proximally from the gastro- oesophageal junction. Apart from specialized intestinal metaplasia, which of the following cell types should also be present for a diagnosis of Barrett’s oesophagus to be made? A. Goblet cell B. Neutrophil C. Lymphocytes D. Epithelial cells E. Macrophages

ANSWER IS A - Goblet cells need to be present for a diagnosis of Barrett's oesophagus to be made.

Prepared by Dr: Mohammed Musa Brema Idress – My best wishes Page 168

Barrett's oesophagus Barrett’s oesophagus is a condition characterized by the metaplastic transformation of squamous oesophageal epithelium to columnar gastric type epithelium. Three types of this metaplastic process are recognized; intestinal (high risk), cardiac and fundic. The latter two categories may cause difficulties in diagnosis.

The most concrete diagnosis can be made when endoscopic features of Barrett’s oesophagus are present together with a deep biopsy that demonstrates not just goblet cell metaplasia but also oesophageal glands.

Barrett's can be sub divided into short (<3cm) and long (>3cm). The length of the affected segment correlates strongly with the chances of identifying metaplasia. The overall prevalence of Barrett's oesophagus is difficult to determine but may be in the region of 1 in 20 and is identified in up to 12% of those undergoing endoscopy for reflux.

A proportion of patients with metaplasia will progress to dysplasia and for this reason individuals identified as having Barrett's should undergo endoscopic surveillance (every 2-5 years). Biopsies should be quadrantic and taken at 1-2cm intervals. Biopsies need to be adequate. Where mass lesions are present consideration should be given to endoscopic sub mucosal resection. Up to 40% of patients will be upstaged from high grade dysplasia to invasive malignancy with such techniques.

Treatment - Long term proton pump inhibitor - Consider pH and manometry studies in younger patients who may prefer to consider an anti reflux procedure - Regular endoscopic monitoring (more frequently if moderate dysplasia). With quadrantic biopsies every 2-3 cm - If severe dysplasia be very wary of small foci of cancer

432. Which of the following amino acids is present in all types of collagen? A. Alanine B. Aspartime C. Glycine D. Tyrosine E. Cystine

ANSWER IS C Collagen has a generic structure of Glycine- X- Y, where X and Y are variable sub units. The relatively small size of the glycine molecule enables collagen to form a tight helical structure.

Collagen - One of the major connective tissue proteins

Prepared by Dr: Mohammed Musa Brema Idress – My best wishes Page 169

- Composed of 3 polypeptide strands that are woven into a helix - Numerous hydrogen bonds exist within molecule to provide additional strength - Many sub types but commonest sub type is I (90% of bodily collagen) - Vitamin c is important in establishing cross links Collagen Diseases - Osteogenesis imperfecta - Ehlers Danlos Osteogenesis imperfecta: -8 Subtypes -Defect of type I collagen -In type I the collagen is normal quality but insufficient quantity -Type II- poor quantity and quality -Type III- Collagen poorly formed, normal quantity -Type IV- Sufficient quantity but poor quality Patients have bones which fracture easily, loose joint and multiple other defects depending upon which sub type they suffer from

Ehlers Danlos: -Multiple sub types -Abnormality of types 1 and 3 collagen -Patients have features of hypermobility. -Individuals are prone to joint dislocations and pelvic organ prolapse. In addition to many other diseases related to connective tissue defects.

433. An 8 year old boy presented with a painless swelling on the superotemporal aspect of his orbit. It was smooth on examination, produced no visual disturbances. Following excision it was found to be lined by squamous epithelium and hair follicles. Which of the following lesions most closely matches these findings? A. Dermoid cyst B. Desmoid tumour C. Lipoma D. Sebaceous cyst E. Schwannoma

ANSWER IS A Dermoid cysts are embryological remnants and may be lined by hair and squamous epithelium (like teratomas). They are often located in the midline and may be linked to deeper structures resulting in a dumbbell shape to the lesion. Complete excision is requires as they have a propensity to local recurrence if not excised.

Desmoid tumours are a different entity; they most commonly develop in ligaments and tendons. They are also referred to as aggressive fibromatosis and consist of fibroblast dense lesions (resembling scar tissue). They should be managed in a similar manner to soft tissue sarcomas.

434. A 55 year old man from Hong Kong presents with left sided otalgia and recurrent episodes of epistaxis. On examination his pharynx appears normal. Examination of his neck reveals left sided cervical lymphadenopathy. What is the most likely underlying diagnosis? A. Antrochoanal polyp B. Nasopharyngeal carcinoma C. Adenocarcinoma of the tonsil

Prepared by Dr: Mohammed Musa Brema Idress – My best wishes Page 170

D. Angiofibroma E. Globus syndrome

ANSWER IS B Given this man ethnic origin and presenting features a nasopharyngeal carcinoma is the most likely underlying diagnosis.

Nasopharyngeal carcinoma - Squamous cell carcinoma of the nasopharynx - Rare in most parts of the world, apart from individuals from Southern China - Associated with Epstein Barr virus infection Presenting features 1. Systemic and local Cervical lymphadenopathy 2. Otalgia 3. Unilateral serous otitis media 4. Nasal obstruction, discharge and/ or epistaxis 5. Cranial nerve palsies e.g. III-VI - Imaging Combined CT and MRI. Treatment - Radiotherapy is first line therapy.

435. Patients with suspected temporal arteritis are often sent for temporal artery biopsy. Which statement is true? A. Temporal artery biopsy is only diagnostic if there is visual loss B. Biopsy is typically taken from the non-symptomatic side to avoid the risk of blindness C. Pre-operative localization with duplex is mandatory D. Biopsies may be non diagnostic in over 50% of cases E. Biopsies are usually performed under general anaesthesia

ANSWER IS D Temporal artery biopsies are frequently non diagnostic. They should be taken from the symptomatic side and though not mandatory a duplex ultrasound is a helpful investigation, particularly if they mark the artery. It is usually performed under local anaesthetic. Temporal artery biopsy - Superficial temporal artery is a terminal branch of the external carotid artery

Main indication - Temporal arteritis American College of Rheumatology guidelines recommend a temporal artery biopsy if: 1) Age of onset older than 50 years 2) New-onset headache or localized head pain 3) Temporal artery tenderness to palpation or reduced pulsation 4) ESR > 50 mm/h

Prepared by Dr: Mohammed Musa Brema Idress – My best wishes Page 171

Histopathology - Vessel wall granulomatous arteritis with mononuclear cell infiltrates and giant cell formation Procedure 1) Position: supine, head 45 degrees 2) USS doppler to locate the superficial temporal artery or palpate 3) Local anaesthetic 4) Artery within temporoparietal fascia 5) Clamp and ligate the vessel 6) Cut 3-5cm 7) Ligate the remaining ends with absorbable suture 8) Close the skin Contraindication - Glucocorticoid therapy > 30 days Risks - Injury to facial or auriculotemporal nerve

436. Which of the following best describes the processes underpinning type IV hypersensitivity reactions? A. Deposition of immune complexes of IgG and antigen at the site of inflammation B. Deposition of IgA complexes at the site of inflammation C. Deposition of IgM and IgG complexes at the site of inflammation D. Degranulation of mast cells at the site of inflammation E. T cell mediated response at the site of inflammation

ANSWER IS E Hypersensitivity reactions: ACID - Type 1 --Anaphylactic - Type 2 --Cytotoxic - Type 3 --Immune complex - Type 4 --Delayed hypersensitivity T Cells are the mediators of type 4 hypersensitivity reactions which are characterized by the absence of immune complex deposition.

437. A 56 year old man is diagnosed with an abdominal aortic aneurysm and undergoes a CT scan to assess the size of the aorta. During the course of his investigations a lesion of the adrenal gland is identified. It measures 1.5 cm in diameter and the gland is otherwise normal. What is the most likely diagnosis? A. Adrenal gland metastasis B. Adrenal gland arterio-venous malformation C. Adrenal cyst D. Phaeochromocytoma E. Adrenal cortical adenoma

ANSWER IS E - 25% of all adrenal lesions >4cm in diameter are malignant

Prepared by Dr: Mohammed Musa Brema Idress – My best wishes Page 172

Incidentalomas of the adrenal gland are common and represent the most likely lesion in this scenario. Clearly the other lesions are all possibilities but are unlikely. Adrenal lesions- Incidental

Incidentaloma of the adrenal glands have become increasingly common as CT scanning of the abdomen is widely undertaken. Prevalences range from 1.5-9% in autopsy studies. Overall, 75% will be non functioning adenomas. However, a thorough diagnostic work up is required to exclude a more significant lesion. Investigations 1. Morning and midnight plasma cortisol measurements 2. Dexamethasone suppression test 3. 24 hour urinary cortisol excretion 4. 24 hour urinary excretion of catecholamines 5. Serum potassium, aldosteron and rennin levels

Management The risk of malignancy is related to the size of the lesion and 25% of all masses greater than 4cm will be malignant. Such lesions should usually be excised. Where a lesion is a suspected metastatic deposit a biopsy may be considered. Smaller, innocent lesions are usually followed up by serial CT scans at 6, 12 and 24 months.

438. A 22 year old man is undergoing an abdominal ultrasound scan as part of a series of investigations for abdominal pain. The radiologist notes that there is evidence of splenic atrophy. What is the most likely cause? A. Letterer-Siwe disease B. Coeliac disease C. Malaria D. Niemann-Pick disease E. Sarcoidosis

ANSWER IS B Splenic atrophy may occur in coeliac disease together with the appearance of Howell-Jolly bodies in erythrocytes. Letterer - Siwe disease is a form of Histiocytosis X in which macrophages proliferate.

Spleen - Embryology: derived from mesenchymal tissue - Shape: orange segment - Position: below 9th-12th ribs - Weight: 75-150g Relations - Superiorly - diaphragm - Anteriorly - gastric impression - Posteriorly - kidney - Inferiorly - colon - Hilum: tail of pancreas and splenic vessels (splenic artery divides here, branches pass to the white pulp transporting plasma)

Prepared by Dr: Mohammed Musa Brema Idress – My best wishes Page 173

- Forms apex of lesser sac (containing short gastric vessels) Contents - White pulp: immune function. Contains central trabecular artery. The germinal centers are supplied by arterioles called penicilliary radicles. - Red pulp: filters abnormal red blood cells Function 1- Filtration of abnormal blood cells and foreign bodies such as bacteria. 2- Immunity: IgM. Production of properdin, and tuftsin which help target fungi and bacteria for phagocytosis. 3- Haematopoiesis: up to 5th month gestation or in haematological disorders. 4- Pooling: storage of 40% platelets. 5- Iron reutilization 6- Storage red blood cells-animals, not humans. 7- Storage monocytes

Disorders of the spleen (massive splenomegaly) 1- Myelofibrosis 2- Chronic myeloid leukaemia 3- Visceral leishmaniasis (kala-azar) 4- Malaria 5- Gaucher's syndrome

Other causes (as above plus) 1- Portal hypertension e.g. secondary to cirrhosis 2- Lymphoproliferative disease e.g. CLL, Hodgkin's 3- Haemolytic anaemia 4- Infection: hepatitis, glandular fever 5- Infective endocarditis 6- Sickle-cell*, thalassaemia 7- Rheumatoid arthritis (Felty's syndrome) *the majority of adult’s patients with sickle-cell will have an atrophied spleen due to repeated infarction

439. Which statement relating to phaeochromocytoma is untrue? A. They are tumours of chromaffin cells in the adrenal medulla. B. They are bilateral in 10% of cases. C. When located in an extra adrenal location have a higher incidence of malignancy. D. May be associated with an elevated urinary VMA. E. Up to 40% may have a blood pressure within the normal range.

ANSWER IS E Normotension is seen in around 10% cases. The remainder show a degree of hypertension. Phaeochromocytoma and adrenal lesions: - Phaeochromocytoma Neuroendocrine tumour of the chromaffin cells of the adrenal medulla. Hypertension and hyperglycaemia are often found.

Prepared by Dr: Mohammed Musa Brema Idress – My best wishes Page 174

- 10% of cases are bilateral. - 10% occur in children. - 11% are malignant (higher when tumour is located outside the adrenal). - 10% will not be hypertensive. Familial cases are usually linked to the Multiple endocrine neoplasia syndromes (considered under its own heading). - Most tumours are unilateral (often right sided) and smaller than 10cm. Diagnosis A. Urine analysis of vanillymandelic acid (VMA) is often used (false positives may occur e.g. in patients eating vanilla ice cream!) B. Blood testing for plasma metanephrine levels. C. CT and MRI scanning are both used to localize the lesion. Treatment - Patients require medical therapy first. An irreversible alpha adrenoreceptor blocker should be given, although minority may prefer reversible blockade. Labetolol may be co-administered for cardiac chronotropic control. Isolated beta blockade should not be considered as it will lead to unopposed alpha activity.

These patients are often volume depleted and will often require moderate volumes of intra venous normal saline perioperatively.

Once medically optimized the phaeochromocytoma should be removed. Most adrenalectomies can now be performed using a laparoscopic approach. The adrenals are highly vascular structures and removal can be complicated by catastrophic haemorrhage in the hands of the inexperienced.

This is particularly true of right sided resections where the IVC is perilously close. Should the IVC be damaged a laparotomy will be necessary and the defect enclosed within a Satin sky style vascular clamp and the defect closed with prolene sutures. Attempting to interfere with the IVC using any instruments other than vascular clamps will result in vessel trauma and make a bad situation much worse.

Incidental adrenal lesions Adrenal lesions may be identified on CT scanning performed for other reasons.

Factors suggesting benign disease on CT include: 1. Size less than 3cm 2. Homogeneous texture 3. Lipid rich tissue 4. Thin wall to lesion All patients with incidental lesions should be managed jointly with an endocrinologist and full work up as described above. Patients with functioning lesions or those with adverse radiological features (Particularly size >3cm) should proceed to surgery.

Prepared by Dr: Mohammed Musa Brema Idress – My best wishes Page 175

440. A 69 year old man presents with a purple lesion on his forearm. It is excised and an 3 cm Merkel cell tumour is diagnosed. Which of the following statements relating to this diagnosis is false? A. He should undergo a sentinel lymph node biopsy. B. Lymphovascular invasion is typically seen histologically C. They are more common in immunosupressed patients D. Histologically they may resemble pyogenic granuloma E. They are associated with visceral metastasis

ANSWER IS D - Merkel cell tumours are rare cutaneous tumours. Histologically they consist of sheets and nodules of hyperchromatic epithelial cells, with high rates of mitosis and apoptosis. As such they are relatively easy to distinguish from pyogenic granuloma which has no features of malignancy and would not show lymphovascular invasion.

Merkel cell tumours of the skin - Rare but aggressive tumour. - Develops from intra epidermal Merkel cells. - Usually presents on elderly, sun damaged skin. The periorbital area is the commonest site. Histologically these tumours appear within the dermis and subcutis. The lesions consist of sheets and nodules of small hyperchromatic epithelial cells with high rates of mitosis and apoptosis. Lymphovascular invasion is commonly seen. - Pre-existing infection with Merkel Cell Polyomavirus is seen in 80% cases. Treatment - Surgical excision is first line. Margins of 1cm are required. Lesions >10mm in diameter should undergo sentinel lymph node biopsy. Adjuvant radiotherapy is often given to reduce the risk of local recurrence. Prognosis - With lymph node metastasis 5 year survival is 50% or less. - Small lesions without nodal spread are usually associated with a 5 year survival of 80%.

441. A 58 year old lady undergoes a screening mammogram and appearances are suspicious for ductal carcinoma in situ. A stereotactic core biopsy is performed. If ductal carcinoma in situ is to be diagnosed, which of the following pathological features must not be present? A. Nuclear pleomorphism B. Coarse chromatin C. Abnormal mitoses D. Angiogenesis E. Dysplastic cells infiltrating the suspensory ligaments of the breast

ANSWER IS E The presence of invasion is a hallmark of invasive disease and thus would not be a feature of DCIS. Angiogenesis may occur in association with high grade DCIS.

Prepared by Dr: Mohammed Musa Brema Idress – My best wishes Page 176

Histopathology of malignancy 1) Abnormal tissue architecture 2) Coarse chromatin 3) Invasion of basement membrane* 4) Abnormal mitoses 5) Angiogenesis 6) De-differentiation 7) Areas of necrosis 8) Nuclear pleomorphism *= Those features that distinguish invasive malignancy from in situ disease

442. Which of the following does not occur as a pathological response to extensive burns? A. Plasma leakage into interstitial space B. Absolute polycythaemia C. Increased haematocrit D. Keratinocyte migration during healing E. Cardiac output reduction by 50% in first 30 minutes

ANSWER IS B - Haemolysis is the main pathological response.

Burns pathology Extensive burns - Haemolysis due to damage of erythrocytes by heat and microangiopathy - Loss of capillary membrane integrity causing plasma leakage into interstitial space - Extravasation of fluids from the burn site causing hypovolaemic shock (up to 48h after injury)- decreased blood volume and increased haematocrit - Protein loss - Secondary infection e.g. Staphylococcus aureus - ARDS - Risk of Curling’s ulcer (acute peptic stress ulcers) - Danger of full thickness circumferential burns in an extremity as these may develop compartment syndrome

Healing - Superficial burns: keratinocytes migrate to form a new layer over the burn site - Full thickness burns: dermal scarring. Usually need keratinocytes from skin grafts to provide optimal coverage.

443. A 67 year old man is investigated for biliary colic and a 4.8 cm abdominal aortic aneurysm is identified. Which of the following statements relating to this condition is untrue? A. The wall will be composed of dense fibrous tissue only B. The majority are located inferior to the renal arteries C. They occur most often in current or former smokers

Prepared by Dr: Mohammed Musa Brema Idress – My best wishes Page 177

D. He should initially be managed by a process of active surveillance E. Aortoduodenal fistula is a recognized complication following repair.

ANSWER IS A They are true aneurysms and have all 3 layers of arterial wall.

Abdominal aorta aneurysm - Abdominal aortic aneurysms are a common problem in vascular surgery. - They may occur as either true or false aneurysm. With the former all 3 layers of the arterial wall are involved, in the latter only a single layer of fibrous tissue forms the aneurysm wall. - True abdominal aortic aneurysms have an approximate incidence of 0.06 per 1000 people. They are commonest in elderly men and for this reason the UK is now introducing the aneurysm screening program with the aim of performing an abdominal aortic ultrasound measurement in all men aged 65 years. Causes - Several different groups of patients suffer from aneurysmal disease. - The commonest group is those who suffer from standard arterial disease, i.e. those who are hypertensive, have diabetes and have been or are smokers. - Other patients such as those suffering from connective tissue diseases such as Marfan's may also develop aneurysms. In patients with abdominal aortic aneurysms the extracellular matrix becomes disrupted with a change in the balance of collagen and elastic fibers. Management - Most abdominal aortic aneurysms are an incidental finding. - Symptoms most often relate to rupture or impending rupture. - 20% rupture anteriorly into the peritoneal cavity. Very poor prognosis. - 80% rupture posteriorly into the retroperitoneal space - The risk of rupture is related to aneurysm size; only 2% of aneurysms measuring less than 4cm in diameter will rupture over a 5 year period. This contrasts with 75% of aneurysms measuring over 7cm in diameter. - This is well explained by La Places' law which relates size to transmural pressure. - For this reason most vascular surgeons will subject patients with an aneurysm size of 5cm or greater to CT scanning of the chest, abdomen and pelvis with the aim of delineating anatomy and planning treatment. Depending upon co- morbidities, surgery is generally offered once the aneurysm is between 5.5cm and 6cm. Indications for surgery A. Symptomatic aneurysms (80% annual mortality if untreated) B. Increasing size above 5.5cm if asymptomatic C. Rupture (100% mortality without surgery) Surgical procedures → Abdominal aortic aneurysm repair Procedure A. GA Invasive monitoring (A-line, CVP, catheter) B. Incision: Midline or transverse Bowel and distal duodenum mobilized to access aorta.

Prepared by Dr: Mohammed Musa Brema Idress – My best wishes Page 178

C. Aneurysm neck and base dissected out and prepared for cross clamp Systemic heparinization D. Cross clamp (distal first) Longitudinal aortotomy Atherectomy deal with back bleeding from lumbar vessels and inferior mesenteric artery Insert graft either tube or bifurcated depending upon anatomy Suture using Prolene (3/0 for proximal, distal anastomosis suture varies according to site) Clamps off: End tidal CO2 will rise owing to effects of reperfusion, at this point major risk of myocardial events. Haemostasis Closure of aneurysm sac to minimize risk of aorto-enteric fistula Closure: Loop 1 PDS or Prolene to abdominal wall Skin- surgeons preference Post operatively - ITU (Almost all) Greatest risk of complications following emergency repair Complications: Embolic- gut and foot infarcts Cardiac - owing to premorbid states, reperfusion injury and effects of cross clamp Wound problems Later risks related to graft- infection and aorto-enteric fistula Special groups - Supra renal AAA: These patients will require a supra renal clamp and this carries a far higher risk of complications and risk of renal failure. - Ruptured AAA: Preoperatively the management depends upon haemodynamic instability. In patients with symptoms of rupture (typical pain, haemodynamic compromise and risk factors) then ideally prompt laparotomy. In those with vague symptoms and haemodynamic stability the ideal test is CT scan to determine whether rupture has occurred or not. Most common rupture site is retroperitoneal 80%.

- These patients will tend to develop retroperitoneal haematoma. This can be disrupted if Bp is allowed to rise too high so aim for Bp 100mmHg. Operative details are similar to elective repair although surgery should be swift, blind rushing often makes the situation worse. Plunging vascular clamps blindly into a pool of blood at the aneurysm neck carries the risk of injury the vena cava that these patients do not withstand. Occasionally a supracoeliac clamp is needed to effect temporary control, although leaving this applied for more than 20 minutes tends to carry a dismal outcome.

EVAR Increasingly patients are now being offered endovascular aortic aneurysm repair. This is undertaken by surgeons and radiologists working jointly. The morphology of the aneurysm is important and not all are suitable. Here is a typical list of those features favoring a suitable aneurysm: A) Long neck B) Straight iliac vessels C) Healthy groin vessels Clearly few AAA patients possess the above and compromise has to be made. The use of fenestrated grafts can allow supra renal AAA to be treated.

Procedure: - GA Radiology or theatre Bilateral groin incisions Common femoral artery dissected out Heparinization Arteriotomy and insertion of guide wire Dilation of

Prepared by Dr: Mohammed Musa Brema Idress – My best wishes Page 179

arteriotomy Insertion of EVAR Device Once in satisfactory position it is released Arteriotomy closed once check angiogram shows good position and no endo- leak.

444. Which of the following statements in relation to the p53 tumour suppressor protein is false? A) It may induce necrosis of cells with non repairable DNA damage B) It is affected in Li Fraumeni syndrome C) It can induce DNA repair D) It can halt the cell cycle E) It may inhibit angiogenesis

ANSWER IS A When DNA cannot be repaired it will induce cellular apoptosis (not necrosis) Genetics and surgical disease - Some of the more commonly occurring genetic conditions occurring in surgical patients are presented here.

Li-Fraumeni Syndrome - Autosomal dominant - Consists of germline mutations to p53 tumour suppressor gene - High incidence of malignancies particularly sarcomas and leukaemias Diagnosed when: - Individual develops sarcoma under 45 years - First degree relative diagnosed with any cancer below age 45 years and another family member develops malignancy under 45 years or sarcoma at any age

BRCA 1 and 2 - Carried on chromosome 17 - Linked to developing breast cancer (60%) risk. - Associated risk of developing ovarian cancer (55% with BRCA 1 and 25% with BRCA 2).

Lynch Syndrome - Autosomal dominant - Develop colonic cancer and endometrial cancer at young age - 80% of affected individuals with get colonic and or endometrial cancer - High risk individuals may be identified using the Amsterdam criteria

Amsterdam criteria:- - Three or more family members with a confirmed diagnosis of colorectal cancer, one of whom is a first degree (parent, child, sibling) relative of the other two. - Two successive affected generations. One or more colon cancers diagnosed under age 50 years. - Familial adenomatous polyposis (FAP) has been excluded. Gardner’s syndrome - Autosomal dominant familial colorectal polyposis

Prepared by Dr: Mohammed Musa Brema Idress – My best wishes Page 180

- Multiple colonic polyps - Extra colonic diseases include: skull osteoma, thyroid cancer and epidermoid cysts - Desmoid tumours are seen in 15% - Mutation of APC gene located on chromosome 5 - Due to colonic polyps most patients will undergo colectomy to reduce risk of colorectal cancer - Now considered a variant of familial adenomatous polyposis coli

445. Which of the following cell types is most likely to be identified in the wall of a fistula in ano? A) Squamous cells B) Goblet cells C) Columnar cells D) Ciliated columnar cells E) None of the above

ANSWER IS A A fistula is an abnormal connection between two epithelial lined surfaces, in the case of a fistula in ano it will be lined by squamous cells.

Fistulas - A fistula is defined as an abnormal connection between two epithelial surfaces. - There are many types ranging from branchial fistulae in the neck to entero- cutaneous fistulae abdominally. - In general surgical practice the abdominal cavity generates the majority and most of these arise from diverticular disease and Crohn's. - As a general rule all fistulae will resolve spontaneously as long as there is no distal obstruction. This is particularly true of intestinal fistulae.

The four types of fistulae are: Enterocutaneous, these link the intestine to the skin. They may be high (>1L) or low output (<1L) depending upon source. Duodenal /jejunal fistulae will tend to produce high volume, electrolyte rich secretions which can lead to severe excoriation of the skin. Colo-cutaneous fistulae will tend to leak faeculent material. Both fistulae may result from the spontaneous rupture of an abscess cavity onto the skin (such as following perianal abscess drainage) or may occur as a result of iatrogenic input. In some cases it may even be surgically desirable e.g. mucous fistula following subtotal colectomy for colitis. - Suspect if there is excess fluid in the drain. Enteroenteric or Enterocolic This is a fistula that involves the large or small intestine. They may originate in a similar manner to enterocutaneous fistulae. A particular problem with this fistula type is that bacterial overgrowth may precipitate malabsorption syndromes. This may be particularly serious in inflammatory bowel disease. - Enterovaginal Aetiology as a above.

Prepared by Dr: Mohammed Musa Brema Idress – My best wishes Page 181

Enterovesicular This type of fistula goes to the bladder. These fistulas may result in frequent urinary tract infections, or the passage of gas from the urethra during urination.

Management - Some rules relating to fistula management: 1. They will heal provided there is no underlying inflammatory bowel disease and no distal obstruction, so conservative measures may be the best option 2. Where there is skin involvement, protect the overlying skin, often using a well fitted stoma bag- skin damage is difficult to treat 3. A high output fistula may be rendered more easily managed by the use of octreotide; this will tend to reduce the volume of pancreatic secretions. 4. Nutritional complications are common especially with high fistula (e.g. high jejunal or duodenal) these may necessitate the use of TPN to provide nutritional support together with the concomitant use of octreotide to reduce volume and protect skin. 5. When managing perianal fistulae surgeons should avoid probing the fistula where acute inflammation is present, this almost always worsens outcomes. 6. When perianal fistulae occur secondary to Crohn's disease the best management option is often to drain acute sepsis and maintain that drainage through the judicious use of setons whilst medical management is implemented. 7. Always attempt to delineate the fistula anatomy, for abscesses and fistulae that have an intra abdominal source the use of barium and CT studies should show a track. For perianal fistulae surgeons should recall Goodsall's rule in relation to internal and external openings.

446. A 22 year old man is referred to the surgical clinic. He has been complaining of varicose veins for many years. On examination he has extensive varicosities of the right leg, there are areas of marked port wine staining. The saphenofemoral junction is competent on doppler assessment. The most likely underlying diagnosis is: A. Deep vein thrombosis B. Klippel-Trenaunay syndrome C. Varicose veins due to sapheno-popliteal junction incompetence D. Sturge - Weber syndrome E. Angiosarcoma

ANSWER IS B Sturge - Weber syndrome is a an arteriovenous malformation affecting the face and CNS, the peripheral vessels are not affected. Simple varicose veins should not typically be associated with port wine staining, nor should a DVT or angiosarcoma.

Klippel-Trenaunay syndrome Klippel-Trenaunay-Weber syndrome generally affects a single extremity, although cases of multiple affected limbs have been reported. The leg is the most common site followed by the arms, the trunk, and rarely the head and the neck

Prepared by Dr: Mohammed Musa Brema Idress – My best wishes Page 182

Signs and symptoms The birth defect is diagnosed by the presence of a combination of these symptoms: - One or more distinctive port-wine stains with sharp borders - Varicose veins - Hypertrophy of bony and soft tissues that may lead to local gigantism or shrinking. - An improperly developed lymphatic system In some cases, port-wine stains (capillary port wine type) may be absent. Such cases are very rare and may be classified as "atypical Klippel-Trenaunay syndrome".

KTS can either affect blood vessels, lymph vessels, or both. The condition most commonly presents with a mixture of the two. Those with venous involvement experience increased pain and complications.

447. A 68 year old man presents with an ulcerated lesion on his right cheek. It is excised and on histological assessment a squamous cell carcinoma is diagnosed. It measures 25mm in diameter and is 4mm deep. Which of the following statements relating to this condition is false? A. In this particular case margins of at least 6mm are required B. Use of cryosurgery to treat this patient lesion would have been unsafe C. Use of radiotherapy to treat this lesion would have been unsafe D. This patient local recurrence rate may approach 15% E. The disease usually spreads via lymphatics

ANSWER IS C Poor prognostic factors in SCC: - Size >20mm (local recurrence rate of up to 15%) - Depth greater than 4mm (risk of metastasis up to 30%) This man has an SCC with significant risk of metastasis. Although cryotherapy may be used to treat SCC it would be most unsafe in this setting as the lesion extends deeply. However, radiotherapy is a safe treatment modality for SCC and may be used in selected cases. It is unwise to use radiotherapy in areas prone to radio-necrosis e.g. the nose.

Squamous cell carcinoma of the skin - Second most common skin malignancy - Derived from epidermal keratinocytes - Commonest in fair skinned individuals in sun exposed sites - May occur in perianal and genital skin especially in association with Human Papilloma Virus 16 and 18 infections. Groups at high risk Renal transplant and on immunosuppression Individuals with HIV those who have received psoralen UVA therapy Chronic wounds (Marjolins ulcer) Xeroderma pigmentosum Oculocutaneous albinism

Prognosis Good Prognosis Poor prognosis Well differentiated tumours Poorly differentiated tumours <20mm diameter >20mm in diameter <2mm deep >4mm deep No associated diseases Immunosupression for whatever reason

Prepared by Dr: Mohammed Musa Brema Idress – My best wishes Page 183

Treatment Surgical excision with 4mm margins if lesion <20mm in diameter. If tumour >20mm then margins should be 6mm.

448. A 23 year old man presents with weight loss fatigue and lymphadenopathy. He is diagnosed with tuberculosis. Which of the following processes most closely matches the underlying pathological process? A. Type 1 hypersensitivity reaction B. Type 2 hypersensitivity reaction C. Type 3 hypersensitivity reaction D. Type 4 hypersensitivity reaction E. None of the above

ANSWER IS D Granulomas (which occur in tuberculosis) are a feature of Type 4 hypersensitivity reactions.

449. A 73 year old man undergoes an emergency amputation for severe lower limb sepsis and gangrene. Post operatively he develops disseminated intravascular coagulation. Which of the following clotting factors will be most rapidly consumed in this process? A. Factor V and VIII B. Factor I C. Factor I and III D. Factor III and VII E. Factor VI and VIII

ANSWER IS A D-I-S-S-E-M-I-N-A-T-E-D D-Dx: D dimer I-Immune complexes S-Snakebite, shock, heatstroke S-SLE E- Eclampsia, HELLP syndrome M-Massive tissue damage I-Infections: viral and bacterial N-Neoplasms A-Acute promyelocytic leukemia T-Tumor products: Tissue Factor (TF) and TF-like factors released by carcinomas of pancreas, prostate, lung, colon, stomach E-Endotoxins (bacterial), D-Dead fetus (retained) - DIC Will tend to consume factors five and eight initially (and platelets). Disseminated intravascular coagulation - Simultaneous coagulation and haemorrhage caused by initially formation of thrombi which consume clotting factors (factors 5,8) and platelets, ultimately leading to bleeding Causes include: 1. Infection 2. Malignancy 3. Trauma e.g. major surgery, burns, shock, dissecting aortic aneurysm 4. Liver disease 5. Obstetric complications

Prepared by Dr: Mohammed Musa Brema Idress – My best wishes Page 184

Key points - Clinically bleeding is usually a dominant feature, bruising, ischaemia and organ failure - Blood tests: prolonged clotting times, thrombocytopenia, decreased fibrinogen, increased fibrinogen degradation products - Treat the underlying cause and supportive management

450. A 52 year old lady is referred to the breast clinic with symptoms of nipple discharge. The discharge is usually thick and green. Which of the following statements relating to the most likely underlying diagnosis is untrue? A. The majority of patients will be smokers B. Typically produces blood stained nipple discharge C. It is not associated with increased risk of breast cancer D. May result in development of mammary duct fistula E. May require total duct excision (Had fields operation) if it fails to resolve

ANSWER IS B Blood stained nipple discharge should always be investigated. Nipple fluid cytology is generally unhelpful.

Discharge of this type of material is most likely to be due to duct ectasia. Green or brown discharge is most common. Blood stained discharge should raise concern of intraductal papilloma or cancer.

Nipple discharge - Causes of nipple discharge Physiological During breast feeding Galactorrhoea Commonest cause may be response to emotional events, drugs such as histamine receptor anatagonists are also implicated Hyperprolactinaemia - Commonest type of pituitary tumour - Microadenomas <1cm in diameter - Macroadenomas >1cm in diameter - Pressure on optic chiasm may cause bitemporal hemianopia Mammary duct ectasia - Dilatation breast ducts. - Most common in menopausal women - Discharge typically thick and green in colour - Most common in smokers Carcinoma - Often blood stained - May be underlying mass or axillary lymphadenopathy Intraductal papilloma - Commoner in younger patients - May cause blood stained discharge - There is usually no palpable lump Assessment of patients - Examine breast and determine whether there is mass lesion present - All mass lesions should undergo Triple assessment.

Prepared by Dr: Mohammed Musa Brema Idress – My best wishes Page 185

Reporting of investigations where a mass lesion is suspected or investigations are requested these are prefixed using a system that denotes the investigation type e.g. M for mammography, followed by a numerical code as shown below: 1- No abnormality 2- Abnormality with benign features 3- Indeterminate probably benign 4- Indeterminate probably malignant 5- Malignant Management of non malignant nipple discharge - Exclude endocrine disease - Nipple cytology unhelpful - Smoking cessation advice for duct ectasia - For duct ectasia with severe symptoms, total duct excision may be warranted

451. Which of the following statements relating to gas gangrene is untrue? A. There is necrosis with putrefaction B. The causative pathogens may be detected on normal perineal skin C. Treatment with low dose penicillin is indicated D. Hyperbaric oxygen may be beneficial E. Clostridium perfringens is a recognized cause

ANSWER IS C Rapid surgery and high dose antibiotics are indicated in the treatment of gas gangrene. Meleney's Gangrene and Necrotizing Fasciitis Necrotizing fasciitis: - Advancing soft tissue infection associated with fascial necrosis - Uncommon, but can be fatal - In many cases there is underlying background immunosuppression e.g. Diabetes - Caused by polymicrobial flora (aerobic and anaerobic) and MRSA is seen increasingly in cases of necrotizing fasciitis - Streptococcus is the commonest organism in isolated pathogen infection (15%) Meleneys gangrene: - Meleneys is a similar principle but the infection is more superficially sited than necrotizing fasciitis and often confined to the trunk

Fournier gangrene: - Necrotizing fasciitis affecting the perineum - Polymicrobial with E-coli and Bacteroides acting in synergy Clinical features 1) Fever 2) Pain 3) Cellulitis 4) Oedema 5) Induration 6) Numbness

Prepared by Dr: Mohammed Musa Brema Idress – My best wishes Page 186

Late findings - Purple/black skin discolouration - Blistering Haemorrhagic bullae - Crepitus - Dirty dish water fluid discharge - Septic shock - Diagnosis is mainly clinical Management - Radical surgical debridement forms the cornerstone of management - Sterile dressing is used to dress the wound - Reconstructive surgery is considered once the infection is completely treated

452. A 30 year old man presents with , a laparotomy is performed, at operation the abdomen is filled with a large amount of gelatinous fluid. What is the most likely underlying diagnosis? A. Infection with entamoeba histolytica B. Pseudomxyoma peritonei C. Metastatic colonic cancer D. Chylous ascites E. None of the above

ANSWER IS B Pseudomyxoma is associated with the deposition of large amounts of gelatinous material. The appendix is the commonest organ or origin. Pseudomyxoma Peritonei - Rare mucinous tumour - Most commonly arising from the appendix (other abdominal viscera are also recognized as primary sites) - Incidence of 1-2/1,000,000 per year - The disease is characterized by the accumulation of large amounts of mucinous material in the abdominal cavity Treatment - Is usually surgical and consists of cytoreductive surgery (and often peritonectomy c.f Sugar baker procedure) combined with intraperitoneal chemotherapy with mitomycin C.

Survival is related to the quality of primary treatment and in Sugar bakers own centre 5 year survival rates of 75% have been quoted. Patients with disseminated intraperitoneal malignancy from another source fare far worse. In selected patients a second look laparotomy is advocated and some practice this routinely.

453. A 30 year old man is suspected of having appendicitis. At operation an inflamed Meckels diverticulum is found. Which of the following vessels is responsible for the blood supply to a Meckels diverticulum? A. Right colic artery B. Vitelline artery C. Appendicular artery

Prepared by Dr: Mohammed Musa Brema Idress – My best wishes Page 187

D. Internal iliac artery E. External iliac artery

ANSWER IS B The vitelline arteries supply a Meckels these are usually derived from the ileal arcades.

Meckel's diverticulum - Congenital abnormality resulting in incomplete obliteration of the vitello- intestinal duct - Normally, in the foetus, there is an attachment between the vitello-intestinal duct and the yolk sac. This disappears at 6 weeks gestation. - The tip is free in majority of cases. - Associated with enterocystomas, umbilical sinuses, and omphaloileal fistulas. - Arterial supply: omphalomesenteric artery. - 2% of population, 2 inches long, 2 feet from the ileocaecal valve. - Typically lined by ileal mucosa but ectopic gastric mucosa can occur, with the risk of peptic ulceration. Pancreatic and jejunal mucosa can also occur. Clinical - Normally asymptomatic and an incidental finding. - Complications are the result of obstruction, ectopic tissue, or inflammation. - Removal if narrow neck or symptomatic. Options are between wedge excision or formal small bowel resection and anastomosis.

454. Which of the following associations are incorrect? A. Afro-Caribbean skin and keloid scarring B. Extensive third degree burns and wound contraction C. Chemotherapy and dehiscence of healed wounds D. Poor healing at the site of previous radiotherapy E. Zinc deficiency and delayed healing

ANSWER IS C

455. A 23 year old man fractures his right tibia in a sporting accident. At which point in the healing process is fracture callus most likely to be visible radiologically? A. 1 day B. 7 days C. 8 weeks D. 6 weeks E. 3 weeks

ANSWER IS E Fracture callus is composed of fibroblasts and chondroblasts and the synthesis of fibrocartilage. It is typically visible on radiographs approximately 3 weeks following injury. If delayed then there may be risk of non union.

Prepared by Dr: Mohammed Musa Brema Idress – My best wishes Page 188

Fracture healing Bone fracture - Bleeding vessels in the bone and periosteum - Clot and haematoma formation - The clot organizes over a week (improved structure and collagen) - The periosteum contains osteoblasts which produce new bone - Mesenchymal cells produce cartilage (fibrocartilage and hyaline cartilage) in the soft tissue around the fracture - Connective tissue + hyaline cartilage = callus - As the new bone approaches the new cartilage, endochondral ossification occurs to bridge the gap - Trabecular bone forms - Trabecular bone is resorbed by osteoclasts and replaced with compact bone

Factors affecting Fracture Healing 1) Age 2) Malnutrition 3) Bone disorders: osteoporosis 4) Systemic disorders: diabetes, Marfan's syndrome and Ehlers-Danlos syndrome cause abnormal musculoskeletal healing. 5) Drugs: steroids, non steroidal anti inflammatory agents. 6) Type of bone: Cancellous (spongy) bone fractures are usually more stable, involve greater surface areas, and have a better blood supply than cortical (compact) bone fractures. 7) Degree of Trauma: The more extensive the injury to bone and surrounding soft tissue, the poorer the outcome. 8) Vascular Injury: Especially the femoral head, talus, and scaphoid bones. 9) Degree of Immobilization 10) Intra-articular Fractures: These fractures communicate with synovial fluid, which contains collagenases that retard bone healing. 11) Separation of Bone Ends: Normal apposition of fracture fragments is needed for union to occur. Inadequate reduction, excessive traction, or interposition of soft tissue will prevent healing. 12) Infection

456. Of the options below, which does not cause lymphadenopathy? A. Kawasaki disease B. Systemic Lupus Erthematosus C. Phenytoin D. Hydrallazine E. Amiodarone

ANSWER IS E Lymphadenopathy - Lymphadenopathy in the neck, axillae, groins and abdomen - Need to note: solitary/multiple, defined/indistinct, hard/rubbery/soft, tender/painless Causes of lymphadenopathy Mnemonic: Hodgkin’s disease H aematological: Hodgkin’s lymphoma, NHL, Leukaemia O ncological: metastases D ermatopathic lympadenitis G aucher's disease K awasaki disease I nfections: TB,

Prepared by Dr: Mohammed Musa Brema Idress – My best wishes Page 189 glandular fever, Syphilis, N iemann Pick disease, S erum sickness, D rug reaction (phenytoin), I mmunological (SLE), S arcoidosis, E ndocrinological (Hyperthyroidism) A ngioimmunoplastic, lymphadenopathy, S LE, E osinophilic granulomatosis

457. A 23 year old man is reviewed on the ward 10 days following a laparotomy. The wound is inspected and is healing well. Which of the following processes is least likely to be occurring in the wound at this stage? A. Angiogenesis B. Synthesis of collagen C. Necrosis of fibroblasts D. Secretion of matrix metalloproteinases by fibroblasts E. Proliferation of fibroblasts

ANSWER IS C Vasculogenesis vs Angiogenesis = Vascu is new. Angi is pre Vasculogenesis is new vessels developing in situ from existing mesenchyme. Angiogenesis is vessels develop from sprouting off pre-existing arteries. Fibroblasts are an important cell type in healing wounds. They typically proliferate in the early phases of wound healing. They release matrix metalloproteinases and these facilitate in the remodeling of the matrix within the healing wound. Necrosis in a healing wound would be unusual as wounds will tend to show clinical evidence of angiogenesis by this time.

458. A 25 year old women presents with a slowly enlarging mass on the side of the face. Clinical examination demonstrates that the mass is located in the tail of the parotid gland. There is no evidence of facial nerve involvement. What is the most likely cause? A. Sialolithiasis B. Adenocarcinoma C. Warthins tumour D. Oncocytoma E. Pleomorphic adenoma

ANSWER IS E Pleomorphic adenomas are the commonest tumours of the parotid gland and are often slow growing, smooth and mobile. Warthins tumours are typically found in elderly males and are composed of multiple cysts and solid components consisting of lymphoid tissue. Warthins tumours are most often found in the tail of the parotid gland, but not in 25 year old females, where a pleomorphic adenoma remains the most likely lesion.

Parotid gland clinical Causes of bilateral parotid enlargement 1. Mumps: Associated with meningoencephalitis, pancreatitis, orchitis, or deafness 2. Parotitis 3. Sialectasis - especially if related to eating

Prepared by Dr: Mohammed Musa Brema Idress – My best wishes Page 190

4. Sjogren's syndrome: dry eyes or mouth, connective tissue disease 5. Sarcoidosis 6. Tuberculosis 7. Alcoholism 8. Myxoedema 9. Cushing's disease 10. Diabetes/insulin resistance 11. Liver cirrhosis 12. Gout 13. Bulimia nervosa 14. Drugs 15. Severe dehydration 16. Malnutrition Causes of unilateral parotid enlargement - Salivary calculus - Tumour Parotid gland tumours - Pleomorphic adenomas are the most common. - Incisional biopsy of parotid masses is not recommended, so superficial parotidectomy is the usual procedure of choice. - Signs of facial nerve palsy and a parotid mass should raise suspicion of malignancy. - Warthins tumours are relatively benign lesions that are slow growing and occur most commonly in elderly male smokers. - Adenoid cystic carcinoma have a tendency for perineural invasion.

459. Beta-naphthalamine is associated with which of the following malignancies? A. Lung cancer B. Bowel cancer C. Bladder cancer D. Liver cancer E. Renal cancer

ANSWER IS C Beta-naphthalamine is used in the rubber industry. The following factors are associated with the development of bladder cancer: 1. Smoking 2. Occupational: aniline dyes used in printing and textile industry, rubber manufacture 3. Schistosomiasis 4. Drugs: cyclophosphamide Occupational cancers - Occupational cancers accounted for 5.3% cancer deaths in 2005. In men the main cancers include: 1. Mesothelioma 2. Bladder cancer 3. Non melanoma skin cancer

Prepared by Dr: Mohammed Musa Brema Idress – My best wishes Page 191

4. Lung cancer 5. Sinonasal cancer Occupations with high levels of occupational tumours include: 1. Construction industry 2. Working with coal tar and pitch 3. Mining 4. Metal workers 5. Working with asbestos (accounts for 98% of all mesotheliomas) 6. Working in rubber industry Shift work has been linked to breast cancer in women (Health and safety executive report RR595).

The latency between exposure and disease is typically 15 years for solid tumours and 20 for leukaemia.

Many occupational cancers are otherwise rare. For example sino nasal cancer is an uncommon tumour, 50% will be SCC. They are linked to conditions such as wood dust exposure and unlike lung cancer is not strongly linked to cigarette smoking. Another typical occupational tumour is angiosarcoma of the liver which is linked to working with vinyl chloride. Again in the non occupational context this is an extremely rare sporadic tumour.

460. A 56 year old man with Wilson’s disease presents with right upper quadrant discomfort. An ultrasound scan is performed and this demonstrates a mass lesion in the right lobe of the liver. What is the most appropriate method of establishing the underlying diagnosis? A. PET CT scan B. Ultrasound guided biopsy C. Measurement of serum alpha fetoprotein D. MRI scan of the liver E. CT scan of the liver

ANSWER IS C High AFP + chronic liver inflammation = Hepatocellular carcinoma. - This is likely to be a hepatocellulcar carcinoma. Diagnosis is usually made by AFP measurement (with further imaging depending on the result). Biopsy should not be performed as it may seed the tumour. Chronic liver diseases such as Wilson’s disease (Hepato-lenticular degeneration) increase the risk.

Liver tumours Primary liver tumours The most common primary tumours are cholangiocarcinoma and hepatocellular carcinoma. Overall metastatic disease accounts for 95% of all liver malignancies making the primary liver tumours comparatively rare. Primary liver tumours include: 1. Cholangiocarcinoma 2. Hepatocellular carcinoma 3. Hepatoblastoma

Prepared by Dr: Mohammed Musa Brema Idress – My best wishes Page 192

4. Sarcomas (Rare) 5. Lymphomas 6. Carcinoids (most often secondary although primary may occur)

Hepatocellular carcinoma: These account for the bulk of primary liver tumours (75% cases). Its worldwide incidence reflects its propensity to occur on a background of chronic inflammatory activity.

Most cases arise in cirrhotic livers or those with chronic hepatitis B infection, especially where viral replication is actively occurring. In the UK it accounts for less than 5% of all cancers, although in parts of Asia its incidence is 100 per 100,000. The majority of patients (80%) present with existing liver cirrhosis, with a mass discovered on screening ultrasound.

Diagnosis 1. CT/ MRI (usually both) are the imaging modalities of choice 2. Alph-fetoprotein is elevated in almost all cases 3. Biopsy should be avoided as it seeds tumours cells through a resection plane. 4. In cases of diagnostic doubt serial CT and aFP measurements are the preferred strategy.

Treatment - Patients should be staged with liver MRI and chest, abdomen and pelvic CT scan. - The testis should be examined in males (testicular tumours may cause raised AFP). PET CT may be used to identify occult nodal disease. - Surgical resection is the mainstay of treatment in operable cases. In patients with a small primary tumour in a cirrhotic liver whose primary disease process is controlled, consideration may be given to primary whole liver resection and transplantation. - Liver resections are an option but since most cases occur in an already diseased liver the operative risks and post-operative hepatic dysfunction are far greater than is seen following metastectomy. - These tumours are not particularly chemo or radiosensitive however, both may be used in a palliative setting. Tumour ablation is a more popular strategy. Survival Poor, overall survival is 15% at 5 years.

Cholangiocarcinoma: This is the second most common type of primary liver malignancy. As its name suggests these tumours arise in the bile ducts. Up to 80% of tumours arise in the extra hepatic biliary tree. Most patients present with jaundice and by this stage the majority will have disease that is not resectable. Primary scelerosing cholangitis is the main risk factor. In deprived countries typhoid and liver flukes are also major risk factors.

Diagnosis 1. Patients will typically have an obstructive picture on liver function tests. 2. CA 19-9, CEA and CA 125 are often elevated 3. CT/ MRI and MRCP are the imaging methods of choice.

Prepared by Dr: Mohammed Musa Brema Idress – My best wishes Page 193

Treatment - Surgical resection offers the best chance of cure. Local invasion of perihilar tumours is a particular problem and this coupled with lobar atrophy will often contra indicate surgical resection. - Palliation of jaundice is important, although metallic stents should be avoided in those considered for resection. Survival is poor, approximately 15% 5 year survival

461. A 55 year old man has suffered from reflux oesophagitis for many years. During a recent endoscopy a biopsy is taken from the distal oesophagus. The histopathology report indicates that cells are identified with features of coarse chromatin and abnormal mitoses. The cells are confined to the superficial epithelial layer only. Which of the following accounts for this process? A. Metaplasia B. Apoptosis C. Autoimmune oesophagitis D. Dysplasia E. Infection with Helicobacter pylori

ANSWER IS D - Dysplasia = pre cancerous Dysplasia tends to develop as a result of prolonged stimulation by precipitants. Removal of these precipitants may possibly reverse these changes. Replacement of differentiated cells with another cell type describes metaplasia rather than dysplasia. The absence of invasion distinguishes this from malignancy.

Dysplasia - Premalignant condition - Disordered growth and differentiation of calls - Alteration in size, shape, and organization of cells - Features increased abnormal cell growth (increased number of mitoses/abnormal mitoses and cellular differentiation) - Underlying connective tissue is not invaded - Causes include smoking, Helicobacter pylori, Human papilloma virus - Main differences to metaplasia is that dysplasia is considered to be part of carcinogenesis (pre cancerous) and is associated with a delay in maturation of cells rather than differentiated cells replacing one another. - The absence of invasion differentiates dysplasia from invasive malignancy. - Severe dysplasia with foci of invasion are well recognized

462. Which one of the following is not a predisposition for developing osteoporosis? A. Obesity B. Long term heparin therapy C. Gastrectomy D. Osteogenesis imperfecta E. Diabetes

Prepared by Dr: Mohammed Musa Brema Idress – My best wishes Page 194

ANSWER IS A Low body weight is a risk factor for osteoporosis.

Osteoporosis - Risk factors 1) Family history 2) Female sex 3) Increasing age 4) Deficient diet 5) Sedentary lifestyle 6) Smoking 7) Premature menopause 8) Low body weight 9) Caucasians and Asians

Diseases which predispose: - Endocrine: glucocorticoid excess (e.g. Cushing's, steroid therapy), hyperthyroidism, hypogonadism (e.g. Turner's, testosterone deficiency), growth hormone deficiency, hyperparathyroidism, diabetes mellitus - Multiple myeloma, lymphoma - Gastrointestinal problems: inflammatory bowel disease, malabsorption (e.g. Coeliac), gastrectomy, liver disease - Rheumatoid arthritis - Long term heparin therapy - Chronic renal failure - Osteogenesis imperfecta, homocystinuria

463. A 63 year old man has a history of claudication that has been present for many years. He is recently evaluated in the clinic and a duplex scan shows that he has an 85% stenosis of the superficial femoral artery. Two weeks later he presents with a 1 hour history of severe pain in his leg. On examination he has absent pulses in the affected limb and it is much cooler than the contra-lateral limb. Which process best accounts for this presentation? A. Thrombosis B. Embolus C. Atheroma growth D. Sub intimal dissection E. Anaemia

ANSWER IS A In an existing lesion a complication such as thrombosis is more likely than embolus. These patients should receive heparin and imaging with duplex scanning. Whilst an early surgical bypass or intraarterial thrombolysis may be indicated, an embolectomy should not generally be performed as the lesion is not an embolus and the operation therefore ineffective.

Prepared by Dr: Mohammed Musa Brema Idress – My best wishes Page 195

Claudication Claudication is a condition in which patients develop pain in a limb during periods of exercise. The underlying disorder is usually that of arterial insufficiency. Atheroma develops in the arterial wall and once this occludes >50-75% of the lumenal diameter the supply to metabolizing tissues distally may become compromised.

The typical claudicant complains of calf pain that is worse on exercise and relieved by rest. This typical description assumes that the SFA is the site of disease, more proximal disease may present with other symptoms such as buttock claudication and impotence.

The history is usually a progressive one, patients presenting as an emergency with severe pain, diminished sensation, pallor and absent pulses have critical limb ischaemia. This may complicate claudication and usually indicates a plaque related complication, such as thrombosis.

Risk factors: risk factors for claudication include smoking, diabetes and hyperlipidaemia.

Diagnosis: diagnostic work -up includes measurement of ankle- brachial pressure indices, duplex scanning and formal angiography.

Treatment: those with long claudication distances, no ulceration or gangrene may be managed conservatively.

Patients with rest pain, ulceration or gangrene will almost always require intervention. All patients should receive an antiplatelet agent and a statin unless there are compelling contra-indications.

464. The following are true of carcinoid tumours except: A. When present in the appendix tip and measure less than 2 cm have an excellent prognosis B. Even when metastatic disease is present it tends to follow a protracted course C. When present in the appendix body tend to present with carcinoid syndrome even when liver metastases are not present D. May be imaged using 5 HIAA radionucleotide scanning E. Advanced appendiceal carcinoids may require right hemicolectomy

ANSWER IS C Rule of thirds: - 1/3 multiple, 1/3 small bowel, 1/3 metastasize, 1/3 second tumour - Liver metastases are necessary for the presence of carcinoid syndrome.

Carcinoid syndrome - Carcinoid tumours secrete serotonin

Prepared by Dr: Mohammed Musa Brema Idress – My best wishes Page 196

- Originate in neuroendocrine cells mainly in the intestine (midgut-distal ileum/appendix) - Can occur in the rectum, bronchi - Hormonal symptoms mainly occur when disease spreads outside the bowel

Clinical features - Onset: years - Flushing face - Palpitations - Tricuspid stenosis causing dyspnoea - Asthma - Severe diarrhoea (secretory, persists despite fasting)

Investigation - 5-HIAA in a 24-hour urine collection - Scintigraphy - CT scan

Treatment - Octreotide - Surgical removal

465. During a difficult femoro-popliteal bypass operation the surgeon inadvertently places a clamp across the femoral nerve. It remains there for most of the procedure. At the end of the operation the nerve is inspected, it is in continuity but has evidence of being crushed. Which of the following is most likely to occur over the following months? A. Wallerian degeneration B. Rapid restoration of neuronal function because the axon itself is intact C. Normal but delayed neuronal transmission due to disruption of the myelin D. Absence of neuroma formation E. None of the above

ANSWER IS A A neuronal injury such as this will result in Wallerian degeneration even though the nerve remains in continuity. Neuromas may well form.

Nerve injury There are 3 types of nerve injury: Neuropraxia - Nerve intact but electrical conduction is affected - Full recovery - Autonomic function preserved - Wallerian degeneration does not occur Axonotmesis - Axon is damaged and the myelin sheath is preserved. The connective tissue framework is not affected. - Wallerian degeneration occurs. Neurotmesis - Disruption of the axon, myelin sheath and surrounding connective tissue. - Wallerian degeneration occurs. Wallerian Degeneration - Axonal degeneration distal to the site of injury.

Prepared by Dr: Mohammed Musa Brema Idress – My best wishes Page 197

- Typically begins 24-36 hours following injury. - Axons are excitable prior to degeneration occurring. - Myelin sheath degenerates and is phagocytosed by tissue macrophages.

Nerve repair - Neuronal repair may only occur physiologically where nerves are in direct contact. Where a large defect is present the process of nerve regeneration is hampered and may not occur at all or result in the formation of a neuroma. Where nerve regrowth occur, it typically occurs at a rate of 1mm per day.

466. A 38 year old lady who smokes heavily presents with recurrent episodes of infection in the right breast. On examination she has an indurated area at the lateral aspect of the nipple areaolar complex. Imaging shows no mass lesions. What is the most likely diagnosis? A. Duct ectasia B. Periductal mastitis C. Paget’s disease of the nipple D. Mondors disease of the breast E. Radial scar

ANSWER IS B Periductal mastitis is common in smokers and may present with recurrent infections. Treatment is with co-amoxyclav. Mondors disease of the breast is a localized thrombophlebitis of a breast vein.

Duct ectasia Duct ectasia is a dilatation and shortening of the terminal breast ducts within 3cm of the nipple. It is common and the incidence increases with age. It typically presents with nipple retraction and occasionally creamy nipple discharge. It may be confused with periductal mastitis, which presents in younger women, the vast majority of which are smokers. Periductal mastitis typically presents with periareolar or sub areolar infections and may be recurrent. Patients with troublesome nipple discharge may be treated by microdochectomy (if young) or total duct excision (if older).

467. You review a 42-year-old woman 8 months following a renal transplant for focal segmental glomerulosclerosis. She is on a combination of tacrolimus, mycophenolate, and prednisolone. She has now presented with a five day history of feeling generally unwell with jaundice, fatigue and arthralgia. On examination she has jaundice, widespread lymphadenopathy and hepatomegaly. What is the most likely diagnosis? A. Hepatitis C B. Epstein-Barr virus C. HIV D. Hepatitis B E. Cytomegalovirus

ANSWER IS B

Prepared by Dr: Mohammed Musa Brema Idress – My best wishes Page 198

Post transplant complications:- - CMV: 4 weeks to 6 months post transplant - EBV: post transplant lymphoproliferative disease. > 6 months post transplant Post transplant lymphoproliferative disorder is most commonly associated with Epstein-Barr virus. It typically occurs 6 months post transplant and is associated with high dose immunosuppressant therapy. Remember cytomegalovirus presents within the first 4 weeks to 6 months post transplant.

Renal transplant: HLA typing and graft failure The human leucocyte antigen (HLA) system is the name given to the major histocompatibility complex (MHC) in humans. It is coded for on chromosome 6.

Some basic points on the HLA system - Class 1 antigens include A, B and C. Class 2 antigens include DP,DQ and DR - When HLA matching for a renal transplant the relative importance of the HLA antigens are as follows DR > B > A

Graft survival • 1 year = 90%, 10 years = 60% for cadaveric transplants • 1 year = 95%, 10 years = 70% for living-donor transplants

Post-op problems • ATN of graft • Vascular thrombosis • Urine leakage • UTI

Hyperacute acute rejection - Due to antibodies against donor HLA type 1 antigens - Rarely seen due to HLA matching

Acute graft failure (< 6 months) - Usually due to mismatched HLA - Other causes include cytomegalovirus infection - Management: give steroids, if resistant use monoclonal antibodies

Causes of chronic graft failure (> 6 months) - Chronic allograft nephropathy - Ureteric obstruction - Recurrence of original renal disease (MCGN > IgA > FSGS)

468. Features which are evaluated for the grading of breast cancer include all the following, except: A. Tubule formation B. Mitoses C. Nuclear pleomorphism D. Tumour necrosis

Prepared by Dr: Mohammed Musa Brema Idress – My best wishes Page 199

E. Coarse chromatin

ANSWER IS D The necrosis of a tumour may be suggestive of a high grade tumour which has out grown its blood supply. However, the grading of breast cancer which classically follows the Bloom -Richardson grading model will tend to favor nuclear appearances (which include mitoses, coarse chromatin and pleomorphism). Tubule formation is an important marker of the degree of differentiation with formation of tubular structures being associated with well differentiated tumours.

Tumour grading Tumours may be graded according to their degree of differentiation, mitotic activity and other features. Grade 1 tumours are the most differentiated and grade 4 the least. The assessment is subjective, in most cases high grade equates to poor prognosis, or at least rapid growth.

Tumours of glandular epithelium will tend to arrange themselves into acinar type structures containing a central lumen. Well differentiated tumours may show excellent acinar formation and poorly differentiated tumours simply clumps of cells around a desmoplastic stroma. Sometimes tumours demonstrate mucous production without evidence of acinar formation. Since mucous production is evidence of a glandular function such tumours are often termed mucinous adenocarcinoma. Squamous cell tumours will typically produce structures resembling epithelial cell components. Well differentiated tumours may also produce keratin (depending upon tissue of origin).

469. A 34 year old man is diagnosed with an aggressive caecal adenocarcinoma. His sister died from the same disease at 38 years of age. His mother died from endometrial cancer at the age of 41. What is the most likely underlying abnormality? A. Familial adenopolypomatosis coli B. Gardeners syndrome C. Microsatellite instability of DNA repair genes D. Deletion of chromosome 6 E. MYH gene mutation

ANSWER IS C Lynch syndrome which is characterized by aggressive right sided colonic malignancy and endometrial cancer is caused by microsatellite instability of DNA repair genes.

Genetics of colorectal cancer The lifetime risk of colorectal cancer in the UK population is 5%. Up to 5% of newly diagnosed bowel cancers will be in those individuals who have a high genetically acquired risk of bowel cancer. Cancers arising in the low-moderate genetic risk group comprise approximately 30% of newly diagnosed bowel cancer.

Prepared by Dr: Mohammed Musa Brema Idress – My best wishes Page 200

Genetics of inherited colorectal cancer syndromes Syndrome Features Genes implicated FAP More than 100 adenomatous polyps affecting the colon and rectum. Duodenal and fundic glandular polyps APC (over 90%) Gardner syndrome As FAP but with desmoid tumours and mandibular osteomas APC Turcots syndrome Polyposis and colonic tumours and CNS tumours APC +MLH1 and PMS2 HNPCC Colorectal cancer without extensive polyposis. Endometrial cancer, renal and CNS MSH2, MLH1, PMS2 and GTBP Peutz-Jeghers syndrome Hamartomatous polyps in GI tract and increased risk of GI malignancy LKB1 andSTK11 (in up to 70%) Cowden disease Multiple hamartomas (see below) PTEN (85%) MYH associated polyposis Autosomal recessive, multiple adenomatous polyps in GI tract, those in colon having somatic KRAS mutations MYH

FAP Autosomal dominant condition, affects 1 in 12,000. Accounts for 0.5% of all CRCs. Lifetime incidence of colorectal cancer in untreated FAP =100%. Up to 25% cases are caused by de-novo germ line mutations and show no prior family history. The APC tumour suppressor gene is affected in most cases.

APC in non inherited colorectal cancer Up to 80% of sporadic colorectal cancers will have somatic mutations that inactivate APC. Both alleles are usually affected. Although the APC protein more than likely has multiple critical cellular functions, the best-established role for APC in the cancer process is as a major binding partner and regulator of the β- catenin protein in the so-called canonical or β- catenin dependent sent signaling pathway.

HNPCC HNPCC cancers differ from conventional tumours in a number of respects. In the colon the tumours are more likely to be right sided, histologically they are more likely to be mucinous and have dense lymphocytic infiltrates. To be diagnosed as having HNPCC individuals must show typically HNPCC tumours in at least three individuals, (one of whom must be a first degree relative to the other two). In at least two successive generations. At least one cancer must be diagnosed under the age of 50.

FAP must be excluded and tumours should be verified by pathological identification (Amsterdam criteria). The genetic changes in HNPCC stem primarily from microsatellite instability affecting DNA mismatch repair genes. In HNPCC the mismatch repair genes most commonly implicated include; MSH2 and MLH1 and these occur in up to 70% of people with HNPCC.

The finding of microsatellite instability is unusual in sporadic colorectal cancers. Approximately 60% of individuals who fulfill the Amsterdam criteria will not be found to have evidence of mismatch repair gene defects on genetic testing. The risk of developing colorectal cancer in those who do not have demonstrated mutation of the mismatch repair genes is increased if they fulfill the Amsterdam criteria, but not the extent that it is increased in those who fulfill the criteria AND have evidence of mismatch repair gene defects.

Prepared by Dr: Mohammed Musa Brema Idress – My best wishes Page 201

KRAS Mutations: The RAS family of small G proteins act as molecular switches downstream of growth factor receptors. KRAS and the other two members of the family; HRAS and NRAS, are the site of mutation in approximately 40% of colorectal cancers. When adenomas are examined the proportion of adenomas less than 1cm showing KRAS mutations was only 10% which contrasts with 50% in those lesions greater than 1cm. p53 mutations The p53 protein functions as a key transcriptional regulator of genes that encode proteins with functions in cell-cycle checkpoints at the G1/S and G2/M boundaries, in promoting apoptosis, and in restricting angiogenesis . As such, selection for p53 defects at the adenoma-carcinoma transition may reflect the fact that stresses on tumor cells activate cell-cycle arrest, apoptotic, and antiangiogenic pathways in cells with wild-type p53 function. Many colonic tumours will demonstrate changes in the p53 gene that may facilitate tumour progression through from adenoma to carcinoma.

Cowden syndrome: Also known as multiple hamartoma syndrome. Rare autosomal dominant condition with incidence of 1 in 200,000.. It is characterized by multiple mucocutaneous lesions, trichilemmomas, oral papillomas and acral keratosis. Most often diagnosed in third decade of life.

Breast carcinoma may occur in up to 50% of patients and conditions such as fibrocystic disease of the breast may occur in 75% of women. Thyroid disease occurs in 75% and may include malignancy. Endoscopic screening will identify disease in up to 85% although the small bowel is rarely involved. There is a 15-20% risk of developing colorectal cancer and regular colonoscopic screening from age 45 is recommended.

Terminology Oncogene: Oncogenes are genes which have the potential to induce cellular proliferation and avoid apoptosis. Oncogene mutations are general gain of function and are therefore dominant. Increased expression of oncogenes are found in most tumours tumour suppressor gene, these genes generally inhibit cellular proliferation or induce apoptosis. Mutations in tumour suppressor genes are generally loss of function mutations, and are therefore recessive. Mutations in both tumour suppressor gene alleles allow cells to proliferate without restraint

470. A 55 year old man undergoes a colonoscopy and a colonic polyp is identified. It has a lobular appearance and is located on a stalk in the sigmoid colon. Which of the processes below best accounts for this disease? A. Apoptosis B. Metaplasia C. Dysplasia D. Calcification E. Degeneration

ANSWER IS C Most colonic polyps described above are adenomas.

Prepared by Dr: Mohammed Musa Brema Idress – My best wishes Page 202

These may have associated dysplasia. The more high grade the dysplasia the greater the level of clinical concern.

Colonic polyps Colonic Polyps May occur in isolation of greater numbers as part of the polyposis syndromes. In FAP greater than 100 polyps are typically present. The risk of malignancy in association with adenomas is related to size and is the order of 10% in a 1cm adenoma. Isolated adenomas seldom give risk of symptoms (unless large and distal). Distally sited villous lesions may produce mucous and if very large electrolyte disturbances may occur.

Follow up of colonic polyps Group Features Action Low risk 1 or 2 adenomas less than 1cm No follow up or re-colonoscopy at 5 years Moderate risk 3 or 4 small adenomas or 1 adenoma greater than 1cm Re-scope at 3 years High risk More than 5 small adenomas or more than 3 with 1 of them greater than 1cm Re scope at 1 year. It is important to stratify patients appropriately and ensure that a complete colonoscopy with good views was performed.

Segmental resection or complete colectomy should be considered when: 1) Incomplete excision of malignant polyp 2) Malignant sessile polyp 3) Malignant pedunculated polyp with submucosal invasion 4) Polyps with poorly differentiated carcinoma 5) Familial polyposis coli -Screening from teenager up to 40 years by 2 yearly sigmoidoscopy/colonoscopy -Panproctocolectomy and Ileostomy or Restorative Panproctocolectomy. - Rectal polypoidal lesions may be amenable to transanal endoscopic microsurgery.

471. A 56 year old lady has just undergone a colonoscopy and a lesion was identified in the caecum. The histology report states that biopsies have been taken from a sessile serrated polyp with traditional features? What is the best management option? A. Perform a right hemicolectomy B. List the patient for colonoscopic polypectomy C. Discharge the patient D. Rescope the patient in 6 months

E. Rescope the patient at 3 years

ANSWER IS B These polyps represent an alternative pathway to progression to carcinoma and may be diagnostically confused with hyperplastic polyps. Hyperplastic polyps are more common in the left colon and confer no increased risk. SSA's are more common in the right colon and are usually larger. Those with "traditional features" on histology have dysplasia with increased risk of malignant transformation.

Prepared by Dr: Mohammed Musa Brema Idress – My best wishes Page 203

472. A 30 year old male presents with gynaecomastia. Clinically he is noted to have a nodule in the left testis. The most likely diagnosis is: A. Oestrogen abuse B. Seminoma with syncytiotrophoblast giant cells C. Teratoma D. Choriocarcinoma E. Leydig cell tumour

ANSWER IS E Leydig cell tumours - Are rare testicular sex cord stromal tumours (which also include sertoli cell tumours) which are associated with hormonal activity. - Patients with Leydig cell tumours may present with gynaecomastia before they notice testicular enlargement. - Majority are benign - Histology: eosinophilic cells in columns

Testicular disorders - Testicular cancer Testicular cancer is the most common malignancy in men aged 20-30 years. Around 95% of cases of testicular cancer are germ-cell tumours. Risk factors for testicular cancer 1. Cryptorchidism 2. Infertility 3. Family history 4. Klinefelter's syndrome 5. Mumps orchitis Features - A painless lump is the most common presenting symptom - Pain may also be present in a minority of men - Other possible features include hydrocele, gynaecomastia Diagnosis - Ultrasound is first-line - CT scanning of the chest/ abdomen and pelvis is used for staging - Tumour markers (see above) should be measured Management - Orchidectomy (Inguinal approach) - Chemotherapy and radiotherapy may be given depending on staging - Abdominal lesions >1cm following chemotherapy may require retroperitoneal lymph node dissection. Prognosis is generally excellent - 5 year survival for seminomas is around 95% if Stage I - 5 year survival for teratomas is around 85% if Stage I

Benign disease Epididymo-orchitis Acute epididymitis is an acute inflammation of the epididymis, often involving the testis and usually caused by bacterial infection.

Prepared by Dr: Mohammed Musa Brema Idress – My best wishes Page 204

- Infection spreads from the urethra or bladder. In men <35 years, gonorrhoea or chlamydia are the usual infections. - Amiodarone is a recognized non infective cause of epididymitis, which resolves on stopping the drug. - Tenderness is usually confined to the epididymis, which may facilitate differentiating it from torsion where pain usually affects the entire testis.

Testicular torsion - Twist of the spermatic cord resulting in testicular ischaemia and necrosis. - Most common in males aged between 10 and 30 (peak incidence 13-15 years) - Pain is usually severe and of sudden onset. - Cremasteric reflex is lost and elevation of the testis does not ease the pain. - Treatment is with surgical exploration. If atorted testis is identified then both testis should be fixed as the condition of bell clapper testis is often bilateral.

473. Which of the following features are not typical of Crohns disease? A. Complex fistula in ano B. Small bowel strictures C. Skip lesions D. Rose thorn ulcers' on barium studies E. Pseudopolyps on colonoscopy

ANSWER IS E Pseudopolyps: are a feature of ulcerative colitis and occur when there is severe mucosal ulceration. The remaining islands of mucosa may then appear to be isolated and almost polypoidal.

Crohns disease Crohns disease is a chronic transmural inflammation of a segment(s) of the gastrointestinal tract and may be associated with extra intestinal manifestations. Frequent disease patterns observed include ileal, ileocolic and colonic disease. Peri- anal disease may occur in association with any of these. The disease is often discontinuous in its distribution. Inflammation may cause ulceration, fissures, fistulas and fibrosis with stricturing. Histology reveals a chronic inflammatory infiltrate that is usually patchy and transmural.

474. A 32 year old lady presents with a 1.5cm pigmented lesion on her back. The surgeon is concerned that this may be a melanoma. What is the most appropriate course of action? A. 2mm punch biopsy from the centre of the lesion B. 4mm punch biopsy from the centre of the lesion C. Wide excision of the lesion with 3cm margins D. Excisional biopsy of the lesion E. Wide excision of the lesion with 1cm margins

ANSWER IS D

Prepared by Dr: Mohammed Musa Brema Idress – My best wishes Page 205

Suspicious naevi should NOT be partially sampled as histological interpretation is severely compromised. Complete excision is mandatory where lesions fulfill diagnostic criteria. However, wide excision for margins may be deferred until definitive histology is available.

Lesions that are suspicious for melanoma should be excised with complete margins. Radical excision is not routinely undertaken for diagnostic purposes and therefore if subsequent histopathological assessment determines that the lesion is a melanoma a re-excision of margins may be required. Incisional punch biopsies of potential melanomas makes histological interpretation difficult and is best avoided.

Malignant melanoma The main diagnostic features (major criteria): 1. Change in size 2. Change in shape 3. Change in colour Secondary features (minor criteria): 1. Diameter >6mm 2. Inflammation 3. Oozing or bleeding 4. Altered sensation

Treatment - Suspicious lesions should undergo excision biopsy. The lesion should be removed in completely as incision biopsy can make subsequent histopathological assessment difficult. - Once the diagnosis is confirmed the pathology report should be reviewed to determine whether further re-excision of margins is required (see below):

Margins of excision-Related to Breslow thickness 1. Lesions 0-1mm thick 1cm 2. Lesions 1-2mm thick 1- 2cm (Depending upon site and pathological features) 3. Lesions 2-4mm thick 2-3 cm (Depending upon site and pathological features) 4. Lesions >4 mm thick 3cm

Further treatments such as sentinel lymph node mapping, isolated limb perfusion and block dissection of regional lymph node groups should be selectively applied.

475. A 70 year old male presents with painless frank haematuria. Clinical examination is unremarkable. Routine blood tests reveal a haemoglobin of 18g/dl but are otherwise normal. What is the most likely underlying diagnosis? A. Squamous cell carcinoma of the bladder B. Adenocarcinoma of the prostate C. Adenocarcinoma of the kidney D. Wilms tumour E. Transitional cell carcinoma of the renal pelvis

Prepared by Dr: Mohammed Musa Brema Idress – My best wishes Page 206

ANSWER IS C Polycythaemia is a recognized feature of renal cell carcinoma. Wilms tumours most commonly occur in children.

Haematuria Causes of haematuria Trauma - Injury to renal tract - Renal trauma commonly due to blunt injury (others penetrating injuries) - Ureter trauma rare: iatrogenic - Bladder trauma: due to RTA or pelvic fractures Infection - Remember TB Malignancy - Renal cell carcinoma (remember paraneoplastic syndromes): painful or painless - Urothelial malignancies: 90% are transitional cell carcinoma, can occur anywhere along the urinary tract. Painless haematuria. - Squamous cell carcinoma and adenocarcinoma: rare bladder tumours - Prostate cancer - Penile cancers: SCC Renal disease - Glomerulonephritis Stones - Microscopic haematuria common Structural abnormalities - Benign prostatic hyperplasia (BPH) causes haematuria due to hypervascularity of the prostate gland - Cystic renal lesions e.g. polycystic kidney disease - Vascular malformations - Renal vein thrombosis due to renal cell carcinoma Coagulopathy - Causes bleeding of underlying lesions Drugs - Cause tubular necrosis or interstitial nephritis: aminoglycosides, chemotherapy - Interstitial nephritis: penicillin, sulphonamides, and NSAIDs - Anticoagulants Benign - Exercise Gynaecological - Endometriosis: flank pain, dysuria, and haematuria that is cyclical Iatrogenic - Catheterization - Radiotherapy; cystitis, severe haemorrhage, bladder necrosis

476. Which of these lesions is most closely associated with Barrett’s oesophagus? A. Squamous cell carcinoma B. Gastro intestinal stromal tumours

Prepared by Dr: Mohammed Musa Brema Idress – My best wishes Page 207

C. Carcinoid tumours D. Leiomyosarcoma E. Adenocarcinoma

ANSWER IS E Barrett’s oesphagus is most closely associated with adenocarcinoma, and it confers a 30 fold increased risk of developing the condition.

Oesophageal cancer - Incidence is increasing - In most cases in the Western world this increase is accounted for by a rise in the number of cases of adenocarcinoma. In the UK adenocarcinomas account for 65% of cases. - Barrett’s oesophagus is a major risk factor for most cases of oesophageal adenocarcinoma. - In other regions of the world squamous cancer is more common and is linked to smoking, alcohol intake, diets rich in nitrosamines and achalasia. - Surveillance of Barrett’s is important as it imparts a 30 fold increase in cancer risk and if invasive malignancy is diagnosed early then survival may approach 85% at 5 years.

Diagnosis - Upper GI endoscopy is the first line test - Contrast swallow may be of benefit in classifying benign motility disorders but has no place in the assessment of tumours - Staging is initially undertaken with CT scanning of the chest, abdomen and pelvis. If overt metastatic disease is identified using this modality then further complex imaging is unnecessary - If CT does not show metastatic disease, then local stage may be more accurately assessed by use of endoscopic ultrasound. - Staging laparoscopy is performed to detect occult peritoneal disease. PET CT is performed in those with negative laparoscopy. Thoracoscopy is not routinely performed.

Treatment - Operable disease is best managed by surgical resection. The most standard procedure is an Ivor- Lewis type oesophagectomy. This procedure involves the mobilization of the stomach and division of the oesophageal hiatus. The abdomen is closed and a right sided thoracotomy performed. The stomach is brought into the chest and the oesophagus mobilized further.

- An intrathoracic oesophagogastric anastomosis is constructed. Alternative surgical strategies include a trans-hiatal resection (for distal lesions), a left thoraco-abdominal resection (difficult access due to thoracic aorta) and a total oesophagectomy (McKeown) with a cervical oesophagogastric anastomosis.

Prepared by Dr: Mohammed Musa Brema Idress – My best wishes Page 208

- The biggest surgical challenge is that of anastomotic leak, with an intrathoracic anastomosis this will result in mediastinitis. With high mortality. The McKeown technique has an intrinsically lower systemic insult in the event of anastmotic leakage.

- In addition to surgical resection many patients will be treated with adjuvant chemotherapy.

477. A 56 year old man presents with epigastric discomfort and episodes of migratory thrombophlebitis. On examination he is mildly jaundiced. A CT scan shows perihilar lymphadenopathy and a mass in the pancreatic head. Which of the following is the most likely underlying diagnosis? A. Squamous cell carcinoma of the pancreas B. Adenocarcinoma of the pancreas C. Insulinoma D. Glucagonoma E. Gastrinoma

ANSWER IS B Adenocarcinoma of the pancreas is the most likely diagnosis and migratory thrombophlebitis is associated with the condition. Squamous cells carcinoma is extremely uncommon in the pancreas. Gastrinoma are extremely rare and thus not the most likely diagnosis.

Pancreatic cancer Adenocarcinoma Risk factors - Smoking, diabetes, Adenoma, Familial adenomatous polyposis - Mainly occur in the head of the pancreas (70%) - Spread locally and metastasizes to the liver - Carcinoma of the pancreas should be differentiated from other periampullary tumours with better prognosis Clinical features 1) Weight loss 2) Painless jaundice 3) Epigastric discomfort (pain usually due to invasion of the coeliac plexus is a late feature) 4) Pancreatitis 5) Trousseau's sign: migratory superficial thrombophlebitis Investigations - USS: May miss small lesions - CT Scanning (pancreatic protocol). If unresectable on CT then no further staging needed. - PET/CT for those with operable disease on CT alone - ERCP/ MRI for bile duct assessment. - Staging laparoscopy to exclude peritoneal disease.

Prepared by Dr: Mohammed Musa Brema Idress – My best wishes Page 209

Management - Head of pancreas: Whipple's resection (SE dumping and ulcers). Newer techniques include pylorus preservation and SMA/ SMV resection. - Carcinoma body and tail: poor prognosis, distal pancreatectomy if operable. - Usually adjuvant chemotherapy for resectable disease - ERCP and stent for jaundice and palliation. - Surgical bypass may be needed for duodenal obstruction.

478. A 22 year old man presents with symptoms of headache, lethargy and confusion. On examination he is febrile and has a right sided weakness. A CT scan shows a ring enhancing lesion affecting the left motor cortex. Which of the following is the most likely diagnosis? A. Arteriovenous malformation B. Cerebral abscess C. Herpes simplex encephalitis D. Metastatic renal adenocarcinoma E. Glioblastoma multiforme

ANSWER IS B The combination of rapidly progressive neurology, fever and headache is highly suggestive of cerebral abscess. CT scanning will show a ring enhancing lesion because the intravenous contrast cannot penetrate the centre of the abscess cavity. HSV encephalitis does not produce ring enhancing lesions.

Brain abscess - CNS abscesses may result from a number of causes including, extension of sepsis from middle ear or sinuses, trauma or surgery to the scalp, penetrating head injuries and embolic events from endocarditis. - The presenting symptoms will depend upon the site of the abscess (those in critical areas e.g. motor cortex) will present earlier. Abscesses have a considerable mass effect in the brain and raised intra cranial pressure is common. - Although fever, headache and focal neurology are highly suggestive of a brain abscess the absence of one or more of these does not exclude the diagnosis, fever may be absent and even if present, is usually not the swinging pyrexia seen with abscesses at other sites. - Assessment of the patient includes imaging with CT scanning. - Treatment is usually surgical, a craniotomy is performed and the abscess cavity debrided. The abscess may reform because the head is closed following abscess drainage.

479. Which of the following are true? Small cell cancer of the lung: A. Accounts for more than 50% of all lung cancers B. Is common in elderly non-smokers C. Is frequently benign and localized at presentation D. May be associated with hypercalcemia E. Has a median survival of 5 years if treated appropriately

Prepared by Dr: Mohammed Musa Brema Idress – My best wishes Page 210

ANSWER IS D Small cell cancers of the lung account for less than one-quarter of all lung cancers. It is named after the small round cells that make up these tumours.

Squamous cell carcinoma makes up nearly 35%, adenocarcinoma 20% and large cell carcinoma approximately 20% of all lung cancers. Small cell carcinoma is a tumour of neuroendocrine origin that may exist in the classical oat cell form or as an intermediate cell type.

Small cell lung cancer is almost always caused by smoking while adenocarcinoma is seen in elderly non-smokers. These tumours are highly malignant and are usually disseminated at presentation.

Small cell lung cancers are often associated with ectopic hormone production and paraneoplastic syndromes such as adrenocorticotrophic hormone (ACTH) (Cushing’s syndrome), parathyroid hormone (hence serum hypercalcemia), syndrome of inappropriate antidiuretic hormone secretion (causes hyponatremia) and myasthenic syndrome.

The median survival of patients with small cell cancer is 3 months if untreated and 12–18 months if treated.

480. A 35-year-old man has a clinical diagnosis of a testicular malignancy in a previously maldescended testis. From the following list of testicular malignancies which is the most likely histological diagnosis? Single best answer question – choose ONE true option only A. Interstitial (Leydig) cell tumour B. Lymphoma C. Seminoma D. Teratoma E. Yolk sac tumour

ANSWER IS C Seminomas and teratomas are of germ cell origin accounting for 85-90 per cent of testicular tumours. Peak incidence of seminomas is 30-50 years compared to 20-30 years for teratomas. Seminomas are the commonest testicular tumours to develop in maldescended testicles. Lymphomas of the testis are far less common with a peak incidence of 60-70 years. They are often bilateral and can be a manifestation of more diffuse disease. Yolk sac tumours occur before the age of three years and are the commonest testicular malignancy in children.

481. The following results came back on a 39-year-old female patient; Hb 10.1 g/dl MCV 85 fl Ferritin 22 ng/ml (22 - 400 ng/ml) B12 248 ng/l (160-900 ng/l)

Prepared by Dr: Mohammed Musa Brema Idress – My best wishes Page 211

Folate 530 ng/l (165-900 ng/l) Transferrin 500 mg/dl (185-370 mg/dl) What is the most likely diagnosis? Single best answer question – choose ONE true option only A. Iron deficiency anaemia B. Anaemia of chronic disease C. Folate deficiency anaemia D. B12 deficiency E. Thalassaemia

ANSWER IS A The low ferritin and high transferrin suggest iron deficiency. In anaemia of chronic disease, the transferrin falls and ferritin is raised as negative and positive inflammatory markers respectively. In thalassaemia ferritin is high.

482. A 35-year old lady develops a DVT 7 days following a panproctocolectomy for Crohn's disease. She is positive for anticardiolipin and lupus anticoagulant antibodies. What is the underlying condition behind this DVT? Single best answer question – choose ONE true option only A. Factor V Leiden B. Protein C deficiency C. Protein S deficiency D. Antiphospholipid syndrome E. Antithrombin III deficiency

ANSWER IS D The presence of the two antibodies indicates that this patient has the hypercoagulable state of antiphospholipid syndrome.

483. A 56-year-old Caucasian man is diagnosed with a liver abscess. He has no recent history of foreign travel, or significant past medical history. Which of the following micro-organisms is the most likely to be the causative agent? Single best answer question – choose ONE true option only A. Candida albicans B. Echinococcus C. Entamoeba histolytica D. Escherichia coli E. Streptococcus milleri

ANSWER IS D Pyogenic abscesses account for 75 per cent of liver abscesses in developed counties. Most pyogenic liver abscesses are secondary to intra-abdominal infections. The most common cause is cholangitis associated with biliary stones followed by diverticulitis. Most are polymicrobial with Gram negative aerobic and anaerobic organisms predominating. Streptococcus milleri usually arises from bacterial endocarditis. Fungal abscesses are associated with immunosuppression. Worldwide, amoebic abscesses are the commonest cause of liver abscess with 10 per cent of the world’s

Prepared by Dr: Mohammed Musa Brema Idress – My best wishes Page 212 population infected with Entamoeba histolytica. Hydatid disease is common in many sheep-raising countries. Echinococcus granulosus is the causative species of Hydatid disease.

484. A 35-year-old man is admitted with ischaemic rest pain in both feet. There is no history of preceding intermittent claudication. He is a heavy smoker. Femoral and popliteal pulses are palpable. From the list below select the most likely diagnosis. Single best answer question – choose ONE true option only A. Atherosclerosis B. Fibromuscular dysplasia C. Polyarteritis nodosa D. Takayasu’s arteritis E. Thromboangiitis obliterans (Buerger’s disease)

ANSWER IS E Buerger’s disease: is characterized by segmental thrombotic occlusions of the small and medium sized arteries usually of the distal lower limb. It occurs predominantly in young male smokers, frequently associated with rest pain and tissue loss. Intermittent claudication is not a major feature. Tobacco abstinence is essential.

Takayasu’s arteritis has a female preponderance manifest by an obliterative arteritis of the aorta and its branches. Polyarteritis nodosa occurs between 40-60 years and affects small and medium sized arteries. Fibromuscular dysplasia affects renal and carotid arteries. Atherosclerosis tends to have a preceding history and very rarely manifests at such an early age.

485. A 30-year-old man presents with a painless swelling behind the anterior edge of the upper third of the right sternomastoid muscle. Fine needle aspiration reveals opalescent fluid. From the list below which is the most likely diagnosis? Single best answer question – choose ONE true option only A. Branchial cyst B. Branchial fistula C. Chemodectoma D. Pharyngeal pouch E. Thyroglossal cyst

ANSWER IS A Branchial cysts originate from embryonic branchial cleft tissue remnants. Most present in the third decade with swelling behind the anterior border of sternomastoid.

These cysts often present when they become infected and contain inflamed lymphoid tissue. Classically opalescent fluid containing cholesterol crystals can be aspirated. Branchial fistulae arising from abnormalities of the first cleft appear as a sinus anterior or posterior to the ear and those of the second, along the anterior boarder of sternomastoid.

Prepared by Dr: Mohammed Musa Brema Idress – My best wishes Page 213

Pharyngeal pouches present behind sternomastoid and thyoglossal cysts, in the midline. Chemodectomas are slow growing, often painful, pulsatile masses at the angle of the mandible.

486. An 81-year-old ex-smoker presents with a 2-day history of worsening left calf pain even at rest. He has been getting pain for the last 10 years but was able to walk 500 meters. What is the underlying pathological process? Single best answer question – choose ONE true option only A. Embolus B. Thrombophilia C. Atheroma D. Thrombocytopenia E. Thrombosis

ANSWER IS E This is acute on chronic ischaemia, caused by rupture of the atheroma and formation of a thrombus on top.

487. A 45-year-old lady presents with right flank mass and haematuria. She also complains of depression, constipation and vomiting. What is the underlying diagnosis? Single best answer question - choose ONE true option only A. Colorectal carcinoma B. Renal stones C. Renal carcinoma D. Bladder carcinoma E. Pyelonephritis

ANSWER IS C This collection of symptoms can all be explained by renal carcinoma with hypercalcaemia as a paraneoplastic syndrome. Paraneoplastic syndromes are non- metastatic systemic effects caused by cancer. Hypercalcaemia causes depression, abdominal pain, lethargy and constipation.

488. Which of the following associated extra-alimentary conditions is more suggestive of ulcerative colitis than Crohn’s disease? Single best answer question – choose ONE true option only A. Ankylosing spondylitis B. Arthropathy C. Erythema nodosum D. Primary sclerosing cholangitis E. Pyoderma gangrenosum

ANSWER IS D Primary sclerosing cholangitis: is far more commonly seen in ulcerative colitis than Crohn’s disease.

Prepared by Dr: Mohammed Musa Brema Idress – My best wishes Page 214

The condition is characterized by a fibrous inflammatory reaction within the biliary tree leading to irregularity with multiple stenosis and biliary obstruction. There is no apparent relationship between duration or severity of ulcerative colitis and ultimately progresses to liver failure.

Arthropathy and ankylosing spondylitis are both features of ulcerative colitis and Crohn’s disease. Erythema nodosum is the commonest cutaneous manifestation of inflammatory bowel disease and occurs slightly more commonly in Crohn’s disease. Conversely pyoderma gangrenosum is slightly more prevalent in ulcerative colitis.

489. A 40-year-old man is diagnosed with right sided synchronous colonic tumours, which are mucinous and poorly differentiated. His mother had died at an early age of ovarian cancer and his maternal grandfather had died aged approximately 40 years of advanced caecal carcinoma. From the following list of inherited syndromes which is most likely to be prevalent in this family? Single best answer question – choose ONE true option only A. Cowdens syndrome B. Familial adenomatous polyposis C. Hereditary non-polyposis colorectal cancer D. Juvenile polyposis E. Peutz-Jeghers syndrome

ANSWER IS C Colonic tumours: associated with hereditary non-polyposis colorectal cancers (HNPCC) tend to have certain distinguishing pathological features. They occur on average twenty years before the peak incidence of sporadic tumours and are frequently synchronous and metachronous with a predilection for the proximal colon. They tend to be mucinous, poorly differentiated and “signet-ring” in appearance. Ovarian cancer is associated with NHPCC along with cancers of the endometrium and stomach.

490. A 16-year-old girl presents with a four month history of an aching discomfort in the distal right femur, relieved with simple analgesia. Plain radiograph of the knee shows an area of slight sclerosis. Which is the most likely diagnosis? Single best answer question – choose ONE true option only A. Chondroma B. Fibroma C. Osteochondroma D. Osteoclastoma E. Osteoid osteoma

ANSWER IS E Osteoid osteomas: are commonest in the femur and tibia. They are unusual as benign tumours in that they produce a constant aching pain which is classically relieved by simple analgesia. They can be difficult to see on plain radiographs.

Prepared by Dr: Mohammed Musa Brema Idress – My best wishes Page 215

Osteochondromas: often appear as a bony pedicle growing away from the epiphyseal plate, covered in a large cartilage cap.

Chondromas: are made up almost entirely of cartilage and are common in the hands and feet.

Fibromas: are well-circumscribed lytic lesions and osteoclastomas are commonly found lying close to the epiphyseal plate with destruction of the overlying cortex.

491. A 56-year-old woman presents with a rapidly growing raised solitary lesion on her face. Over the course of four weeks it has reached two centimeters in diameter. The lesion has now developed a necrotic, crusted centre. From the following list of skin lesions which is the most likely diagnosis? Single best answer question – choose ONE true option only A. Basal cell carcinoma B. Histiocytoma C. Keratocanthoma D. Pyogenic granuloma E. Squamous cell carcinoma

ANSWER IS C Keratocanthomas: are epidermal nodules almost indistinguishable from squamous cell carcinomas; however unlike squamous cell carcinomas, they grow very rapidly. Having attained the size of two to three centimeters over several weeks they spontaneously involute leaving a pitted scar. Histiocytomas appear as firm flesh coloured nodules mainly on the lower limb. Pyogenic granulomas also grow rapidly, however they usually occur on the fingers after trivial trauma and appear as raised, wet, pedunculated lesions. Basal cell carcinomas are slow growing.

492. A 21 year old man undergoes drainage of perianal abscess. Which is the best method to manage the wound? Single best answer question – choose ONE true option only A. Primary closure B. Delayed primary closure C. Healing by secondary intention D. Permanent fistula E. Tertiary closure by skin graft

ANSWER IS C Infected wounds are best left open to heal by secondary intention. Primary closure is achieved by suturing or stapling or steristrip. Delayed primary closure is needed for contaminated wounds that are left open day 1 and closed by suturing by day 3 to 5 before excessive granulation is started.

493. A 26-year-old man presents with a hard, non-tender lump in his left testicle. Both AFP and HCG are raised. What is the most likely nature of the lump? Single best answer question – choose ONE true option only

Prepared by Dr: Mohammed Musa Brema Idress – My best wishes Page 216

A. Hematoma B. Abscess C. Teratoma D. Seminoma E. Hydrocoele

ANSWER IS C Testicular lumps are often noticed after trauma. Seminomas do not cause a raised AFP and peak age is 30-40 years. Teratomas are derived from multipotent germ cells so cause both AFP and B-HCG to rise, peak age being 20-30 years.

494. Clostridium difficile diarrhoea is best treated with? Single best answer question – choose ONE true option only A. Intravenous vancomycin B. Oral metronidazole C. Oral amoxicillin + clavulanic acid D. Intravenous timentin E. Oral erythromycin

ANSWER IS B The standard treatment for clostridium difficile diarrhoea is oral metronidazole or oral vancomycin. Options C & D are common culprits in predisposing to this infection.

495. An absolute contraindication to renal transplantation is? Single best answer question – choose ONE true option only A. ABO incompatability B. Rhesus incompatability C. Zero major histo-compatibility complex match D. Size mismatch E. CMV positive donor to CMV negative recipient

ANSWER IS A Unlike ABO, Rhesus antigens do not play a major part in renal transplant rejection. With modern immunosuppressive therapy, it is now possible for live donation to take place between individuals with a six major histo-compatibility complex mismatch. Although size may play a part when performing transplants in children, it is not an absolute contraindication. In the case of a CMV positive donor to a CMV negative recipient, the recipient will require prophylaxis with ganciclovir.

496. A 32-year-old woman who had undergone an appendicectomy 2-weeks ago presents to her General Practitioner with a 24-hour history of right upper quadrant abdominal pain, fever with chills and rigors, and shortness of breath. She says that the pain is radiating to her right shoulder tip. On examination, her temperature is 38.9º C, pulse rate is 106/min and she is acutely tender over the right hypochondric region. Chest X-ray reveals a right-sided basal atelectasis and mild pleural effusion. From the options below choose the ONE that you think is the

Prepared by Dr: Mohammed Musa Brema Idress – My best wishes Page 217 most likely diagnosis in this patient: Single best answer question – choose ONE true option only. A. Emphysema of right lung B. Acute pancreatitis C. Sclerosing cholangitis D. Subphrenic abscess E. Pyonephrosis

ANSWER IS D Subphrenic abscess usually arises 3 to 6 weeks following abdominal surgery, mainly to the biliary tract, duodenum or stomach, or following a perforated viscous or anastamotic leakage.

The subphrenic space is in direct contact with the para-colic gutter thereby allowing peritoneal contamination such as bile, blood or bowel contents to spread. Subphrenic abscess is right-sided in about 50%, left-sided in 25% and bilateral in 25% of patients.

Clinical features include pyrexia with chills and rigors, anorexia, loss of appetite and loss of weight. Diaphragmatic irritation may affect the lung, resulting in chest pain, dyspnoea and non-productive cough. Basal atelectasis, pneumonia and pleural effusion are recognized complications of this condition. Ultrasound scan is the investigation of choice to diagnose subphrenic abscess, and, if an abscess is identified, ultrasound guided percutaneous drainage catheter may be placed at the same time.

497. Systemic Inflammatory Response Syndrome (SIRS) is present if a patient has? Single best answer question – choose ONE true option only A. Temperature of 37.5°C, a leukocyte count of 11,000/mm3, heart rate 150 beats/min B. Temperature of 38°C, a leukocyte count of 5,000/mm3, respiratory rate 15 breaths/min C. Temperature of 35.5°C, a leukocyte count of 8,000/mm3, pCO2 50mmHg D. Temperature of 38.0°C, a leukocyte count of 2,000/mm3, respiratory rate 30 breaths/min E. Temperature of 37.5°C, a leukocyte count of 11,000/mm3, heart rate 100 beats/min

ANSWER IS D A patient is said to have SIRS if he satisfies two or more of the following criteria: temperature > 38°C or < 36°C; heart rate > 90 beats/minute; respiratory rate > 20/min (or PaCO2 < 32mm Hg); leukocyte count > 12,000/mm3 or < 4,000/mm3 (or > 10% immature (band) cells).

498. A 48-year-old woman presents with a painless slow growing mass in the left parotid gland. There is no apparent involvement of the facial nerve. From the

Prepared by Dr: Mohammed Musa Brema Idress – My best wishes Page 218 following list, which is the most likely finding on biopsy? Single best answer question – choose ONE true option only A. Adenoid cystic carcinoma B. Adenolymphoma (Warthin’s tumour) C. Monomorphic adenoma D. Mucoepidermoid tumour E. Pleomorphic adenoma

ANSWER IS E Pleomorphic adenoma: is the most common benign salivary gland tumour accounting for approximately 80% of parotid tumours. It contains mixed epithelial and mesenchymal elements and generally develops superficial to the facial nerve. Facial nerve involvement is suggestive of malignant disease and adenoid cystic carcinomas in particular are unusual in their predilection for peri-neural spread.

Adenolymphomas constitute 18% of all salivary gland neoplasms occurring in the sixth and seventh decades of life.

Monomorphic adenomas are rare and although 90% of malignant mucoepidermoid tumours arise in the parotid, they are much rarer than pleomorphic adenomas.

499. A 33-year-old woman underwent a total unilateral thyroid lobectomy for a suspicious dominant nodule. Subsequent histology demonstrated multi-focal papillary cancer with 18mm and 4mm foci. Resection margins were clear, with no lymphadenopathy noted at operation. Which of the following treatment options is the most appropriate next stage of treatment? Single best answer question – choose ONE true option only A. Completion total thyroidectomy and radical neck dissection B. Completion total thyroidectomy C. Local radiotherapy D. Radioactive iodine (131I) imaging and ablation E. Regular out-patient follow-up with thyroglobulin measurements

ANSWER IS B Micropapillary tumours (<10mm diameter) are frequently co-incidental findings and can be safely managed without total thyroidectomy in the absence of clinically overt contralateral or metastatic disease. Larger tumours, especially when multi-focal, should be considered for completion thyroidectomy. There is no evidence to support radical block dissection of the neck. Excision of locally infiltrated structures may be required with extensive extrathyroidal disease, along with radiotherapy. Radioactive iodine is a useful means of detecting metastatic disease after total thyroidectomy. Measurement of thyroglobulin is a sensitive indicator of recurrent disease after total thyroidectomy when the patient is on full thyroxin replacement therapy.

500. Patient presents with a painful slowly enlarging left parotid mass and a facial nerve palsy. The most likely lesion is: Single best answer question - choose ONE true option only

Prepared by Dr: Mohammed Musa Brema Idress – My best wishes Page 219

A. Pleomorphic adenoma B. Warthin’s tumour C. Myoepithelioma D. Adenoid cystic carcinoma E. Ductal papilloma

ANSWER IS D The facial nerve involvement points to a malignant tumour. The other tumours are all benign. Benign tumours are also more often painless.

501. 33-year-old cyclist is brought to the Accident and Emergency department after he was hit by a car travelling at about 40-50 mph on a dual carriage way. On examination, he has severe bruising over his left antero-lateral chest wall and there is a puncture wound at the level of left 6th ICS. There is decreased air entry over the left lower lobe of lung and bowel sounds are heard in the chest. A chest X- ray reveals fracture of the lower four ribs on the left side and a raised left dome of the diaphragm. From the options below choose the ONE which you think is the most likely diagnosis in this patient: Single best answer question – choose ONE true option only. A. Tension pneumothorax B. Traumatic haemothorax C. Diaphragmatic rupture D. Cardiac tamponade

ANSWER IS C The clinical signs and symptoms in this patient are most likely to be due to a ruptured diaphragm. Direct penetrating injury to the thoraco-abdominal region is a common cause for diaphragmatic rupture.

The injury could be at any level between the 4th and the 10th inter-costal space, depending on the patient’s respiratory pattern.

The other causes include rib fractures and a sudden increase in thoraco-abdominal pressure, as occurs when the patient with a closed glottis is hit in the abdomen. Patients with rupture of the diaphragm may be present with hypotension, tachycardia, tachypnoea, chest pain and decreased air-entry in the lung base of the affected side. However, diaphragmatic rupture may be difficult to detect clinically, and thus may result in significant morbidity or sometimes mortality. Rupture may be seen on plain chest radiograph especially with the abnormal location of the naso-gastric tube; the accuracy of this method, however, is modest. Fracture(s) of the lower ribs on the affected side may or may not be present. The differential diagnoses for a raised left hemidiaphragm, both clinically and in plain radiography, includes, phrenic nerve palsy, atelectasis, diaphragmatic hernia and distended abdominal viscera

502. A 74-year-old gentleman presents to his General Practitioner with a four- month history of vague lower abdominal pain, abdominal distension and absolute

Prepared by Dr: Mohammed Musa Brema Idress – My best wishes Page 220 constipation. On further questioning he says that he has noticed a change in his bowel habits, a sense of incomplete evacuation of the bowel and his stools to be mixed with blood. He has lost more than a stone in weight during this period. On examination, the abdomen is soft but a mass is felt over the left iliac fossa. Plain abdominal X-ray reveals a markedly dilated colon. From the options below choose the ONE that you think is the most likely diagnosis in this patient: Single best answer question – choose ONE true option only. A. Toxic B. Acute colonic pseudo-obstruction C. Sigmoid volvulus D. Colonic carcinoma E. Diverticular disease

ANSWER IS D Elderly patients with per rectal bleeding, change in bowel habits, and weight loss should be considered to have colonic cancer unless proven otherwise. Increase in age is a risk factor for developing colonic cancer. The other risk factors, among others, include a family history of colon cancer, familial adenomatous polyposis, diet rich in red meat and long standing ulcerative colitis or Crohn’s disease. The clinical presentation of patients with colonic malignancy depends on the site of the tumour.

Right-sided colonic carcinoma commonly presents with anaemia, tiredness, malaise, pallor and loss of weight, while the left sided colonic carcinoma presents with change in the bowel habits, bleeding per rectum and intestinal obstruction. Rectal carcinoma, in addition to the features seen in left-sided colonic carcinoma, is associated with a sense of incomplete evacuation of the bowel (tenesmus).

Investigations for suspected colonic malignancy includes full blood cell count, renal function and electrolytes, liver function tests (to rule out hepatic involvement), plain X-ray of the abdomen, and ultrasound and CT scans. Carcinoembryonic antigen (CEA) is the commonly used tumour marker to diagnose colonic malignancy and subsequently to assess the progress, including recurrence.

503. A 33-year-old company executive presents to his General Practitioner with a six-week history of upper abdominal pain and diarrhoea. He also says that he has had a few episodes of ‘dark-coloured’ vomitus during this period. General examination is unremarkable. Endoscopic examination of his upper gastro- intestinal region reveals multiple ulcers in the stomach and duodenum. From the options below choose the ONE that you think is the most likely diagnosis in this patient: Single best answer question – choose ONE true option only. A. Carcinoid tumour B. Gastrinoma C. Somatostatinoma D. Vasoactive intestinal peptide tumour E. ACTH secreting tumour

Prepared by Dr: Mohammed Musa Brema Idress – My best wishes Page 221

ANSWER IS B Gastrinomas, which occur primarily in the pancreas and duodenum, are malignant in nearly two-thirds of cases. The patients may present with upper abdominal/epigastric pain and vomiting; the vomitus may be ‘coffee-ground’ due to bleeding from the ulcers.

Ninety percent of patients with gastrinomas develop peptic ulceration. Gastrinomas may either arise sporadically or as part of Zollinger-Ellison syndrome (peptic ulceration, gastric acid hypersecretion and islet cell tumour of the pancreas). Sporadic Zollinger-Ellison occurs most frequently in the fifth decade of life.

Approximately 20% of patients with Zollinger-Ellison syndrome have MEN type I syndrome. An elevated basal gastric acid output >15mEq/h and a serum gastrin >1000pg/ml are suggestive of a gastrinoma. If it is difficult to make a diagnosis, a secretin stimulation test may be indicated. Lesions are localized by somatostatin- receptor scintography.

A CT scan may be indicated to exclude metastases. The treatment of this condition is either conservative (high dose proton pump inhibitors) or surgical. Surgical resection may be aided by intra-operative ultrasound and/or intra-operative endoscopy.

504. A 47-year-old barmaid presents to the Accident and Emergency department with a 12-hour history of right upper quadrant pain and vomiting. She says that the pain is radiating to her right scapula and exacerbates on breathing. She appears pale and mildly jaundiced. On examination, her pulse rate is 98/min, blood pressure is 126/84 mmHg and temperature is 37.6º C. Abdominal examination reveals tenderness over the right hypochondrium but no mass is palpable. Plain radiographs of the abdomen (supine) and chest (erect) are unremarkable. From the options below choose the ONE that you think is the most likely diagnosis in this patient: Single best answer question – choose ONE true option only. A. Perforated peptic ulcer B. Acute pancreatitis C. Acute biliary cholangitis D. Acute cholecystitis E. Infective hepatitis

ANSWER IS D The history, signs and symptoms in this patient are suggestive of acute cholecystitis. Acute cholecystitis is more common in females over the age of 40 and with high BMI. Gallstones are the commonest cause for acute cholecystitis. Obstruction of the common bile duct due to stones leads to accumulation of bile and inflammation, resulting in an acutely inflamed gall bladder.

Other risk factors for acute cholecystitis include alcohol abuse and tumours of the gall bladder. The signs and symptoms of acute cholecystitis include severe right hypochondrial pain exacerbated by respiration, and vomiting, and increase in temperature. The rise in temperature is frequently mild to moderate; a very high

Prepared by Dr: Mohammed Musa Brema Idress – My best wishes Page 222 temperature with or without chills and rigors may point to a diagnosis of acute cholangitis.

A tender, inflamed gall bladder may be palpable in some patients. Likewise, jaundice may or may not be present. The differential diagnoses for acute cholecystitis include acute pancreatitis, peptic ulcer disease or perforated peptic ulcer, appendicitis, acute infective hepatitis and pleurisy.

505. A 54-year-old male presents to his General Practitioner with a two-week history of sweating, headache, constipation and itchy lesions over his back. On examination, his blood pressure is 162/94 mmHg and his pulse rate is 102/min. Twenty-four hour urinary catecholamines, metanephrines and vanillyl-mandellic acid are found to be elevated. A CT and a 131I-meta-iodo-benzyl-guanidine scan confirm a phaeochromocytoma. He is subsequently found to have a medullary carcinoma of the thyroid. From the options below choose the ONE that you think is the most likely diagnosis in this patient: Single best answer question – choose ONE true option only. A. Multiple Endocrine Neoplasia I B. Secondary hyperparathyroidism C. Multiple Endocrine Neoplasia IIB D. Carcinoid tumour E. Multiple Endocrine Neoplasia IIA

ANSWER IS E Multiple Endocrine Neoplasia II (MEN II) is an autosomal dominant disorder caused by mutations in the RET proto-oncogene. MEN II has 3 distinct subtypes - MEN IIA, MEN IIB, and familial medullary thyroid carcinoma-only. MEN II describes the association of medullary thyroid carcinoma, phaeochromocytomas and parathyroid tumours.

MEN IIB is characterized by MEN IIA plus Marfanoid features and mucosal neuromas. In MEN IIB, the medullary cancer is very aggressive with most patients dying before developing either a phaeochromocytoma or hyperparathyroidism. A patient with medullary carcinoma of the thyroid may present with diarrhoea due to elevated prostaglandin or calcitonin levels.

Patients with hypercalcemia may present with constipation, polyuria, polydipsia, depression, nephrolithiasis, glucose intolerance, gastroesophageal reflux, loss of bone density and fatigue. Patients with pheochromocytomas may present with hypertension, tachycardia, sweating and headaches. Cutaneous lichen amyloidosis in patients with MEN IIA manifests as multiple pruritic scaly skin lesions in the scapular area of the back.

506. A 22-year-old female is referred by her General Practitioner to the rapid access breast clinic with a lump in her left breast. She says that it has been present for about 8-weeks now and is painless. There is no bleeding or discharge from her nipples. On examination, there is a 2-cm sized, firm, mobile and smooth lump in

Prepared by Dr: Mohammed Musa Brema Idress – My best wishes Page 223 the upper outer quadrant of her left breast. There is no palpable axillary lymphadenopathy. Her paternal aunt died from breast cancer at the age of 62. From the options below choose the ONE that you think is the most likely diagnosis in this patient: Single best answer question – choose ONE true option only. A. Cystosarcomma phyllodes B. Fibroadenoma C. Fibrocystic disease D. Mondor’s disease E. Paget’s disease

ANSWER IS B Fibroadenoma: is the most commonly diagnosed breast tumour in women under 30 years of age. They are benign tumours originating from the breast lobule. They show proliferation of both epithelium and connective tissue elements, and is considered as an 'Aberration of Normal Development and Involution (ANDI)'.

Most fibroadenomas measure 2-3 cm in diameter. Fibroadenoma is common between 16 and 24 yrs of age; the incidence decreases towards menopause. Fibroadenomas are usually mobile, firm and smooth lumps (but sometimes may be lobulated). It may be multiple in approximately 10% of the cases.

The diagnosis is confirmed by triple assessment: (i) clinical examination (ii) radiological assessment (mammography or ultrasound scan), and (iii) cytological/histological (fine needle aspiration, core biopsy). Over a 5-year period, 50% increase in size, 25% remain stable and 25% decrease in size. Risk of malignant transformation is approximately 1 in 1,000.

507. A 25-year-old footballer presents to the Accident and Emergency department with pain in his left lower leg after he was violently kicked (in his leg) during a tackle. On examination, his pulse rate is 96/min and blood pressure is 116/74 mmHg. There is considerable bruising over the posterior aspect of his leg, and that part of the limb is tense, swollen and tender. He complains of altered sensation over the dorsum of his foot. Dorsiflexion of the foot and extension of the toes are painful and limited. Although he had normal anterior tibial and dorsalis pedis pulsations when he was brought to the Accident and Emergency department, they soon become weak and difficult to palpate. Plain radiograph of this limb does not reveal any fractures. From the options below choose the most appropriate cause for this patient’s signs and symptoms: Single best answer question - choose ONE true option only. A. Deep venous thrombosis B. Torn muscle bellies of gastrocnemius and soleus C. Compartment syndrome D. Ruptured Achilles tendon E. Thrombosis of the popliteal artery

ANSWER IS C

Prepared by Dr: Mohammed Musa Brema Idress – My best wishes Page 224

Compartment syndrome: is defined as an increase in the interstitial fluid pressure within an osteofascial compartment of sufficient magnitude to cause a compromise of the microcirculation leading to necrosis of the affected nerve(s) and muscle(s). It is a well-recognized and important complication of lower limb injuries, most commonly seen after fractures and crush injury, although it can occur in the absence of bony injury.

The other causes for compartment syndrome include electrical injuries, deep thermal burns, venom from snake bites, restricting tourniquets, and fluid extravasation (e.g. intravenous regional anaesthesia). The patient may present with unremitting pain that is not relieved by high doses of opioid analgesics. Severe pain in response to passive stretch of the ischaemic muscles is by far the most dramatic and reliable clinical sign of compartment syndrome.

Sensory loss occurs before motor loss. Early in its development, the peripheral pulses are normal, as is the colour and temperature of the affected part, since it is the microvasculature that is initially affected. Loss of peripheral pulses is usually a late and often sinister sign.

With progression of the condition, the limb becomes tense and swollen, and if left treated, the muscle weakness progresses to paralysis. Untreated, irreversible myoneural necrosis occurs within 6-8 hours. The areas of muscle may also infarct causing rhabdomyolysis, hyperkalaemia, hyperphosphataemia, high uric acid levels and metabolic acidosis.

508. A 51-year-old lady presents to the Surgical Emergency Assessment Unit with a 12-hour history of central colicky abdominal pain and vomiting. She has undergone a subtotal colectomy and formation of an end ileostomy for ulcerative colitis 7 years ago. Her ileostomy has not functioned for 2 days. On examination, she is tender over the upper abdomen and the abdomen is mildly distended. Plain abdominal radiograph reveals number of small loops in the centre of the abdomen. From the options below choose the ONE that you think is the most likely diagnosis in this patient: Single best answer question – choose ONE true option only. A. Acute colonic pseudo-obstruction B. Incarcerated incisional hernia C. Bacterial peritonitis D. Adhesional small bowel obstruction E. Sigmoid volvulus

ANSWER IS D Small bowel obstructions: make up 80-85% of all intestinal obstructions. Of these, adhesions accounts for nearly 90% of all small bowel obstructions. Adhesions usually develop following laparotomy and/or surgery to the bowel. It can occur as a sequlae of ‘minor’ abdominal surgeries such as appendicectomies or ‘major’ surgeries such as resection of large sections of the bowel.

Prepared by Dr: Mohammed Musa Brema Idress – My best wishes Page 225

In females, gynaecological procedures are an important cause; in addition, pelvic inflammatory disease can also lead to adhesions even in the absence of a surgical intervention in the abdomen. The cardinal features of small bowel obstruction are pain, vomiting and abdominal distension; untreated, this leads to constipation with reduction in flatus which then becomes absolute.

The pain is usually colicky due to excessive peristalsis, but may become continuous if strangulation or perforation occurs. Vomiting is early in high small bowel obstruction, late in low small bowel obstruction and delayed or absent in large bowel obstruction. The management involves appropriate resuscitation of the patient and surgical exploration of the abdomen to relieve the obstruction.

509. In a patient with bladder transitional cell carcinoma, the occupation most likely to have exposed them to a carcinogen is? Single best answer question – choose ONE true option only A. Painter B. Peanut farmer C. Printer D. Dye worker E. Miner

ANSWER IS D Beta-naphthylamine exposure is related to transitional cell carcinoma, and this is used in the dye industry. Painting and printing are linked to benzopyrene exposure and an increased risk of lung cancer. Mining is linked to chromium, asbestos, arsenic and nickel exposure, and increases the risk of lung cancer. Peanut farming causes exposure to aflatoxin and increases the risk of hepatocellular carcinoma.

510. 50-year-old man has an adrenal mass noted on CT scan. This is a metastasis. The most likely primary source is: Single best answer question – choose ONE true option only A. Kidney B. Lung C. Prostate D. Pancreas E. Colorectal

ANSWER IS B Tumours commonly metastasizing to the adrenal glands are lung and breast cancers Kidney and prostate more commonly metastasize to lung and bone, pancreas to liver and colorectal to lung and liver

511. A 70-year-old female presents with increasing pain in the right femur. X-rays reveal a single lytic lesion consistent with bony metastasis. Which of the following is LEAST likely to be the primary: Single best answer question – choose ONE correct option only? A. Colon

Prepared by Dr: Mohammed Musa Brema Idress – My best wishes Page 226

B. Breast C. Bronchus D. Kidney E. Thyroid

ANSWER IS A The five tumours commonly metastasizing to bone are breast, bronchus, kidney, thyroid and prostate, however prostate tend to cause sclerotic lesions. Colorectal cancer commonly metastasizes to lung and liver.

512. An 18-month-old presents with septic arthritis of the left hip. The most likely causative organism is? Single best answer question – choose ONE true option only A. Neisseria gonorrhoeae B. Haemophilus influenzae C. Staphylococcus aureus D. Haemolytic streptococcus E. Salmonella

ANSWER IS B Haemophilus influenza: is the most common cause of septic arthritis in infants <5 years, Staphylococcus aureus is the second commonest cause. Staphlococcus aureus is most common in children >5 years and adults >50 years, and second most common in adults <50 years.

In adults <50 years the commonest cause is Neisseria gonorrhoeae. Salmonella may be seen in sickle cell patients. Haemolytic streptococcus is a more common cause of osteomyelitis in children <5 years.

513. A 56 year old man from Egypt has suffered from recurrent attacks of haematuria for many years. He presents with suprapubic discomfort and at cystoscopy is found to have a mass lesion within the bladder. What is the most likely diagnosis? A. Squamous cell carcinoma B. Transitional cell papilloma C. Adenocarcinoma D. Leiomyosarcoma E. Rhabdomyosarcoma

ANSWER IS A In Egypt Schistosomiasis is more common than in the UK and may cause recurrent episodes of haematuria. In those affected with the condition who develop a bladder neoplasm, an SCC is the most common type. Bladder cancer Bladder cancer is the second most common urological cancer.It most commonly affects males aged between 50 and 80 years of age. Those who are current, or previous (within 20 years), smokers have a 25 fold increased risk of the disease.

Prepared by Dr: Mohammed Musa Brema Idress – My best wishes Page 227

Occupational exposure to hydrocarbons such as alanine increases the risk. Although rare in the UK, chronic bladder inflammation arising from Schistosomiasis infection remains a common cause of squamous cell carcinomas, in those countries where the disease is endemic.

Benign tumours Benign tumours of the bladder including inverted urothelial papilloma and nephrogenic adenoma are uncommon.

Bladder malignancies Transitional cell carcinoma (>90% of cases) • Squamous cell carcinoma ( 1-7% - except in regions affected by schistosomiasis), Adenocarcinoma (2%)

Transitional cell carcinomas may arise as solitary lesions, or may be multifocal, owing to the effect of "field change" within the urothelium. Up to 70% of TCC's will have a papillary growth pattern.

These tumours are usually superficial in location and accordingly have a better prognosis. The remaining tumours show either mixed papillary and solid growth or pure solid growths. These tumours are typically more prone to local invasion and may be of higher grade, the prognosis is therefore worse. Those with T3 disease or worse have a 30% (or higher) risk of regional or distant lymph node metastasis.

TNM Staging: - Stage description T0: No evidence of tumour Ta: Non invasive papillary carcinoma T1: Tumour invades sub epithelial connective tissue T2a: Inner half of detrusor invaded T2b: Outer half of detrusor invaded T3 : Tumour extends to perivesical fat T4a: Invasion of uterus, prostate or bowel T4b: Invasion of other abdominal organs N0: No nodal disease N1: Single lymph node metastasis (up to 2cm) N2: Single node >2cm or multiple nodes up to 5cm N3: Nodes over 5cm M1: Distant disease

Presentation - Most patients (85%) will present with painless, macroscopic haematuria. In those patients with incidental microscopic haematuria, up to 10% of females aged over 50 will be found to have a malignancy (once infection excluded).

Staging - Most will undergo a cystoscopy and biopsies or TURBT, this provides histological diagnosis and information relating to depth of invasion. Locoregional spread is

Prepared by Dr: Mohammed Musa Brema Idress – My best wishes Page 228

best determined using pelvic MRI and distant disease CT scanning. Nodes of uncertain significance may be investigated using PET CT.

Treatment - Those with superficial lesions may be managed using TURBT in isolation. Those with recurrences or higher grade/ risk on histology may be offered intravesical chemotherapy. Those with T2 disease are usually offered either surgery (radical cystectomy and ileal conduit) or radical radiotherapy. Prognosis T1 90% T2 60% T3 35% T4a 10-25% Any T, N1-N2 30%

514. A newborn infant is noted to have a unilateral cleft lip only. What is the most likely explanation for this process? A. Incomplete fusion of the second branchial arch B. Incomplete fusion of the nasolabial muscle rings C. Incomplete fusion of the first branchial arch D. Incomplete fusion of the third branchial arch E. Incomplete fusion of the secondary palate

ANSWER IS B Unilateral isolated cleft lip represents a failure of nasolabial ring fusion. It is not related to branchial arch fusion. Arch disorders have a far more profound phenotype and malformation sequences.

Cleft lip and palate Cleft lip and palate are the most common congenital deformity affecting the orofacial structures. Whilst they may be an isolated developmental malformation they are also a recognized component of more than 200 birth defects. The incidence is as high as 1 in 600 live births. The commonest variants are: A. Isolated cleft lip (15%) B. Isolated cleft palate (40%) C. Combined cleft lip and palate (45%)

The aetiology of the disorder is multifactorial; both genetic (affected first degree relative increases risk) and environmental factors play a role.

Cleft lip: occurs as a result of disruption of the muscles of the upper lip and nasolabial region. These muscles comprise a chain of muscles vs; nasolabial, bilabial and labiomental. Defects may be unilateral or bilateral.

Cleft palate: the primary palate consists of all anatomical structures anterior to the incisive foramen. The secondary palate lies more posteriorly and is sub divided into the hard and soft palate. Cleft palate occurs as a result of non fusion of the two palatine shelves. Both hard and soft palate may be involved. Complete cases are associated with complete separation of the nasal septum and vomer from the palatine processes. Treatment: Surgical reconstruction is the mainstay of management. The procedures are planned according to the extent of malformation and child age. Simple defects

Prepared by Dr: Mohammed Musa Brema Idress – My best wishes Page 229 are managed as a single procedure. Complex malformations are usually corrected in stages. Affected individuals have a higher incidence of hearing and speech problems.

515. A 72 year old woman with back pain and chronic renal failure has the following blood test results: Reference range Ca2+ 2.03 (2.15-2.55 mmol/l) Parathyroid hormone 10.4 (1-6.5 pmol/l ) Phosphate 0.80 (0.6-1.25 mmol/l). What is the most likely diagnosis? A. Hypoparathyroidism B. Primary hyperparathyroidism C. Secondary hyperparathyroidism D. Tertiary hyperparathyroidism E. Pseudohypoparathyroidism

ANSWER IS C In relation to secondary hyperparathyroidism; there is a HIGH PTH and the Ca2+ is NORMAL or LOW. In secondary hyperparathyroidism there in hyperplasia of the parathyroid glands in response to chronic hypocalcaemia (or hyperphosphataemia) and is a normal physiological response. Calcium is restored from bone, kidneys and the gastrointestinal system.

Differential diagnoses - It is important to consider the rare but relatively benign condition of benign familial hypocalciuric hypercalcaemia, caused by an autosomal dominant . Diagnosis - Is usually made by genetic testing and concordant biochemistry (Serum Calcium: Creatinine clearance ratio <0.01distinguished from primary hyperparathyroidism). Treatment Primary hyperparathyroidism Indications for surgery:- 1) Elevated serum Calcium > 1mg/dL above normal 2) Hypercalciuria > 400mg/day 3) Creatinine clearance < 30% compared with normal 4) Episode of life threatening hypercalcaemia 5) Nephrolithiasis 6) Age < 50 years • Neuromuscular symptoms 7) Reduction in bone mineral density of the femoral neck, lumbar spine, or distal radius of more than 2.5 standard deviations below peak bone mass (T score lower than -2.5) - Secondary hyperparathyroidism usually managed with medical therapy. Indications for surgery in secondary (renal) hyperparathyroidism: 1) Bone pain 2) Persistent pruritus 3) Soft tissue calcifications - Tertiary hyperparathyroidism Allow 12 months to elapse following transplant as many cases will resolve. The presence of an autonomously functioning parathyroid gland may require surgery. If the culprit gland can be identified then

Prepared by Dr: Mohammed Musa Brema Idress – My best wishes Page 230

it should be excised. Otherwise total parathyroidectomy and re-implantation of part of the gland may be required.

516. An 8 year old child presents with enlarged tonsils that meet in the midline and are covered with a white film that bleeds when you attempt to remove it. He is pyrexial but otherwise well. A. Infectious mononucleosis B. Acute bacterial tonsillitis C. Quinsy D. Lymphoma E. Diptheria

ANSWER IS B In acute tonsillitis the tonsils will often meet in the midline and may be covered with a membrane. Individuals who are systemically well are unlikely to have diptheria.

517. A 10 year old child presents with enlarged tonsils that meet in the midline. Oropharyngeal examination confirms this finding and you also notice peticheal haemorrhages affecting the oropharynx. On systemic examination he is noted to have splenomegaly. A. Infectious mononucleosis B. Acute bacterial tonsillitis C. Quinsy D. Lymphoma E. Diptheria

ANSWER IS A A combination of pharyngitis and tonsillitis is often seen in glandular fever. Antibiotics containing penicillin may produce a rash when given in this situation, leading to a mistaken label of allergy.

518. A 19 year old man has had a sore throat for the past 5 days. Over the past 24 hours he has notices increasing and severe throbbing pain in the region of his right tonsil. He is pyrexial and on examination he is noted to have a swelling of this area. A. Infectious mononucleosis B. Acute bacterial tonsillitis C. Quinsy D. Lymphoma E. Diptheria

ANSWER IS C Unilateral swelling and fever is usually indicative of quinsy. Surgical drainage usually produces prompt resolution of symptoms.

519. A 4 year old boy is brought to the clinic by his mother who has noticed a small lesion at the external angle of his eye. On examination there is a small cystic

Prepared by Dr: Mohammed Musa Brema Idress – My best wishes Page 231 structure which has obviously been recently infected. On removal of the scab, there is hair visible within the lesion. What is the most likely diagnosis? A. Dermoid cyst B. Desmoid cyst C. Sebaceous cyst D. Epidermoid cyst E. Keratoacanthoma

ANSWER IS A - Dermoid cysts occur at sites of embryonic fusion and may contain multiple cell types. They occur most often in children.

- The lesion is unlikely to be a desmoid cyst as these are seldom located either at this site or in this age group. In addition they do not contain hair. Sebaceous cysts will usually have a punctum and contain a cheesy material. Epidermoid cysts contain keratin plugs.

Dermoid cysts A cutaneous dermoid cyst may develop at sites of embryonic developmental fusion. They are most common in the midline of the neck, external angle of the eye and posterior to the pinna of the ear. They typically have multiple inclusions such as hair follicles that bud out from its walls. They may develop at other sites such as the ovary and in these sites are synonymous with teratomas.

A desmoid tumour Is a completely different entity and may be classified either as low grade fibrosarcomas or non aggressive fibrous tumours. They commonly present as large infiltrative masses. They may be divided into abdominal, extra abdominal and intra abdominal. All types share the same biological features. Extra abdominal desmoids have an equal sex distribution and primarily arise in the musculature of the shoulder, chest wall, back and thigh.

Abdominal desmoids usually arise in the musculoaponeurotic structures of the abdominal wall. Intra abdominal desmoids tend to occur in the mesentery or pelvic side walls and occur most frequently in patients with familial adenomatous polyposis coli syndrome.

520. Which of the following tumours are most likely to give rise to para-aortic nodal metastasis early? A. Ovarian B. Bladder C. Rectal D. Caecal E. Cervical

ANSWER IS A

Prepared by Dr: Mohammed Musa Brema Idress – My best wishes Page 232

Ovarian tumours are supplied by the ovarian vessels, these branch directly from the aorta. The cervix drains to the internal and external iliac nodes.

Para-aortic lymphadenopathy - Organ sites that may metastasize (early) to the para-aortic lymph nodes: A. Testis B. Ovary C. Uterine fundus - Many other organs may result in para-aortic nodal disease. However, these deposits will represent a much later stage of the disease, since other nodal stations are involved earlier.

521. Which of the following statements relating to malignant mesothelioma is false? A. It may be treated by extrapleural pneumonectomy. B. It is linked to asbestos exposure. C. It is linked to cigarette smoking independent of asbestos exposure. D. It may occur intra abdominally. E. It is relatively resistant to radiotherapy

ANSWER IS C It is not linked to cigarette smoking. When identified at an early stage a radical resection is the favored option. Radiotherapy is often given perioperatively. However, it is not a particularly radiosensitive tumour. Combination chemotherapy gives some of the best results and most regimes are cisplatin based.

Occupational cancers Occupational cancers accounted for 5.3% cancer deaths in 2005. In men the main cancers include: 1) Mesothelioma 2) Bladder cancer 3) Non melanoma skin cancer 4) Lung cancer 5) Sino nasal cancer Occupations with high levels of occupational tumours include: 1) Construction industry 2) Working with coal tar and pitch 3) Mining 4) Metalworkers 5) Working with asbestos (accounts for 98% of all mesotheliomas) 6) Working in rubber industry Shift work has been linked to breast cancer in women (Health and safety executive report RR595). The latency between exposure and disease is typically 15 years for solid tumours and 20 for leukaemia.

Many occupational cancers are otherwise rare. For example sinonasal cancer is an uncommon tumour, 50% will be SCC. They are linked to conditions such as wood

Prepared by Dr: Mohammed Musa Brema Idress – My best wishes Page 233 dust exposure and unlike lung cancer is not strongly linked to cigarette smoking. Another typical occupational tumour is angiosarcoma of the liver which is linked to working with vinyl chloride. Again in the non occupational context this is an extremely rare sporadic tumour.

522. A 33 year old man is recovering following a protracted stay on the intensive care unit recovering from an anastomotic leak following a difficult Trans-hiatal oesophagectomy. His progress is slow, and the intensive care doctors receive the following thyroid function test results: TSH 1.0 u/L Free T4 8 T3 1.0 (1.2-3.1 normal). A. Sick euthyroid B. Hyperthyroidism C. Hypothyroidism D. Normal euthyroid E. Anxiety state

ANSWER IS A Sick euthyroid syndrome is caused by systemic illness. With this, the patient may have an apparently low total and free T4 and T3, with a normal or low TSH. Note that the levels are only mildly below normal.

523. A 28 year old female presents to the general practitioner with symptoms of fever and diarrhoea. As part of her diagnostic evaluation the following thyroid function tests are obtained: TSH < 0.01 Free T4 30 T3 4.0. What is the diagnosis? A. Sick euthyroid B. Hyperthyroidism C. Hypothyroidism D. Normal euthyroid E. Anxiety state

ANSWER IS B The symptoms are suggestive of hyperthyroidism. This is supported by the abnormal blood results; suppressed TSH with an elevated T3 and T4.

Hyperthyroidism Causes of hyperthyroidism include: 1. Diffuse toxic goiter (Graves Disease) 2. Toxic nodular goiter 3. Toxic nodule • Rare causes

Graves disease: Characterized by a diffuse vascular goiter that appears at the same time as the clinical manifestations of hyperthyroidism. It is commonest in younger females and may be associated with eye signs. Thyrotoxic symptoms will predominate. Up to 50% of patients will have a familial history of autoimmune disorders. The glandular hypertrophy and hyperplasia occur as a result of the thyroid stimulating effects of the TSH receptor antibodies.

Prepared by Dr: Mohammed Musa Brema Idress – My best wishes Page 234

Toxic nodular goiter: In this disorder the goiter is present for a long period of time prior to the development of clinical symptoms. In most goiters the nodules are inactive and in some cases it is the internodular tissue that is responsible for the goiter.

Toxic nodule: Overactive, autonomously functioning nodule. It may occur as part of generalized nodularity or be a true toxic adenoma. The TSH levels are usually low as the autonomously functioning thyroid tissue will exert a negative feedback effect. Signs and symptoms: Symptoms Signs Lethargy Tachycardia Emotionally labile Agitation Heat intolerance Hot, moist palms Weight loss Exopthalmos Excessive appetite Thyroid goiter and bruit Palpitations Lid lag/retraction Diagnosis The most sensitive test for diagnosing hyperthyroidism is plasma T3 (which is raised). Note in hypothyroidism the plasma T4 and TSH are the most sensitive tests. A TSH level of <0.5U/L suggests hyperthyroidism. TSH receptor antibodies may be tested for in the diagnosis of Graves.

Treatment First line treatment for Graves disease is usually medical and the block and replace regime is the favored option. Carbimazole is administered at higher doses and thyroxin is administered orally. Patient are maintained on this regime for between 6 and 12 months. Attempts are then made to wean off medication. Where relapse then occurs the options are between ongoing medical therapy, radioiodine or surgery.

524. A 19 year old lady presents with palpitations. The medical officer takes a blood sample for thyroid function tests. The following results are obtained: TSH > 6.0 Free T4 20 T3 2.0. What is the diagnosis? A. Sick euthyroid B. Hyperthyroidism C. Hypothyroidism D. Anxiety state E. Factitious hyperthyroidism

ANSWER IS C An elevated TSH with normal T4 indicates partial thyroid failure. This is caused by Hashimotos, drugs (lithium, antithyroids) and dyshormogenesis.

525. Which of the following conditions is least likely to exhibit the Koebner phenomenon? A. Vitiligo B. Molluscum contagiosum C. Lichen planus D. Psoriasis E. Lupus vulgaris

Prepared by Dr: Mohammed Musa Brema Idress – My best wishes Page 235

ANSWER IS E Lupus vulgaris is not associated with the Koebner phenomenon.

Koebner phenomenon The Koebner phenomenon describes skin lesions which appear at the site of injury. It is seen in: 1. Psoriasis 2. Vitiligo 3. Warts • Lichen planus 4. Lichen sclerosus 5. Molluscum contagiosum

526. Which of the tumour types listed below is found most frequently in a person with aggressive fibromatosis? A. Medullary thyroid cancer B. Basal cell carcinoma of the skin C. Desmoid tumours D. Dermoid tumours E. Malignant melanoma

ANSWER IS C Aggressive fibromatosis - Aggressive fibromatosis is a disorder consisting of desmoid tumours, which behave in a locally aggressive manner. - Desmoid tumours may be identified in both abdominal and extra-abdominal locations. Metastatic disease is rare. The main risk factor (for abdominal desmoids) is having APC variant of familial adenomatous polyposis coli. Most cases are sporadic. - Treatment is by surgical excision.

527. 20 year old man is suspected of having an inflamed Meckels diverticulum. At which of the following locations is it most likely to be found? A. Approximately 60 cm distal to the ileo-caecal valve B. Approximately 60 cm proximal to the ileocaecal valve C. Approximately 200cm distal to the ileocaecal valve D. Approximately 200cm proximal to the ileocaecal valve E. 50cm distal to the DJ flexure

ANSWER IS B Rule of 2's 1) 2% of population 2) 2 inches (5cm) long 3) 2 feet (60 cm) from the ileocaecal valve 4) 2 x's more common in men 5) 2 tissue types involved They are typically found 2 feet proximal to the ileocaecal valve (or approximately 60cm).

Prepared by Dr: Mohammed Musa Brema Idress – My best wishes Page 236

Meckel's diverticulum Congenital abnormality resulting in incomplete obliteration of the vitello-intestinal duct. Normally, in the foetus, there is an attachment between the vitello-intestinal duct and the yolk sac. This disappears at 6 weeks gestation. The tip is free in majority of cases. Associated with enterocystomas, umbilical sinuses, and omphaloileal fistulas.

Arterial supply: omphalomesenteric artery. 2% of population, 2 inches long, 2 feet from the ileocaecal valve. Typically lined by ileal mucosa but ectopic gastric mucosa can occur, with the risk of peptic ulceration. Pancreatic and jejunal mucosa can also occur.

Clinical Normally asymptomatic and an incidental finding. Complications are the result of obstruction, ectopic tissue, or inflammation. Removal if narrow neck or symptomatic. Options are between wedge excision or formal small bowel resection and anastomosis.

528. A 43 year old lady presents with severe chest pain. Investigations demonstrate a dissecting aneurysm of the ascending aorta which originates at the aortic valve. What is the optimal long term treatment? A. Endovascular stent B. Medical therapy with beta blockers C. Medical therapy with ACE inhibitors D. Sutured aortic repair E. Aortic root replacement

ANSWER IS E Proximal aortic dissections are generally managed with surgical aortic root replacement. The proximal origin of the dissection together with chest pain (which may occur in all types of aortic dissection) raises concerns about the possibility of coronary ostial involvement (which precludes stenting). There is no role for attempted suture repair in this situation.

Aortic dissection More common than rupture of the abdominal aorta, 33% of patients die within the first 24 hours, and 50% die within 48 hours if no treatment received Associated with hypertension. Features of aortic dissection: tear in the intimal layer, followed by formation and propagation of a subintimal hematoma. Cystic medial necrosis (Marfan's). Most common site of dissection: 90% occurring within 10 centimeters of the aortic valve

Stanford Classification Type Location Treatment A Ascending aorta/ aortic root Surgery- aortic root replacement B Descending aorta Medical therapy with antihypertensives DeBakey classification Type Site affected I Ascending aorta, aortic arch, descending aorta II Ascending aorta only III Descending aorta distal to left subclavian artery

Prepared by Dr: Mohammed Musa Brema Idress – My best wishes Page 237

Clinical features Tearing, sudden onset chest pain (painless 10%) • Hypertension or Hypotension A blood pressure difference greater than 20 mm Hg. Neurologic deficits (20%)

Investigations CXR: widened mediastinum, abnormal aortic knob, ring sign, deviation trachea/oesophagus. CT (spiral), MRI, Angiography (95% of patients diagnosed)

Management Beta-blockers: aim HR 60-80 bpm and systolic BP 100-120 mm Hg. Urgent surgical intervention: type A dissections. This will usually involve aortic root replacement.

529. A 45 year old lady presents with a pathological fracture of her femoral shaft. She is a poor historian, but it transpires that she underwent a thyroidectomy 1 year previously. She has no other illness or co morbidities. What is the most likely underlying diagnosis? A. Hyperparathyroidism B. Metastatic papillary carcinoma of the thyroid C. Metastatic medullary carcinoma of the thyroid D. Metastatic follicular carcinoma of the thyroid E. None of the above

ANSWER IS D Follicular carcinomas are a recognized cause of bone metastasis. Papillary lesions typically spread via the lymphatics.

Thyroid malignancy Papillary carcinoma Commonest sub-type. Accurately diagnosed on fine needle aspiration cytology Histologically they may demonstrate psammoma bodies (areas of calcification) and so called 'orphan Annie' nuclei. They typically metastasize via the lymphatics and thus laterally located apparently ectopic thyroid tissue is usually a metastasis from a well differentiated papillary carcinoma.

Follicular carcinoma Are less common than papillary lesions. Like papillary tumours they may present as a discrete nodule. Although they appear to be well encapsulated macroscopically there invasion on microscopic evaluation. Lymph node metastases are uncommon and these tumours tend to spread haematogenously. This translates into a higher mortality rate. Follicular lesions cannot be accurately diagnosed on fine needle aspiration cytology and thus all follicular FNA's will require at least a hemi thyroidectomy.

Anaplastic carcinoma Less common and tend to occur in elderly females. Disease is usually advanced at presentation and often only palliative decompression and radiotherapy can be offered.

Prepared by Dr: Mohammed Musa Brema Idress – My best wishes Page 238

Medullary carcinoma These are tumours of the parafollicular cells ( C Cells) and are of neural crest origin. The serum calcitonin may be elevated which is of use when monitoring for recurrence. They may be familial and occur as part of the MEN -2A disease spectrum. Spread may be either lymphatic or haematogenous and as these tumours are not derived primarily from thyroid cells they are not responsive to radioiodine.

Lymphoma These respond well to radiotherapy. Radical surgery is unnecessary once the disease has been diagnosed on biopsy material. Such biopsy material is not generated by an FNA and thus a core biopsy has to be obtained (with care!).

530. A 35 year old Singaporean female attends a varicose vein pre operative clinic. On auscultation a mid diastolic murmur is noted at the apex. The murmur is enhanced when the patient lies in the left lateral position. What is the most likely cause of the cardiac murmur in the patient? A. Pulmonary stenosis B. Mitral regurgitation C. Tricuspid regurgitation D. Aortic stenosis E. Mitral stenosis F. Aortic sclerosis

ANSWER IS E A mid diastolic murmur at the apex is a classical description of a mitral stenosis murmur. The most common cause is rheumatic heart disease.

Complications of mitral stenosis include atrial fibrillation, stroke, myocardial infarction and infective endocarditis.

531. A 22 year old intravenous drug user is found to have a femoral abscess. The nursing staff contact the on call doctor as the patient has a temperature of 39oC. He is found to have a pan systolic murmur loudest at the left sternal edge at the 4th intercostal space. What is the most likely cause of the cardiac murmur in the patient? A. Pulmonary stenosis B. Mitral regurgitation C. Tricuspid regurgitation D. Aortic stenosis E. Mitral stenosis F. Aortic sclerosis

ANSWER IS C Intravenous drug users are at high risk of right sided cardiac valvular endocarditis. The character of the murmur fits with a diagnosis of tricuspid valve endocarditis.

Prepared by Dr: Mohammed Musa Brema Idress – My best wishes Page 239

532. An 83 year old woman is admitted with a left intertrochanteric neck of femur fracture. On examination the patient is found to have an ejection systolic murmur loudest in the aortic region. There is no radiation of the murmur to the carotid arteries. Her ECG is normal. What is the most likely cause of the cardiac murmur in the patient? A. Pulmonary stenosis B. Mitral regurgitation C. Tricuspid regurgitation D. Aortic stenosis E. Mitral stenosis F. Aortic sclerosis

ANSWER IS F The most likely diagnosis is aortic sclerosis. The main differential diagnosis is of aortic stenosis, however as there is no radiation of the murmur to the carotids and the ECG is normal, and this is less likely.

Cardiac murmurs Type of Murmur Conditions 1- Ejection systolic: Aortic stenosis, Pulmonary stenosis, HOCM, ASD, Fallot's 2- Pan-systolic: Mitral regurgitation, Tricuspid regurgitation, VSD 3- Late systolic: Mitral valve prolapsed, Coarctation of aorta 4- Early diastolic: Aortic regurgitation 5- Graham-Steel murmur: (pulmonary regurgitation) 6- Mid diastolic: Mitral stenosis 7- Austin-Flint murmur (severe aortic regurgitation)

533. A 55 year old lady presents with discomfort in the right breast. On clinical examination a small lesion is identified and clinical appearances suggest fibroadenoma. Imaging confirms the presence of a fibroadenoma alone. A core biopsy is taken, this confirms the presence of the fibroadenoma. However, the pathologist notices that a small area of lobular carcinoma in situ is also present in the biopsy. What is the best management? A. Whole breast irradiation B. Simple mastectomy C. Mastectomy and sentinal lymph node biopsy D. Wide local excision and sentinel lymph node biopsy E. Breast MRI scan

ANSWER IS E Lobular carcinoma in situ has a low association with invasive malignancy. It is seldom associated with microcalcification and therefore MRI is the best tool for determining disease extent. Resection of in situ disease is not generally recommended and most surgeons would simply pursue a policy of close clinical and radiological follow up.

Prepared by Dr: Mohammed Musa Brema Idress – My best wishes Page 240

Lobular carcinoma of the breast Lobular breast cancers are less common than their ductal counterparts. They typically present differently, the mass is usually more diffuse and less obvious on the usual imaging modalities of ultrasound and mammography. This is significant since the disease may be under staged resulting in inadequate treatment when wide local excision is undertaken. In women with invasive lobular carcinoma it is usually safest to perform an MRI scan of the breast, if breast conserving surgery is planned.

Lobular carcinomas are also more likely to be multifocal and metastasize to the contralateral breast. Lobular carcinoma in situ is occasionally diagnosed incidentally on core biopsies. Unlike DCIS, lobular carcinoma in situ is far less strongly associated with foci of invasion and is usually managed by close monitoring.

534. Which one of the following may be associated with an increased risk of venous thromboembolism? A. Diabetes B. Cannula C. Hyperthyroidism D. Tamoxifen E. Amiodarone

ANSWER IS D - Consider thromboembolism in breast cancer patients on tamoxifen! Venous thromboembolism Risk factors Common predisposing factors include malignancy, pregnancy and the period following an operation. The comprehensive list below is partly based on the 2010 SIGN venous thromboembolism (VTE) guidelines: General Increased risk with advancing age, obesity, family history of VTE, pregnancy (especially puerperium), immobility, hospitalization, anaesthesia, central venous catheter: femoral >> subclavian

Underlying conditions Malignancy, thrombophilia: e.g. Activated protein C resistance, protein C and S deficiency, heart failure, antiphospholipid syndrome, Behcet's, polycythaemia, nephrotic syndrome, sickle cell disease, paroxysmal nocturnal haemoglobinuria, hyperviscosity syndrome and homocystinuria

Medication Combined oral contraceptive pill: 3rd generation more than 2nd generation, hormone replacement therapy, raloxifene and tamoxifen, antipsychotics (especially olanzapine) have recently been shown to be a risk factor

Also state that the following are risk factors for recurrent VTE: - Previous unprovoked VTE, male sex, obesity, thrombophilias

Prepared by Dr: Mohammed Musa Brema Idress – My best wishes Page 241

535. A 42 year old lady is investigated for symptoms of irritability and altered bowel habit. On examination she is noted to have a smooth enlargement of the thyroid gland. As part of her investigations thyroid function tests are requested, these are as follows: TSH 0.1 mug/l Free T4 35 pmol/l. The most likely underlying diagnosis is: A. Multinodular goiter B. Follicular carcinoma of the thyroid gland C. Graves disease D. Pregnancy E. None of the above

ANSWER IS C TSH receptor antibodies will cause stimulation of the thyroid to synthesize T4. However, this will have a negative feedback effect on the pituitary causing decrease in TSH levels. Where hyperthyroidism occurs secondary to pregnancy the TSH is typically elevated.

Thyroid disease Patients may present with a number of different manifestations of thyroid disease. They can be broadly sub classified according to whether they are euthyroid or have clinical signs of thyroid dysfunction. In addition it needs to be established whether they have a mass or not.

Assessment History and Examination including USS. If a nodule is identified then it should be sampled ideally via an image guided fine needle aspiration. Radionucleotide scanning is of limited use

Thyroid Tumours Papillary carcinoma, Follicular carcinoma, Anaplastic carcinoma, Medullary carcinoma and Lymphoma's

Multinodular goiter One of the most common reasons for presentation. Provided the patient is euthyroid and asymptomatic and no discrete nodules are seen, they can be reassured. In those with compressive symptoms surgery is required and the best operation is a total thyroidectomy. Subtotal resections were practiced in the past and simply result in recurrent disease that requires a difficult revisional resection.

Endocrine dysfunction In general these patients are managed by physicians initially. Surgery may be offered alongside radio iodine for patients with Graves’s disease that fails with medical management or in patients who would prefer not to be irradiated (e.g. pregnant women). Patients with hypothyroidism do not generally get offered a thyroidectomy. Sometimes people inadvertently get offered resections during the early phase of Hashimotos thyroiditis, however, with time the toxic phase passes and patients can simply be managed with thyroxin.

Prepared by Dr: Mohammed Musa Brema Idress – My best wishes Page 242

Complications following surgery Anatomical such as recurrent laryngeal nerve damage. • Bleeding. Owing to the confined space haematoma's may rapidly lead to respiratory compromise owing to laryngeal oedema. Damage to the parathyroid glands resulting in hypocalcaemia.

536. Which of the following is not a risk factor for developing tuberculosis? A. Gastrectomy B. Solid organ transplantation with immunosupression C. Intravenous drug use D. Haematological malignancy E. Amiodarone

ANSWER IS E Risk factors for developing active tuberculosis include: Silicosis, chronic renal failure, HIV positive, solid organ transplantation with immunosuppression, intravenous drug use, haematological malignancy, anti-TNF treatment and previous gastrectomy

Tuberculosis Tuberculosis (TB) is an infection caused by Mycobacterium tuberculosis that most commonly affects the lungs. Understanding the pathophysiology of TB can be difficult - the key is to differentiate between primary and secondary disease.

Primary tuberculosis A non-immune host who is exposed to M. tuberculosis may develop primary infection of the lungs. A small lung lesion known as a Ghon focus develops. The Ghon focus is composed of tubercle-laden macrophages. The combination of a Ghon focus and hilar lymph nodes is known as a Ghon complex

In immunocompotent people the initially lesion usually heals by fibrosis. Those who are immunocompromized may develop disseminated disease (miliary tuberculosis).

Secondary (post-primary) tuberculosis If the host becomes immunocompromized the initial infection may become reactivated. Reactivation generally occurs in the apex of the lungs and may spread locally or to more distant sites. Possible causes of immunocomprize include: 1. Immunosuppressive drugs including steroids 2. HIV 3. Malnutrition - The lungs remain the most common site for secondary tuberculosis. Extra-pulmonary infection may occur in the following areas: 1. Central nervous system (tuberculous meningitis - the most serious complication) Vertebral bodies (Pott's disease) 2. Cervical lymph nodes (scrofuloderma) 3. Renal 4. Gastrointestinal tract.

Prepared by Dr: Mohammed Musa Brema Idress – My best wishes Page 243

537. Which of the following is not true of gastric cancer? A. There is an association with blood group A B. Adenocarcinoma is the most common subtype C. Individuals with histological evidence of signet ring cells have a lower incidence of lymph node metastasis D. Lymphomas account for 5% cases E. In Western Countries a more proximal disease distribution has been noted

ANSWER IS C Signet ring cells are features of poorly differentiated gastric cancer associated with a increased risk of metastatic disease.

Gastric cancer Overview There are 700,000 new cases of gastric cancer worldwide each year. It is most common in Japan and less common in western countries. It is more common in men and incidence rises with increasing age. The exact cause of many sporadic cancer is not known, however, familial cases do occur in HNPCC families. In addition, smoking and smoked or preserved foods increase the risk. Japanese migrants retain their increased risk (decreased in subsequent generations). The distribution of the disease in western countries is changing towards a more proximal location (perhaps due to rising obesity).

Pathology There is some evidence of support a stepwise progression of the disease through intestinal metaplasia progressing to atrophic gastritis and subsequent dysplasia, through to cancer. The favoured staging system is TNM. The risk of lymph node involvement is related to size and depth of invasion; early cancers confined to submucosa have a 20% incidence of lymph node metastasis.

Tumours of the gastro-oesophageal junction are classified as below: A. Type 1: True oesophageal cancers and may be associated with Barrett's oesophagus. B. Type 2: Carcinoma of the cardia, arising from cardiac type epithelium or short segments with intestinal metaplasia at the oesophagogastric junction. C. Type 3: Sub cardial cancers that spread across the junction. Involve similar nodal stations to gastric cancer.

Groups for close endoscopic monitoring A. Intestinal metaplasia of columnar type B. Atrophic gastritis • Low to medium grade dysplasia C. Patients who have previously undergone resections for benign peptic ulcer disease (except highly selective vagotomy). Staging CT scanning of the chest abdomen and pelvis is the routine first line staging investigation in most centers. Laparoscopy to identify occult peritoneal disease. PET CT (particularly for junctional tumours)

Prepared by Dr: Mohammed Musa Brema Idress – My best wishes Page 244

Treatment - Proximally sited disease greater than 5-10cm from the OG junction may be treated by subtotal gastrectomy. - Total gastrectomy if tumour is <5cm from OG junction • For type 2 junctional tumours (extending into oesophagus) oesophagogastrectomy is usual. - Endoscopic sub mucosal resection may play a role in early gastric cancer confined to the mucosa and perhaps the sub mucosa (this is debated) - Lymphadenectomy should be performed. - A D2 lymphadenectomy is widely advocated by the Japanese, the survival advantages of extended lymphadenectomy have been debated. However, the overall recommendation is that a D2 nodal dissection be undertaken. - Most patients will receive chemotherapy either pre or post operatively.

Prognosis UK Data - Disease extent Percentage 5 year survival - All RO resections 54% - Early gastric cancer 91% - Stage 1 87% - Stage 2 65% - Stage 3 18%

Operative procedure Total Gastrectomy , lymphadenectomy and Roux en Y anastomosis - General anaesthesia Prophylactic intravenous antibiotics Incision: Rooftop. Perform a thorough laparotomy to identify any occult disease. Mobilize the left lobe of the liver off the diaphragm and place a large pack over it. Insert a large self retaining retractor e.g. omnitract or Balfour (take time with this, the set up should be perfect). Pack the small bowel away. Begin by mobilizing the omentum off the transverse colon. Proceed to detach the short gastric vessels. Mobilize the pylorus and divide it at least 2cm distally using a linear cutter stapling device. - Continue the dissection into the lesser sac taking the lesser omentum and left gastric artery flush at its origin. The lymph nodes should be removed en bloc with the specimen where possible. - Place 2 stay sutures either side of the distal oesophagus. Ask the anaesthetist to pull back on the nasogastric tube. Divide the distal oesophagus and remove the stomach. - The oesphago jejunal anastomosis should be constructed. Identify the DJ flexure and bring a loop of jejunum up to the oesophagus (to check it will reach). Divide the jejunum at this point. Bring the divided jejunum either retrocolic or antecolic to the oesophagus. - Anastamose the oesophagus to the jejunum, using either interrupted 3/0 vicryl or a stapling device. - Then create the remainder of the Roux en Y reconstruction distally. Place a jejunostomy feeding tube. Wash out the abdomen and insert drains (usually the anastomosis and duodenal stump).

Prepared by Dr: Mohammed Musa Brema Idress – My best wishes Page 245

- Help the anaesthetist insert the nasogastric tube (carefully!) Close the abdomen and skin. Enteral feeding may commence on the first post-operative day. However, most surgeons will leave patients on free NG drainage for several days and keep them nil by mouth.

538. What is the most common presentation of a parotid gland tumour? A. Parapharyngeal mass B. Mass at anterior border of masseter C. Mass inferior to the angle of the mandible D. Mass behind the angle of the mandible E. Mass anterior to the ear

ANSWER IS D Parotid tumours may present at any region in the gland. However, most lesions will be located behind the angle of the mandible, inferior to the ear lobe. Tumours of the deep lobe of the parotid may present as a parapharyngeal mass and large lesions may displace the tonsil.

Parotid gland malignancy Most parotid neoplasms (80%) are benign lesions. Most commonly present with painless mass in cheek region. Up to 30% may present with pain, when this is associated with a discrete mass lesion in the parotid it usually indicates perineural invasion. Perineural invasion is very unlikely to occur in association with benign lesions 80% of patients with facial nerve weakness caused by parotid malignancies will have nodal metastasis and a 5 year survival of 25% Types of malignancy Mucoepidermoid carcinoma - 30% of all parotid malignancies usually low potential for local invasiveness and metastasis (depends mainly on grade)

Adenoid cystic carcinoma Unpredictable growth patter tendency for perineural spread Nerve growth may display skip lesions resulting in incomplete excision. Distant metastasis more common (visceral rather than nodal spread) 5 year survival 35%. Mixed tumours often a malignancy occurring in a previously benign parotid lesion Acinic cell carcinoma Intermediate grade malignancy May show perineural invasion Low potential for distant metastasis 5 year survival 80% Adenocarcinoma Develops from secretory portion of gland Risk of regional nodal and distant metastasis 5 year survival depends upon stage at presentation, may be up to 75% with small lesions with no nodal involvement Lymphoma Large rubbery lesion, may occur in association with Warthins tumours Diagnosis should be based on regional nodal biopsy rather than parotid resection Treatment is with chemotherapy (and radiotherapy)

539. A 52 year old male is referred to urology clinic with impotence. He is known to have hypertension. He does not have any morning erections. On further

Prepared by Dr: Mohammed Musa Brema Idress – My best wishes Page 246 questioning the patient reports pain in his buttocks, this worsens on mobilizing. On examination there is some muscle atrophy. The penis and scrotum are normal. What is the most likely diagnosis? A. Leriche syndrome B. S4-S5 cord lesion C. Pudendal nerve lesion D. Psychological impotence E. Beta blocker induced impotence

ANSWER IS A Leriche syndrome - Classically, it is described in male patients as a triad of symptoms: 1. Claudication of the buttocks and thighs 2. Atrophy of the musculature of the legs 3. Impotence (due to paralysis of the L1 nerve) Leriche syndrome, is atherosclerotic occlusive disease involving the abdominal aorta and/or both of the iliac arteries. - Management involves correcting underlying risk factors such as hypercholesterolaemia and stopping smoking. - Investigation is usually with angiography.

Leriche syndrome atheromatous disease involving the iliac vessels. Blood flow to the pelvic viscera is compromised. Patients may present with buttock claudication and impotence (in this particular syndrome). - Diagnostic work up will include angiography, where feasible iliac occlusions are usually treated with endovascular angioplasty and stent insertion.

540. A 23 year old man presents with diarrhoea and passage of mucous. He is suspected of having ulcerative colitis. Which of the following is least likely to be associated with this condition? A. Superficial mucosal inflammation in the colon B. Significant risk of dysplasia in long standing disease C. Episodes of large bowel obstruction during acute attacks D. Haemorrhage E. Disease sparing the anal canal

ANSWER IS C Large bowel obstruction is not a feature of UC, patients may develop megacolon. However, this is a different entity both diagnostically and clinically. Ulcerative colitis does not affect the anal canal and the anal transitional zone. Inflammation is superficial. Dysplasia can occur in 2% overall, but increases significantly if disease has been present over 20 years duration. Granulomas are features of crohn's disease.

Other features: Disease maximal in the rectum and may spread proximally, Contact bleeding, Longstanding UC crypt atrophy and metaplasia/dysplasia

Prepared by Dr: Mohammed Musa Brema Idress – My best wishes Page 247

Ulcerative colitis Ulcerative colitis is a form of inflammatory bowel disease. Inflammation always starts at rectum, does not spread beyond ileocaecal valve (although backwash ileitis may occur) and is continuous. The peak incidence of ulcerative colitis is in people aged 15-25 years and in those aged 55-65 years. It is less common in smokers. The initial presentation is usually following insidious and intermittent symptoms.

Features include: 1. Bloody diarrhoea 2. Urgency 3. Tenesmus 4. Abdominal pain, particularly in the left lower quadrant 5. Extra-intestinal features (see below)

Questions regarding the 'extra-intestinal' features of inflammatory bowel disease are common. Extraintestinal features include sclerosing cholangitis, iritis and ankylosing spondylitis. Common to both Crohn's disease (CD) and Ulcerative colitis (UC)

Notes Related to disease activity - Arthritis: pauciarticular, asymmetric Erythema nodosum Episcleritis Osteoporosis - Arthritis is the most common extraintestinal feature in both CD and UC Episcleritis is more common in Crohns disease

Unrelated to disease activity - Arthritis: polyarticular, symmetric Uveitis Pyoderma gangrenosum Clubbing Primary sclerosing cholangitis - Primary sclerosing cholangitis is much more common in UC Uveitis is more common in UC

Pathology Red, raw mucosa, bleeds easily, no inflammation beyond submucosa (unless fulminant disease), widespread superficial ulceration with preservation of adjacent mucosa which has the appearance of polyps ('pseudopolyps'), inflammatory cell infiltrate in lamina propria, neutrophils migrate through the walls of glands to form crypt abscesses, depletion of goblet cells and mucin from gland epithelium, granulomas are infrequent Barium enema Loss of haustrations, superficial ulceration, 'pseudopolyps', long standing disease: colon is narrow and short -'drainpipe colon' Endoscopy Superficial inflammation of the colonic and rectal mucosa, Continuous disease from rectum proximally, Superifical ulceration, mucosal islands, loss of vascular definition and continuous ulceration pattern.

Prepared by Dr: Mohammed Musa Brema Idress – My best wishes Page 248

Management Patients with long term disease are at increased risk of development of malignancy. Acute exacerbations are generally managed with steroids, in chronic patient agents such as azathioprine and infliximab may be used. Individuals with medically unresponsive disease usually require surgery- in the acute phase a sub total colectomy and end ileostomy. In the longer term a proctectomy will be required. An ileoanal pouch is an option for selected patients

541. A 23 year old lady undergoes a total thyroidectomy as treatment for a papillary carcinoma of the thyroid. The pathologist examines histological sections of the thyroid gland and identifies a psammoma body. What are these primarily composed of? A. Clusters of calcification B. Aggregations of neutrophils C. Aggregations of macrophages D. Giant cells surrounding the tumour E. Clusters of oxalate crystals

ANSWER IS A Psammoma bodies consist of clusters of microcalcification. They are most commonly seen in papillary carcinomas.

Thyroid malignancy Papillary carcinoma Commonest sub-type, Accurately diagnosed on fine needle aspiration cytology, Histologically they may demonstrate psammoma bodies (areas of calcification) and so called 'orphan Annie' nuclei. They typically metastasize via the lymphatics and thus laterally located apparently ectopic thyroid tissue is usually a metastasis from a well differentiated papillary carcinoma.

Follicular carcinoma Are less common than papillary lesions, Like papillary tumours they may present as a discrete nodule. Although they appear to be well encapsulated macroscopically there invasion on microscopic evaluation. Lymph node metastases are uncommon and these tumours tend to spread haematogenously. This translates into a higher mortality rate. Follicular lesions cannot be accurately diagnosed on fine needle aspiration cytology and thus all follicular FNA's will require at least a hemi thyroidectomy.

Anaplastic carcinoma Less common and tend to occur in elderly females, Disease is usually advanced at presentation and often only palliative decompression and radiotherapy can be offered.

Medullary carcinoma: These are tumours of the parafollicular cells ( C Cells) and are of neural crest origin. The serum calcitonin may be elevated which is of use when monitoring for recurrence.

Prepared by Dr: Mohammed Musa Brema Idress – My best wishes Page 249

They may be familial and occur as part of the MEN -2A disease spectrum. Spread may be either lymphatic or haematogenous and as these tumours are not derived primarily from thyroid cells they are not responsive to radioiodine.

Lymphoma These respond well to radiotherapy. Radical surgery is unnecessary once the disease has been diagnosed on biopsy material. Such biopsy material is not generated by an FNA and thus a core biopsy has to be obtained (with care!)

542. A 52 year old woman attends clinic for investigation of abdominal pain and constipation. On examination you note blue lines on the gum margin. She mentions that her legs have become weak in the past few days. What is the most likely diagnosis? A. Acute intermittent porphyria B. Lead poisoning C. Constipation D. Guillan Barre syndrome E. Rectal carcinoma

ANSWER IS B The combination of abdominal pain and a motor peripheral neuropathy, should indicate this diagnosis. The blue line along the gum margin can occur in up to 20% patients with lead poisoning.

Lead poisoning Along with acute intermittent porphyria, lead poisoning should be considered in questions giving a combination of abdominal pain and neurological signs

Features Abdominal pain, peripheral neuropathy (mainly motor), fatigue, constipation, blue lines on gum margin (only 20% of adult patients, very rare in children)

Investigations The blood lead level is usually used for diagnosis. Levels greater than 10 mcg/dl are considered significant, Full blood count: microcytic anaemia. Blood film shows red cell abnormalities including basophilic stippling and clover-leaf morphology, Raised serum and urine levels of delta aminolaevulinic acid may be seen making it sometimes difficult to differentiate from acute intermittent porphyria, Urinary coproporphyrin is also increased (urinary porphobilinogen and uroporphyrin levels are normal to slightly increased)

Management - various chelating agents are currently used: Dimercaptosuccinic acid (DMSA), D-penicillamine, EDTA, Dimercaprol

543. A 20 year old lady is referred to the vascular clinic. She has been feeling generally unwell for the past six weeks. She works as a typist and has noticed

Prepared by Dr: Mohammed Musa Brema Idress – My best wishes Page 250 increasing pain in her forearms whilst working. On examination she has absent upper limb pulses. Her ESR is measured and mildly elevated. What IS the most likely underlying diagnosis? A. Wegeners granulomatosis B. Polyarteritis nodosa C. Giant cell arteritis D. Takayasu's arteritis E. Buergers disease

ANSWER IS D Takayasus arteritis: may be divided into acute systemic phases and the chronic pulseless phase. In the latter part of the disease process the patient may complain of symptoms such as upper limb claudication. In the later stages of the condition the vessels will typically show changes of intimal proliferation, together with band fibrosis of the intima and media.

544. You are referred a patient by their general practitioner (GP). They underwent a renal transplantation 3 years ago. The patient has been complaining of headaches and dizzy spells. The GP reports an abrupt onset of unexplained hypertension. Select the most likely complication for this case. A. Acute graft rejection B. Chronic graft rejection C. Delayed graft function D. Lymphocele E. Renal artery stenosis

ANSWER IS E Renal artery stenosis complicates 2–10% of renal transplants. It may be confirmed on Doppler ultrasound. It can be managed using percutaneous angioplasty and stent placement.

545. You review a 50-year-old man in clinic. He had a renal transplantation 200 days ago. He feels tired but otherwise well. His renal function is deteriorating and he has developed proteinuria. Select the most likely complication for this case. A. Acute graft rejection B. Chronic graft rejection C. Delayed graft function D. Lymphocele E. Renal artery stenosis

ANSWER IS B This gentleman has chronic allograft nephropathy. It is characterized by slowly progressive graft dysfunction which leads to chronic renal failure. In addition many patients develop nephrotic-range proteinuria. Histopathological examination of a renal allograft biopsy may show varying combinations of lesions. These include: chronic transplant glomerulopathy, ischaemic glomerulopathy, interstitial fibrosis, tubular atrophy, arteriosclerosis and arteriolar hyalinosis.

Prepared by Dr: Mohammed Musa Brema Idress – My best wishes Page 251

546. A 25-year-old man is on the intensive therapy unit (ITU) following a severe head injury. He has been confirmed brainstem dead. He is a suitable candidate for organ donation but is not on the organ donation register (ODR). His relatives say he never expressed a wish to donate his organs. They themselves do not object and agree with donation. Select the most likely course of action for this case. A. Discuss organ donation with relatives B. Seek consent from the coroner C. Respect relatives wishes, unable to go ahead with organ harvest D. Invalid consent, unable to go ahead with organ harvest E. Not suitable for organ donation

ANSWER IS B The coroner would have to be contacted and if he consents then one can proceed with organ procurement.

547. Brainstem death is confirmed in a 34-year-old woman on ITU. She is on the organ donation register. You discuss this with her relatives and they object to organ donation. Select the most likely course of action for this case. A. Seek consent from the coroner B. Respect relatives wishes, unable to go ahead with organ harvest C. Invalid consent, unable to go ahead with organ harvest D. Not suitable for organ donation E. Valid consent, proceed with organ harvest.

ANSWER IS E The patient had expressed a wish to donate her organs by registering with the organ donation register (ODR). There is no evidence that the patient changed her mind before death. Her relatives holding an opposing view does not invalidate the consent. Time should be spent with the relatives fully informing them about organ donation and address any issues they have.

NB: It is very important that candidates realize that in reality, transplant teams are unlikely to proceed with organ harvesting unless the donor’s relatives are also agreeable to this. This scenario may arise in the MRCS Part B exam (e.g. as a communication station) and every effort must be made to provide information and seek agreement from the relatives, failing which, senior advice must be sought.

548. A 46-year-old man is on ITU following a road traffic collision. He has been confirmed brainstem dead. He is a suitable candidate for organ donation but is not on the register. He does not have any close relatives. A friend is not sure what his wishes would have been. Select the most likely course of action for this case. A. Seek consent from the coroner B. Respect relatives wishes, unable to go ahead with organ harvest C. Invalid consent, unable to go ahead with organ harvest D. Not suitable for organ donation E. Valid consent, proceed with organ harvest.

Prepared by Dr: Mohammed Musa Brema Idress – My best wishes Page 252

ANSWER IS A The Human Tissue Act (1961) states that only the person lawfully in possession of the body or his/her designated other can authorize the removal of organs or tissues from the body. It states: “The person lawfully in possession of the body has powers and duties in connection with removal of organs. The person authorizes the removal of any part from the body for the said purposes (therapeutic or medical education, or research) if having made such reasonable enquiry as may be practicable, he has no reason to believe: (a) that the deceased had expressed an objection to his body being so dealt with after his death, and had not withdrawn it; or (b) that the surviving spouse or any surviving relative of the deceased objects to the body being so dealt with.”

Where a deceased person is in an NHS hospital or organization, the person with control and management of the hospital (usually the chief executive/medical director) is the person lawfully in possession of the body until such time as it is claimed by the person who has right to possession for the purpose of disposal (usually the executor or next of kin of the patient) or, by reason of their statutory obligations, the coroner or procurator fiscal (Department of Health Working Party – Code of Practice for the Diagnosis of Brain Stem Death 1998). In situations where the cause of death was sudden, not due to natural causes or unexpected and/or the doctor has not treated the deceased during his/her last illness, the coroner/procurator fiscal must be informed (Coroner’s Act 1988).

The medical staff involved in the care of the patient should contact the coroner/procurator fiscal to obtain authorization for organ/tissue donation or the donor transplant co-coordinator/tissue co-coordinator may do this on behalf of the medical staff.

549. A 50-year-old woman on ITU is confirmed brainstem dead. She is on the ODR and her relatives have already expressed a wish that her organs are harvested. According to her relatives she is fit and well. In the past she has been ‘cured’ following treatment for early breast cancer, for which she was given the 'all clear' 7 years ago. Select the most likely course of action for this case. A. Seek consent from the coroner B. Respect relatives wishes, unable to go ahead with organ harvest C. Invalid consent, unable to go ahead with organ harvest D. Not suitable for organ donation E. Valid consent, proceed with organ harvest.

ANSWER IS E The Advisory Committee on the Safety of Blood, Tissues and Organs issued revised guidelines in April 2014. Organ donation is not permitted from patients with a history of metastatic cancer or any form of haematological malignancy (e.g. leukaemia or lymphoma) or cerebral lymphoma. Although certain tumours are considered „high risk‟ (e.g. melanoma, breast cancer, lung cancer, bowel cancer, renal cancer and sarcoma), individuals with these tumours may still be organ donors

Prepared by Dr: Mohammed Musa Brema Idress – My best wishes Page 253 if their cancer was at an early stage, and treated successfully without evidence of recurrence for over 5 years.

550. At the 6-month review following a liver transplant a patient says they feel unwell. He/she has been feeling tired, have a sore throat and night sweats. According to your notes he/she has also lost weight. Select the most likely diagnosis. A. Biliary obstruction B. Chronic graft rejection C. Hepatic artery thrombosis D. Hepatitis E. Post-transplant lymphoproliferative disorder.

ANSWER IS E Post-transplant lymphoproliferative disorders (PTLDs) may develop at any time after transplantation. They are usually associated with Epstein–Barr virus infection and are a consequence of the high level of immunosuppression achieved with current regimens. The usual management is to reduce the immunosuppression and treatment with acyclovir.

551. 5 days post-transplant a patient develops a high temperature and their liver function tests have deteriorated. Blood cultures are positive for Escherichia coli. Select the most likely diagnosis. A. Biliary obstruction B. Chronic graft rejection C. Hepatic artery thrombosis D. Hepatitis E. Post-transplant lymphoproliferative disorder

ANSWER IS C A high temperature, deranged liver function tests and positive blood cultures in the postoperative period following liver transplantation is virtually pathognomic of hepatic artery thrombosis. Doppler ultrasound is used to assess hepatic artery patency. Arteriography can be used if Doppler fails to visualize the artery. Patients presenting in the early postoperative period usually require retransplantation.

552. 2 days post-transplant a patient deteriorates and has a seizure. Blood tests show grossly deranged liver function tests and confirm a suspected coagulopathy. Select the most likely diagnosis. A. Biliary obstruction B. Chronic graft rejection C. Hepatic artery thrombosis D. Primary graft failure E. Post-transplant lymphoproliferative disorder

ANSWER IS D Approximately 7% of grafts undergo primary failure.

Prepared by Dr: Mohammed Musa Brema Idress – My best wishes Page 254

It is a very serious complication with a poor prognosis. Urgent re-transplantation is required.

553. A 25-year-old woman with cystic fibrosis underwent a heart and lung transplant 4 months ago, and is currently receiving triple immunotherapy. She attends the follow-up clinic for a routine check-up. She reports sore gums and excessive facial hair. On examination she is hypertensive. Urea & electrolyte results are as follows: Na+ 139 mmol/liter, K+ 5.7 mmol/liter, urea 9.9 mmol/liter, creatinine 140 µmol/liter. Select the most likely complication for this case. A. Acute rejection B. Arterial thrombosis C. Azathioprine side-effects D. Chronic rejection E. Cyclosporine side-effects

ANSWER IS E Cyclosporine A is an example of a calcineurin inhibitor. Cyclosporine-based triple immunosuppression with corticosteroids and azathioprine remains the most popular regimen in the UK. It is used prophylactically and therapeutically to address rejection following organ transplantation. Side effects of cyclosporine include nephrotoxicity, hypertension, hirsutism, tremor, gingival hyperplasia and hepatotoxicity. Long-term use increases the risk of development of malignancy (5% of patients), most commonly basal or squamous cell carcinomas.

554. A 58-year-old man had a liver transplant 7 weeks ago. He attends The Emergency Department with a 24-h history of malaise, fever and myalgia and respiratory distress going upstairs. He is currently receiving triple immunosuppression. On examination, he is unwell and dyspnoeic at rest. He has a pulse rate of 105/min, blood pressure is 95/60 mmHg, temperature is 38.3°C and respiratory rate is 28 breaths/min. Oxygen saturation is 90% on air. Select the most likely complication for this case. A. Azathioprine side-effects B. Cyclosporine side-effects C. Cytomegalovirus infection D. Graft-versus-host disease E. Malignancy

ANSWER IS C In addition to the development of malignancy, immunosuppression increases the risk of infection. Cytomegalovirus (CMV) is a member of the herpes group of viruses. Primary infection in a seronegative individual who receives a graft from a seropositive donor typically occurs 6 weeks post transplantation, and results in the most severe disease.

The main symptoms of CMV infection are usually non-specific and include fever, night sweats, fatigue and myalgia. Retinitis is pathognomonic, but rarely seen in the transplant population. Respiratory distress, noticed at first during exercise, may give

Prepared by Dr: Mohammed Musa Brema Idress – My best wishes Page 255 a clue to early CMV pneumonitis. Patients may also present with CMV encephalitis or gastrointestinal infection, characterized by dysphagia, diarrhoea, nausea and abdominal pain. Reactivation of latent CMV infection may also occur in immunosuppressed patients, although the infection is usually less severe.

555. A 32 year old man presents to the vascular clinic with symptoms of foot pain during exertion. He is a heavy smoker and has recently tried to stop smoking. On examination he has normal pulses to the level of the popliteal. However, foot pulses are absent. A diagnostic angiogram is performed which shows an abrupt cut off at the level of the anterior tibial artery, together with the formation of corkscrew shaped collateral vessels distally. What IS the most likely underlying diagnosis? A. Wegeners granulomatosis B. Polyarteritis nodosa C. Giant cell arteritis D. Takayasu's arteritis E. Buergers disease

ANSWER IS E Buergers disease: is most common in young male smokers. This demographic is changing in those areas where young female smokers are more common. In the acute lesion the internal elastic lamina of the vessels is usually intact. As the disease progresses the changes progress to hypercellular occlusive thrombus. Tortuous corkscrew collaterals may reconstitute patent segments of the distal tibial or pedal vessels.

556. A 78 year old man presents with symptoms of headaches and deteriorating vision. He notices that there is marked pain on the right hand side of his face when he combs his hair. What IS the most likely underlying diagnosis? A. Wegeners granulomatosis B. Polyarteritis nodosa C. Giant cell arteritis D. Takayasu's arteritis E. Buergers disease

ANSWER IS C Temporal arteritis: may present acutely with symptoms of headache and visual loss, or with a less acute clinical picture. Sight may be threatened and treatment with immunosupressants should be started promptly. The often requested temporal artery biopsy (which can be the bane of many surgeons) is often non diagnostic and unhelpful.

Vasculitis The vasculitides are a group of conditions characterized by inflammation of the blood vessel walls. This may, in turn, compromise vessel integrity. Constitutional symptoms may be present. Whilst certain disease subtypes are reported to affect specific vessels, there is often a degree of overlap clinically.

Prepared by Dr: Mohammed Musa Brema Idress – My best wishes Page 256

Vessel diameter and vasculitis classification Aorta and branches. Takayasu's arteritis, Buergers disease, Giant cell arteritis

Large and medium sized arteries, Buergers disease, Giant cell arteritis, Polyarteritis nodosa

Medium sized: muscular arteries, Polyarteritis nodosa, and Wegener’s granulomatosis Small muscular arteries, Wegener’s granulomatosis, Rheumatoid vasculitis

Specific conditions, Takyasu's arteritis, Inflammatory, obliterative arteritis affecting aorta and branches, Females> Males, Symptoms may include upper limb claudication

Clinical findings include diminished or absent pulses, ESR often affected during the acute phase Buergers disease, Segmental thrombotic occlusions of the small and medium sized lower limb vessels, Commonest in young male smokers, Proximal pulses usually present, but pedal pulses are lost. An acute hypercellular occlusive thrombus is often present, Tortuous corkscrew shaped collateral vessels may be seen on angiography

Giant cell arteritis: Systemic granulomatous arteritis that usually affects large and medium sized vessels, Females > Males, Temporal arteritis is commonest type, Granulomatous lesions may be seen on biopsy (although up to 50% are normal) Polyarteritis nodosa, Systemic necrotizing vasculitis affecting small and medium sized muscular arteries, Most common in populations with high prevalence of hepatitis B. Renal disease is seen in 70% cases, Angiography may show saccular or fusiform aneurysms and arterial stenoses

Wegener’s granulomatosis: Predominantly affects small and medium sized arteries Systemic necrotizing granulomatous vasculitis, Cutaneous vascular lesions may be seen (ulceration, nodules and purpura). Sinus imaging may show mucosal thickening and air fluid levels

Treatment: Conditions such as Buergers disease are markedly helped by smoking cessation. Immunosupression is the main treatment for vasculitides.

557. A 28 year old man presents with right upper quadrant pain and hydatid disease is suspected. Which of the following statements relating to the disease is untrue? A. First line treatment is with pentavalent antimony. B. Peritoneal contamination with active daughter cysts may complicate surgery. C. CT scanning of the liver may show a floating membrane. D. Biliary communication with the cysts may occur. E. It is caused by Echinococcus granulosus

ANSWER IS A

Prepared by Dr: Mohammed Musa Brema Idress – My best wishes Page 257

Drug treatment is with albendazole or mebendazole. Praziquantzel may be used in the pre operative stages.

Hydatid cysts Hydatid cysts are endemic in Mediterranean and Middle Eastern countries. They are caused by the tapeworm parasite Echinococcus granulosus. An outer fibrous capsule is formed containing multiple small daughter cysts. These cysts are allergens which precipitate a type 1 hypersensitivity reaction.

Clinical features are as follows: Up to 90% cysts occur in the liver and lungs, Can be asymptomatic, or symptomatic if cysts > 5cm in diameter. Morbidity caused by cyst bursting, infection and organ dysfunction (biliary, bronchial, renal and cerebrospinal fluid outflow obstruction) • In biliary rupture there may be the classical triad of; biliary colic, jaundice, and urticaria CT is the best investigation to differentiate hydatid cysts from amoebic and pyogenic cysts. Surgery is the mainstay of treatment (the cyst walls must not be ruptured during removal and the contents sterilized first).

558. A 33 year old man is involved in a road traffic accident. He is initially stable and transferred to the accident and emergency department. On arrival he is catheterized. One minute later he becomes hypotensive, with evidence of angioedema surrounding his penis. What is the most likely explanation for this event? A. Type V latex hypersensitivity reaction B. Type IV latex hypersensitivity reaction C. Type III latex hypersensitivity reaction D. Type I latex hypersensitivity reaction E. Type II latex hypersensitivity reaction

ANSWER IS D Sudden collapse and angioedema following exposure to latex (of which most urinary catheters are manufactured) suggests a type I hypersensitivity reaction.

559. A 43 year old female develops severe chest wall cellulitis following a mastectomy. On examination the skin is markedly erythematous. Which of the acute inflammatory mediators listed below is least likely to produce vasodilation? A. Complement component C5a B. Lysosomal compounds C. Histamine D. Serotonin E. Prostaglandins

ANSWER IS D Erythema is a classical feature of acute inflammation. Potent mediators of vascular dilatation include; histamine, prostaglandins, nitric oxide, platelet activating factor,

Prepared by Dr: Mohammed Musa Brema Idress – My best wishes Page 258 complement C5a (and C3a) and lysosomal compounds. Although serotonin is associated with acute inflammation it is a vasoconstrictor.

The effects of serotonin are dependent upon the state of the vessels in the tissues. Intact and healthy tissues and vessels will respond to a serotonin infusion with vasodilation (hence the flushing seen in carcinoid syndrome). In contrast it worsens cardiac ischaemia in myocardial infarcts when released from damaged platelets.

560. A 43 year old man presents with a 3 week history of malaise, sore throat, odynophagia and dysphagia. On examination he is found to have patchy white spots in his oropharynx. An upper GI endoscopy is performed and similar lesions are identified in the oesophagus. Which investigation is most likely to identify the underlying pathology in this case? A. Serum urea and electrolytes B. Oesophageal biopsy for culture C. Oesophageal biopsy for histology D. Glucose tolerance testing E. Viral serology

ANSWER IS E Oesophageal candidiasis is associated with immunosupression; mainly in patients on chemotherapy, with haematological malignancy, HIV or inhaled steroids. In patients with HIV, oesophageal candidiasis is part of the spectrum of AIDS defining illnesses and usually occurs when the CD4 count is less than 200. Others include PCP pneumonia and CMV infections.

Oesophageal candidiasis Characterized by white spots in the oropharynx with extension into the oesophagus. Associated with broad spectrum antibiotic usage, immunosupression and immunological disorders. Patients may present with oropharyngeal symptoms, odynophagia and dysphagia.

Treatment is directed both at the underlying cause (which should be investigated for) and with oral antifungal agents.

561. A 78 year old man has undergone a hemi-arthroplasty for a intracapsular hip fracture. Post operatively he develops electrolyte derangement and receives intravenous fluids. Over the previous 24 hours he develops marked abdominal distension. On examination he has a tense, tympanic abdomen which is not painful. A contrast enema shows flow of contrast through to the caecum and through the ileocaecal valve. What is the most likely cause of obstruction? A. Malignant obstruction B. Olgilvies syndrome C. Volvulus D. Diverticular stricture E. Ischaemic stricture

Prepared by Dr: Mohammed Musa Brema Idress – My best wishes Page 259

ANSWER IS B Patients with electrolyte disturbance and previous surgery may develop colonic pseudoobstruction (olgilvies syndrome). The diagnosis is made using a contrast enema and treatment is usually directed at the underlying cause with colonic decompression if indicated.

562. A 67 year old man has had multiple episodes with fever and left iliac fossa pain. These have usually resolved with courses of intravenous antibiotics. He is admitted with a history of increasing constipation and abdominal distension. A contrast x-ray is performed which shows flow of contrast to the sigmoid colon, here the contrast flows through a long narrow segment of colon into dilated proximal bowel. What is the most likely cause of obstruction? A. Malignant obstruction B. Olgilvies syndrome C. Volvulus D. Diverticular stricture E. Ischaemic stricture

ANSWER IS D The long history of left iliac fossa pain and development of bowel obstruction suggests a diverticular stricture. These may contain a malignancy and most will require resection.

563. A 78 year old lady from a nursing home is admitted with a 24 hour history of absolute constipation and abdominal pain. On examination she has a distended soft mass in her left iliac fossa. An x-ray is performed which shows a large dilated loop of bowel in the left iliac fossa which contains a fluid level. What is the most likely cause of obstruction? A. Malignant obstruction B. Olgilvies syndrome C. Volvulus D. Diverticular stricture E. Ischaemic stricture

ANSWER IS C Sigmoid volvulus may present with an asymmetrical mass in an elderly patient. It may contain a fluid level, visible on plain films.

Colonic obstruction - Cause Features Treatment: Cancer Usually insidious onset, History of progressive constipation, Systemic features (e.g. anaemia), Abdominal distension, Absence of bowel gas distal to site of obstruction - Establish diagnosis (e.g. contrast enema/ endoscopy) Laparotomy and resection, stenting, defunctioning colostomy or bypass Diverticular stricture: Usually history of previous acute diverticulitis, Long history of altered bowel habit, Evidence of diverticulosis on imaging or Once diagnosis established, usually surgical resection Colonic stenting should not be

Prepared by Dr: Mohammed Musa Brema Idress – My best wishes Page 260 endoscopy performed for benign disease

Volvulus: Twisting of bowel around its mesentery, Sigmoid colon affected in 76% cases. Patients usually present with abdominal pain, and constipation, Examination usually shows asymmetrical distension, Plain X-rays usually show massively dilated sigmoid colon, loss of haustra and "U" shape are typical, the loop may contain fluid levels

Initial treatment is to untwist the loop, a flexible sigmoidoscopy may be needed. Those with clinical evidence of ischaemia should undergo surgery Patient with recurrent volvulus should undergo resection

Acute colonic pseudoobstruction Symptoms and signs of large bowel obstruction with no lesion, Usually associated with metabolic disorders, Usually a cut off in the left colon (82% cases), Although abdomen tense and distended, it is usually not painful, All patients should undergo contrast enema (may be therapeutic!) - Colonoscopic decompression Correct metabolic disorders IV neostigmine Surgery

564. A 52 year old man with dyspepsia is found to have a duodenal ulcer. A CLO test is taken and is positive. Which statement relating to the likely causative organism is false? A. It is a gram negative organism B. It lives only on gastric type mucosa C. It may occupy areas of ectopic gastric metaplasia D. In patients who are colonized there is commonly evidence of fundal gastritis on endoscopy E. It produces a powerful urease that forms the basis of the Clo test

ANSWER IS D Helicobacter pylori accounts for >75% cases of duodenal ulceration. It may be diagnosed with either serology, microbiology, histology or CLO testing.

Helicobacter pylori rarely produces any typical features on endoscopy. Where infection is suspected the easiest course of action is to take an antral biopsy for Clo testing in the endoscopy suite.

Helicobacter Pylori Infection with Helicobacter Pylori is implicated in many cases of duodenal ulceration and up to 60% of patients with gastric ulceration. It is a gram negative, helical shaped rod with microaerophillic requirements. It has the ability to produce a urease enzyme that will hydrolyze urea resulting in the production of ammonia.

The effect of ammonia on antral G cells is to cause release of gastrin via a negative feedback loop. Once infection is established the organism releases enzymes that

Prepared by Dr: Mohammed Musa Brema Idress – My best wishes Page 261 disrupt the gastric mucous layer. Certain subtypes release cytotoxins cag A and vac A gene products.

The organism incites a classical chronic inflammatory process of the gastric epithelium. This accounts for the development of gastric ulcers. The mildly increased acidity may induce a process of duodenal gastric metaplasia. Whilst duodenal mucosa cannot be colonized by H-Pylori, mucosa that has undergone metaplastic change to the gastric epithelial type may be colonized by H- Pylori with subsequent inflammation and development of duodenitis and ulcers.

In patients who are colonized there is a 10-20% risk of peptic ulcer, 1-2% risk gastric cancer and <1% risk MALT lymphoma.

565. A 34 year old male presents with painful rectal bleeding and a fissure in ano is suspected. On examination he has an epithelial defect at the mucocutaenous defect that is located anteriorly. Approximately what proportion of patients with fissure in ano will present with this pattern of disease? A. 90% B. 10% C. 50% D. 25% E. 100%

ANSWER IS B Only a minority of patients with fissure in ano will have an anteriorly sited fissure. They are particularly rare in males and an anterior fissure in a man should prompt a search for an underlying cause.

Anal fissure Anal fissures are a common cause of painful, bright red, rectal bleeding. Most fissures are idiopathic and present as a painful mucocutaneous defect in the posterior midline (90% cases). Fissures are more likely to be anteriorly located in females, particularly if they are multiparous. Multiple fissures and those which are located at other sites are more likely to be due to an underlying cause. Diseases associated with fissure in ano include: A. Crohns disease B. Tuberculosis C. Internal rectal prolapse Diagnosis: In most cases the defect can be visualized as a posterior midline epithelial defect. Where symptoms are highly suggestive of the condition and examination findings are unclear an examination under anaesthesia may be helpful. Atypical disease presentation should be investigated with colonoscopy and EUA with biopsies of the area. Treatment Stool softeners are important as the hard stools may tear the epithelium and result in recurrent symptoms.

Prepared by Dr: Mohammed Musa Brema Idress – My best wishes Page 262

The most effective first line agents are topically applied GTN (0.2%) or Diltiazem (2%) paste. Side effects of diltiazem are better tolerated.

Resistant cases may benefit from injection of botulinum toxin or lateral internal sphincterotomy (beware in females). Advancement flaps may be used to treat resistant cases. Sphincterotomy produces the best healing rates. It is associated with incontinence to flatus in up to 10% of patients in the long term.

566. The pathogenesis of osteopetrosis is best explained by a defect in which of the following? A. Osteoclast function B. PTH receptors C. Osteoblast function D. Calcium resorption in proximal tubule E. Calcium absorption

ANSWER IS A Osteopetrosis Overview Aka marble bone disease, rare disorder of defective osteoclast function resulting in failure of normal bone resorption, stem cell transplant and interferon-gamma have been used for treatment

567. A 13 month old boy is brought to the surgical clinic by his mother because his left testicle is not located in the scrotum. At which of the following sites would the testicle be located if it were an ectopic testis? A. Canalicular B. Inguinal C. External inguinal ring D. Superficial inguinal pouch E. High scrotal

ANSWER IS D Ectopic testes: are those that come to lie outside the normal range of embryological descent (i.e. in the superficial inguinal pouch). Other sites of ectopic testes include; base of penis, femoral and perineal.

Testicular disorders-paediatric Testicular disorders Testicular disorders are some of the commonest conditions present in paediatric urological practice.

Cryptorchidism The embryological descent of the testicle from within the abdominal cavity may be subject to a number of variations. Distinctions need to be made clinically from a non descended testis and a testis that is retractile. Testis that lie outside the normal path of embryological descent are termed ectopic testis. Undescended testis occurs in 1%

Prepared by Dr: Mohammed Musa Brema Idress – My best wishes Page 263 of male infants. Where the testis does not lie in an intra scrotal location, its location should be ascertained. Where both testes are absent the infant may be intersex.

MRI scanning may reveal intra-abdominal testes; however a GA is often needed to perform this investigation in this age group. Testes that are undescended should be placed in the scrotum after 1 year of age as the testosterone surge that may facilitate descent occurs at 6 months of age. Where the testes lie distally e.g. Superficial inguinal pouch an open orchidopexy is the procedure of choice. With abdominal testes a laparoscopy should be performed. The risk of seminoma is increased in individuals with a non descended testes and this risk is not reduced by orchidopexy.

Testicular torsion Typically the patient has severe sudden onset of scrotal pain. The difficulty in paediatric practice is the lack of clear history. On examination the testis is tender and enlarged. Management is by surgical exploration. Delay beyond 6 hours is associated with low salvage rates. A torted hyatid produces pain that is far more localized and the testis itself should feel normal. However, diagnostic doubt often exists and in such cases surgical exploration is warranted.

568. A splenectomy increases the risk of infection from all the following organisms except? A. Pneumococcus B. Klebsiella C. Haemophilus influenzae D. Staphylococcus aureus E. Neisseria meningitidis

ANSWER IS D Staphylococcus aureus infection following splenectomy is no more common than in non splenectomised individuals. The other organisms are encapsulated, which is why they are more likely to cause overwhelming post splenectomy sepsis.

Post splenectomy sepsis The loss of splenic function renders individuals at increased risk of fulminant sepsis. Young children are at the highest risk, especially in the first 2 years following surgery. Surgery for trauma is associated with a lower risk than when splenectomy is performed as a treatment for haematological disorders. Infection with encapsulated organisms poses the greatest risk, these organisms may be opsonised, but this then goes undetected at an immunological level due to loss of the spleen.

Prophylactic vaccinations are usually administered to reduce the risk of pneumococcal septicaemia. Since the vaccine only covers up to 80% of pneumococcal infections, patients will usually receive long term, low dose penicillin prophylaxis in addition to vaccination.

Prepared by Dr: Mohammed Musa Brema Idress – My best wishes Page 264

569. Which of the following is not an extraintestinal feature Crohns disease? A. Iritis B. Clubbing C. Aphthous ulcers D. Erythema multiforme E. Pyoderma gangrenosum

ANSWER IS D Extraintestinal manifestation of inflammatory bowel disease: A PIE SAC Aphthous ulcers, Pyoderma gangrenosum, Iritis, Erythema nodosum, Sclerosing cholangitis, Arthritis and Clubbing

Crohns disease Crohns disease is a chronic transmural inflammation of a segment(s) of the gastrointestinal tract and may be associated with extra intestinal manifestations. Frequent disease patterns observed include ileal, ileocolic and colonic disease. Peri- anal disease may occur in association with any of these. The disease is often discontinuous in its distribution. Inflammation may cause ulceration, fissures, fistulas and fibrosis with stricturing. Histology reveals a chronic inflammatory infiltrate that is usually patchy and transmural.

Ulcerative colitis Vs Crohns Crohn's disease Ulcerative colitis Distribution Mouth to anus Rectum and colon Macroscopic changes Cobble stone appearance, apthoid ulceration Contact bleeding Depth of disease Transmural inflammation Superficial inflammation Distribution pattern Patchy Continuous Histological features Granulomas (non caseating epithelioid cell aggregates with Langhans' giant cells) Crypt abscesses, Inflammatory cells in the lamina propria

Extraintestinal manifestations of Crohns Related to disease extent Unrelated to disease extent Aphthous ulcers (10%) Sacroiliiitis (10-15%) Erythema nodosum (5- 10%) Ankylosing spondylitis (1-2%) Pyoderma gangrenosum (0.5%) Primary sclerosing cholangitis (Rare). Acute arthropathy (6-12%) Gallstones (up to 30%) Ocular complications (up to 10%) Renal calculi (up to 10%)

570. Which of the following is not considered a risk factor for the development of oesophageal malignancy? A. Oesophageal metaplasia B. Smoking C. Excessive intake of alcoholic spirits D. Achalasia E. Blood group O

ANSWER IS E Blood group O is not a risk factor for oesophageal cancer. Achalasia is associated with the risk of developing squamous cell carcinoma of the oesophagus.

Prepared by Dr: Mohammed Musa Brema Idress – My best wishes Page 265

Oesophageal cancer Incidence is increasing, In most cases in the Western world this increase is accounted for by a rise in the number of cases of adenocarcinoma. In the UK adenocarcinomas account for 65% of cases.

Barrett’s oesophagus is a major risk factor for most cases of oesophageal adenocarcinoma. In other regions of the world squamous cancer is more common and is linked to smoking, alcohol intake, diets rich in nitrosamines and achalasia. Surveillance of Barrett’s is important as it imparts a 30 fold increase in cancer risk and if invasive malignancy is diagnosed early then survival may approach 85% at 5 years.

Diagnosis Upper GI endoscopy is the first line test, Contrast swallow may be of benefit in classifying benign motility disorders but has no place in the assessment of tumours, Staging is initially undertaken with CT scanning of the chest, abdomen and pelvis. If overt metastatic disease is identified using this modality then further complex imaging is unnecessary, If CT does not show metastatic disease, then local stage may be more accurately assessed by use of endoscopic ultrasound. Staging laparoscopy is performed to detect occult peritoneal disease. PET CT is performed in those with negative laparoscopy. Thoracoscopy is not routinely performed.

Treatment - Operable disease is best managed by surgical resection. The most standard procedure is an Ivor- Lewis type oesophagectomy. This procedure involves the mobilization of the stomach and division of the oesophageal hiatus. The abdomen is closed and a right sided thoracotomy performed. The stomach is brought into the chest and the oesophagus mobilized further. An intrathoracic oesophagogastric anastomosis is constructed.

Alternative surgical strategies include a trans-hiatal resection (for distal lesions), a left thoraco-abdominal resection (difficult access due to thoracic aorta) and a total oesophagectomy (McKeown) with a cervical oesophagogastric anastomosis. The biggest surgical challenge is that of anastomotic leak, with an intrathoracic anastomosis this will result in mediastinitis. With high mortality. The McKeown technique has an intrinsically lower systemic insult in the event of anastmotic leakage.

- In addition to surgical resection many patients will be treated with adjuvant chemotherapy.

571. A 32 year old lady is diagnosed with medullary carcinoma of the thyroid and has undergone resection of the tumour. Which blood test is most commonly performed for the diagnosis or assessment of this thyroid disorder? A. Measurement of antibodies to TSH receptor B. Thyroid peroxidase antibodies C. Thyroglobulin antibodies

Prepared by Dr: Mohammed Musa Brema Idress – My best wishes Page 266

D. Serum calcitonin

ANSWER IS D Measurement of basal or stimulated calcitonin concentrations is used to assess the completeness of surgical resection, and is of use in detecting diseases recurrences during follow up.

572. A 20 year old lady has undergone a total thyroidectomy for a well differentiated papillary carcinoma. She attends clinic and is well and the surgeon wishes to screen for disease recurrence. Which blood test IS most commonly performed for the diagnosis or assessment of this thyroid disorder? A. Measurement of antibodies to TSH receptor B. Thyroid peroxidase antibodies C. Thyroglobulin antibodies D. Serum calcitonin

ANSWER IS C Antibodies to thyroglobulin, the major constituent of colloid and precursor of thyroid hormones may be elevated in those with metastatic or recurrent thyroid cancer. Results may be erronoeous in those with other thyroid disorders.

573. A 33 year old lady presents with a recently diagnosed goiter and a diagnosis of Hashimotos thyroiditis is suspected. Which blood test IS most commonly performed for the diagnosis or assessment of this thyroid disorder? A. Measurement of antibodies to TSH receptor B. Thyroid peroxidase antibodies C. Thyroglobulin antibodies D. Serum calcitonin

ANSWER IS B Antibodies to thyroid peroxidase are found in most patients with Graves’s disease or Hashimotos thyroiditis.

Blood testing in thyroid disease - Assay Usage Thyroid peroxidase (microsomal) antibodies • Found in autoimmune disease affecting the thyroid (Hashimotos 100%) and Graves (70%) - Antibodies to TSH receptor • Individuals with Graves disease (95%) - Thyroglobulin antibodies • Not useful for clinically distinguishing between different types of thyroid disease, may be used as part of thyroid cancer follow up - Calcitonin: Released from the parafollicular cells, Usually found in patient with medullary carcinoma of the thyroid

574. Which one of the following complications is least associated with ventricular septal defects? A. Right heart failure B. Aortic regurgitation

Prepared by Dr: Mohammed Musa Brema Idress – My best wishes Page 267

C. Eisenmenger's complex D. Infective endocarditis E. Atrial fibrillation

ANSWER IS E Atrial fibrillation is associated more with atrial septal defects Ventricular septal defect Ventricular septal defects are the most common cause of congenital heart disease. They close spontaneously in around 50% of cases. Non-congenital causes include post myocardial infarction Features Classically a pan-systolic murmur which is louder in smaller defects Complications 1. Aortic regurgitation* 2. Infective endocarditis 3. Eisenmenger's complex 4. Right heart failure

*aortic regurgitation is due to a poorly supported right coronary cusp resulting in cusp prolapse

575. A 24 year old man from Sudan presents with a lymphadenopathy and weight loss. A diagnosis of tuberculosis is suspected and a lymph node biopsy is performed. Staining with which of the agents below is most likely to facilitate identification of the causative organism? A. Gram stain B. Ziehl-Neelsen stain C. Von Kossa stain D. Van Gieson stain E. Masson Trichrome stain

ANSWER IS B Ziehl-Neelsen stain is typically used to identify mycobacteria. They are not stained in the Gram staining process. Van Gieson and Masson trichrome are histological staining methods for identification of connective tissues. The Von Kossa technique is useful for identifying tissue mineralization.

576. Which of the following is not a pathological feature of breast cancer? A. Resemblance to ductal epithelial cells B. Angiogenesis C. Nuclear pleomorphism D. Metastatic calcification E. Vascular invasion

ANSWER IS D Dystrophic calcification may be present in breast malignancy and is the basis for the breast screening programme.

Prepared by Dr: Mohammed Musa Brema Idress – My best wishes Page 268

Metastatic calcification is calcification which occurs in otherwise normal tissues, usually as a result of hypercalcaemia.

Invasive ductal carcinoma is the most common type of breast cancer, unless the tumour is very poorly differentiated there is usually some resemblance to ductal epithelial cells.

Breast cancer pathology The histological features of breast cancer depend upon the underlying diagnosis. The invasive component is usually comprised of ductal cells (unless it is an invasive lobular cancer). In situ lesions may co-exist (such as DCIS). Typical changes seen in conjunction with invasive breast cancer include: 1. Nuclear pleomorphism 2. Coarse chromatin 3. Angiogenesis 4. Invasion of the basement membrane 5. Dystrophic calcification (may be seen on mammography) 6. Abnormal mitoses 7. Vascular invasion 8. Lymph node metastasis - The primary tumour is graded on a scale of 1-3 where 1 is the most benign lesion and 3 the most poorly differentiated.

- Immunohistochemistry for oestrogen receptor and herceptin status is routinely performed.

- The grade, lymph node stage and size are combined to provide the Nottingham prognostic index.

577. A 69 year old male presents with haematuria. He worked in the textile industry. He has a left flank mass. A CT IVU shows a lesion of the left renal pelvis. What is the most likely underlying diagnosis? A. Renal cell carcinoma B. Renal transitional cell carcinoma C. Nephroblastoma D. Neuroblastoma E. Angiomyolipoma F. Renal squamous cell carcinoma G. Retroperitoneal fibrosis

ANSWER IS B TCC is a rare form of renal cancer, accounting for approximately 7% of all renal tumours. Risk factors include exposure to chemicals in the textile, plastic and rubber industry.

578. A 2 year old boy presents with a right renal mass. On examination he has an irregular mass arising from the right flank and is hypertensive. A CT scan shows a

Prepared by Dr: Mohammed Musa Brema Idress – My best wishes Page 269 non calcified irregular lesion affecting the apex of the right kidney and the right adrenal gland. What is the most likely underlying diagnosis? A. Renal cell carcinoma B. Renal transitional cell carcinoma C. Nephroblastoma D. Neuroblastoma E. Angiomyolipoma F. Renal squamous cell carcinoma G. Retroperitoneal fibrosis

ANSWER IS C Wilm's tumour of the kidney: is the most common renal tumour in children. Both nephroblastoma and neuroblastoma may occupy the adrenal and apex of the kidney. In the case of neuroblastoma the lesion will have arisen from the adrenal, in the case of nephroblastoma the lesion will have arisen from the kidney. Hypertension is more commonly associated with nephroblastoma. Neuroblastomas are usually calcified, whereas nephroblastomas are not and this may be of diagnostic usefulness pre operatively.

579. A 35 year old male presents with haematuria. He is found to have bilateral masses in the flanks. He has a history of epilepsy and learning disability. What is the most likely underlying diagnosis? A. Renal cell carcinoma B. Renal transitional cell carcinoma C. Nephroblastoma D. Neuroblastoma E. Angiomyolipoma F. Renal squamous cell carcinoma G. Retroperitoneal fibrosis

ANSWER IS E This patient has tuberous sclerosus. This is associated with angiomyolipoma, which is present in 60-80% patients. It is a benign lesion.

Angiomyolipoma: 80% of these hamartoma type lesions 50% of patients with lesions >4cm occur sporadically, the remainder are seen in those with tuberous sclerosis. Tumour is composed of blood vessels, smooth muscle and fat, Massive bleeding may occur in 10% of cases will have symptoms and will require surgical resection

580. An 18 month old boy presents with recurrent urinary tract infections. As part of the diagnostic work-up he is noted to have abnormal renal function. An ultrasound scan is performed and shows bilateral hydronephrosis. What is the most likely underlying diagnosis? A. Urethral valves B. Meatal stenosis C. Hydronephrosis D. Pelvico-ureteric junction obstruction

Prepared by Dr: Mohammed Musa Brema Idress – My best wishes Page 270

E. Benign prostatic hyperplasia

ANSWER IS A A posterior urethral valve is an obstructive, developmental uropathy that usually affects male infants (incidence 1 in 8000). Diagnostic features include bladder wall hypertrophy, hydronephrosis and bladder diverticula.

Urethral valves Posterior urethral valves are the commonest cause of infravesical outflow obstruction in males. They may be diagnosed on ante natal ultrasonography. Because the bladder has to develop high emptying pressures in utero the child may develop renal parenchymal damage. This translates to renal impairment noted in 70% of boys at presentation.

Treatment is with bladder catheterization. Endoscopic valvotomy is the definitive treatment of choice with cystoscopic and renal follow up.

581. At which of the following anatomical sites does dormant tuberculosis most frequently reactivate? A. Apex of the lung B. Base of the lung C. Brain D. Terminal ileum E. Lumbar spine

ANSWER IS A TB reactivation most commonly occurs at the lung apex. This site is better oxygenated than elsewhere allowing the mycobacteria to multiply more rapidly and then spread both locally and distantly.

582. What is the commonest tumour type encountered in the colon? A. Squamous cell carcinoma B. Adenocarcinoma C. Lymphoma D. Anaplastic carcinoma E. Sarcoma

ANSWER IS B Adenocarcinoma are the most common and typically arise as a result of the adenoma - carcinoma sequence.

Colorectal cancer Annually about 150,000 new cases are diagnosed and 50,000 deaths from the disease. About 75% will have sporadic disease and 25% will have a family history. Colorectal tumours comprise a spectrum of disease ranging from adenomas through to polyp cancers and frank malignancy. Polyps may be categorized into: neoplastic polyps, adenomatous polyps and non neoplastic polyps. The majority of adenomas

Prepared by Dr: Mohammed Musa Brema Idress – My best wishes Page 271 are polypoidal lesions, although flat lesions do occur and may prove to be dysplastic. Non-neoplastic polyps include hyperplastic, juvenile, hamartomatous, inflammatory, and lymphoid polyps, which have not generally been thought of as precursors of cancer.

Three characteristics of adenomas that correlate with malignant potential have been characterized. These include increased size, villous architecture and dysplasia. For this reason most polyps identified at colonoscopy should be removed. The transformation from polyp to cancer is described by the adenoma - carcinoma sequence and its principles should be appreciated. Essentially genetic changes accompany the transition from adenoma to carcinoma; key changes include APC, c- myc, K RAS mutations and p53 deletions.

583. Which of the following changes are most likely to be identified in the aortic wall of a 38 year old lady with a Marfans syndrome and a dissecting aortic aneurysm? A. Transmural aortitis B. Cystic medial necrosis C. Foamy macrophages D. Dense dystrophic calcification E. None of the above

ANSWER IS B Cystic medial necrosis ( or cystic medial degeneration) occurs when basophils and mucoid material lie in between the intimal elastic fibers of the aorta. It is typically found in the aortic degeneration of Marfans syndrome, but may also be seen in aortic degeneration in older adults.

Aortic dissection More common than rupture of the abdominal aorta, 33% of patients die within the first 24 hours, and 50% die within 48 hours if no treatment received. Associated with hypertension.

Features of aortic dissection: tear in the intimal layer, followed by formation and propagation of a subintimal hematoma. Cystic medial necrosis (Marfan's), Most common site of dissection: 90% occurring within 10 centimeters of the aortic valve

Stanford Classification Type Location Treatment A Ascending aorta/ aortic root Surgery- aortic root replacement B Descending aorta Medical therapy with antihypertensives

DeBakey classification Type Site affected I Ascending aorta, aortic arch, descending aorta II Ascending aorta only III Descending aorta distal to left subclavian artery

Clinical features Tearing, sudden onset chest pain (painless 10%), Hypertension or Hypotension, A blood pressure difference greater than 20 mm Hg, Neurologic deficits (20%)

Prepared by Dr: Mohammed Musa Brema Idress – My best wishes Page 272

Investigations CXR: widened mediastinum, abnormal aortic knob, ring sign, deviation trachea/oesophagus, CT (spiral), MRI, Angiography (95% of patients diagnosed)

Management Beta-blockers: aim HR 60-80 bpm and systolic BP 100-120 mm Hg. Urgent surgical intervention: type A dissections. This will usually involve aortic root replacement.

584. A male infant is born by emergency cesarean section at 39 weeks gestation for foetal distress. Soon after the birth the baby becomes progressively hypoxic and on examination is found to have a scaphoid abdomen. What is the most likely underlying diagnosis? A. Intestinal malrotation B. Hiatus hernia C. Foramen of Bochdalek hernia D. Foramen of Morgagni hernia E. Tracheooesphageal fistula

ANSWER IS C The finding of a scaphoid abdomen and respiratory distress suggests extensive intra thoracic herniation of the abdominal contents. This is seen most frequently with Bochdalek hernias. Morgagni hernias seldom present in such a dramatic fashion. The other options do not typically present with the symptoms and signs described.

Embryology of the diaphragm and diaphragmatic hernia:- Embryology The diaphragm is formed between the 5th and 7th weeks of gestation through the progressive fusion of the septum transversum, pleuroperitoneal folds and via lateral muscular ingrowth. The pleuroperitoneal membranes from which the diaphragm originates originate at somites located in cervical segments 3 to 5, which accounts for the long path taken by the phrenic nerve.

The components contribute to the following diaphragmatic segments: 1. Septum transversum 2. Central tendon 3. Pleuroperitoneal membranes 4. Parietal membranes surrounding viscera 5. Cervical somites C5 to C7 6. Muscular component of the diaphragm Diaphragmatic hernia: Type of hernia Features Morgagni Anteriorly located Minimal compromise on lung development Minimal signs on antenatal ultrasound Usually present later Usually good prognosis Bochdalek hernia Posteriorly located Larger defect Often diagnosed antenatally Associated with pulmonary hypoplasia Poor prognosis

The posterior hernias of Bochdalek: are the most common type and if not diagnosed antenatally will typically present soon after birth with respiratory distress. The

Prepared by Dr: Mohammed Musa Brema Idress – My best wishes Page 273 classical finding is that of a scaphoid abdomen on clinical examination because of herniation of the abdominal contents into the chest.

Bochdalek hernias are associated with a number of chromosomal abnormalities such as Trisomy 21 and 18. Infants have considerable respiratory distress due to hypoplasia of the developing lung. Historically this was considered to be due to direct compression of the lung by herniated viscera. This view over simplifies the situation and the pulmonary hypoplasia occurs concomitantly with the hernial development, rather than as a direct result of it.

The pulmonary hypoplasia is associated with pulmonary hypertension and abnormalities of pulmonary vasculature. The pulmonary hypertension renders infants at risk of right to left shunting (resulting in progressive and worsening hypoxia). Diagnostic work up of these infants includes chest x-rays/ abdominal ultrasound scans and cardiac echo. Surgery forms the mainstay of treatment and both thoracic and abdominal approaches may be utilized. Following reduction of the hernial contents a careful search needs to be made for a hernial sac as failure to recognize and correct this will result in a high recurrence rate. Smaller defects may be primarily closed, larger defects may require a patch to close the defect.

Malrotation of the viscera: is a recognized association and may require surgical correct at the same procedure (favoring an abdominal approach). The mortality rate is 50-75% and is related to the degree of lung compromise and age at presentation (considerably better in infants >24 hours old).

585. A 72 year old lady falls and lands on her left hip. She attends the emergency department and is given some paracetamol by the junior doctor and discharged. Several months later she presents with ongoing pain and discomfort of the hip. Avascular necrosis of the femoral head is suspected. Which of the following features is least likely to be present? A. Non union of the fracture B. Angiogenesis at the fracture site C. Increased numbers of fibroblasts at the fracture site D. Osteochondritis dissecans E. Apoptosis of osteoblasts

ANSWER IS E Apoptosis is not a feature of necrotic cell death. By this stage there would usually be attempted repair so angiogenesis and proliferation of fibroblasts would be expected. These cells may differentiate further to become osteoblasts which in turn will lay down new matrix.

Avascular necrosis Cellular death of bone components due to interruption of the blood supply, causing bone destruction, Main joints affected are hip, scaphoid, lunate and the talus. It is not the same as non union. The fracture has usually united. Radiological evidence is

Prepared by Dr: Mohammed Musa Brema Idress – My best wishes Page 274 slow to appear. Vascular ingrowth into the affected bone may occur. However, many joints will develop secondary osteoarthritis.

Causes P ancreatitis L upus A lcohol S teroids T rauma I diopathic, infection C aisson disease, collagen vascular disease R adiation, rheumatoid arthritis A myloid G aucher disease S ickle cell disease Presentation Usually pain, Often despite apparent fracture union.

Investigation MRI scanning will show changes earlier than plain films. Treatment In fractures at high risk sites anticipation is key. Early prompt and accurate reduction is essential. Non weight bearing may help to facilitate vascular regeneration. Joint replacement may be necessary, or even the preferred option (e.g. Hip in the elderly).

586. Which one of the following is least associated with the development of colorectal cancer in patients with ulcerative colitis? A. Unremitting disease B. Disease duration > 10 years C. Onset before 15 years old D. Poor compliance to treatment E. Disease confined to the rectum

ANSWER IS E Ulcerative colitis and colorectal cancer Overview Risk of colorectal cancer is 10-20 times that of general population, the increased risk is mainly related to chronic inflammation, worse prognosis than patients without ulcerative colitis (partly due to delayed diagnosis), lesions may be multifocal

Factors increasing risk of cancer 1) Disease duration > 10 years 2) Patients with pancolitis onset before 15 years old 3) Unremitting disease 4) Poor compliance to treatment

587. A 52 year old male presents with tearing central chest pain. On examination he has an aortic regurgitation murmur. An ECG shows ST elevation in leads II, III and aVF. What is the most likely cause of chest pain? A. Pulmonary embolism B. Anterior myocardial infarction C. Inferior myocardial infarction D. Proximal aortic dissection E. Distal aortic dissection F. Boerhaave Syndrome

Prepared by Dr: Mohammed Musa Brema Idress – My best wishes Page 275

G. Mallory Weiss tear H. Perforated gastric ulcer

ANSWER IS D An inferior myocardial infarction and AR murmur should raise suspicions of an ascending aorta dissection rather than an inferior myocardial infarction alone. Also the history is more suggestive of a dissection. Other features may include pericardial effusion, carotid dissection and absent subclavian pulse.

588. A 52 year old male presents with central chest pain and vomiting. He has drunk a bottle of vodka. On examination there is some mild crepitus in the epigastric region. What is the most likely cause of chest pain? A. Pulmonary embolism B. Anterior myocardial infarction C. Inferior myocardial infarction D. Proximal aortic dissection E. Distal aortic dissection F. Boerhaave Syndrome G. Mallory Weiss tear H. Perforated gastric ulcer

ANSWER IS F The Mackler triad for Boerhaave syndrome: vomiting, thoracic pain, subcutaneous emphysema. It commonly presents in middle aged men with a background of alcohol abuse.

589. A 52 year old male presents with central chest pain. On examination he has an mitral regurgitation murmur. An ECG shows ST elevation in leads V1 to V6. There is no ST elevation in leads II, III and aVF. What is the most likely cause of chest pain? A. Pulmonary embolism B. Anterior myocardial infarction C. Inferior myocardial infarction D. Proximal aortic dissection E. Distal aortic dissection F. Boerhaave Syndrome G. Mallory Weiss tear H. Perforated gastric ulcer

ANSWER IS B The most likely diagnosis is an anterior MI. As there are no ST changes in the inferior leads, aortic dissection is less likely.

Chest pain Aortic dissection This occurs when there is a flap or filling defect within the aortic intima. Blood tracks into the medial layer and splits the tissues with the subsequent creation of a false lumen. It most commonly occurs in the ascending aorta or just distal to the left

Prepared by Dr: Mohammed Musa Brema Idress – My best wishes Page 276 subclavian artery (less common). It is most common in Afro-carribean males aged 50-70 years.

Patients usually present with a tearing intrascapular pain, which may be similar to the pain of a myocardial infarct. The dissection may spread either proximally or distally with subsequent disruption to the arterial branches that are encountered. In the Stanford classification system the disease is classified into lesions with a proximal origin (Type A) and those that commence distal to the left subclavian (Type B).

Diagnosis may be suggested by a chest x-ray showing a widened mediastinum. Confirmation of the diagnosis is usually made by use of CT angiography, Proximal (Type A) lesions are usually treated surgically, and type B lesions are usually managed non operatively.

Pulmonary embolism Typically sudden onset of chest pain, haemoptysis, hypoxia and small pleural effusions may be present. Most patients will have an underlying deep vein thrombosis.

Diagnosis may be suggested by various ECG findings including S waves in lead I, Q waves in lead III and inverted T waves in lead III. Confirmation of the diagnosis is usually made through use of CT pulmonary angiography. Treatment is with anticoagulation, in those patients who develop a cardiac arrest or severe compromise from their PE, consideration may be given to thrombolysis.

Myocardial infarction Traditionally described as sudden onset of central, crushing chest pain. It may radiate into the neck and down the left arm. Signs of autonomic dysfunction may be present. The presenting features may be atypical in the elderly and those with diabetes.

Diagnosis is made through identification of new and usually dynamic ECG changes (and cardiac enzyme changes). Inferior and anterior infarcts may be distinguished by the presence of specific ECG changes (usually II, III and aVF for inferior, leads V1-V5 for anterior).

Treatment is with oral antiplatelet agents, primary coronary angioplasty and/ or thrombolysis.

Perforated peptic ulcer Patients usually develop sudden onset of epigastric abdominal pain, it may be soon followed by generalized abdominal pain. There may be features of antecedent abdominal discomfort, the pain of gastric ulcer is typically worse immediately after eating.

Prepared by Dr: Mohammed Musa Brema Idress – My best wishes Page 277

Diagnosis may be made by erect chest x-ray which may show a small amount of free intraabdominal air (very large amounts of air are more typically associated with colonic perforation).

Treatment is usually with a laparotomy, small defects may be excised and overlaid with an omental patch, and larger defects are best managed with a partial gastrectomy.

Boerhaaves syndrome Spontaneous rupture of the oesophagus that occurs as a result of repeated episodes of vomiting. The rupture is usually distally sited and on the left side. Patients usually give a history of sudden onset of severe chest pain that may complicate severe vomiting. Severe sepsis occurs secondary to mediastinitis. Diagnosis is CT contrast swallow. Treatment is with thoracotomy and lavage, if less than 12 hours after onset then primary repair is usually feasible, surgery delayed beyond 12 hours is best managed by insertion of a T tube to create a controlled fistula between oesophagus and skin. Delays beyond 24 hours are associated with a very high mortality rate.

590. A 3 month old boy is suspected of having hypospadias. At which of the following locations is the urethral opening most frequently located in boys suffering from the condition? A. On the distal ventral surface of the penis B. On the proximal ventral surface of the penis C. On the distal dorsal surface of the penis D. On the proximal dorsal surface of the penis E. At the base of the scrotum

ANSWER IS A The defect is located ventrally and most often distally. Proximally located urethral openings are well recognized. Circumcision may compromise reconstruction.

Hypospadias The urethral meatus opens on the ventral surface of the penis. There is also a ventral deficiency of the foreskin. The urethral meatus may open more proximally in the more severe variants. However, 75% of the openings are distally located. The incidence is 1 in 300 male births.

Features include: 1. Absent frenular artery 2. Ventrally opened glans 3. Skin tethering to hypoplastic urethra 4. Splayed columns of spongiosum tissue distal to the meatus 5. Deficiency of the foreskin ventrally

Management: - No routine cultural circumcisions, Urethroplasty, Penile reconstruction

Prepared by Dr: Mohammed Musa Brema Idress – My best wishes Page 278

- The foreskin is often utilized in the reconstructive process. In boys with very distal disease no treatment may be needed.

591. A 52 year old male attends for a preoperative assessment for an inguinal hernia repair. You notice that the chest x-ray shows a loculated left pleural effusion. On further questioning the patient reports that he worked as a builder 30 years ago. What is the most likely cause for the effusion? A. Asbestosis B. Pneumonia C. Mesothelioma D. Silicosis E. Left ventricular failure

ANSWER IS C This patient has a risk of asbestos exposure through his occupation as a builder. As there is latent period of 30 years and a complicated effusion, the most likely cause is mesothelioma.

Mesothelioma Features Dyspnoea, weight loss, chest wall pain, Clubbing, 30% present as painless pleural effusion Only 20% have pre-existing asbestosis, History of asbestos exposure in 85-90%, latent period of 30-40 years

Basics Malignancy of mesothelial cells of pleura, Metastases to contralateral lung and peritoneum • Right lung affected more often than left Management Investigation: pleural biopsy, CT Scanning, (PET Scanning if surgery considered). Symptomatic: Industrial compensation. Chemotherapy, Surgery if operable Prognosis: poor, median survival 12 months

592. A 64-year-old woman who is reviewed due to multiple non-healing leg ulcers. She reports feeling generally unwell for many months. Examination findings include a blood pressure of 138/72 mmHg, pulse 90 bpm, pale conjunctivae and poor dentition associated with bleeding gums. What is the most likely underlying diagnosis? A. Thyrotoxicosis B. Vitamin B12 deficiency C. Vitamin C deficiency D. Diabetes mellitus E. Sarcoidosis

ANSWER IS C Bleeding gums and poor healing are suggestive of vitamin C deficiency.

Prepared by Dr: Mohammed Musa Brema Idress – My best wishes Page 279

Vitamin C deficiency - Vitamin C deficiency (scurvy) leads to defective synthesis of collagen resulting in capillary fragility (bleeding tendency) and poor wound healing Features Gingivitis, loose teeth, poor wound healing, bleeding from gums, haematuria, epistaxis, general malaise

593. Which of the following is not a typical feature of neuropraxia? A. Transient delay in neuronal transmission B. Axonal degeneration distal to the site of injury C. Absence of neuroma formation D. Preservation of autonomic function E. Absence of axonal degeneration proximal to the site of injury

ANSWER IS B

594. A 44 year old lady presents with a pathological fracture of the left femur. She has previously undergone a renal transplant for end stage renal failure. Her blood test results are as follows: Serum Ca2+ 2.80, PTH 88 pg/ml, Phosphate 0.30. A surgeon decides to perform a parathyroidectomy on the basis of these results. When the glands are assessed histologically, which of the appearances is most likely to be identified? A. Metaplasia the gland B. Hypertrophy of the gland C. Hyperplasia of the gland D. Parathyroid carcinoma E. Necrosis of the parathyroid gland

ANSWER IS C This is likely to be a case of tertiary hyperparathyroidism (high Calcium, high PTH, low phosphate). Therefore the glands will be hyperplastic. Hypertrophy is not correct as this implies an increase in size without an increase in cellularity. - This mistake has cost many candidates marks in the MRCS exams over the years!

595. A 56 year old man presents with lethargy, haematuria and haemoptysis. On examination he is hypertensive and has a right loin mass. A CT scan shows a lesion affecting the upper pole of the right kidney, it has a small cystic centre. Which of the options below is the most likely diagnosis? A. Squamous cell carcinoma of the kidney B. Nephroblastoma C. Renal adenocarcinoma D. Transitional cell carcinoma of the kidney E. Polycystic kidney disease

ANSWER IS C Renal adenocarcinoma are the most common renal tumours.

Prepared by Dr: Mohammed Musa Brema Idress – My best wishes Page 280

These will typically affect the renal parenchyma. Transitional cell carcinoma will usually affect urothelial surfaces. Nephroblastoma would be very rare in this age group. Renal adenocarcinoma may produce cannon ball metastasis in the lung which cause haemoptysis, this is not a feature of PKD.

Renal tumours Renal cell carcinoma: Renal cell carcinoma is an adenocarcinoma of the renal cortex and is believed to arise from the proximal convoluted tubule. They are usually solid lesions, up to 20% may be multifocal, 20% may be calcified and 20% may have either a cystic component or be wholly cystic.

They are often circumscribed by a pseudocapsule of compressed normal renal tissue. Spread may occur either by direct extension into the adrenal gland, renal vein or surrounding fascia. More distant disease usually occurs via the haematogenous route to lung, bone or brain. Renal cell carcinoma comprise up to 85% of all renal malignancies. Males are more commonly affected than females and sporadic tumours typically affect patients in their sixth decade. Patients may present with a variety of symptoms including; haematuria (50%), loin pain (40%), mass (30%) and up to 25% may have symptoms of metastasis. Less than 10% have the classic triad of haematuria, pain and mass.

Investigation - Many cases will present as haematuria and be discovered during diagnostic work up. Benign renal tumours are rare, so renal masses should be investigated with multislice CT scanning. Some units will add and arterial and venous phase to the scan to demonstrate vascularity and evidence of caval ingrowth. - CT scanning of the chest and abdomen to detect distant disease should also be undertaken. - Routine bone scanning is not indicated in the absence of symptoms. - Biopsy should not be performed when a nephrectomy is planned but is mandatory before any ablative therapies are undertaken. - Assessment of the functioning of the contra lateral kidney.

Management - T1 lesions may be managed by partial nephrectomy and this gives equivalent oncological results to total radical nephrectomy. Partial nephrectomy may also be performed when there is inadequate reserve in the remaining kidney.

- For T2 lesions and above a radical nephrectomy is standard practice and this may be performed via a laparoscopic or open approach. Preoperative embolization is not indicated nor is resection of uninvolved adrenal glands. During surgery early venous control is mandatory to avoid shedding of tumour cells into the circulation.

- Patients with completely resected disease do not benefit from adjuvant therapy with either chemotherapy or biological agents. These should not be administered outside the setting of clinical trials.

Prepared by Dr: Mohammed Musa Brema Idress – My best wishes Page 281

- Patients with transitional cell cancer will require a nephroureterectomy with disconnection of the ureter at the bladder.

596. A 34-year-old man is taken immediately to theatre with aortic dissection. You note he is tall with pectus excavatum and arachnodactyly. His condition is primarily due to a defect in which one of the following proteins? A. Polycystin-1 B. Fibrillin C. Type IV collagen D. Type I collagen E. Elastin

ANSWER IS B Although fibrillin is the primary protein affected (due to a defect in the fibrillin-1 gene) it should be noted that fibrillin is used as a substrate of elastin.

Marfan's syndrome Marfan's syndrome is an autosomal dominant connective tissue disorder. It is caused by a defect in the fibrillin-1 gene on chromosome 15 and affects around 1 in 3,000 people.

Features Tall stature with arm span to height ratio > 1.05, high-arched palate, arachnodactyly pectus excavatum, pes planus, scoliosis of > 20 degrees, heart: dilation of the aortic sinuses (seen in 90%) which may lead to aortic aneurysm, aortic dissection, aortic regurgitation, mitral valve prolapse (75%), lungs: repeated pneumothoraces eyes: upwards lens dislocation (superotemporal ectopia lentis), blue sclera, myopia, dural ectasia (ballooning of the dural sac at the lumbosacral level)

The life expectancy of patients used to be around 40-50 years. With the advent of regular echocardiography monitoring and beta-blocker/ACE-inhibitor therapy this has improved significantly over recent years. Aortic dissection and other cardiovascular problems remain the leading cause of death however.

597. Which of the following are not typical of Lynch syndrome? A. It is inherited in an autosomal recessive manner B. Affected patients are more likely to develop right colon mucinous tumours than the general population C. Affected individuals have an 80% lifetime risk of colon cancer D. Endometrial cancer is seen in 80%. E. Gastric cancers are more common

ANSWER IS A Lynch syndrome is inherited in an autosomal dominant fashion. It is characterized by microsatellite instability in the DNA mismatch repair genes. Colonic tumours in

Prepared by Dr: Mohammed Musa Brema Idress – My best wishes Page 282 patients with Lynch syndrome are more likely to be right sided tumours and to be poorly differentiated.

598. An enthusiastic medical student approaches you with a list of questions about blood transfusion reactions. Which of her following points is incorrect? A. Graft versus host disease involves neutrophil proliferation B. Thrombocytopaenia may occur in women with a prior pregnancy C. IgA antibodies may cause blood pressure compromise during transfusion D. Hypocalcaemia can occur E. Iron overload can be avoided by chelation therapy

ANSWER IS A - Mnemonic for transfusion reactions: Got a bad unit - G raft vs. Host disease, O verload, T hrombocytopaenia - A lloimmunization - B lood pressure unstable, A cute haemolytic reaction, D elayed haemolytic reaction - U rticaria, N eutrophilia - I nfection, T ransfusion associated lung injury

GVHD results from lymphocytic proliferation. The patient's own lymphocytes are similar to the donor's lymphocytes, therefore don't perceive them as being foreign. The donor lymphocytes, however, sees the recipient lymphocytes as being foreign. Therefore they proliferate causing severe complications.

Thrombocytopaenia occurs a few days after transfusion and may resolve spontaneously. Patients with IGA antibodies need IgA deficient blood transfusions.

Blood transfusion reactions: Immune mediated Non immune mediated Pyrexia Hypocalcaemia Alloimmunization CCF Thrombocytopaenia Infections Transfusion associated lung injury Hyperkalaemia Graft vs Host disease Urticaria Acute or delayed haemolysis. ABO incompatibility and Rhesus incompatibility

Notes: GVHD: lymphocyte proliferation causing organ failure Transfusion associated lung injury: neutrophil mediated allergic pulmonary oedema ABO and Rhesus incompatibility: causes acute haemolytic transfusion reaction leading to agglutination and haemolysis

599. An 82 year old lady presents with a carcinoma of the caecum. Approximately what proportion of patients presenting with this diagnosis will have synchronous lesions? A. <1% B. 60% C. 50% D. 20% E. 5%

Prepared by Dr: Mohammed Musa Brema Idress – My best wishes Page 283

ANSWER IS E Synchronous colonic tumours are seen in 5% cases and all patients having a flexible sigmoidoscopy should have completion colonoscopy if tumours or polyps are found

Synchronous lesions may occur in up to 5% of patients with colorectal cancer. A full and complete lumenal study with either colonoscopy, CT cologram or barium enema is mandatory in all patients being considered for surgery.

600. A 28 year old lady presents with benign cyclical mastalgia. Which of the following is not a recognized treatment for the condition? A. Evening primrose oil B. Bromocriptine C. Methotrexate D. Danazol E. Tamoxifen

ANSWER IS C Surgical excision of tender breast tissue is inappropriate Methotrexate: is used for the treatment of breast cancer. Whilst the use of tamoxifen is of benefit other agents such as flaxseed oil or evening primrose oil should be tried first. Danazol is effective, but many women dislike the side effects.

Benign cyclical mastalgia Benign cyclical mastalgia is a common cause of breast pain in younger females. It varies in intensity according to the phase of the menstrual cycle. It is not associated with point tenderness of the chest wall (more likely to be Tietze's syndrome).

The underlying cause is difficult to pinpoint, examination should focus on identifying focal lesions (such as cysts) that may be treated to provide symptomatic benefit. Women should be advised to wear a supportive bra. Conservative treatments include flax seed oil and evening primrose oil. There is slightly more evidence in favor of flax seed oil, though neither has performed much better than placebo in RCT's.

Hormonal agents such as bromocriptine and danazol may be more effective. However, many women discontinue these therapies due to adverse effects.

601. A 39 year old lady has undergone surgery for breast cancer. As part of the histopathology report the pathologist provides the surgeon with a Nottingham Prognostic Index score of 6.4. He also states that the tumour size is 2cm. Which of the following inferences can be made in relation to this statement? A. The tumour is likely to be grade 1 B. Vascular invasion is present C. Lymph node metastasis are definitely present D. The tumour is oestrogen receptor positive E. None of the above

Prepared by Dr: Mohammed Musa Brema Idress – My best wishes Page 284

ANSWER IS C A score of this value is unlikely to be reached with a grade 1 tumour and a size of 2cm. Therefore lymph node metastasis is definitely present. In addition since the maximal score for lymph node metastasis is 3 the tumour is likely be of a higher grade (see below). The Nottingham Prognostic Index provides no information about oestrogen receptor status or the presence or absence of vascular invasion.

Nottingham prognostic index The Nottingham Prognostic Index can be used to give an indication of survival following breast cancer surgery. In this system the tumour size is weighted less heavily than other major prognostic parameters. Calculation of NPI: Tumour Size (S) x 0.2 + Lymph node score (N) + Grade score (G) NPI = (0.2 × S) + N + G Lymph nodes grade (N) : 0 node = 1, 1-3 nodes = 2, >3 nodes = 3 Tumour grade (G): grade 1= 1, grade 2= 2, grade3 = 3

Prognosis Score Percentage 5 year survival 2.0 to 2.4 = 93%, 2.5 to 3.4 = 85%, 3.5 to 5.4 = 70%, >5.4 = 50% This data was originally published in 1992. It should be emphasized that other factors such as vascular invasion and receptor status also impact on survival and are not included in this data and account for varying prognoses often cited in the literature.

602. In patients with multiple endocrine neoplasia type IIb which of the following clinical appearances is the patient most likely to display? A. Acromegalic facies B. Turners type features C. Profound kyphoscoliosis D. Multiple bony exostoses E. Marfanoid features

ANSWER IS E

603. A 32 year old man undergoes an appendicectomy. A large carcinoid tumour is identified and a completion right hemicolectomy is performed. He is well for several months and then develops symptoms of palpitations and facial flushing. Which of the following diagnostic markers should be requested? A. Alpha feto protein B. 5-Hydroxyindoleacetic acid C. Urinary catecholamines D. Urinary VMA measurements E. None of the above

ANSWER IS B 5 HIAA is the most commonly used diagnostic marker for carcinoid syndrome.

Prepared by Dr: Mohammed Musa Brema Idress – My best wishes Page 285

Carcinoid syndrome Carcinoid tumours secrete serotonin, Originate in neuroendocrine cells mainly in the intestine (midgut-distal ileum/appendix), Can occur in the rectum, bronchi, Hormonal symptoms mainly occur when disease spreads outside the bowel

Clinical features Onset: years - Flushing face - Palpitations - Tricuspid stenosis causing dyspnoea - Asthma - Severe diarrhoea (secretory, persists despite fasting)

Investigation 5-HIAA in a 24-hour urine collection - Scintigraphy - CT scan

Treatment Octreotide, Surgical removal

604. Which of the following breast tumours is most commonly associated with a risk of metastasis to the contralateral breast? A. Invasive ductal carcinoma B. Invasive lobular carcinoma C. Phyllodes tumour D. Pagets disease of the breast E. Atypical ductal hyperplasia

ANSWER IS B Risk of metastasis to the contralateral breast is a classical feature of invasive lobular carcinoma.

Lobular carcinoma of the breast Lobular breast cancers are less common than their ductal counterparts. They typically present differently, the mass is usually more diffuse and less obvious on the usual imaging modalities of ultrasound and mammography. This is significant since the disease may be under staged resulting in inadequate treatment when wide local excision is undertaken. In women with invasive lobular carcinoma it is usually safest to perform an MRI scan of the breast, if breast conserving surgery is planned.

Lobular carcinomas are also more likely to be multifocal and metastasize to the contralateral breast. Lobular carcinoma in situ is occasionally diagnosed incidentally on core biopsies. Unlike DCIS, lobular carcinoma in situ is far less strongly associated with foci of invasion and is usually managed by close monitoring.

605. With respect to oncogenes which statement is false? A. Mutations in oncogenes lead to cell survival B. Cells with oncogene mutations are resistant to cell necrosis C. Oncogene mutations must typically involve two allelic mutations for biological effects to become manifest D. May prevent cellular apoptosis E. Include MYC mutations in the development of Burkitts Lymphoma

Prepared by Dr: Mohammed Musa Brema Idress – My best wishes Page 286

ANSWER IS C

606. Which is the characteristic finding on a blood film post splenectomy? A. Stipple cell B. Tear drop cell C. Reticulocytes D. Howell-Jolly bodies E. Schistocyte

ANSWER IS D Blood film in hyposplenism: Howell-Jolly bodies Pappenheimer bodies Poikilocytes (Target cells) Erythrocyte containing siderotic granules Heinz bodies

Splenectomy Indications A. Trauma: 1/4 are iatrogenic B. Spontaneous rupture: EBV C. Hypersplenism: hereditary spherocytosis or elliptocytosis etc D. Malignancy: lymphoma or leukaemia E. Splenic cysts, hydatid cysts, splenic abscesses

Post splenectomy changes Platelets will rise first (therefore in ITP should be given after splenic artery clamped), Blood film will change over following weeks, Howell Jolly bodies will appear, Other blood film changes include target cells and Pappenheimer bodies, Increased risk of post splenectomy sepsis, therefore prophylactic antibiotics and pneumococcal vaccine should be given.

Post splenectomy sepsis Typically occurs with encapsulated organisms and Opsonisation occurs but then not recognized

607. Abdominal wound dehiscence A. In the immediate post operative period is a result of prolonged preoperative starvation B. Does not require operative repair C. Is treated by a tight abdominal corset post operatively D. Is more likely if the patient has had external beam radiation in the region of the incision E. Is preventable by good suture selection and technique

ANSWER IS: A. FALSE B. FALSE C. FALSE D. TRUE

Prepared by Dr: Mohammed Musa Brema Idress – My best wishes Page 287

E. TRUE Abdominal wound dehiscence can be prevented even in high risk patients by use of good technique adequate suture length and deep tension sutures if necessary. Pre- op starvation, if it has resulted in a degree of malnutrition may delay wound healing but is unlikely to cause wound dehiscence immediately post-op. It may require operative repair, especially if it is due to poor suturing technique. It should not be treated with corset

608. In patients with pseudomembranous colitis (True or False) A. Pigment-containing macrophages accumulate in the colonic mucosa B. Cholesterol gallstones are common C. There is crypt damage throughout the mucosal layer D. Aplain radiograph of the abdomen typically shows narrowed haustra E. Sigmoidoscopy shows a pale red pseudomembrane

ANSWER IS A. FALSE B. FALSE C. FALSE D. FALSE E. FALSE A pseudomembrane may form on the colonic mucosa in response to several types of colonic injury such as intestinal ischaemia and mercury poisoning. The commonest cause of pseudomembranous colitis is colonization of the colon by Clostridium difficile after treatment with broad-spectrum antibiotics. Patients are usually pyrexial and have profuse diarrhoea, although the passage of blood and mucus is not usual. Plain radiographs of the abdomen may be non-specific, but a distended colon with blunted haustra and an irregular edge are suggestive of pseudomembranous colitis. Areas of crypt damage tend to occur in foci of inflammation with the intervening mucosa appearing relatively undamaged. The sigmoidoscopic appearance is typically that of a yellowish-white membrane contrasting with the reddened colonic mucosa

609. Helicobacter pylori:- A. Is found in 10% of a healthy population B. Is associated with gastric cancer C. Is spread by the faeco-oral route D. Is associated with chronic gastritis E. Is a potent secretor of the enzyme urease F. Is eradicated by sucralfate alone G. Adhere to gastric mucosa in an alkaline layer H. Is associated with duodenal ulcer relapse True False

ANSWER IS A. FALSE B. TRUE C. TRUE

Prepared by Dr: Mohammed Musa Brema Idress – My best wishes Page 288

D. TRUE E. TRUE F. FALSE G. TRUE H. TRUE Helicobacter pylori is found in 50% of the healthy population and lives closely applied to the gastric epithelium under the mucus layer. It is transmitted by the faecal-oral, or oral-oral route. It is responsible for atrophic gastritis type B and duodenal ulceration, and implicated in development of gastric cancers. H. pylori secretes urease.

610. Which of the following are true? Small cell cancer of the lung: CHOOSE SINGLE BEST ANSWER. A. Accounts for more than 50% of all lung cancers B. Is common in elderly non-smokers C. Is frequently benign and localized at presentation D. May be associated with hypercalcemia E. Has a median survival of 5 years if treated appropriately

ANSWER IS D Small cell cancers of the lung account for less than one-quarter of all lung cancers. It is named after the small round cells that make up these tumours. Squamous cell carcinoma makes up nearly 35%, adenocarcinoma 20% and large cell carcinoma approximately 20% of all lung cancers.

Small cell carcinoma is a tumour of neuroendocrine origin that may exist in the classical oat cell form or as an intermediate cell type. Small cell lung cancer is almost always caused by smoking while adenocarcinoma is seen in elderly non-smokers. These tumours are highly malignant and are usually disseminated at presentation. Small cell lung cancers are often associated with ectopic hormone production and paraneoplastic syndromes such as adrenocorticotrophic hormone (ACTH) (Cushing’s syndrome), parathyroid hormone (hence serum hypercalcemia), syndrome of inappropriate antidiuretic hormone secretion (causes hyponatremia) and myasthenic syndrome. The median survival of patients with small cell cancer is 3 months if untreated and 12-18 months if treated.

611. A 75-year-old man with prostatism has a serum prostate-specific antigen (PSA) concentration of 15 ng/l. Which of the following statements is true with regard to this result? Single best answer question, choose ONE true option only A. It could be explained by prostatitis B. It is diagnostic of malignancy C. It is likely to be invalidated if he underwent a digital rectal examination 48 h before the blood sample was taken D. It is prognostically highly significant E. It is unremarkable in a man of this age

Prepared by Dr: Mohammed Musa Brema Idress – My best wishes Page 289

ANSWER IS A Prostatitis and acute urinary retention can both result in increased serum PSA concentrations; digital rectal examination has a minor and only transient effect. This level is certainly compatible with malignancy, but is not diagnostic of it. In general, the higher the PSA, the greater the likelihood of malignancy, but some patients with malignancy have normal levels (often taken as less than 4 ng/l, but are actually age- dependent). The absolute PSA concentration correlates poorly with prognosis in prostatic cancer.

612. Osteoporosis is most reliably diagnosed by which one of the following techniques? Single best answer question “choose ONE true option only A. Dual-energy X-ray absorptiometry (DEXA) B. Measurement of serum osteocalcin C. Measurement of urinary collagen telopeptide D. Quantitative computed tomography (CT) E. Quantitative ultrasonography

ANSWER IS A DEXA is currently the most precise and accurate method for diagnosing osteoporosis. Quantitative CT of the lumbar spine is less precise (and more expensive, and involves a greater exposure to radiation).

Quantitative ultrasound measurements on the calcaneum are being evaluated but have not been proven to be superior to DEXA. Biochemical indices of bone formation (e.g. osteocalcin) and breakdown (N- and C-terminal cross-linking telopeptides of type-1 collagen) are of value in monitoring the response of patients to treatment, but not for diagnosis.

613. A 70-year-old man with symptoms of prostatism has a serum prostate specific antigen (PSA) concentration of 20 ng/l (normal value, < 4 ng/l). Which one of the following statements about the clinical importance of this result is the most likely? Single best answer question “choose ONE true option only A. It could be elevated because a digital rectal examination was performed 48 h before the blood sample was taken B. It could be elevated because of cancer elsewhere in the urinary tract C. It is diagnostic of prostatic cancer D. It is more likely to reflect prostatic cancer than benign prostatic hypertrophy E. It is normal for a man of this age

ANSWER IS D PSA is specific to the prostate, although not for cancer. Its concentration in the serum increases with age and in benign prostatic hypertrophy, but a concentration this high is more likely to be due to cancer than benign disease. PSA increases only slightly, and transiently, after digital rectal examination.

Prepared by Dr: Mohammed Musa Brema Idress – My best wishes Page 290

614. A woman presented with diarrhoea that has persisted up to 2 weeks after cholecystectomy. What is the most likely cause of the diarrhoea? Single best answer question “choose ONE true option only A. Salmonella spp. B. Yersinia C. Campylobacter D. Bile acid malabsorption E. Lactose malabsorption

ANSWER IS D Bile acid malabsorption: is an under diagnosed cause of chronic diarrhoea and is a recognized cause of diarrhoea after cholecystectomy. Bile acids are present in increased concentrations in the colon (particularly deoxycholate and chenodeoxycholate) and lead to diarrhoea by causing both increased colonic secretions and motility.

Diagnosis is made with the selenium-75-homocholic acid taurine (SeHCAT) test, where radiolabelled bile acid analogue is administered; % age retention at 7 days is calculated (less than 19% retention is abnormal). Treatment of bile acid malabsorption is with cholestyramine, which binds and inactivates bile acids in the colon.

615. A 50-year-old woman, who is a non-smoker, presents with bilateral claudication of her arms when performing tasks such as using a paint-roller when decorating. She is noted to have absent radial pulses and be normotensive. Investigations reveal a raised ESR of 80 mm/h and a CRP of 30 mg/l. What is the most likely diagnosis? Single best answer question “choose ONE true option only A. Antiphospholipid antibody syndrome (APLS) B. Takayasu,s arteritis C. Giant-cell arteritis D. Polyarteritis nodosa (PAN) E. Berger’s disease

ANSWER IS B This would be an unusual presentation for APLS and the acute-phase response is not usually elevated. The patient has a large-vessel arteritis, hence Takayasu,s arteritis is likely. Giant-cell arteritis (GCA) is also a large-vessel arteritis but rarely involves the limb arteries. Berger’s disease is associated with smoking and is a small-vessel vasculitis.

616. In the investigation of a woman presenting with jaundice and itching, which one of the following would most strongly support the diagnosis of primary biliary cirrhosis (PBC)? Single best answer question “ choose ONE true option only A. Raised serum IgM B. Positive antinuclear antibodies C. Antimitochondrial antibodies (M2 pattern) D. Raised serum IgA

Prepared by Dr: Mohammed Musa Brema Idress – My best wishes Page 291

E. Anti-smooth muscle antibodies

ANSWER IS C M2 AMA is highly indicative of PBC. A polyclonal increase in IgM (sometimes associated with elevated IgG) is typical but not specific for PBC. Raised serum IgA and anti-smooth muscle antibodies would not support a diagnosis of PBC.

617. A 72-year-old man presents to A&E with acute retention of urine. He has a serum creatinine concentration of 522 mmol/l. Which of the following features would suggest a cause for his renal failure and suggest that it is longstanding? Single best answer question “ choose ONE true option only A. Anaemia B. Small kidneys on ultrasound examination C. High serum parathyroid hormone concentration D. Hypotension E. Hyperuricaemia

ANSWER IS B The kidneys are usually decreased in size in chronic renal failure, except when this is due to amyloid or polycystic disease. Hypertension can be both a cause and a consequence of renal failure. Uric acid nephropathy can cause renal failure but hyperuricaemia is usual in chronic renal failure, due to decreased excretion. Anaemia (due to decreased erythropoietin secretion) and secondary hyperparathyroidism (decreased calcitriol secretion causes hypocalcaemia) are features, not causes, of chronic renal failure.

618. A 21-year-old man takes advantage of his employer’s health benefits and undergoes a private health screen. Urinalysis is positive for protein. His serum urea and creatinine concentrations are normal. Which of the following would be the most important investigations to perform next? Single best answer question “ choose ONE true option only A. Creatinine clearance B. Measurement of urine protein immediately on getting out of bed in the morning C. Mid-stream urine for culture and sensitivity D. Urine protein electrophoresis E. 24-hour urine protein excretion

ANSWER IS B Proteinuria: can occur because of glomerular damage, tubular damage (hence the decreased absorption of the small amount of low molecular weight proteins that are normally filtered by the glomeruli), an increased concentration of low molecular weight proteins in the blood (e.g. immunoglobulin light chains in myeloma “ Bence Jones “ protein) and secretion of proteins into the urine (e.g. in urinary tract infection).

Prepared by Dr: Mohammed Musa Brema Idress – My best wishes Page 292

Tubular proteins and Bence Jones protein can be detected by electrophoresis of concentrated urine. However, particularly in a young person, orthostatic proteinuria should be excluded before proceeding to further investigation.

In this benign condition, protein may be present in the urine when the individual is erect, but not when they have been supine for a period.

619. A 24-year-old man is being provided with long-term parenteral nutrition for Crohns disease. His energy requirement is estimated to be approximately 597 J (2500 kcal)/24 h. Which of the following combinations of lipid emulsion and dextrose solutions would be most appropriate to provide his energy requirements? Single best answer question “ choose ONE true option only A. Dextrose 50% 500 ml, lipid emulsion 20% 500 ml B. Dextrose 50% 500 ml, lipid emulsion 20% 750 ml C. Dextrose 50% 500 ml, lipid emulsion 20% 1000 ml D. Dextrose 50% 750 ml, lipid emulsion 20% 500 ml E. Dextrose 50% 1000 ml, lipid emulsion 20% 500 ml

ANSWER IS D Dextrose 50% and 20% lipid emulsions each provide about 478 J (2000 kcal)/l. His requirements would thus be met by a combined total of about 1250 ml of these fluids. It is usually recommended that lipid should provide not more than half the total energy requirements: in the combination in option B, it would provide 60%; in D, 40%. The former is too high. The other combinations either provide too much or too little energy.

620. A 47-year-old man with a long history of alcohol abuse is admitted to hospital with acute abdominal pain and is diagnosed clinically as having acute pancreatitis. This is confirmed by finding a high serum amylase activity and by the results of an ultrasound scan. He has not had a similar illness before. His serum is seen to be lipaemic: serum triglyceride concentration is 26 mmol/l. Which of the following is the most likely cause of the hypertriglyceridaemia? Single best answer question “ choose ONE true option only A. Decreased pancreatic lipase secretion B. Deficiency of apolipoprotein C-II C. Deficiency of LDL (low-density lipoprotein) receptors D. Deficiency of lipoprotein lipase E. Increased synthesis of VLDL (very low-density lipoprotein)

ANSWER IS E Hypertriglyceridaemia: occurs frequently in association with a high alcohol intake, and is due to the increased synthesis of triglyceride for incorporation into VLDL in the liver. This is thought to be as a result of the altered redox state consequent on alcohol metabolism, which favours fat synthesis.

Lipoprotein lipase is responsible for the clearance of VLDL and chylomicrons from the plasma: it is activated by apolipoprotein C-II. Lipoprotein lipase deficiency and

Prepared by Dr: Mohammed Musa Brema Idress – My best wishes Page 293 apolipoprotein C-II deficiency are rare, inherited causes of severe hypertriglyceridaemia, which usually present in childhood.

LDL receptors are involved in the transport of cholesterol from the plasma to the interior of cells: deficiency is the cause of familial hypercholesterolaemia. Pancreatic lipase is normally secreted into the gut. Although high plasma activities are found in pancreatitis, this does not affect the metabolism of lipoproteins.

621. A 41-year-old patient complains of tinnitus. Which of the following statements is correct regarding the clinical physiology of the ear? Single best answer question “ choose ONE true option only A. High-frequency waves are detected in the scala tympani B. Low-frequency waves are detected in the scala vestibuli C. The scala media is filled with perilymph D. The scala media contains the organ of Corti E. Normal hearing frequency only ranges from 20 to 2000 Hz

ANSWER IS D Low-frequency waves are detected in the scala tympani, high-frequency waves are detected in the scala vestibuli. Both contain perilymph. The scala media is filled with potassium-rich endolymph. Normal hearing frequency ranges from 20 to 20,000 Hz.

622. In breast cancer, which receptor will indicate the likelihood of responding to antihormonal treatment such as Tamoxifen? Single best answer question “ choose ONE true option only A. Oestrogen Receptor B. Her II receptor C. Progesterone receptor D. Tam-I receptor E. CK5/6

ANSWER IS A The presence of Oestrogen receptor (ER) is detected using Immunohistochemistry and its presence indicates the likelihood of benefiting from antihormonal treatment such as Tamoxifen.

The use of such treatment reduces recurrence by nearly 50%. The presence of progesterone receptor (PR) reflects on the functionality of the ER. Therefore if the tumour is ER positive and PR positive it will respond better to antihormonal treatment compared to an ER positive PR negative tumour. Her II positive tumours are targeted using the new treatment Herceptin.

623. The commonest histological type of invasive breast cancer is? Single best answer question “ choose ONE true option only A. Lobular B. Tubular C. Mucinous

Prepared by Dr: Mohammed Musa Brema Idress – My best wishes Page 294

D. Medullary E. Ductal

ANSWER IS E 75% of invasive breast cancer are ductal no special type NST. They are called so because they don’t have a special acinar pattern. Though this type is called ductal it arises from the breast epithelium and not from the ducts. The second commonest type is lobular cancer which has the tendency to be bilateral. Tubular cancer makes up 3-5% of all invasive breast cancers, and when it is a pure tubular cancer (not mixed with other types of cancer) it does not have the tendency to metastasize.

624. The presence of Orphan Annie nuclei on histopathology is found in which type of thyroid cancer? Single best answer question “ choose ONE true option only A. Papillary carcinoma B. Follicular carcinoma C. Anaplastic carcinoma D. Medullary carcinoma E. Primary thyroid lymphoma

ANSWER IS A Orphan Annie nuclei are pale empty looking nuclei which are typically seen in cases of papillary adenocarcinoma of the thyroid. The papillary type is the most common thyroid cancer (70%). Most tumours are thyroid stimulating hormone (TSH) dependant. It is usually seen in younger patients with history of irradiation to the neck. This type can be multi-focal and can spread early to the lymph nodes, and can present as an enlarged lymph node. Follicular carcinoma is the second commonest malignant thyroid tumour while anaplastic carcinoma is the least common with the worst prognosis. Medullary carcinoma of C cells can be part of MEN syndrome.

625. In one of the following is NOT true? Single best answer question “ choose ONE true option only A. The causative organism is Trypanosoma brucei B. The organism destroys the ganglionic cells of the myenteric plexus C. Clinically it presents with a picture similar to achalasia D. It can mimic the presentation of malignancy E. It is associated with cardiomyopathy

ANSWER IS A The causative agent of Chagas disease (pseudo achalasia) is chronic infection with the parasite Trypanosoma cruzi which are found in Latin America mainly Brazil. The protozoan parasite trypanosome brucei along with trypanosome gambiensi cause sleeping sickness and is found in Africa.

This infection affects reticulo-endothelial system, muscles and nervous system. This is why Chagas disease is associated with cardiomyopathy, megacolon, megaureter and megaduodenum. Treatment is by cardiomyotomy, preferably laparoscopic

Prepared by Dr: Mohammed Musa Brema Idress – My best wishes Page 295 though in unfit patients the alternative treatment is endoscopic pneumatic dilatation or botulinum toxin injections.

626. The commonest underlying cause for meconium ileus in a newborn is? Single best answer question “ choose ONE true option only A. Hirschsprung,s disease B. Intussusceptions C. D. Imperforate anus E. Cystic fibrosis

ANSWER IS E Most newborn babies with meconium ileus have cystic fibrosis. Cystic fibrosis is an autosomal recessive disorder associated with the production of abnormal mucous with high salt content. Sweat chloride test is used to confirm the diagnosis, but it should be done after the 6th week of life. The main organs affected are the lungs and the pancreas. The pancreas undergoes dysplasia, so with the loss of the pancreatic enzymes and with the presence of abnormal mucous, thick meconium forms which causes intraluminal intestinal obstruction usually at the level of the terminal ileum. Affected bowel will be dilated and filled with heavy meconium making it prone to volvulus.

627. A patient diagnosed with thyroglossal cyst is going to be admitted for surgery. The most important preoperative test is? Single best answer question “ choose ONE true option only A. T3 and T4 B. Thyroid Stimulating Hormone (TSH) levels C. ECG D. Isotope scan E. Cyst ultrasound scan

ANSWER IS D Thyroglossal cyst results from the persistence of the thyroglossal tract. It presents clinically as a neck swelling. Though it most commonly occurs at the level of the hyoid bone in the midline, it can be anywhere between the foramen caecum, above the manubrium, and between the sternomastoid muscles.

The cystic nature can be confirmed using ultrasound scan. Thyroid function tests are usually normal. Since thyroglossal cysts can contain the only functioning thyroid tissue in the body, isotope scan should be planned before surgery to identify all functioning thyroid tissue.

628. The commonest benign tumour of the oesophagus is? Single best answer question “ choose ONE true option only A. Fibroma B. Pseudopolyps C. Adenoma

Prepared by Dr: Mohammed Musa Brema Idress – My best wishes Page 296

D. Leiomyoma E. Fibrous polyps

ANSWER IS D Most of the tumours of the oesophagus are malignant and symptomatic benign tumours account for less than 1% of oesophageal tumours.

Leiomyoma is the commonest benign tumour which occurs commonly in the lower part of the oesophagus. The majority of those are asymptomatic. The commonest symptomatic presentation is dysphagia. The tumours are well encapsulated. Endoscopic resection for small pedunculated lesions and localized resection for larger lesions is the treatment if symptomatic.

The second commonest benign tumours are fibrous or fibrovascular polyps which occur commonly in the upper third of the oesophagus.

629. One of the following is not a pathological characteristic of Crohn,s disease? Single best answer question “ choose ONE true option only. A. There are abnormal areas intervening between normal mucosa skip lesions B. Usually limited to mucosa C. Ulceration in the bowel is usually deep D. Non-caseating granuloma E. Can affect any part of the alimentary tract

ANSWER IS B Crohn,s disease is an inflammatory bowel disease that affects any part of the alimentary tract. The terminal ileum is affected in nearly 75% of the cases. The inflammation is transmural affecting the whole thickness of the affected bowel.

Since the inflammation is transmural and the ulcers associated with the disease are deep, fistulae formation, perforations, abscesses and strictures are not unusual. Perianal disease is common in Crohn,s presenting with features such as fissures, abscesses and fistulae.

Inflammatory bowel diseases are associated with extra-intestinal manifestations affecting the skin (pyoderma gangrenosum, erythema nodosum), eyes (iritis), joints (sacroiliitis, ankylosing spondylitis), biliary tree (sclerosing cholangitis, chronic active hepatitis).

630. In polyps of the colon the type that has the highest malignant potential is? Single best answer question “ choose ONE true option only A. Tubulovillous polyp B. Villous adenoma C. Juvenile polyp D. Tubular adenoma E. Metaplastic polyp

Prepared by Dr: Mohammed Musa Brema Idress – My best wishes Page 297

ANSWER IS B Metaplastic polyps: are the commonest type of rectal polyps. They are not neoplastic and they have no malignant potential. Juvenile polyps are the commonest polyps in children. They occur commonly in the rectum though they can occur anywhere in the large bowel.

Peutz-Jeghers syndrome: is the combination of intestinal hamartomatous polyp with peri-oral pigmentation. The polyps are usually in the small intestine and they are wide spread. Presentation is often with bleeding or intussusception.

Adenomas of the colon have malignant potential, with the highest potential in the villous adenoma. Villous adenomas are usually large and sessile and found commonly in the rectum while tubular adenomas are usually small (less than 2cm) and commonly multiple.

631. Consider maldescent of the testes, which of the following is a site of ectopic testes? Single best answer question “ choose ONE true option only A. Inguinal B. Intra-canalicular C. External inguinal ring D. High scrotal E. Base of the penis

ANSWER IS E Maldescended testes are either incompletely descended testes or ectopic testes. Incompletely descended testes occur in 20% of the cases where the testes fail to reach the scrotum but lie along the path. The common sites are intra-abdominal, inguinal, intra-canalicular, external inguinal ring and high scrotal. On the other hand ectopic testis occurs when the testes leaves the normal course of descent. The common sites are superficial inguinal pouch, perineum, base of the penis and the femoral triangle.

632. Which of the following is true for a patient of Familial Adenomatous Polyposis (FAP)? Single best answer question “ choose ONE true option only A. Family screening for the condition should start at 10 and be done yearly until 35years old B. Desmoid tumours occurring as part of FAP are benign and do not cause any problems C. Colonic cancer is the only malignancy associated with FAP D. FAP affects 1 in 1000 people E. The FAP adenoamatous Polyposis coli (APC) gene shows an autosomal recessive form of inheritance

ANSWER IS A FAP affects 1 in 10,000. Men and women are equally affected. About one-third of cases seem to arise from new mutations.

Prepared by Dr: Mohammed Musa Brema Idress – My best wishes Page 298

Desmoid tumours, which are solid tumours of the connective tissues, particularly within the abdomen, occur in 5 to 10 per cent of people with FAP. Although they are not malignant, they can cause major problems by invading surrounding tissues, and they can be very difficult to cut out.

Individuals with desmoid tumours have a one in five risk of death at an average age of about 35. Individuals affected with FAP have approximately a 10 per cent lifetime risk of developing a duodenal malignancy.

FAP affects 1 in 10,000. Men and women are equally affected. About one-third of cases seem to arise from new mutations. Family screening starts at 10 and is yearly until 35, after that 3 yearly. Screening involves a flexible sigmoidoscopy or colonoscopy.

633. Which ONE of the following statements is true about the differences between Crohn,s Disease (CD) and Ulcerative Colitis (UC)? Single best answer question “ choose ONE true option only A. Both Crohn,s and UC start then spread from the Rectal mucosa B. Crohn,s disease has a clear demarcation between involved and uninvolved mucosa, where as Ulcerative colitis shows skip lesions with normal mucosa in between C. The incidence of gallstones and kidney stones is increased in Crohn,s but not in UC D. Weight loss is more commonly seen in UC than in Crohn,s E. Fistulas are often seen in both diseases

ANSWER IS C UC always involves the rectum with disease then spreading proximally throughout the colon. There can be backwash ileitis where there is some involvement of the Terminal Ileum, but UC is really a colonic disease. UC remains confined to the rectum in approximately 25% of cases. In the remainder of cases, UC spreads proximally and contiguously. Pancolitis occurs in 10% of patients.

CD, on the other hand, consists of segmental involvement by a nonspecific granulomatous inflammatory process. The most important pathologic feature is involvement of all layers of the bowel, not just the mucosa and the submucosa, as is characteristic of UC.

CD is discontinuous, with skip areas interspersed between one or more involved areas. Late in the disease, the mucosa develops a cobblestone appearance, which results from deep longitudinal ulcerations interlaced with intervening normal mucosa.

The 3 major patterns of involvement in CD are:- 1) Disease in the ileum and caecum, occurring in 40% of patients; 2) Disease confined to the small intestine, occurring in 30% of patients; and 3) Disease confined to the colon, occurring in 25% of patients.

Prepared by Dr: Mohammed Musa Brema Idress – My best wishes Page 299

The involvement of the small bowel in Crohn,s has an effect on nutrient absorption, i.e. malabsorption of fat and bile salts. Gallstones are formed because of increased cholesterol concentration in the bile, caused by a reduced bile salt pool.

Patients who have CD with ileal disease or resection are likely to form calcium oxalate kidney stones. With the fat malabsorption, unabsorbed long-chain fatty acids bind calcium in the lumen. Oxalate in the lumen normally is bound to calcium. Calcium oxalate is poorly soluble and poorly absorbed; however, if calcium is bound to malabsorbed fatty acids, oxalate combines with sodium to form sodium oxalate, which is soluble and is absorbed in the colon (enteric hyperoxaluria). The development of calcium oxalate stones in CD requires an intact colon to absorb oxalate. Patients with ileostomies do not develop calcium oxalate stones.

Weight loss is more commonly seen with Crohn,s disease due to the loss of absorptive capacity in the Small bowel. UC is a more likely to present with bloody diarrhoea, and symptoms suggestive of an inflamed rectum, such as tenesmus, and urgency.

Fistulas are never seen in UC because there is only mucosal and submucosal involvement in the large bowel. Crohn's however has full thickness involvement of the bowel wall and can therefore form a fistula with nearby structures e.g. recto- vaginal, recto-vesical.

634. Which ONE of the following is true of Toxic Megacolon? Single best answer question “ choose ONE true option only A. Toxic megacolon is due to large bowel obstruction causing dilatation of the colon B. Toxic megacolon does not occur in Crohn,s disease C. On abdominal X-ray a colonic diameter of 8cm is significant D. Mortality rates in Toxic megacolon of any aetiology are 40% E. First line treatment is surgical resection

ANSWER IS C The hallmarks of toxic megacolon, a potentially lethal condition, are nonobstructive colonic dilatation larger than 6 cm and signs of systemic toxicity.

Toxic megacolon may complicate any number of colitides, including inflammatory, ischaemic, infectious, radiation, and pseudomembranous colitis. In a study of 1936 patients over 19 years Toxic megacolon was found in 2.3% of Crohn,s disease admissions.

The transverse colon is often used as the point of reference for dilatation, its normal diameter = 5.5 - 6.5 cm. Loss of colonic haustra is also seen, and possible thumb printing, a coarse, irregular, mucosal pattern of the large bowel

Prepared by Dr: Mohammed Musa Brema Idress – My best wishes Page 300

Mortality rates have been falling due to better care and earlier surgical involvement, and are now quoted as being 5-20%. Once perforation has occurred mortality rates are thought to increase 5 fold.

During the initial resuscitation, fluid and electrolyte replacement, and transfusion should be aggressive. Broad-spectrum intravenous antibiotics with coverage equivalent to ampicillin, gentamicin, and metronidazole should be initiated. All medications that may affect colonic motility must be stopped. These include narcotics, antidiarrheals, and anticholinergic agents. The patient should be put on bowel rest, and a nasogastric tube placed to assist with GI decompression

635. A 20-year-old male presents with a painless lump in his testicle. Which ONE of the following is true of testicular malignancies? Single best answer question - choose ONE true option only A. Testicular tumours can present with gynaecomastia

B. Teratomas are the commonest type of testicular malignancy C. In pure Seminomas Alpha-Fetoprotein (AFP) is markedly raised D. Teratomas are less aggressive than seminomas E. An orchidectomy for malignancy should be performed through a scrotal incision

ANSWER IS A Gynaecomastia is a rare presenting feature but can occur in sex hormone secreting interstitial cell tumours, and may be noticed before the lump is seen.

Seminomas are germ-cell tumours arising from the seminiferous epithelium. Seminoma is the commonest form of malignant testicular tumour at 40% of the total incidence. - Its peak incidence is 30-50. Teratomas are also germ-cell tumours; part of the non-seminomatous germ-cell group.

Nonseminomatous germ cell tumour frequency: 1. Embryonal carcinoma (20-25%) 2. Teratocarcinoma (25-30%) 3. Teratoma (5-10%) 4. Pure choriocarcinoma (1%)

AFP is not produced by pure seminomas; it is produced by Yolk-sac elements, which are not present in Seminomas. Beta human chorionic gonadotrophin (B.hCG) and AFP are used to aid diagnosis and monitoring of response to treatment in Teratomas. In seminomas B.hCG is more useful tumour-marker, along with Lactate dehydrogenase, which is less specific but can give an idea of tumour burden.

Teratomas are more aggressive than seminomas. Seminomas tend to spread by the lymphatics to the para-aortic nodes, where as teratomas spread haematogenously and occasionally to lymph nodes.

Prepared by Dr: Mohammed Musa Brema Idress – My best wishes Page 301

If clinical and lab findings are highly suggestive of malignancy an INGUINAL approach is used to allow control of the blood vessels and the Vas. These vessels must both be divided above a tourniquet placed on them. If the scrotum is incised there is a risk of seeding and recurrence. If there is doubt a frozen section can be done but should also be through an inguinal incision.

636. A patient presents with renal cyst on ultrasound scan. Which ONE of the following is true of renal cystic disease? Single best answer question “ choose ONE true option only A. Simple renal cysts decrease in incidence with age B. Adult polycystic disease always presents in adult life C. Childhood polycystic disease has a milder perinatal form, which is rarely fatal D. Both adult and childhood polycystic disease can also be associated with cysts in the liver E. Adult polycystic disease can be unilateral

ANSWER IS D Simple renal cysts increase in incidence with age; they are a common finding at autopsy, or an incidental finding at USS. These cysts may be single or multiple, and are usually fluid filled. They rarely cause problems, but can cause pain if there is haemorrhage into a cyst.

Adult polycystic kidney disease (APKD) can present from childhood to late in adult life. It has an autosomal dominant inheritance with high penetrance. Renal function is maintained until the cysts compress the surrounding parenchyma.

There are several subgroups of childhood polycystic kidney disease (CPKD) the perinatal form accounts for 10% of childhood PKD and involves around 90% of the nephrons. The baby may be still born or die from renal failure soon after birth. Neonatal, infantile, and juvenile represent a progressive less severe involvement with a longer survival.

Both forms of polycystic kidney disease can have liver cysts. In adult polycystic kidney disease the liver cysts have no functional significance. In childhood polycystic kidney disease there may be more significant liver disease ranging from bile-duct proliferation and cysts to hepatic fibrosis.

Adult PKD is always bilateral with grossly enlarged kidneys weighing 1000g or more. Cysts range in size from a few mm to 100mm and cause much distortion.

637. An elderly man presents with painless haematuria. The last episode in the past was painful. Which ONE of the following statements is true about his possible diagnosis? Single best answer question - choose ONE true option only A. Painless haematuria cannot be a urinary calculus B. Squamous cell carcinoma is associated with exposure to certain dyes, and smokers

Prepared by Dr: Mohammed Musa Brema Idress – My best wishes Page 302

C. Squamous cell carcinomas are usually solid and invasive D. Carcinoma-in-situ is not a precursor to invasive carcinoma E. Metastatic lung carcinoma does not produce bladder secondaries

ANSWER IS C Urinary calculi can cause painless haematuria, although painful haematuria is more commonly seen in the hospital setting. Squamous cell carcinoma arises from metaplastic squamous epithelium, which occurs most often in association with bladder calculi. A history of previously ignored painful haematuria could point towards bladder calculi.

Transitional Carcinoma is associated with some aniline dyes in the textile industry, reagents in rubber, smoking and analgesic abuse. Squamous cell carcinoma is associated with metaplastic epithelium seen in association with bladder calculi and schistosomiasis.

Squamous cell carcinoma is usually solid and invasive; its prognosis depends on the individual stage and grade of the tumour, but generally is not as good.

Carcinoma-in situ is a precursor of invasive transitional cell carcinoma. It is often seen as a field change in between areas of malignancy.

Secondary tumours, by haematogenous and lymphatic spread, can be found in the bladder; although they are rare, direct spread from the cervix, prostate or rectum is more common.

638. Which ONE of the following statements is true about risk factors for breast cancer? Single best answer question choose ONE true option only A. The incidence of breast cancer rises steeply up to 40-45 then falls B. An early menarche lowers your risk of breast cancer C. A woman having her first child in her mid-late thirties still has a lower risk of breast cancer than a nulliparous woman D. In the UK the risk of any woman developing breast cancer is 11% E. The identification of BRCA1 + 2 genes confers an increased risk of breast cancer only

ANSWER IS D The incidence of breast cancer does rise steeply until 40-45, at which point the incidence still increases but less steeply.

There is a significantly higher risk of developing breast cancer amongst women with an early age at menarche (i.e. <11). Age at menopause is also important. A woman who goes through the menopause at 45 has only half the breast cancer risk of someone whose menopause happens at 55. The longer the overall length of menarche the greater the breast cancer risk, as this increases lifetime exposure to oestrogens.

Prepared by Dr: Mohammed Musa Brema Idress – My best wishes Page 303

Nulliparous women have an increased risk of developing breast cancer. The risk reduction conferred by having children is only if a woman has children at an earlier age. If a woman has her first child in her mid to late thirties, she is then at a greater risk of breast cancer than a nulliparous woman. Pregnancy decreases breast tissue susceptibility to somatic mutations; thus, the earlier the first pregnancy, the shorter the susceptibility period

The lifetime risk quoted on current breast cancer websites is 11% i.e. 1 in 9 women will develop breast cancer. A positive family history then increases this risk, depending on the number and age of first-degree relatives affected.

BRCA2 mutations are associated with other types of cancers, such as prostate, pancreatic, fallopian tube, bladder, non-Hodgkin lymphoma, and basal cell carcinoma.

639. A woman presents with a thyroid mass and symptoms of hyperthyroidism. Which ONE of the following is true of hyperthyroidism? Single best answer question choose ONE true option only A. Toxic nodular goiters are the commonest cause of hyperthyroidism B. Graves disease is due to production of an autoantibody of the Ig-M class, which binds to thyroid epithelial cells and mimics the action of TSH C. Less than 1% of functioning adenomas show enough secretory activity to cause hyperthyroidism D. In Graves disease Pretibial myxoedema is more common than exopthalmos E. Hyperthyroidism can cause distal myopathy

ANSWER IS C Graves’s disease is the commonest cause of hyperthyroidism (thyrotoxicois). It is usually associated with a diffuse goiter. The thyroid is moderately enlarged with increased vascularity.

The autoantibody in Graves’s disease is an Ig-G antibody known as a long-acting thyroid stimulator (LATS). It stimulates the function and growth of the thyroid follicular epithelium.

Thyroid adenomas may be so small that they only show up on a 131I-radio-istope scan. Histologically they are composed of thyroid follicles.

Exopthalmos, pretibial myxoedema, (due to accumulation of mucopolysaccharides in the deep dermis of the skin) and finger-clubbing are only seen in Graves’s disease. Pretibial myxoedema, and finger-clubbing are rare but exopthalmos is common. Hyperthyroidism is known to cause proximal rather than distal myopathy.

640. An elderly man presents with a firm unilateral thyroid mass, which appears malignant. Which ONE of the following is true of malignant thyroid disease? Single best answer question - choose ONE true option only A. 80% of Follicular lesions are malignant

Prepared by Dr: Mohammed Musa Brema Idress – My best wishes Page 304

B. Fine needle aspiration (FNA) will not differentiate between benign papillary adenomas and malignant papillary carcinoma C. Treatment of follicular carcinoma involves thyroid lobectomy with frozen section, and if confirmed as follicular carcinoma total thyroidectomy D. Papillary carcinoma tends to metastasize to lung, and the brain E. Medullary carcinoma of the thyroid occurs as part of MEN1

ANSWER IS C 20% of follicular lesions are malignant. Follicular carcinoma tends to be unilateral and encapsulated. Whereas papillary carcinoma is unencapsulated and multifocal.

FNA can be used to differentiate papillary carcinoma from papillary adenoma. It is follicular lesions which need tissue the architecture provided by a histology specimen to distinguish benign from malignant lesions. The cytology provided by an FNA is not adequate.

If a frozen section confirms a follicular adenoma there is no need for further surgery. Papillary carcinoma spread locally to regional lymph nodes, where as follicular carcinoma spreads to the lung bone and brain.

Medullary carcinoma can occur as part of MEN2a and MEN2b. It arises from the para-follicular C Cells. Tumours produce calcitonin, which can be used to monitor for the presence of metastatic disease.

641. Which ONE of the following is true about the pathology of atherosclerosis? Single best answer question choose ONE true option only A. Atherosclerosis affects only large arteries B. The accumulation of lipid-laden macrophages occurs late on in the development of atherosclerosis C. Fatty streaks in the aorta do not develop until 40 plus D. Platelet derived growth factor (PDGF) plays an important role in the development of an atheromatous plaque E. Developing atherosclerotic plaques acquire their own microvascular network called vasa recta

ANSWER IS D Atherosclerosis affects large and medium sized arteries. Lipid laden macrophages appear in the subendothelial zone early in the development of atherosclerosis. A more advanced plaque contains proliferating smooth muscle.

Fatty streaks in the aorta and the coronary arteries can be seen in the majority of people by age 20.

PDGF stimulate the proliferation of intimal smooth muscle cells and their subsequent synthesis of collagen and elastin. PDGF is secreted by macrophages and endothelium.

Prepared by Dr: Mohammed Musa Brema Idress – My best wishes Page 305

The developing plaque had its own blood supply the vasa vasorum. The vasa recta supply the collecting ducts in the kidney. Haemorrhage of these vessels contributes towards plaque progression.

642. Which ONE of the following is true of Aneurysm pathology? Single best answer question choose ONE true option only A. A false aneurysm is the same thing as a dissecting aneurysm B. 25% of people with one aneurysm will also have other aneurysms elsewhere C. The intimal entry tear in a dissecting aneurysm is usually in a diseased area of the aorta D. The commonest location for Mycotic aneurysms is the aorta E. Syphilitic aneurysms often result in rupture

ANSWER IS B A false aneurysm is a blood-filled space due to an organized haematoma following a vascular rupture. A dissecting aneurysm is due to blood being forced through a tear in the intima of a vessel. Blood collects in the aortic media and can track back into the pericardium leading to possible fatal haemopericardium. Dissecting aneurysms are seen in Marfan,s syndrome and the elderly. 25% of people with one aneurysm will also have other aneurysms elsewhere.

In most cases of dissecting aneurysm the intimal tear is seen in disease free section of the aorta; occasionally a tear will occur around atheromatous plaques.

Mycotic aneurysms are due to weakening of the arterial wall commonly secondary to bacterial infection. Lesions are commonest in the cerebral & limb arteries but almost any area can be affected. Syphilitic aneurysms rarely rupture, instead causing aortic incompetence.

643. Which ONE of the following is true of Cushing’s disease? Single best answer question choose ONE true option only A. The commonest cause of Cushing’s syndrome is iatrogenic B. Cushing’s syndrome is due to an excess of aldosteron C. True Cushing’s disease is due to an adrenal adenoma D. Removal of a pituitary adenoma carries a high risk of Nelson’s syndrome E. At autopsy 50% of men over 50 have microscopic cancer in the prostate

ANSWER IS A By far the commonest cause of Cushing’s syndrome is long-term steroid use for chronic conditions such as arthritis or asthma.

Cushing’s syndrome results from excess cortisol. The natural causes are, excess ACTH secretion by the adenohypophysis; adrenal cortical neoplasms; ectopic ACTH secretion e.g. by Oat cell carcinoma of the bronchus.

Prepared by Dr: Mohammed Musa Brema Idress – My best wishes Page 306

Cushing’s syndrome refers to the constellation of bodily responses to excess glucocorticoids. True Cushing’s disease is due to hypersecretion of ACTH by an adenoma of the corticotrophs in the pituitary gland.

Nelson’s syndrome is the name given to hyperpigmentation, ACTH excess and pituitary expansion secondary to bilateral adrenalectomy. There is a 20% risk of developing Nelson’s syndrome after a bilateral adrenalectomy, but the risk can be minimized by pituitary irradiation.

Cushing’s syndrome tends to cause hypertension rather than hypotension. Other features of Cushing’s syndrome include central obesity, proximal myopathy, osteoporosis, striae and poor wound healing, diabetes mellitus, depression and psychosis, and menorrhagia in women.

644. Which ONE of the following is true of lung cancer and its staging? Single best answer question choose ONE true option only A. T1 applies to a tumour no larger than 1cm B. A tumour invading the pleura is T3 C. A 2cm node has an approximately 60-70% chance of showing malignant infiltration D. The majority of malignant lung lesions are primary lung cancers E. Small cell tumours are derived from bronchopulmonary tissues and include adenocarcinoma

ANSWER IS C T1 applies to tumours 3cm and smaller. T2 applies to tumours within a lobar bronchus but more than 2cm from the carina, or to invasion of the pleura.

T3 applies to any tumour with invasion into adjacent structures, involvement of the main bronchus less than 2cm form the carina, or any tumour associated with a pleural effusion.

A 1cm lymph node has a less than 7% chance of having malignant infiltration. From 1-2cm there is a 40% chance of malignant infiltration; and at 3 cm the chance of malignant infiltration is 66%.

The majority of lung malignancies are deposits from elsewhere. Only 20% of primary lung cancer will not have metastasized by time of diagnosis.

Small cell carcinomas (often called oat-cell tumours) are thought to be derived from APUD cells. Non-small cell carcinomas are derived from bronchopulmonary tissue and include adenocarcinoma, squamous cell carcinoma and large-cell undifferentiated carcinoma.

645. A man presents with dysphonia and a possible laryngeal tumour. Which ONE of the following statements about laryngeal tumours is true? Single best answer question choose ONE true option only

Prepared by Dr: Mohammed Musa Brema Idress – My best wishes Page 307

A. The most common site for laryngeal a malignancy is the supraglottis B. Haematogenous metastatic spread is common at the time of diagnosis C. Associated dysplastic change is uncommon D. The commonest form of benign laryngeal tumours are papillomas E. Radiotherapy is not used in treatment of laryngeal tumours

ANSWER IS D Most laryngeal tumours arise on the vocal cords. At diagnosis most tumours have not spread beyond local nodal involvement.

As in squamous carcinoma of the cervix neoplasia is thought to be preceded by a phase of dysplasia. Low-grade dysplasia may be reversible on withdrawal of causative factors. Papillomas may be multiple in children but tend to be single in adults. - Treatment is by resection and / or radiotherapy.

646. A 60-year-old lady presents with a parotid mass. Which ONE of the following statements is true about parotid lesions? Single best answer question choose ONE true option only A. True cut biopsy is indicated for diagnosis B. Most parotid tumours are malignant C. Function of the trigeminal nerve should be carefully assessed D. Parotid secretions constitute the majority of saliva volume E. Pleomorphic adenomas have the potential to transform into a malignant tumour

ANSWER IS E Fine needle aspiration and imaging are used in diagnosis. True cut biopsy is contra- indicated where there is the potential for malignant disease, as it can cause seeding along the tract. In parotid lesions there is also is also the possibility of damaging the facial nerve. - 80-85% of parotid tumours are benign.

The facial nerve courses through the parotid gland and its function should be carefully assessed. The trigeminal nerve is not found near the parotid gland.

The parotid gland is the largest of the salivary glands but produces mucinous secretions. Saliva is predominantly serous fluid, which is produced by the submandibular glands.

Pleomorphic adenomas may undergo true malignant change, often indicated by a sudden increase in size. They usually transform into squamous cell carcinoma.

647. A 65-year-old man presents with retention. Which ONE of the following statements about prostatic pathology is True? Single best answer question choose ONE true option only A. Benign prostatic hyperplasia affects 50% of men over 45

Prepared by Dr: Mohammed Musa Brema Idress – My best wishes Page 308

B. The severity of symptoms is related to the size of the prostate gland C. Finasteride a 5-alpha-reductase inhibitor will decrease the size of the prostate gland within a week and therefore allow a trial without catheter D. Benign prostatic hyperplasia (BPH) will not increase PSA (prostate specific antigen) E. The aim of transurethral prostatectomy is to core out the whole prostate

ANSWER IS A The proportion of men affected increases with increasing age. Many of this 50% will have only mild or moderate symptoms, which are acceptable to them without treatment. The severity of symptoms are not related to the size of the gland but to the encroachment on the prostatic urethra and the resulting obstruction to urinary flow.

Finasteride is an alpha-reductase inhibitor, which blocks the conversion of testosterone to dihydrotestosterone and reduces the size of the hyperplastic glands. A 6-month trial of treatment is required and symptoms may be reduced or delay or even avoid surgery. Alpha-adrenergic blockers can be used to reduce bladder neck hypertrophy, and take effect within a week.

BPH can cause a small rise in PSA, but levels over 10-15 nanograms/mL are highly indicative of carcinoma.

The aim of TURP is to remove the bulk of the prostate gland and leave the compressed but normal peripheral tissue. By leaving the normal peripheral tissue the sub-capsular venous plexus is protected.

648. A 41-year-old woman underwent renal transplantation 18 months ago. She attends follow-up clinic and is currently asymptomatic. On examination her blood pressure is 150/110 and urinalysis reveals 3+ protein. Urine culture is negative. A renal ultrasound scan reveals a normal collecting system. Renal biopsy demonstrates intrarenal arteriosclerosis with associated glomerular atrophy and interstitial fibrosis. Select the most likely complication for this case. A. Acute rejection B. Chronic rejection C. Graft-versus-host disease D. Malignancy E. Primary graft non-function

ANSWER IS B Chronic rejection is characterized clinically by a progressive deterioration in graft function occurring months to years after transplantation and is associated with typical histological changes of graft atherosclerosis and fibrosis. By contrast, acute rejection occurs within the first 3 months after transplantation, and hyperacute rejection occurs within hours.

Prepared by Dr: Mohammed Musa Brema Idress – My best wishes Page 309

For the diagnosis of chronic rejection to be made, other causes of graft dysfunction must be excluded (e.g. infection, calcineurin antagonist toxicity, etc), and a transplant graft biopsy is required to confirm the diagnosis histologically.

For renal transplantation, graft dysfunction is manifested by a rise in serum creatinine as a result of progressive decline in the glomerular filtration rate. There is associated proteinuria and worsening hypertension, with the diastolic component classically rising in advance of the systolic component.

649. Which ONE of the following is true about prostate cancer? Single best answer question – choose ONE true option only A. Prostate carcinoma arises in the paraurethral tissue B. Malignant change may occur in the compressed normal tissue left behind after TURP C. Average survival after diagnosis of metastatic disease is 6-months D. Stage A tumors are nodular on rectal examination E. At autopsy 50% of men over 50 have microscopic cancer in the prostate

ANSWER IS B Carcinoma of the prostate arises in the peripheral glands rather than the para- urethral glands, which is why it often takes longer to become symptomatic.

Malignant change may occur in the compressed normal tissue left behind after TURP. Average survival after diagnosis of metastatic disease is 2-years.

A Stage A malignant prostate appears normal or smoothly enlarged by benign hyperplasia. Stage B has a nodular asymmetrical surface. Stage C presents with a large hard irregular gland.

At autopsy a third of men over 50 have microscopic malignant change in the prostate gland, and at 90 the percentage becomes 90%.

650. For a patient with obstructive jaundice, which ONE of the following statements is TRUE? Single best answer question – choose ONE true option only A. Clinically detectable jaundice does not occur until the bilirubin is above 60micromol/l B. Bile acid deposition in the skin can cause intense itching C. Most biliary strictures are due to inflammatory scarring after cholecystitis D. Cholangiocarcinoma is the commonest neoplasm causing obstructive jaundice E. Primary sclerosing cholangitis (PSC) is more commonly seen in Crohn’s disease than in Ulcerative colitis

ANSWER IS B Clinically detectable jaundice occurs above 30micromol/L, at 60micromol/L the jaundice should be obvious.

Prepared by Dr: Mohammed Musa Brema Idress – My best wishes Page 310

Biliary obstruction can dam back bile acids, which then raises their blood concentration and causes skin deposition, and intense itching.

The majority of biliary strictures are iatrogenic after surgery, only a small number are post inflammatory disease.

Adenocarcinoma of the head of the pancreas is the commonest neoplasm leading to obstructive jaundice. Cholangiocarcinoma is a rare tumour arising from biliary epithelium. Unlike adenocarcinoma of the pancreas these tumours are slow growing and often metastasize late so that complete resection is often possible. 90% of pancreatic carcinomas are adenocarcinomas derived from the ductal cells of the exocrine pancreas; these have only a 5% 5 year survival rate.

Sclerosing cholangitis is a rare condition thought to be of autoimmune origin; it results in progressive fibrosis of the biliary system. PSC may arise spontaneously but is usually in association with long-standing U.C, rather than Crohn’s. Areas of short localized disease can be treated with an indwelling stent.

651. Which ONE of the following is true about skin lesions? Single best answer question – choose ONE true option only A. Epidermal cysts are filled with sebaceous material B. Removal of an inflamed epidermal cyst is the best treatment C. Dermoid cysts can contain hair, keratin and sebaceous glands D. Pilonidal lesions rarely recur after surgery E. Kaposi’s sarcoma is most often found on the trunk, and may be a presenting sign of AIDS

ANSWER IS C Epidermal cysts although called sebaceous cysts are lined by stratified squamous epithelium and filled with keratin, consistent with their derivation from a hair follicle.

Removal of an inflamed epidermal cyst is not the best treatment as it is very difficult to define any tissue planes and remove the cyst whole. If very painful the cyst contents can be drained but removal should be left until the inflammation has settled.

Dermoid cysts arise from cystic change in epithelial remnants left behind at lines of embryological fusion. They are usually found in the midline of the scalp, neck and lower jaw. Treatment is by excision.

Pilonidal lesions include cysts, sinuses and abscesses, and occur due to a nest of hairs, which have worked their way into the dermis. Surgical treatment aims to eliminate the nidus of hairs, but unfortunately they do often recur.

Kaposi’s Sarcoma appears as multiple nodules or plaques, all of which are primary tumours. They most commonly occur on the limbs. The tumours are composed of

Prepared by Dr: Mohammed Musa Brema Idress – My best wishes Page 311 proliferating dysplastic fibroblasts, along with chronic inflammation. They should be treated with excision biopsy.

652. A 40-year-old female suffers from recurrent urinary tract infections caused by proteus spp isolated from her mid-stream urine samples. Cystoscopic bladder examination confirms the presence of a bladder calculus. What is the most likely composition of the bladder calculus? Single best answer question – choose ONE true option only A. Cystine B. Magnesium-ammonium-phosphate C. Calcium oxalate D. Uric acid E. Xanthine

ANSWER IS B Magnesium-ammonium-phosphate calculi account for approximately one-sixth of all bladder stones and are particularly associated with proteus spp infections (e.g. proteus mirabilis and proteus vulgaris). The most common chemical composition for bladder calculi is calcium oxalate or mixed calcium phosphate/oxalate, representing around 75% of bladder stones.

Uric acid calculi, which are associated with gout, comprise approximately 5% of bladder stones. Cystine and xanthine calculi are associated with inborn errors of metabolism.

653. A 55-year-old male with a two year history of back pain presents with worsening symptoms. Which of the following signs is most suggestive of Acute Cauda Equina Syndrome? Single best answer question – choose ONE true option only A. Muscle wasting B. Hyperactive lower limb reflexes C. Loss of sensation to the lateral aspect of the foot D. Loss of perianal tone E. Positive Babinski sign

ANSWER IS D Cauda Equina Syndrome (CES) is caused when there is compression of the lumbosacral nerves below the level of the spinal cord. It typically presents with by low back pain, unilateral or bilateral sciatica, bladder or bowel dysfunction and perianal sensory loss or hypoaesthesia.

Loss of perianal tone is characteristic of CES. Muscle wasting suggests chronic CES. Hyperactive lower limb reflexes and a positive Babinski sign are upper motor neuron signs and therefore may the diagnosis of CES unlikely.

654. Which muscle initiates abduction of the shoulder? A. Infraspinatus

Prepared by Dr: Mohammed Musa Brema Idress – My best wishes Page 312

B. Latissimus dorsi C. Supraspinatus D. Deltoid E. Teres major

ANSWER IS C The intermediate portion of the deltoid muscle is the chief abductor of the humerus. However, it can only do this after the movement has been initiated by supraspinatus. Damage to the tendon of supraspinatus is a common form of rotator cuff disease.

655. Which of the following nerves is most commonly damaged during a superficial parotidectomy? A. Greater auricular B. Facial C. Greater occipital D. Accessory E. Ophthalmic branch of the trigeminal

ANSWER IS A The greater auricular nerve and in particular its lobular branch is commonly injured in parotid surgery and consent usually makes particular reference to this. In a superficial parotidectomy, the facial nerve should not be injured and this is less common than a greater auricular nerve injury.

656. A 34 year old man is shot in the postero- inferior aspect of his thigh. Which of the following lies at the most lateral aspect of the popliteal fossa? A. Popliteal artery B. Popliteal vein C. Common peroneal nerve D. Tibial nerve E. Small saphenous vein

ANSWER IS C The contents of the popliteal fossa are (from medial to lateral): 1. Popliteal artery 2. Popliteal vein 3. Tibial nerve 4. Common peroneal nerve

The sural nerve is a branch of the tibial nerve and usually arises at the inferior aspect of the popliteal fossa. However, its anatomy is variable.

657. A 67 year old man has an abdominal aortic aneurysm which displaces the left renal vein. Which branch of the aorta is most likely to affected at this level? A. Inferior mesenteric artery B. Superior mesenteric artery C. Coeliac axis

Prepared by Dr: Mohammed Musa Brema Idress – My best wishes Page 313

D. Testicular artery E. None of the above

ANSWER IS B The left renal vein lies behind of the SMA as it branches off the aorta. Whilst juxtarenal AAA may sometimes require the division of the left renal vein, direct involvement of the SMA may require a hybrid surgical bypass and subsequent endovascular occlusion.

658. A 12 year old boy undergoes surgery for recurrent mastoid infections. Post operatively he complains of an altered taste sensation. Which of the following nerves has been injured? A. Glossopharyngeal B. Greater petrosal C. Olfactory D. Trigeminal E. Chorda tympani

ANSWER IS E The chorda tympani branch of the facial nerve passes forwards through its canaliculus into the middle ear, and crosses the medial aspect of the tympanic membrane. It then passes antero-inferiorly in the infratemporal fossa. It distributes taste fibers to the anterior two thirds of the tongue.

659. What is the origin of the superior gluteal artery? A. Internal iliac artery B. External iliac artery C. Femoral artery D. Common iliac artery E. Circumflex femoral artery

ANSWER IS A The inferior gluteal artery arises from the anterior trunk of the internal iliac artery The superior gluteal artery arises from the posterior trunk of the internal iliac artery

660. The first root of the brachial plexus commonly arises at which of the following levels? A. C6 B. C5 C. C3 D. C2 E. C8

ANSWER IS B It begins at C5 and has 5 roots. It ends with a total of 15 nerves of these 5 are the main nerves to the upper limb (axillary, radial, ulnar, musculocutaneous and median)

Prepared by Dr: Mohammed Musa Brema Idress – My best wishes Page 314

661. What is the anatomical level of the transpyloric plane? A. T11 B. T12 C. L1 D. L4 E. T10

ANSWER IS C Transpyloric plane = Level of the body of L1 1. Pylorus stomach 2. Left kidney hilum (L1- left one!) 3. Right hilum of the kidney (1.5cm lower than the left) 4. Fundus of the gallbladder 5. Neck of pancreas 6. Duodenojejunal flexure 7. Superior mesenteric artery 8. Portal vein 9. Left and right colic flexure 10. Root of the transverse mesocolon 11. 2nd part of the duodenum 12. Upper part of conus medullaris 13. Spleen

662. When the brachial plexus is injured in the axilla as a result of a crutch palsy, which of the nerves listed is most commonly affected? A. Thoracodorsal nerve B. Suprascapular nerve C. Radial nerve D. Ulnar nerve E. Long thoracic nerve

ANSWER IS C The radial nerve is most commonly injured and results in a wrist drop. The ulnar nerve arises from the medial cord and is rarely affected as a result of this injury mechanism.

663. A 35 year old man falls and sustains a fracture to the medial third of his clavicle. Which vessel is at greatest risk of injury? A. Subclavian vein B. Subclavian artery C. External carotid artery D. Internal carotid artery E. Vertebral artery

ANSWER IS A The subclavian vein lies behind subclavius and the medial part of the clavicle. It rests on the first rib, below and in front of the third part of the subclavian artery, and then

Prepared by Dr: Mohammed Musa Brema Idress – My best wishes Page 315 on scalenus anterior which separates it from the second part of the artery (posteriorly).

664. The pudendal canal is a fascial canal located on the lateral wall of the ischioanal fossa. In this location, it lies on the inferior border of which of the following muscles? A. Coccygeus B. Obturator internus C. Pubococcygeus D. Iliococcygeus E. Piriformis

ANSWER IS B The coccygeus, pubococcygeus and iliococcygeus form part of the pelvic diaphragm and are not related to it. The piriformis exits the pelvis via the greater sciatic foramen and is not associated with the canal in the ischiorectal fossa.

Pudendal canal:- The pudendal canal is located along the lateral wall of the ischioanal fossa at the inferior margin of the obturator internus muscle. It extends from the lesser sciatic foramen to the posterior margin of the urogenital diaphragm. It conveys the internal pudendal vessels and nerve.

665. Where are the greatest proportion of musculi pectinati found? A. Right ventricle B. Left ventricle C. Right atrium D. Pulmonary valve E. Aortic valve

ANSWER IS C The musculi pectinati are found in the atria, hence the reason that the atrial walls in the right atrium are irregular anteriorly. The musculi pectinati of the atria are internal muscular ridges on the anterolateral surface of the chambers and they are only present in the area derived from the embryological true atrium.

666. Which of the nerves listed below provides sensory innervations to the skin overlying the lateral aspect of the nose? A. Infratrochlear nerve B. Zygomatic nerve C. Nasopalatine nerve D. Lateral nasal branches of the ethmoidal nerve E. Frontal nerve

ANSWER IS D The lateral aspect of the external nose is innervated by lateral nasal branches of the anterior ethmoidal nerve.

Prepared by Dr: Mohammed Musa Brema Idress – My best wishes Page 316

The ethmoidal nerve is a branch of the nasociliary nerve which is one of the divisions of the trigeminal.

667. Which of the following structures separates the subclavian artery and vein? A. Digastric muscle B. Prevertebral fascia C. Anterior scalene muscle D. Middle scalene muscle E. Omohyoid

ANSWER IS C The anterior scalene muscle is an important anatomical landmark and separates the subclavian vein (anterior) from the subclavian artery (posterior).

668. A 33 year old man is stabbed in the right chest and undergoes a thoracotomy. The right lung is mobilized and the pleural reflection at the lung hilum is opened. Which of the structures listed below does not lie within this region? A. Pulmonary artery B. Azygos vein C. Pulmonary vein D. Bronchus E. None of the above

ANSWER IS B The pleural reflections encase the hilum of the lung and continue inferiorly as the pulmonary ligament. It encases the pulmonary vessels and bronchus. The azygos vein is not contained within it.

669. A 56 year old man requires long term parenteral nutrition and the decision is made to insert a PICC line for long term venous access. This is inserted into the basilic vein at the region of the elbow. As the catheter is advanced, into which venous structure is the tip of the catheter most likely to pass from the basilic vein? A. Subclavian vein B. Axillary vein C. Posterior circumflex humeral vein D. Cephalic vein E. Superior vena cava

ANSWER IS B The basilic vein drains into the axillary vein and although PICC lines may end up in a variety of fascinating locations the axillary vein is usually the commonest site following from the basilic. The posterior circumflex humeral vein is encountered prior to the axillary vein. However, a PICC line is unlikely to enter this structure because of its angle of entry into the basilic vein.

670. An individual is noted to have a left sided superior vena cava. By which pathway is blood from this system most likely to enter the heart?

Prepared by Dr: Mohammed Musa Brema Idress – My best wishes Page 317

A. Via the coronary sinus B. Via the azygos venous system and into the superior vena cava C. Via anomalies in the pulmonary vascular bed D. Via the left atrium and persistent foramen ovale E. Directly into the roof of the right atrium

ANSWER IS A Persistent left superior vena cava is the most common anomaly of the thoracic venous system. It is prevalent in 0.3% of the population and is a benign entity of failed involution during embryogenesis. Superior vena cava →Drainage:- Head and neck Upper limbs Thorax Part of abdominal walls

Formation Subclavian and internal jugular veins unite to form the right and left brachiocephalic veins. These unite to form the SVC. Azygos vein joins the SVC before it enters the right atrium

Relations Anterior: Anterior margins of the right lung and pleura Posteromedial: Trachea and right vagus nerve Posterolateral: Posterior aspects of right lung and pleura Posterior: Pulmonary hilum Right lateral: Right phrenic nerve and pleura Left lateral: Brachiocephalic artery and ascending aorta

Developmental variations Anomalies of the connection of the SVC are recognized. In some individuals a persistent left sided SVC drains into the right atrium via an enlarged orifice of the coronary sinus. More rarely the left sided vena cava may connect directly with the superior aspect of the left atrium, usually associated with an un-roofing of the coronary sinus.

The commonest lesion of the IVC is for its abdominal course to be interrupted, with drainage achieved via the azygos venous system. This may occur in patients with left sided atrial isomerism.

671. A 40 year old lady trips and falls through a glass door and sustains a severe laceration to her left arm. Amongst her injuries it is noticed that she has lost the ability to adduct the fingers of her left hand. Injury to which of the following nerves is most likely to account for her examination findings? A. Ulnar B. Median C. Radial

Prepared by Dr: Mohammed Musa Brema Idress – My best wishes Page 318

D. Musculocutaneous E. Axillary

ANSWER IS A The interossei are supplied by the ulnar nerve.

672. A 53 year old man is undergoing a radical gastrectomy for carcinoma of the stomach. Which of the following structures will need to be divided to gain access to the coeliac axis? A. Lesser omentum B. Greater omentum C. Falciform ligament D. Median arcuate ligament E. Gastrosplenic ligament

ANSWER IS A The lesser omentum will need to be divided. During a radical gastrectomy this forms one of the nodal stations that will need to be taken.

673. A 76 year old man complains of symptoms of claudication. The decision is made to measure his ankle brachial pressure index. The signal from the dorsalis pedis artery is auscultated with a hand held doppler device. This vessel is the continuation of which of the following? A. Posterior tibial artery B. Anterior tibial artery C. Peroneal artery D. Popliteal artery E. None of the above

ANSWER IS B The dorsalis pedis is a continuation of the anterior tibial artery.

674. A 67 year old man is due to undergo a revisional total hip replacement using a posterior approach. After dividing gluteus maximus in the line of its fibers there is brisk arterial bleeding. Which of the following vessels is likely to be responsible? A. Profunda femoris artery B. External iliac artery C. Internal iliac artery D. Obturator artery E. Inferior gluteal artery

ANSWER IS E The inferior gluteal artery runs on the deep surface of the gluteus maximus muscle. It is a branch of the internal iliac artery. It is commonly divided during the posterior approach to the hip joint.

Prepared by Dr: Mohammed Musa Brema Idress – My best wishes Page 319

675. A 17 year old lady presents with right iliac fossa pain and diagnosed as having acute appendicitis. You take her to theatre to perform a laparoscopic appendicectomy. During the procedure the scrub nurse distracts you and you inadvertently avulse the appendicular artery. The ensuing haemorrhage is likely to be supplied directly from which vessel? A. Inferior mesenteric artery B. Superior mesenteric artery C. Ileo-colic artery D. Internal iliac artery E. None of the above

ANSWER IS C The appendicular artery is a branch of the ileocolic artery. Gluteus maximus Inferior gluteal N & A Gluteus medius & minimus ...sup gluteal N & A

676. A 63 year old man who smokes heavily presents with dyspepsia. He is tested and found to be positive for helicobacter pylori infection. One evening he has an episode of haematemesis and collapses. What is the most likely vessel to be responsible? A. Portal vein B. Short gastric arteries C. Superior mesenteric artery D. Gastroduodenal artery E. None of the above

ANSWER IS D The motor and sensory fibers of the genitofemoral nerve are tested in the cremasteric reflex. A small contribution is also played by the ilioinguinal nerve and thus the reflex may be lost following an inguinal hernia repair.

677. Which of the structures listed below are not located within the mediastinum? A. Thymus B. Heart C. Great vessels D. Arch of azygos vein E. Vertebral bodies

ANSWER IS E The vertebral bodies lie outside of the mediastinum, as do the lungs

678. A 22 year old man is stabbed in the chest at the level of the junction between the sternum and manubrium. Which structure is at greatest risk? A. Left atrium B. Oesophagus C. Thyroid gland D. Inferior vena cava

Prepared by Dr: Mohammed Musa Brema Idress – My best wishes Page 320

E. Aortic arch

ANSWER IS E At the level of the Angle of Louis (Manubriosternal angle), is the surface marking for the aortic arch. The oesophagus is posteriorly located and at less risk.

679. Which of these nerves passes through the greater and lesser sciatic foramina? A. Pudendal nerve B. Sciatic nerve C. Superior gluteal nerve D. Inferior gluteal nerve E. Posterior cutaneous nerve of the thigh

ANSWER IS A Structures passing through the lesser and greater sciatic foramina (medial to lateral): PIN A. Pudendal nerve B. Internal pudendal artery C. Nerve to obturator internus The pudendal nerve originates from the ventral rami of the second, third, and fourth sacral nerves (S2, S3, S4).

It passes between the piriformis and coccygeus muscles and exits the pelvis through the greater sciatic foramen. It crosses the spine of the ischium and reenters the pelvis through the lesser sciatic foramen. It passes through the pudendal canal.

The pudendal nerve gives off the inferior rectal nerves. It terminates into 2 branches: perineal nerve, and the dorsal nerve of the penis or the dorsal nerve of the clitoris.

680. A cervical rib is due to which of the following? A. Hyperplasia of the annulus fibrosus B. Proliferation of the nucleus pulposus C. Fusion of the transverse processes of the 6th and 7th cervical vertebrae D. An accessory cervical vertebra E. Elongation of the transverse processes of the 7th cervical vertebra

ANSWER IS E Cervical ribs occur as a result of the elongation of the transverse process of the 7th cervical vertebra. It is usually a fibrous band that attaches to the first thoracic rib.

681. Which of the structures listed below is not a content of the carotid sheath? A. Internal jugular vein B. Internal carotid artery C. Vagus nerve D. Recurrent laryngeal nerve E. Common carotid artery

Prepared by Dr: Mohammed Musa Brema Idress – My best wishes Page 321

ANSWER IS D Contents of carotid sheath: 1. Common carotid artery 2. Internal carotid artery 3. Internal jugular vein 4. Vagus nerve

682. A 28 year old rugby player injures his right humerus and on examination is noted to have a minor sensory deficit overlying the point of deltoid insertion into the humerus. Which of the nerves listed below is most likely to have been affected? A. Radial B. Axillary C. Musculocutaneous D. Median E. Subscapular

ANSWER IS B This patch of skin is supplied by the axillary nerve

683. A 22 year old man is undergoing a wedge excision of his great toenail. As the surgeon passes a needle into the area to administer local anaesthetic, the patient notices a sharp pain. By which pathway will this sensation be conveyed to the central nervous system? A. Anterior corticospinal tract B. Posterior spinocerebellar tract C. Cuneate fasciculus D. Vestibulospinal tract E. Spinothalamic tract

ANSWER IS E - Spinothalamic tract- Pain and temperature - Vestibulospinal tract- Motor neuronal signals relating to posture - Cuneate fasciculus- Fine touch, pressure and proprioception - Posterior spinocerebellar tract- Proprioceptive signals to cerebellum - Anterior corticospinal tract- Conveys motor signals from precentral gyrus to motor cells within the cord - Pain impulses are transmitted via the spinothalamic tract

684. A 73 year old lady is admitted with brisk rectal bleeding. Despite attempts at resuscitation the bleeding proceeds to cause haemodynamic compromise. An upper GI endoscopy is normal. A mesenteric angiogram is performed and a contrast blush is seen in the region of the sigmoid colon. The radiologist decides to embolize the vessel supplying this area. At what spinal level does it leave the aorta? A. L2 B. L1

Prepared by Dr: Mohammed Musa Brema Idress – My best wishes Page 322

C. L4 D. L3 E. T10

ANSWER IS D The inferior mesenteric artery leaves the aorta at L3. It supplies the left colon and sigmoid. Its proximal continuation to communicate with the middle colic artery is via the marginal artery.

685. Inspection of the left ventricle reveals all except which of the following? A. Papillary muscles B. Trabeculae carnae C. Chordae tendinae D. Conus arteriosus E. Openings of the venae cordis minimae

ANSWER IS D The conus arteriosus (infundibulum) is the smooth walled outflow tract of the right ventricle leading to the pulmonary trunk.

686. A 23 year old lady with troublesome axillary hyperhidrosis is undergoing a thorascopic sympathectomy to treat the condition. Which of the following structures will need to be divided to access the sympathetic trunk? A. Intercostal vein B. Intercostal artery C. Parietal pleura D. Visceral pleura E. None of the above

ANSWER IS C The sympathetic chain lies posterior to the parietal pleura. During a thorascopic sympathetomy this structure will need to be divided. The intercostal vessels lie posteriorly. They may be damaged with troublesome bleeding but otherwise are best left alone as deliberate division will not improve surgical access.

687. In which space is a lumbar puncture performed? A. Subdural space B. Epidural space C. Subarachnoid space D. Extradural space E. Intraventricular space

ANSWER IS C Samples of CSF are normally obtained by inserting a needle between the third and fourth lumbar vertebrae. The tip of the needle lies in the sub arachnoid space, the spinal cord terminates at L1 and is not at risk of injury. Clinical evidence of raised intracranial pressure is a contraindication to lumbar puncture.

Prepared by Dr: Mohammed Musa Brema Idress – My best wishes Page 323

688. A 21 year old man is stabbed in the antecubital fossa. A decision is made to surgically explore the wound. At operation the surgeon dissects down onto the brachial artery. A nerve is identified medially, which nerve is it likely to be? A. Radial B. Recurrent branch of median C. Anterior interosseous D. Ulnar E. Median

ANSWER IS E 689. A 65 year old man with long standing atrial fibrillation develops an embolus to the lower leg. The decision is made to perform an embolectomy, utilizing a trans popliteal approach. After incising the deep fascia, which of the following structures will the surgeons encounter first on exploring the central region of the popliteal fossa? A. Popliteal vein B. Common peroneal nerve C. Popliteal artery D. Tibial nerve E. None of the above

ANSWER IS D The tibial nerve lies superior to the vessels in the inferior aspect of the popliteal fossa. In the upper part of the fossa the tibial nerve lies lateral to the vessels, it then passes superficial to them to lie medially. The popliteal artery is the deepest structure in the popliteal fossa.

690. A 43 year old man is undergoing a right hemicolectomy and the ileo-colic artery is ligated. From which of the following vessels is it derived? A. Inferior mesenteric artery B. Superior mesenteric artery C. Coeliac axis D. Aorta E. None of the above

ANSWER IS B The ileocolic artery is a branch of the SMA and supplies the right colon and terminal ileum. The transverse colon is supplied by the middle colic artery. As veins accompany arteries in the mesentery and are lined by lymphatics, high ligation is the norm in cancer resections. The ileo-colic artery branches off the SMA near the duodenum.

691. A 53 year old man is undergoing a distal pancreatectomy for trauma. Which of the following vessels is responsible for the arterial supply to the tail of the pancreas? A. Splenic artery

Prepared by Dr: Mohammed Musa Brema Idress – My best wishes Page 324

B. Pancreaticoduodenal artery C. Gastric artery D. Hepatic artery E. Superior mesenteric artery

ANSWER IS A Pancreatic head is supplied by the pancreaticoduodenal artery Pancreatic tail is supplied by branches of the splenic artery There is an arterial watershed in the supply between the head and tail of the pancreas. The head is supplied by the pancreaticoduodenal artery and the tail is supplied by branches of the splenic artery. 692. A 43 year old lady presents with varicose veins and undergoes a saphenofemoral disconnection, long saphenous vein stripping to the ankle and isolated hook phlebectomies. Post operatively she notices an area of numbness superior to her ankle. What is the most likely cause for this? A. Sural nerve injury B. Femoral nerve injury C. Saphenous nerve injury D. Common peroneal nerve injury E. Superficial peroneal nerve injury

ANSWER IS C The sural nerve is related to the short saphenous vein. The saphenous nerve is related to the long saphenous vein below the knee and for this reason full length stripping of the vein is no longer advocated.

693. Which of the following muscles does not attach to the radius? A. Pronator quadratus B. Biceps C. Brachioradialis D. Supinator E. Brachialis

ANSWER IS E The brachialis muscle inserts into the ulna. The other muscles are all inserted onto the radius.

694. A 25 year old man is stabbed in the upper arm. The brachial artery is lacerated at the level of the proximal humerus, and is being repaired. A nerve lying immediately lateral to the brachial artery is also lacerated. Which of the following is the nerve most likely to be? A. Ulnar nerve B. Median nerve C. Radial nerve D. Intercostobrachial nerve E. Axillary nerve

Prepared by Dr: Mohammed Musa Brema Idress – My best wishes Page 325

ANSWER IS B The brachial artery begins at the lower border of teres major and terminates in the cubital fossa by branching into the radial and ulnar arteries. In the upper arm the median nerve lies closest to it in the lateral position. In the cubital fossa it lies medial to it.

695. What is the course of the median nerve relative to the brachial artery in the upper arm? A. Medial to anterior to lateral B. Lateral to posterior to medial C. Medial to posterior to lateral D. Medial to anterior to medial E. Lateral to anterior to medial

ANSWER IS E Relations of median nerve to the brachial artery: Lateral -> Anterior -> Medial The median nerve descends lateral to the brachial artery, it usually passes anterior to the artery to lie on its medial side. It passes deep to the bicipital aponeurosis and the median cubital vein at the elbow. It enters the forearm between the two heads of the pronator teres muscle.

696. To draw some blood from a patient’s median cubital vein you will insert the needle in the: Single best answer question – choose ONE true option only. A. Posterior aspect of the knee B. Femoral triangle C. Dorsal surface of the hand D. Anterior aspect of the elbow E. Deltopectoral triangle.

ANSWER IS D The veins of the arm carry blood from the extremities of the limb, as well as drain the arm itself. The two main veins are the basilic and the cephalic veins. There is a connecting vein between the two, the median cubital vein, which passes through the cubital fossa on the anterior aspect of the elbow and is clinically important for venepuncture (withdrawing blood).

The median nerve and brachial artery are the deep relations of the median cubital vein and can be damaged during venous access.

697. A 17-year-old male is stabbed over the left lower ribs whilst travelling home from a night out. He is alert and haemodynamically stable on admission. A nasogastric tube is inserted and a chest x-ray shows this to be in the thoracic cavity and a traumatic diaphragmatic injury is thought likely. What is the best imaging technique to visualize the anatomy of the diaphragm? Select one answer only. A. Abdominal CT B. Barium swallow

Prepared by Dr: Mohammed Musa Brema Idress – My best wishes Page 326

C. Chest CT D. Gastrograffin swallow E. MRI

ANSWER IS E Diaphragmatic injuries result from either blunt or penetrating trauma. A traumatic diaphragmatic rupture is more commonly diagnosed on the left side, perhaps because the liver obliterates the defect or protects it on the right side. In addition, the appearance of bowel, stomach or a nasogastric (NG) tube is more easily detected in the left side of the chest. Right diaphragmatic ruptures are rarely diagnosed in the early post-injury period. The liver often prevents herniation of other abdominal organs into the chest.

This, however, may not be representative of the true incidence of laterality and autopsy studies have revealed that left- and right-sided ruptures occur almost equally. Blunt trauma produces large radial tears measuring 5–15 cm, most often at the posterolateral aspect of the diaphragm. In contrast, penetrating trauma usually create only small linear incisions or perforations, which are less than 2 cm in size and may often take some time, even years, to develop into diaphragmatic hernias.

If a laceration of the left diaphragm is suspected, a NG tube should be inserted. If the tube appears in the thoracic cavity on the chest film, the need for special contrast studies can be eliminated. Minimally invasive endoscopic procedures (thoracoscopy) may be helpful in evaluating the injury to the diaphragm in indeterminate cases.

Abdominal computed tomography scan is usually not helpful because of its poor visualization of the diaphragm. Magnetic resonance imaging is more accurate in visualizing the anatomy of the diaphragm. It is very sensitive and specific and so is the investigation of choice. Surgical repair is necessary, even for small tears, because the defect will not heal spontaneously.

698. Which of the following statements concerning the sympathetic nervous system is most accurate? Select one answer only. A. Cell bodies of the postganglionic neurons lie in the intermediolateral horn of the spinal cord B. Efferent neurons leave the spinal cord from all thoracic and lumbar segments C. Preganglionic neurons reach the adrenal medulla without synapsing D. The sympathetic chain extends three quarters of the length of the vertebral column E. The vagus nerve carries postganglionic sympathetic neurons.

ANSWER IS C The sympathetic nerves originate in the spinal cord from segments T1– L2.The preganglionic neurons have their cell bodies in the intermediolateral horn of the spinal cord. They leave the spinal cord in the anterior roots and pass to the paired sympathetic chains, which run the entire length of the vertebral column. There are no sympathetic fibers in the vagus nerve.

Prepared by Dr: Mohammed Musa Brema Idress – My best wishes Page 327

699. A 17-year-old male is seen in A&E after being attacked with a knife. He is haemodynamically normal, but has sustained a deep laceration over the thenar eminence of his left hand. It is decided to explore this wound under general anaesthetic. Which of the following nerves is most likely to be injured following a deep laceration to the thenar eminence? Single best answer - select one answer only. A. Anterior interosseous nerve B. Deep branch of the Ulnar nerve C. Recurrent branch of the median nerve D. Superficial branch of the Ulnar nerve E. Superficial radial nerve.

ANSWER IS C The anterior interosseous nerve arises below the two heads of pronator teres and runs along the interosseous membrane with flexor digitorum profundus and flexor pollicis longus and ends beneath pronator quadratus. The superficial radial nerve lies deep to brachioradialis and as such does not enter the thenar eminence.

The Ulnar nerve passes superficial to the flexor retinaculum through Guyon,s canal. It divides into the superficial and deep terminal branches at the pisiform bone. However, the recurrent branch of the median nerve would be at risk, and so movements of the thumb would be restricted.

700. A 54-year-old obese female presents with intermittent episodes of altered sensation in her right hand which mostly occur at night. The altered sensation occurs in palmar aspect of the hand over the radial 3 and a half digits. She is thought to have a diagnosis of Carpal tunnel syndrome. In addition to the median nerve, the Carpal tunnel also contains the tendons flexor digitorum superficialis (FDS), flexor digitorum profundus (FDP), flexor carpi radialis (FCR) and which other? Single best answer - select one answer only. A. Abductor pollicis brevis (APB B. Abductor pollicis longus (APL) C. Extensor pollicis longus D. Flexor pollicis longus E. Pronator quadrates.

ANSWER IS D The carpal tunnel contains the median nerve, as well as the tendons of the flexor carpi radialis, flexor pollicis longus, and flexor digitorum superficialis and flexor digitorum profundus brevis tendons. The ulnar artery and nerve, and the flexor carpi ulnaris tendon, lie outside the tunnel.

701. You review a 56-year-old woman in the outpatient’s clinic following myocardial revascularization using the internal thoracic arteries. Despite a strong recovery and good postoperative course she complains that since the operation she has had chronic pain in her shoulder and that her shoulder appears to be

Prepared by Dr: Mohammed Musa Brema Idress – My best wishes Page 328 getting smaller. When you examine her you notice atrophy of the shoulder muscles and winging of the scapula. What is the most likely cause of these symptoms? Select one answer only. A. Damage to the axillary nerve B. Damage to the long thoracic nerve C. Damage to the radial nerve D. Damage to the spinal accessory nerve E. Damage to the whole brachial plexus.

ANSWER IS B Damage to the long thoracic nerve causes winging of the scapula owing to weakness of the serratus anterior. Spinal accessory nerve damage will cause weakness in shrugging the shoulders.

The axillary nerve contains fibers of C5 and C6 to supply predominantly the deltoid muscle. It conveys some sensory fibers to the lateral aspect of the forearm. The radial nerve supplies the extensor carpi radialis and ulnaris, plus extensor digitorum, so its injury results in wrist drop. Froment’s (pincer) sign is associated with an ulnar nerve injury, where the adductor pollicis is weak and so the flexor pollicis longus accommodates. When a sheet of paper is placed between the thumb and index finger of a patient with a weak adductor pollicis, he/she will flex the thumb at the interphalangeal joint to grip the sheet. Sciatic nerve injuries result in foot drop, so the high-stepping gait.

702. You are assisting in fitting an inferior vena cava (IVC) filter. The consultant asks you about the anatomy of the IVC. Which one of the following is true? Select one answer only. A. It commences at the level of L3 B. It enters the thoracic cavity at the level of T8 C. It lies anterior to the aorta D. It lies anterior to the caudate lobe of the liver E. It lies to the left of the aorta.

ANSWER IS B The IVC commences opposite the L5 vertebra, formed by the confluence of the right and left common iliac veins. It runs on the right of the aorta upwards towards the diaphragm and extends to the central tendinous diaphragm at the level of the body of the T8 vertebra. The IVC lies posterior to the bare area and caudate lobe of the liver. Due to its compound embryological origin, the tributaries are not identical to the branches of the abdominal aorta. On the right side, the testicular vein drains into the inferior vena cava; but on the left, the testicular vein drains into the left renal vein.

703. A 72-year-old male is undergoing a coronary artery bypass grafting procedure. To allow access to the thorax, a median sternotomy is performed by the operating surgeon, disrupting the manubriosternal joint. What type of joint is this? Single best answer - select one answer only.

Prepared by Dr: Mohammed Musa Brema Idress – My best wishes Page 329

A. Condylar B. Fibrous C. Primary cartilagenous D. Secondary cartilagenous E. Synovial.

ANSWER IS D All joints in the body can be broadly divided into three types: fibrous, cartilaginous (primary and secondary) and synovial (typical and atypical). Fibrous joints unite the bones of the vault of the skull at the sutures; the bone ends are joined by fibrous tissue. Movement is negligible in fibrous joints.

All midline joints – symphysis pubis, manubriosternum, xiphisternum and intervertebral discs – are examples of secondary cartilaginous joints. Primary cartilaginous joints are formed when a bone unites with a cartilage. They are immobile and strong. All epiphyses and the attachments of ribs to their costal cartilages are examples of primary cartilaginous joints.

704. You are treating a patient with portal hypertension and request a scan to look for a portal vein thrombosis. Which of the following would help you locate the portal vein on the scan images? Single best answer question – choose ONE true option only. A. It arises at the union of the splenic and inferior mesenteric veins B. It lies directly anterior to the aorta C. It lies in the free edge of the lesser omentum D. It lies in front of the hepatic artery at the porta hepatis E. It receives the ligamentum teres at its right branch.

ANSWER IS C The portal vein is the continuation of the superior mesenteric vein after it has received the splenic vein behind the neck of the pancreas. It lies behind the bile duct and the hepatic artery. It lies anterior to the inferior vena cava. The ligamentum teres and the paraumbilical veins join the left branch.

705. In locating the ureter on a plain abdominal x-ray, which landmark would you use to find it? A. It crosses the pelvic brim overlying the sacral foramina B. It extends inferiorly from the 12th thoracic vertebra C. It lies slightly medial to the tips of the transverse processes of the lumbar vertebrae D. It passes towards the contralateral ischial spine E. It extends beyond the pubic tubercle.

ANSWER IS C The hila of the kidney lie at around the level of the first lumbar vertebra. The ureters extend inferiorly from here and are projected slightly medial to the tips of the transverse processes of the lumbar vertebrae.

Prepared by Dr: Mohammed Musa Brema Idress – My best wishes Page 330

The ureters cross the pelvic brim at the sacroiliac joints, pass over the aforementioned joints and then towards the ischial spine and then passes towards (and not beyond, on radiographs) the pubic tubercle.

706. You are reviewing a patient on the neurosurgical ward who has a complex skull base tumour described on the MRI brain report as 'invading the superior orbital fissure'. Which of the following nerves pass through the superior orbital fissure? Select one answer only. A. Facial nerve B. Mandibular division of the trigeminal nerve C. Branches of the ophthalmic division of the trigeminal nerve D. Orbital branches of the pterygopalatine ganglion E. Supra-orbital nerve.

ANSWER IS C The lacrimal, frontal and nasociliary branches of ophthalmic nerve (cranial nerve V1), oculomotor nerve (CN III), trochlear nerve (CN IV), abducens nerve (CN VI) and sympathetic nerve fibers all pass through the superior orbital fissure. The supra- orbital nerve passes through the supra-orbital foramen and the orbital branches of the pterygopalatine ganglion pass through the inferior orbital fissure

707. A 47-year-old male is brought in as a trauma call after falling from his motorbike onto his right shoulder. On examination of his right upper limb it is noted he cannot abduct his shoulder or flex his elbow. His forearm is held in a pronated position. He is noted to have an absent biceps tendon reflex. Which of the following parts of the brachial plexus mediate this reflex? Single best answer - select one answer only. A. Lower trunk B. Medial cord C. Middle trunk D. Posterior cord E. Upper trunk

ANSWER IS E The musculocutaneous nerve (C5, C6) is a branch of the lateral cord of the brachial plexus and supplies the biceps. Each C5 and C6 spinal nerve is formed from dorsal (sensory) and ventral (motor) roots, mediating the afferent and efferent reflex arcs respectively. Each spinal nerve divides into a dorsal and ventral ramus. The C5 and C6 ventral rami form the upper trunk of the brachial plexus. The lateral cord is formed from anterior divisions of the upper (C5/C6) and middle (C7) trunks. So, neither the medial cord nor the middle trunk mediates the reflex.

708. A 47-year-old female undergoes low anterior resection to treat a rectal adenocarcinoma. During the surgery, great care is taken to preserve the pelvic splanchnic nerves. From which nerve roots do these arise? Single best answer - select one answer only.

Prepared by Dr: Mohammed Musa Brema Idress – My best wishes Page 331

A. L2-4 B. L3-5 C. L4,5-S1 D. S1-3 E. S2-4

ANSWER IS E The pelvic plexuses supply the viscera of the pelvic cavity. They are situated at the sides of the rectum in men and at the sides of the rectum and vagina in women. The pelvic plexuses are formed on either side by a continuation of the hypogastric plexus, by the sacral parasympathetic efferent fibers from the second, third and fourth sacral nerves and by a few filaments from the first two sacral ganglia.

In general, the muscles of the bladder (detrusor muscle) and rectum are innervated by the pelvic splanchnic nerves, the smooth muscle of the internal sphincter of the bladder through the superior hypogastric plexus and the smooth muscles of the internal sphincter of the anal canal by the branches from the sacral ganglia (which pass through the sacral plexuses). The pelvic splanchnic nerves supply the colon distal to the splenic flexure.

Normal sensations of distension of bladder and rectum pass through the pelvic splanchnic nerves.

709. The inferior sagittal sinus: Single best answer question – choose ONE true option only. A. Drains directly into the confluence of sinuses B. Is found in the falx cerebelli C. Is formed within the free, inferior border of the falx cerebri D. Drains into the superior petrosal sinus E. Contains valves, unlike the other venous dural sinuses

ANWER IS C The inferior sagittal sinus is enclosed in the posterior half or two thirds of the free margin of the falx cerebri. It is cylindrical in shape. It increases in size as it passes backward and ends in the straight sinus. It receives several veins from the falx cerebri and occasionally receives a few veins from the medial surfaces of the hemispheres.

710. Which one of the following muscles in the hand is supplied by the median nerve? Single best answer question – choose ONE true option only. A. Radial two interossei B. Abductor pollicis brevis C. Ulnar two lumbricales D. Adductor pollicis E. Extensor pollicis.

Prepared by Dr: Mohammed Musa Brema Idress – My best wishes Page 332

ANSWER IS B The median nerve supplies the following structures in the hand: The abductor pollicis brevis, flexor pollicis brevis, opponens pollicis The radial two lumbricales The palmar surface of the radial three and half digits The ulnar nerve supplies all the interossei and the rest of the hand muscles.

711. Intervertebral disc prolapse in the lumbar spine most often affects the L4/L5 and L5/S1 discs. In a man presenting with acute back pain following an episode of lifting a heavy weight, reduced force of which of the following movements would most suggest an L4/L5 rather than an L5/S1 disc lesion?. Single best answer question – choose ONE true option only. A. Ankle plantar flexion B. Eversion of the foot C. Extension of great toe D. Inversion of the foot E. Knee extension.

ANSWER IS C In the lumbar spine (in contrast to the cervical spine) nerve roots emerge below their respective vertebrae: thus the majority of L4/5 disc prolapses would be expected to affect the L5 root and the majority of L5/S1 disc prolapses would affect the S1 nerve root. Knee extension is mediated primarily by L3/4; ankle dorsiflexion by L4/L5; inversion of the foot by L4 alone; eversion of the foot by S1, and ankle plantar flexion by S1 and 2. Although L5 contributes to hip abduction and extension, knee flexion and ankle dorsiflexion, weakness is often minimal because of the contribution of other roots to these movements and tends to be maximal in extension of the toes, particularly the great toe.

712. A 32-year-old female complains of a long history of pain and paraesthesiae along the ulnar border of her left arm and forearm precipitated by placing her upper limb in certain positions. She reports episodes of dropping items in her left hand recently and also episodes of left arm swelling. On examination wasting of the small muscles of the left hand is noted. Which of the following is the most likely diagnosis? Select one answer only. A. Carpal tunnel syndrome B. Cubital tunnel syndrome C. Erbs palsy D. Klumpke’s paralysis E. Thoracic outlet syndrome

ANSWER IS E Cervical rib is present in about 0.5% of the population, of which 60% are symptomatic. Symptoms due to the presence of a cervical rib depend on the structure it is compressing. Neurological symptoms are the most common presentation, usually compression of the C8 and T1 nerve roots, which causes pain and paraesthesiae on the ulnar aspect of the arm and forearm and wasting of the small muscles of the hand. Vascular changes are seen less often.

Prepared by Dr: Mohammed Musa Brema Idress – My best wishes Page 333

The arm can become swollen as a result of venous compression. Compression of the subclavian artery can lead to thrombus formation, emboli, ischaemic changes, and even gangrene.

713. You are performing a cranial nerve examination on a patient who has a mild head injury. You ask them to stick out their tongue. With regards to the examination of the tongue: A. It is depressed by the hyoglossus B. It is passive during the voluntary phase of swallowing C. It is protruded by the styloglossus D. It is retracted by the hyoglossus muscle E. It receives sensory innervation from the vagus nerve

ANSWER IS A The sensory innervation to the tongue is from the VII th (facial) and IX th (glossopharyngeal) cranial nerves and the lingual nerve. The tongue deviates to the side of a XII th cranial nerve lesion on protrusion, is active during the first stage of swallowing and contains the lingual tonsil in the dorsum of its posterior third. The tongue is retracted up and back by the styloglossus muscle, protruded by genioglossus and depressed by the hyoglossus.

714. The total length of the trachea is 10cm. At what level does the trachea begin? A. Bifurcation of common carotid artery B. Cricoid cartilage C. Hyoid bone D. Suprasternal notch E. Thyroid cartilage

ANSWER IS B The trachea commences just below the cricoid cartilage (at the level of C6). Within the thorax and on the right, the trachea is in contact with the pleura, vagus and subclavian artery. On its left, the trachea is in contact with the left recurrent laryngeal nerve, aortic arch and the left common carotid and subclavian arteries. The trachea ends at the upper border of T5, where it bifurcates.

715. A 25-year-old is involved in a high speed road traffic accident and complains of left upper quadrant pain. After a normal primary survey and fluid resuscitation, he is deemed haemodynamically stable but complains of persistent left upper quadrant pain. Examination reveals a soft abdomen with moderate left upper quadrant tenderness, and no evidence of peritonism. What would be the most appropriate next step in his management? A. Continue the trauma management sequence B. Diagnostic peritoneal lavage C. Abdominal ultrasonography D. CT Scan of the abdomen with continued resuscitation E. Emergency laparotomy.

Prepared by Dr: Mohammed Musa Brema Idress – My best wishes Page 334

ANSWER IS D The spleen should never normally be palpable even on deep inspiration. Mobilization of the splenic flexure can cause troublesome fractures to the splenic capsule resulting in significant blood loss that can be difficult to stop. Occasionally, splenectomy may be required to stop this.

The splenic artery is a direct branch of the coeliac plexus. The tail of pancreas abuts the splenic hilum and although classified as a „disaster‟ it is possible to cause irreparable damage to this area which contains the islet (B) cells that produce insulin. Appreciation of this anatomy is therefore essential.

The gastro-splenic and spleno-renal (Lieno renal) ligaments: the former carries the short gastric vessels that require ligation during splenectomy and gastrectomy. The scenario suggests splenic trauma.

The mainstay of treatment for splenic trauma is conservation wherever possible. It would be important to confirm splenic injury and severity. The latter is described by the American Association for the Surgery of Trauma and is graded from 1 to 5. Each grade is subdivided into haematoma and laceration with grade 5 including vascular injury to the hilum.

Grades 1 to 4 can be treated non-operatively and approximately 65% of blunt splenic injuries can be treated non-operatively with a 98% success rate. Haematomas are usually subcapsular and lacerations involve the capsule and parenchyma. Grade 4 injuries involve segmental or hilar vessel lacerations with major revascularization to > 25% of the spleen. Of the options offered in the question, the above-mentioned injuries are best delineated by CT scanning of the abdomen.

716. An 82-year-old woman with atrial fibrillation develops a sudden arterial occlusion of her right arm due to a brachial embolism. Which statement pertaining to the arterial system of the upper limb best accords with usual clinical findings? Single best answer question – choose ONE true option only. A. The brachial artery bifurcates into the ulnar and radial arteries just below the level of the elbow crease B. The brachial artery is crossed by the median nerve immediately above the elbow C. A large single brachial vein accompanies the artery on its medial side D. Profunda brachii arises from the brachial artery a hand’s breadth above the elbow E. A brachial artery embolus is especially serious because of the poor collateral circulation around the elbow joint.

ANSWER IS A The median nerve crosses from medial to lateral at the mid-humerus. The artery is accompanied by two vena comitantes and gives off its profunda branch near the upper end of the humeral shaft, where it accompanies the radial nerve. As with all joints, there is an excellent circulation around the elbow joint.

Prepared by Dr: Mohammed Musa Brema Idress – My best wishes Page 335

717. A 19-year-old male is stabbed in the back suffering a left lateral hemisection of the spinal cord at the T5 segment. Which of the following findings is most likely? Select one answer only. A. Loss of ipsilateral ankle jerk B. Loss of pain in the ipsilateral foot C. Babinski sign in the ipsilateral foot D. Preservation of fine touch sensation in the ipsilateral foot E. Wasting of the contralateral quadriceps muscle.

ANSWER IS C Lesions involving hemisection of the cord may cause an ipsilateral loss of fine touch and a contralateral loss of pain and temperature sensation below the lesion. There is an ipsilateral upper neuron lesion, with brisk reflexes, a Babinski sign, but no muscle wasting.

718. The popliteal fossa forms the posterior aspect of the knee. Which structure forms the floor of the popliteal fossa? A. The biceps femoris B. The gastrocnemius C. The knee joint capsule D. The semimembranosus E. The semitendinosus.

ANSWER IS C The popliteal fossa lies behind the knee and is diamond shaped. It is bound above laterally by the biceps femoris and above medially by the semimembranosus. It is bound by the two heads of the gastrocnemius below. In addition to lymph nodes the fossa contains (from superficial to deep) branches of the sciatic nerve, the popliteal vein, popliteal artery.

The sciatic nerve divides into the tibial and common peroneal nerves. The floor of the popliteal fossa is formed by the posterior surface of the knee joint capsule, and by the posterior surface of the femur.

719. A 65-year-old diabetic with hypertension and a long smoking history is due to undergo a CABG for triple vessel disease. Which of the following grafts would be likely to have the highest patency rates at 10 years? Select one answer only. A. Left internal mammary B. Long saphenous vein C. Radial artery D. PTFE graft E. Cryopreserved allograft vein.

ANSWER IS A During the first year following a CABG up to 15% of vein grafts occlude. Between 1–6 years the graft attrition is 1% to 2% per year and between 6–10 years it is 4% per year. By 10 years after bypass surgery, only 60% are patent and only 50% are free of

Prepared by Dr: Mohammed Musa Brema Idress – My best wishes Page 336 significant stenosis. The internal thoracic (mammary) artery has a reported patency of up to 90% at 10 years.

The left internal thoracic artery is usually anastomosed to the left anterior descending artery (or anterior interventricular artery). A number of studies have been carried out to assess the use of alternative grafts for coronary bypass surgery, including PTFE and cryopreserved allograft veins (CAVs). The results so far have been disappointing.

720. A 72-year-old male presents with a swelling in the anterior triangle of the neck. Biopsy confirms a superficial submandibular gland carcinoma. Which structure is least likely to be involved? Select one answer only. A. Deep cervical fascia B. Platysma C. The cervical branch of the facial nerve D. The facial artery E. The facial vein

ANSWER IS C The submandibular gland is a lobulated gland made up of a superficial and a deep part, which are continuous with each other around the posterior border of the mylohyoid muscle. Part of the gland lies infero-laterally, enclosed in an investing layer of deep cervical fascia, platysma muscle and skin. Laterally it is crossed by the cervical branch of the facial nerve and vein.

The facial artery is related to the posterior and superior aspects of the superficial part of the gland. The marginal mandibular branch of the facial nerve passes beneath platysma and depressor angulioris in the proximity of the submandibular gland where it may be damaged during surgery.

721. You are the neurosurgery CT2 attending a trauma call for a patient with head and facial injuries. Following a CT head, the radiologist calls you to say the patient has a fracture through the superior orbital fissure. Which of the following nerves pass through the superior orbital fissure? Select one answer only. A. Mandibular division of the trigeminal nerve B. Maxillary division of the trigeminal nerve C. Oculomotor nerve D. Orbital branches of the pterygopalatine ganglion E. Supra-orbital nerve

ANSWER IS C The oculomotor nerve (cranial nerve III), trochlear nerve (CN IV), ophthalmic nerve (CN Va), abducent nerve (CN VI) and sympathetic nerve fibers all pass through the superior orbital fissure. The supra-orbital nerve passes through the supra-orbital foramen and the orbital branches of the pterygopalatine ganglion pass through the inferior orbital fissure.

Prepared by Dr: Mohammed Musa Brema Idress – My best wishes Page 337

722. A 38-year-old builder’s labourer sustained a severe fracture of his left elbow, which damaged the ulnar nerve behind the medial epicondyle of the humerus. A month later, he still has a total ulnar nerve paralysis. Which clinical sign is most likely to be present on examination? Single best answer question – choose ONE true option only. A. Sensory loss over the ulnar 3½ digits on the ulnar side of the hand B. Inability to grip a sheet of paper between his fingers when the hand is placed flat on the table C. Excessive sweating over the ulnar border of the left hand D. Index and middle fingers on the affected side are held in the claw position E. Marked wasting of the thenar eminence.

ANSWER IS B The ulnar nerve (usually) supplies sensation to the skin of the fifth and the ulnar side of the fourth finger, front and back. Injury to the ulnar nerve may result in sympathetic interruption, with absence of sweating in the affected area.

The thenar muscles are supplied by the median nerve and are therefore spared. Although the fourth and fifth digits are held in the clawed position when the nerve is injured at the wrist, a high lesion paralyses the long flexors to these two fingers and results in the loss of this sign. A test for paralysis of the palmar interossei, supplied by the ulnar nerve, is the inability to adduct the fingers and thus to be unable to grip a sheet of paper between them.

723. In a 25-year-old football player undergoing an inguinal hernia repair, what would the surgeon note in order to identify the superficial inguinal ring? Single best answer - select one answer only. A. A V-shaped defect in the external oblique B. Intercrural fibers C. The conjoint tendon and the transversalis fascia D. That the ilioinguinal nerve is within the spermatic cord E. That the inferior epigastric vessels are lateral to the deep inguinal ring

ANSWER IS B The inguinal canal is a short oblique passage through the anterior abdominal wall. The relatively weak transversalis fascia and the conjoint tendon form the posterior wall. The deep ring is a hiatus in the transversalis fascia, with the inferior epigastric vessels lying immediately medial to it.

The superficial inguinal ring is a triangular hiatus in the external oblique aponeurosis. The visible landmark for the superficial inguinal ring are the intercrural fibers that run at right angles across the external oblique aponeurosis.

In females the inguinal canal carries the round ligament of the uterus and the ilioinguinal nerve. In males, the ilioinguinal nerve is carried in the inguinal canal but is not found within the spermatic cord.

Prepared by Dr: Mohammed Musa Brema Idress – My best wishes Page 338

724. A 32-year-old man is brought in after a RTA in which he was doing 40 mph and collided with an on-coming vehicle after he lost concentration momentarily. He was not wearing a seatbelt and a sternal fracture is seen on imaging. There is concern about a myocardial contusion. What is the best way to establish this diagnosis? Select one answer only. A. ECG B. Direct inspection of myocardium C. Serial troponin levels and ECG D. Transoesophageal echo E. Transthoracic echo.

ANSWER IS C Blunt cardiac injury, commonly seen in patients with decelerating trauma, can cause valvular disruption, pericardial effusion, myocardial muscle contusion and cardiac chamber rupture. Patients with myocardial contusion may complain of chest discomfort/pain and may be hypotensive. However, cardiogenic shock is rarely seen with myocardial contusion alone.

A two-dimensional echocardiography may reveal an abnormal ventricular wall motion. The electrocardiographic changes are variable: They include multiple premature ventricular contractions, unexplained sinus tachycardia, atrial fibrillation, bundle branch block (usually right), non-specific ST and T wave changes and features suggestive of frank myocardial infarction.

In current practice, the widespread availability of cardiospecific troponin I and troponin T assays has made it easier to detect myocardial injury, especially in patients with no major signs of cardiac injury. Troponin I and troponin T have also facilitated the stratification of patients at risk for life threatening complications.

Serial measurements of troponin I or T in combination with ECG have shown to be sufficient for identifying the vast majority of patients at risk, thus avoiding extensive diagnostic screening. Moreover, patients with normal troponin I or T concentrations and ECG may be safely discharged after a period of observation.

725. An anaesthetist performs a successful block of the median nerve at the elbow. Which neurological sign is likely to be present on examination? Single best answer question – choose ONE true option only. A. Inability to flex the fingers B. An obvious wrist drop deformity C. The palm of the hand is totally anaesthetized D. Inability to abduct and adduct the fingers E. Loss of sensation over thenar eminence

ANSWER IS E The median nerve supplies all the muscles in the anterior compartment of the forearm, apart from the flexor carpi ulnaris and the ulnar half of the flexor digitorum

Prepared by Dr: Mohammed Musa Brema Idress – My best wishes Page 339 profundus; so the little and ring fingers can still be flexed. The radial nerve supplies the extensors – hence no wrist drop.

The ulnar nerve supplies the skin of the ulnar side of the hand, hence no anaesthesia there. It also supplies the interossei muscles of the hand, which effect abduction and adduction of the fingers. Absence of thumb abduction, due to paralysis of abductor pollicis brevis, is a good test for median nerve paralysis

726. Of the spectrum of deformities associated with Pes Cavus, which of these is caused by compensatory use of the extensor digitorum to dorsiflex the ankle? A. Calluses B. Clawing of the Toes C. Contracture of the Plantar Fascia D. Increased Calcaneal Angle E. Metatarsalgia.

ANSWER IS B Pes cavus is a high arch of the foot that does not flatten with weight bearing. The etiology of pes cavus can be identified approximately 80% of the time. The causes include malunion of calcaneal or talar fractures, burns, sequelae resulting from compartment syndrome, residual clubfoot, and neuromuscular disease. The remaining 20% of cases are idiopathic and nonprogressive.

Identifying the etiology is essential to determine if the deformity is progressive, which assists in operative planning. All of these deformities above occur in Pes Cavus, however clawing of the toes occurs when the intrinsic muscles develop contractures and the long extensor to the toes (extensor digitorum), recruited to assist in ankle dorsiflexion, causes claw toe deformity due to a change in its axis of pull.

727. Which nerve is potentially at greatest danger of iatrogenic injury during Total Hip Arthroplasty via the posterior approach? A. Femoral Nerve B. Lateral Cutaneous Nerve of the Thigh C. Obturator Nerve D. Sciatic Nerve E. Superior Gluteal Nerve

ANSWE IS D Iatrogenic injury to the femoral or obturator nerves are rare, accounting for less than 10% of all nerve injuries following hip surgery. Schmalzreid et al conducted a large study documenting 3126 consecutive total hip replacements and found that 80% of nerve injuries occurred to the sciatic nerve (47% peroneal division alone, 30% to peroneal and tibial divisions, 3% to tibial division alone). Lateral skin incision should be planned to avoid the lateral cutaneous nerve of the thigh. The superior gluteal nerve is retracted with the muscle intra-op and is unlikely to be injured.

Prepared by Dr: Mohammed Musa Brema Idress – My best wishes Page 340

728. A 64-year-old man is admitted with a tearing chest pain radiating through to his back. He is haemodynamically stable and a CT angiogram shows a type A thoracic aneurysm with dissection. Which of the following vessels normally arise from the aortic arch? Select one answer only. A. Left subclavian artery B. Left vertebral artery C. Right internal mammary artery D. Right common carotid artery E. Right subclavian artery

ANSWER IS A The arch of the aorta commences from the manubriosternal joint and passes backwards over the left bronchus to reach the body of T4 vertebra just to the left of the midline. The arch is crossed on its left side by the phrenic and vagus nerves as they pass downwards in front of and behind the lung root, respectively. The left vertebral artery and the right internal mammary artery come off the left and right subclavian arteries, respectively.

729. A 21-year-old male is seen in A&E after an assault outside a night club with pain in his jaw. An X-ray confirms a fracture of the mandible. Which of the following regions is the weakest part of the mandible? Select one answer only. A. Angle B. Canine region C. Coronoid process D. Head E. Ramus.

ANSWER IS B Mandibular fractures are commonly the result of blows to the face and it is not uncommon for both sides to be affected. Fractures of the condyles lead to temporo- mandibular joint dislocation and usually are as a result of a blow to the chin. The canine region is the weakest part of the bone due to the length of the root. Angle fractures can run downwards and forwards, which leads to impaction and so prevents displacement. Fractures of the body always cause laceration to the buccal mucosa.

730. A 78-year-old man had poliomyelitis as a child, which left him with total paralysis of the left deltoid muscle. Which feature is most likely to be present on clinical examination? Single best answer question – choose ONE true option only. A. Anaesthesia over the „epaulette‟ region of the left shoulder B. The acromion process of the scapula forms the most lateral bony landmark of the left shoulder C. Drooping of the left shoulder compared to the right side D. Detectable weakness in drawing the arm forward and internally rotating the shoulder when this is compared with the right side E. Full abduction of the shoulder can be achieved by the action of the intact supraspinatus muscle on the left side.

Prepared by Dr: Mohammed Musa Brema Idress – My best wishes Page 341

ANSWER IS D Even if the supraspinatus is fully functional, it is far too weak a muscle to be able to abduct the whole weight of the arm. The deltoid, in addition to being the powerful abductor of the humerus, also assists in flexion and medial rotation (and extension and lateral rotation) of the shoulder by means of its anterior and posterior fibers, respectively. Weakness of these movements compared to the normal side can be detected on careful examination.

731. An 18-year-old male was thrown from moving car suffering a severe head injury in the process. He is placed on the ITU and an intra-cranial pressure bolt is inserted. Once correctly calibrated this gives a reading of 25mmHg.Which of the following is true about cerebral blood flow (CBF)? Single best answer - select one answer only. A. In the normal subject CBF fluctuates widely between a lying and a standing position B. It is directly autoregulated by arterial PCO2 C. It is directly autoregulated by arterial PO2 D. It is increased in the normal subject by the administration of mannitol E. It is unrelated to intracranial pressure

ANSWER IS C Cerebral perfusion pressure is calculated by the mean arterial blood pressure minus the intracranial pressure. It is tightly regulated so that perfusion pressure, and hence cerebral blood flow, fluctuates very little despite many postural changes. The autoregulatory stimuli are PO2 and pH (PCO2 exerts its effect via dissociation into bicarbonate and hydrogen ions and hence a fall in pH).

Mannitol is used to reduce intracranial oedema, and hence intracranial pressure, in cases of head injury. This has the effect of increasing cerebral blood flow, but it has no effect in normal subjects.

732. Following excision of a left cervical rib a 28-year-old female is found to have a milky, white fluid in her drain. What structure is most likely to have been damaged? Select one answer only. A. Cisterna chyli B. Hemiazygos vein C. Left subclavican artery D. Left subclavican vein E. Thoracic duct

ANSWER IS E The thoracic duct leaves the cysterna chyli at the level of L1–L2, ascends into the thorax to the right of the descending thoracic aorta, crosses the midline gradually to reach the left border of the oesophagus (plane of Louis, T4), where it continues to run upwards, reaching the root of the neck.

Prepared by Dr: Mohammed Musa Brema Idress – My best wishes Page 342

It then curves behind the carotid sheath and arches over the left subclavian artery to drain into the left brachiocephalic vein, although it can also drain into the other adjacent major veins. It carries lymph from the lower limbs, abdominal and pelvic regions, left thorax, left head and neck plus the left arm.

733. A 27-year-old male is rushed into A&E after being involved in a RTA. From the history he was a front seat passenger in a stolen car that crashed into a wall at approximately 70mph. He is complaining of generalized pain and on examination prominent marks are seen corresponding to his seatbelt. His trauma series chest x- ray shows an obliterated aortic knuckle and there is concern about blunt aortic rupture. Which of the following imaging modalities is the gold standard in diagnosing this? Select one answer only. A. Aortography B. Contrast- enhanced CT of the chest C. MRI D. Transoesophageal echocardiography E. Transthoracic echocardiography

ANSWER IS A Traumatic aortic disruption, a time-sensitive injury, is a common cause of sudden death after an automobile collision or a fall from great height. A complete tear through the tunica intima, media and adventitia usually leads to rapid exsanguination and death. In aortic rupture survivors, immediate death is prevented due to the vascular continuity maintained by a pseudoaneurysm within an intact adventitial layer or a mediastinal haematoma.

A large mediastinal haematoma may shift the trachea to the right. This condition has a variable course ranging from a relatively clinically silent period due to the contained rupture (pseudoaneurysm), to rupture of the pseudoaneurysm, exsanguination and death. Radiographic findings may include a widened mediastinum, obliteration of the aortic knuckle, deviation of the trachea to the right, obliteration of the space between the pulmonary artery and the aorta (obscuration of AP (aorto-pulmonary) window), depression of the left main stem bronchus, deviation of the oesophagus (nasogastric tube) and fractures of the first or second rib or scapula.

False-positive and false-negative findings occur with each radiographic sign and, rarely (1–2%), no mediastinal or initial chest X-ray abnormality is present in patients with great vessel injury. Although transoesophageal echocardiography is a useful, the less invasive diagnostic tool, aortography is the gold standard in the diagnosis of blunt aortic rupture. Helical contrast-enhanced computed tomography (CT) of the chest is also an accurate screening method for patients with suspected blunt aortic injury. However, a patient who is haemodynamically abnormal should not be placed in a CT scanner.

In stable patients, if enhanced helical CT of the chest is negative for mediastinal haematoma and aortic rupture, no further diagnostic imaging is necessary. If it is

Prepared by Dr: Mohammed Musa Brema Idress – My best wishes Page 343 positive for blunt aortic rupture, the extent of the injury can best be ascertained by aortography.

734. A 50-year-old carpenter presented with swelling and pain anterior to his knee. Which knee bursa is likely to be involved? A. The deep infrapatellar bursa behind the infrapatellar bursa B. The popliteal bursa protecting the patellar tendon C. The prepatellar bursa protecting the patella D. The superficial infrapatellar bursa which lies between the ligamentum patellae and skin E. The suprapatellar bursa which communicates with the knee joint.

ANSWER IS C There are four anterior bursae: suprapatellar, prepatellar, superficial infrapatellar and deep infrapatellar. There are two posterior bursae: popliteal and semimembranosus. The deep infrapatellar bursa lies between the ligamentum patellae and the tibia. The superficial infrapatellar bursa lies between the skin and the lower half of the ligamentum patellae. The prepatellar bursa lies in the subcutaneous tissue over the lower half of the patella and upper part of the ligamentum patellae.

735. A cerebral angiogram is performed on a 37-year-old woman, following a suspected aneurysmal bleed. Which anatomical feature should be considered when interpreting the angiogram? Single best answer question – choose ONE true option only. A. The middle cerebral artery is the largest single component of the circle of Willis B. The posterior cerebral artery is clearly seen on a lateral carotid angiogram C. The vertebral arteries meet at the foramen magnum to form the basilar artery D. The middle cerebral artery courses over the lateral aspect of the temporal lobe of the cerebrum E. The middle meningeal artery is an extracranial branch of the internal carotid artery

ANSWER IS A The middle meningeal artery is a branch of the maxillary artery, one of the terminal branches of the external carotid. The posterior cerebral artery arises from the termination of the basilar artery, which itself arises from the two vertebral arteries that meet on the under surface of the brainstem – so the posterior cerebral artery can only be visualized by vertebral angiography. The middle cerebral artery is indeed the largest component of the circle of Willis, being, in effect, the termination of the internal carotid. It passes through the lateral sulcus of the cerebrum between the temporal and frontal lobes.

736. What is the normal location of the major duodenal papilla? Single best answer question – choose ONE true option only.

Prepared by Dr: Mohammed Musa Brema Idress – My best wishes Page 344

A. Superior part of the duodenum B. Ascending part of the duodenum C. Descending part of the duodenum D. Horizontal part of the duodenum E. Duodenal bulb.

ANSWER IS C The major duodenal papilla is situated at the medial side of the descending portion of the duodenum, a little below its middle and about 7 to 10 cm from the pylorus. The common bile and pancreatic ducts unite and open by a common orifice on the summit of the duodenal papilla.

737. A 28-year-old man, who is a keen bodybuilder, presents with a short history of left upper limb discomfort and difficulty in moving his shoulder. On examination he is noticed to have winging of the left scapula. There is no wasting of the shoulder girdle muscles. With stabilization of the scapula, he has a full range of movement and is able to elevate the shoulder. Sensory testing is normal, as are upper limb reflexes. What is the likely anatomical origin of his problem? Single best answer question – choose ONE true option only. A. C3, 4 nerve root B. Long thoracic nerve C. Diffuse left brachial plexus injury D. Spinal accessory nerve (cranial nerve XI) E. C5, 6 nerve root

ANSWER IS B Seven muscles attach the scapula (shoulder blade) to the chest wall and help maintain normal scapular control. These muscles are trapezius, levator scapulae, rhomboids major, rhomboids minor, pectoralis minor, omohyoid and serratus anterior. The latissimus dorsi has a small attachment at the base of the scapula but does not significantly contribute to scapular stability.

Of these muscles, the serratus anterior and the trapezius are the most important. A winging scapula is nearly always associated with partial or complete paralysis of either of these muscles. Weakness or paralysis of the serratus anterior, secondary to palsy of the long thoracic nerve, is the commonest cause of winging.

The long thoracic nerve (origin C5,6 motor roots, with sometimes a contribution from C4 +/- C7) is thin, fragile and runs an anatomical course in the neck and upper thorax that makes it susceptible to damage by compression or trauma. Commoner causes include: surgery (e.g. radical mastectomy, lymph node biopsy from axilla); stretch injury during sports (as in this case); viral/ post-infectious (brachial neuritis); other causes of neuropathy (vascular, toxic etc.).

Accessory nerve (XI) damage can also produce scapular winging via weakness of trapezius, but this would be milder and would be expected to be associated with weakness of shoulder elevation, which this patient does not have. The other options

Prepared by Dr: Mohammed Musa Brema Idress – My best wishes Page 345 listed would tend to be associated with other symptoms and signs. Nerve conduction studies and electromyography would help confirm the diagnosis.

738. In a 27-year-old restrained passenger from a high speed collision with a tree, you suspect a deceleration injury to the aorta. Which feature on chest X-ray would most reliably support your suspicion? Single best answer question – choose ONE true option only. A. Obliteration of the aortic knuckle B. Obscuration of the aorto-pulmonary window C. Presence of a pleural cap D. Depression of the left main stem bronchus E. Widened mediastinum

ANSWER IS E All of the above can be associated with a traumatic aortic injury. However, they are non-specific signs with high false positive and false negative rates. Occasionally, no chest X-ray changes are visible following traumatic aortic injury. In a trauma supine CXR, a widened mediastinum is the most consistent feature of aortic disruption. About 3% of these patients would prove to have aortic injury on contrast CT or aortic angiogram.

739. A 23-year-old motorcyclist is involved in a road traffic accident and sustains a fracture of the femoral shaft. Which prominent ridge runs along the posterior aspect of the femoral shaft? Intertrochanteric ridge A. Linea alba B. Linea albuginea C. Linea aspera D. Linea semilunaris

ANSWER IS D The linea aspera runs along the posterior aspect of the femoral shaft. It is the point of attachment for many muscles of the lower limb, and for the intermuscularseptum. The intertrochanteric ridge is positioned at the proximal anterior femur.

740. You are called to the Trauma Room of the Emergency Department and find a 34-year-old motorcyclist with a blunt chest injury. On examination you notice reduced breath sounds and a dull percussion note over the right hemithorax. Trauma care is initiated and includes the insertion of an intercostal drain which quickly yields 1600mls of blood. His blood pressure is recorded as 100/65mmHg. What is the most appropriate management of this injury? A. Continued non-surgical management B. Emergency thoracoscopy C. Emergency thoracotomy in the Emergency Department D. Emergency thoracotomy in the Operating Theatre E. Massive blood transfusion.

Prepared by Dr: Mohammed Musa Brema Idress – My best wishes Page 346

ANSWER IS D This patient has a massive haemothorax, which is defined as the rapid accumulation of more than 1500mL (or one third) of a patient’s blood volume in the chest cavity. ATLS guidelines recommend that emergency thoracotomy is performed by a thoracic surgeon in an operating theatre environment in such cases. Blood transfusion may be required, but massive transfusion may not be necessary if surgical methods of haemostasis are rapidly delivered.

Emergency room thoracotomy should only be performed in the context of: i) penetrating chest injury plus cardiac arrest (with previously witnessed cardiac activity) or unresponsive hypotension (<70mmHg SBP). ii) blunt chest injury with massive haemothorax and unresponsive hypotension (<70mmHg).

741. In an Urology MDT, a pelvic MRI of a 45-year-old gentleman with HIV, presenting with acute on chronic urinary retention is discussed. It shows an irregular mass encasing the urethra within the deep perineal pouch. In relation to the deep perineal pouch, which is the most likely structure/tissue that the mass will invade? Single best answer - select one answer only. A. The bulbourethral glands B. The internal urethral sphincter C. The penile urethra D. The pudendal nerve E. The superficial perineal fascia

ANSWER IS A In men, the deep perineal pouch contains the membranous urethra, sphincter urethra, bulbourethral glands, deep transverse perineal muscles, internal pudendal vessels and dorsal nerves of the penis. The bulb of the penis lies in the superficial perineal pouch. The superficial perineal fascia forms the boundary for the superficial perineal pouch. The bulbourthethral glands lie on each side of the membranous urethra.

742. Stimulation of the sympathetic nervous system leads to which of be following? Select one answer only. A. Bronchial constriction B. Decreased sweating C. Increased peristalsis D. Inhibition of lipolysis E. Peripheral vasoconstriction

ANSWER IS E Stimulation of the sympathetic nervous system occurs in response to stress. It leads to an increase in heart rate, bronchial dilatation, sweating, reduced peristalsis, and constriction of peripheral blood vessels. There is also an increase in lipolysis

Prepared by Dr: Mohammed Musa Brema Idress – My best wishes Page 347

743. Following a rock-climbing accident in which a foothold gave way, leaving him suspended by one arm, a young man develops weakness of his right hand. He can manoeuvre his arm into any position but cannot use the hand effectively. What structure is most likely to have been damaged in this accident? Single best answer question – choose ONE true option only. A. The C6 nerve root B. The C7 nerve root C. The T1 nerve root D. The ulnar nerve E. The radial nerve

ANSWER IS C He has sustained an injury to the brachial plexus, affecting the lowest roots (C8, T1), which provides the motor supply to the intrinsic muscles of the hand and the long flexors and extensors of the fingers. This deformity is known as Klumpke‟s paralysis. C6 and C7 mediate the movements of the shoulder and elbow, and C7 the movements of the elbow and wrist. The radial nerve is required for normal positioning of the arm. A proximal ulnar nerve lesion affects the small muscles of the hand and wrist flexion but not the positioning of the arm.

744. A 50-year-old man who had suffered long-standingback pain presents to the accident and emergency department with a sudden deterioration and pain radiating to the lateral malleolus. What is the most likely cause of these symptoms? A. A prolapsed intervertebral disc at the L4/5 interspace B. A prolapsed intervertebral disc at the L5/S1 interspace C. A prolapsed intervertebral disc compressing the cauda equina D. A prolapsed intervertebral disc compressing the sacral segments of the cord E. Collapse of a vertebral body secondary to osteoporosis with subsequent compression of the spinal canal.

ANSWER IS B A prolapsed L5–S1 disc presses on the S1 spinal nerve (the L5 nerve passes above the prolapsed disc in the intervertebral foramen and so escapes damage). At the level of prolapse, the spinal canal contains the cauda equina and not cord per se. The S1 dermatome lies over the lateral malleolus. Exaggerated reflexes are diagnostic of an upper motor neuron lesion.

The S2 dermatome occupies the posterior aspect of the calf. Osteoporosis can cause painful collapse of vertebral bodies but rarely causes neurological symptoms and most often affects older women.

745. A 43-year-old female presents with pain and paraesthesia of the medial aspect of the foot, exacerbated by walking. She is diagnosed with tarsal tunnel syndrome and receives appropriate management. Which nerve has been injured? A. Common peroneal nerve B. Femoral nerve

Prepared by Dr: Mohammed Musa Brema Idress – My best wishes Page 348

C. Lateral cutaneous nerve D. Sural nerve E. Tibial nerve

ANSWER IS E The tibial nerve originates from the sciatic nerve. It runs inferior to the flexor retinaculum of the foot alongside the tibial artery and vein, the tibialis posterior muscle, and the flexor hallucislongus and flexor digitorumlongus muscles. If the nerve is compressed within the tarsal tunnel, then the clinical syndrome described above can result.

746. You are assisting your neurosurgical consultant performing an anterior cervical discectomy. He is concerned about injuring the recurrent laryngeal nerve. Which of the following is correct regarding the right recurrent laryngeal nerve? Single best answer - select one answer only. A. Arises from a cranial nerve passing through the foramen magnum B. Is related to the inferior thyroid artery C. Passes behind the ligamentum arteriosum D. Runs posterior to the oesophagus but anterior to the trachea E. Winds around the aortic arch.

ANSWER IS B The recurrent laryngeal nerves arise from the vagus. On the right, the recurrent laryngeal nerve winds around the subclavian artery; on the left, it winds around the aortic arch, passing behind the ligamentum arteriosum. Both nerves run in a groove between the trachea and oesophagus and are closely related to the inferior thyroid artery.

747. Following stimulation of the parasympathetic nervous system which one of the following are you likely to observe? Select one answer only. A. Pupillary dilation B. Increased heart rate C. Smooth muscle relaxation D. Reduced glandular secretion E. Decreased force of contraction of the heart

ANSWER IS E The parasympathetic nervous system functionally antagonises the sympathetic nervous system. Stimulation reduces heart rate and force of contraction, constricts the pupils, stimulates smooth muscle contraction and increases glandular secretion.

748. When operating on a femoral hernia in the femoral triangle, care should be taken to avoid damage to which structure? A. Large bowels could become incarcerated in the hernia B. The cribriform fascia transmits the superficial branches of the femoral artery C. The deep inguinal nodes those lie lateral to the femoral vein D. The femoral vein which lies medial to the femoral nerve

Prepared by Dr: Mohammed Musa Brema Idress – My best wishes Page 349

E. The profunda femoris artery which lies medial to the femoral artery

ANSWER IS D The superficial epigastric and superficial external pudendal arteries pass through the saphenous opening (the superficial circumflex and deep external pudendal arteries pierce the fascia lata). The femoral nerve, artery and vein lie in that order from lateral to medial. The profunda femoris artery is a lateral branch of the femoral artery. The deep inguinal nodes lie medial to the femoral vein. The femoral sheath encloses the femoral vessels for up to 3 cm beyond the inguinal ligament, where the sheath terminates by fusing with the adventitia of both vessels.

749. A 63-year-old female undergoes a subtotal thyroidectomy for a toxic multinodular goiter. Which of the following statements is most likely to be true? Select one answer only. A. Damage to the external laryngeal nerve causes loss of low pitched phonation B. The inferior thyroid artery arises from the origin of the external carotid artery C. The inferior thyroid artery should be ligated as far laterally as possible D. The isthmus lies anterior to the thyroid cartilage E. The superior thyroid artery enters the upper pole of the thyroid gland close to the recurrent laryngeal nerve.

ANSWER IS C The superior thyroid artery arises from the external carotid artery and enters the upper pole of the thyroid gland close to the external laryngeal nerve, which supplies the cricothyroid muscle, a tensor of the vocal cord. Damage to this nerve causes the loss of high-pitched phonation. The inferior thyroid artery, absent in 5%, arises from the thyrocervical trunk of the subclavian artery.

The inferior thyroid artery should be ligated as far laterally as possible to avoid damaging the recurrent laryngeal nerve. Damage to one recurrent laryngeal nerve causes a weakened voice, damage to both causes semiadduction and respiratory difficulties. The isthmus is normally in front of the second and third tracheal rings, although variations are common.

750. The posterior tibial artery is a branch of the popliteal artery and supplies the posterior compartment of the leg. What is the main branch of the posterior tibial artery? A. Dorsalis pedis artery B. Gives rise to the anterior tibial artery C. Gives rise to the peroneal artery D. Lateral calcaneal branch E. Medial and lateral genicular arteries.

ANSWER IS C The popliteal artery divides into the anterior and posterior tibial arteries at the lower border of the popliteus muscle, anterior to the fibrous arch of the soleus muscle. The

Prepared by Dr: Mohammed Musa Brema Idress – My best wishes Page 350 peroneal artery is the first branch of the posterior tibial artery. At the ankle joint, the posterior tibial artery passes deep to the flexor retinaculum.

751. Which of the following is not a content of the cavernous sinus? A. Oculomotor nerve B. Internal carotid artery C. Ophthalmic nerve D. Abducens nerve E. Optic nerve

ANSWER IS E Mnemonic for contents of cavernous sinus: O TOM CAT O:occulomotor nerve (III) T:trochlear nerve (IV) O:ophthalmic branch of trigeminal nerve (V1) M:maxillary branch of trigeminal nerve (V2) C:carotid artery (internal carotid) A:abducent nerve (VI) T:trochlear nerve

OTOM=lateral wall components CA= components within sinus The optic nerve lies above and outside the cavernous sinus.

752. Surgical occlusion of which of these structures, will result in the greatest reduction in hepatic blood flow? A. Portal vein B. Common hepatic artery C. Right hepatic artery D. Coeliac axis E. Left hepatic artery

ANSWER IS A The portal vein transports 70% of the blood supply to the liver, while the hepatic artery provides 30%. The portal vein contains the products of digestion. The arterial and venous blood is dispersed by sinusoids to the central veins of the liver lobules; these drain into the hepatic veins and then into the IVC. The caudate lobe drains directly into the IVC rather than into other hepatic veins.

753. A 43 year old man is due to undergo an excision of the sub mandibular gland. Which of the following incisions is the most appropriate for this procedure? A. A transversely orientated incision 3cm below the mandible B. A transversely orientated incision immediately inferior to the mandible C. A vertical incision 3 cm anterior to the angle of the mandible and extending inferiorly D. A transversely orientated incision 2cm above the mandible

Prepared by Dr: Mohammed Musa Brema Idress – My best wishes Page 351

E. A transversely orientated incision 12cm below the mandible

ANSWER IS A To access the sub mandibular gland a transverse incision 3cm below the mandible should be made. Incisions located higher than this may damage the marginal mandibular branch of the facial nerve.

754. A 5 year old boy presents with recurrent headaches. As part of his assessment he undergoes an MRI scan of his brain. This demonstrates enlargement of the lateral and third ventricles. Where is the most likely site of obstruction? A. Foramen of Luschka B. Foramen of Magendie C. Foramen of Munro D. Aqueduct of Sylvius E. None of the above

ANSWER IS D The CSF flows from the 3rd to the 4th ventricle via the Aqueduct of Sylvius.

Cerebrospinal fluid:- The CSF fills the space between the arachnoid mater and pia mater (covering surface of the brain). The total volume of CSF in the brain is approximately 150ml. Approximately 500 ml is produced by the ependymal cells in the choroid plexus (70%), or blood vessels (30%). It is reabsorbed via the arachnoid granulations which project into the venous sinuses.

Circulation:- 1. Lateral ventricles (via foramen of Munro) 2. 3rd ventricle 3. Cerebral aqueduct (aqueduct of Sylvius) 4. 4th ventricle (via foramina of Magendie and Luschka) 5. Subarachnoid space 6. Reabsorbed into the venous system via arachnoid granulations into superior sagittal sinus

Composition:- Glucose: 50-80mg/dl Protein: 15-40 mg/dl Red blood cells: Nil White blood cells: 0-3 cells/ mm3

755. A 23 year old man presents with appendicitis. A decision is made to perform an appendicectomy. The operation commences with a 5cm incision centered on McBurneys point. Which of the following structures will be encountered first during the dissection? A. External oblique aponeurosis B. Internal oblique muscle

Prepared by Dr: Mohammed Musa Brema Idress – My best wishes Page 352

C. Transversalis fascia D. Rectus sheath E. Peritoneum

ANSWER IS A The external oblique will be encountered first in this location. The rectus sheath lies more medially. The external oblique muscle is the most superficial of the abdominal wall muscles. It originates from the 5th to 12th ribs and passes inferomedially to insert into the linea alba, pubic tubercle and anterior half of the iliac crest. It is innervated by the thoracoabdominal nerves (T7-T11) and sub costal nerves.

756. A 23 year old man is undergoing an inguinal hernia repair. The surgeons mobilize the spermatic cord and place it in a hernia ring. A small slender nerve is identified superior to the cord. Which nerve is it most likely to be? A. Iliohypogastric nerve B. Pudendal nerve C. Femoral branch of the genitofemoral nerve D. Ilioinguinal nerve E. Obturator nerve

ANSWER IS D The ilioinguinal nerve passes through the inguinal canal and is the nerve most commonly identified during hernia surgery. The genitofemoral nerve splits into two branches, the genital branch passes through the inguinal canal within the cord structures. The femoral branch of the genitofemoral nerve enters the thigh posterior to the inguinal ligament, lateral to the femoral artery. The iliohypogastric nerve pierces the external oblique aponeurosis above the superficial inguinal ring.

Ilioinguinal nerve:- Arises from the first lumbar ventral ramus with the iliohypogastric nerve. It passes inferolaterally through the substance of psoas major and over the anterior surface of quadratus lumborum. It pierces the internal oblique muscle and passes deep to the aponeurosis of the external oblique muscle. It enters the inguinal canal and then passes through the superficial inguinal ring to reach the skin.

Branches:- To supply those muscles of the abdominal wall through which it passes. Skin and fascia over the pubic symphysis, superomedial part of the femoral triangle, surface of the scrotum, root and dorsum of penis or labum majus in females.

757. A 34 year old man undergoes excision of a sarcoma from the right buttock. During the procedure the sciatic nerve is sacrificed. Which of the following will not occur as a result of this process? A. Loss of extension at the knee joint B. Foot drop C. Inability to extend extensor hallucis longus

Prepared by Dr: Mohammed Musa Brema Idress – My best wishes Page 353

D. Unchanged sensation to the posterior aspect of the thigh E. Loss of sensation to the posterior aspect of the lower leg

ANSWER IS A Extension of the knee joint is caused by the obturator and femoral nerves. Sensation to the posterior aspect of the thigh is via the posterior cutaneous nerve of the thigh and this is a direct branch from the plexus itself.

758. Where does the spinal cord terminate in neonates? A. L1 B. L2 C. L3 D. L4 E. L5

ANSWER IS C At the 3rd month the foetus's spinal cord occupies the entire length of the vertebral canal. The vertebral column then grows longer exceeding the growth rate of the spinal cord. This results with the cord being at L3 at birth and L1-2 by adulthood.

Spinal cord:- Located in a canal within the vertebral column that affords it structural support. Rostrally it continues to the medulla oblongata of the brain and caudally it tapers at a level corresponding to the L1-2 interspace (in the adult), a central structure, the filum terminale anchors the cord to the first coccygeal vertebra. The spinal cord is characterized by cervico-lumbar enlargements and these, broadly speaking, are the sites which correspond to the brachial and lumbar plexuses respectively.

There are some key points to note when considering the surgical anatomy of the spinal cord:

* During foetal growth the spinal cord becomes shorter than the spinal canal, hence the adult site of cord termination at the L1-2 level.

* Due to growth of the vertebral column the spine segmental levels may not always correspond to bony landmarks as they do in the cervical spine.

* The spinal cord is incompletely divided into two symmetrical halves by a dorsal median sulcus and ventral median fissure. Grey matter surrounds a central canal that is continuous rostrally with the ventricular system of the CNS.

* The grey matter is sub divided cytoarchitecturally into Rexeds laminae.

* Afferent fibers entering through the dorsal roots usually terminate near their point of entry but may travel for varying distances in Lissauers tract. In this way they may establish synaptic connections over several levels

Prepared by Dr: Mohammed Musa Brema Idress – My best wishes Page 354

* At the tip of the dorsal horn are afferents associated with nociceptive stimuli. The ventral horn contains neurons that innervate skeletal muscle.

The key point to remember when revising CNS anatomy is to keep a clinical perspective in mind. So it is worth classifying the ways in which the spinal cord may become injured. These include:

Trauma either direct or as a result of disc protrusion Neoplasia either by direct invasion (rare) or as a result of pathological vertebral fracture Inflammatory diseases such as Rheumatoid disease, or OA (formation of osteophytes compressing nerve roots etc. Vascular either as a result of stroke (rare in cord) or as complication of aortic dissection Infection historically diseases such as TB, epidural abscesses.

The anatomy of the cord will, to an extent dictate the clinical presentation. Some points/ conditions to remember:

Brown- Sequard syndrome-Hemisection of the cord producing ipsilateral loss of proprioception and upper motor neuron signs, plus contralateral loss of pain and temperature sensation. The explanation of this is that the fibers decussate at different levels. Lesions below L1 will tend to present with lower motor neuron signs

759. A 45 year old man is undergoing a low anterior resection for a carcinoma of the rectum. Which of the following fascial structures will need to be divided to mobilize the mesorectum from the sacrum and coccyx? A. Denonvilliers fascia B. Colles fascia C. Sibson’s fascia D. Waldeyers fascia E. None of the above

ANSWER IS D Fascial layers surrounding the rectum: A. Anteriorly: lies the fascia of Denonvilliers B. Posteriorly: lies Waldeyers fascia Waldeyers fascia separates the mesorectum from the sacrum and will need to be divided.

760. A 10 year old child has a grommet inserted for a glue ear. What type of epithelium is present on the external aspect of the tympanic membrane? A. Stratified squamous B. Ciliated columnar C. Non ciliated columnar D. Non stratified squamous

Prepared by Dr: Mohammed Musa Brema Idress – My best wishes Page 355

E. None of the above

ANSWER IS A The external aspect of the tympanic membrane is lined by stratified squamous epithelium. This is significant clinically in the development of middle ear infections when this type of epithelium may migrate inside the middle ear.

Ear- anatomy:- The ear is composed of three anatomically distinct regions.

External ear Auricle is composed of elastic cartilage covered by skin. The lobule has no cartilage and contains fat and fibrous tissue. External auditory meatus is approximately 2.5cm long. Lateral third of the external auditory meatus is cartilaginous and the medial two thirds is bony.

The region is innervated by the greater auricular nerve. The auriculotemporal branch of the trigeminal nerve supplies most the of external auditory meatus and the lateral surface of the auricle.

Middle ear Space between the tympanic membrane and cochlea. The aditus leads to the mastoid air cells is the route through which middle ear infections may cause mastoiditis. Anteriorly the eustacian tube connects the middle ear to the nasopharynx.

The tympanic membrane consists of: 1. Outer layer of stratified squamous epithelium. 2. Middle layer of fibrous tissue. 3. Inner layer of mucous membrane continuous with the middle ear. The tympanic membrane is approximately 1cm in diameter. The chorda tympani nerve passes on the medial side of the pars flaccida.

The middle ear is innervated by the glossopharyngeal nerve and pain may radiate to the middle ear following tonsillectomy.

Ossicles 1. Malleus attaches to the tympanic membrane (the Umbo). 2. Malleus articulates with the incus (synovial joint). 3. Incus attaches to stapes (another synovial joint).

Internal ear Cochlea, semi circular canals and vestibule

Organ of corti is the sense organ of hearing and is located on the inside of the cochlear duct on the basilar membrane.

Prepared by Dr: Mohammed Musa Brema Idress – My best wishes Page 356

Vestibule accommodates the utricule and the saccule. These structures contain endolymph and are surrounded by perilymph within the vestibule.

The semicircular canals lie at various angles to the petrous temporal bone. All share a common opening into the vestibule.

761. A 73 year old lady is admitted with acute mesenteric ischaemia. A CT angiogram is performed and a stenotic lesion is noted at the origin of the superior mesenteric artery. At which of the following levels does this branch from the aorta? A. L1 B. L2 C. L3 D. L4 E. L5

ANSWER IS A The SMA leaves the aorta at L1. It passes under the neck of the pancreas prior to giving its first branch the inferior pancreatico-duodenal artery.

Superior mesenteric artery:- Branches off aorta at L1 Supplies small bowel from duodenum (distal to ampulla of vater) through to mid transverse colon Takes more oblique angle from aorta and thus more likely to receive emboli than coeliac axis

Relations of superior mesenteric artery Superiorly: Neck of pancreas Postero-inferiorly: Third part of duodenum Uncinate process Posteriorly Left renal vein Right Superior mesenteric vein

Branches of the superior mesenteric artery 1. Inferior pancreatico-duodenal artery 2. Jejunal and ileal arcades 3. Ileo-colic artery 4. Right colic artery 5. Middle colic artery

762. The following statements relating to the musculocutaneous nerve are true except? A. It arises from the lateral cord of the brachial plexus B. It provides cutaneous innervation to the lateral side of the forearm C. If damaged, then extension of the elbow joint will be impaired D. It supplies the biceps muscle

Prepared by Dr: Mohammed Musa Brema Idress – My best wishes Page 357

E. It runs beneath biceps

ANSWER IS C It supplies biceps, brachialis and coracobrachialis. If damaged then elbow flexion will be impaired.

Musculocutaneous nerve:- Branch of lateral cord of brachial plexus

Pathway It penetrates the coracobrachialis muscle Passes obliquely between the biceps brachii and the brachialis to the lateral side of the arm Above the elbow it pierces the deep fascia lateral to the tendon of the biceps brachii Continues into the forearm as the lateral cutaneous nerve of the forearm

Innervates 1. Coracobrachialis 2. Biceps brachii 3. Brachialis

763. Which of the following structures does not pass through the foramen ovale? A. Lesser petrosal nerve B. Accessory meningeal artery C. Maxillary nerve D. Emissary veins E. Otic ganglion

ANSWER IS C Mnemonic: OVALE O tic ganglion V3 (Mandibular nerve:3rd branch of trigeminal) A ccessory meningeal artery L esser petrosal nerve E missary veins

Remember for ovale = LOve 1- Lesser petrosal 2- Otic ganglion 3- V3 4- Emissary veins 5- Accessory meningeal

764. Which of the cranial nerves listed below is least likely to carry parasympathetic fibers? A. III B. VII

Prepared by Dr: Mohammed Musa Brema Idress – My best wishes Page 358

C. IX D. X E. II

ANSWER IS E Cranial nerves carrying parasympathetic fibers X IX VII III (1973)

The parasympathetic functions served by the cranial nerves include: 1. III (oculomotor) → Pupillary constriction and accommodation 2. VII (facial) → Lacrimal gland, submandibular and sublingual glands 3. IX (glossopharyngeal) → Parotid 4. X (vagus) → Heart and abdominal viscera - The optic nerve carries no parasympathetic fibers. The cranial preganglionic parasympathetic nerves arise from specific nuclei in the CNS. These synapse at one of four parasympathetic ganglia; otic, pterygopalatine, ciliary and submandibular. From these ganglia the parasympathetic nerves complete their journey to their target tissues via CN V (trigeminal) branches (ophthalmic nerve CNV branch 1, Maxillary nerve CN V branch2, mandibular nerve CN V branch 3)

Cranial nerves:- Cranial nerve lesions Olfactory nerve: May be injured in basal skull fractures or involved in frontal lobe tumour extension. Loss of olfactory nerve function in relation to major CNS pathology is seldom an isolated event and thus it is poor localizer of CNS pathology.

Optic nerve: Problems with visual acuity may result from intra ocular disorders. Problems with the blood supply such as amaurosis fugax may produce temporary visual distortion. More important surgically is the pupillary response to light. The pupillary size may be altered in a number of disorders. Nerves involved in the resizing of the pupil connect to the pretectal nucleus of the high midbrain, bypassing the lateral geniculate nucleus and the primary visual cortex. From the pretectal nucleus neurons pass to the Edinger - Westphal nucleus, motor axons from here pass along with the oculomotor nerve. They synapse with ciliary ganglion neurons; the parasympathetic axons from this then innervate the iris and produce miosis. The miotic pupil is seen in disorders such as Horner's syndrome or opiate overdose.

Mydriasis is the dilatation of the pupil in response to disease, trauma, drugs (or the dark!). It is pathological when light fails to induce miosis. The radial muscle is innervated by the sympathetic nervous system. Because the parasympathetic fibers travel with the oculomotor nerve they will be damaged by lesions affecting this nerve (e.g. cranial trauma).

The response to light shone in one eye is usually a constriction of both pupils. This indicates intact direct and consensual light reflexes. When the optic nerve has an afferent defect the light shining on the affected eye will produce a diminished pupillary response in both eyes. Whereas light shone on the unaffected eye will

Prepared by Dr: Mohammed Musa Brema Idress – My best wishes Page 359 produce a normal pupillary response in both eyes. This is referred to as the Marcus Gunn pupil and is seen in conditions such as optic neuritis. In a total CN II lesion shining the light in the affected eye will produce no response.

Oculomotor nerve: The pupillary effects are described above. In addition it supplies all ocular muscles apart from lateral rectus and superior oblique. Thus the affected eye will be deviated inferolaterally. Levator palpebrae superioris may also be impaired resulting in impaired ability to open the eye.

Trochlear nerve: The eye will not be able to look down. Trigeminal nerve: Largest cranial nerve. Exits the brainstem at the pons. Branches are ophthalmic, maxillary and mandibular. Only the mandibular branch has both sensory and motor fibers. Branches converge to form the trigeminal ganglion (located in Meckels cave). It supplies the muscles of mastication and also tensor veli palatine, mylohyoid, anterior belly of digastric and tensor tympani. The detailed descriptions of the various sensory functions are described in other areas of the website. The corneal reflex is important and is elicited by applying a small tip of cotton wool to the cornea, a reflex blink should occur if it is intact. It is mediated by: the nasociliary branch of the ophthalmic branch of the trigeminal (sensory component) and the facial nerve producing the motor response. Lesions of the afferent arc will produce bilateral absent blink and lesions of the efferent arc will result in a unilateral absent blink.

Abducens nerve: The affected eye will have a deficit of abduction. This cranial nerve exits the brainstem between the pons and medulla. It thus has a relatively long intra cranial course which renders it susceptible to damage in raised intra cranial pressure. Facial nerve Emerges from brainstem between pons and medulla. It controls muscles of facial expression and taste from the anterior 2/3 of the tongue. The nerve passes into the petrous temporal bone and into the internal auditory meatus. It then passes through the facial canal and exits at the stylomastoid foramen. It passes through the parotid gland and divides at this point. It does not innervate the parotid gland. Its divisions are considered in other parts of the website. Its motor fibers innervate orbicularis oculi to produce the efferent arm of the corneal reflex. In surgical practice it may be injured during parotid gland surgery or invaded by malignancies of the gland and a lower motor neuron on the ipsilateral side will result.

Vestibulo-cochlear nerve: Exits from the pons and then passes through the internal auditory meatus. It is implicated in sensorineural hearing loss. Individuals with sensorineural hearing loss will localize the sound in Weber’s test to the normal ear. Rinnes test will be reduced on the affected side but should still work. These two tests will distinguish sensorineural hearing loss from conductive deafness. In the latter condition Weber’s test will localize to the affected ear and Rinnes test will be impaired on the affected side. Surgical lesions affecting this nerve include CNS tumours and basal skull fractures. It may also be damaged by the administration of ototoxic drugs (of which gentamicin is the most commonly used in surgical practice).

Prepared by Dr: Mohammed Musa Brema Idress – My best wishes Page 360

Glossopharyngeal nerve: Exits the pons just above the vagus. Receives sensory fibers from posterior 1/3 tongue, tonsils, pharynx and middle ear (otalgia may occur following tonsillectomy). It receives visceral afferents from the carotid bodies. It supplies parasympathetic fibers to the parotid gland via the otic ganglion and motor function to stylopharyngeaus muscle. The sensory function of the nerve is tested using the gag reflex.

Vagus nerve: Leaves the medulla between the olivary nucleus and the inferior cerebellar peduncle. Passes through the jugular foramen and into the carotid sheath. Details of the functions of the vagus nerve are covered in the website under relevant organ sub headings. Accessory nerve Exists from the caudal aspect of the brainstem (multiple branches) supplies trapezius and sternocleidomastoid muscles. The distal portion of this nerve is most prone to injury during surgical procedures.

Hypoglossal nerve: Emerges from the medulla at the preolivary sulcus, passes through the hypoglossal canal. It lies on the carotid sheath and passes deep to the posterior belly of digastric to supply muscles of the tongue (except palatoglossus). Its location near the carotid sheath makes it vulnerable during carotid endarterectomy surgery and damage will produce ipsilateral defect in muscle function.

765. A 72 year old man is undergoing an open abdominal aortic aneurysm repair. The aneurysm is located in a juxtarenal location and surgical access to the neck of aneurysm is difficult. Which of the following structures may be divided to improve access? A. Cisterna chyli B. Transverse colon C. Left renal vein D. Superior mesenteric artery E. Coeliac axis

ANSWER IS C The left renal vein will be stretched over the neck of the anuerysm in this location and is not infrequently divided. This adds to the nephrotoxic insult of juxtarenal aortic surgery as a supra renal clamp is also often applied. Deliberate division of the Cisterna Chyli will not improve access and will result in a chyle leak. Division of the transverse colon will not help at all and would result in a high risk of graft infection. Division of the SMA is pointless for a juxtarenal procedure.

766. An occlusion of the anterior cerebral artery may compromise the blood supply to the following structures except: A. Medial inferior surface of the frontal lobe B. Corpus callosum C. Medial surface of the frontal lobe D. Olfactory bulb E. Brocas area

Prepared by Dr: Mohammed Musa Brema Idress – My best wishes Page 361

ANSWER IS E Brocas area is usually supplied by branches from the middle cerebral artery.

Circle of Willis:- The two internal carotid arteries and two vertebral arteries form an anastomosis known as the Circle of Willis on the inferior surface of the brain. Each half of the circle is formed by: 1. Anterior communicating artery 2. Anterior cerebral artery 3. Internal carotid artery 4. Posterior communicating artery 5. Posterior cerebral arteries and the termination of the basilar artery

Vertebral arteries ❖ Enter the cranial cavity via foramen magnum ❖ Lie in the subarachnoid space ❖ Ascend on anterior surface of medulla oblongata ❖ Unite to form the basilar artery at the base of the pons Branches: 1. Posterior spinal artery 2. Anterior spinal artery 3. Posterior inferior cerebellar artery Basilar artery Branches: 1. Anterior inferior cerebellar artery 2. Labyrinthine artery 3. Pontine arteries 4. Superior cerebellar artery 5. Posterior cerebral artery Internal carotid arteries Branches: 1. Posterior communicating artery 2. Anterior cerebral artery 3. Middle cerebral artery 4. Anterior choroid artery

767. Parasympathetic fibers innervating the parotid gland originate from which of the following? A. Submandibular ganglion B. Otic ganglion C. Ciliary ganglion D. Pterygopalatine ganglion E. None of the above

ANSWER IS B Secretion of saliva by the parotid gland is controlled by nerve fibers originating in the inferior salivatory nucleus; these leave the brain via the tympanic nerve (branch of

Prepared by Dr: Mohammed Musa Brema Idress – My best wishes Page 362 glossopharyngeal nerve (CN IX), travel through the tympanic plexus (located in the middle ear), and then form the lesser petrosal nerve until reaching the otic ganglion. After synapsing in the Otic ganglion, the postganglionic (postsynaptic) fibers travel as part of the auriculotemporal nerve (a branch of the mandibular nerve (V3) to reach the parotid gland.

768. Following an oesophagogastrectomy the surgeons will anastomose the oesophageal remnant to the stomach, which of the following is not part of the layers that comprise the oesophageal wall? A. Serosa B. Adventitia C. Muscularis propria D. Submucosa E. Mucosa

ANSWER IS A The oesophageal wall lacks the serosa layer The wall lacks a serosa which can make the wall hold sutures less securely.

769. Which of the following structures suspends the spinal cord in the dural sheath? A. Filum terminale B. Conus medullaris C. Ligamentum flavum D. Denticulate ligaments E. Anterior longitudinal ligament

ANSWER IS D The spinal cord is approximately 45cm in men and 43cm in women. The denticulate ligament is a continuation of the pia mater (innermost covering of the spinal cord) which has intermittent lateral projections attaching the spinal cord to the dura mater.

770. Where is the 'safe triangle' for chest drain insertion located? A. 4th intercostal space, mid axillary line B. 5th intercostal space, mid axillary line C. 4th intercostal space, mid scapular line D. 5th intercostal space, mid scapular line E. 4th intercostal space, mid clavicular line

ANSWER IS B 'Safe Triangle' for chest drain insertion: 5th intercostal space, mid axillary line

Chest drains:- There are a number of different indications for chest drain insertion. In general terms large bore chest drains are preferred for trauma and haemothorax drainage.

Prepared by Dr: Mohammed Musa Brema Idress – My best wishes Page 363

Smaller diameter chest drains can be used for pneumothorax or pleural effusion drainage.

Insertion can be performed either using anatomical guidance or through ultrasound guidance. In the exam, the anatomical method is usually tested.

It is advised that chest drains are placed in the 'safe triangle'. The triangle is located in the mid axillary line of the 5th intercostal space. It is bordered by: Anterior edge latissimus dorsi, the lateral border of pectoralis major, a line superior to the horizontal level of the nipple, and the apex below the axilla.

Another triangle is situated behind the scapula. It is bounded above by the trapezius, below by the latissimus dorsi, and laterally by the vertebral border of the scapula; the floor is partly formed by the rhomboid major. If the scapula is drawn forward by folding the arms across the chest, and the trunk bent forward, parts of the sixth and seventh ribs and the interspace between them become subcutaneous and available for auscultation. The space is therefore known as the triangle of auscultation.

771. A 32 year old rugby player is hit hard on the shoulder during a rough tackle. Clinically, his arm is hanging loose on the side. It is pronated and medially rotated. What structure is most likely to have been compromised? A. Brachial trunks C5-6 B. Brachial trunks C6-7 C. Brachial trunks C8 - T1 D. Anterior interosseous nerve E. Posterior interosseous nerve

ANSWER IS A The patient has an Erb's palsy involving brachial trunks C5-6.

772. Your consultant decides to perform an open inguinal hernia repair under local anaesthesia. Which of the following dermatomal levels will require blockade? A. T10 B. T12 C. T11 D. S1 E. S2

ANSWER IS B

773. A 40 year old professional singer is admitted for a right thyroid lobectomy. Post operatively, she is unable to sing high notes. Which of the following muscles is likely to demonstrate impaired function? A. Thyroarytenoid B. Posterior cricoarytenoid C. Cricothyroid D. Thyrohyoid

Prepared by Dr: Mohammed Musa Brema Idress – My best wishes Page 364

E. Lateral cricoarytenoid

ANSWER IS C The most likely injury is to the superior thyroid nerve which innervates the cricothyroid muscle. Since this tenses the vocal cords, singing high notes becomes a problem.

774. A 44 year old man is undergoing a parotidectomy and the surgeon is carefully preserving the facial nerve. Unfortunately his trainee then proceeds to divide it. Which of the following will not be affected as a result? A. Taste sensation from anterior two thirds of the tongue B. Closing the ipsilateral eyelid C. Raising the ipsilateral side of the lip D. Ipsilateral corneal reflex E. None of the above

ANSWER IS A The chorda tympani branches inside the facial canal and will therefore be unaffected by this most unfortunate event! The corneal reflex is mediated by the ophthalmic branch of the trigeminal nerve sensing the stimulus on the cornea, lid or conjunctiva; the facial nerve initiates the motor response of the reflex.

Facial nerve:- The facial nerve is the main nerve supplying the structures of the second embryonic branchial arch. It is predominantly an efferent nerve to the muscles of facial expression, digastric muscle and also to many glandular structures. It contains a few afferent fibers which originate in the cells of its genicular ganglion and are concerned with taste.

Supply - 'face, ear, taste, tear' - Face: muscles of facial expression - Ear: nerve to stapedius - Taste: supplies anterior two-thirds of tongue - Tear: parasympathetic fibers to lacrimal glands, also salivary glands

Pathway Subarachnoid path Origin: motor- pons, sensory- nervus intermedius Pass through the petrous temporal bone into the internal auditory meatus with the vestibulocochlear nerve. Here they combine to become the facial nerve.

Facial canal path The canal passes superior to the vestibule of the inner ear At the medial aspect of the middle ear, it becomes wider and contains the geniculate ganglion. - 3 branches: 1. Greater petrosal nerve

Prepared by Dr: Mohammed Musa Brema Idress – My best wishes Page 365

2. Nerve to stapedius 3. Chorda tympani

Stylomastoid foramen Passes through the stylomastoid foramen (tympanic cavity anterior and mastoid antrum posteriorly) Posterior auricular nerve and branch to posterior belly of digastric and stylohyoid muscle

Face Enters parotid gland and divides into 5 branches: 1. Temporal branch 2. Zygomatic branch 3. Buccal branch 4. Marginal mandibular branch 5. Cervical branch

775. A 45 year old lady develops severe back pain and on examination is found to have clinical evidence of an L5/ S1 radiculopathy. Her symptoms deteriorate and eventually a laminectomy is performed. During a posterior surgical approach the surgeons encounter a tough ligamentous structure lying anterior to the spinous processes. This structure is most likely to be the: A. Transverse spinal ligament B. Supraspinal ligament C. Anterior longitudinal ligament D. Ligamentum flavum E. Posterior longitudinal ligament ANSWER IS D

776. Which of the following does not pass through the superior orbital fissure? A. Lacrimal nerve B. Abducens nerve C. Ophthalmic artery D. Trochlear nerve E. Superior ophthalmic vein

ANSWER IS C Mnemonic for the nerves passing through the supraorbital fissure: Live Frankly To See Absolutely No Insult 1. Lacrimal 2. Frontal 3. Trochlear 4. Superior Division of Oculomotor 5. Abducens 6. Nasociliary 7. Inferior Division of Oculomotor nerve

Prepared by Dr: Mohammed Musa Brema Idress – My best wishes Page 366

The ophthalmic artery arises from the internal carotid immediately after it has pierced the dura and arachnoid. It runs through the optic canal below the optic nerve and within its dural and arachnoid sheaths. It terminates as the supratrochlear and dorsal nasal arteries.

777. An 18 year old man undergoes a tonsillectomy for attacks of recurrent acute tonsillitis. Whilst in recovery he develops a post operative haemorrhage. Which of the following vessels is the most likely culprit? A. Facial vein B. External palatine vein C. External carotid artery D. Internal jugular vein E. None of the above

ANSWER IS B The external palatine vein lies immediately lateral to the tonsil and if damaged may be a cause of reactionary haemorrhage following tonsillectomy.

Tonsil:- Anatomy Each palatine tonsil has two surfaces, a medial surface which projects into the pharynx and a lateral surface that is embedded in the wall of the pharynx.

They are usually 25mm tall by 15mm wide, although this varies according to age and may be almost completely atrophied in the elderly. Their arterial supply is from the tonsillar artery, a branch of the facial artery. Its veins pierce the constrictor muscle to join the external palatine or facial veins. The external palatine vein is immediately lateral to the tonsil, which may result in haemorrhage during tonsillectomy. Lymphatic drainage is the jugulodigastric node and the deep cervical nodes.

Tonsillitis Usually bacterial (50%)- group A Streptococcus. Remainder viral. May be complicated by development of abscess (quinsy). This may distort the uvula. - Indications for tonsillectomy include recurrent acute tonsillitis, suspected malignancy, enlargement causing sleep apnoea. - Dissection tonsillectomy is the preferred technique with haemorrhage being the commonest complication. Delayed otalgia may occur owing to irritation of the glossopharyngeal nerve.

778. Which of the nerves listed below is responsible for providing innervation to the lower molar teeth? A. Greater palatine nerve B. Nasopalatine nerve C. Inferior alveolar nerve D. Zygomatic nerve E. Mandibular nerve

Prepared by Dr: Mohammed Musa Brema Idress – My best wishes Page 367

ANSWER IS C The branches of the lower molar and premolar teeth are supplied by branches of the inferior alveolar nerve. Those of the canine and incisors by the incisive branch of the same nerve. The gingiva and supporting structures are innervated by the lingual nerve.

779. A patient is found to have an ischaemic left colon. Which artery arising from the aorta at around the level of L3 is most likely to account for this situation? A. Superior mesenteric artery B. Inferior mesenteric artery C. Superior rectal artery D. Ileocolic artery E. Middle colic artery

ANSWER IS B Only the IMA is likely to affect the left side of the colon and originate at L3

780. At which level does the aorta traverse the diaphragm? A. T10 B. T9 C. T8 D. T11 E. T12

ANSWER IS E Memory aid: T8: (8 letters) = vena cava T10: (10 letters) = oesophagus T12: (12 letters) = aortic hiatus

781. What is the arterial blood supply to the lacrimal apparatus? A. Nasociliary artery B. Supra orbital artery C. Internal carotid artery D. Ophthalmic artery E. Supra trochlear artery

ANSWER IS D The ophthalmic artery supplies the gland

Lacrimal apparatus:- Comprises the lacrimal gland and its ducts, lacrimal canaliculi, lacrimal sac and nasolacrimal duct. It lies anteriorly in the superolateral region of the orbit and is divided into 2 parts by the levator palpebrae superioris. Numerous ducts empty glandular secretions into the lateral part of the superior fornix of the conjunctiva. The fluid so produced finally accumulates in the lacrimal lake from which it drains via the lacrimal canaliculi, one with each eyelid. Passing medially, the lacrimal canaliculi

Prepared by Dr: Mohammed Musa Brema Idress – My best wishes Page 368 eventually join the lacrimal sac between the anterior and posterior lacrimal crests, posterior to the medial palpebral ligament and anterior to the lacrimal part of the orbicularis oculi muscle. When the orbicularis oculi muscle contracts during blinking, the small lacrimal part of the muscle dilates the lacrimal sac and draws tears into it.

Innervation Secretomotor fibers from the parasympathetic nervous system. The preganglionic parasympathetic neurons leave the CNS in the facial nerve, enter the greater petrosal nerve and continue with this nerve until it becomes the nerve of the pterygoid canal. The nerve of the pterygoid canal eventually joins the pterygopalatine ganglion where the pre-ganglionic parasympathetic neurons synapse on post ganglionic parasympathetic neurons. The post ganglionic neurons join the maxillary nerve and continue with it until the zygomatic nerve branches from it, and travels with the zygomatic nerve until it gives off the zygomaticotemporal nerve which eventually distributes the post ganglionic parasympathetic fibers in a small branch that joins the lacrimal nerve.

Sympathetic innervation follows a similar path to the parasympathetic path described above. Post ganglionic sympathetic fibers originating in the superior cervical ganglion travel along the plexus surrounding the internal carotid. They leave this plexus as the deep petrosal nerve and join the parasympathetic fibers in the nerve of the pterygoid canal. Passing through the pterygopalatine ganglion, the parasympathetic fibers from this point onwards follow the same path as the parasympathetic fibers to the lacrimal gland.

Vessels Arterial supply is from branches of the ophthalmic artery and venous drainage is to the ophthalmic veins

782. A 24 year old lady is stabbed in the buttock. Following the injury the wound is sutured in the emergency department. Eight weeks later she attends the clinic, as she walks into the clinic room she has a waddling gait and difficulty with thigh abduction. On examination she has buttock muscle wasting. Which nerve has been injured? A. Superior gluteal nerve B. Obturator nerve C. Sciatic nerve D. Femoral nerve E. Inferior gluteal nerve

ANSWER IS A Damage to the superior gluteal nerve will result in a Trendelenburg gait. Trendelenburg test:- Injury or division of the superior gluteal nerve results in a motor deficit that consists of weakened abduction of the thigh by gluteus medius, a disabling gluteus medius limp and a compensatory list of the body to the weakened gluteal side. The

Prepared by Dr: Mohammed Musa Brema Idress – My best wishes Page 369 compensation results in a gravitational shift so that the body is supported on the unaffected limb.

When a person is asked to stand on one leg, the gluteus medius usually contracts as soon as the contralateral leg leaves the floor, preventing the pelvis from dipping towards the unsupported side. When a person with paralysis of the superior gluteal nerve is asked to stand on one leg, the pelvis on the unsupported side descends, indicating that the gluteus medius on the affected side is weak or non functional ( apositive Trendelenburg test).

This eponymous test also refers to a vascular investigation in which tourniquets are placed around the upper thigh, these can help determine whether saphenofemoral incompetence is present.

783. At which level is the hilum of the left kidney located? A. L1 B. L2 C. T12 D. T11 E. L3

ANSWER IS A Remember L1 ('left one') is the level of the hilum of the left kidney

784. During a radical neck dissection, division of which of the following fascial layers will expose the ansa cervicalis? A. Pretracheal fascia B. Carotid sheath C. Prevertebral fascia D. Investing layer of fascia E. Sibson’s fascia

ANSWER IS A The ansa cervicalis lies anterior to the carotid sheath. It may be exposed by division of the pretracheal fascia at the posterolateral aspect of the thyroid gland. The pre vertebral fascia lies more posteriorly and division of the investing layer of fascia will not expose this nerve.

Ansa cervicalis:- Superior root: Branch of C1 anterolateral to carotid sheath Inferior root: Derived from C2 and C3 roots, passes posterolateral to the internal jugular vein (may lie either deep or superficial to it) Innervation 1. Sternohyoid 2. Sternothyroid 3. Omohyoid

Prepared by Dr: Mohammed Musa Brema Idress – My best wishes Page 370

The ansa cervicalis lies anterior to the carotid sheath. The nerve supply to the inferior strap muscles enters at their inferior aspect. Therefore when dividing these muscles to expose a large goiter, the muscles should be divided in their upper half.

785. A 73 year old lady presents with symptoms of faecal incontinence. On examination she has weak anal sphincter muscles. What are the main nerve root values of the nerves supplying the external anal sphincter? A. S2, 3 B. L5, S1 C. S4,5 D. S5 E. S2, 3, 4

ANSWER IS E S2, 3, 4 keeps the poo off the floor The external anal sphincter is innervated by the inferior rectal branch of the pudendal nerve, this has root values of S2, 3 and the perineal branch of S4.

Anal sphincter:- Internal anal sphincter composed of smooth muscle continuous with the circular muscle of the rectum. It surrounds the upper two- thirds of the anal canal and is supplied by sympathetic nerves.

External anal sphincter is composed of striated muscle which surrounds the internal sphincter but extends more distally. The nerve supply of the external anal sphincter is from the inferior rectal branch of the pudendal nerve (S2 and S3) and the perineal branch of the S4 nerve roots.

786. A 22 year old falls over and lands on a shard of glass. It penetrates the palmar aspect of his hand, immediately lateral to the pisiform bone. Which of the following structures is most likely to be injured? A. Palmar cutaneous branch of the median nerve B. Lateral tendons of flexor digitorum superficialis C. Ulnar artery D. Flexor carpi radialis tendons E. Lateral tendons of flexor digitorum profundus

ANSWER IS C The ulnar nerve and artery are at most immediate risk in this injury.

Fascia and compartments of the palm:- The fascia of the palm is continuous with the antebrachial fascia and the fascia of the dorsum of the hand. The palmar fascia is thin over the thenar and hypothenar eminences. In contrast, the central palmar fascia is relatively thick. The palmar aponeurosis covers the soft tissues and overlies the flexor tendons. The apex of the palmar aponeurosis is continuous with the flexor retinaculum and the palmaris

Prepared by Dr: Mohammed Musa Brema Idress – My best wishes Page 371 longus tendon. Distally, it forms four longitudinal digital bands that attach to the bases of the proximal phalanges, blending with the fibrous digital sheaths.

A medial fibrous septum extends deeply from the medial border of the palmar aponeurosis to the 5th metacarpal. Lying medial to this are the hypothenar muscles. In a similar fashion, a lateral fibrous septum extends deeply from the lateral border of the palmar aponeurosis to the 3rd metacarpal. The thenar compartment lies lateral to this area.

Lying between the thenar and hypothenar compartments is the central compartment. It contains the flexor tendons and their sheaths, the lumbricals, the superficial palmar arterial arch and the digital vessels and nerves. The deepest muscular plane is the adductor compartment, which contains adductor pollicis.

Short muscles of the hand These comprise the lumbricals and interossei. The four slender lumbrical muscles flex the fingers at the metacarpophalangeal joints and extend the interphalangeal joint. The four dorsal interossei are located between the metacarpals and the four palmar interossei lie on the palmar surface of the metacarpals in the interosseous compartment of the hand.

Long flexor tendons and sheaths in the hand The tendons of FDS and FDP enter the common flexor sheath deep to the flexor retinaculum. The tendons enter the central compartment of the hand and fan out to their respective digital synovial sheaths. Near the base of the proximal phalanx, the tendon of FDS splits to permit the passage of FDP. The FDP tendons are attached to the margins of the anterior aspect of the base of the distal phalanx. The fibrous digital sheaths contain the flexor tendons and their synovial sheaths. These extend from the heads of the metacarpals to the base of the distal phalanges

787. A 72 year old man has a fall. He is found to have a fractured neck of femur and goes on to have a left hip hemiarthroplasty. Two months post operatively he is found to have an odd gait. When standing on his left leg his pelvis dips on the right side. There is no foot drop. What is the cause? A. Sciatic nerve damage B. L5 radiculopathy C. Inferior gluteal nerve damage D. Previous poliomyelitis E. Superior gluteal nerve damage

ANSWER IS E This patient has a trendelenburg gait caused by damage to the superior gluteal nerve causing weakness of the abductor muscles. Classically a patient is asked to stand on one leg and the pelvis dips on the opposite side. The absence of a foot drop excludes the possibility of polio or L5 radiculopathy.

Prepared by Dr: Mohammed Musa Brema Idress – My best wishes Page 372

Gluteal region:- 1. Gluteal muscles 2. Gluteus maximus: inserts to gluteal tuberosity of the femur and iliotibial tract 3. Gluteus medius: attach to lateral greater trochanter 4. Gluteus minimis: attach to anterior greater trochanter - All extend and abduct the hip Deep lateral hip rotators 1. Piriformis 2. Gemelli 3. Obturator internus 4. Quadratus femoris Nerves Superior gluteal nerve (L5, S1) 1. Gluteus medius 2. Gluteus minimis 3. Tensor fascia lata Inferior gluteal nerve = Gluteus maximus

Damage to the superior gluteal nerve will result in the patient developing a Trendelenberg gait. Affected patients are unable to abduct the thigh at the hip joint. During the stance phase, the weakened abductor muscles allow the pelvis to tilt down on the opposite side. To compensate, the trunk lurches to the weakened side to attempt to maintain a level pelvis throughout the gait cycle. The pelvis sags on the opposite side of the lesioned superior gluteal nerve.

788. Which of the following structures lies posterior to the femoral nerve in the femoral triangle? A. Adductor longus B. Pectineus C. Psoas major D. Iliacus E. None of the above

ANSWER IS D The iliacus lies posterior to the femoral nerve in the femoral triangle. The femoral sheath lies anterior to the iliacus and pectineus muscles.

Mnemonic for femoral nerve supply = (don't) M I S V Q Scan for PE M edial cutaneous nerve of the thigh I ntermediate cutaneous nerve of the thigh S aphenous nerve V astus Q uadriceps femoris S artorius PE ectineus

Prepared by Dr: Mohammed Musa Brema Idress – My best wishes Page 373

789. You are assisting in an open right adrenalectomy for a large adrenal adenoma. The consultant is distracted and you helpfully pull the adrenal into the wound to improve the view. Unfortunately this is followed by brisk bleeding. The vessel responsible for this is most likely to be: A. Portal vein B. Phrenic vein C. Right renal vein D. Superior mesenteric vein E. Inferior vena cava

ANSWER IS E It drains directly via a very short vessel. If the sutures are not carefully tied then it may be avulsed off the IVC. An injury best managed using a Satin sky clamp and a 6/0 prolene suture.

790. A 28 year old lady requires an episiotomy for a ventouse vaginal delivery. Which of the nerves listed below will usually be anaesthetised to allow the episiotomy? A. Femoral B. Ilioinguinal C. Pudendal D. Genitofemoral E. Sacral plexus

ANSWER IS C The pudendal nerve innervates the posterior vulval area and is routinely blocked in procedures such as episiotomy. Pudendal nerve:- The pudendal nerve arises from nerve roots S2, S3 and S4 and exits the pelvis through the greater sciatic foramen. It re-enters the perineum through the lesser sciatic foramen. It travels inferior to give innervation to the anal sphincters and external urethral sphincter. It also provides cutaneous innervation to the region of perineum surrounding the anus and posterior vulva.

Traction and compression of the pudendal nerve by the foetus in late pregnancy may result in late onset pudendal neuropathy which may be part of the process involved in the development of faecal incontinence.

791. A motorcyclist is involved in a road traffic accident. He suffers a complex humeral shaft fracture which is plated. Post operatively he complains of an inability to extend his fingers. Which of the following structures is most likely to have been injured? A. Ulnar nerve B. Radial nerve C. Median nerve D. Axillary nerve E. None of the above

Prepared by Dr: Mohammed Musa Brema Idress – My best wishes Page 374

ANSWER IS B Mnemonic for radial nerve muscles: BEST B rachioradialis E xtensors S upinator T riceps The radial nerve is responsible for innervation of the extensor compartment of the forearm.

792. An enthusiastic surgical registrar undertakes his first solo splenectomy. The operation is far more difficult than anticipated and the registrar leaves a tube drain to the splenic bed at the end of the procedure. Over the following 24 hours approximately 500ml of clear fluid has entered the drain. Biochemical testing of the fluid is most likely to reveal: A. Elevated creatinine B. Elevated triglycerides C. Elevated glucagon D. Elevated amylase E. None of the above

ANSWER IS D During splenectomy the tail of the pancreas may be damaged. The pancreatic duct will then drain into the splenic bed, amylase is the most likely biochemical finding. Glucagon is not secreted into the pancreatic duct.

Splenic anatomy:- The spleen is the largest lymphoid organ in the body. It is an intraperitoneal organ, the peritoneal attachments condense at the hilum where the vessels enter the spleen. Its blood supply is from the splenic artery (derived from the coeliac axis) and the splenic vein (which is joined by the IMV and unites with the SMV).

Embryology: derived from mesenchymal tissue Shape: clenched fist Position: below 9th-12th ribs Weight: 75-150g Relations Superiorly- diaphragm Anteriorly- gastric impression Posteriorly- kidney Inferiorly- colon Hilum- tail of pancreas and splenic vessels Forms apex of lesser sac (containing short gastric vessels)

793. A 48 year old lady is undergoing an axillary node clearance for breast cancer. Which of the structures listed below are most likely to be encountered during the axillary dissection? A. Cords of the brachial plexus

Prepared by Dr: Mohammed Musa Brema Idress – My best wishes Page 375

B. Thoracodorsal trunk C. Internal mammary artery D. Thoracoacromial artery E. None of the above

ANSWER IS B Beware of damaging the thoracodorsal trunk if a latissimus dorsi flap reconstruction is planned. The thoracodorsal trunk runs through the nodes in the axilla. If injured it may compromise the function and blood supply to latissimus dorsi, which is significant if it is to be used as a flap for a reconstructive procedure.

794. A 53 year old lady is recovering following a difficult mastectomy and axillary nodal clearance for carcinoma of the breast. She complains of shoulder pain and on examination has obvious winging of the scapula. Loss of innervation to which of the following is the most likely underlying cause? A. Latissimus dorsi B. Serratus anterior C. Pectoralis minor D. Pectoralis major E. Rhomboids

ANSWER IS B Winging of the scapula is most commonly the result of long thoracic nerve injury or dysfunction. Iatrogenic damage during the course of the difficult axillary dissection is the most likely cause in this scenario. Damage to the rhomboids may produce winging of the scapula but would be rare in the scenario given.

795. How many phalanges are there in the hand? A. 14 B. 12 C. 13 D. 10 E. 8

ANSWER IS A Each digit has 3 phalanges, the thumb has two the total is therefore 14.

796. A 56 year old lady is referred to the colorectal clinic with symptoms of pruritus ani. On examination a polypoidal mass is identified inferior to the dentate line. A biopsy confirms squamous cell carcinoma. To which of the following lymph node groups will the lesion potentially metastasize? A. Internal iliac B. External iliac C. Mesorectal D. Inguinal E. None of the above

Prepared by Dr: Mohammed Musa Brema Idress – My best wishes Page 376

ANSWER IS D Lesions distal to the dentate line drain to the inguinal nodes. Occasionally this will result in the need for a block dissection of the groin.

797. A 20 year old man is hit with a hammer on the right side of the head. He dies on arrival in the emergency department. Which of these features is most likely to be found at post mortem? A. Hydrocephalus B. Supra tentorial herniation C. Laceration of the middle meningeal artery D. Sub dural haematoma E. Posterior fossa haematoma

ANSWER IS C This will account for the scenario given where there is a brief delay prior to death. The other options are less acute and a supratentorial herniation would not occur in this setting.

798. Which of the following ligaments contains the artery supplying the head of femur in children? A. Transverse ligament B. Ligamentum teres C. Iliofemoral ligament D. Ischiofemoral ligament E. Pubofemoral ligament

ANSWER IS B Hip joint:- ◆ Head of femur articulates with acetabulum of the pelvis ◆ Both covered by articular hyaline cartilage ◆ The acetabulum forms at the union of the ilium, pubis, and ischium ◆ The triradiate cartilage (Y-shaped growth plate) separates the pelvic bones ◆ The acetabulum holds the femoral head by the acetabular labrum ◆ Normal angle between femoral head and femoral shaft is 130o

Ligaments ❖ Transverse ligament: joints anterior and posterior ends of the articular cartilage ❖ Head of femur ligament (ligamentum teres): acetabular notch to the fovea. Contains arterial supply to head of femur in children.

Extracapsular ligaments ❖ Iliofemoral ligament: inverted Y shape. Anterior iliac spine to the trochanteric line ❖ Pubofemoral ligament: acetabulum to lesser trochanter ❖ Ischiofemoral ligament: posterior support. Ischium to greater trochanter. Blood supply Medial circumflex femoral and lateral circumflex femoral arteries (Branches of profunda femoris).

Prepared by Dr: Mohammed Musa Brema Idress – My best wishes Page 377

Also from the inferior gluteal artery. These form an anastomosis and travel to up the femoral neck to supply the head.

799. A 72 year old man develops a hydrocele which is being surgically managed. As part of the procedure the surgeons divide the tunica vaginalis. From which of the following is this structure derived? A. Peritoneum B. External oblique aponeurosis C. Internal oblique aponeurosis D. Transversalis fascia E. Rectus sheath

ANSWER IS A The tunica vaginalis is derived from peritoneum, it secretes the fluid that fills the hydrocele cavity.

800. A 43 year old lady is donating her left kidney to her sister and the surgeons are harvesting the left kidney. Which of the following structures will lie most anteriorly at the hilum of the left kidney? A. Left renal artery B. Left renal vein C. Left ureter D. Left ovarian vein E. Left ovarian artery

ANSWER IS B The renal veins lie most anteriorly, then artery and ureter lies posteriorly.

801. What is the sensory nerve supply to the angle of the jaw? A. Maxillary branch of the trigeminal nerve B. Mandibular branch of the trigeminal nerve C. C3-C4 D. Greater auricular nerve (C2-C3) E. Buccal branch of the facial nerve

ANSWER IS D The trigeminal nerve is the major sensory nerve to the face except over the angle of the jaw. The angle of the jaw is innervated by the greater auricular nerve.

802. A 63 year old man is undergoing a coronary artery bypass procedure. During the median sternotomy which structure would routinely require division? A. Parietal pleura B. Interclavicular ligament C. Internal mammary artery D. Brachiocephalic vein E. Left vagus nerve

Prepared by Dr: Mohammed Musa Brema Idress – My best wishes Page 378

ANSWER IS B The interclavicular ligament lies at the upper end of a median sternotomy and is routinely divided to provide access. The pleural reflections are often encountered and should not be intentionally divided, if they are, then a chest drain will need to be inserted on the affected side as collections may then accumulate in the pleural cavity. Other structures encountered include the pectoralis major muscles, again if the incision is truly midline then these should not require formal division.

The close relationship of the brachiocephalic vein should be borne in mind and it should be avoided, iatrogenic injury to this structure will result in considerable haemorrhage.

803. Which of the following structures separates the subclavian artery from the subclavian vein? A. Scalenus anterior B. Scalenus medius C. Sternocleidomastoid D. Pectoralis major E. Pectoralis minor

ANSWER IS A The artery and vein are separated by scalenus anterior. This muscle runs from the transverse processes of C3,4,5 and 6 to insert onto the scalene tubercle of the first rib.

804. A 56 year old lady is due to undergo a left hemicolectomy for carcinoma of the splenic flexure. The surgeons decide to perform a high ligation of the inferior mesenteric vein. Into which of the following does this structure usually drain? A. Portal vein B. Inferior vena cava C. Left renal vein D. Left iliac vein E. Splenic vein

ANSWER IS E Beware of ureteric injury in colonic surgery. The inferior mesenteric vein drains into the splenic vein, this point of union lies close to the duodenum and this surgical maneouvre is a recognized cause of ileus.

805. A man undergoes a high anterior resection for carcinoma of the upper rectum. Which of the following vessels will require ligation? A. Superior mesenteric artery B. Inferior mesenteric artery C. Coeliac axis D. Perineal artery E. Middle colic artery

Prepared by Dr: Mohammed Musa Brema Idress – My best wishes Page 379

ANSWER IS B The IMA is usually divided during anterior resection. Not only is this borne out of oncological necessity but it also permits sufficient colonic mobilization for anastomosis.

806. A 43 year old lady is due to undergo an axillary node clearance as part of treatment for carcinoma of the breast. Which of the following fascial layers will be divided during the surgical approach to the axilla? A. Sibson’s fascia B. Pre tracheal fascia C. Waldayers fascia D. Clavipectoral fascia E. None of the above

ANSWER IS D The clavipectoral fascia is situated under the clavicular portion of pectoralis major. It protects both the axillary vessels and nodes. During an axillary node clearance for breast cancer the clavipectoral fascia is incised and this allows access to the nodal stations.

The nodal stations are; level 1 nodes inferior to pectoralis minor, level 2 lie behind it and level 3 above it. During a Patey Mastectomy surgeons divide pectoralis minor to gain access to level 3 nodes. The use of sentinel node biopsy (and stronger assistants!) have made this procedure far less common.

807. What are the boundaries of the 'safe triangle' for chest drain insertion? A. Bounded by trapezius, latissimus dorsi, and laterally by the vertebral border of the scapula B. Bounded by latissimus dorsi, pectoralis major, line superior to the nipple and apex at the axilla C. Bounded by latissimus dorsi, serratus anterior, line superior to the nipple and apex at the axilla D. Bounded by trapezius, deltoid, rhomboid major and teres minor E. Bounded by trapezius, deltoid and latissimus dorsi

ANSWER IS B Chest drains:- There are a number of different indications for chest drain insertion. In general terms large bore chest drains are preferred for trauma and haemothorax drainage. Smaller diameter chest drains can be used for pneumothorax or pleural effusion drainage.

Insertion can be performed either using anatomical guidance or through ultrasound guidance. In the exam, the anatomical method is usually tested.

It is advised that chest drains are placed in the 'safe triangle'. The triangle is located in the mid axillary line of the 5th intercostal space. It is bordered by:

Prepared by Dr: Mohammed Musa Brema Idress – My best wishes Page 380

Anterior edge latissimus dorsi, the lateral border of pectoralis major, a line superior to the horizontal level of the nipple, and the apex below the axilla.

Another triangle is situated behind the scapula. It is bounded above by the trapezius, below by the latissimus dorsi, and laterally by the vertebral border of the scapula; the floor is partly formed by the rhomboid major. If the scapula is drawn forward by folding the arms across the chest, and the trunk bent forward, parts of the sixth and seventh ribs and the interspace between them become subcutaneous and available for auscultation. The space is therefore known as the triangle of auscultation.

808. Where is a gomphoses type of fibrous joint typically found? A. Teeth B. Skull C. Manubriosternum D. Ribs E. Femur

ANSWER IS A - EXAMPLES OF FIBROUS JOINTS : Sutures- skull - Gomphoses-Peg to socket e.g. teeth - Syndesmosis-tibiofibular joint

809. The vertebral artery traverses all of the following except? A. Transverse process of C6 B. Transverse process of the axis C. Vertebral canal D. Foramen magnum E. Intervertebral foramen

ANSWER IS E The vertebral artery passes through the foramina which are located in the transverse processes of the cervical vertebra, it does not traverse the intervertebral foramen.

810. A 60 year old female attends the preoperative hernia clinic. She reports some visual difficulty. On examination she is noted to have a homonymous hemianopia. Where is the lesion most likely to be? A. Frontal lobe B. Pituitary gland C. Parietal lobe D. Optic chiasm E. Optic tract

ANSWER IS E Lesions before optic chiasm:- A. Monocular vision loss = Optic nerve lesion B. Bitemporal hemianopia = Optic chiasm lesion

Prepared by Dr: Mohammed Musa Brema Idress – My best wishes Page 381

Lesions after the optic chiasm:- A. Homonymous hemianopia = Optic tract lesion B. Upper quadranopia = Temporal lobe lesion C. Lower quadranopia = Parietal lobe lesion Unfortunately we thought as surgeons we could forget about visual field defects! However the college seem to like them. Remember a homonymous hemianopia is indicative of an optic tract lesion. Parietal lobe lesions tend to cause inferior quadranopias and there is a bitemporal hemianopia with optic chiasm lesion or pituitary tumours.

Visual field defects:- left homonymous hemianopia means visual field defect to the left, i.e. Lesion of right optic tract homonymous quadrantanopias: PITS (Parietal-Inferior, Temporal-Superior) incongruous defects = optic tract lesion; congruous defects = optic radiation lesion or occipital cortex

Homonymous hemianopia A. Incongruous defects: lesion of optic tract B. Congruous defects: lesion of optic radiation or occipital cortex C. Macula sparing: lesion of occipital cortex

Homonymous quadrantanopias Superior: lesion of temporal lobe Inferior: lesion of parietal lobe Mnemonic = PITS (Parietal-Inferior, Temporal-Superior)

Bitemporal hemianopia Lesion of optic chiasm Upper quadrant defect > lower quadrant defect = inferior chiasmal compression, commonly a pituitary tumour Lower quadrant defect > upper quadrant defect = superior chiasmal compression, commonly a craniopharyngioma

811. A 34 year old male is being examined in the pre-operative assessment clinic. A murmur is identified in the 4th intercostal space just next to the left side of the sternum. From where is it most likely to have originated? A. Mitral valve B. Aortic valve C. Pulmonary valve D. Right ventricular aneurysm E. Tricuspid valve

ANSWER IS E The tricuspid valve is generally referred to being best auscultated adjacent to the sternum. The plane of projected sound from the mitral area is best heard in the region of the cadiac apex.

Prepared by Dr: Mohammed Musa Brema Idress – My best wishes Page 382

Heart sounds:- Sites of auscultation = Valve Site Pulmonary valve = Left second intercostal space, at the upper sternal border Aortic valve = Right second intercostal space, at the upper sternal border Mitral valve = Left fifth intercostal space, just medial to mid clavicular line Tricuspid valve = Left fourth intercostal space, at the lower left sternal border

812. During an Ivor Lewis Oesophagectomy for carcinoma of the lower third of the oesophagus which structure is divided to allow mobilization of the oesophagus? A. Vagus nerve B. Azygos vein C. Right inferior lobar bronchus D. Phrenic nerve E. Pericardiophrenic artery

ANSWER IS B The azygos vein is routinely divided during an oesophagectomy to allow mobilization. It arches anteriorly to insert into the SVC on the right hand side.

813. Which of the following statements relating to quadratus lumborum is false? A. Causes flexion of the thoracic spine B. Causes the rib cage to be pulled down C. Innervated by anterior primary rami of T12 and L1-3 D. Attached to the iliac crest E. Inserts into the 12th rib

ANSWER IS A Quadratus lumborum Origin: Medial aspect of iliac crest and iliolumbar ligament Insertion: 12th rib Action: Pulls the rib cage inferiorly. Lateral flexion. Nerve supply: Anterior primary rami of T12 and L1-3

The rectus abdominis causes flexion of the thoracic spine and therefore the statement suggesting that quaratus lumborum does so is incorrect.

814. A 23 year old climber falls and fractures his humerus. The surgeons decide upon a posterior approach to the middle third of the bone. Which of the following nerves is at greatest risk in this approach? A. Ulnar B. Antebrachial C. Musculocutaneous D. Radial E. Intercostobrachial

ANSWER IS D

Prepared by Dr: Mohammed Musa Brema Idress – My best wishes Page 383

The radial nerve wraps around the humerus and may be injured during a posterior approach. An IM nail may be preferred as it avoids the complex dissection needed for direct bone exposure.

815. A 68 year old man with critical limb ischaemia is undergoing a femoro-distal bypass graft. During mobilization of the proximal part of the posterior tibial artery which of the following is at greatest risk of injury? A. Tibial nerve B. Sciatic nerve C. Saphenous nerve D. Common peroneal nerve E. Medial superior genicular artery

ANSWER IS A The tibial nerve is closely related to the posterior tibial artery. The tibial nerve crosses the vessel posteriorly approximately 2.5cm distal to its origin. At its origin the nerve lies medial and then lateral after it crosses the vessel as described.

816. A 67 year old man is undergoing an angiogram for gastro intestinal bleeding. The radiologist advances the catheter into the coeliac axis. At what spinal level does this vessel typically arise from the aorta? A. T10 B. L3 C. L4 D. T12 E. None of the above

ANSWER IS D The coeliac axis lies at T12, it takes an almost horizontal angle off the aorta. It has three major branches.

817. Which muscle does not insert on the medial surface of the greater trochanter? A. Gemelli B. Obturator internus C. Piriformis D. Quadratus femoris E. Obturator externus

ANSWER IS D Mnemonic for muscle attachment on greater trochanter is POGO: 1. Piriformis 2. Obturator internus 3. Gemelli 4. Obturator externus The quadratus femoris fibers pass laterally to be inserted into the quadrate tubercle on the intertrochanteric crest of the femur. The other muscles all insert on the trochanteric fossa lying medial to the greater trochanter.

Prepared by Dr: Mohammed Musa Brema Idress – My best wishes Page 384

818. What is the largest branch of the brachial artery? A. Radial artery B. Ulnar artery C. Profunda brachii artery D. Humeral nutrient artery E. Ulnar collateral artery

ANSWER IS C The profunda brachii artery is the largest branch and then continues in the radial groove of the humerus

819. During a radical gastrectomy for carcinoma of the stomach the surgeons remove the omentum. What is the main source of its blood supply? A. Ileocolic artery B. Superior mesenteric artery C. Gastroepiploic artery D. Middle colic artery E. Inferior mesenteric artery

ANSWER IS C The vessels supplying the omentum are the omental branches of the right and left gastro-epiploic arteries. The colonic vessels are not responsible for the arterial supply to the omentum. The left gastro-epiploic artery is a branch of the splenic artery and the right gastro-epiploic artery is a terminal branch of the gastroduodenal artery.

820. A 38 year old lady is due to undergo a parathyroidectomy for hyperparathyroidism. At operation the inferior parathyroid gland is identified as being enlarged. A vessel is located adjacent to the gland laterally. This vessel is most likely to be the: A. External carotid artery B. Common carotid artery C. Internal carotid artery D. External jugular vein E. None of the above

ANSWER IS B The common carotid artery is a lateral relation of the inferior parathyroid.

821. A 45 year old man has a long femoral line inserted to provide CVP measurements. The catheter passes from the common iliac vein into the inferior vena cava. At which of the following vertebral levels will this occur? A. L5 B. L4 C. S1 D. L3 E. L2

Prepared by Dr: Mohammed Musa Brema Idress – My best wishes Page 385

ANSWER IS A The common iliac veins fuse with the IVC at L5.

822. Following a carotid endarterectomy a man notices that he has a weakness of his tongue. Damage to which of the following nerves is the most likely explanation for this process? A. Hypoglossal B. Accessory C. Ansa cervicalis D. Vagus E. Cervical plexus

ANSWER IS A The hypoglossal nerve innervates the tongue and is one of the structures more commonly at risk in carotid surgery.

823. At which of the following levels does the inferior vena cava exit the abdominal cavity? A. T6 B. T7 C. T10 D. T8 E. T12

ANSWER IS D

824. Which of the following structures lies deepest in the popliteal fossa? A. Popliteal artery B. Popliteal vein C. Tibial nerve D. Common peroneal nerve E. Popliteal lymph nodes

ANSWER IS A From superficial to deep:- The common peroneal nerve exits the popliteal fossa along the medial border of the biceps tendon. Then the tibial nerve lies lateral to the popliteal vessels to pass posteriorly and then medially to them. The popliteal vein lies superficial to the popliteal artery, which is the deepest structure in the fossa.

825. Which of the following nerves is responsible for innervation of the triceps muscle? A. Radial B. Ulnar C. Axillary D. Median E. None of the above

Prepared by Dr: Mohammed Musa Brema Idress – My best wishes Page 386

ANSWER IS A To remember nerve roots and their reflexes: A) 1-2 Ankle (S1-S2) B) 3-4 Knee (L3-L4) C) 5-6 Biceps (C5-C6) D) 7-8 Triceps (C7-C8) The radial nerve innervates all three heads of triceps, with a separate branch to each head.

826. Which of the positions listed below best describes the location of the coeliac autonomic plexus? A. Anterolateral to the aorta B. Posterolateral to the aorta C. Anterolateral to the sympathetic chain D. Anteromedial to the sympathetic chain E. Posterior to L1

ANSWER IS A Coeliac plexus:- The coeliac plexus is the largest of the autonomic plexuses. It is located on a level of the last thoracic and first lumbar vertebrae. It surrounds the coeliac axis and the SMA. It lies posterior to the stomach and the lesser sac. It lies anterior to the crura of the diaphragm and the aorta. The plexus and ganglia are joined by the greater and lesser splanchnic nerves on both sides and branches from both the vagus and phrenic nerves.

827. An intravenous drug user develops a false aneurysm and requires emergency surgery. The procedure is difficult and the femoral nerve is inadvertently transected. Which of the following muscles is least likely to be affected as a result? A. Sartorius B. Vastus medialis C. Pectineus D. Quadriceps femoris E. Adductor magnus

ANSWER IS E Mnemonic for femoral nerve supply = (don't) M I S V Q Scan for PE M edial cutaneous nerve of the thigh I ntermediate cutaneous nerve of the thigh S aphenous nerve V astus Q uadriceps femoris S artorius PE ectineus Adductor magnus is innervated by the obturator and sciatic nerve. The pectineus muscle is sometimes supplied by the obturator nerve but this is variable. Since the question states least likely, the correct answer is adductor magnus

Prepared by Dr: Mohammed Musa Brema Idress – My best wishes Page 387

828. What is the nerve root value of the external urethral sphincter? A. S4 B. S1, S2, S3 C. S2, S3, S4 D. L3, L4, L5 E. L5, S1, S2

ANSWER IS C The external urethral sphincter is innervated by branches of the pudendal nerve, therefore the root values are S2, S3, S4.

829. A 45 year old man is stabbed in the abdomen and the inferior vena cava is injured. How many functional valves does this vessel usually have? A. 0 B. 1 C. 3 D. 2 E. 4

ANSWER IS A Mnemonic for the Inferior vena cava tributaries: I Like To Rise So High: 1. Iliacs 2. Lumbar 3. Testicular 4. Renal 5. Suprarenal 6. Hepatic vein The lack of valves in the IVC is important clinically when it is cannulated during cardiopulmonary bypass, using separate SVC and IVC catheters, such as when the right atrium is to be opened. Note that there is a non functional valve between the right atrium and inferior vena cava.

830. Which of the following structures does not pass posteriorly to the medial malleolus? A. Posterior tibial artery B. Tibial nerve C. Tibialis anterior tendon D. Tendon of flexor digitorum longus E. Tendon of flexor hallucis longus

ANSWER IS C Mnemonic for structures posterior to the medial malleolus: Tom Dick And Nervous Harry T ibialis posterior tendon flexor Digitorum longus A rtery N erve

Prepared by Dr: Mohammed Musa Brema Idress – My best wishes Page 388

H allucis longus

831. Which of the following statements relating to the root of the neck is false? A. The lung projects into the neck beyond the first rib and is constrained by Sibson's fascia B. The subclavian artery arches over the first rib anterior to scalenus anterior C. The trunks of the brachial plexus lie posterior to the subclavian artery on the first rib D. The roots and trunks of the Brachial plexus lie between scalenus anterior and scalenus medius muscles E. The thyrocervical trunk is a branch of the subclavian artery

ANSWER IS B The subclavian artery lies posterior to scalenus anterior, the vein lies in front. Sibson's fascia is another name for the suprapleural membrane.

832. A patient presents to the clinic following a surgical procedure. She complains that she is unable to shrug her shoulder. What is the most likely underlying nerve injury? A. Accessory nerve B. Cervical plexus C. Ansa cervicalis D. Long thoracic nerve E. Axillary nerve

ANSWER IS A The accessory nerve may be injured in operations in the posterior triangle. Injury will affect trapezius.

833. The cords of the brachial plexus are most closely related to which of the following vessels? A. Subclavian artery B. Axillary artery C. Axillary vein D. Subclavian vein E. Brachial artery

ANSWER IS B The trunks are related to the subclavian artery superiorly. The cords of the plexus surround the axillary artery, they are named according to their positions relative to this structure.

834. Which of the following are not generally supplied by the right coronary artery? A. The sinoatrial node B. The circumflex artery C. The atrioventricular node

Prepared by Dr: Mohammed Musa Brema Idress – My best wishes Page 389

D. Most of the right ventricle E. The right atrium

ANSWER IS B The circumflex artery is generally a branch of the left coronary artery.

835. A 44 year old man has a malignant melanoma and is undergoing a block dissection of the groin. The femoral triangle is being explored for intra operative bleeding. Which of the following forms the medial border of the femoral triangle? A. Femoral artery B. Biceps femoris C. Adductor longus D. Sartorius E. Adductor magnus

ANSWER IS C Adductor longus forms the medial boundary of the femoral triangle

836. The foramen marking the termination of the adductor canal is located in which of the following? A. Adductor longus B. Adductor magnus C. Adductor brevis D. Sartorius E. Semimembranosus

ANSWER IS B The foramen marking the distal limit of the adductor canal is contained within adductor magnus. The vessel passes through this region to enter the popliteal fossa.

837. Which of the following is the first vessel to branch from the external carotid artery? A. Superior thyroid artery B. Inferior thyroid artery C. Lingual artery D. Facial artery E. Occipital artery

ANSWER IS A Mnemonic:(Order in which they branch off) Some (sup thyroid) Attending (Ascending Pharyngeal)Like (Lingual)Freaking (Facial)Out (Occipital)Potential (Post auricular)Medical (Maxillary)Students (Sup temporal)

The first branch of the external carotid artery is the superior thyroid artery. The inferior thyroid artery is derived from the thyrocervical trunk.

Prepared by Dr: Mohammed Musa Brema Idress – My best wishes Page 390

838. A motorcyclist is injured in a road traffic accident and is not wearing a helmet. He suffers a severe closed head injury and develops raised intracranial pressure. The first cranial nerve to be affected by this process is likely to be: A. Oculomotor B. Hypoglossal C. Motor branch of the trigeminal D. Abducens E. Sensory branch of the trigeminal

ANSWER IS D The abducens nerve (CN VI) has a long intra cranial course and is thus susceptible to raised intra cranial pressure. It also passes over the petrous temporal bone and 6th nerve palsies are also seen in mastoiditis.

839. A 32 year old man is undergoing a splenectomy. Division of which of the following will be necessary during the procedure? A. Left crus of diaphragm B. Short gastric vessels C. Gerotas fascia D. Splenic flexure of colon E. Marginal artery

ANSWER IS B During a splenectomy the short gastric vessels which lie within the gastrosplenic ligament will need to be divided. The splenic flexure of the colon may need to be mobilized. However, it will almost never need to be divided, as this is watershed area that would necessitate a formal colonic resection in the event of division.

840. Which structure separates the cephalic vein and the brachial artery in the antecubital fossa? A. Brachioradialis muscle B. Biceps muscle C. Origin of flexor digitorum profundus muscle D. Pronator quadratus muscle E. Origin of flexor digitorum superficialis muscle

ANSWER IS B

841. A 24 year old motor cyclist is involved in a road traffic accident. He suffers a tibial fracture which is treated with an intra medullary nail. Post operatively he develops a compartment syndrome. Surgical decompression of the anterior compartment will relieve pressure on all of the following muscles except? A. Peroneus brevis B. Peroneus tertius C. Extensor digitorum longus D. Tibialis anterior E. None of the above

Prepared by Dr: Mohammed Musa Brema Idress – My best wishes Page 391

ANSWER IS A The anterior compartment contains: 1. Tibialis anterior 2. Extensor digitorum longus 3. Peroneus tertius 4. Extensor hallucis longus 5. Anterior tibial artery All the muscles are innervated by the deep peroneal nerve.

842. A 43 year old lady underwent an attempted placement of a central line into the internal jugular vein. Unfortunately, the doctor damaged the carotid artery and this necessitated surgical exploration. As the surgeons incise the carotid sheath a nerve is identified lying between the internal jugular vein and the carotid artery. Which of the following is this nerve most likely to be? A. Glossopharyngeal nerve B. Hypoglossal nerve C. Superior laryngeal nerve D. Recurrent laryngeal nerve E. Vagus

ANSWER IS E The vagus lies in the carotid sheath. The hypoglossal nerve crosses the sheath, but does not lie within it.

843. A patient has a chest drain insertion. There is fresh blood at the chest drain insertion area. Which vessel has been damaged? A. Pericardiophrenic artery B. Intercostal vein C. Right ventricle D. Vagus artery E. Intercostal artery

ANSWER IS E The intercostal vein is more superior than the artery and is thus slightly less susceptible to injury. Within the intercostal spaces there are thin, strong muscles, intercostal vessels, nerves and lymphatics. There are 3 intercostal muscle layers corresponding to the lateral abdominal wall; external, internal, innermost intercostals. At the mid axillary line there are thin intracostals which is an extension of the internal intercostal muscle.

In each intercostal space lies the neurovascular bundle, comprising, from superior to inferiorly; the posterior intercostal vein, artery and nerve, lying protected in the subcostal groove of the rib above and situated between the second and third layer of the intercostal muscles. These blood vessels anastomose anteriorly with the anterior intercostal vessels, which arise from the internal thoracic artery and vein.

Prepared by Dr: Mohammed Musa Brema Idress – My best wishes Page 392

844. Two teenagers are playing with an airgun when one accidentally shoots his friend in the abdomen. He is brought to the emergency department. On examination there is a bullet entry point immediately to the right of the rectus sheath at the level of the 1st lumbar vertebra. Which of the following structures is most likely to be injured by the bullet? A. Head of pancreas B. Right ureter C. Right adrenal gland D. Fundus of the gallbladder E. Gastric antrum

ANSWER IS D The fundus of the gallbladder lies at this level and is the most superficially located structure.

845. Which of the following muscles inserts onto the lesser tuberostiy of the humerus? A. Subscapularis B. Deltoid C. Supraspinatus D. Teres minor E. Infraspinatus

ANSWER IS A With the exception of subscapularis which inserts into the lesser tuberosity, the muscles of the rotator cuff insert into the greater tuberosity.

846. Which of the following nerves is not contained within the posterior triangle of the neck? A. Accessory nerve B. Phrenic nerve C. Greater auricular nerve D. Ansa cervicalis E. Lesser occiptal nerve

ANSWER IS D Ansa cervicalis is a content of the anterior triangle of the neck.

847. A 42 year old lady is reviewed in the outpatient clinic following a routine surgical procedure. She complains of diminished sensation at the dorso-lateral aspect of her foot. Which of the following nerves is most likely to be affected? A. Sural B. Superficial peroneal C. Deep peroneal D. Medial plantar E. Lateral plantar

Prepared by Dr: Mohammed Musa Brema Idress – My best wishes Page 393

ANSWER IS A The sural nerve supplies the lateral aspect of the foot. It runs alongside the short saphenous vein and may be injured in short saphenous vein surgery.

848. Which of the following anatomical planes separates the prostate from the rectum? A. Sibson’s fascia B. Denonvilliers fascia C. Levator ani muscle D. Waldeyers fascia E. None of the above

ANSWER IS B The Denonvilliers fascia separates the rectum from the prostate. Waldeyers fascia separates the rectum from the sacrum

849. A 56 year old lady is undergoing an adrenalectomy for Conn’s syndrome. During the operation the surgeon damages the middle adrenal artery and haemorrhage ensues. From which of the following structures does this vessel originate? A. Aorta B. Renal artery C. Splenic artery D. Coeliac axis E. Superior mesenteric artery

ANSWER IS A The middle adrenal artery is usually a branch of the aorta, the lower adrenal artery typically arises from the renal vessels.

850. A 73 year old lady suffers a fracture at the surgical neck of the humerus. The decision is made to operate. There are difficulties in reducing the fracture and a vessel lying posterior to the surgical neck is injured. Which of the following is this vessel most likely to be? A. Axillary artery B. Brachial artery C. Thoracoacromial artery D. Transverse scapular artery E. Posterior circumflex humeral artery

ANSWER IS E The circumflex humeral arteries lie at the surgical neck and is this scenario the posterior circumflex is likely to be injured. The thoracoacromial and transverse scapular arteries lie more superomedially. The posterior circumflex humeral artery is a branch of the axillary artery.

Prepared by Dr: Mohammed Musa Brema Idress – My best wishes Page 394

851. A 22-year-old man sustained facial injuries playing rugby. Which radiological investigation is the most helpful to plan surgery? A. Angiography B. Computed tomography (CT) scanning C. Facial view X-rays showing fluid in the paranasal sinuses D. MRI E. Waters’ facial view

ANSWER IS B Computed tomography (CT) scanning is essential when planning reconstruction of facial fractures, especially complex ones. CT scanning also delineates soft tissue damage adequately. Indirect signs of a fracture include soft tissue swelling, periorbital or intracranial air and fluid in the paranasal sinus.

Four facial X-rays are usually taken: Waters‟ view (posterior–anterior (PA) with cephalad angulation), Caldwell (PA view), lateral view and occipito-submentovertex view. Waters‟ view tends to show all facial structures the best. Lines of Dolan (I–III) are three anatomic contours that correspond to facially important structures.

852. During pericardiectomy, sudden bleeding was noticed due to accidental injury to a major vascular structure in the pericardium. The surgeon inserted his left index finger through the transverse pericardial sinus, pulled forward on the two large vessels lying ventral to his finger and compressed these vessels with his thumb to control bleeding. Which vessels were these? Single best answer question – choose ONE true option only. A. Pulmonary trunk and brachiocephalic trunk B. Pulmonary trunk and aorta C. Pulmonary trunk and superior vena cava D. Superior vena cava and aorta E. Superior vena cava and right pulmonary artery

ANSWER IS B The transverse pericardial sinus is an area of the pericardial cavity located behind the aorta and pulmonary trunk and anterior to the superior vena cava. Therefore, the two large vessels lying ventral to his finger are the pulmonary trunk and aorta; the large vessel lying dorsal to his finger is the superior vena cava.

853. A 45-year-old male with a history of gastro-oesophageal reflux disease undergoes laparoscopic reduction of hiatus hernia and fundoplication. During the procedure, dissection is performed to identify the boundaries of the oesophageal hiatus. What structure accompanies the oesophagus through this hiatus? Single best answer - select one answer only. A. Azygous vein B. Inferior vena cava C. Phrenic nerves D. Thoracic duct E. Vagus nerves

Prepared by Dr: Mohammed Musa Brema Idress – My best wishes Page 395

ANSWER IS E The oesophagus passes through the diaphragm at the level of the T10 vertebrae, and is accompanied by the left and right branches of the vagus nerve. The aorta passes through at T12, accompanied by the azygous vein and thoracic duct, and the IVC passes through at the level of T8 and is accompanied by the right phrenic nerve.

854. A 42-year-old man with a history of road traffic accident and injury to his back and neck presents with global muscle wasting of the left hand. Which is the nerve or nerve root most likely to be involved? Single best answer question – choose ONE true option only. A. Radial nerve B. Median nerve C. Ulnar nerve D. T1 nerve root E. C7 nerve root.

ANSWER IS D The median nerve supplies the lateral two lumbricals, opponens pollicis, abductor pollicis brevis and flexor pollicis brevis; the remainder are served by the ulnar nerve. Global muscle wasting of the hand indicates damage to both the median and ulnar nerves with damage to the T1 nerve root. Isolated wasting of abductor pollicis brevis occurs in association with median nerve damage from carpal tunnel syndrome. More extensive wasting may suggest a broader diagnosis such as syringomyelia or motor neuron disease.

855. The femoral triangle of Scarpa in the thigh contains some important structures. What are the contents of the femoral triangle? A. The femoral canal lies lateral to the femoral vein B. The femoral canal which is the most lateral structure in the femoral sheath C. The femoral sheath which contains the femoral vessels D. The long saphenous vein E. The pubic branch of the inferior epigastric vein.

ANSWER IS C The femoral sheath is a downward continuation of the abdominal fascia about 2.5 cm below the inguinal ligament. It is believed to allow for femoral vessel movement in the inguinal region during movement of the hip. The mnemonic „NAVEL‟ describes some of the key structures in this region (from lateral to medial): femoral nerve, artery, vein, empty space, lymphatics (Cloquet‟s).

Apart from the femoral nerve, that lies most laterally, all the other structures in the mnemonic are encased in the femoral sheath. The mentioned empty space and lymphatic compartments form the „femoral canal‟, where femoral hernias may occur. Therefore, the femoral canal is in the most medial portion of the femoral sheath. The femoral ring is the abdominal opening into the femoral canal. At 2–3 cm below the inguinal ligament, the femoral sheath fuses with the adventitia of the femoral vessels.

Prepared by Dr: Mohammed Musa Brema Idress – My best wishes Page 396

The pubic branch of the inferior epigastric artery replaces the obturator artery in about 30% of cases, and may be at risk in a femoral hernia repair.

856. A tumour in the medial wall of the body of the lateral ventricle will involve which of the following structures? Single best answer question – choose ONE true option only. A. Caudate nucleus of the corpus striatum B. Stria terminalis C. Posterior part of the septum pellucidum D. Terminal vein E. Undersurface of the corpus callosum

ANSWER IS C The central part or body of the lateral ventricle extends from the interventricular foramen to the splenium of the corpus callosum. It is an irregularly curved cavity, triangular on transverse section, with a roof, a floor and a medial wall.

The roof is formed by the undersurface of the corpus callosum; the floor by the following parts, enumerated in their order of position, from before backward: the caudate nucleus of the corpus striatum, the stria terminalis and the terminal vein, the lateral portion of the upper surface of the thalamus, the choroid plexus and the lateral part of the fornix; the medial wall is the posterior part of the septum pellucidum, which separates it from the opposite ventricle.

A tumour in the medial wall of the body of the lateral ventricle will involve the posterior part of the septum pellucidum.

857. A 25-year-old male who is in Burns intensive care having sustained 42% burns in a house fire is on day 12 post injury and remains intubated and ventilated. The Consultant Anaesthetist feels he will not be ready for extubation for a while and the decision is made to perform a tracheostomy. A size 7 tracheostomy tube is used. Which of the following does the tube size relate to? Select one answer only. A. Distance from tracheostomy tube tip to carina B. Distance of tracheostomy tube from vocal cords C. Internal diameter of the tracheostomy tube D. Length of the tracheostomy tube E. Outer diameter of the tracheostomy tube

ANSWER IS C The first tracheal ring is complete in children and must not be excised for fear of tracheal stenosis. The preferred method is excision of rings 2–4. Tracheostomy should be performed if prolonged intubation is expected; either percutaneous or open approaches are used.

Fenestrated tubes allow air to be diverted superiorly through the vocal cords so speech can be possible, but this can also be attained if the tube is occluded with a

Prepared by Dr: Mohammed Musa Brema Idress – My best wishes Page 397 finger. A size 7 tracheostomy tube relates to an internal diameter (ID) of 7 mm, not the outer diameter or length.

858. A 58-year-old diabetic male presents with chest pain radiating to the back and an initial chest X ray shows a widened mediastinum. Further imaging demonstrates a thoracic aortic aneurysm and the decision is made for surgical management. What type of incision allows the best access to the thoracic aorta? Select one answer only. A. Median sternotomy B. Rooftop incision C. Thoracoabdominal D. Thoracoscopic E. Thoracotomy

ANSWER IS E Thoracic aortic aneurysms are far less common than abdominal aneurysms. Previous dissection may lead to weakness of the aortic wall, and thus predispose to aneurysm formation. There is no common association with neurofibromatosis. They are just as likely to rupture as abdominal aneurysms; according to the law of Laplace the wall tension is proportional to the radius at a given pressure. Access to the thoracic aorta is best afforded by a thoracotomy rather than a median sternotomy, as it is a posterior mediastinal structure.

859. The blade of a retractor has rested on the psoas muscle during a lengthy operative procedure. This has resulted in a femoral nerve palsy. In the post- operative period the patient will experience: Single best answer question – choose ONE true option only. A. Inability to flex the knee only B. Inability to flex the knee and numbness over the thigh C. Numbness over the anterior thigh only D. Inability to extend the knee and numbness over the anterior thigh E. Inability to flex the hip and numbness over the anterior thigh.

ANSWER IS D The femoral nerve is the largest branch of the lumbar plexus (L2-4). It forms in the abdomen within the substance of the psoas major muscle and descends posterolaterally through the pelvis to the midpoint of the inguinal ligament.

It supplies the anterior thigh muscles – the quadriceps group extend the leg at the knee, it also supplies other anterior thigh muscles - iliacus and sartorius which allow flexion of the thigh at the hip joint. The femoral nerve also gives several branches to the skin on the anteriomedial side of the lower limb.

860. A 49-year-old woman has been complaining of a tingling feeling in her right hand at night and is under the impression that her hand is swollen although there is no obvious oedema. In the last few days she has noticed numbness in her right index finger and the tip of her thumb, especially while working. Which nerve is

Prepared by Dr: Mohammed Musa Brema Idress – My best wishes Page 398 most likely to be responsible for her symptoms? Single best answer question – choose ONE true option only. A. Ulnar nerve B. Median nerve C. Radial nerve D. Nerve root C7 E. Nerve root C8.

ANSWER IS B One of the most common lesions at this site is carpal tunnel syndrome, in which the median nerve is compressed as it passes deep to the flexor retinaculum. The usual presentation is with acroparaesthesias. This consists of numbness, tingling and burning sensations felt in the hand and fingers; the pain sometimes radiates up the forearm as far as the elbow or even as high as the shoulder or root of the neck. Although the paraesthesias are sometimes restricted to the radial fingers, they may affect all the digits as some fibers from the median nerve are distributed to the fifth finger through a communication with the ulnar nerve in the palm.

The attacks of pain and paraesthesias are most common at night and often wake the patient from sleep. They are then relieved by shaking the hand. The hand tends to feel numb and useless on waking in the morning but recovers after it has been used for some minutes. The symptoms may recur during the day following use, or at other times if the patient sits with the hands immobile. Such symptoms of acroparaesthesias may persist for many years without the appearance of symptoms of median nerve damage.

In other patients, weakness of the thenar muscles develops, particularly with abduction of the thumb, and is associated with atrophy of the lateral aspect of the thenar eminence. Sensory loss may appear over the tips of the median innervated fingers.

861. The femoral artery is the terminal branch of the external iliac artery behind the inguinal ligament. Which branch of the femoral artery supplies the femoral head? A. Deep external pudendal artery B. Profunda femoris artery C. Superficial circumflex iliac artery D. Superficial external pudendal artery E. Superficial epigastric artery

ANSWER IS B The surface marking of the femoral artery is the mid-inguinal point, which lies just medial to the position of the deep inguinal ring, which is at the midpoint of the inguinal ligament. The lateral and medial circumflex femoral arteries are branches of the profunda femoris artery, and they contribute to the trochanteric anastomosis, feeding the femoral head via their subretinacular branches. The femoral artery is subcutaneous in the femoral triangle, separated from skin by the fascia lata.

Prepared by Dr: Mohammed Musa Brema Idress – My best wishes Page 399

862. A young man sustains brachial plexus injury following a stabbing incident. He presented with weakness of elbow and wrist dorsiflexion. Which part of the brachial plexus has been transected? Select one answer only. A. C 5 nerve root B. The superior trunk C. The posterior cord D. The axillary nerve E. The ulnar nerve.

ANSWER IS C The posterior cord of the brachial plexus gives off the radial nerve which supplies the triceps and the extensor carpi radialis. The brachial plexus is derived from the anterior rami of C5,6,7,8,T1. It is described in terms of roots, trunks, divisions, cords and nerves. The musculocutaneous nerve (C5,6,7) is a continuation of the lateral cord, the median nerve (C5,6,7,8,T1) is derived from the medial and lateral cords, the ulnar (C8,T1) nerve is a continuation of the medial cord, and the radial nerve (C5,6,7,8,T1) a continuation of the posterior cord. The thoracodorsal nerve is a branch of the posterior cord.

863. A 45-year-old man has been sent to the accident and emergency department by his general practitioner who suspects that he is suffering from caudaequina syndrome. Which of the following clinical findings supports this diagnosis? Select one answer only. A. An MRI of the spine reveals a disc prolapse at T11 B. There is a positive Babinski sign C. The patients chief complaint is of a shooting pain radiating down the back of his legs D. The patient is in urinary retention, with reduced anal tone and bilateral lower limb weakness E. The patient has purely sensory loss in all of the lumber dermatomes.

ANSWER IS D Below the conus medullaris, the spinal canal contains the caudaequina which branches off the lower end of the spinal cord and contains the nerve roots from L1-5 and S1-5. These roots from L4-S4 join in the sacral plexus. Compression to this area results in caudaequina syndrome.

Signs include weakness of the muscles of the lower extremities, detrusor weaknesses causing urinary retention and post-void residual incontinence. There may be decreased anal tone and fecal incontinence; saddle anaesthesia; bilateral leg pain and weakness; and bilateral absence of ankle reflexes. Pain may be absent.

Below the L1 vertebra (transpyloric plane), the anterior and posterior nerve roots pass almost vertically downwards through the subarachnoid space and form the caudaequina. This consists only of anterior and posterior nerve roots. The subcostal plane lies at L3. A positive Babinski sign represents an upper motor neuron defect.

Prepared by Dr: Mohammed Musa Brema Idress – My best wishes Page 400

864. An 18-year-old male has come back from a camping holiday with his friends complaining of numb, swollen and blistered feet. What is the main cause of this trench foot condition? A. Chilblains B. Cold and damp conditions while wearing constricting foot wear C. Foot infection D. Frost bite E. Gout.

ANSWER IS B Trench foot or cold immersion foot (or hand) is caused due to a non-freezing injury of the hands or feet. This is typically seen in soldiers, sailors, or fishermen, who are chronically exposed to wet conditions and temperatures just above freezing, i.e. 1.6°C to 10°C (35°F to 50°F).It occurs due to microvascular endothelial damage, stasis and vascular occlusion. Although the entire foot may appear black, deep tissue destruction may not be present. An alternating arterial vasospasm and vasodilatation occurs, with the affected tissue first cold and anaesthetic, progressing to hyperaemia in 24 to 48 h.

This then leads to an intense painful burning and dysaesthesia, as well as tissue damage characterized by oedema, blistering, redness, ecchymosis, and ulcerations. Pruritic, red–purple skin lesions are a feature of chilblain or pernio. Complications of trench foot include local infection, cellulitis, lymphagitis, and gangrene.

865. A patient undergoes mastectomy and axillary node clearance for her right sided breast cancer. Post operatively she notices numbness in her axilla and upper inner arm. This is because of the division of which nerve? Single best answer question – choose ONE true option only. A. Internal thoracic nerve B. Intercostobrachial nerve C. Long thoracic nerve D. Thoracodorsal nerve E. Axillary nerve.

ANSWER IS B During axillary node clearance the intercostobrachial nerve which gives the cutaneous innervations of the axilla and the inner upper arm is encountered and is divided in many of the cases to ease the exposure to the axilla. The long thoracic nerve of Bell (nerve to serratus anterior) and the thoracodorsal nerve to latissimus dorsi are encountered during dissection and they are preserved.

866. You are treating a patient with an abdominal stab wound, which you suspect has lacerated the hepatic portal vein. Which of the following facts would allow you to locate the hepatic portal vein? Single best answer - choose ONE correct answer. A. It forms anterior to the neck of the pancreas B. It lies anterior to the first part of the duodenum C. It lies posterior to the hepatic artery proper

Prepared by Dr: Mohammed Musa Brema Idress – My best wishes Page 401

D. It lies posterior to the inferior vena cava E. It runs posterior to the epiploic foramen.

ANSWER IS C The portal vein forms the anterior boundary of the epiploic foramen, lying behind the bile duct and hepatic artery proper. It lies in front of the inferior vena cava, as it lies behind the pancreas and the first part of the duodenum. It is formed by the union of the splenic and superior mesenteric veins.

867. A 38-year-old man is diagnosed with a sliding hiatus hernia. Which pair of structures travel through the same diaphragmatic aperture as the hernia? Select one answer only. A. Azygous vein and inferior vena cava B. Azygous vein and right phrenic nerve C. Hemiazygous vein and oesophagus D. Vagus nerve and azygous vein E. Vagus nerve and oesophagus

ANSWER IS E The sliding hernia occurs as a result of the stomach and oesophagus moving cranially through the oesophageal opening of the diaphragm into the thoracic cavity. Three sets of structures pierce the diaphragm at three vertebral levels: 1. T8 = Right phrenic nerve and IVC; these pierce the diaphragm through the central tendon (caval opening) 2. T10 = Vagus nerve and Oesophagus; these pierce the diaphragm through its right crus (oesophageal opening) 3. T12 = Azygous vein, thoracic duct and abdominal aorta; these pass posterior to the diaphragm through the aortic opening.

868. The thyroid gland in some cases can have a thyroidea ima artery that supplies the isthmus of the thyroid. If present, it would take origin: Single best answer question – choose ONE true option only. A. From the inferior thyroid artery B. Directly from the thyrocervical trunk C. From the superior thyroid artery D. From the brachiocephalic trunk E. Directly from the external carotid artery

ANSWER IS D The thyroidea ima artery, when present, arises from the brachiocephalic trunk (innominate artery) and ascends in front of the trachea to the lower part of the thyroid gland, which it supplies. It varies greatly in size and appears to compensate for deficiency or absence of one of the other thyroid vessels. It occasionally arises from the aorta, the right common carotid, the subclavian or the internal mammary.

Prepared by Dr: Mohammed Musa Brema Idress – My best wishes Page 402

869. You are examining a child in the ENT clinic whose mother says has had previous problems with his Eustachian tubes but has no other details. The Eustachian tube: A. Can be obstructed by an enlarging palatine tonsil B. Closes during swallowing C. Drains the inner ear D. Gives attachment to the tensor veli palatini muscle E. Pierces the pharyngobasilar fascia to drain into the oropharynx.

ANSWER IS D The pharyngotympanic tube drains the middle ear into the nasopharynx by piercing the pharyngobasilar fascia. The cartilaginous part gives attachment to the tensor veli palatini muscle. It opens on swallowing under the action of the salpingopharyngeus and tensor palati muscles, and is obstructed by an enlarging pharyngeal tonsil (adenoids).

870. A 20-year-old man presents to A&E with a traumatic pneumothorax. He is obviously tachypnoeic but maintains adequate oxygen saturation. Which of the following is least likely to contribute to his respiratory effort? Select one answer only. A. Contraction of the intercostal muscles B. Descent of the hemidiaphragms C. An increase in the vertical dimension of the chest D. The long thoracic nerve of Bell (supplying the serratus anterior) E. Upward/forward movement of the first rib

ANSWER IS E In cases of impaired ventilation, tachypnoea can compensate the impaired gas exchange to an extent until fatigue of the respiratory muscles occurs. A number of factors are involved in respiration: - An increase in the vertical dimension of the chest on inspiration - An upwards and outwards movement of the ribs - A rise and fall of the hemi diaphragms.

The serratus anterior (supplied by the long thoracic nerve) is also involved in respiration. Note that the first rib does not move during respiration.

871. Following an elective splenectomy in a 21-year-old gentleman with Thalassaemia you are asked by the discharge co-coordinating team to review a discharge summary detailing the operative steps and subsequent treatment. Which of the following would be correct? Single best answer. A. The splenic artery was ligated after the splenic vein and lifelong oral Penicillin prophylaxis should not be offered to patients B. The splenic artery was ligated after the splenic vein and lifelong oral Penicillin prophylaxis should be offered to patients C. The splenic artery was ligated before the splenic vein and lifelong oral Penicillin prophylaxis should be offered to patients

Prepared by Dr: Mohammed Musa Brema Idress – My best wishes Page 403

D. The splenic artery was ligated before the splenic vein and lifelong oral Penicillin prophylaxis should not be offered to patients E. The splenic artery was ligated at the hilum and lifelong oral Penicillin prophylaxis should be offered to patients.

ANSWER IS C The spleen lies on the diaphragm, separated from the ninth to 11th ribs by the costodiaphragmatic recess. The left extremity of the lesser sac extends into the hilum. The peritoneal fold known as the lienorenal ligament (which contains the splenic vessels and the tail of the pancreas touching the spleen) passes from the left kidney to the spleen. Another peritoneal fold, the gastrosplenic ligament, carries the short gastric and gastroepiploic vessels and joins the spleen to the stomach.

The spleen is not palpable in the healthy individual, and must be enlarged by two to four times its normal size before it appears in the left hypochondrium.

The short gastric vessels need to be divided. In this scenario, the Splenic artery is usually ligated along the upper border of the tail of the Pancreas. The artery is ligated before the vein to minimize splenic engorgement.

Current guidelines: dictate that after splenectomy, lifelong prophylactic antibiotics should be offered to patients considered at continued high risk of pneumococcal infection using oral penicillins or macrolides (this advice is regularly reviewed in light of local pneumococcal resistance patterns).

Patients not at high risk should be counseled regarding the risks and benefits of lifelong antibiotics and may choose to discontinue them. All patients should carry a supply of appropriate antibiotics for emergency use.

873. A cut wound at the wrist affecting the median nerve will cause the following? Single best answer question – choose ONE true option only. A. The long flexors will be affected causing wasting the front of the forearm on the long run B. Flexor pollicis longus will be affected C. The hand is held typically with the index finger straight „pointing finger‟ D. Abduction of the thumb is affected E. Loss of sensation over the ulnar aspect of ring finger.

ANSWER IS D All the answers, except E, are features of high median nerve injury (more at the level of elbow), which affects the long flexors of the forearm (except flexor carpi ulnaris and flexor digitorum profundus) and the pronators. Lesions at the wrist level affect the sensation in the radial 3 ½ fingers (but not the ulnar aspect of ring finger which is supplied by ulnar nerve). The median nerve at the wrist also supplies the three thenar muscles (abductor and flexor pollicis brevis, and opponens pollicis) and the two radial lumbricals.

Prepared by Dr: Mohammed Musa Brema Idress – My best wishes Page 404

874. A 31-year-old basketball player felt a pop and sustained a non-contact pivoting injury to his knee. Immediate clinical examination shows haemoarthrosis. What is the most likely structure to have been damaged? A. Anterior cruciate ligament B. Lateral collateral ligament C. Medial meniscus tear D. Posterior cruciate ligament E. Quadriceps tendon rupture.

ANSWER IS A The knee joint is a synovial and a hinge joint. The capsule of the knee joint is thin anteriorly and posteriorly but reinforced on either side by strong collateral ligaments. Both the anterior and posterior cruciate ligaments are intracapsular, but extrasynovial. The medial and lateral menisci are C shaped, with their anterior and posterior horns attached to the intercondylar eminence of the tibia and their outer borders to the joint capsule.

The meniscofemoral ligament is adjacent to the posterior cruciate ligament and attaches the posterior border of the lateral meniscus close to the femoral attachment of the posterior cruciate ligament. It stabilizes the meniscus during rotation of the femur on the tibia. The oblique popliteal ligament is a lateral expansion from the insertion of semimembranosus which slopes up to the popliteal surface of the femur. The obliquity of this ligament limits rotation-extension in the „screw home‟ or locked position.

The anterior cruciate ligament, running upwards and backwards from the anterior part of the tibial plateau towards the lateral condyle of the femur, prevents backward displacement of the femur on the tibial plateau. It also limits extension of the lateral condyle of the femur, which then causes medial rotation of the femur in the „screw-home position of full extension. It is commonly injured in pivoting injuries, and often a pop is heard. The middle geniculate artery has branches within the ligament and therefore rupture results in an immediate haemarthrosis.

The posterior cruciate ligament, running obliquely from the posterior part of the tibia towards the medial condyle of the femur, prevents the femur from sliding forwards off the tibial plateau. In the weight-bearing, flexed knee it is the only stabilizing factor for the femur and its attached quadriceps. The suprapatellar bursa is continuous with the synovial cavity of the joint, and therefore it provides a route for injecting or withdrawing fluid into or from the joint. After injuries to the joint, fluid accumulates (effusion) in the suprapatellar bursa, causing typical fullness around the knee. The pre- and infrapatellar bursae, however, do not communicate with the joint.

875. In performing a lumbar puncture, the operator needs to be familiar with the anatomy involved. Which anatomical feature is relevant to this procedure? Single best answer question – choose ONE true option only. A. In the newborn baby, the spinal cord occupies the full length of the dural sac

Prepared by Dr: Mohammed Musa Brema Idress – My best wishes Page 405

B. The dural sac in the adult terminates at the lower end of the sacral canal C. The spinal cord in the normal adult terminates anywhere from opposite the body of T12 to the body of L3; however, the commonest level is at the disc space between L1 and L2 D. The spinal cord in the average male is 12 inches (30 cm) in length E. The extradural space comprises a thin layer of avascular connective tissue

ANSWER IS C The spinal cord in both adult men and women is 18 inches (45 cm) in length. In the newborn it terminates at L3. The dural sac in the adult extends to the level of the second sacral segment. The spinal extradural space contains loose fat (which allows the ready diffusion of local anaesthetic in an extradural block), together with the extensive vertebral venous plexus of veins.

876. A 46-year-old female presents with lipodermatosclerosis, haemosiderosis and an ulcer on the medial aspect of the left calf. She is diagnosed with venous insufficiency and undergoes surgery. What is the surface anatomical location of the saphenofemoral junction? A. 1cm medial to the femoral pulse B. 2cm inferior to mid-inguinal point C. 2cm inferior to the midpoint of the inguinal ligament D. 2cm lateral and inferior to the pubic tubercle E. 4cm lateral and inferior to the pubic tubercle

ANSWER IS E The saphenofemoral junction occurs 4cm lateral and inferior to the pubic tubercle. The long saphenous vein passes through the fascia lata to confluence with the femoral vein.

877. In the breast the connective tissue that extends from deep layer of superficial fascia to the dermis is called? Single best answer question – choose ONE true option only. A. Fibrous bands of Halstead B. Tubercles of Montgomery C. Retromammary bursa D. Ligaments of Cooper E. Poupart‟s ligament

ANSWER IS D The suspensory ligaments of the breast (Ligaments of Cooper) are fibrous bands of connective tissue that interdigitate between the breast tissues and extend from the deep layers of the superficial fascia to the dermis. These ligaments provide some shape to the breast tissue and are partly responsible for the “peau d‟orange” appearance associated with lymphoedema of malignancy.

Tubercles of Montgomery are small accessory glands on the areola. The retromammary bursa is the posterior aspect of the breast between the deep layer of

Prepared by Dr: Mohammed Musa Brema Idress – My best wishes Page 406 the superficial fascia and deep investing fascia of pectoralis major. Poupart‟s ligament is the inguinal ligament.

878. A 23-year-old male presents with a unilateral testicular swelling and is diagnosed with a teratoma. He undergoes orchidectomy and makes a full recovery. During his regular follow-up, a CT scan of the abdomen and pelvis reveals lymph nodes of more than 1 cm in diameter. Which lymph nodes are likely to be enlarged? A. External iliac lymph nodes B. Inguinal lymph nodes C. Internal iliac lymph nodes D. Para-aortic lymph nodes E. Sacral lymph nodes

ANSWER IS D The testes develop in the embryological abdomen, and so lymphatic drainage follows the route of the gonadal vessels to arrive at the para-aortic lymph nodes. The scrotum drains to the inguinal nodes.

879. A 76-year-old male is undergoing an oesophagectomy for adenocarcinoma of the distal oesophagus thought to be secondary to long standing GORD. Which part of the diaphragm does the oesophagus pass through? Select one answer only. A. Central tendon B. Left crus C. Median arcuate ligament D. Peripheral muscular part E. Right crus

ANSWER IS E It is important to know what structures pass through the diaphragm, at what vertebral levels, and also which specific part of the diaphragm. The aortic opening lies behind the diaphragm at T12 at vertebral level T12. It contains the aorta, thoracic duct, and often the azygos vein.

The oesophageal hiatus is at vertebral level T10 behind the seventh costal cartilage, and is formed by the medial fibers of the right crus of the diaphragm as they arch across to the left side of the midline. They form a tubular envelope around the oesophagus with an elliptical opening.

In addition to the oesophagus, it transmits the vagus nerve, left gastric artery and vein. The inferior vena cava passes through at T8 in the central tendon. The right phrenic nerve passes with it. The sympathetic trunk passes posterior to the medial arcuate ligament lying on psoas major. Each half of the diaphragm is supplied by its own phrenic nerve (C3, C4, C5) which is both motor and sensory. The intercostal nerves send some

Prepared by Dr: Mohammed Musa Brema Idress – My best wishes Page 407 proprioceptive fibers to the periphery of the diaphragm. The greater, lesser and least splanchnic nerves pierce each crus.

880. A 69-year-old diabetic lady with a history of a stroke affecting the right cerebral hemisphere is referred to the vascular surgeons as a Carotid duplex of the left shows a 75% stenosis of the internal carotid artery (ICA). Which of the following vessels is the first to arise from the ICA? Select one answer only. A. Anterior cerebral artery B. Middle cerebral artery C. Ophthalmic artery D. Posterior cerebral artery E. Superior thyroid artery.

ANSWER IS C The ICA does not give rise to any branches within the neck, the superior thyroid artery is usually the first branch of the external carotid artery. The ophthalmic artery is a branch of the internal carotid and enters the orbit through the optic foramen. The ophthalmic artery then crosses the optic nerve to the medial side of the orbit accompanied by the nasociliary nerve. The supraorbital and nasal arteries are two branches of the ophthalmic artery that supply part of the skin of the forehead. The ICA bifurcates into the anterior and middle cerebral arteries. The posterior cerebral artery is formed by the bifurcation of the basilar artery which arises from the junction of the left and right vertebral arteries.

881. You are inserting a central line into the internal jugular vein. Which of the following is a tributary of the internal jugular vein? A. The external jugular vein B. The inferior thyroid vein C. The middle thyroid vein D. The retromandibular vein E. The transverse cervical vein.

ANSWER IS C The internal jugular vein receives the facial, pharyngeal, lingual and superior and middle thyroid veins. It is formed from the sigmoid and inferior petrosal sinus and continues to the brachiocephalic vein. The external jugular arises from the junction of the posterior auricular vein and the posterior division of the retromandibular vein and drains into the subclavian. The transverse cervical vein is a tributary of the external jugular vein.

882. You see in clinic a 36-year-old man referred by his GP with a feeling of numbness in his left leg. On examination, he has decreased position sense and light touch and vibration sensation affecting his left leg to the upper part of the thigh. No other neurological deficit is demonstrable. Which of the following is the most likely cause of this presentation? Single best answer question – choose ONE true option only.

Prepared by Dr: Mohammed Musa Brema Idress – My best wishes Page 408

A. Left dorsal column lesion B. Left spinothalamic tract lesion C. Peripheral polyneuropathy D. Partial section of the spinal cord E. Sensory root compression.

ANSWER IS A The dorsal columns carry ipsilateral proprioreception, light touch and vibration sensation. Spinothalamic tract lesions cause contralateral loss of pain and temperature sensation. Partial section of the cord tends to cause contralateral pain and temperature sensation and ipsilateral loss of the modalities carried in the dorsal columns; there may also be upper motor neuron weakness. Sensory root compression typically causes pain in the dermatome supplied by the root; peripheral polyneuropathy is usually bilateral.

883. Following a left carotid endarterectomy, a 74-year-old man is found to have altered tongue movements. When asked to protrude his tongue, it deviates to the left. Which of the following structures could have been affected intra-operatively to cause this? A. Ansa cervicalis B. Hypoglossal nerve C. Lingual nerve D. Marginal mandibular nerve E. Vagus nerve

ANSWER IS B Damage to the hypoglossal nerve will result in weakness of the extrinsic muscles of the tongue on the ipsilateral side. The tongue will therefore deviate.

884. A 4-year-old boy is rushed into A&E by his mum who is concerned he has aspirated on one of his leg bricks he likes to put into his mouth. On examination, the child appears well with sats of 98%. A chest X-ray shows a foreign body impacted in the right main bronchus. Which of the following is the most appropriate management? Select one answer only. A. Conservative management B. CT guided removal C. Fibre-optic bronchoscopy to attempt to remove it D. Rigid bronchoscopy to attempt to remove it E. Thoracoscopic removal.

ANSWER IS D Inhaled or aspirated foreign bodies are more commonly seen in children than adults. A history of foreign body aspiration must be taken seriously, as the consequences of a retained foreign body in the lung may be significant. Following a careful history and examination, posteroanterior and lateral chest radiography may help to localize the foreign object. However, some objects may be radiolucent and may not be seen on the chest radiograph.

Prepared by Dr: Mohammed Musa Brema Idress – My best wishes Page 409

Further investigation is imperative, the next step being either fibre-optic or preferably rigid bronchoscopy. Rigid bronchoscopy is preferred for recovery of foreign body aspiration because it allows protection of the airway and controlling the foreign body during recovery. The anatomy of the bronchial tree, in particular a less angulated right main bronchus, makes this the commonest site of impaction.

885. During the course of an abdominal hysterectomy, you are asked to ligate and divide the uterine arteries. From which artery do they originate? Single best answer - select one answer only. A. Abdominal aorta B. Common iliac artery C. External iliac artery D. Internal iliac artery E. Median sacral artery.

ANSWER IS D The uterine arteries originate from the internal iliac artery. From there, they traverse the ureter and travel within the cardinal and broad ligaments before anastomosing with the ovarian artery.

886. In a fracture of the surgical neck of humerus (Neer‟s classification Group III) the following important structure might be affected? Single best answer question – choose ONE true option only. A. The axillary artery B. The axillary vein C. The radial nerve D. The axillary nerve E. The profunda brachii artery

ANSWER IS D The important anatomical relationships to the humerus are the axillary nerve and circumflex humeral vessels at the surgical neck, the ulnar nerve at the posterior aspect of the medial epicondyle, and the radial nerve and the profunda brachii vessels at the surgical groove. Fractures affecting those sites might affect the related structures. In proximal humeral fractures, Surgical neck fractures (Neer group III) might cause axillary nerve damage, anatomical neck fractures (Neer Group II) might result in avascular necrosis of the humeral head while greater tuberosity fractures (Neer group IV) might result in painful arc syndrome.

887. A 14-year-old footballer presents with severe pain and swelling of the left ankle. Plain radiographs confirm a bimalleolar ankle fracture. The distal tibia articulates with which bone? A. Cuboid B. Medial cuneiform C. Navicular D. Os calcis

Prepared by Dr: Mohammed Musa Brema Idress – My best wishes Page 410

E. Talus

ANSWER IS E The ankle joint is formed of the distal tibia and fibula, articulating with the talus. The cuboid, cuneiform and navicular bones form the mid foot, with the oscalcis (or calcaneum) forming part of the hind foot.

888. You are the CT2 on a cardiothoracic surgery rotation. A patient you are reviewing has a complex thoracic tumour 'compressing the right phrenic nerve' according to the CT report. Which of the following is correct about the right phrenic nerve? Select one answer only. A. Arises from the ventral rami of the second to fifth cervical nerves B. Lies in contact with the pericardium of the right atrium C. Lies superficial to the pre-vertebral fascia D. Passes medially across the border of stylopharyngeus muscle E. Pierces the diaphragm at the level of T12 vertebra.

ANSWER IS B The phrenic nerve originates from the ventral rami of the third to fifth cervical nerves. It passes inferiorly down the neck to the lateral border of scalenus anterior muscle. Then, it passes medially across the scalenus anterior parallel to the internal jugular vein. At this point it is deep to the prevertebral fascia, the transverse cervical artery and the suprascapular artery.

Within the thorax, the right phrenic nerve is in contact with mediastinal pleura laterally and medially, the following venous structures: right brachiocephalic vein; superior vena cava; pericardium of the right atrium and inferior vena cava. The right phrenic nerve pierces the diaphragm in its tendinous portion just slightly lateral to the inferior vena caval foramen. This is at the level of T8 vertebra (T12 level is the level of aortic opening in the diaphragm).

889. A 17-year-old woman presents to her GP who after investigation diagnoses Horner’s syndrome secondary to a cervical rib. Which of the following is NOT a feature of Horner’s syndrome? Select one answer only. A. Disruption of sympathetic supply to the pulmonary plexus B. Loss of sweating over the C4 dermatome C. Ptosis D. Pupillary constriction E. Vasomotor changes in the arm.

ANSWER IS A In Horner’s syndrome there is: ptosis, pupillary constriction and occasional enophthalmos, and dryness and flushing of the skin of the head and neck. The sympathetic supply to the lungs is preserved as this originates below the lesion directly from the T1–T4 ganglia of the sympathetic chain. Sympathetic fibers pass to the arm via grey rami from the middle and inferior cervical sympathetic ganglia through all the roots of the brachial plexus.

Prepared by Dr: Mohammed Musa Brema Idress – My best wishes Page 411

890. A 25-year-old woman with history of high-grade fever, gradual-onset loss of consciousness and a petechial rash is suspected of having bacterial meningitis. As part of the diagnostic procedure, a lumbar puncture is to be performed. To avoid injury to the spinal cord and nerves you must insert the spinal needle just below the spine of the fourth lumbar vertebra. What anatomical landmark would you use to identify the spine of the fourth lumbar vertebra? Single best answer question – choose ONE true option only. A. Pubic symphysis B. Iliac crest C. Xiphoid process D. Iliac tuberosity E. Umbilicus.

ANSWER IS B The fourth lumbar vertebra (L4) is a relatively safe level for performing a lumbar puncture. Since the conus medullaris is at the inferior border of L1 or the superior border of L2, it should be safe to insert a needle either above or below L4. The anatomical landmark used to identify L4 is the top of the iliac crest.

The line connecting the top of the two iliac crests, the supracrestal line, passes through the spinous process of the L4 vertebra. Therefore, by finding the tops of the iliac crests, you should be able to identify L4.

891. A 62-year-old male with alcoholic liver disease is deteriorating in ICU following insertion of a Sengstaken-Blakemore tube for bleeding oesophageal varices. His chest X-ray shows a pneumomediastinum and he is thought to have perforated his oesophagus. Which of the following structures which run adjacent to the oesophagus makes an indentation on the left lung, but not the right lung? Select one answer only. A. Azygos vein B. Oesophagus C. Phrenic nerve D. Superior vena cava E. Thoracic duct

ANSWER IS E Impressions on the mediastinal surface of the right lung include the trachea, vagus, superior vena cava, right atrium and subclavian artery. The oesophagus grooves the left lung above the arch of the aorta and below the hilum. There is also a vertical groove for the oesophagus on the right lung.

892. During a laparoscopic low anterior resection for a mid-rectal neoplasm, the splenic flexure has to be mobilized to ensure a tension-free anastomosis. During dissection, the anatomical structure in relation to the body of the pancreas is which of the following? A. The greater curve of the stomach B. The inferior mesenteric vein

Prepared by Dr: Mohammed Musa Brema Idress – My best wishes Page 412

C. The right kidney D. The transverse mesocolon E. Lies in the transpyloric plane.

ANSWER IS D The head of the pancreas is related to the hilum but does not overlie the right kidney. It is, however, anterior to the left kidney. The transpyloric plane (L1) transects the pancreas obliquely, passing through the midpoint of the neck, with most of the head below the plane, and most of the body and tail above.

The transverse mesocolon is attached to the head, neck and body of the pancreas. The uncinate process lies posterior to the superior mesenteric vessels, and the inferior mesenteric vein passes behind the body of the pancreas, where it joins the splenic vein. Entering the lesser sac between the transverse mesocolon and body of pancreas is a common method for laparoscopic mobilization of the splenic flexure.

893. The anatomical description of Smith’s fracture is? Single best answer question – choose ONE true option only. A. It is an intra-articular distal radial fracture with volar displacement B. It is an extra-articular proximal radial fracture with volar displacement C. It is an extra-articular distal radial fracture with volar displacement D. It is an extra-articular fracture with dorsal and radial displacements of the distal fragment E. It is an intra-articular distal radial fracture either volar or dorsal where the articular surface of the distal radius subluxes from the carpals.

ANSWER IS C The three commonest distal radial fractures are Colles‟, Smith’s and Barton’s. Colles‟ and Smith’s are extra-articular while Barton’s is Intra-Articular. Answer D describes Colles‟ fracture while answer E describes Barton’s Fracture.

894. A young patient is tilting her pelvis while walking. Which clinical test is used to assess the hip abductors? A. Barlow’s test B. Ober test C. Ortolani test D. Thomas test E. Trendelenburg test

ANSWER IS E Trendelenburg test: This test is performed with the patient standing. The patient is asked to raise one leg; the test is positive if the hip on the raised side drops. - A positive test suggests weakness of the abductors of the other hip. Ober's tests for tight iliotibial band. The Thomas test, is for tight hip flexors. Barlow and Ortolani tests can be used in combination at birth and can elicit a dislocated hip, assess its reducibility and diagnose an unstable hip which is dislocatable.

Prepared by Dr: Mohammed Musa Brema Idress – My best wishes Page 413

895. A 24-year-old woman attends the neurological clinic for review of multiple sclerosis, diagnosed 2 years before. She had presented with blurring of vision and mild pain in her left eye, which had resolved over a period of 3 months and had not recurred. On examination now, the following observations are made: light shone in the left eye causes constriction of the left and right pupils; light shone into the right eye causes constriction of the right and left pupils but when the light is shone back into the left eye, the left pupil dilates slightly. Which of the following is the most likely site of the lesion responsible? Single best answer question – choose ONE true option only. A. Left ciliary ganglion B. Left oculomotor nerve C. Left optic nerve D. Right ciliary ganglion E. Right optic nerve.

ANSWER IS C The abnormal response is on the left; the right pupil shows a normal direct and consensual response. The normal constriction of the left pupil indicates that the efferent pathway (involving the Edinger–Westphal nucleus, oculomotor nerve and ciliary ganglion) is intact. The defect is in the afferent pathway on the left, which involves fibers in the optic nerve.

The sign demonstrated is a relative afferent pupillary defect (RAPD), implying partial damage only to the afferent pathway: if all function is lost, neither the direct reflex nor consensual reflex (constriction of the right pupil in response to light shone into the left eye) will be present.

896. A patient complains of deficit in the cutaneous field halfway down the anterior surface of the thigh. This: Single best answer question – choose ONE true option only. A. Is due to damage to the sciatic nerve B. Would result from compression of the ventral roots of L5 to S2 C. Would result from damage to a nerve accompanying the artery in the adductor canal D. Could be the result of nerve damage during surgical procedures in the femoral sheath E. Would result from damage to the nerve that innervates the pectineus muscle

ANSWER IS E The pectineus is supplied by the second, third and fourth lumbar nerves through the femoral nerve and by the third lumbar through the accessory obturator when this exists. Occasionally it receives a branch from the obturator nerve. The anterior surface of the thigh receives its innervation from, the femoral nerve as well so that is the nerve most likely to be injured.

In the thigh, the anterior division of the femoral nerve gives off anterior cutaneous branches. The anterior cutaneous branches comprise the intermediate and medial

Prepared by Dr: Mohammed Musa Brema Idress – My best wishes Page 414 cutaneous nerves. The intermediate cutaneous nerve pierces the fascia lata (and generally the sartorius) about 7.5 cm below the inguinal ligament and divides into two branches that descend in immediate proximity along the forepart of the thigh to supply the skin as low as the front of the knee.

Here they communicate with the medial cutaneous nerve and the infrapatellar branch of the saphenous, to form the patellar plexus. In the upper part of the thigh, the lateral branch of the intermediate cutaneous communicates with the lumboinguinal branch of the genitofemoral nerve.

897. In an adult female the breast lies over these structures except? Single best answer question – choose ONE true option only. A. Pectoralis major B. Serratus anterior C. Internal oblique D. External oblique E. Rectus sheath

ANSWER IS C The breast extends from the second to the 6th rib overlying the pectoralis major muscle, extending over the serratus anterior laterally, the rectus sheath inferomedially, the external oblique inferolaterally and the costal cartilage medially.

898. A 48-year-old male presents with symptoms of fulminant hepatic failure secondary to alcohol abuse. He is noted to have hepatosplenomegaly, anorectal varices and caput medusa. Which anatomical structure is just anterior to the origin of the hepatic portal vein? A. Hepatic flexure B. Inferior vena cava C. Pancreatic body D. Pancreatic neck E. Pancreatic tail

ANSWER IS D The hepatic portal vein is formed of the confluence of the splenic and superior mesenteric veins, just posterior to the pancreatic neck. It supplies 75% of all blood flow to the liver. The inferior vena cava lies posterior to its origin.

899. During a coronary angiography a tight stenosis is noted at the proximal aspect of the left coronary artery. Which of the following is the most important branch of the left coronary artery? Select one answer only. A. Anterior interventricular B. Atrioventricular nodal C. Circumflex D. Marginal E. Posterior interventricular.

Prepared by Dr: Mohammed Musa Brema Idress – My best wishes Page 415

ANSWER IS A The left coronary artery divides into the left anterior interventricular artery also known as the left anterior descending artery (LAD) and circumflex artery. It’s most important branch is the anterior interventricular artery which supplies the anterior aspect of both ventricles and passes around the apex of the heart to anastomose with the posterior interventricular artery. The right coronary artery gives off a posterior interventricular branch. The right coronary artery supplies the AV node in 80% of cases.

900. A 61-year-old female is commenced on anti-hypertensives by her GP as her blood pressure has been elevated on the last three visits. Following her first dose at home, she noted her tongue became very swollen and she had some difficulty breathing. She was rushed to the emergency department and a diagnosis of angio- neurotic oedema was made. Which group of the following anti-hypertensives is most likely to have precipitated this? Select one answer only. A. ACE inhibitors B. α blockers C. β blockers D. Calcium channel blockers E. Thiazide diuretics.

ANSWER IS A Angio-neurotic oedema is tongue swelling secondary to ACE inhibitors. The most important initial management is to secure the airway; a nasotracheal tube may be required. Steroids (which may take 6 hours to take full effect) and antihistamine should be commenced immediately.

901. An 18-year-old female presented with ear-ache, a conductive deafness and a temperature of 39oC. What is the most common cause? A. Paget’s disease of bone B. Acoustic neuroma C. Otosclerosis D. Fracture through the petrous temporal bone E. Otitis media

ANSWER IS E The most common causes of conductive deafness include wax, acute otitis media, secretory otitis media, chronic otitis media, barotrauma, otosclerosis and injuries to the tympanic membrane and otitis externa. Other less common causes include tumours of the middle ear and traumatic ossicular dislocation.

Sensory neural deafness is caused by a number of causes including infections such as mumps, herpes zoster, meningitis and syphilis. Other causes include congenital– maternal rubella, cytomegalovirus, toxoplasmosis, prolonged exposure to loud noises, drugs (aspirin, aminoglycosides), Meniere‟s disease, head injury and acoustic neuroma.

Prepared by Dr: Mohammed Musa Brema Idress – My best wishes Page 416

Metabolic causes include diabetes and hypothyroidism. In Paget’s disease there may be a mixed hearing loss i.e. conduction and sensorineural deafness. This is due to direct involvement of the ossicles of the inner ear due to ankylosis of the stapes, or by impeachment of bone on the eighth cranial nerve in the auditory foramen.

902. Which of the structures listed below lies posterior to the carotid sheath at the level of the 6th cervical vertebra? A. Hypoglossal nerve B. Vagus nerve C. Cervical sympathetic chain D. Ansa cervicalis E. Glossopharyngeal nerve

ANSWER IS C The carotid sheath is crossed anteriorly by the hypoglossal nerves and the ansa cervicalis. The vagus lies within it. The cervical sympathetic chain lies posteriorly between the sheath and the prevertebral fascia.

903. A sprinter attends A&E with severe leg pain. He had forgotten to warm up and ran a 100m sprint race. Towards the end of the race he experienced pain in the posterior aspect of his thigh. The pain worsens, localizing to the lateral aspect of the knee. The sprinter is unable to flex the knee. What structure has been injured? A. Anterior cruciate ligament B. Posterior cruciate ligament C. Semimembranosus tendon D. Semitendinosus tendon E. Biceps femoris tendon

ANSWER IS E The biceps femoris is commonly injured in sports that require explosive bending of the knee as seen in sprinting, especially if the athlete has not warmed up first. Avulsion most commonly occurs where the long head attaches to the ischial tuberosity. Injuries to biceps femoris are more common than to the other hamstrings.

904. A 24 year old man falls and lands astride a manhole cover. He suffers from an injury to the anterior bulbar urethra. Where will the extravasated urine tend to collect? A. Lesser pelvis B. Connective tissue of the scrotum C. Deep perineal space D. Ischiorectal fossa E. Posterior abdominal wall

ANSWER IS B This portion of the urethra is contained between the perineal membrane and the membranous layer of the superficial fascia. As these are densely adherent to the

Prepared by Dr: Mohammed Musa Brema Idress – My best wishes Page 417 ischiopubic rami, extravasated urine cannot pass posteriorly because the 2 layers are continuous around the superficial transverse perineal muscles.

905. A 73 year old man presents with symptoms of mesenteric ischaemia. As part of his diagnostic work up a diagnostic angiogram is performed .The radiologist is attempting to cannulate the coeliac axis from the aorta. At which of the following vertebral levels does this is usually originate? A. T10 B. L2 C. L3 D. T8 E. T12

ANSWER IS E Coeliac trunk branches: Left Hand Side (LHS) 1. Left gastric 2. Hepatic 3. Splenic The coeliac axis branches off the aorta at T12.

906. A 43 year old man is diagnosed as having a malignancy of the right adrenal gland. The decision is made to resect this via an open anterior approach. Which of the following will be most useful during the surgery? A. Division of the coronary ligaments of the liver B. Mobilization of the colonic hepatic flexure C. Division of the right renal vein D. Division of the ligament of Trietz E. Division of the right colic artery

ANSWER IS B Mobilization of the hepatic flexure and right colon are standard steps in open adrenal surgery from an anterior approach. Mobilization of the liver is seldom required.

907. A 45 year old man presents with a lipoma located posterior to the posterior border of the sternocleidomastoid muscle, approximately 4cm superior to the middle third of the clavicle. During surgical excision of the lesion troublesome bleeding is encountered. Which of the following is the most likely source? A. Internal jugular vein B. External jugular vein C. Common carotid artery D. Vertebral artery E. Second part of the subclavian artery

ANSWER IS B

Prepared by Dr: Mohammed Musa Brema Idress – My best wishes Page 418

The external jugular vein runs obliquely in the superficial fascia of the posterior triangle. It drains into the subclavian vein. During surgical exploration of this area the external jugular vein may be injured and troublesome bleeding may result. The internal jugular vein and carotid arteries are located in the anterior triangle. The third, and not the second, part of the subclavian artery is also a content of the posterior triangle

908. The sciatic nerve lies deep to the following structures except: A. Gluteus maximus B. The femoral cutaneous nerve C. Long head of biceps femoris D. Gluteus medius E. Branch of the inferior gluteal artery

ANSWER IS D The gluteus medius does not extend around to the sciatic nerve

909. Which of the following upper limb muscles is not innervated by the radial nerve ? A. Extensor carpi ulnaris B. Abductor digiti minimi C. Anconeus D. Supinator E. Brachioradialis

ANSWER IS B Mnemonic for radial nerve muscles: BEST B rachioradialis E xtensors S upinator T riceps Abductor digiti minimi is innervated by the ulnar nerve

910.Which of the following forms the floor of the anatomical snuffbox? A. Radial artery B. Cephalic vein C. Extensor pollicis brevis D. Scaphoid bone E. Cutaneous branch of the radial nerve

ANSWER IS D The scaphoid bone forms the floor of the anatomical snuffbox. The cutaneous branch of the radial nerve is much more superficially and proximally located.

911. During a liver resection a surgeon performs a pringles manoeuvre to control bleeding. Which of the following structures will lie posterior to the epiploic foramen at this level?

Prepared by Dr: Mohammed Musa Brema Idress – My best wishes Page 419

A. Hepatic artery B. Cystic duct C. Greater omentum D. Superior mesenteric artery E. Inferior vena cava

ANSWER IS E Bleeding from liver trauma or a difficult cholecystectomy can be controlled with a vascular clamp applied at the epiploic foramen.

The epiploic foramen has the following boundaries: Anteriorly: (in the free edge of the lesser omentum): Bile duct to the right, portal vein behind and hepatic artery to the left. Posteriorly: Inferior vena cava Inferiorly: 1st part of the duodenum Superiorly: Caudate process of the liver

912. A 72 year old lady is suspected of having a femoral hernia. At which of the following sites is it most likely to be identifiable clinically ? A. Mid inguinal point B. Above and medial to the pubic tubercle C. Below and lateral to the pubic tubercle D. Midpoint of the inguinal ligament E. 3 cm superomedially to the superficial inguinal ring

ANSWER IS C Femoral hernias exit the femoral canal below and lateral to the pubic tubercle. Femoral hernia occur mainly in women due to their difference in pelvic anatomy. They are at high risk of strangulation and therefore should be repaired.

913. Which muscle is responsible for causing flexion of the distal interphalangeal joint of the ring finger? A. Flexor digitorum superficialis B. Lumbricals C. Palmar interossei D. Flexor digitorum profundus E. Flexor digiti minimi brevis

ANSWER IS D Flexor digitorum superficialis and flexor digitorum profundus are responsible for causing flexion. The superficialis tendons insert on the bases of the middle phalanges; the profundus tendons insert on the bases of the distal phalanges. Both tendons flex the wrist, MCP and PIP joints; however, only the profundus tendons flex the DIP joints.

Prepared by Dr: Mohammed Musa Brema Idress – My best wishes Page 420

914. A 34 year old lady undergoes a thyroidectomy for Graves’s disease. Post operatively she develops a tense haematoma in the neck. In which of the following fascial planes will it be contained? A. Gerotas fascia B. Waldeyers fascia C. Pretracheal fascia D. Sibson’s fascia E. Clavipectoral fascia

ANSWER IS C The pretracheal fascia encloses the thyroid and is unyielding. Therefore tense haematomas can develop.

915. A 32 year old lady complains of carpal tunnel syndrome. The carpal tunnel is explored surgically. Which of the following structures will lie in closest proximity to the hamate bone within the carpal tunnel ? A. The tendon of abductor pollicis longus B. The tendons of flexor digitorum profundus C. The tendons of flexor carpi radialis longus D. Median nerve E. Radial artery

ANSWER IS B The carpal tunnel contains nine flexor tendons:- 1. Flexor digitorum profundus 2. Flexor digitorum superficialis 3. Flexor pollicis longus The tendon of flexor digitorum profundus lies deepest in the tunnel and will thus lie nearest to the hamate bone.

916. A 45 year old man sustains a significant head injury and a craniotomy is performed. The sigmoid sinus is bleeding profusely, into which of the following structures does it drain ? A. Internal jugular vein B. Straight sinus C. Petrosal sinus D. Inferior sagittal sinus E. External jugular vein

ANSWER IS A The sigmoid sinus is joined by the inferior petrosal sinus to drain into the internal jugular vein.

917. Which nerve supplies the interossei of the fourth finger? A. Radial B. Median C. Superficial ulnar

Prepared by Dr: Mohammed Musa Brema Idress – My best wishes Page 421

D. Deep ulnar E. Posterior interosseous

ANSWER IS D Mnemonic : PAD and DAB - Palmer interossei Adduct - Dorsal interossei Abduct

918. In which of the following cranial bones does the foramen spinosum lie ? A. Sphenoid bone B. Frontal bone C. Temporal bone D. Occipital bone E. Parietal bone

ANSWER IS A The foramen spinosum (which transmits the middle meningeal artery and vein) lies in the sphenoid bone. R -v2 O-v3 S-MMA L-Ica And superior orbital

919. Which of the following is not considered a major branch of the descending thoracic aorta ? A. Bronchial artery B. Mediastinal artery C. Inferior thyroid artery D. Posterior intercostal artery E. Oesophageal artery

ANSWER IS C They all branches of thoracic aorta except inferior thyroid The inferior thyroid artery is usually derived from the thyrocervical trunk, a branch of the subclavian artery.

920. An 18 year old lady with troublesome hyperhidrosis of the hands and arms is due to undergo a sympathectomy to treat the condition. Which of the following should the surgeons divide to most effectively treat her condition? A. Sympathetic ganglia at T1, T2 and T3 B. Sympathetic ganglia at T2 and T3 C. Sympathetic ganglia at T1 and T2 D. Stellate ganglion E. Superior cervical ganglion

Prepared by Dr: Mohammed Musa Brema Idress – My best wishes Page 422

ANSWER IS B To treat hyperhidrosis the sympathetic ganglia at T2 and T3 should be divided. Dividing the other structures listed would either carry a risk of Horner’s syndrome or be ineffective.

921. A 44 year old lady is recovering following a transphenoidal hypophysectomy. Unfortunately there is a post operative haemorrhage. Which of the following features is most likely to occur initially? A. Cavernous sinus thrombosis B. Abducens nerve palsy C. Bi-temporal hemianopia D. Inferior homonymous hemianopia E. Central retinal vein occlusion

ANSWER IS C The pituitary is covered by a sheath of dura and an expanding haematoma at this site may compress the optic chiasm in the same manner as an expanding pituitary tumour.

922. During a right hemicolectomy the caecum is mobilized. As the bowel is retracted medially a vessel is injured, posterior to the colon. Which of the following is the most likely vessel? A. Right colic artery B. Inferior vena cava C. Aorta D. External iliac artery E. Gonadal vessels

ANSWER IS E The gonadal vessels and ureter are important posterior relations that are at risk during a right hemicolectomy.

923. A 53 year old man with a carcinoma of the lower third of the oesophagus is undergoing an oesophagogastrectomy. As the surgeons mobilize the lower part of the oesophagus, where are they most likely to encounter the thoracic duct? A. Anterior to the oesophagus B. On the left side of the oesophagus C. On the right side of the oesophagus D. Immediately anterior to the azygos vein E. Posterior to the oesophagus

ANSWER IS E The thoracic duct lies posterior to the oesophagus and passes to the left at the level of the Angle of Louis. It enters the thorax at T12 together with the aorta.

924. Which of the following represents the root values of the sciatic nerve? A. L4 to S3

Prepared by Dr: Mohammed Musa Brema Idress – My best wishes Page 423

B. L1 to L4 C. L3 to S1 D. S1 to S4 E. L5 to S1

ANSWER IS A The sciatic nerve most commonly arises from L4 to S3.

925. The common peroneal nerve, or its branches, supply the following muscles except: A. Peroneus longus B. Tibialis anterior C. Extensor hallucis longus D. Flexor digitorum brevis E. Extensor digitorum longus

ANSWER IS D Flexor digitorum is supplied by the tibial nerve.

926. An 83 year old lady presents with a femoral hernia and undergoes a femoral hernia repair. Which of the following forms the posterior wall of the femoral canal? A. Pectineal ligament B. Lacunar ligament C. Inguinal ligament D. Adductor longus E. Sartorius

ANSWER IS A Borders of the femoral canal:- Laterally: Femoral vein Medially: Lacunar ligament Anteriorly: Inguinal ligament Posteriorly: Pectineal ligament

927. A 45 year man presents with hand weakness. He is given a piece of paper to hold between his thumb and index finger. When the paper is pulled, the patient has difficulty maintaining a grip. Grip pressure is maintained by flexing the thumb at the interphalangeal joint. What is the most likely nerve lesion? A. Posterior interosseous nerve B. Deep branch of ulnar nerve C. Anterior interosseous nerve D. Superficial branch of the ulnar nerve E. Radial nerve

ANSWER IS B

Prepared by Dr: Mohammed Musa Brema Idress – My best wishes Page 424

This is a description of Froment's sign, which tests for ulnar nerve palsy. It mainly tests for the function of adductor pollicis. This is supplied by the deep branch of the ulnar nerve. Remember the anterior interosseous branch (of the median nerve), which innervates the flexor pollicis longus (hence causing flexion of the thumb IP joint), branches off more proximally to the wrist.

928. Which of the following statements relating to the right phrenic nerve is false? A. It lies deep to the prevertebral layer of deep cervical fascia B. Crosses posterior to the 2nd part of the subclavian artery C. It runs on the anterior surface of the scalene muscle D. On the right side it leaves the mediastinum via the vena cava hiatus at a level of T8 E. The right phrenic nerve passes over the right atrium

ANSWER IS B Phrenic nerve Origin = C3,4,5 Supplies: Diaphragm, sensation central diaphragm and pericardium

Path The phrenic nerve passes with the internal jugular vein across scalenus anterior. It passes deep to prevertebral fascia of deep cervical fascia. Left: crosses anterior to the 1st part of the subclavian artery. Right: Anterior to scalenus anterior and crosses anterior to the 2nd part of the subclavian artery. On both sides, the phrenic nerve runs posterior to the subclavian vein and posterior to the internal thoracic artery as it enters the thorax.

Right phrenic nerve In the superior mediastinum: anterior to right vagus and laterally to superior vena cava Middle mediastinum: right of pericardium It passes over the right atrium to exit the diaphragm at T8

Left phrenic nerve Passes lateral to the left subclavian artery, aortic arch and left ventricle. Passes anterior to the root of the lung. Pierces the diaphragm alone

929. Which of the following cranial foramina pairings are incorrect? A. Carotid canal and internal carotid artery. B. Foramen ovale and mandibular nerve. C. Optic canal and ophthalmic artery. D. Optic canal and ophthalmic nerve. E. Foramen rotundum and maxillary nerve.

ANSWER IS D

Prepared by Dr: Mohammed Musa Brema Idress – My best wishes Page 425

The optic canal transmits the optic nerve. The ophthalmic nerve traverses the superior orbital fissure.

930. A 22 year old man is involved in a fight and sustains a stab wound in his upper forearm. On examination there is a small, but deep laceration. There is an obvious loss of pincer movement involving the thumb and index finger with minimal loss of sensation. The most likely nerve injury is to the: A. Ulnar nerve B. Radial nerve C. Anterior interosseous nerve D. Axillary nerve E. Median nerve

ANSWER IS C The anterior interosseous nerve is a motor branch of the median nerve just below the elbow. When damaged it classically causes: 1. Pain in the forearm 2. Loss of pincer movement of the thumb and index finger (innervates the long flexor muscles of flexor pollicis longus & flexor digitorum profundus of the index and middle finger) 3. Minimal loss of sensation due to lack of a cutaneous branch

931. A 66 year old man is undergoing a left nephro-ureterectomy. The surgeons remove the ureter, which of the following is responsible for the blood supply to the proximal ureter? A. Branches of the renal artery B. External iliac artery C. Internal iliac artery D. Direct branches from the aorta E. Common iliac artery

ANSWER IS A The proximal ureter is supplied by branches from the renal artery.

932. Which of the following structures does not pass behind the lateral malleolus? A. Peroneus brevis tendon B. Sural nerve C. Short saphenous vein D. Peroneus longus tendon E. Tibialis anterior tendon

ANSWER IS E Tibialis anterior tendon passes anterior to the medial malleolus.

933. A 78 year old man presents with symptoms consistent with intermittent claudication. To assess the severity of his disease you decide to measure his ankle

Prepared by Dr: Mohammed Musa Brema Idress – My best wishes Page 426 brachial pressure index. To do this you will identify the dorsalis pedis artery. Which of the following statements relating to this vessel is false? A. It originates from the peroneal artery B. It is crossed by the tendon of extensor hallucis brevis C. Two veins are usually closely related to it D. It passes under the inferior extensor retinaculum E. The tendon of extensor digitorum longus lies lateral to it

ANSWER IS A The dorsalis pedis artery is a direct continuation of the anterior tibial artery

934. Which of the following is not a content of the anterior triangle of the neck? A. Vagus nerve B. Submandibular gland C. Phrenic nerve D. Internal jugular vein E. Hypoglossal nerve

ANSWER IS C The phrenic nerve is a content of the posterior triangle. The anterior triangle contains the carotid sheath and its contents.

935. A 32 year old attends neurology clinic complaining of tingling in his hand. He has radial deviation of his wrist and there is mild clawing of his fingers, with the 4th and 5th digits being relatively spared. What is the most likely lesion? A. Ulnar nerve damage at the wrist B. Ulnar nerve damage at the elbow C. Radial nerve damage at the elbow D. Median nerve damage at the wrist E. Median nerve damage at the elbow

ANSWER IS B The ulnar paradox- the higher the lesion, the less the clawing of the fingers seen clinically. At the elbow the ulnar nerve lesion affects the flexor carpi ulnaris and flexor digitorum profundus.

936. A 22 year old man is undergoing an endotracheal intubation. Which of the following vertebral levels is consistent with the origin of the trachea? A. C2 B. T1 C. C6 D. C4 E. C3

ANSWER IS C - The trachea commences at C6.

Prepared by Dr: Mohammed Musa Brema Idress – My best wishes Page 427

- It terminates at the level of T5 (or T6 in tall subjects in deep inspiration).

937. A young child undergoes a difficult craniotomy for fulminant mastoiditis and associated abscess. During the procedure the trigeminal nerve is severely damaged within Meckels cave. Which deficit is least likely to be present? A. Anaesthesia over the ipsilateral anterior aspect of the scalp B. Loss of the corneal reflex C. Weakness of the ipsilateral masseter muscle D. Anaesthesia of the anterior aspect of the lip E. Anaesthesia over the entire ipsilateral side of the face

ANSWER IS E The angle of the jaw is not innervated by sensory fibers of the trigeminal nerve and is spared in this type of injury.

Remember the trigeminal nerve provides motor innervation to the muscles of mastication. The close proximity of the site of injury to the motor fibers is likely to result in at least some compromise of motor muscle function.

938. Which of the following is not a branch of the external carotid artery? A. Facial artery B. Lingual artery C. Superior thyroid artery D. Mandibular artery E. Maxillary artery

ANSWER IS D External carotid artery branches mnemonic: 'Some Angry Lady Figured Out PMS' 1. Superior thyroid (superior laryngeal artery branch) 2. Ascending pharyngeal 3. Lingual 4. Facial (tonsillar and labial artery) 5. Occipital 6. Posterior auricular 7. Maxillary (inferior alveolar artery, middle meningeal artery) 8. Superficial temporal

939. A 23 year old man is stabbed in the groin, several structures are injured and the adductor longus muscle has been lacerated. Which of the following nerves is responsible for the innervation of adductor longus? A. Femoral nerve B. Obturator nerve C. Sciatic nerve D. Common peroneal nerve E. Ilioinguinal nerve

Prepared by Dr: Mohammed Musa Brema Idress – My best wishes Page 428

ANSWER IS B The adductors are innervated by the obturator nerve

940. Which of the following statements relating to the basilar artery and its branches is false? A. The superior cerebellar artery may be decompressed to treat trigeminal neuralgia B. Occlusion of the posterior cerebral artery causes contralateral loss of the visual field C. The oculomotor nerve lies between the superior cerebellar and posterior cerebral arteries D. The posterior inferior cerebellar artery is the largest of the cerebellar arteries arising from the basilar artery E. The labyrinthine branch is accompanied by the facial nerve

ANSWER IS D The posterior inferior cerebellar artery is the largest of the cerebellar arteries arising from the vertebral artery. The labyrinthine artery is long and slender and may arise from the lower part of the basilar artery.

It accompanies the facial and vestibulocochlear nerves into the internal auditory meatus. The posterior cerebral artery is often larger than the superior cerebellar artery and it is separated from the vessel, near its origin, by the oculomotor nerve. Arterial decompression is a well established therapy for trigeminal neuralgia.

941. Which of the following muscles does not receive any innervation from the sciatic nerve? A. Semimembranosus B. Quadriceps femoris C. Biceps femoris D. Semitendinosus E. Adductor magnus

ANSWER IS B The sciatic nerve is traditionally viewed as being a nerve of the posterior compartment. It is known to contribute to the innervation of adductor magnus (although the main innervation to this muscle is from the obturator nerve). The quadriceps femoris is nearly always innervated by the femoral nerve.

942. A 23 year old man is involved in a fight and is stabbed in his upper arm. The ulnar nerve is transected. Which of the following muscles will not demonstrate compromised function as a result? A. Flexor carpi ulnaris B. Medial half of flexor digitorum profundus C. Palmaris brevis D. Hypothenar muscles E. Pronator teres

Prepared by Dr: Mohammed Musa Brema Idress – My best wishes Page 429

ANSWER IS E M edial lumbricals A dductor pollicis F lexor digitorum profundus/Flexor digiti minimi I nterossei A bductor digiti minimi and opponens

Innervates all intrinsic muscles of the hand (EXCEPT 2: thenar muscles & first two lumbricals - supplied by median nerve)

Pronator teres is innervated by the median nerve. Palmaris brevis is innervated by the ulnar nerve

943. Which of the structures listed below overlies the cephalic vein? A. Extensor retinaculum B. Bicipital aponeurosis C. Biceps muscle D. Antebrachial fascia E. None of the above

ANSWER IS E The cephalic vein is superficially located in the upper limb and overlies most the fascial planes. It pierces the coracoid membrane (continuation of the clavipectoral fascia) to terminate in the axillary vein. It lies anterolaterally to biceps.

944. Which of the following pairings are incorrect? A. Aortic bifurcation and L4 B. Transpyloric plane and L1 C. Termination of dural sac and L4 D. Oesophageal passage through diaphragm and T10 E. Transition between pharynx and oesophagus at C6

ANSWER IS C Vena cava T8 (eight letters) Oesophagus T10 (ten letters) Aortic hiatus T12 (twelve letters) It terminates at S2, which is why it is safe to undertake an LP at L4/5 levels. The spinal cord itself terminates at L1.

945. A 22 year old man is involved in a fight. He sustains a laceration to the posterior aspect of his wrist. In the emergency department the wound is explored and the laceration is found to be transversely orientated and overlies the region of the extensor retinaculum, which is intact. Which of the following structures is least likely to be injured in this scenario? A. Dorsal cutaneous branch of the ulnar nerve B. Tendon of extensor indicis C. Basilic vein

Prepared by Dr: Mohammed Musa Brema Idress – My best wishes Page 430

D. Superficial branch of the radial nerve E. Cephalic vein

ANSWER IS B The extensor retinaculum attaches to the radius proximal to the styloid, thereafter it runs obliquely and distally to wind around the ulnar styloid (but does not attach to it). The extensor tendons lie deep to the extensor retinaculum and would therefore be less susceptible to injury than the superficial structures.

946. Which of the following is not a content of the porta hepatis? A. Portal vein B. Hepatic artery C. Cystic duct D. Lymph nodes E. None of the above

ANSWER IS C The cystic duct lies outside the porta hepatis and is an important landmark in laparoscopic cholecystectomy. The structures in the porta hepatis are: 1. Portal vein 2. Hepatic artery 3. Common hepatic duct These structures divide immediately after or within the porta hepatis to supply the functional left and right lobes of the liver. The porta hepatis is also surrounded by lymph nodes, that may enlarge to produce obstructive jaundice and parasympathetic nervous fibers that travel along vessels to enter the liver.

947. Which of the following structures is not closely related to the carotid sheath? A. Sternothyroid muscle B. Sternohyoid muscle C. Hypoglossal nerve D. Superior belly of omohyoid muscle E. Anterior belly of digastric muscle

ANSWER IS E At its lower end the carotid sheath is related to sternohyoid and sternothyroid. Opposite the cricoid cartilage the sheath is crossed by the superior belly of omohyoid. Above this level the sheath is covered by the sternocleidomastoid muscle. Above the level of the hyoid the vessels pass deep to the posterior belly of digastric and stylohyoid. Opposite the hyoid bone the sheath is crossed obliquely by the hypoglossal nerve

948. A 21 year old develops tonsillitis. He is in considerable pain. Which of the following nerves is responsible for the sensory innervation of the tonsillar fossa? A. Facial nerve B. Trigeminal nerve

Prepared by Dr: Mohammed Musa Brema Idress – My best wishes Page 431

C. Glossopharyngeal nerve D. Hypoglossal nerve E. Vagus

ANSWER IS C The glossopharyngeal nerve is the main sensory nerve for the tonsillar fossa. A lesser contribution is made by the lesser palatine nerve. Because of this otalgia may occur following tonsillectomy.

949. A man has an incision sited that runs 8cm from the deltopectoral groove to the midline. Which of the following is not at risk of injury? A. Cephalic vein B. Shoulder joint capsule C. Axillary artery D. Pectoralis major E. Trunk of the brachial plexus

ANSWER IS B This region will typically lie medial to the joint capsule.

950. A surgeon is due to perform a laparotomy for perforated duodenal ulcer. An upper midline incision is to be performed. Which of the following structures is the incision most likely to divide? A. Rectus abdominis muscle B. External oblique muscle C. Linea alba D. Internal oblique muscle E. None of the above

ANSWER IS C Upper midline abdominal incisions will involve the division of the linea alba. Division of muscles will not usually improve access in this approach and they would not be routinely encountered during this incision.

951. A 59 year old man is undergoing an extended right hemicolectomy for a carcinoma of the splenic flexure of the colon. The surgeons divide the middle colic vein close to its origin. Into which of the following structures does this vessel primarily drain? A. Superior mesenteric vein B. Portal vein C. Inferior mesenteric vein D. Inferior vena cava E. Ileocolic vein

ANSWER IS A The middle colonic vein drains into the SMV, if avulsed during mobilization then dramatic haemorrhage can occur and be difficult to control.

Prepared by Dr: Mohammed Musa Brema Idress – My best wishes Page 432

952. A 23 year old man is stabbed in the chest approximately 10cm below the right nipple. In the emergency department a abdominal ultrasound scan shows a large amount of intraperitoneal blood. Which of the following statements relating to the likely site of injury is untrue? A. Part of its posterior surface is devoid of peritoneum. B. The quadrate lobe is contained within the functional right lobe. C. Its nerve supply is from the coeliac plexus. D. The hepatic flexure of the colon lies posterio-inferiorly. E. The right kidney is closely related posteriorly

ANSWER IS B The right lobe of the liver is the most likely site of injury. Therefore the answer is B as the quadrate lobe is functionally part of the left lobe of the liver. The liver is largely covered in peritoneum. Posteriorly there is an area devoid of peritoneum (the bare area of the liver). The right lobe of the liver has the largest bare area (and is larger than the left lobe)

953. A 22 year old man is involved in a fight and sustains a skull fracture with an injury to the middle meningeal artery. A craniotomy is performed, and with considerable difficulty the haemorrhage from the middle meningeal artery is controlled by ligating it close to its origin. What is the most likely sensory impairment that the patient may notice post operatively? A. Parasthesia of the ipsilateral external ear B. Loss of taste sensation from the anterior two thirds of the tongue C. Parasthesia overlying the angle of the jaw D. Loss of sensation from the ipsilateral side of the tongue E. Loss of taste from the posterior two thirds of the tongue

ANSWER SI A The auriculotemporal nerve is closely related to the middle meningeal artery and may be damaged in this scenario. The nerve supplied sensation to the external ear and outermost part of the tympanic membrane. The angle of the jaw is innervated by C2,3 roots and would not be affected. The tongue is supplied by the glossopharyngeal nerve.

954. A 72 year old man presents with haemoptysis and undergoes a bronchoscopy. The carina is noted to be widened. At which level does the trachea bifurcate? A. T3 B. T5 C. T7 D. T2 E. T8

ANSWER IS B The trachea bifurcates at the level of the fifth thoracic vertebra. Or the sixth in tall subjects.

Prepared by Dr: Mohammed Musa Brema Idress – My best wishes Page 433

955. A 23 year old man is injured during a game of rugby. He suffers a fracture of the distal third of his clavicle, it is a compound fracture and there is evidence of arterial haemorrhage. Which of the following vessels is most likely to be encountered first during subsequent surgical exploration? A. Posterior circumflex humeral artery B. Axillary artery C. Thoracoacromial artery D. Sub scapular artery E. Lateral thoracic artery

ANSWER IS C The thoracoacromial artery arises from the second part of the axillary artery. It is a short, wide trunk, which pierces the clavipectoral fascia, and ends, deep to pectoralis major by dividing into four branches.

956. The following are true of the femoral nerve, except: A. It is derived from L2, L3 and L4 nerve roots B. It supplies sartorius C. It supplies quadriceps femoris D. It gives cutaneous innervations via the saphenous nerve E. It supplies adductor longus

ANSWER IS E Adductor longus is supplied by the obturator nerve.

957. Where is the vomiting centre located? A. Medulla oblongata B. Substantia nigra C. Antrum of stomach D. Pons E. Midbrain

ANSWER IS A ABC's of Non- GI causes of vomiting:- 1. Acute renal failure 2. Brain (Increased ICP) 3. Cardiac (Inferior MI) 4. DKA 5. Ears (labyrinthitis) 6. Foreign substances (Tylenol, theo, etc) 7. Glaucoma 8. Hyperemesis Gravidarum 9. Infections (pyelonephritis, meningitis)

958. Which of the following nerves conveys sensory information from the laryngeal mucosa? A. Glossopharyngeal

Prepared by Dr: Mohammed Musa Brema Idress – My best wishes Page 434

B. Laryngeal branches of the vagus C. Ansa cervicalis D. Laryngeal branches of the trigeminal E. None of the above

ANSWER IS B The laryngeal branches of the vagus supply sensory information from the larynx.

959. Which of the following nerves passes through the greater sciatic foramen and innervates the perineum? A. Pudendal B. Sciatic C. Superior gluteal D. Inferior gluteal E. Posterior cutaneous nerve of the thigh

ANSWER IS A 3 divisions of the pudendal nerve: 1. Rectal nerve 2. Perineal nerve 3. Dorsal nerve of penis/ clitoris All these pass through the greater sciatic foramen. The pudendal nerve innervates the perineum. It passes between piriformis and coccygeus medial to the sciatic nerve.

960. Which of the following is true in relation to the sartorius muscle? A. Innervated by the deep branch of the femoral nerve B. Inserts at the fibula C. It is the shortest muscle in the body D. Forms the Pes anserinus with Gracilis and semitendinous muscle E. Causes extension of the knee

ANSWER IS D It is innervated by the superficial branch of the femoral nerve. It is a component of the pes anserinus.

961. A 38 year old man falls onto an outstretched hand. Following the accident he is examined in the emergency department. On palpating his anatomical snuffbox there is tenderness noted in the base. What is the most likely injury in this scenario? A. Rupture of the tendon of flexor pollicis B. Scaphoid fracture C. Distal radius fracture D. Rupture of flexor carpi ulnaris tendon E. None of the above

Prepared by Dr: Mohammed Musa Brema Idress – My best wishes Page 435

ANSWER IS B A fall onto an outstretched hand is a common mechanism of injury for a scaphoid fracture. This should be suspected clinically if there is tenderness in the base of the anatomical snuffbox. A tendon rupture would not result in bony tenderness.

962. A 25 year old man sustains a severe middle cranial fossa basal skull fracture. Once he has recovered it is noticed that he has impaired tear secretion. This is most likely to be the result of damage to which of the following? A. Stellate ganglion B. Ciliary ganglion C. Otic ganglion D. Trigeminal nerve E. Greater petrosal nerve

ANSWER IS E The greater petrosal nerve may be injured and carries fibers for lacrimation

963. Which of the following structures passes through the quadrangular space near the humeral head? A. Axillary artery B. Radial nerve C. Axillary nerve D. Median nerve E. Transverse scapular artery

ANSWER IS C The quadrangular space is bordered by the humerus laterally, subscapularis superiorly, teres major inferiorly and the long head of triceps medially. It lies lateral to the triangular space. It transmits the axillary nerve and posterior circumflex humeral artery.

964. Which of the following pairings of foramina and their contents is not correct? A. Superior orbital fissure and the oculomotor nerve B. Foramina rotundum and the maxillary nerve C. Jugular foramen and the hypoglossal nerve D. Foramina spinosum and the middle meningeal artery E. Carotid canal and the internal carotid artery

ANSWER IS C The hypoglossal nerve passes through the hypoglossal canal

965. A 55 year old man with carcinoma of the larynx is undergoing a difficult laryngectomy. The surgeons divide the thyrocervical trunk, from which of the following vessels does this structure most commonly originate? A. Subclavian artery B. Common carotid artery C. Vertebral artery

Prepared by Dr: Mohammed Musa Brema Idress – My best wishes Page 436

D. External carotid artery E. Internal carotid artery

ANSWER IS A The thyrocervical trunk is a branch of the subclavian artery. It arises from the first part between the subclavian artery and the inner border of scalenus anterior. It branches off the subclavian distal to the vertebral artery.

966. What is the first branch of the axillary artery? A. Subscapular artery B. Lateral thoracic artery C. Thoracoacromial artery D. Superior thoracic artery E. Anterior circumflex humeral artery

ANSWER IS D - The superior thoracic artery is the first branch of the axillary artery arises from the first part - Two branches arise from the second part, thoracoacromial and lateral thoracic - Three branches from the third part, subscapular artery, anterior and posterior circumflex humeral arteries

967. The following structures are closely related to the brachiocephalic artery except: A. Trachea posteriorly B. Right brachiocephalic vein C. Inferior thyroid vein D. Right recurrent laryngeal nerve E. None of the above

ANSWER IS D There is no brachiocephalic artery on the left, however the left brachiocephalic vein lies anteriorly to the roots of all the 3 great arteries (including the brachiocephalic artery). The right recurrent laryngeal nerve has no relation to the brachiocephalic artery.

968. Which of the following structures separates the ulnar artery from the median nerve? A. Brachioradialis B. Pronator teres C. Tendon of biceps brachii D. Flexor carpi ulnaris E. Brachialis

ANSWER IS B It lies deep to pronator teres and this separates it from the median nerve.

Prepared by Dr: Mohammed Musa Brema Idress – My best wishes Page 437

969. Which muscle is supplied by the superficial peroneal nerve? A. Peroneus tertius B. Sartorius C. Adductor magnus D. Peroneus brevis E. Gracilis

ANSWER IS D Superficial peroneal nerve:- - Supplies: Lateral compartment of leg: peroneus longus, peroneus brevis (action: eversion and plantar flexion) - Sensation over dorsum of the foot (except the first web space, which is innervated by the deep peroneal nerve) Path ❖ Passes between peroneus longus and peroneus brevis along the length of the proximal one third of the fibula. ❖ 10-12 cm above the tip of the lateral malleolus, the superficial peroneal nerve pierces the fascia. ❖ 6-7 cm distal to the fibula, the superficial peroneal nerve bifurcates into intermediate and medial dorsal cutaneous nerves

970. A 32 year old motorcyclist is involved in a road traffic accident. His humerus is fractured and severely displaced. At the time of surgical repair the surgeon notes that the radial nerve has been injured. Which of the following muscles is least likely to be affected by an injury at this site? A. Extensor carpi radialis brevis B. Brachioradialis C. Abductor pollicis longus D. Extensor pollicis brevis E. None of the above

ANSWER IS E Muscles supplied by the radial nerve = BEST 1. Brachioradialis 2. Extensors 3. Supinator 4. Triceps The radial nerve supplies the extensor muscles, abductor pollicis longus and extensor pollicis brevis (the latter two being innervated by the posterior interosseous branch of the radial nerve).

971. A man develops an infection in his external auditory meatus. The infection is extremely painful. Which of the following nerves conveys sensation from this region? A. Occipital branch of the trigeminal nerve B. Vestibulocochlear nerve C. Facial nerve

Prepared by Dr: Mohammed Musa Brema Idress – My best wishes Page 438

D. Auriculotemporal nerve E. Maxillary branch of the trigeminal nerve

ANSWER IS D Tensor tympania and stapedius are the only two muscles of the middle ear. Contraction of tensor tympani will tend to dampen the vibrations produced by loud sounds, it is innervated by a branch of the trigeminal nerve. The stapedius dampens movements of the ossicles in response to loud sounds and is innervated by a branch of the facial nerve. The auriculotemporal nerve, which is derived from the mandibular branch of the trigeminal nerve supplies this area.

972. Which muscle is responsible for causing flexion of the interphalangeal joint of the thumb? A. Flexor pollicis longus B. Flexor pollicis brevis C. Flexor digitorum superficialis D. Flexor digitorum profundus E. Adductor pollicis

ANSWER IS A There are 8 muscles: 1. Two flexors (flexor pollicis brevis and flexor pollicis longus) 2. Two extensors (extensor pollicis brevis and longus) 3. Two abductors (abductor pollicis brevis and longus) 4. One adductor (adductor pollicis) 5. One muscle that opposes the thumb by rotating the CMC joint (opponens pollicis). Flexor and extensor longus insert on the distal phalanx moving both the MCP and IP joints.

973. Which of the following structures separates the posterior cruciate ligament from the popliteal artery? A. Oblique popliteal ligament B. Transverse ligament C. Popliteus tendon D. Biceps femoris E. Semitendinosus

ANSWER IS A The posterior cruciate ligament is separated from the popliteal vessels at its origin by the oblique popliteal ligament. The transverse ligament is located anteriorly.

974. How many compartments are there in the lower leg? A. 2 B. 1 C. 3

Prepared by Dr: Mohammed Musa Brema Idress – My best wishes Page 439

D. 5 E. 4

ANSWER IS E The posterior compartment of the lower leg has both superficial and deep posterior layers, together with the anterior and lateral compartments this allows for four compartments. Decompression of the deep posterior compartment during fasciotomy may be overlooked with significant sequelae.

975. Which structure is least likely to be found at the level of the sternal angle? A. Left brachiocephalic vein B. Intervertebral discs T4-T5 C. Start of aortic arch D. 2nd pair of costal cartilages E. Bifurcation of the trachea into left and right bronchi

ANSWER IS A The left brachiocephalic vein lies posterior to the manubrium, at the level of its upper border. The sternal angle refers to the transition between manubrium and sternum and therefore will not include the left brachiocephalic vein.

976. A 53 year old man is undergoing a left hemicolectomy for carcinoma of the descending colon. From which embryological structure is this region of the gastrointestinal tract derived? A. Vitellino-intestinal duct B. Hind gut C. Mid gut D. Fore gut E. Woolffian duct

ANSWER IS B The left colon is embryologically part of the hind gut. Which accounts for its separate blood supply via the IMA.

977. Which of the following statements concerning the anatomy of the spinal cord is most accurate? Select one answer only. A. Afferent sensory neurones have their cell bodies in the anterior (ventral) root ganglion B. In a transverse section the white matter is seen as an H-shaped area C. Motor neurons leave via the anterior (ventral) root D. The main blood supply to the spinal cord is derived from paired anterior spinal arteries and a posterior spinal artery E. The spinal cord ends at the lower border of the second lumbar vertebra

ANSWER IS C

Prepared by Dr: Mohammed Musa Brema Idress – My best wishes Page 440

The spinal cord extends from the foramen magnum to the lower border of the first lumbar vertebra. The cord is divided into segments. At each segment a pair of spinal nerves is given off. The anterior (ventral) root of the spinal nerve carries motor neurons. The posterior (dorsal) root carries sensory neurons, which have their cells of origin in the dorsal root ganglion.

In cross-section the grey matter forms a central H-shape and is surrounded by white matter. The white matter is divided in to dorsal, lateral and ventral columns. The spinal cord is supplied by the anterior and right and left posterior spinal arteries. This is supplemented by various anastomoses with radicular arteries.

978. Entrapment syndrome could affect the deepest content in the popliteal fossa causing calf cramps and toes tingling. Which of the following structures is the deepest content of the popliteal fossa? A. Genicular branch of the obturator nerve B. Long saphenous vein C. Posterior tibial artery D. Popliteal artery E. Tibial nerve.

ANSWER IS D The popliteal fossa is a diamond-shaped space at the back of the knee. The superomedial border is formed by the semitendinous and semimembranous muscles and the superolateral border by biceps femoris muscle. The inferior borders are formed by the medial and lateral heads of gastrocnemius. The contents include the popliteal artery, politeal vein, tibial nerve, common peroneal nerve, small saphenous vein, posterior cutaneous nerve of the thigh and the genicular branch of the obturator nerve. There are also lymph nodes and connective tissue.

979. The axillary artery is divided into three parts by which muscle? Single best answer question – choose ONE true option only. A. Teres major B. Teres minor C. Pectoralis major D. Pectoralis minor E. Latissimus dorsi

ANSWER IS D The axillary artery is the continuation of the subclavian artery starting from the outer part of the 1st rib to the lower border of teres major. Along with the axillary vein and the brachial plexus it is enclosed in the axillary sheath. The vein is medial to the artery and the cords of the brachial plexus lie around it. Pectoralis major covers most of the artery except for its lateral edge. The branches are; First part: superior thoracic artery Second part: acromiothoracic and lateral thoracic arteries Third Part: Subscapular, anterior circumflex humeral and posterior circumflex humeral arteries.

Prepared by Dr: Mohammed Musa Brema Idress – My best wishes Page 441

980. A 25-year-old woman attends the Coloproctology clinic with a 4-month history of intermittent, painful, fresh PR bleeding. She is found to have anal spasm and a chronic posterior anal fissure. What will be the most appropriate first line management? Single best answer - select one answer only. A. Advancement flap B. Examination under anaesthesia and Botulinum A toxin C. Flexible sigmoidoscopy and high fibre diet D. Lateral sphincterotomy E. 2% Diltiazem ointment

ANSWER IS E Anterior anal fissures are commoner in women due to the lack of support to the anal canal anteriorly. Recent studies have demonstrated good results with topical Diltiazem. Other options include lateral sphincterotomy, Botulinum toxin and advancement flap. The latter is considered for low pressure fissures.

The accepted treatment protocol for anal fissures is firstly topical 0.2% GTN or 2% Diltiazem followed by BOTOX and then lateral sphincterotomy or advancement flap. The latter is considered in women with low pressure anal fissure.

981. A 25-year-old female presents with pain in the right hand which is associated with a tingling sensation and weakness. After further investigation she is diagnosed with thoracic outlet syndrome. The affected neurovascular bundle runs between scalenus anterior and medius. Where do these muscles insert? A. Body of the sternum B. Clavicle C. First rib D. Manubrium E. Second rib.

ANSWER IS C Thoracic outlet syndrome is caused by compression of the neurovascular bundle that passes between the anterior and middle scalene muscles. The brachial plexus and the subclavian artery and (rarely) the subclavian vein can be affected, leading to a variety of upper limb, neck and upper back symptoms. Cervical ribs, Pancoast‟s tumour and pregnancy can all cause the condition, but it is usually secondary to trauma or repetitive strain injuries.

The scalene muscle is composed of three pairs of constituent muscles, namely scalenus anterior (originating at anterior tubercles of the transverse processes of C3- 6 vertebrae), scalenus medius (originating from the posterior tubercles of C2-6 vertebrae) and scalenus posterior (originating from the posterior tubercles of the transverse processes of C4-6). The anterior and medius muscles insert into the first rib, the posterior component inserts into the second.

Prepared by Dr: Mohammed Musa Brema Idress – My best wishes Page 442

982. Of the following intrinsic muscles of the larynx, which tenses (stretches) the vocal folds? Single best answer question – choose ONE true option only. A. Posterior cricoarytenoid B. Lateral cricoarytenoid C. Thyroarytenoid D. Transverse arytenoid E. Cricothyroid muscle

ANSWER IS E The cricothyroid, triangular in form, arises from the front and lateral part of the cricoid cartilage; its fibers diverge and are arranged in two groups. The lower fibers constitute a pars obliqua and slant backward and lateral ward to the anterior border of the inferior cornu.

The anterior fibers, forming pars recta, run upward, backward and lateral ward to the posterior part of the lower border of the lamina of the thyroid cartilage. The external laryngeal branch of the superior laryngeal nerve supplies the cricothyroid. The cricothyroids produce tension and elongation of the vocal folds by drawing up the arch of the cricoid cartilage and tilting back the upper border of its lamina. The distance between the vocal processes and the angle of the thyroid is so increased and the folds are consequently elongated.

983. A 62-year-old ex-smoker has been diagnosed with non-small cell lung carcinoma by endobronchial biopsy. At standard mediastinoscopy, what is the least likely lymph node station to be sampled? Single best answer question – choose ONE true option only. A. Paratracheal nodes B. Subcarinal nodes C. Tracheobronchial nodes D. Aortopulmonary nodes E. Pretracheal nodes

ANSWER IS D During mediastinoscopy, the pretracheal fascia is incised and blunt dissection creates a tunnel inferiorly. The linear passage of the mediastinoscope along this tunnel allows visualization of the nodes lying to the front (pretracheal) and sides (paratracheal) of the trachea. The subcarinal and tracheobronchial nodes can also be reached at the distal end of this tunnel. The aortopulmonary nodes, however, cannot easily be reached as the aorta is “in the way” of the advancing finger or mediastinoscope.

984. A 34-year-old man has been admitted following a stab wound to the right upper quadrant. During laparotomy profuse hepatic bleeding is encountered and your consultant performs Pringles manoeuvre which is successful and then packs the abdomen. Which of the following is correct concerning the lesser sac and its related anatomy? Single best answer - select one answer only. A. Its left lateral border is formed by the falciform ligament

Prepared by Dr: Mohammed Musa Brema Idress – My best wishes Page 443

B. Its anterior relation is the posterior aspect of the stomach C. Its posterior relations include the spleen D. The aditus to the lesser sac is bounded anteriorly by the left free margin of the lesser omentum containing the portal vein, hepatic artery and bile duct E. The posterior wall is formed partly by the lesser omentum.

ANSWER IS B The lesser sac is a peritoneal cavity located behind the stomach. It may be entered by passing a finger into the epiploic foramen, immediately behind the portal vein in the free edge of the lesser omentum.

The transverse mesocolon and pancreas form the posterior wall. The gastrosplenic and lienorenal ligaments form its left lateral border. The falciform ligament arises from the anterior abdominal wall and demarcates the left and the right lobe of the liver.

Pringles manoeuvre involves compressing the structures in the free edge of the lesser omentum anterior to the aditus to the lesser sac. This is the right free margin rather than the left.

985. The following structure passes under the brachioradialis emerging distally on its medial side. Distally it is covered only by skin and fascia and lies on the radius? Single best answer question – choose ONE true option only. A. Ulnar artery B. Radial artery C. Median nerve D. Profunda brachii artery E. Radial nerve.

ANSWER IS B This describes the course of the radial artery which starts at the level of the neck of the radius lying on the tendon of the biceps. It passes under the brachioradialis as described reaching the wrist where its pulsations are felt against the radius. After that it winds laterally and enters the palm between the heads of the first dorsal interosseous muscle, it ends as the deep palmar arch supplying the hand.

The ulnar artery on the other hand passes inferiorly and medially in the anterior compartment of the forearm. It runs laterally to the ulnar nerve deep to the flexor carpi ulnaris and ends in the hand forming the superficial palmar arch.

986. Club foot deformity (Congential Talipes Equino Varus) is the most common birth defect. Which of the following is responsible for the hind foot equinus deformity? A. Achilles tendon B. Peroneus brevis C. Peroneus tertius D. Tibialis anterior

Prepared by Dr: Mohammed Musa Brema Idress – My best wishes Page 444

E. Tibialis posterior.

ANSWER IS A Peroneus tertius dorsiflexes and everts the foot. The extensor digitorum longus and brevis, extensor hallucis longus and tibialis anterior muscles are also in the anterior compartment of the leg. Tibialis posterior plantar flexes and inverts the foot. Soleus is also in the posterior compartment of the leg and plantar flexes the foot at the ankle joint. Peroneus longus and brevis are in the lateral compartment of the leg. They both plantar flex and evert the foot.

987. A 34-year-old woman is seen in the orthopaedic outpatients department. She is a keen jogger and has noticed numbness in the first, second and third toes as well as a dull ache. Which nerve is most likely to have been affected? A. The deep peroneal nerve B. The lateral cutaneous nerve of the thigh C. The lateral plantar nerve D. The medial plantar nerve E. The sural nerve.

ANSWER IS D The medial plantar nerve is a branch of the posterior tibial nerve (L4 – S3; a branch of the sciatic nerve) and supplies the medial three and a half toes on the plantar surfaces and the dorsal surfaces proximal to the nail beds. Excessive stretching of the medial plantar nerve can result in tarsal tunnel syndrome. Also note that the medial branch of the superficial peroneal nerve partly supplies the skin of the medial big toe and adjacent parts of the second and third toes.

988. Which one of the following muscles is attached to the tibial tuberosity? Single best answer question – choose ONE true option only. A. Pectineus B. Vastus intermedius C. Tensor fascia lata D. Short head of the biceps femoris E. Adductor brevis

ANSWER IS B The tuberosity of the tibia gives attachment to the ligamentum patellae (which is the single strong tendon of the quadriceps femoris, including rectus femoris, vasti medialis, intermedius and lateralis). A bursa intervenes between the deep surface of the ligament and the part of the bone immediately above the tuberosity.

989. Which of the following structures passes through the canal of Guyon? Single best answer question – choose ONE true option only. A. Ulnar nerve B. Median nerve C. Sciatic nerve D. Accessory nerve

Prepared by Dr: Mohammed Musa Brema Idress – My best wishes Page 445

E. Sural nerve.

ANSWER IS A The canal of Guyon is a fibrous band of fascia covering both the ulnar artery and nerve at the level of the wrist. Both the artery and nerve pass in front of the retinaculum lateral to the pisiform bone. The ulnar artery lies lateral to the nerve at this level. The ulnar nerve might be occasionally compressed at the canal of Guyon.

990. The psoas major muscle: Single best answer question – choose ONE true option only. A. Flexes the thigh at the hip joint B. Extends the thigh at the hip joint C. Adducts the thigh at the hip joint D. Abducts the thigh at the hip joint E. Assists in the full contraction of the diaphragm.

ANSWER IS A The psoas major muscle joins the iliacus muscle which originates broadly over the inner aspect of the iliac wing of the pelvis. This becomes the iliopsoas muscle and inserts on the lesser trochanter of the femur and thus flexes the thigh at the hip joint.

991. Following a difficult tracheostomy, the surgeon is concerned about the nerve supply to the trachea being damaged. Which of the following nerves supply the trachea? Select one answer only. A. Phrenic nerve B. Glossopharyngeal nerve C. Recurrent laryngeal nerve D. Superior laryngeal nerve E. External laryngeal nerve.

ANSWER IS C The trachea attaches to the larynx via the cricotracheal membrane at about the level of the sixth cervical vertebrae, and descends through the neck and superior mediastinum to terminate at about the level of the disc between the fourth and fifth thoracic vertebrae, although this bifurcation descends as the trachea is stretched during inspiration.

The tracheal rings consist of hyaline cartilage. It is supplied by the recurrent laryngeal nerve which is sensory below the level of the vocal cords and motor to all muscles of the larynx with the exception of the cricothyroid, which is supplied by superior laryneal nerve.

992. You are performing a parotidectomy. Which of the following lie within the gland you are excising? A. External carotid artery B. Masseter

Prepared by Dr: Mohammed Musa Brema Idress – My best wishes Page 446

C. Maxillary artery D. Superficial temporal artery E. Trigeminal nerve

ANSWER IS A The parotid gland lies deep to masseter, medial pterygoid, stylomandibular ligament, superficial temporal artery, maxillary artery and facial nerve. Within the gland lie the facial nerve, retromandibular vein, external carotid artery and auriculotemporal nerve.

993. A 72-year-old farmer presents with a lesion on his left cheek which has been present for several months. On examination, it has nodular appearance with rolled edges and visible telangiectasia. Clinically it is thought to be a basal cell carcinoma (BCC) and the decision is made to proceed with treatment. What proportion of BCCs occur in the head and neck region? Select one answer only. A. 1% B. 10% C. 30% D. 50% E. 90%

ANSWER IS E BCC is the commonest skin tumour on the face and 90% of all BCCs occur in the head and neck region. Sunlight exposure and genetic factors are the main risk factors.

BCCs rarely, if ever, metastasize and treatment is with complete surgical excision. Radiotherapy is reserved for recurrences, which are typically aggressive if they recur on the cheeks, nasolabial folds, medial canso and preauricular region.

The phrase „rather a large scar than a small tomb‟ (Sir Harold Gillies) should always be taken into consideration when planning surgical excision – tumours under 1 cm should have at least a 5 mm excision margin and those over 1 cm should have at least a 1 cm margin.

994. A 76-year-old male who is a long term smoker presents with a headache and facial swelling. On further questioning he reports episodes of haemoptysis and 11/2 stone in weight loss. Examination reveals a plethoric face and neck, and distended veins on his upper chest and neck. A CT scan reveals a bronchial tumour causing SVC obstruction. At what level does the SVC enter the right atrium? Select one answer only. A. At the level the azygos vein drains into it B. Behind the first costal cartilage C. Behind the third costal cartilage D. Level of sternal angle E. Vertebral level T8.

Prepared by Dr: Mohammed Musa Brema Idress – My best wishes Page 447

ANSWER IS C The SVC drains all the structures above the diaphragm except the heart and lungs. It also receives the azygos vein, which drains the lumbar and subcostal regions. The SVC is formed behind the first costal cartilage by the union of the right and left brachiocephalic veins. It ends behind the third costal cartilage as it enters the right atrium. The SVC has no valves. The thoracic duct drains into the left brachiocephalic vein (or sometimes into the subclavian or internal jugular vein).

995. A man undergoes a laparoscopy at the beginning of surgery for cholecystectomy. He is found to have an incidental patent processus vaginalis. Which of the following is he predisposed to? Single best answer - select one answer only. A. Direct inguinal hernia B. Indirect inguinal hernia C. Recurrent prostatitis D. Testicular torsion E. Undescended testis

ANSWER IS B The processus vaginalis is a parietal peritoneal sac which passes through the internal ring of the inguinal canal in the fetus, but which is normally obliterated after birth except for a small part that becomes the tunica vaginalis of the testis. The testis descends through the canal as a retroperitoneal structure and is therefore outside and behind the processus vaginalis. In cases of a persistent processus vaginalis, indirect inguinal hernias can ensue.

996. A 23 year old presented with a humeral fracture on the left side affecting the area of the spiral groove. The nerve most likely to be affected by this injury is? Single best answer question – choose ONE true option only. A. Ulnar nerve B. Radial nerve C. Axillary nerve D. Median nerve E. Posterior interosseous nerve.

ANSWER IS B The radial nerve runs in the spiral groove and injury at that level will affect the extensors of the wrist and fingers resulting in „wrist drop‟. Sensory loss will be localized to the back of the radial side of the hand.

997. during a posterior approach to the right hip joint, you are asked to identify the gluteal muscles, you correctly identify the most superficial gluteal muscle which is innervated by the inferior gluteal nerve, what movements does this muscle produce? A. Hip Adduction and medial rotation B. Hip extension and knee flexion C. Hip Extension and lateral rotation

Prepared by Dr: Mohammed Musa Brema Idress – My best wishes Page 448

D. Hip Flexion alone E. Hip flexion and knee extension.

ANSWER IS C Gluteus maximus is supplied by the inferior gluteal nerve, and extends and laterally rotates the hip. The superior and inferior gluteal arteries supply gluteus maximus and are branches off the internal iliac artery. The gluteus minimus and gluteus medius are supplied by the superior gluteal nerve and arteries.

998. the lumbosacral plexus is formed by the ventral rami of T12 – S3, and gives several peripheral nerve branches. Which branch originates from L2-L4? Select one answer only. A. The sciatic nerve B. The genitofemoral nerve C. The femoral nerve D. The superior gluteal nerve E. The inferior gluteal nerve

ANSWER IS C The sciatic nerve comprises nerve roots L4, 5, S1, 2, 3.

999. You are assisting your ENT consultant performing a thyroidectomy for malignancy. Regarding the surgical anatomy of the thyroid gland: A. The inferior parathyroid glands are more constant in position than the superior parathyroid glands B. The middle thyroid veins are more constant in position than the superior and inferior thyroid veins C. The thyroid gland does not have a definite capsule D. There is a ligament, Berry’s ligament connecting the thyroid to the cricoid cartilage and upper trachea E. Unilateral recurrent laryngeal nerve division results in the contralateral vocal cord lying in the mid- or cadaveric position.

ANSWER IS D The thyroid gland has a definite, fine capsule, which allows a capsular dissection to preserve the recurrent laryngeal nerves. The superior parathyroid glands are more constant in position than the inferior. Because of their embryological migration, the inferior glands may be situated among the pretracheal lymph nodes or in the thymus as far as 10 cm from the thyroid. The middle thyroid veins are the least constant of the thyroid veins.

The superior veins drain into the internal jugular vein; the inferior veins are very constant and drain into the brachiocephalic veins; and the middle veins are very variable and often multiple. Unilateral recurrent laryngeal nerve section results in the ipsilateral vocal cord lying motionless in the mid- or cadaveric position. The voice is hoarse and weak. If both recurrent laryngeal nerves are divided, then the glottic space is narrowed and stridor develops.

Prepared by Dr: Mohammed Musa Brema Idress – My best wishes Page 449

1000. A 17-year-old girl is brought to A&E following a horse riding accident. She is very distressed and in a lot of pain. From the history the horse fell backwards and landed on the left side of her chest. On examination there is extensive bruising over the left chest and it can be seen to move in when she inspires. Auscultation reveals bilateral air entry and her trachea is central. What is the most likely diagnosis? Select one answer only. A. Chylothorax B. Flail chest C. Haemothorax D. Pneumothorax E. Tension pneumothorax.

ANSWER IS B A flail chest occurs when a segment of the thoracic wall does not have bony continuity with the rest of the thoracic cage. Flail chest classically results from blunt trauma, as in high-speed road-traffic accidents. A transfer of significant kinetic energy to the rib cage can cause fracture of the ribs in multiple areas leading to a segment of the thoracic wall to „float‟ independently of the rest of the chest wall. A flail chest can arise when two or more ribs are fractured in two or more places.

The presence of a flail chest segment results in severe disruption of normal chest wall movement leading to paradoxical breathing (when the injured segment of the thoracic cage moves in an opposite direction to the rest of the chest wall). Initial management of flail chest includes adequate ventilation, administration of humidified oxygen, adequate analgesia and fluid resuscitation. However, in the absence of systemic hypotension (from other associated causes), intravenous fluid resuscitation should be carefully monitored to prevent over hydration.

The injured lung in a flail chest is very sensitive to both under-resuscitation and fluid overload. The definitive management of flail chest is to re-expand the affected (contused or collapsed) lung, ensure adequate oxygenation, and provide sufficient pain relief and judicious fluid resuscitation. Since prevention of hypoxia is of paramount importance some patients will benefit from a short period of intubation and ventilation, the timing guided by the respiratory rate, arterial oxygen tension and other vital respiratory parameters. Surgical stabilization of the chest is an option but is rarely necessary in the management of flail chest.

1001. You are excising the right suprarenal gland and your consultant asks you about its anatomy. Which of the following descriptions is correct? Single best answer - select one correct answer. A. It is crescentic in shape B. It has a longer vein than the left suprarenal gland C. It lies anterior to the inferior vena cava D. It receives an arterial supply from the subcostal artery E. It touches the bare area of the liver

Prepared by Dr: Mohammed Musa Brema Idress – My best wishes Page 450

ANSWER IS E The left suprarenal vein is longer than the right, entering the left renal vein. Both glands receive an arterial supply directly from the aorta, as well as from the renal and inferior phrenic arteries. The right suprarenal gland is pyramidal in shape; the left is crescentic in shape. The anterior surface of the right suprarenal gland is overlapped medially by the inferior vena cava.

1002. A seven-year-old boy presents with a displaced supracondylar fracture of his right humerus. On examination there is no distal sensory deficit, but he is unable to flex his thumb and index finger to make the “OK” sign. Which nerve has been damaged? Single best answer question – choose ONE true option only. A. Median nerve B. Radial Nerve C. Posterior Interosseous Nerve D. Ulnar Nerve E. Anterior Interosseous Nerve

ANSWER IS E The anterior Interosseous Nerve is a branch of the median nerve. It does not have a sensory component and supplies both flexor pollicis longus and flexor digitorum profundus to the index and middle fingers. Both of these muscles are required to make an “OK” sign. It is the most commonly injured nerve in supracondylar fractures of the elbow.

1003. A 39-year-old female sustains a displaced intracapsular fracture of neck of femur following a fall from a horse. What is the most common complication of this injury? A. A vascular necrosis B. Chondrolysis C. Hip instability D. Non-union

ANSWER IS A Posteriorly the capsule is attached half-way along the neck. The intertrochanteric crest lies posteriorly. A fracture of the neck of the femur causes lateral rotation of the thigh. The blood supply to the head of the femur passes through the retinacula on the neck. The obturator externus winds round the inferior aspect of the neck. Nonunion rate is around 20%, whereas AVN is about 30%.

1004. Which of the following is false regarding the lumbosacral plexus? Select one answer only. A. The anterior divisions of anterior rami supply the adductor muscles B. The lumbosacral trunk lies on the piriformis muscle C. The lumbosacral trunk underlies the common iliac vessels D. The parasympathetic nerves originate from S2/3 spinal segments E. The sympathetic trunk sends grey rami to all roots.

Prepared by Dr: Mohammed Musa Brema Idress – My best wishes Page 451

ANSWER IS B Sympathetic grey rami connect with all spinal nerve roots. The splanchnic nerves also known as the Nervi erigentes originate from S2, S3 and often S4. The lumbosacral trunk (L4/5) passes anterior to the ala of the sacrum, under cover of the common iliac vessels, and joins the S1 ventral ramus above the piriformis muscle. The adductor muscles are supplied by the L2–L4 roots of the lumbar plexus. They develop by medial migration from the flexor (hamstring) compartment, and are therefore supplied by anterior divisions.

1005. A 24-year-old footballer presents with a swollen painful knee. On examination he is noted to have significant joint effusion and a positive Lachman test. Which component of the knee joint has been injured? A. Anterior cruciate ligament B. Lateral collateral ligament C. Medial collateral ligament D. Patellar tendon E. Posterior cruciate ligament.

ANSWER IS A The anterior cruciate ligament provides stability to the knee joint. High impact sports that involve rapid changes of direction or direct lower limb trauma are often associated with injuries to the ligament. It originates from the lateral femoral condyle to insert into the intercondyloidtibial eminence. Lachman‟s test requires the knee to be slightly flexed, and is positive when the tibia can be pulled abnormally anteriorly from a fixed femur.

1006. The first compartment under the extensor retinaculum of the wrist contains which of the following tendons? Single best answer question – choose ONE true option only. A. Extensor Carpi Radialis Longus and Extensor Carpi Radialis Brevis B. Extensor Pollicis Longus C. Extensor Pollicis Brevis and Abductor Pollicis Longus D. Extensor Carpi Ulnaris E. Extensor Digiti Minimi.

ANSWER IS C There are six extensor compartments in the extensor retinaculum on the dorsum of the wrist. They contain in order: 1st - Extensor Pollicis Brevis and Abductor Pollicis Longus 2nd - Extensor Carpi Radialis Longus and Extensor Carpi Radialis Brevis 3rd - Extensor Pollicis Longus 4th - Extensor Digitorum and Extensor Indicis 5th - Extensor Digiti Minimi 6th - Extensor Carpi Ulnaris. The second and third compartments are separated by Lister‟s Tubercle.

1007. You are excising the right suprarenal gland and you are having difficulty locating it. Which anatomical considerations of the right suprarenal gland would assist you? A. It drains into the right renal vein

Prepared by Dr: Mohammed Musa Brema Idress – My best wishes Page 452

B. It extends anterior to the inferior vena cava (IVC) C. It lies against the caudate lobe of the liver D. It lies on the ninth rib E. It receives blood from the right inferior phrenic artery

ANSWER IS E The bare area of the liver is in direct contact with the right suprarenal gland and the diaphragm. The right suprarenal gland extends medially behind the IVC, separated from the 12th rib by the diaphragm. It typically has three arterial supplies. It receives blood from the superior suprarenal artery (via the inferior phrenic artery), the middle suprarenal artery (directly via the abdominal aorta) and the inferior suprarenal artery (via the right renal artery). The venous drainage is into the IVC by a very short vessel. The left suprarenal gland drains into its corresponding renal vein.

1008. you have performed a liver biopsy, and shortly after the procedure the patient develops pain on the tip of his right shoulder. Which nerve is most likely to be responsible for his pain? Single best answer - choose ONE true option only. A. Right phrenic nerve B. Axillary nerve C. Right vagus D. Right sympatheticus E. Intercostobrachial nerve

ANSWER IS A The phrenic nerve on both sides originates from the ventral rami of the third to fifth cervical nerves. It passes inferiorly down the neck to the lateral border of the scalenus anterior, and then it passes medially across the border of scalenus anterior parallel to the internal jugular vein that lies inferomedially. The right phrenic nerve pierces the diaphragm in its tendinous portion just slightly lateral to the inferior vena caval foramen.

It then forms three branches on the inferior surface of the diaphragm: anterior, lateral and posterior. These ramify out in a radial manner from the point of perforation to supply all but the periphery of the muscle.

1009. There are three main foramina: the superior and inferior orbital fissures and the optic canal. Which of the following enter the orbit via the optic canal? A. Abducens nerve B. Lacrimal nerve C. Ophthalmic artery D. Superior and Inferior ophthalmic veins E. Trochlear nerve.

ANSWER IS C The optic nerve and ophthalmic artery pass through the optic canal. The optic nerve and retina are part of the central nervous system. The optic nerve is invested by all the meningeal layers.

Prepared by Dr: Mohammed Musa Brema Idress – My best wishes Page 453

Since the central retinal artery travels in the optic nerve after branching off the ophthalmic artery, damage to the optic nerve commonly causes retinal infarction. The other options in this question pass through the superior orbital fissure.

1010. A 15-year-old rugby player sustains a head injury during a match and collapses after initially playing on. He is taken to A+E where a CT scan demonstrates a large left extradural haematoma. At operation, damage to the middle meningeal artery is noted to be the cause. Which of the following is correct regarding this artery? Select one answer only. A. Arises from the superficial temporal branch of the external carotid artery B. Enters the skull through the foramen ovale C. Is unlikely to be damaged in fractures at the pterion D. Originates in the pterygopalatine fossa E. Supplies part of the dura mater and the calvaria

ANSWER IS E The middle meningeal artery divides from the maxillary branch of the external carotid, entering the cranium through the foramen spinosum. It is the largest artery that supplies the dura and also the calvaria. While the maxillary artery does give off branches in the pterygopalatine fossa region, the middle meningeal artery comes off more proximally, deep to the ramus of the mandible.

It is particularly prone to damage caused by temporal bone fractures. The anterior division runs beneath the pterion, which is the junction of the parietal, frontal and squamous temporal bones with the greater wing of the sphenoid, and is prone to damage from fractures here.

1011. You are required to insert a chest drain in a patient with penetrating trauma. Which anatomical landmark is least useful to you? Single best answer question – choose ONE true option only. A. Anterior border of latissimus dorsi B. The mid-clavicular line C. The sixth rib D. The mid-axillary line E. Inferolateral border of pectoralis major.

ANSWER IS B The “safe triangle” for the insertion of an intercostal drain is bounded anteriorly by the inferolateral border of pectoralis major, posteriorly by the anterior border of latissimus dorsi and inferiorly by the axial plane at the level of the nipple.

In practice, however, one must note that the position of the nipple is highly variable and so the drain is best sited within the 5th intercostal space (i.e. immediately above the 6th rib as located by palpation). The mid-axillary line may be used within this triangle to help guide placement of the incision. The mid-clavicular line should not be used for placement of an intercostal tube drain.

Prepared by Dr: Mohammed Musa Brema Idress – My best wishes Page 454

1012. A 50-year-old roofer attends the outpatient clinic with bilateral groin lumps which you decide are bilateral direct inguinal hernia. He asks your advice about the best repair. What would you recommend? Single best answer - select one answer only. A. Intraperitoneal repair B. Lichtenstein tension free repair C. Shouldice repair D. Totally extra-peritoneal repair E. Trans-abdominal peritoneal repair.

ANSWER IS D Inguinal hernia may be repaired under local anaesthetic or general anaesthetic. Local anaesthetic techniques include Lichtenstein mesh repair or Shouldice repair. A herniotomy would not be appropriate in a man but would be suitable for repair of inguinal hernia in children less than 12 years old.

Laparoscopic intraperitoneal repair is an acceptable treatment; however, the long term results are unknown. The Bassini repair has fallen out of favour. The current favoured method is totally extraperitoneal (TEP repair). The recurrence rates are equivalent to Lichtenstein repair but analgesic requirements are less, return to work is quicker and chronic pain is infrequent.

1013. A man is treated with a right sided intercostal chest drain for haemothorax after an RTC. He subsequently develops a winged scapula on the right side. Which nerve has been damaged? Single best answer question – choose ONE true option only. A. Dorsal Scapular Nerve B. Long Thoracic Nerve C. Suprascapular Nerve D. Musculocutaneous Nerve E. Thoracodorsal Nerve.

ANSWER IS B The Long Thoracic Nerve arises from the roots of C5, C6 and C7. The nerve passes just posterior to the mid-axillary line deep to the fascia of serratus anterior and supplies this muscle in a segmental fashion. Serratus anterior protracts the scapula in punching and pushing and keeps the vertebral border of the scapula in firm apposition with the chest wall. The nerve is potentially damaged by malpositioned chest drains.

1014. From lateral to medial what are the structures in the femoral triangle. Single best answer question – choose ONE true option only. A. Femoral vein, femoral artery, femoral nerve B. Femoral nerve, femoral artery, femoral vein C. Femoral nerve, femoral artery, femoral vein, long saphenous vein D. Long saphenous vein, femoral vein, femoral artery, femoral nerve E. Short saphenous vein, femoral vein, femoral artery, femoral nerve.

Prepared by Dr: Mohammed Musa Brema Idress – My best wishes Page 455

ANSWER IS C From lateral to medial, the femoral triangle contains the femoral nerve and its branches, the femoral artery and its branches, including the profunda femoris and the femoral vein with its main tributary the long saphenous vein. The short saphenous vein enters the popliteal vein in the popliteal fossa.

1015. A young man sustains a skull-base fracture at the middle cranial fossa which injures his right abducens (VI) nerve. Which signs are most likely to be present on clinical examination? Single best answer question – choose ONE true option only. A. There is ptosis on the right side B. The pupil on the right side is constricted and fails to respond to light C. The right eyelid is numb D. The patient is unable to deviate his right eye medially E. The patient is unable to deviate his right eye laterally

ANSWER IS E The abducent nerve innervates the lateral rectus muscle of the eye exclusively; the sole effect of damage to this nerve is that the patient is unable to abduct (laterally deviate) the eye.

1016. during resection of a mandibular tumour, there is concern the lingual nerve may have been inadvertently damaged. The lingual nerve is a branch of which of the following nerves? Select one answer only. A. Chorda tympani B. Hypoglossal nerve C. Mandibular division of trigeminal nerve D. Marginal mandibular branch of the facial nerve E. Maxillary division of the trigeminal nerve.

ANSWER IS C The trigeminal nerve innervates the muscles of mastication and is the main sensory nerve of the head. Its three sensory nuclei are in the midbrain, pons and medulla oblongata. The motor nucleus is in the upper part of the pons. The sensory and motor roots emerge from the pons and pass into the middle cranial fossa. At this point the sensory route forms the trigeminal ganglion where the majority of its cell bodies lie. The ganglion gives off three divisions (ophthalmic Va, maxillary Vb and mandibular Vc). The lingual nerve is a branch of the mandibular division.

1017. Which of the following types of nerve fiber does the spinothalamic tract carry? A. Descending motor fibers B. First order fibers carrying light touch and proprioception C. First order fibers carrying pain and temperature sensation D. Second order fibers carrying light touch and proprioception E. Second order fibers carrying pain and temperature sensation

Prepared by Dr: Mohammed Musa Brema Idress – My best wishes Page 456

ANSWER IS E The spinothalamic tract conveys pain, temperature, touch and pressure sensations from one side of the body to the opposite side of the brain. Vibration and position sense are conveyed via the posterior column. The first neuron of the spinothalamic tract synapses in the posterior horn; the next neuron crosses to the right side of the spinal cord and synapse in the thalamus, after ascending through the cord and brainstem; the third neuron arises in the thalamus to pass to the cortex.

The secondary axons of the spinothalamic tract ascend through the brainstem to synapse in the thalamus. Axons from the cervical region synapse medially while axons from the lumbar region synapse laterally. A lesion of the spinothalamic tract anywhere in the brainstem would lead to a loss of pain sensations from the opposite side of the body.

Temperature and touch sensations would also be diminished from the opposite side of the body but not totally lost because other pathways may also convey these modalities. A lesion of the spinothalamic tract at the level of the spinal cord would lead to loss of pain sensations on the opposite side, beginning one level below the level of the lesion.

1018. A pre-operative patient is on β-blockers. Where in the thoracic cage are the beta-1 adrenoceptors concentrated? Single best answer question – choose ONE true option only. A. Cardiac ventricles B. Pulmonary trunk C. Aortic arch D. Lungs E. Carotid sinus.

ANSWER IS A The beta-1 adrenoceptors are located within the ventricles of the heart. The atria contain cholinergic receptors. The aortic arch contains baroreceptors. The Lungs contain beta-2 adrenoceptors. The carotid sinus contains baroreceptors and is in the neck, not the thorax.

1019. While performing a hysterectomy the surgeon ligated the uterine artery on either side. The uterine artery arises from the: Single best answer question – choose ONE true option only. A. Abdominal aorta B. External iliac artery C. Inferior rectal artery D. Internal iliac artery E. Ovarian artery.

ANSWER IS D The uterine artery arises from the anterior division of the internal iliac artery and runs medially on the levator ani toward the uterine cervix. It crosses above and in

Prepared by Dr: Mohammed Musa Brema Idress – My best wishes Page 457 front of the ureter, to which it supplies a small branch, about 2 cm from the cervix. Reaching the side of the uterus, it ascends in a tortuous manner between the two layers of the broad ligament to the junction of the Fallopian tube and uterus.

It then runs laterally toward the hilum of the ovary and ends by joining with the ovarian artery. It supplies branches to the uterine cervix and others that descend on the vagina. The branches descending on the vagina anastomose with branches of the vaginal arteries and form with them two median longitudinal vessels, the azygos arteries of the vagina, one of which runs down in front of and the other behind the vagina.

It supplies numerous branches to the body of the uterus and from its terminal portion branches are distributed to the Fallopian tube and the round ligament of the uterus.

1020. An 18-year-old falls out of a tree and injures his arm by using it to break his fall. Subsequently his hand appears clawed. On examination, his hand is extended at all the MCP joints and all the interphalangeal joints are in fixed flexion. The wrist flexors appear to be slightly weaker. What is likely to be the injury? Single best answer question – choose ONE true option only. A. Ulnar nerve injury B. C8, T1 root injury C. Median nerve injury D. C5, C6 root injury E. Radial nerve injury.

ANSWER IS B This is a classical history for a “Klumpke‟s Palsy”. It is produced by a traction type injury to the lower brachial plexus nerve roots. It is commonly associated with traction to the arm when it is in an extended overhead position. Occasionally damage to T1 may also cause a Horner’s syndrome.

All the intrinsic muscles of the hand are affected as opposed to an ulnar nerve lesion producing a claw hand affecting only the little and ring fingers. In ulnar nerve injuries, the index and middle fingers are spared as the radial 2 lumbricals are supplied by the median nerve.

1021. During total hip arthroplasty via the posterior approach in the gluteal region you are asked to locate the sciatic nerve. Where is the sciatic nerve likely to be found? A. Deep in the upper outer quadrant B. Deep to the obturator internus muscle C. Deep to the piriformis muscle D. Medial to the inferior gluteal vessels E. On the capsule of the hip joint.

Prepared by Dr: Mohammed Musa Brema Idress – My best wishes Page 458

ANSWER IS C The sciatic nerve appears in the gluteal region below the lower border of the piriformis muscle in the vast majority of cases, deep to the lower medial quadrant, and lateral to the inferior gluteal vessels. In its descent, it is separated from the capsule of the hip joint anteriorly by the obturator internus tendon, and by the gemelli muscles.

1022. A patient has been diagnosed with a fast-growing pituitary adenoma. Magnetic resonance image (MRI) scanning reveals suprasellar extension. Which structure is most likely affected? Single best answer question – choose ONE true option only. A. Abducens nerve B. Hypothalamus C. Oculomotor nerve D. Third ventricle E. Optic nerve

ANSWER IS E The pituitary gland occupies the sella turcica, which is a cup-shaped depression in the basisphenoid bone. The roof of the sella is formed by the diaphragma sella, a fold of dura, which is perforated to allow passage of the pituitary stalk. Above the diaphragma lie the suprasellar cistern, the optic chiasm and the anterior cerebral arteries.

The lateral walls of the pituitary fossa are formed by the cavernous sinuses which contain the internal carotid arteries and cranial nerves 3, 4, the first and second divisions of 5, and 6. Behind the sella is the pontine cistern containing the basilar artery. The cavernous sinus, pituitary gland, and stalk and median eminence all show significant enhancement after the administration of intravenous contrast medium.

1023. A 28-year-old pregnant woman presents with bilateral painful lower limbs with varicosities in the long saphenous vein distribution at the level of the perforating veins bilaterally. What is the diagnostic investigation of choice for this lady? A. Ankle Brachial Pressure Index (ABPI B. Lower limb arteriogram C. Lower limb Contrast CT angiogram D. Lower limb duplex venous ultrasound E. Lower limb MRI angiogram

ANSWER IS D Varicose veins are usually acquired and due to incompetent venous valves. Incompetence at the saphenofemoral junction is much more common than sapheno popliteal incompetence (85% versus15%). There is a higher incidence in pregnant women. Duplex ultrasonography is the preoperative investigation of choice.

Prepared by Dr: Mohammed Musa Brema Idress – My best wishes Page 459

Venous ulcers are due to high venous pressure, commonly due to failure of the calf muscle pump. The other investigations relate to arterial disease and are more invasive. Duplex ultrasound venography is safe in a pregnant patient, can exclude DVT, which is a risk factor due to the hypercoaguable state in pregnancy.

1024. A 79-year-old lady suffers a displaced transverse fracture of her left olecranon. The fracture is treated conservatively in a cast. Which function is she most likely to have difficulty with? Single best answer question – choose ONE true option only. A. Brushing her hair B. Fastening buttons C. Pouring a kettle D. Pushing to standing from an armchair E. Reaching into a high cupboard.

ANSWER IS D Triceps attaches to the olecranon and is responsible for extension of the elbow. If olecranon fractures are treated conservatively, an excellent range of movement can be achieved, however functional outcome is impaired due to lack of power of extension. This would be most apparent pushing up against gravity as you have to do when pushing out of a chair.

Brushing hair, reaching into cupboards and pouring kettles are functions mainly achieved by movements of the shoulder, whilst fastening buttons requires dexterity and may be adversely affected by injuries to the wrist or hand.

1025. You are setting up monopolar diathermy at the start of an operation and your consultant asks you for some facts regarding the set up and use of monopolar surgical diathermy. How might you respond? A. It uses an alternating current of 400 Hz B. It requires a patient plate electrode of at least 20 cm2 C. It produces a local heating effect up to 1000 °C D. It cannot be used in patients with pacemakers E. It produces burn injuries that are usually partial thickness.

ANSWER IS C Monopolar diathermy uses an alternating current at a frequency of 400kHz to 10 MHz Current passes down the diathermy forceps, which may be applied to surgical forceps holding tissue, causing a local heating effect up to 1000oC through the patient plate electrode. The patient plate electrode must be at least 70 cm2 in size.

Monopolar diathermy may be used in patients with a pacemaker, but should be used in short bursts of less than 2 seconds, and the diathermy circuit should be away from the site of the pacemaker. It is preferable to use bipolar diathermy in these patients. Diathermy burns are usually full thickness.

Prepared by Dr: Mohammed Musa Brema Idress – My best wishes Page 460

1026. A 51-year-old female is rushed in following a road traffic accident. The patient was haemodynamically stable and a right sided haemothorax was noted. A CT scan showed a contained haematoma of the azygos vein. At what level does the azygos vein enter the superior vena cava? Select one answer only. A. T2 B. T4 C. T6 D. T8 E. T10

ANSWER IS B The azygos vein receives blood from the posterior intercostal veins and segmental veins of the abdomen, It is located on the right, and on the left a variable pattern exists, which mostly involves an accessory hemiazygos vein (superiorly) and azygos vain (inferiorly). The azygos vein is formed at the level of the right renal vein (either as a posterior tributary of the IVC, or as a confluence of the right ascending lumbar vein and right subcostal vein). It passes through the diaphragm via the aortic opening at T12 and ascends on the right side of the vertebral bodies posterior to the oesophagus. It terminates by arching over the hilum of the right lung to enter the SVC at T4 level.

The azygos vein does not extend higher than T4, and therefore the 2nd, 3rd and 4th right intercostal veins drain into the right superior intercostal vein which itself drains into the azygos vein. Other tributaries draining into it include the lower 8 right posterior intercostal veins, bronchial and oesophageal veins and the 2 hemiazygos veins.

1027. The superior cerebral veins drain into the: Single best answer question – choose ONE true option only. A. Cavernous sinus B. Great cerebral vein C. Inferior petrosal sinus D. Superior petrosal sinus E. Superior sagittal sinus

ANSWER IS E The superior cerebral veins, eight to twelve in number, drain the superior, lateral and medial surfaces of the cerebral hemispheres and are mainly lodged in the sulci between the gyri, but some run across the gyri. They open into the superior sagittal sinus. The anterior veins run nearly at right angles to the sinus. The posterior and larger veins are directed obliquely forward and open into the sinus in a direction more or less opposed to the current of the blood contained within it.

1028. What is the function of the middle nasal meatus? A. It contains the bulla ethmoidalis of the middle ethmoidal air sinus B. It drains the nasolacrimal duct C. It drains the posterior ethmoidal air sinus

Prepared by Dr: Mohammed Musa Brema Idress – My best wishes Page 461

D. It drains the sphenoidal air cells E. It is lined by olfactory epithelium containing the primary olfactory neurons.

ANSWER IS A The nasolacrimal duct drains into the inferior meatus; the sphenoidal air sinus drains into the sphenoethmoidal recess; and the posterior ethmoidal air sinus drains into the superior meatus. The middle meatus contains the bulla ethmoidalis of the middle ethmoidal air sinus, which drains through a hiatus in the bulla. Olfactory epithelium containing the primary olfactory neurons lines the superior nasal recess (roof of the nasal cavity) under cover of the cribriform plate of the ethmoid bone.

1029. Following a very difficult right thoracic outlet syndrome decompression in a 35-year-old female there is some respiratory difficulty and there is concern the right phrenic nerve may be injured. Which of these descriptions most accurately describes the muscle relations of the phrenic nerve? Select one answer only. A. Anterior to scalenus anterior B. Anterior to scalenus medius C. Anterior to trapezius D. Posterior to scalenus anterior E. Posterior to scalenus medius.

ANSWER IS A The phrenic nerve arises from C3, C4 and C5 deep to the scalenus anterior and medius muscles, and runs on scalenus anterior, over the anterior part of the dome of the pleura, to enter the mediastinum posterior to the subclavian vein. Here the right phrenic nerve spirals forward to lie on the SVC, right atrium and inferior vena cava (IVC), and traverses the diaphragm via the caval orifice. The vagus nerve gives off the recurrent laryngeal nerve.

1030. What is the mortality rate associated with ruptured abdominal aortic aneurysms (AAA) presenting to hospital? Single best answer - select one answer only. A. 30% B. 40% C. 50% D. 60% E. 70%.

ANSWER IS C The mortality rate of ruptured abdominal aortic aneurysm (AAA) presenting to hospital is 50%. The overall mortality from ruptured AAA is 75%. Size is the most important risk factor for rupture, although rate of expansion is also significant. The patient typically presents with a hypotensive episode or collapse associated with severe central abdominal pain radiating to the back and the flanks.

Prepared by Dr: Mohammed Musa Brema Idress – My best wishes Page 462

1031. A patient with a dislocated shoulder is thought to have an associated axillary nerve injury. What would you expect to find on examination? Single best answer question – choose ONE true option only. A. No sensory loss and weakness of shoulder abduction B. Sensory loss over the anterior aspect of the arm but no motor loss C. Sensory loss over the anterior aspect and weakness of shoulder abduction D. Sensory loss over the lateral aspect and inability to initiate abduction E. Sensory loss over the lateral aspect of the arm and weakness of shoulder abduction

ANSWER IS E The axillary nerve supplies the “regimental patch” area of skin over the upper lateral aspect of the arm. Its motor component is to deltoid and to teres minor. The motor deficit resulting from an axillary nerve injury is weakness of these muscles. The most noticeable is abduction, since flexion and extension are assisted by pectoralis major and latissimus dorsi respectively. There is not complete loss of abduction as supraspinatus will still be intact which mainly initiates abduction.

1032. A 24-year-old rugby player was injured in the face during a match. Eye injuries are common in sports. Which complication results from fractures of the orbital floor ('blowout fractures')? A. Damage to the mental nerve leading to cheek numbness B. Diplopia especially on upward gaze C. Hyphema D. Ocular muscles rupture E. Retinal detachment.

ANSWER IS B Blowout fractures occur along the orbital floor, as this is the thinnest part of the bone. Structures may herniate through into the ethmoidal or maxillary sinuses, causing the „trapdoor‟ appearance on plain X-ray. Ocular injury occurs in 24% of cases, with exophthalmos and diplopia the commonest signs. Computed tomography (CT) helps to delineate the fracture and aids in planning surgery if required. Damage to the infraorbital nerve leads to cheek numbness.

1033. A 17-year-old female skier presents with an acutely painful and swollen ankle joint after falling at speed. She is diagnosed with a bi-malleolar ankle fracture and undergoes open reduction and internal fixation. Which ligament attaches to the medial malleolus? A. Anterior talofibular ligament B. Calcaneofibular ligament C. Deltoid ligament D. Posterior talofibular ligament E. Syndesmotic ligament.

Prepared by Dr: Mohammed Musa Brema Idress – My best wishes Page 463

ANSWER IS C The deltoid ligament attaches to the medial malleolus of the tibia, and inserts via four separate parts into the sustentaculum tali of the oscalcis, the calcaneonavicular ligament, the navicular tuberosity and the medial surface of the talus.

1034. You are reviewing a patient in the intensive care unit with post-operative sepsis and feel they need inotropic support. Regarding normal neurotransmitters in the body which one of the following is correct? Select one answer only. A. About a third of postganglionic sympathetic neurons secrete noradrenaline B. Adrenaline and noradrenaline act on muscarinic and nicotinic receptors C. All preganglionic neurons in the autonomic nervous system are cholinergic D. Most postganglionic sympathetic neurons secrete acetylcholine E. The adrenal cortex secretes both adrenaline and noradrenaline.

ANSWER IS C Acetylcholine and noradrenaline are the two main neurotransmitters secreted in the autonomic nervous system. Neurons secreting acetylcholine are called cholinergic and those secreting noradrenaline are termed adrenergic. Both parasympathetic and sympathetic preganglionic neurons are cholinergic.

Almost all parasympathetic postganglionic neurons are also cholinergic, however most sympathetic postganglionic neurons are adrenergic. Acetylcholine receptors are generally either muscarinic or nicotinic, and adrenergic receptors are divided into either alpha or beta. The adrenal medulla secretes both adrenaline and noradrenaline in response to sympathetic stimulation.

1035. You are reviewing a patient in the ENT clinic with a painful swelling around the jaw and you suspect there is a mass in the submandibular salivary gland. The submandibular gland: A. Has the facial artery running through it B. Has the hypoglossal nerve running through it C. Is deep to the hyoglossus muscle D. Lies entirely below the lower mandible E. Lies below the digastric muscle.

ANSWER IS A The submandibular gland consists of a deep and a superficial part. The superficial part lies in the digastric triangle (above and between the two bellies of the digastric muscle). The hypoglossal nerve runs medial to the superficial part of the gland. The gland is superficial to the mylohyoid and hyoglossus muscles. A third of the submandibular gland lies below the lower border of the mandible and two-thirds above it.

1036. A patient with a history of stable angina undergoes a coronary angiography. This shows triple vessel disease. What is the usual treatment for such a patient? Select one answer only. A. Angioplasty

Prepared by Dr: Mohammed Musa Brema Idress – My best wishes Page 464

B. Angioplasty and stent insertion C. Coronary artery bypass grafting D. Medical treatment E. Surveillance.

ANSWER IS C According to the Coronary Artery Surgery Study the patient groups that derive particular benefit from coronary artery bypass grafting (CABG) are those with triple vessel disease, and those with >50% left main stem stenosis. Those with single or double vessel disease are usually more amenable to percutaneous intervention. Post-myocardial infarction, unstable angina is a primary indication for urgent CABG. Valvular disease with concomitant coronary artery disease is usually treated operatively.

1037. Concerning the uterus: Single best answer question – choose ONE true option only. A. The uterine wall consists of two layers: the myometrium and the endometrium B. The uterus is frequently retroverted in older women C. The body of the uterus is enclosed between the layers of the round ligament D. The principal support of the uterus is the uterosacral ligament E. Peritoneum covers only the superior aspect of the uterus.

ANSWER IS B The uterus is normally bent anteriorly between the cervix and the body of the uterus, and the entire uterus is as a general rule inclined anteriorly (anteverted). However, the uterus is frequently retroverted (inclined posteriorly) in older women.

The wall of the uterus consists of three layers: the perimetrium, the myometrium, and the endometrium. The body of the uterus is enclosed between the layers of the broad ligament and is freely mobile. The uterus is covered by peritoneum anteriorly and superiorly except for the vaginal part of the cervix, and the principal support of the uterus is the pelvic floor.

1038. Which of the structures below is not contained within the carpal tunnel at the wrist? Single best answer question – choose ONE true option only. A. Flexor digitorum Superficialis B. Palmaris longus tendon C. Median nerve D. Flexor digitorum profundus tendons E. Flexor pollicis longus tendon.

ANSWER IS B Palmaris longus lies anterior to the flexor retinaculum, partly adherent to the retinaculum. The tendons of flexor pollicis longus and flexor carpi radialis lie within their own synovial sheaths within the carpal tunnel.

Prepared by Dr: Mohammed Musa Brema Idress – My best wishes Page 465

1039. A variety of fracture patterns result from injurious forces upon the patella. Which is the most common following a direct blow (compressive force) to the patella? A. Marginal B. Osteochondral (sleeve) fractures C. Stellate D. Transverse E. Vertical.

ANSWER IS C Indirect trauma such as knee flexion injuries usually causes a transverse fracture of the patella. Stellate (comminuted) fractures are more frequently seen with direct trauma. Undisplaced fractures are treated with a straight cylinder cast, displaced transverse fractures need internal fixation to repair the extensor mechanism, and comminuted fractures can be treated conservatively, but patellectomy may be required to prevent damage to the patellofemoral joint. However, complete patellectomy should be avoided when possible.

1040. A 69-year-old man falls sustaining a hyperextension injury to his neck. On examination, he has weakness in his upper and lower limbs, but sensation is preserved. Which spinal tract injury explains this presentation? Select one answer only. A. The dorsal column B. The lateral spinothalamic tract C. The ventral spinothalamic tract D. The lateral corticospinal tract E. The ventral corticospinal tract.

ANSWER IS D The lateral corticospinal tract transmits the voluntary motor signals. The dorsal columns are white matter tracts formed by the fasciculus gracilis and fasciculus cuneatus. The fasciculus gracilis lies medial to the fasciculus cuneatus. They carry fine touch and proprioception. Uncrossed fibers are added from medial to lateral as the cord is ascended. Therefore the fasciculus gracilis mainly carries sensory fibers from the lower limbs, and the fasciculus cuneatus carries those from the upper limbs. The lateral spinothalamic tracts carry crossed axons conducting pain and temperature sensations.

1041. A 21-year-old presents with pes planus and weak foot. Which Tendon, lying immediately posterior to the medial malleolus, is responsible for this presentation? A. Flexor Digitorum Longus B. Flexor Hallucis Longus C. Gastrocnemius D. Soleus E. Tibialis posterior

Prepared by Dr: Mohammed Musa Brema Idress – My best wishes Page 466

ANSWER IS E The lateral malleolus is grooved by the peroneus brevis tendon; the medial malleolus by the tibialis posterior tendon. The superficial part of the deltoid ligament is attached to the sustentaculum tali of the calcaneus. Posterior to the medial malleolus, the tibial nerve lies posterior to the posterior tibial artery, and these lie between the flexor hallucis longus tendons posteriorly, and the flexor digitorum longus tendons anteriorly.

1042. Which tendon can be seen during routine arthroscopy of the shoulder joint? Single best answer question – choose ONE true option only. A. Short head of biceps B. Pectoralis major C. Long head of biceps D. Pectoralis minor E. Latissimus Dorsi

ANSWER IS C Long head of biceps passes through the shoulder joint and is invested in a synovial sleeve. As such it will be the only tendon visible during arthroscopy of the shoulder other than the tendons of the rotator cuff.

1043. Exomphalos is associated with all of the following, except: Single best answer - select one answer only. A. Beckwith–Wiedemann syndrome B. Cardiac malformations C. Edward syndrome D. Patau syndrome E. Potter syndrome

ANSWER IS E Exomphalos is the failure of closure of the abdomen at the umbilical ring. Incidence is 1 in 5,000 to 1 in 10,000 live births although the trend is decreasing (partly related to termination of pregnancy following identification of other lethal structural or chromosomal abnormalities.

The incidence of associated abnormalities is between 40% - 70%, including chromosonal (trisomies 13, 18 and 21), cardiac, genitourinary, gastrointestinal, craniofacial, and pulmonary hypoplasia. Syndromes associated include Beckwith– Wiedemann syndrome, Edward syndrome and Patau syndrome.

Potter syndrome or sequence relates to renal abnormalities. Oligohydramnios as a result of renal abnormalities, urinary tract obstruction or amniotic fluid leakage may lead to fetal compression with joint contractures (arthrogryposis), pulmonary hypoplasia and squashed facies.

1044. During a difficult oesophagectomy in a 55-year-old man there is concern the thoracic duct may have been injured in the mediastinum. Which of the following

Prepared by Dr: Mohammed Musa Brema Idress – My best wishes Page 467 descriptions best describes the route of the thoracic duct within the mediastinum in relation to the oesophagus? Select one answer only. A. It passes anterior and to the left of the oesophagus at T5 level B. It passes anterior and to the right of the thoracic duct at T5 level C. It passes posterior and to the left of the oesophagus at T5 level D. It passes posterior and to the left of the oesophagus at T10 level E. It passes posterior and to the right of the thoracic duct at T5 level.

ANSWER IS C The thoracic duct begins below the diaphragm as the cysterna chyli and then ascends through the aortic opening in the diaphragm, to the right of the descending aorta. It passes behind the oesophagus and then to the left of the oesophagus at the level of T5. It then runs upwards on the left side of the oesophagus into the neck. Here it crosses the subclavian artery to enter the left brachiocephalic vein. At the root of the neck, the thoracic duct receives the left jugular, subclavian and bronchomediastinal lymph trunks, although they may occasionally drain directly into the adjacent large vessels. The thoracic duct therefore conveys all the lymph from the lower limbs, pelvic cavity, abdominal cavity, left side of the thorax, head and neck and the left arm.

1045. Itching sensation from the skin immediately over the base of the spine of your scapula is mediated through the: Single best answer question – choose ONE true option only. A. Accessory nerve B. Dorsal primary ramus of T2 C. Dorsal root of T2 D. Ventral primary ramus of T2 E. Ventral root of T2.

ANSWER IS B Dorsal and ventral primary rami are the first branches off spinal nerves. Dorsal rami provide sensory innervation to the skin over the back and give motor innervation to the true back muscles; ventral rami supply sensory innervation to the skin over the limbs and the skin over the ventral side of the trunk.

Ventral rami also give motor innervation to the skeletal muscles of the neck, trunk and extremities. The accessory nerve, which innervates the trapezius, is not responsible for any sensory innervation. The dorsal and ventral roots of spinal nerves are not directly responsible for any sensory innervation to the skin. Dorsal and ventral rootlets emerge from the spinal cord to form the dorsal and ventral roots. The ventral roots contain efferent motor fibers to skeletal muscles, while the dorsal roots contain afferent sensory fibers. These roots combine to form the spinal nerve, which then gives off the primary rami.

1046. A 7-year-old boy is seen by his GP for otitis media of his left ear, on further questioning it emerges he has had recurrent upper respiratory tract infections and

Prepared by Dr: Mohammed Musa Brema Idress – My best wishes Page 468 has erythematous swollen tonsils at present. Which part of the skull does the Eustachian tube penetrate? Select one answer only. A. Greater wing of the sphenoid B. Lesser wing of the sphenoid C. Occipital bone D. Petrous temporal bone E. Squamous temporal bone.

ANSWER IS D The Eustachian (Phanyngotympanic, auditory) tube connects the tympanic cavity to the nasopharynx. The posterolateral third is bony and the remainder cartilaginous. The Eustachian tube in a child is shorter and more horizontal. The opening of the auditory tube lies above the soft palate adjacent to the tubal tonsil. The bony part of the Eustachian tube perforates the petrous temporal bone.

1047. In the clinical examination of the chest, accurate knowledge of the surface markings of the lungs is essential. Which of the following corresponds to the clinical situation? Single best answer question – choose ONE true option only. A. The apex of the lung corresponds precisely to the upper border of the medial third of the clavicle B. The oblique fissure of the lung corresponds to the medial border of the scapula when the arm is fully abducted C. The transverse (horizontal) fissure of the right lung corresponds to the right fifth intercostal space D. The lower border of the lung on each side corresponds to the tenth rib in the mid-axillary line E. The lower border of the lung reaches the twelfth rib posteriorly.

ANSWER IS B The apex of the lung extends about 4 cm above the medial one-third of the clavicle. The oblique fissure does indeed correspond closely to the medial border of the scapula when the arm is fully abducted. The transverse fissure of the right lung corresponds to the level of the fourth rib. The lower border of the lung on each side corresponds to the eighth rib in the mid-axillary line and the tenth rib posteriorly.

1048. An 18-month-old child is brought to see you in clinic as his mother has noted mucus and blood staining on nappies, and has seen a bright red mass intermittently protruding from the anus. You diagnose a partial rectal prolapse. Which of the following is true regarding partial rectal prolapses? Select one answer only. A. It involves all layers of the rectal wall B. It is associated with a significant increased risk of rectal carcinoma C. It is more common in adults than children D. It should be treated with DeLorme’s procedure E. The majority resolve with conservative treatment

Prepared by Dr: Mohammed Musa Brema Idress – My best wishes Page 469

ANSWER IS E Complete rectal prolapse involves all layers of the rectal wall and is more commonly seen in elderly women. Partial rectal prolapse is more common in children and involves the mucosal layer only. The majority of cases of partial rectal prolapse in children resolve with conservative treatment.

DeLorme‟s procedure is excision of the mucosal component of a prolapse with plication of the muscle from below, and is generally used in adults. There is an associated increased risk of rectal carcinoma in adults with complete rectal prolapse, but this risk has not been reported in children with partial rectal prolapse.

1049. A patient has a fall onto his shoulder and is unable to internally rotate the shoulder against resistance. Other movements are relatively well preserved. X-rays taken show no bony injury. Which of the following muscles is likely to be injured? Single best answer question – choose ONE true option only. A. Supraspinatus B. Infraspinatus C. Teres Minor D. Deltoid E. Subscapularis

ANSWER IS E Subscapularis produces internal rotation of the shoulder joint. It arises from the costal surface of the scapula and inserts into the lesser tubercle.

Pectoralis major and latissimus dorsi contribute to internal rotation so there is not a complete loss of internal rotation if only subscapularis has been injured.

Supraspinatus initiates abduction and helps in abducting the shoulder joint between 90° and 120°. Teres minor along with infraspinatus externally rotates the shoulder. Deltoid flexes and abducts the shoulder.

1050. The greater trochanter of the femur is a bony prominence of the proximal femur that serves as insertional muscle attachment. Which muscle is attached to the greater trochanter of the femur? A. Gluteus maximus B. Piriformis C. Psoas D. Sartorius E. Vastus lateralis.

ANSWER IS B Piriformis, gluteus medius, gluteus minimus, obturator internus, obturator externus, quadratus femoris and the gemelli are all attached to the greater trochanter of the femur. Psoas and iliacus both insert on the lesser trochanter.

Prepared by Dr: Mohammed Musa Brema Idress – My best wishes Page 470

1051. An elderly man with pre-existing cervical spondylosis, falls sustaining hyperextension injury to his neck. On examination he has motor deficit worse in the upper limb than the lower limb. Which spinal cord injury explains this presentation? Select one answer only. A. Complete spinal cord injury B. Anterior cord syndrome C. Brown-Sequard syndrome D. Central cord syndrome E. Posterior cord syndrome.

ANSWER IS D The central cord syndrome is thought to be due to vascular compromise of the cord in the distribution of the anterior spinal artery. Infarction of the cord in the territory of this artery could also result in the anterior cord syndrome. Anterior cord syndrome is characterized by paraplegia and a dissociated sensory loss with loss of pain and temperature sensation.

Position, vibration and deep pressure sensations, all functions of the posterior column, are preserved. Posterior cord syndrome is a rare, incomplete lesion with primary damage to the posterior cord, featuring preservation of motor function, pain and temperature sensations with loss of proprioception and fine touch sensation below the level of lesion.

Brown–Sequard syndrome, resulting from hemisection of the cord, usually causes ipsilateral motor loss and contralateral loss of pain and temperature sensation; there is also associated loss of position sense. The central cord syndrome is characterized by a disproportionately greater loss of motor power in the upper extremities than in the lower extremities, with varying degrees of sensory loss.

The arms and hands are most severely affected since the motor fibers to the cervical segments are topographically arranged toward the centre of the cord. Anterior cord syndrome has the poorest prognosis of all incomplete spinal injuries. In contrast to all the above, a complete spinal cord injury would feature a complete loss of sensory and motor function below the level of lesion.

1052. Which of the following statements concerning neurotransmitters is most accurate? Select one answer only. A. Adrenaline and noradrenaline act on muscarinic and nicotinic receptors B. Few postganglionic parasympathetic neurons secrete acetylcholine C. Few postganglionic sympathetic neurons secrete noradrenaline D. No preganglionic neurons in the autonomic nervous system are cholinergic E. The adrenal medulla secretes both adrenaline and noradrenaline

ANSWER IS E Acetylcholine and noradrenaline are the two main neurotransmitters secreted in the autonomic nervous system. Neurons secreting acetylcholine are called cholinergic

Prepared by Dr: Mohammed Musa Brema Idress – My best wishes Page 471 and those secreting noradrenaline are termed adrenergic. Both parasympathetic and sympathetic preganglionic neurons are cholinergic.

Almost all parasympathetic postganglionic neurons are also cholinergic, however most sympathetic postganglionic neurons are adrenergic. Acetylcholine receptors are generally either muscarinic or nicotinic, and adrenergic receptors are divided into either alpha or beta. The adrenal medulla secretes both adrenaline and noradrenaline in response to sympathetic stimulation.

1053. The modified synovial joint allows flexion and extension in one plane. Which of the lower limb joints is a modified synovial joint? A. The calcaneocuboid joint B. The hip joint C. The inferior tibiofibular joint D. The intermetatarsal joints E. The knee

ANSWER IS E The knee is a modified synovial hinge joint. The tibiotalar joint is a synovial hinge joint. The inferior tibiofibular joint is fibrous.

1054. You are in theatre performing a formal tracheostomy. When performing this procedure: A. It is best performed with the neck slightly flexed B. Midline dissection provides a relatively avascular plane C. Once the tracheostomy cuff is inflated further fixation is not required D. The first and second tracheal rings are incised E. The skin incision is placed between the cricoid cartilage and hyoid.

ANSWER IS B Tracheostomy is a definitive surgical airway. The head and neck are extended, and a 3–4 cm transverse collar incision is made between the cricoid cartilage and sternal notch. Once the incision is deepened through the platysma muscle, the remainder of dissection should continue in the midline in a relatively avascular plane down to the pretracheal fascia.

This layer is then incised to reveal the tracheal rings. Care must be taken not to incise the first ring (the cricoid cartilage). A vertical incision through the second and third rings is usual. Once the airway is in the trachea, the cuff may be inflated. Further fixation is then mandatory to prevent the tracheostomy tube from being displaced.

1055. A patient attends clinic complaining of pins and needles affecting the radial (lateral) 3 digits of his hand. The symptoms are worse at night, and also after driving long distances. Which of the following motor signs would confirm the likely diagnosis? Single best answer question – choose ONE true option only. A. Inability to flex the radial 3 digits B. Inability to adduct all the fingers of his hand

Prepared by Dr: Mohammed Musa Brema Idress – My best wishes Page 472

C. Inability to abduct all the fingers of his hand D. Weakness of opposition of the thumb E. The likely diagnosis would not cause a motor deficit.

ANSWER IS D The history described above is typical of carpal tunnel syndrome. Entrapment of the median nerve at the carpal tunnel affects the muscles of the thenar eminence. These are abductor pollicis brevis, flexor pollicis brevis and opponens pollicis. The nerve supply of flexor pollicis brevis is extremely variable however, so the best test to perform would be to see if opposition is affected.

In addition the motor branch of the median nerve after the level of the carpal tunnel also innervates the radial 2 lumbricals. Flexion would not be affected as the nerve supply to the long flexors comes off the median nerve proximal to the carpal tunnel. Abduction and adduction of the fingers is produced by the interossei, which are supplied by the ulnar nerve.

1056. Which one of the following structures is not encountered during a right hemicolectomy? Single best answer question – choose ONE true option only. A. The right ureter B. The right gonadal artery C. The inferior mesenteric artery D. The second part of the duodenum E. The middle colic artery.

ANSWER IS C The inferior mesenteric artery supplies the large bowel from the transverse colon distally and is not encountered in a right hemicolectomy which involves excision of the caecum, ascending colon and the middle third of the transverse colon. The transverse colon crosses the second part of the duodenum.

The right ureter and gonadal vessels although both retroperitoneal, are both posteriorly related to the ascending colon and are easily injured during dissection. The middle colic artery is the terminal branch of the superior mesenteric artery and its right branch is usually included in the excision during a right hemicolectomy.

1057. A 19-year-old male is brought into A & E following a stabbing. On examination a wound is seen in the left supraclavicular fossa, but he is otherwise haemodynamically stable. A chest X-ray shows the left hemidiaphragm is elevated and there is concern that the phrenic nerve may be affected. Which of the following descriptions best describe the route by which the phrenic nerve enters the chest? Select one answer only. A. Anterior to subclavian artery and posterior to the subclavian vein. Medial to the lower border of scalenus anterior B. Anterior to the subclavian artery and posterior to the subclavian vein. Lateral to the lower border of scalenus anterior

Prepared by Dr: Mohammed Musa Brema Idress – My best wishes Page 473

C. Anterior to the subclavian vein and artery. Medial to the lower border of scalenus anterior D. Posterior to subclavian artery and vein. Medial to the lower border of scalenus anterior E. Posterior to subclavian vein and artery. Lateral to the lower border of scalenus anterior.

ANSWER IS A The autonomic fibers in the phrenic nerve are sympathetic and pass from the superior (C1–C4) and middle (C5/6) sympathetic cervical ganglia as grey rami into the C3–C5 roots of the phrenic nerve, and innervate blood vessels in the diaphragm.

The nerve lies on the fibrous pericardium and is sensory to the mediastinal and diaphragmatic pleura, and also to the diaphragmatic peritoneum. The phrenic nerve enters the chest by descending from the medial lower border of the scalenus anterior muscle between the subclavian vein anteriorly and artery posteriorly.

1058. What is the most useful test to clinically distinguish between an upper and lower motor neuron lesion of the facial nerve? A. Blow cheeks out B. Loss of chin reflex C. Close eye D. Raise eyebrow E. Open mouth against resistance

ANSWER IS D Upper motor neuron lesions of the facial nerve- Paralysis of the lower half of face. Lower motor neuron lesion- Paralysis of the entire ipsilateral face.

1059. An 18 year old man is stabbed in the axilla during a fight. His axillary artery is lacerated and repaired. However, the surgeon neglects to repair an associated injury to the upper trunk of the brachial plexus. Which of the following muscles is least likely to demonstrate impaired function as a result? A. Palmar interossei B. Infraspinatus C. Brachialis D. Supinator brevis E. None of the above

ANSWER IS A The palmar interossei are supplied by the ulnar nerve. Which lies inferiorly and is therefore less likely to be injured.?

1060. A 23 year old man is involved in a fight, during the dispute he sustains a laceration to the posterior aspect of his right arm, approximately 2cm proximal to the olecranon process. On assessment in the emergency department he is unable

Prepared by Dr: Mohammed Musa Brema Idress – My best wishes Page 474 to extend his elbow joint. Which of the following tendons is most likely to have been cut? A. Triceps B. Pronator teres C. Brachioradialis D. Brachialis E. Biceps

ANSWER IS A The triceps muscle extends the elbow joint. The other muscles listed all produce flexion of the elbow joint. Triceps:- Origin Long head - infraglenoid tubercle of the scapula. Lateral head - dorsal surface of the humerus, lateral and proximal to the groove of the radial nerve Medial head - posterior surface of the humerus on the inferomedial side of the radial groove and both of the intermuscular septae Insertion Olecranon process of the ulna. Here the olecranon bursa is between the triceps tendon and olecranon. Some fibers insert to the deep fascia of the forearm, posterior capsule of the elbow (preventing the capsule from being trapped between olecranon and olecranon fossa during extension) Innervation - Radial nerve Blood supply - Profunda brachii artery Action - Elbow extension. The long head can adduct the humerus and extend it from a flexed position Relations - The radial nerve and profunda brachii vessels lie between the lateral and medial heads

1061. A 25 year old man undergoes an excision of a pelvic chondrosarcoma, during the operation the obturator nerve is sacrificed. Which of the following muscles is least likely to be affected as a result? A. Adductor longus B. Pectineus C. Adductor magnus D. Sartorius E. Gracilis

ANSWER IS D Sartorius is supplied by the femoral nerve. In approximately 20% of the population, pectineus is supplied by the accessory obturator nerve.

1062. You excitedly embark on your first laparoscopic cholecystectomy and during the operation the anatomy of Calots triangle is more hostile than anticipated. Whilst trying to apply a haemostatic clip you avulse the cystic artery. This is followed by brisk haemorrhage. From which source is this most likely to originate ?

Prepared by Dr: Mohammed Musa Brema Idress – My best wishes Page 475

A. Right hepatic artery B. Portal vein C. Gastroduodenal artery D. Liver bed E. Common hepatic artery

ANSWER IS A The cystic artery is a branch of the right hepatic artery. There are recognized variations in the anatomy of the blood supply to the gallbladder. However, the commonest situation is for the cystic artery to branch from the right hepatic artery.

1063. A 43 year old man suffers a pelvic fracture which is complicated by an injury to the junction of the membranous urethra to the bulbar urethra. In which of the following directions is the extravasated urine most likely to pass? A. Posteriorly into extra peritoneal tissues B. Laterally into the buttocks C. Into the abdomen D. Anteriorly into the connective tissues surrounding the scrotum E. None of the above

ANSWER IS D The superficial perineal pouch is a compartment bounded superficially by the superficial perineal fascia, deep by the perineal membrane (inferior fascia of the urogenital diaphragm), and laterally by the ischiopubic ramus. It contains the crura of the penis or clitoris, muscles, viscera, blood vessels, nerves, the proximal part of the spongy urethra in males, and the greater vestibular glands in females.

When urethral rupture occurs as in this case the urine will tend to pass anteriorly because the fascial condensations will prevent lateral and posterior passage of the urine.

Urogenital triangle The urogenital triangle is formed by the: 1. Ischiopubic inferior rami 2. Ischial tuberosities 3. A fascial sheet is attached to the sides, forming the inferior fascia of the urogenital diaphragm. It transmits the urethra in males and both the urethra and vagina in females. The membranous urethra lies deep to this structure and is surrounded by the external urethral sphincter.

Superficial to the urogenital diaphragm lies the superficial perineal pouch. In males this contains: 1. Bulb of penis 2. Crura of the penis 3. Superficial transverse perineal muscle 4. Posterior scrotal arteries

Prepared by Dr: Mohammed Musa Brema Idress – My best wishes Page 476

5. Posterior scrotal nerves In females the internal pudendal artery branches to become the posterior labial arteries in the superficial perineal pouch.

1064. Which of the following does not pass through the superior orbital fissure? A. Oculomotor nerve B. Abducens nerve C. Ophthalmic artery D. Ophthalmic division of the trigeminal nerve E. Ophthalmic veins

ANSWER IS C The ophthalmic artery, a branch of the internal carotid enters the orbit with the optic nerve in the canal.

1065. Which nerve supplies the 1st web space of the foot? A. Popliteal nerve B. Superficial peroneal nerve C. Deep peroneal nerve D. Tibial nerve E. Saphenous nerve

ANSWER IS C The first web space is innervated by the deep peroneal nerve. Deep peroneal nerve:- Origin: From the common peroneal nerve, at the lateral aspect of the fibula, deep to peroneus longus Nerve root values: L4, L5, S1, S2 Course and relation Pierces the anterior intermuscular septum to enter the anterior compartment of the lower leg Passes anteriorly down to the ankle joint, midway between the two malleoli. Terminates In the dorsum of the foot Muscles innervated: 1. Tibialis anterior 2. Extensor hallucis longus 3. Extensor digitorum longus 4. Peroneus tertius 5. Extensor digitorum brevis Cutaneous innervations: Web space of the first and second toes Actions: - Dorsiflexion of ankle joint - Extension of all toes (extensor hallucis longus and extensor digitorum longus) Inversion of the foot After its bifurcation past the ankle joint, the lateral branch of the deep peroneal nerve innervates the extensor digitorum brevis and the extensor hallucis brevis. The medial branch supplies the web space between the first and second digits.

Prepared by Dr: Mohammed Musa Brema Idress – My best wishes Page 477

1066. During the course of a radical gastrectomy the surgeons detach the omentum and ligate the right gastro-epiploic artery. From which vessel does it originate? A. Superior mesenteric artery B. Inferior mesenteric artery C. Coeliac axis D. Common hepatic artery E. Gastroduodenal artery

ANSWER IS E The gastroduodenal artery arises at the superior part of the duodenum and descends behind it to terminate at its lower border. It terminates by dividing into the right gastro-epiploic artery and the superior pancreaticoduodenal artery.

The right gastro-opiploic artery passes to the left and passes between the layers of the greater omentum to anastomose with the left gastro-epiploic artery.

1067. Which of the following is not an intrinsic muscle of the hand? A. Opponens pollicis B. Palmaris longus C. Flexor pollicis brevis D. Flexor digiti minimi brevis E. Opponens digiti minimi

ANSWER IS B Mnemonic for intrinsic hand muscles ='A OF A OF A' A bductor pollicis brevis O pponens pollicis F lexor pollicis brevis A dductor pollicis (thenar muscles) O pponens digiti minimi F lexor digiti minimi brevis A bductor digiti minimi (hypothenar muscles) Palmaris longus originates in the forearm.

1068. A man with lung cancer and bone metastasis in the thoracic spinal vertebral bodies, sustains a pathological fracture at the level of T4. The fracture is unstable and the spinal cord is severely compressed at this level. Which of the findings below will not be present 6 weeks after injury? A. Extensor plantar reflexes B. Spasticity of the lower limbs C. Diminished patellar tendon reflex D. Urinary incontinence E. Sensory ataxia

ANSWER IS C

Prepared by Dr: Mohammed Musa Brema Idress – My best wishes Page 478

A thoracic cord lesion causes spastic paraperesis, hyperrflexia and extensor plantar responses (UMN lesion), incontinence, sensory loss below the lesion and 'sensory' ataxia. These features typically manifest several weeks later, once spinal shock (in which areflexia predominates) has resolved.

1069. Through which of the following foramina does the genital branch of the genitofemoral nerve exit the abdominal cavity? A. Superficial inguinal ring B. Sciatic notch C. Obturator foramen D. Femoral canal E. Deep inguinal ring

ANSWER IS E The genitofemoral nerve divides into two branches as it approaches the inguinal ligament. The genital branch passes anterior to the external iliac artery through the deep inguinal ring into the inguinal canal. It communicates with the ilioinguinal nerve in the inguinal canal (though this is seldom of clinical significance).

1070. A 28 year old man lacerates the posterolateral aspect of his wrist with a knife in an attempted suicide. On arrival in the emergency department the wound is inspected and found to be located over the lateral aspect of the extensor retinaculum (which is intact). Which of the following structures is at greatest risk of injury? A. Superficial branch of the radial nerve B. Radial artery C. Dorsal branch of the ulnar nerve D. Tendon of extensor carpi radialis brevis E. Tendon of extensor digiti minimi

ANSWER IS A The superficial branch of the radial nerve passes superior to the extensor retinaculum in the position of this laceration and is at greatest risk of injury. The dorsal branch of the ulnar nerve and artery also pass superior to the extensor retinaculum n but are located medially.

1071. A 43 year old man is reviewed in the clinic following a cardiac operation. A chest x-ray is performed and a circular radio-opaque structure is noted medial aspect of the 4th intercostal space on the left. Which of the following procedures is the patient most likely to have undergone? A. Aortic valve replacement with metallic valve B. Tricuspid valve replacement with metallic valve C. Tricuspid valve replacement with porcine valve D. Pulmonary valve replacement with porcine valve E. Mitral valve replacement with metallic valve

ANSWER IS E

Prepared by Dr: Mohammed Musa Brema Idress – My best wishes Page 479

Prosthetic heart valves on Chest X-rays:- The aortic and mitral valves are most commonly replaced and when a metallic valve is used, can be most readily identified on plain x-rays. The presence of cardiac disease (such as cardiomegaly) may affect the figures quoted here.

Aortic: Usually located medial to the 3rd interspace on the right. Mitral: Usually located medial to the 4th interspace on the left. Tricuspid: Usually located medial to the 5th interspace on the right.

Please note that these are the sites at which an artificial valve may be located and are NOT the sites of auscultation.

1072. A 63 year old lady is diagnosed as having an endometrial carcinoma arising from the uterine body. To which nodal region will the tumour initially metastasise? A. Para aortic nodes B. Iliac lymph nodes C. Inguinal nodes D. Pre sacral nodes E. Mesorectal lymph nodes

ANSWER IS B Tumours of the uterine body will tend to spread to the iliac nodes initially. Tumour expansion crossing different nodal margins this is of considerable clinical significance, if nodal clearance is performed during a Wertheim’s type hysterectomy.

1073. Transection of the radial nerve at the level of the axilla will result in all of the following except: A. Loss of elbow extension. B. Loss of extension of the interphalangeal joints. C. Loss of metacarpophalangeal extension. D. Loss of triceps reflex. E. Loss of sensation overlying the first dorsal interosseous.

ANSWER IS B These may still extend by virtue of retained lumbrical muscle function.

1074. Which of the following structures is not located in the superficial perineal space in females? A. Posterior labial arteries B. Pudendal nerve C. Superficial transverse perineal muscle D. Greater vestibular glands E. None of the above

Prepared by Dr: Mohammed Musa Brema Idress – My best wishes Page 480

ANSWER IS B The pudendal nerve is located in the deep perineal space and then branches to innervate more superficial structures.

1075. Which of the following is not a branch of the hepatic artery? A. Pancreatic artery B. Cystic artery C. Right gastric artery D. Right hepatic artery E. Gastroduodenal artery

ANSWER IS A The pancreatic artery is a branch of the splenic artery.

1076. Which of the following structures does not pass behind the piriformis muscle in the greater sciatic foramen? A. Sciatic nerve B. Posterior cutaneous nerve of the thigh C. Inferior gluteal artery D. Obturator nerve E. None of the above

ANSWER IS D The obturator nerve does not pass through the greater sciatic foramen.

1077. A 56 year old man is undergoing a right nephrectomy. The surgeons divide the renal artery. At what level does this usually branch off the abdominal aorta? A. T9 B. L2 C. L3 D. T10 E. L4

ANSWER IS B The renal arteries usually branch off the aorta on a level with L2.

1078. A 23 year old man is shot in the chest during a robbery. The left lung is lacerated and is bleeding. An emergency thoracotomy is performed. The surgeons place a clamp over the hilum of the left lung. Which of the following structures lies most anteriorly at this level? A. Vagus nerve B. Oesophagus C. Descending aorta D. Phrenic nerve E. Azygos vein

ANSWER IS D

Prepared by Dr: Mohammed Musa Brema Idress – My best wishes Page 481

The phrenic nerve lies anteriorly at this point. The vagus passes anteriorly and then arches backwards immediately superior to the root of the left bronchus, giving off the recurrent laryngeal nerve as it does so.

1079. A 22 year old man presents with appendicitis. At operation the appendix is retrocaecal and difficult to access. Division of which of the following anatomical structures should be undertaken? A. Ileocolic artery B. Mesentery of the caecum C. Gonadal vessels D. Lateral peritoneal attachments of the caecum E. Right colic artery

ANSWER IS D The commonest appendiceal location is retrocaecal. Those struggling to find it at operation should trace the tenia to the caecal pole where the appendix is located. If it cannot be mobilized easily then division of the lateral caecal peritoneal attachments (as for a right hemicolectomy) will allow caecal mobilization and facilitate the procedure.

1080.Which of the following muscles does not adduct the shoulder? A. Teres major B. Pectoralis major C. Coracobrachialis D. Supraspinatus E. Latissimus dorsi

ANSWER IS D Supraspinatus is an abductor of the shoulder.

1081. Which of these muscles is innervated by the cervical branch of the facial nerve? A. Masseter B. Sternocleidomastoid C. Platysma D. Geniohyoid E. Sternothyroid

ANSWER IS C The cervical branch of the facial nerve innervates platysma.

1082. During a thyroidectomy the surgeons ligate the inferior thyroid artery. From which vessel does this structure usually originate? A. External carotid artery B. Thyrocervical trunk C. Internal carotid artery D. Subclavian artery

Prepared by Dr: Mohammed Musa Brema Idress – My best wishes Page 482

E. Vertebral artery

ANSWER IS B The inferior thyroid artery originates from the thyrocervical trunk. This is a branch of the subclavian artery.

1083. A 56 year old man is left impotent following an abdomino-perineal excision of the colon and rectum. What is the most likely explanation? A. Psychosexual issues related to an end colostomy B. Damage to the sacral venous plexus during total mesorectal excision C. Damage to the left ureter during sigmoid mobilization D. Damage to the hypogastric plexus during mobilization of the inferior mesenteric artery E. Damage to the internal iliac artery during total mesorectal excision

ANSWER IS D Autonomic nerve injury is the most common cause.

1084. A 73 year old man is due to undergo a radical prostatectomy for carcinoma of the prostate gland. To which of the following lymph nodes will the tumour drain primarily? A. Para aortic B. Internal iliac C. Superficial inguinal D. Mesorectal E. None of the above

ANSWER IS B The prostate lymphatic drainage is primarily to the internal iliac nodes and also the sacral nodes. Although internal iliac is the first site.

1085. Which of the following statements relating to the vertebral column is false? A. There are 7 cervical vertebrae B. The cervical and lumbar lordosis are secondary curves developing after birth due to change in shape of the intervertebral discs C. The lumbar vertebrae do not have a transverse process foramina D. The lumbar vertebrae receive blood directly from the aorta E. The spinous process is formed by the junction of the pedicles posteriorly

ANSWER IS E The spinous process is formed by 2 laminae posteriorly.

1086. A 78 year old lady falls over in her nursing home and sustains a displaced intracapsular fracture of the femoral neck. A decision is made to perform a hemi arthroplasty through a lateral approach. Which of these vessels will be divided to facilitate access? A. Saphenous vein

Prepared by Dr: Mohammed Musa Brema Idress – My best wishes Page 483

B. Superior gluteal artery C. Superficial circumflex iliac artery D. Profunda femoris artery E. Transverse branch of the lateral circumflex artery

ANSWER IS E During the Hardinge style lateral approach the transverse branch of the lateral circumflex artery is divided to gain access.

1087. A 73 year old man undergoes a sub total oesophagectomy with anastomosis of the stomach to the cervical oesophagus. Which vessel will be primarily responsible for the arterial supply to the oesophageal portion of the anastomosis? A. Superior thyroid artery B. Internal carotid artery C. Direct branches from the thoracic aorta D. Inferior thyroid artery E. Subclavian artery

ANSWER IS D The cervical oesophagus is supplied by the inferior thyroid artery. The thoracic oesophagus (removed in this case) is supplied by direct branches from the thoracic aorta.

1088. Which of the following structures is not closely related to the brachial artery? A. Ulnar nerve B. Median nerve C. Cephalic vein D. Long head of triceps E. Median cubital vein

ANSWER IS C The cephalic vein lies superficially and on the contralateral side of the arm to the brachial artery.

1089. Which ligament keeps the head of the radius connected to the radial notch of the ulna? A. Annular (orbicular) ligament B. Quadrate ligament C. Radial collateral ligament of the elbow D. Ulnar collateral ligament E. Radial collateral ligament

ANSWER IS A The annular ligament connects the radial head to the radial notch of the ulna.

Prepared by Dr: Mohammed Musa Brema Idress – My best wishes Page 484

1090. A 38 year old man presents to the clinic with shoulder weakness. On examination he has an inability to initiate shoulder abduction. Which of the nerves listed below is least likely to be functioning normally? A. Suprascapular nerve B. Medial pectoral nerve C. Axillary nerve D. Median nerve E. Radial nerve

ANSWER IS A Suprascapular nerve:- The suprascapular nerve arises from the upper trunk of the brachial plexus. It lies superior to the trunks of the brachial plexus and passes inferolaterally parallel to them. It passes through the scapular notch, deep to trapezius.

It innervates both supraspinatus and infraspinatus and initiates abduction of the shoulder. If damaged, patients may be able to abduct the shoulder by leaning over the affected side and deltoid can then continue to abduct the shoulder.

1091. Which of the following statements relating to the Cavernous Sinus is false? A. The pituitary gland lies medially B. The internal carotid artery passes through it C. The temporal lobe of the brain is a lateral relation D. The mandibular branch of the trigeminal and optic nerve lie on the lateral wall E. The ophthalmic veins drain into the anterior aspect of the sinus

ANSWER IS D The veins that drain into the sinus are important as sepsis can cause cavernous sinus thrombosis. The maxillary branch of the trigeminal and not the mandibular branches pass through the sinus

1092. Which of the following is not a branch of the subclavian artery? A. Superior thyroid artery B. Vertebral artery C. Thyrocervical trunk D. Internal thoracic artery E. Dorsal scapular artery

ANSWER IS A Mnemonic for the branches of the subclavian artery: VIT C & D V ertebral artery I nternal thoracic T hyrocervical trunk C ostalcervical trunk D orsal scapular Superior thyroid artery is a branch of the external carotid artery.

Prepared by Dr: Mohammed Musa Brema Idress – My best wishes Page 485

1093. During the repair of an atrial septal defect the surgeons note that blood starts to leak from the coronary sinus. Which structure forms the largest tributary of the coronary sinus? A. The besian veins B. Great cardiac vein C. Oblique vein D. Small cardiac veins E. None of the above

ANSWER IS B The great cardiac vein runs in the anterior interventricular groove, and is the largest tributary of the coronary sinus. The besian veins drain into the heart directly.

1094. Which of the following vessels provides the greatest contribution to the arterial supply of the breast? A. External mammary artery B. Thoracoacromial artery C. Internal mammary artery D. Lateral thoracic artery E. Subclavian artery

ANSWER IS C 60% of the arterial supply to the breast is derived from the internal mammary artery. The external mammary and lateral thoracic arteries also make a significant (but lesser) contribution. This is of importance clinically in performing reduction mammoplasty procedures.

1095. Which of the following muscles is supplied by the external laryngeal nerve? A. Transverse arytenoid B. Cricothyroid C. Thyro-arytenoid D. Posterior crico-arytenoid E. Oblique arytenoid

ANSWER IS B The others are all supplied by the recurrent laryngeal nerve.

1096. A 28 year old man has sustained a non salvageable testicular injury to his left testicle. The surgeon decides to perform an orchidectomy and divides the left testicular artery. From which of the following does this vessel originate? A. Abdominal aorta B. Internal iliac artery C. Inferior epigastric artery D. Inferior vesical artery E. External iliac artery

ANSWER IS A: The testicular artery is a branch of the abdominal aorta.

Prepared by Dr: Mohammed Musa Brema Idress – My best wishes Page 486

1097. During a carotid endarterectomy the internal carotid artery is cross clamped. Assuming that no shunt is inserted, which of the following vessels will not have diminished or absent flow as a result? A. Anterior cerebral artery B. Ophthalmic artery C. Middle cerebral artery D. Maxillary artery E. None of the above

ANSWER IS D Mnemonic for branches of the cerebral portion of the internal carotid artery 'Only Press Carotid Arteries Momentarily' - Only = Opthalmic - Press = Posterior communicating - Carotid = Choroidal - Arteries = Anterior cerebral - Momentarily = Middle cerebral The maxillary artery is a branch of the external carotid artery.

1098. A 72 year old lady with osteoporosis falls and sustains an intracapsular femoral neck fracture. The fracture is completely displaced. Which of the following vessels is the main contributor to the arterial supply of the femoral head? A. Deep external pudendal artery B. Superficial femoral artery C. External iliac artery D. Circumflex femoral arteries E. Superficial external pudendal artery

ANSWER IS D The vessels which form the anastomoses around the femoral head are derived from the medial and lateral circumflex femoral arteries. These are usually derived from the profunda femoris artery.

1099. A 21 year old man is hit with a hammer and sustains a depressed skull fracture at the vertex. Which of the following sinuses is at risk in this injury? A. Superior sagittal sinus B. Inferior petrosal sinus C. Transverse sinus D. Inferior sagittal sinus E. Straight sinus

ANSWER IS A The superior sagittal sinus is at greatest risk in this pattern of injury. This sinus begins at the front of the crista galli and courses backwards along the falx cerebri. It becomes continuous with the right transverse sinus near the internal occipital protuberance.

Prepared by Dr: Mohammed Musa Brema Idress – My best wishes Page 487

1100. What is the most appropriate description of the transplanted tissue if a patient receives a porcine aortic valve replacement? A. Autograft B. Allograft C. Monograft D. Isograft E. Xenograft.

ANSWER IS E A xenograft (or heterograft) is an organ or tissue transferred from one species to another species.

1101. What is the most appropriate description of the transplanted tissue if a 25- year-old woman with end-stage renal failure receives a kidney for transplantation from her identical twin sister? A. Autograft B. Allograft C. Monograft D. Isograft E. Xenograft.

ANSWER IS D An isograft is an organ or tissue obtained from a donor genetically identical to the recipient.

1102. What is the most appropriate description of the transplanted tissue if a woman undergoes renal transplantation whose kidney had been harvested from a patient with confirmed brainstem death? A. Autograft B. Allograft C. Monograft D. Isograft E. Xenograft.

ANSWER IS B An allograft (or homograft) is an organ or tissue transplanted from a donor of the same species but different genetic make-up.

1103. What is the most appropriate description of the transplanted tissue if a 16 year-old boy who suffers burns to his chest and undergoes a partial-thickness skin graft from his thigh to cover part of his chest? A. Autograft B. Allograft C. Monograft D. Isograft E. Xenograft.

Prepared by Dr: Mohammed Musa Brema Idress – My best wishes Page 488

ANSWER IS A An autograft is tissue taken from one site and grafted to another site on the same person.

Complications following renal transplant Renal transplantation is widely practiced. The commonest technical related complications are related to the ureteric anastomosis. The warm ischaemic time is also of considerable importance and graft survival is directly related to this. Long warm ischaemic times increase the risk of acute tubular necrosis which may occur in all types of renal transplanation and provided other insults are minimized, will usually recover. Organ rejection may occur at any phase following the transplantation process.

Immunological complications - Types of organ rejection 1) Hyperacute. This occurs immediately through presence of pre formed antibody (such as ABO incompatibility). 2) Acute. Occurs during the first 6 months and is usually T cell mediated. Usually tissue infiltrates and vascular lesions. 3) Chronic. Occurs after the first 6 months. Vascular changes predominate.

Hyperacute Renal transplants are most susceptible to this process. Risk factors include major HLA mismatch and ABO incompatibility. The rejection occurs almost immediately and the macroscopic features may become manifest following completion of the vascular anastomosis and removal of clamps. The kidney becomes mottled, dusky and the vessels will thrombose. The only treatment is removal of the graft, if left in situ it will result in abscess formation.

Acute All organs may undergo acute rejection. Mononuclear cell infiltrates predominate. All types of transplanted organ are susceptible and it may occur in up to 50% cases. Most cases can be managed medically.

Chronic Again all transplants with HLA mismatch may suffer this fate. Previous acute rejections and other immunosensitizing events all increase the risk. Vascular changes are most prominent with myointimal proliferation leading to organ ischaemia. Organ specific changes are also seen such as loss of acinar cells in pancreas transplants and rapidly progressive coronary artery disease in cardiac transplants.

1104. A 34 year old lady presents with a long standing offensive discharge from the ear and on examination is noted to have a reduction in her hearing of 40 decibels compared to the opposite side. What is the most likely cause? A. Otosclerosis B. Acute otitis media C. Cholesteatoma

Prepared by Dr: Mohammed Musa Brema Idress – My best wishes Page 489

D. Long standing perforation of the pars tensa E. Otitis externa

ANSWER IS C The combination of long history, offensive discharge and hearing loss is strongly suggestive of cholesteatoma.

1105. A dentist treating a woman with rheumatoid arthritis for recurrent episodes of dental sepsis notices that both parotid and submandibular glands are symmetrically enlarged. What is the most likely diagnosis? A. Sjogren's syndrome B. Mikulicz's syndrome C. Alcoholism D. Liver cirrhosis E. Metastatic cancer

ANSWER IS A Sjogren's is associated with autoimmune disorders. Mikulicz's is similar but there is no sicca or arthritis.

1106. A 29 year old man has had a sore throat for the past 5 days. Over the past 24 hours he has notices increasing and severe throbbing pain in the region of his right tonsil. He is pyrexial and on examination he is noted to have a swelling of this area. What is the most likely cause? A. Tonsillar cancer B. Lymphoma C. Quinsy D. Glandular fever E. Common cold

ANSWER IS C Unilateral swelling and fever is usually indicative of quinsy. Surgical drainage usually produces prompt resolution of symptoms.

1107. A 42 year old singer is admitted for a thyroidectomy. Post operatively she is only able to make a gargling noise. Her voice sounds breathy. What is the most likely explanation? A. Bilateral inferior laryngeal nerve injury B. Unilateral inferior laryngeal nerve injury C. Unilateral superior laryngeal nerve injury D. Bilateral superior laryngeal nerve injury E. Lacunar infarct

ANSWER IS B. This patient has diplophonia which causes a gargling sound. This is associated with dysphagia. This can also be caused by a vagus nerve lesion, but the recurrent laryngeal nerve is more at risk of damage.

Prepared by Dr: Mohammed Musa Brema Idress – My best wishes Page 490

1108. A 53 year old man presents with a mass lesion slightly inferior to the tragus of his right ear. An FNA and USS are performed and a 2cm pleomorphic adenoma is diagnosed. What is the most appropriate course of action? A. USS guided core biopsy B. Radical parotidectomy C. Superficial parotidectomy D. Discharge E. MRI scanning of the region

ANSWER IS C Pleomorphic adenoma of the parotid = surgical excision Pleomorphic adenomas usually benign tumours. However, they will enlarge over time and a proportion can undergo malignant transformation. Therefore, all pleomorphic adenomas are excised and a superficial parotidectomy is generally the procedure of choice. The facial nerve is preserved. More recently, there has been a move towards partial superficial parotidectomy. However, complete resection of the lesion is still mandatory.

1109. An 8 year old boy presents with a lump in his neck. On examination, he is found to have a lump in the midline of the neck immediately below the hyoid which moves on swallowing. What is the most likely diagnosis? A. Benign lymphadenopathy B. Malignant lymphadenopathy C. Thyroglossal cyst D. Branchial cyst E. Dermoid cyst

ANSWER IS C Thyroglossal cysts are the commonest midline cysts in the neck found in children Both dermoid and thyroglossal cysts are often located in the midline. Dermoid cysts are usually found above the hyoid and do not move on swallowing. Thyroglossal cysts lie below the hyoid and do move on swallowing.

1110. A 14 year old boy presents with enlarged tonsils that meet in the midline. Oropharyngeal examination confirms this finding and you also notice peticheal haemorrhages affecting the oropharynx. On systemic examination he is noted to have splenomegaly. What is the most likely cause? A. Oral candidiasis B. Infection with Streptococcus pyogenes C. Infection with Rickettsia rickettsii D. Infection with Epstein Barr virus E. Infection with Staphylococcus aureus

ANSWER IS D A combination of pharyngitis and tonsillitis is often seen in glandular fever. Antibiotics containing penicillin may produce a rash when given in this situation, leading to a mistaken label of allergy. Infection with candidiasis can occur in

Prepared by Dr: Mohammed Musa Brema Idress – My best wishes Page 491 individuals with systemic illness of which splenomegaly may be a feature. However, its unlikely to only affect the tonsils.

1111. A 70 year old lady presents with a painless neck lump. There is a mass noted beneath the sternocleidomastoid muscle. There is a long history and somewhat unkindly her husband remarked on her rather noticeable halitosis. What is the most likely explanation? A. Branchial cyst B. Branchial fistula C. Pharyngeal pouch D. Thyroglossal cyst E. Dental caries

ANSWER IS C Usual history of regurgitation of undigested food or coughing at night. Associated with halitosis and throat infections.

1112 A 4 year old is brought to the general practitioner by her mother. She has been distressed with ear pain for the past 14 hours. She is constantly touching and pulling at her ear. Whilst she is sat in the waiting room her mother notices a discharge of foul smelling fluid from the ear, following which the pain resolves. What is the most likely cause? A. Acute otitis media B. Otitis externa C. Cholesteatoma D. Mastoiditis E. Otosclerosis

ANSWER IS A In young children acute suppurative otitis media is a common condition. Rupture of the tympanic membrane is a rare, but recognized complication.

1113. A 40 year old woman presents as an emergency with a painful mass underneath her right mandible. The mass has appeared over the previous week with the pain worsening as the lump has increased in size. On examination, there is a 4 cm mass underneath her mandible, there is no associated lymphadenopathy. What is the most likely diagnosis? A. Submandibular gland cancer B. Submandibular gland calculus C. Carotid body tumour D. Papilloma of Wharton’s duct E. Carotid body aneurysm

ANSWER IS B The sub mandibular gland is the most common site for salivary calculi. Patients will usually complain of pain, which is worse on eating. When the lesion is located

Prepared by Dr: Mohammed Musa Brema Idress – My best wishes Page 492 distally the duct may be laid open and the stone excised. Otherwise the gland will require removal

1114. A 46-year-old woman is referred to endocrine surgery for a possible thyroidectomy. She has a tender neck swelling. Blood results are as follows: TSH <0.1 mU/l T4 188 nmol/l Hb 14.2 g/dl Plt 377 * 10^9/l WBC 6.4 * 10^9/l ESR 65 mm/hr Technetium thyroid scan shows decreased uptake globally What is the most likely diagnosis? A. Sick thyroid syndrome B. Acute bacterial thyroiditis C. Hashimoto's thyroiditis D. Subacute thyroiditis E. Toxic multinodular goiter

ANSWER IS D This patient does not need surgery! Subacute thyroiditis is suggested by the tender goiter, hyperthyroidism and raised ESR. The globally reduced uptake on technetium thyroid scan is also typical. This should resolve without any active intervention.

1115. Which of the following statements relating to branchial cysts is untrue? A. The greater auricular nerve may be divided during excision B. They typically occur in young adults C. They move upwards on swallowing D. They are rare over the age of 40 years E. They are usually located in the anterior triangle of the neck

ANSWER IS C Nerves at risk during branchial cyst excision: Mandibular branch of facial nerve, greater auricular nerve and accessory nerve.

They do not move on swallowing. They should be diagnosed with caution in those aged >40 years, as lumps in this age group may in fact be metastatic disease from oropharyngeal cancer.

1116. A 30 year old lady is found to have a thyroglossal cyst that has been recently infected and the patient requests treatment. What is the most appropriate course of action? A. Surgical treatment with resection of cyst, associated track, central portion of the hyoid and wedge of tongue muscle behind the hyoid B. Excision of the cyst alone with ligation of the track C. Laying open of the track and excision of the cyst D. Phenolization of the cyst

Prepared by Dr: Mohammed Musa Brema Idress – My best wishes Page 493

E. Marsupialization of the cyst

ANSWER IS A Recurrence following attempted resection of thyroglossal cysts is very common. Complete excision of the cyst and its track and origin is mandatory (Sis trunks procedure).

1117. A 22 year old female attends clinic after noticing a painless neck lump. On examination she is noted to have bilateral thyroid masses and multicentric nodes near the base of the thyroid. Her corrected Ca is 2.18. What is the most likely diagnosis? A. Sporadic medullary carcinoma of the thyroid B. Medullary carcinoma of the thyroid associated with multiple endocrine neoplasia C. Follicular thyroid carcinoma D. Anaplastic thyroid carcinoma E. Toxic nodular goiter

ANSWER IS B Medullary thyroid cancer is a tumour of the parafollicular cells of the thyroid. Less than 10% of thyroid cancers are of this type. Patients typically present in children or young adults. Diarrhoea occurs in 30% of cases. Toxic nodular goiter is very rare. In sporadic medullary thyroid cancer, patients typically present with a unilateral solitary nodule and it tends to spread early to the neck lymph nodes. In association with multiple endocrine neoplasia (MEN) syndromes, medullary thyroid cancers are always bilateral and multicentric. It may be the presenting feature in MEN 2a and 2b; almost all MEN 2a patients develop medullary thyroid carcinoma.

1118. A 42 year old singer is admitted for a thyroidectomy. Post operatively the patient develops stridor and is unable to speak. What is the most likely underlying cause? A. Bilateral superior laryngeal nerve injury B. Unilateral inferior recurrent laryngeal nerve injury C. Unilateral superior laryngeal nerve injury D. Bilateral inferior recurrent laryngeal nerve injury E. Hypoglossal nerve injury

ANSWER IS D This patient has aphonia due to bilateral damage to the recurrent laryngeal nerve.

1119. A 10 year old girl presents with epistaxis. From which of the following regions is the bleeding most likely to originate? A. Posterior nasal space B. Alar rim C. Kiesselbach's plexus D. Cribriform plate E. None of the above

Prepared by Dr: Mohammed Musa Brema Idress – My best wishes Page 494

ANSWER IS C Kiesselbachs plexus has an arterial supply derived from both the internal and external carotid arteries and is the commonest area for bleeding in idiopathic epistaxis.

1120. A 56 year old man presents with a painless swelling in the upper part of the anterior triangle of his neck. On examination a mass lesion involving the sub mandibular gland is identified. On CT scanning this is shown to be a solid lesion. There is no regional lymphadenopathy. Two fine needle aspirates have failed to be diagnostic. Which of the following is the most appropriate management option? A. Sub mandibular gland excision B. Incisional biopsy of the mass C. Manage conservatively and repeat the CT scan in 6 months D. Sub mandibular gland excision and radical neck dissection E. Diagnostic excision of the superficial lobe of the submandibular gland

ANSWER IS A There is a 50% risk that this lesion is malignant (in some series up to 70%). Therefore the gland should be excised entirely. At this stage a radical neck dissection is not justified.

1121. A 36-year-old woman who presented with a goiter is diagnosed with autoimmune thyroiditis. Which one of the following types of thyroid cancer is she predisposed to developing? A. Anaplastic B. Lymphoma C. Medullary D. Follicular E. None of the above

ANSWER IS B Hashimoto's thyroiditis is associated with thyroid lymphoma Rarely, a patient with Hashimotos thyroiditis may develop a lymphoma of the thyroid gland. The exact aetiology of thyroid gland lymphoma is unknown. Hashimotos thyroiditis is a definite risk factor. It is possible that the lymphoma may represent the expansion of a clone of immortalized intrathyroidal lymphocytes.

Lymphoma of the gland is characterized by rapid thyroid growth despite T4 therapy. Whilst adenocarcinoma of the thyroid may occur in association with thyroiditis there are no studies, to date, showing a correlation between these two conditions. The commonest sequelae of thyroiditis are hypothyroidism.

1122. An elderly diabetic male presents with a severe deep seated otalgia and a facial nerve palsy, he has completed a course of amoxycillin with no benefit. What is the most likely diagnosis? A. Malignant otitis externa B. Otosclerosis

Prepared by Dr: Mohammed Musa Brema Idress – My best wishes Page 495

C. Acoustic neuroma D. Meniers disease E. Viral illness

ANSWER IS A A combination of severe otalgia and facial nerve palsy in a diabetic should raise suspicion of malignant otitis externa. This is a condition caused by pseudomonas. It commences as otitis externa and then progresses to involve the temporal bone. Spread of the disease outside the external auditory canal occurs through the fissures of Santorini and the osseocartilaginous junction.

1123. A 25 year old lady presents with an swelling located at the anterior border of the sternocleidomastoid muscle. The swelling is intermittent and on examination it is soft and fluctuant. What is the most likely diagnosis? A. Thyroglossal cyst B. Branchial cyst C. Carotid body tumour D. Infection with Bartonella E. Dental abscess

ANSWER IS B Branchial cysts are remnants of the branchial cleft. They may become infected.

1124. A 4 year old child presents with a 4-5 day history of feeling generally unwell and also of having a sore throat. On examination, there is marked cervical lymphadenopathy, the oropharynx is covered with a thick grey membrane which bleeds following attempted removal. What is the most likely diagnosis? A. Acute streptococcal pharyngitis B. Infection with Epstein Barr virus C. Diphtheria D. Ludwig’s angina E. Ingestion of caustic soda

ANSWER IS C Infection with diphtheria classically causes a systemic illness that lasts several days. The tonsils or pharynx can be covered in a thick grey membrane which bleeds on attempted removal. There is often quite marked cervical adenopathy and some individuals can have abulls neck appearance. Death can occur through airway compromise, which is why the often described attempted removal of the pseudomembrane so beloved of examiners, is, in practice rather a foolish thing to attempt in a young child!

1125. A 40 year old professional singer is admitted for a right thyroid lobectomy. Post operatively, she is unable to sing high notes. Which of the following muscles is likely to demonstrate impaired function? A. Thyroarytenoid B. Posterior cricoarytenoid

Prepared by Dr: Mohammed Musa Brema Idress – My best wishes Page 496

C. Cricothyroid D. Thyrohyoid E. Lateral cricoarytenoid

ANSWER IS C The most likely injury is to the superior thyroid nerve which innervates the cricothyroid muscle. Since this tenses the vocal cords, singing high notes becomes a problem.

1126. Which of the following does not cause parotid enlargement? A. Liver cirrhosis B. Myxoedema C. Amiodarone D. Tuberculosis E. Sjogrens syndrome

ANSWER IS C Drugs commonly implicated in parotid gland enlargement include: Thiouracil, isoprenaline, phenylbutazone, high oestrogen dose contraceptive pills.

1127. At which of the following time frames is secondary haemorrhage most likely to occur following tonsillectomy? A. Between 5 and 10 days after surgery B. During the first 6 hours after surgery C. Between 6 and 12 hours after surgery D. Upon resumption of normal feeding E. Between 2 and 3 days post operatively

ANSWER IS A Haemorrhage in the first 6 hours after surgery is termed reactionary haemorrhage. Feeding does not increase the risk and may actually lower the risks of infection developing.

1128. A 4 year old child is brought to the clinic by his father. They are concerned because the child has been noted to have a small epithelial defect anterior to the left ear and is has been noted to discharge foul smelling material for the past 2 days. What is the most likely explanation? A. Otitis externa B. Otitis media C. Preauricular sinus D. Cholesteatoma E. Long standing pars flaccida perforation

ANSWER IS C Preauricular sinuses that are deeper may accumulate secretions and produce foul smelling discharge.

Prepared by Dr: Mohammed Musa Brema Idress – My best wishes Page 497

1129. A 32 year old lady is referred to the clinic with recurrent infections and abscesses in the neck. On examination, she has a midline defect with an overlying scab, which moves upwards on tongue protrusion. What is the most likely diagnosis? A. Branchial cyst B. Branchial fistula C. Thyroglossal cyst D. Sebaceous cyst E. Dermoid cyst

ANSWER IS C Thyroglossal cyst is always located in the midline as it is this route that the thyroid takes during its embryological descent. Its connection with the foramen caecum means it will move on tongue protrusion.

1130. A 8 year old boy with recurrent attacks of otitis media is suspected of developing a glue ear. If his sound conduction is tested, which of the following is most consistent with a unilateral middle ear effusion? A. Negative Rinne's test on the ipsilateral side B. Positive Rinne's test on the ipsilateral side C. Positive Weber’s and Rinnes tests on the ipsilateral side D. Positive Rinne's test on the contralateral side E. Negative Weber’s test only on the contralateral side

ANSWER IS A Rinne's test will localize to the affected side (i.e. it is negative in conductive deafness). In a positive Rinne's test sound heard by air conduction is better than that conveyed by bone conduction. Reduction of both air and bone conduction in equal measure is a feature of sensorineural hearing loss.

1131. A 43 year old lady is diagnosed as having a malignant lesion in the inferior aspect of her left breast. There is palpable axillary lymphadenopathy. What is the most appropriate course of action? A. Mastectomy and axillary node clearance B. Wide local excision and axillary node clearance C. Wide local excision and sentinel lymph node biopsy D. Image guided fine needle aspiration of the axillary nodes E. CT scanning of the chest, abdomen and pelvis

ANSWER IS D Where axillary nodal involvement is suspected from the outset it is important to establish whether this is the case prior to surgery. This is because, if axillary metastatic disease is present then the correct management would be an axillary node clearance and this is irrespective of the surgical plans for the breast primary. In the case of breast cancer, image guided FNAC is acceptable as it is accurate and if carcinoma cells are identified at FNA then axillary node clearance can be performed.

Prepared by Dr: Mohammed Musa Brema Idress – My best wishes Page 498

If FNAC is negative then a sentinel node biopsy should accompany excision of the primary tumour. Where the axilla is clinically clear on palpation and imaging then a sentinel lymph node biopsy should accompany excision of the primary tumour.

1132. A 44 year old lady presents with a mass lesion in the upper outer quadrant of the left breast. On clinical examination she has a 2cm mass lesion which on core biopsy is demonstrated to have invasive ductal carcinoma. An FNA of a bulky axillary lymph node contains malignant cells. What is the correct course of action? A. Wide local excision and axillary node clearance B. Radical mastectomy and axillary node clearance C. Simple mastectomy and sentinel node biopsy D. Wide local excision and sentinel node biopsy E. Excision biopsy and sentinel node biopsy

ANSWER IS A Although the primary lesion is small enough for breast conserving surgery, the presence of overt axillary lymph node metastasis will attract a recommendation for axillary node clearance. Note that an excision biopsy is not appropriate for malignant cases.

1133. A 43 year old lady has recently undergone a wide local excision and sentinel lymph node biopsy for carcinoma of the breast. Of the factors listed below, which will provide the most important prognostic information? A. Mitotic number B. Grade C. Nodal status D. Size E. Oestrogen receptor status

ANSWER IS C Nodal status is the single most important prognostic factor in breast cancer. Nodal status is important because it serves as a marker of tumour metastatic potential. This translates to survival advantages of up to 40% at five years. Both grade and size are of secondary importance as they both less concerning in the absence of nodal involvement.

1134. A 53 year old lady has recently entered the UK from the far East and she undergoes a thyroidectomy for a very large multinodular goiter with retrosternal extension. The operation is relatively straightforward otherwise. The patient is returned to the recovery area and suddenly becomes profoundly dyspnoeic and hypoxic. On examination, the wound is healthy and dry and the drain empty. What is the most likely cause? A. Contained haematoma B. Unilateral recurrent laryngeal nerve injury C. Unilateral superior laryngeal nerve injury D. Tracheomalacia

Prepared by Dr: Mohammed Musa Brema Idress – My best wishes Page 499

E. Hypocalcaemia

ANSWER IS D With long standing very large goiter, there is a risk of tracheomalacia. In the UK this is very rarely seen. However, patients from other countries can have enourmous goiter’s and this then becomes a concern. The fact that the wound is satisfactory makes a bleed less likely and whilst total thyroidectomy can be a risk factor for hypocalcaemia, this is a little early.

1135. A 45 year old lady presented with a 2cm mobile breast mass. A mammogram is indeterminate (M3), USS shows benign changes (U2), clinical examination is also indeterminate (P3). What is the next most appropriate course of action? A. Re-assure and discharge B. Fine needle aspiration cytology C. Excision biopsy D. Image guided core biopsy E. Wide local excision

ANSWER IS D All discrete breast lumps, including those that seem benign, should have a confirmed histological diagnosis. In this case, a core biopsy has not yet been performed. This may yield a diagnosis that is concordant with imaging findings. In which case, this concludes the investigative process (if benign). If it remains unclear, excision biopsy will be needed.

1136. A 27 year old lady presents with a breast lump. She has previously undergone a breast augmentation with an implant. What is the imaging technique of choice? A. Ultrasound B. CT scanning C. MRI D. PET CT E. Mammography

ANSWER IS A Unless there are concerns about implant rupture, the imaging of a breast lump in a young patient with implants would be USS initially. If this is not conclusive then MRI should be performed.

Breast imaging Breast imaging is undertaken with a combination of USS and mammography in most women presenting with a palpable lump. In younger patients, the denser breasts encountered in this group may make mammography less informative.

In women who present with breast implants with a palpable lump, the imaging of choice remains the same. However, specialized imaging techniques may be needed

Prepared by Dr: Mohammed Musa Brema Idress – My best wishes Page 500 to obtain optimal mammographic views. Where there are specific concerns about a breast implant, rather than a lump, the imaging modality of choice is MRI scanning. MRI scanning may be beneficial in screening younger patients with a family history and also in patients with lobular cancers who are being considered for breast conserving surgery.

1137. A 52 year old woman with known Hashimotos thyroiditis presents with a neck swelling. She describes it as rapidly increasing in size over 3 months and she complains of dysphagia to solids. On examination, there is an asymmetrical swelling of the thyroid gland. What is the most likely diagnosis? A. Anaplastic thyroid cancer B. Follicular thyroid cancer C. Papillary thyroid cancer D. Lymphoma E. Toxic adenoma

ANSWER IS D Thyroid lymphoma (Non Hodgkin's B cell lymphoma) is rare. It should be considered in patients with a background of Hashimoto's thyroiditis and a rapid growth in size of the thyroid gland. Diagnosis can be made with core needle biopsy; however an incisional biopsy may be needed. Radiotherapy is the main treatment option.

1138. A 34 year old female presents with a thyroid nodule. She has a family history of thyroid disease and both her sisters have undergone total thyroidectomies. Her past medical history includes hypertension which has been difficult to manage. What is the most likely underlying thyroid lesion? A. Papillary thyroid cancer B. Follicular adenoma C. Medullary thyroid cancer D. Anaplastic thyroid cancer E. Lymphoma of the thyroid

ANSWER IS C This is a typical scenario for medullary carcinoma in which a phaeochromocytoma may also be present. It may be inherited in an autosomal dominant fashion and affected family members may be offered prophylactic thyroidectomy.

1139. A 29 year old man presents to the clinic with a recurrent thyroid cyst. It has been drained now on three occasions. Each time the cyst is aspirated and cytology is reassuring. What is the most appropriate course of action? A. Reassure and discharge B. Resection of the ipsilateral thyroid lobe C. Enucleation of the cyst D. Total thyroidectomy E. Radio-isotope scan

ANSWER IS B

Prepared by Dr: Mohammed Musa Brema Idress – My best wishes Page 501

Persist refilling cysts may be associated with a well differentiated tumour and should be removed by lobectomy.

Management of thyroid disease - Lesion Management 1. Large multinodular goiter → Surgery for pressure symptoms. Total thyroidectomy is treatment of choice 2. Toxic nodule → Hemithyroidectomy 3. Follicular lesion (THY 3f) → Hemithyroidectomy to establish diagnosis 4. Papillary thyroid cancer → Total thyroidectomy and central compartment nodal dissection (extended lymphadenectomy as required) 5. Follicular thyroid cancer → Total thyroidectomy (usually completion as already had hemithyroidectomy) 6. Anaplastic thyroid cancer → Palliative radiotherapy 7. Medullary thyroid cancer →Total thyroidectomy (screen for other MEN tumours) 8. Lymphoma of the thyroid → Consider core biopsy 9. Persistent refilling cysts → Hemithyroidectomy 10. Graves disease with significant eye signs →Total thyroidectomy 11. Graves disease without significant eye signs → Patient choice (radioiodine Vs surgery)

1140. A 56 year old woman has undergone a wide local excision and axillary node sample. The histology shows a 2.5cm invasive ductal carcinoma grade 1 which is completely excised. None of the axillary lymph nodes show evidence of metastatic disease. What is the most appropriate course of action? A. Arrange routine surgical follow up B. Completion mastectomy C. Radiotherapy to breast and axilla D. Administration of tamoxifen E. Radiotherapy to breast alone

ANSWER IS E This woman has a good prognosis. However, irradiation of the breast is mandatory after breast cancer has been treated using breast conserving surgery. Note that only the breast needs to be irradiated in this case and not the axilla.

1141. A 20 year old lady presents with a lump in the upper outer aspect of her right breast. On examination, she has a firm mobile mass in the upper outer quadrant of her right breast. What is the most likely underlying disease process?

A. Atypical ductal hyperplasia B. Breast cyst C. Duct ectasia D. Fibroadenoma E. Fat necrosis

Prepared by Dr: Mohammed Musa Brema Idress – My best wishes Page 502

ANSWER IS D Fibroadenomas account for 60% of discrete breast lesions in the 18-25 year age group. They are mobile lesions. Core biopsy should be performed in lesions measuring more than 4cm in diameter.

1142. An 82 year old woman from a nursing home is admitted to the orthopaedic ward with a hip fracture. She is acutely confused and agitated. Her Calcium is 2.95 (elevated) and her PTH is 12 (elevated). What is the best course of action? A. Administration of intravenous dextrose B. Administration of intravenous crystalloid C. Sestamibi scan D. Administration of intravenous colloid E. Emergency parathyroidectomy

ANSWER IS B Whilst she may have an underlying parathyroid lesion, resuscitation should occur first. Initially, this should be with crystalloid.

1143. A 45 year old man is referred to the breast clinic with gynaecomastia. He takes the drugs listed below. Which is least likely to be the cause of his symptoms? A. Spironolactone B. Carbimazole C. Chlorpromazine D. Cimetidine E. Methyldopa

ANSWER IS B Mnemonic for drugs causing gynaecomastia: DISCO 1. D igitalis 2. I soniazid 3. S pironolactone 4. C imetidine 5. O estrogen Mnemonic for causes of gynaecomastia: METOCLOPRAMIDE 1. M etoclopramide 2. E ctopic oestrogen 3. T rauma skull/tumour breast, testes 4. O rchitis 5. C imetidine, Cushing’s syndrome 6. L iver cirrhosis 7. O besity 8. P araplegia 9. R A 10. A cromegaly 11. M ethyldopa 12. I soniazid 13. D igoxin

Prepared by Dr: Mohammed Musa Brema Idress – My best wishes Page 503

14. E thionamide - Carbimazole is not associated with gynaecomastia.

1144. A 72 year old female is found to have a malignant lesion in her left arm. She had a mastectomy of the left breast 10 years ago and has chronic lymph oedema of the left arm. What is the most likely cause of the malignancy? A. Lymphangiosarcoma B. Lymphoma C. Myeloma D. Angiomyolipoma E. Giant cell tumour

ANSWER IS A Lymphangiosarcoma is a rare condition arising as a result of chronic oedema. It is an aggressive malignancy.

1145. A 39 year old lady presents with a mass lesion in her right breast. Clinical examination, biopsy and imaging confirm a 2.5 cm lesion in the upper inner quadrant of her right breast and a 1.5 cm lesion at the central aspect of the same breast. Her axilla shows lymphadenopathy and a fine needle aspirate from the node shows malignant cells. What is the best course of action? A. Simple mastectomy and axillary node clearance B. Wide local excisions of the two lesions and axillary node clearance C. Simple mastectomy and sentinel node biopsy D. Wide local excisions of the two lesions and sentinel node biopsy E. Radical mastectomy and axillary node clearance

ANSWER IS A Multifocal breast cancer is best treated with mastectomy A combination of established axillary disease and multifocal invasive lesions attracts an indication for mastectomy and axillary clearance. A radical mastectomy is less frequently indicated in modern surgical practice, disease that is locally advanced is often best down staged using medical therapy, rather than embarking on the operations for breast cancer that were first popularized over 100 years ago.

1146. A 55 year old women presents with nipple discharge. On examination, she has a slit like retraction of the nipple in the centre of this area is a small amount of cheese like material. No discrete mass lesion is palpable in the underlying breast. Which of the factors listed below is most strongly associated with developing this condition? A. BRCA 1 positivity B. Smoking C. Alcoholism D. Obesity E. Type II diabetes mellitus

ANSWER IS B

Prepared by Dr: Mohammed Musa Brema Idress – My best wishes Page 504

Smoking is a major risk factor for duct ectasia

1147. A 56 year old man is investigated with an abdominal CT scan for a change of bowel habit towards constipation. It shows no colonic lesions. However, a right sided adrenal lesion is noted and measures 2.5cm in diameter. What is the most appropriate course of action? A. Arrange an MRI of the adrenal gland B. Arrange an adrenal USS C. Arrange an image guided core biopsy D. Undertake an adrenalectomy E. Arrange a hormonal assay

ANSWER IS E The vast majority of small adrenal lesions are incidental, benign and non functioning adenomas. Apart from minimal workup, no further investigation is needed. Of note, if there are concerns about malignancy, the only surgical option is adrenalectomy.

1148. A 59 year old man is referred with symptoms of dysphagia. On examination, he has a large goiter and on imaging there is significant retrosternal extension and features of a multinodular goiter. What is the most appropriate course of action? A. Total thyroidectomy B. Subtotal thyroidectomy C. Thyroid lobectomy D. Radio-iodine E. Conservative management

ANSWER IS A Subtotal thyroidectomy is no longer routinely undertaken in this group.

1149. A 58 year old male is referred to endocrinology clinic for a parathyroidectomy by the F1 in medicine. His corrected calcium is 2.85 (2.2-2.6), PTH 7.5 (3-7) and 24 hour urinary calcium is 1.5 (2.5-7.5). What is the diagnosis? A. Primary hyperparathyroidism B. Secondary hyperparathyroidism C. Tertiary hyperparathyroidism D. Familial hypocalciuric hypercalcaemia E. Hypercalacemia associated with malignancy

ANSWER IS D This F1 should have spoken to his senior. This patient has familial hypocalciuric hypercalcaemia, which requires no further action. A calcium to creatinine clearance ratio of <0.01 will confirm this diagnosis.

1150. A 50 year old lady is commenced on tamoxifen for the treatment of an oestrogen receptor positive breast cancer. Which of the following malignancies are associated with tamoxifen use? A. Adenocarcinoma of the colon

Prepared by Dr: Mohammed Musa Brema Idress – My best wishes Page 505

B. Hodgkin’s lymphoma C. Adenocarcinoma of the lung D. Ovarian cancer E. Endometrial cancer

ANSWER IS E Tamoxifen is an oestrogen receptor antagonist in breast tissues. However, at other sites, such as the endometrium it may act as an agonist. Hence the reason for increasing risk of endometrial cancer.

Tamoxifen Synthetic partial oestrogen agonist, acts primarily by binding to the oestrogen receptor. Half life of 7 days, takes 4 weeks for drug to reach plasma steady state. Should usually be considered in patients with oestrogen receptor positive tumours (alternative agents may be preferred in some groups). Although antagonistic with respects to breast tissue tamoxifen may serve as an agonist at other sites. Therefore risk of endometrial cancer is increased, preservation of bone density and decreased cardiovascular risks.

Climateric side effects are common, 3% stop taking the drug because of these. Aromatase inhibitors are an alternative class of drugs, these work by blocking the peripheral aromatization of androgens (post menopausal women produce oestrogens in this way). They may treat cancers for which tamoxifen is no longer effective.

1151. A 48 year old lady presents with discomfort in the right breast. On examination, she has a discrete, soft, fluctuant area in the upper outer quadrant of her right breast. A mammogram is performed and a halo sign is seen by the radiologist. What is the most likely explanation for this process? A. Breast cyst B. Fibroadenoma C. Radial scar D. Atypical ductal hyperplasia E. Ductal carcinoma in situ

ANSWER IS A Lesions such as breast cysts compress the underlying fat and produce a radiolucent area (halo sign). If symptomatic, these cysts should be aspirated.

1152. Which investigation is best for initial assessment of recurrence of follicular carcinoma of the thyroid? A. Free T4 B. Thyroid stimulating hormone C. Scintigraphy D. Serum thyroglobulin E. USS thyroid gland

Prepared by Dr: Mohammed Musa Brema Idress – My best wishes Page 506

ANSWER IS D Elevated thyroglobulin levels raises suspicion of recurrence.

1153. A 33 year old lady attends the clinic with a 3 month history of palpitations and irritability. Her thyroid function, PTH and calcium are measured: Thyroid function Free T4 40 pmol/L TSH < 0.1 miu/L Free T3 25 p mol/L (normal 3.5-7.7 p mol/L) PTH 10pg/ml (normal 10-55pg/ml) (Normal values listed in reference range link) What is the most likely diagnosis? A. Hypothyroidism B. Hyperthyroidism C. Hypoparathyroidism D. Hyperparathyroidism E. Euthyroid

ANSWER IS B Elevated T4 and suppressed TSH make this the most likely diagnosis. The PTH level is normal.

1154. A 48 year old lady undergoes a redo thyroidectomy for a multinodular goiter. 24 hours post operatively she develops oculogyric crises and diffuse muscle spasm. What is the most appropriate course of action? A. Arrange a CT scan of the head B. Administration of intravenous magnesium C. Administration of intravenous calcium D. Return the patient to theatre for wound exploration E. Call an anaesthetist to paralyze and ventilate the patient

ANSWER IS C She has most likely developed hypocalcaemic tetany and will require immediate calcium supplementation.

1155. A 32 year old lady has previously undergone a wide local excision and axillary node clearance (5 nodes positive) for an invasive ductal carcinoma. It is oestrogen receptor negative, HER 2 positive, vascular invasion is present. She has a lesion suspicious for metastatic disease in the left lobe of her liver. Of the agents listed below, which is the most likely to be beneficial in this setting? A. Cetuximab B. Bevacizumab C. Trastuzumab D. Basiliximab E. Imatinib

ANSWER IS C

Prepared by Dr: Mohammed Musa Brema Idress – My best wishes Page 507

This lady's young age, coupled with ER negativity and extensive nodal disease with suspicion of metastatic disease makes her a candidate for treatment with trastuzumab (herceptin).

1156. Which blood test is most likely to be helpful in the following situation? A 20 year old lady has undergone a total thyroidectomy for a well differentiated papillary carcinoma. She attends clinic and is well and the surgeon wishes to screen for disease recurrence. A. Calcitonin B. Antibodies to TSH receptor C. Antibodies to parafollicular c cells D. Thyroid peroxidase antibodies E. Thyroglobulin antibodies

ANSWER IS E Antibodies to thyroglobulin, the major constituent of colloid and precursor of thyroid hormones may be elevated in those with metastatic or recurrent thyroid cancer. Results may be erroneous in those with other thyroid disorders.

1157. A 23 year old lady has Graves disease that has relapsed on stopping anti thyroid drugs, radioiodine is offered as the next treatment by the endocrinologists. Which statement is false? A. Close contact with children is not permitted for up to 4 weeks following treatment B. 15% of patients with opthalmopathy will see worsening of eye signs C. Symptomatic improvement takes 6-8 weeks D. Up to 80% of patients will become hypothyroid E. It increases the risk of parathyroid carcinoma

ANSWER IS E - Radio-iodine- may worsen opthalmopathy, contraindicated in pregnancy and those wishing to conceive within 6 months. - Radioiodine vs. Surgery:- Surgery - Symptomatic improvement within 10 days - No effect on opthalmopathy - Risk of damage to adjacent anatomical structures - No restrictions on contact Radioiodine - Symptomatic improvement takes up to 2 months - Eye signs may worsen - No risk of anatomical damage - No contact with children for 4 weeks

1158. Which of the blood tests listed below is most likely to be abnormal in a 33 year old lady presents with a recently diagnosed goiter and a diagnosis of Hashimotos thyroiditis is suspected?

Prepared by Dr: Mohammed Musa Brema Idress – My best wishes Page 508

A) TSH receptor antibodies B) Thyroid peroxidase antibodies C) Thyroglobulin antibodies D) Antibodies to parafollicular c cells E) Calcitonin

Antibodies to thyroid peroxidase are found in most patients with Graves disease or Hashimotos thyroiditis.

1159. A 28 year old female presents with a painless lump in the upper outer quadrant of her left breast. Imaging using ultrasound is indeterminate (U3). Two core biopsies have now been performed and both show normal breast tissue (B1). What is the most appropriate course of action? A) Arrange for imaging surveillance at 3 monthly intervals B) Undertake a wide local excision of the lump C) Reassure the patient and discharge D) Undertake an excision biopsy of the lump E) Arrange for a breast CT scan

ANSWER IS D Wide local excision and excision biopsy are different procedures. The imaging and biopsy results are not concordant. At this stage an excision biopsy is the safest option. CT scanning is seldom helpful in breast surgery.

1160. A 19 year old male presents with bilateral gynaecomastia, poor vision and nipple discharge. Which of the following blood tests is most likely to be abnormal? A) Oestrogen B) Testosterone C) β HCG D) Prolactin E) Calcitonin

ANSWER IS D A combination of nipple discharge, gynaecomastia and poor vision may well be associated with a prolactinoma. The poor vision results from compression of the optic chiasm resulting in bi temporal hemianopia.

1161. A 46 year old man is admitted to hospital with a femoral shaft fracture that occurred suddenly whilst he was out walking his dog. On examination, there is no neurovascular deficit distal to the fracture site. He has a large firm nodule in the left lobe of the thyroid, there is no associated lymphadenopathy. What is the most likely underlying cause? A) Papillary thyroid cancer B) Follicular thyroid cancer C) Thyroid gland lymphoma D) Anaplastic thyroid cancer E) Medullary thyroid cancer

Prepared by Dr: Mohammed Musa Brema Idress – My best wishes Page 509

ANSWER IS B Follicular carcinomas may metastasise haematogenously (often to bone) where they may give rise to pathological fractures as in this case.

1162. A 52 year old woman presents with a neck swelling. Her GP reports that her TSH value is low at 0.01. A scintigraphy demonstrates a hot nodule. What is the most likely diagnosis? A) Graves disease B) Toxic adenoma C) Papillary thyroid cancer D) Hashimotos thyroiditis E) De Quervains thyroiditis

ANSWER IS B This lady has thyrotoxicosis (low TSH) and a hot solitary nodule indicating a toxic adenoma. Thyroid cancer rarely causes thyrotoxicosis or hot nodules.

1163. A 44 year old lady presents with a mass in the upper outer quadrant of her right breast. Imaging, histology and clinical examination confirm a 1.5cm malignant mass lesion with no clinical evidence of axillary nodal disease. What is the most appropriate treatment? A) Radical chemoradiotherapy alone B) Excision biopsy and sentinel node biopsy C) Wide local excision and sentinel node biopsy D) Mastectomy and axillary node sampling E) Wide local excision and axillary node sampling

ANSWER IS C Small peripheral breast cancers are well suited to breast conserving surgical techniques. A small peripheral lesion such as this would usually be suitable for breast conserving surgery. Since imaging and clinical examination is not suspicious for axillary disease, a sentinel lymph node biopsy should be performed. Axillary node sampling is a procedure that become obsolete with the introduction of sentinel node biopsy.

1164. Which of the following are not true of follicular thyroid cancer? A) They often appear to be encapsulated. B) Those with a Hurthle cell subtype have an excellent prognosis. C) Haematogenous metastasis is more common than in Papillary carcinoma. D) The overall mortality rate is 24%. E) Vascular invasion is seen in up to 60% of cases.

ANSWER IS B The Hurthle cell subtype have a worse prognosis.

1165. A 56 year old lady undergoes a mastectomy as treatment for multifocal ductal carcinoma in situ. Two weeks post operatively she attends the clinic and

Prepared by Dr: Mohammed Musa Brema Idress – My best wishes Page 510 complains of a diffuse swelling at the surgical site. On examination she has a large, fluctuant area underlying the mastectomy skin flaps. She is otherwise well. What is the most likely cause? A) Abscess B) Seroma C) Haematoma D) Disease recurrence E) Arteriovenous malformation

ANSWER IS B Seromas are very common after breast surgery. The exposed raw surfaces created during the elevation of the skin flaps are a common cause. Treatment usually involves percutaneous drainage under aseptic conditions.

1166. You are the specialist trainee in endocrinology clinic. The medical team have referred a man for a parathyroidectomy who has a corrected calcium of 2.82 (elevated) and a PTH of 11 (elevated). Which of the following is not an indication for parathyroidectomy? A) Nephrolithiasis B) Reduction in bone mineral density of the femoral neck, lumbar spine, or distal radius of more than 2.5 standard deviations below peak bone mass C) Age < 50 years D) Episode of life threatening hypercalcaemia E) None of the above

ANSWER IS E All of the situations listed are indications for parathyroidectomy. See below for more information.

Parathyroid glands and disorders of calcium metabolism Hyperparathyroidism ◆ Disease type A. Primary hyperparathyroidism - Hormone profile 1) PTH (Elevated) 2) Ca2+ (Elevated) 3) Phosphate (Low) 4) Urine calcium : creatinine clearance ratio > 0.01 - Clinical features 1) May be asymptomatic if mild 2) Recurrent abdominal pain (pancreatitis, renal colic) 3) Changes to emotional or cognitive state - Cause - Most cases due to solitary adenoma (80%), multifocal disease occurs in 10- 15% and parathyroid carcinoma in 1% or less B. Secondary hyperparathyroidism - Hormone profile

Prepared by Dr: Mohammed Musa Brema Idress – My best wishes Page 511

1) PTH (Elevated) 2) Ca2+ (Low or normal) 3) Phosphate (Elevated) 4) Vitamin D levels (Low) - Clinical features 1) May have few symptoms 2) Eventually may develop bone disease, osteitis fibrosa cystica and soft tissue calcifications - Cause - Parathyroid gland hyperplasia occurs as a result of low calcium, almost always in a setting of chronic renal failure C. Tertiary hyperparathyroidism - Hormone profile 1) Ca2+ (Normal or high) 2) PTH (Elevated) 3) Phosphate levels (Decreased or Normal) 4) Vitamin D (Normal or decreased) 5) Alkaline phosphatase (Elevated) - Clinical features 1) Metastatic calcification 2) Bone pain and / or fracture 3) Nephrolithiasis 4) Pancreatitis - Cause - Occurs as a result of ongoing hyperplasia of the parathyroid glands after correction of underlying renal disorder, hyperplasia of all 4 glands is usually the cause Differential diagnoses It is important to consider the rare but relatively benign condition of benign familial hypocalciuric hypercalcaemia, caused by an autosomal dominant genetic disorder. Diagnosis is usually made by genetic testing and concordant biochemistry (urine calcium : creatinine clearance ratio <0.01-distinguished from primary hyperparathyroidism).

Treatment:- A. Primary hyperparathyroidism - Indications for surgery 1) Elevated serum Calcium > 1mg/dL above normal 2) Hypercalciuria > 400mg/day 3) Creatinine clearance < 30% compared with normal 4) Episode of life threatening hypercalcaemia 5) Nephrolithiasis 6) Age < 50 years 7) Neuromuscular symptoms 8) Reduction in bone mineral density of the femoral neck, lumbar spine, or distal radius of more than 2.5 standard deviations below peak bone mass (T score lower than -2.5)

Prepared by Dr: Mohammed Musa Brema Idress – My best wishes Page 512

B. Secondary hyperparathyroidism - Usually managed with medical therapy. - Indications for surgery in secondary (renal) hyperparathyroidism: 1) Bone pain 2) Persistent pruritus 3) Soft tissue calcifications C. Tertiary hyperparathyroidism Allow 12 months to elapse following transplant as many cases will resolve The presence of an autonomously functioning parathyroid gland may require surgery. If the culprit gland can be identified then it should be excised. Otherwise total parathyroidectomy and re-implantation of part of the gland may be required.

1167. A 52 year old woman presents with a neck swelling. On examination she is noted to have single nodule on the thyroid gland. A CXR shows two mass lesions. What is the most likely cause? A. Papillary thyroid cancer B. Toxic adenoma C. Follicular thyroid cancer D. Anaplastic thyroid cancer E. Medullary thyroid cancer

ANSWER IS C A solitary nodule with signs of haematogenous spread indicates a follicular tumour. Note that papillary tumours tend to be multinodular and spread via the lymphatic system. Lymphatic spread from a papillary thyroid cancer is nearly always to neck nodes in the first instance and mediastinal lymphadenopathy is vanishingly rare. Lung lesions are typically synonymous with haematogenous metastasis of which a follicular lesion is the most likely culprit.

1168. A 48 year old lady with thyrotoxicosis is referred to the clinic, she was poorly controlled on carbimazole and has received orbital radiotherapy for severe proptosis. This has improved matters but she relapsed on stopping her carbimazole. What is the most appropriate course of action? A. Administration of radio-iodine B. Subtotal thyroidectomy C. External beam irradiation to the thyroid D. Hemithyroidectomy E. Total thyroidectomy

ANSWER IS E The recurrence of symptoms following medical therapy attracts a recommendation for definitive treatment. Since radio-iodine can worsen eye signs, this is not a wise choice. Surgery in the form of a total thyroidectomy would be curative.

1169. A 55 year old women presents with nipple discharge. On examination, she has a slit like retraction of the nipple in the centre of this area is a small amount of

Prepared by Dr: Mohammed Musa Brema Idress – My best wishes Page 513 cheese like material. No discrete mass lesion is palpable in the underlying breast. What is the commonest underlying cause? A. Invasive lobular carcinoma B. Duct ectasia C. Fibroadenoma D. Phyllodes tumour E. Inflammatory carcinoma

ANSWER IS B Duct ectasia is a common alteration in the breast that occurs with aging. As the ducts shorten and dilate a degree of symmetrical slit like retraction occurs. A small amount of cheese like discharge may occur.

1170. Which of the following statements regarding papillary carcinoma of the thyroid is false? A. They account for the majority of thyroid carcinomas B. Spread predominantly via the lymphatics C. May be diagnosed using fine needle aspiration cytology D. When viewed microscopically may demonstrate 'orphan Annie 'nuclei E. Have a five year survival of 65% if confined to the thyroid alone

ANSWER IS E The prognosis for localized papillary carcinomas is excellent. Survival rates at 5 years approach 90%.

1171. A 35 year old woman has undergone a wide local excision. The histology shows an invasive lobular carcinoma present at 3 of the resection margins. Cavity shavings taken at the original operation are also involved. Sentinel node biopsy was negative. What is the most appropriate course of action? A. Arrange for re-excision of margins B. Arrange for breast radiotherapy alone C. Arrange for completion mastectomy alone D. Arrange for radical radiotherapy to the breast and axilla E. Arrange for completion mastectomy and axillary node clearance

ANSWER IS C This patient has an extensive disease process and lobular cancers are notorious for being multifocal. In this case a mastectomy is the safest next step. Radiotherapy is not appropriate in this setting as the margins are not clear.

1172. A 20-year-old male presents to the ENT clinic with increasing swelling at the anterior border of sternocleidomastoid. The swelling was initially pain free, but is now painful, with an increase in size over the last week. Lymph node examination of the neck is normal. The swelling has no overlying punctum, and is not pulsatile. What is the most likely diagnosis? Select one answer only. A. Branchial cyst B. Carotid body tumour

Prepared by Dr: Mohammed Musa Brema Idress – My best wishes Page 514

C. Lymphoma D. Pharyngeal pouch E. Tuberculous lymphadenopathy

ANSWER IS A Branchial cysts are present at birth, but present usually between 15 and 25 years, when the potential space left from the second branchial arch growing down distends. Males and females are equally commonly affected. The swelling occurs at the anterior border of sternocleidomastoid and is usually painless, unless it becomes infected, in which case it enlarges and becomes painful, as in this case.

1173. A 52-year-old man recently arrived in the UK from Mexico presents to ENT clinic with a slow growing painless pulsatile lump in the anterior triangle of the neck, deep to sternocleidomastoid. What investigation would be best to plan for surgical excision of this lesion? Select one answer only. A. CT B. IV digital subtraction arteriography C. MRI D. USS E. X-ray

ANSWER IS B The patient has a carotid body tumour. These are more common in Peru and Mexico. While CT, MRI and USS would be sufficient to see the tumour itself, IV digital subtraction arteriography is superior in allowing assessment of the cerebral circulation and any collaterals.

1174. A 13-year-old boy presents to the emergency department. He is known to have chronic suppurative otitis media. He has a severe headache, fever, and otorrhoea. He reports altered taste. On examination he has pain on lateral rotation of the neck and tenderness and oedema over the right mastoid. He is tachycardic. What is the most likely complication of chronic suppurative otitis media that he has? Select one answer only. A. Cortical thrombophlebitis B. Extra-dural abscess C. Lateral sinus thrombosis D. Meningitis E. Subdural abscess.

ANSWER IS C All of the conditions listed are known complications of chronic suppurative otitis media, and all can cause headache, fever and otorrhoea. Lateral sinus thrombosis causes papilloedema, a raised intracranial pressure, and tenderness and oedema over the mastoid (Griesinger sign).

If the thrombosis extends into the jugular bulb and internal jugular vein, then this can result in neck pain on rotation. Compression of the ninth, tenth and eleventh

Prepared by Dr: Mohammed Musa Brema Idress – My best wishes Page 515 cranial nerves can also occur and this would alter the patient’s taste in the posterior third of the tongue, due to glossopharyngeal compression, and can result in tachycardia, due to vagus nerve compression.

1175. A 3-year-old boy presents to the emergency department with a severe sore throat, temperature of 38.70C and noisy breathing. The child is sat forward and drooling. What is the most important first step you should perform in the management of this child? Select one answer only. A. Examination of the oropharynx B. Flexible largyngoscopy C. IV access and blood cultures D. IV antibiotics E. Urgently call the paediatric anaesthetist

ANSWER IS E The child has acute epiglottitis. Acute epiglottitis tends to present with a rapidly progressive sore throat and in the late stages, may be associated with inspiratory stridor. The child will be toxic with a raised temperature and tends to sit forward and drool. The loose connective tissue swelling in the epiglottis can occlude the airway and care must be taken to avoid this. Any attempts to examine the child, including simple cannulation can precipitate laryngospasm and airway occlusion. Therefore the most important step in managing this patient is to get the help of someone with experience in managing children's airways.

1176. An unimmunized 2-year-old child is brought to the emergency department by her parents with a 1day history of a sore throat. On examination the child is drooling, with noisy breathing and a temperature of 39.40C. What is the most likely causative organism? Select one answer only. A. Group C streptococcus B. Haemophilus influenzae type B C. Moraxella catarrhalis D. Neisseria meningitidis E. Staphylococcus aureus.

ANSWER IS B The child in this scenario has acute epiglottitis. Acute epiglottitis tends to present with a rapidly progressive sore throat and in the late stages, the patient may have associated inspiratory stridor. The child will be toxic, with a raised temperature and tends to sit forward in the tripod position and drool. The peak incidence is between 2-7 years. All the bacteria listed are known to cause acute epiglottitis, however, some are more common in adults.

The commonest cause in children has classically been Haemophilus influenzae type B, accounting for approximately 90%, however the incidence has decreased due to vaccination. Currently the commonest cause in children is group A beta-haemolytic streptococcus. The most likely cause in this case is Haemophilus influenzae type B as the child has not been immunized.

Prepared by Dr: Mohammed Musa Brema Idress – My best wishes Page 516

1177. A 13-year-old boy, known to have sinusitis, presents to the emergency department with severe pain over the right side of his nose and right cheek. Over the last 24 hours he has developed a fever and blurred vision. On examination he has tenderness, erythema and induration over the right cheek and around the eye, which is bulging. What is the most likely complication of sinusitis that he has developed? Select one answer only. A. Cavernous sinus thrombosis B. Meningitis C. Orbital cellulitis D. Osteomyelitis E. Subperiosteal abscess.

ANSWER IS C All of the conditions listed are complications of acute sinusitis. The thin walls of the sinuses and fact that the sinuses all share a boundary with the orbit predispose patients with acute sinusitis to developing them. Orbital cellulitis causes erythema, induration and tenderness over the orbit. The key features that would make you suspicious of it being more advanced than a preseptal cellulitis are proptosis, conjunctival oedema (chemosis) and loss of colour differentiation. A CT scan of the orbits and sinuses would confirm the extent.

1178. A 40-year-old male presents to the ENT clinic with non-tender 3cm hard and rubbery mass just anterior and superior to the angle of the mandible. The skin overlying the mass is normal. The patient has drooping of the corner of the mouth on the affected side. What is the most appropriate investigation for this patient? Select one answer only. A. Core needle biopsy B. Excisional biopsy with frozen sections C. Incisional biopsy D. Trucut biopsy E. Wedge biopsy

ANSWER IS B The patient has a mass in the parotid gland, the involvement of the facial nerve suggests this lesion is more likely to be malignant. The investigation of parotid lumps is controversial, (particularly if the lump shows no clinical signs of malignancy), with some debate existing regarding ultrasound and fine needle biopsy. The principle with regards to investigating these lumps is however that you should not cut into the lump and risk spread by seeding along the tract. Given that all the other types of biopsy listed would cut into the lump, excisional biopsy is the only available option that does not risk spread.

1179. A 54-year-old man, who underwent a parotidectomy six months ago, returns to clinic. He complains of flushing and sweating over the temple, cheek and upper neck prior to and during eating. What is the most likely diagnosis? Select one answer only. A. Boerhaave syndrome

Prepared by Dr: Mohammed Musa Brema Idress – My best wishes Page 517

B. Brown syndrome C. Edwards syndrome D. Felty syndrome E. Frey syndrome

ANSWER IS E Frey syndrome is a known complication of parotidectomy with gustatory sweating due to inappropriate regeneration of sympathetic and parasympathetic nerve fibers. Boerhaave syndrome is a rupture of the oesophagus. Brown syndrome is a rare form of strabismus. Edwards syndrome is trisomy 18 and Felty syndrome is rheumatoid arthritis, splenomegaly and neutropaenia.

1180. A 52-year-old female presents to the ENT clinic with fatigue, a dry mouth and eyes and, repeated episodes of oral thrush. She is on etanercept for rheumatoid arthritis. What is the most likely diagnosis? Select one answer only. A. Chronic recurrent sialoadenitis B. Drug induced sialomegaly C. Granulomatous sialoadenitis D. Primary Sjogren’s syndrome E. Secondary Sjogren’s syndrome

ANSWER IS E Sjogren’s syndrome affects 3-4% of UK adults, with the average age at presentation between 40 and 60 years. 90% of cases occur in females. Sjogren’s presents with xerostomia, xerophthalmia, and fatigue and may have repeated oral thrush (secondary to lack of saliva). Periductal lymphocytes are found in multiple organs. Secondary Sjogren’s is characterized by a coexisting connective tissue abnormality, most commonly rheumatoid arthritis, (etanercept is a TNF alpha blocker used for rheumatoid arthritis).

1181. A 90-year-old on diuretics with ill-fitting dentures, presents to the emergency department with a one day history of pain and swelling of the left parotid, worse on speaking and eating. The patient has a temperature of 38.90C. Which bacteria is most likely to be responsible for this patient's symptoms? Select one answer only. A. Klebsiella pneumoniae B. Mycobacterium tuberculosis C. Pseudomonas aeruginosa D. Staphylococcus aureus E. Streptococcus peumoniae.

ANSWER IS D The commonest cause of acute parotits is viral, (mumps). Acute bacterial parotitis can occur, particularly in neonates and the elderly and debilitated. Poor oral hygiene and dehydration are risk factors. The commonest bacterial cause is staphylococcus aureus.

Prepared by Dr: Mohammed Musa Brema Idress – My best wishes Page 518

1182. A 47-year-old man is referred to clinic with unilateral swelling affecting the right parotid gland. He reports painless swelling, increasing in size over the last few years. Examination confirms intact facial nerve function, although inspection of the mouth reveals displacement of the right tonsil and pillar of the fauces towards the midline. Select the most likely diagnosis. A. Adenocarcinoma B. Adenoid cystic carcinoma C. Epidermoid carcinoma D. Acinic cell carcinoma E. Pleomorphic adenoma

ANSWER IS E This tumour is a slow-growing lesion that is made up of glandular and stromal elements. It does not have a true capsule but a pseudo-capsule resulting from fibrosis of the adjacent compressed salivary tissue.

Multiple projections extending through ‘defects’ in the pseudo-capsule into the normal surrounding gland prevent its treatment by enucleation. Clinically, pleomorphic adenomas present as slow-growing painless masses. There is rarely disruption of the facial nerve, although location in the deep part of the gland may displace adjacent structures inside the mouth.

The diagnosis is confirmed by fine-needle aspiration. Such tumours should be removed by superficial parotidectomy when the tumour is in the superficial lobe of the gland, or by total parotidectomy with preservation of the facial nerve in those tumours involving the deep part of the gland. Those arising in the submandibular gland should be treated by excision of the gland.

1183. A 61-year-old woman has been referred urgently by her general practitioner with a history of painful swelling of the right parotid gland. She has recently developed facial nerve palsy on the right side. Examination reveals a cystic mass that is fixed over the parotid gland, and associated cervical lymphadenopathy. Fine-needle aspiration demonstrates the presence of atypical mucous cells. Select the most likely diagnosis. A. Metastatic carcinoma B. Monomorphic adenoma (synonym: adenolymphoma – Warthin’s tumour) C. Mucoepidermoid carcinoma D. Pleomorphic adenoma E. Adenocarcinoma

ANSWER IS C This is the most common malignant salivary gland tumour. Associated pain is not a reliable indicator of malignancy but it is associated with a worse prognosis in proven malignancy. Presentation with facial nerve palsy is very suggestive of a malignant tumour.

Prepared by Dr: Mohammed Musa Brema Idress – My best wishes Page 519

Twenty-five per cent of patients have associated lymphadenopathy at the time of presentation. Histologically, the tumour is composed of epidermoid cells and mucous cells, which secrete mucus into the stroma of the tumour, giving rise to its cystic nature. Tumours may be graded as high- or low-grade, although their biological behaviour tends to be unpredictable. The 10-year survival rates are 40% and 80% for high-grade and low-grade tumours, respectively.

1184. A 71-year-old man presents with swelling of the left parotid gland. Examination reveals a non-tender enlargement of the left parotid gland, with bilateral cervical and axillary lymphadenopathy. There is intact facial nerve function. Full blood count: haemoglobin 8.8 g/dl, white cell count 2.2 x 109/liter, platelets 45 x 109/liter. Select the most likely diagnosis. A. Acinic cell carcinoma B. Adenocarcinoma C. Adenoid cystic carcinoma D. Epidermoid carcinoma E. Lymphoma

ANSWER IS E Primary salivary gland lymphomas are uncommon, accounting for 10% of all salivary malignancies. The majority tend to be non-Hodgkin’s lymphoma. Patients with non- Hodgkin’s usually present with localized or generalized non-tender lymphadenopathy. Approximately 40% of patients have bone marrow involvement at presentation, resulting in cytopenia. There may be associated splenic or hepatic enlargement.

1185. A 33-year-old non-smoker who presents with a 6-month history of recurrent, intermittent left-sided neck swelling associated with pain and dry mouth. He can feel a hard mass in the floor of his mouth. Choose the most likely diagnosis from the list below. A. Cervical nodal metastasis B. Branchial cyst C. Cystic hygroma D. Deep neck space abscess E. Salivary gland swelling

ANSWER IS E

1186. A 25-year-old female presents with mild dysphagia and a palpable mass on the left of the midline. Choose the most likely diagnosis from the list below. A. Solitary thyroid nodule B. Thyroid nodule in multinodular goiter C. Cervical nodal metastasis D. Cystic hygroma E. Deep neck space abscess

ANSWER IS B

Prepared by Dr: Mohammed Musa Brema Idress – My best wishes Page 520

1187. You are assisting in a sentinel node biopsy for a patient with known breast cancer. Which of the following statements is correct? Select ONE answer only. A. At least 10 nodes must be sampled to give an accurate nodal stage B. Sentinel node biopsy involves identifying the first 4 nodes of a basin through which the majority of lymph from the breast drains C. Lymphoscintigraphy can be performed up to one week in advance of the procedure D. Blue dye is contraindicated in pregnancy E. There is a high incidence of brachial plexus traction following sentinel node.

ANSWER IS D Sentinel node biopsy reduces the number of patients who require a lymph node dissection, thereby reducing complications such as lymphoedema. However, there is still a false negative result and as such patients should be counseled accordingly. It involves identifying the first nodes which drain the primary tumour and is usually 1-2 nodes but can be more. A combination of radioactive colloid and blue dye (patent blue) can be used but the blue dye is contraindicated in pregnancy.

1188. A 64-year-old female presents with a painless lump she has noticed on the front of her neck which moves on swallowing. On examination, she is noted to have a thyroid nodule and cervical lymphadenopathy. An FNA of the thyroid nodule reveals psammoma bodies. What is the most likely diagnosis? Select one answer only. A. Anaplastic thyroid carcinoma B. Follicular thyroid carcinoma C. Lymphoma D. Medullary thyroid carcinoma E. Papillary thyroid carcinoma

ANSWER IS E Papillary carcinoma of the thyroid gland accounts for around two-thirds of thyroid malignancies. The tumour spreads via lymphatics and at presentation over half of patients will have cervical lymph node involvement. Histologically Psammoma bodies are typical. Follicular thyroid carcinoma accounts for 20% of thyroid tumours and classically spreads via the bloodstream. Therefore lymph node involvement at presentation is very uncommon. Medullary thyroid carcinoma occurs in 5-10% of thyroid malignancies. It is derived from C cells and therefore produces calcitonin which can be measured in the blood. Anaplastic carcinoma is a very aggressive tumour type that is most commonly found in the elderly. It has a very poor prognosis with few patients surviving beyond 1 year.

1189. A 46-year old pre-menopausal woman presents with a rapidly growing lump in her left breast. There is no associated nipple discharge or ulceration. What is the likely diagnosis? Select ONE answer only. A. Phyllodes tumour B. Ductal carcinoma C. Fibroadenoma

Prepared by Dr: Mohammed Musa Brema Idress – My best wishes Page 521

D. Breast cyst E. Paget’s disease

ANSWER IS A Phyllodes tumour occur in premenopausal women and manifest as rapidly growing tumours, often enlarging over a few weeks. They rarely involve the nipple areolar complex or ulcerate. Overlying skin may have a shiny appearance and reveal presence of superficial veins. Early tumours can be confused with fibroadenoma’s which typically present 15 years earlier. Approximately 60-70% are benign with the chance of malignancy increasing with age.

1190. You are attending a lecture on breast cancer and are asked which type is most common. Which of the following is the correct answer? Select ONE answer only. A. Ductal B. Lobular C. Mucinous D. Papillary E. Medullary.

ANSWER IS A Ductal carcinoma accounts for approximately 75% of all cases and is composed of invasive ductal cancer (55%) and ductal carcinoma in situ (DCIS). It arises from the breast ducts. In situ disease carries an increased lifetime risk of developing invasive carcinoma. Invasive lobular carcinoma (ILC) is the next commonest type of breast cancer (10%) with the other types being relatively rare. Although having a slightly better prognosis, ILC is more often multifocal, bilateral and mammographically occult.

1191. A 23-year-old lady presents with a painless mobile lump in the left outer breast. It has been present for the last nine weeks and seems to fluctuate in size with her menstrual cycle. What is the likely diagnosis? Single best answer - select one answer only. A. Abscess B. Blocked Montgomery Gland C. Fibroadenoma D. Mondor’s Disease E. Sebaceous cyst.

ANSWER IS C The most likely diagnosis for a breast lump in this age group is a fibroadenoma and a history of fluctuation in size with menstrual cycle is typical. Montgomery glands are situated within the areolar skin. Mondor’s disease is superficial thrombophlebitis of a chest wall vein – it can occur following trauma or exercise and is tender. Sebaceous cyst is a skin cyst and therefore fixed to the skin. An abscess is typically painful and with a short history.

Prepared by Dr: Mohammed Musa Brema Idress – My best wishes Page 522

1192. A 55-year-old female is referred to a general surgery clinic after a dermatologist has diagnosed a migratory necrolytic erythema. The patient also has a history of diabetes mellitus and weight loss. What is the most likely diagnosis? Select one answer only. A. Cushings’ syndrome B. Gastrinoma C. Glucagonoma D. Insulinoma E. Phaeochromocytoma

ANSWER IS C Glucagonomas are very rare tumours with an incidence of around 1 in 20 million. There are often metastases present when the patient presents. Classic features of glucagonomas are migratory necrolytic erythema, diabetes mellitus and weight loss. The rash is very characteristic and may be sufficient evidence to raise the concern itself. It is secondary to a zinc deficiency in the skin.

1193. A 35-year old woman with known node positive breast cancer and a positive family of the disease has undergone a mastectomy, axillary clearance and chemotherapy. She returns for the pathology result and asks you about her 5-year prognosis. Which is one the following is correct? Select ONE answer only. A. The Nottingham Prognostic Index is purely a mathematical equation and should not be applied to the individual patient B. Her risk of recurrence should be expected given her strong family history C. The Nottingham Prognostic Index is calculated from size of the lesion, the number of lymph nodes involved and tumour grade D. It is impossible to predict prognosis from pathological results alone E. Immune modulators would prolong survival in this patient.

ANSWER IS C The Nottingham prognostic index was constructed in 1982 based on a retrospective analysis of 9 factors in 387 patients. It divides patient prognosis into good (83% 10 year survival) moderate (52% 10 year survival) and poor (13% 10 year survival). There is an updated classification which an additional 2 further groups but this has not been widely adopted. It is used to compare survival outcomes between breast units.

1194. A 41-year-old woman who is six weeks postpartum presents with a tender periareolar lump with nipple retraction. What is the likely diagnosis? Single best answer - select one answer only. A. Abscess B. Cyst C. Fibroadenoma D. Intraductal papilloma E. Malignancy.

ANSWER IS A

Prepared by Dr: Mohammed Musa Brema Idress – My best wishes Page 523

The most likely diagnosis of a tender lump in this breastfeeding patient is a postpartum abscess. The nipple retraction is caused by inflammatory fibrosis.

1195. A 45-year old woman, who has a 3-month daughter and is breast feeding, attends outpatients with a finding of a new breast lump. How should she be managed? Select ONE answer only. A. Lumps are common in breast feeding woman and she should be re-assured and reviewed in 6 months B. Radiological imaging and clinical examination in the first instance C. Mammogram in combination with tissue biopsy D. Mammogram in combination with tissue cytology and clinical examination E. Radiological imaging in combination with tissue biopsy, clinical examination and lymph node staging.

ANSWER IS B In this lady the most likely diagnosis is benign breast disease associated with the heightened hormonal state secondary to breast feeding (a galactocoele). She should have a clinical assessment and imaging which as she is over 35 should be with mammography and ultrasound. The need for a tissue diagnosis (FNA/Core biopsy) would depend on these results. If possible a biopsy should be avoided as it is associated with an unacceptable risk of a mammary fistula which is very difficult to treat. It is often sufficient to repeat imaging at an interval if the initial imaging is not suspicious.

1196. A 35-year-old woman asks to see you in clinic to discuss further management as she has recently tested BRCA 1 positive. She is very keen to reduce her chance of developing cancer since her mother died aged 37. She has not yet completed her family and is hoping to have another child soon. What is the most appropriate advice or intervention for her to consider at present? Single best answer - select one answer only. A. Advice on self-examination B. Mammographic screening C. Risk reducing mastectomy D. Risk reducing bilateral salpingoophrectomy E. Tamoxifen.

ANSWER IS C The overall risk for breast cancer in BRCA 1 patients is 60-90% by age 80 years and 40 – 60% lifetime risk of ovarian cancer which increases from age 40 years. She wants to reduce the chance of developing cancer, therefore self-examining and screening may allow earlier detection but will not reduce the chance of her developing cancer. In any case she would start having mammograms from age 40 years and MRI screening at her current age following national guidelines.

Risk reducing bilateral salpingoophrectomy would reduce the chance of her developing ovarian cancer but is offered after the age of 40 when the risk increases and currently she still wishes to extend her family. Women who have their ovaries

Prepared by Dr: Mohammed Musa Brema Idress – My best wishes Page 524 removed before menopause also reduce their risk of breast cancer by 50%, even when hormone replacement therapy is given.

Tamoxifen is teratogenic so not suitable in this case as chemoprophylaxis. Although all of these routes should be discussed in detail with the patient, it is risk reducing mastectomy that would meet her current request. It is important to note that this does not remove the risk completely hence ‘risk reducing’. It would reduce her risk to 5% over her lifetime which is less than that of the general population.

1197. A 67-year old lady with known breast cancer is seen in your clinic with regard to breast reconstruction options. Which of the following regarding free tissue transfer is correct? Select ONE answer only. A. There are fewer complications than with local pedicled options B. Only the abdomen can be used as a donor site C. It is better to perform a free flap after any radiotherapy treatment has been completed D. Flap necrosis can occur in the outer zones of donor skin as perfusion here is not as reliable as in the centre E. A free flap is contra-indicated because of the patient’s age.

ANSWER IS D Free tissue transfer is a longer procedure and as such, more complications can arise. Donor sites include abdomen, thigh and buttock. A large trial suggests age is not a barrier for free tissue transfer. Radiotherapy affects vessel wall composure and as such immediate free transfers are easier to perform than delayed. Flap necrosis commonly occurs on the outer zones, for this reason the outer section is often discarded. Even with a reliable blood supply the rate of DIEP flap loss is up to 10% (as shown by the recent national mastectomy and breast reconstruction audit).

1198. A 35-year-old female is referred by her GP with hyperhidrosis. On further questioning it emerges this is intermittent in nature and is precipitated by exercise and overeating. In addition to this she reports severe headaches and palpitations at the same time. On one occasion her blood pressure was measured and was found to be 190/110. What is the most likely diagnosis? Select one answer only. A. Cushing syndrome B. Gastrinoma C. Neuroblastoma D. Phaeochromocytoma E. Zollinger-Ellison syndrome

ANSWER IS D Phaeochromocytomas are rare catecholamine-producing tumours which arise from sympathetic paraganglia cells which are collections of adrenaline- secreting chromaffin cells. They occur in around one in a million people and are the underlying cause for less than 1% of hypertension. Around 10% are bilateral, 10% are malignant, and 10% are extra-adrenal.

Prepared by Dr: Mohammed Musa Brema Idress – My best wishes Page 525

The most common extra-adrenal site is the organ of Zucker kandl which is by the aortic bifurcation. The most common symptom is sweating, and other symptoms include paroxysmal headaches, palpitations pericordial pain. Pre-operative management with α and ß blockers is crucial prior to anaesthetic induction to minimize morbidity and mortality.

1199. A 69-year old woman has undergone mastectomy and axillary surgery for breast cancer. She returns to clinic. During the consultation she asks you what stage her cancer is at. She has read a little on the internet but does not wholly understand the implication of stage on her overall survival. Which of the following can you advise? Select ONE answer only. A. Staging cannot be assessed without full radiological imaging including a CT chest/abdomen/pelvis and PET scan B. Staging is a medical term used for cancer research purposes, it has no implication on clinical disease free survival period C. Breast cancer has 4 stages, where stage 4 has a better prognosis than stage 1 D. Staging is based on histopathological analysis of the cancer and follows the TNM classification, where T is tumour size, N is nodal involvement and M is metastasis E. An axillary dissection should be performed in order to establish whether she has lymph node metastasis in order to fully stage her disease.

ANSWER IS D Stage of disease is determined by histological analysis of the tumour specimen and lymph. Follow up scans such as CT are reserved for follow up where distant metastatic spread is suspected. If this hasn’t been assessed it is designated Mx Staging of a disease allows identification of those patients who would benefit from entry into trials as well as informing the patient on their likely disease free survival period.

1200. A 23-year old women who has given birth to one child present with a growing mobile lump in her right breast. It is painless with no associated nipple discharge. What is the likely diagnosis? Select ONE answer only. A. Breast cyst B. Ductal carcinoma C. Paget’s disease D. Fibroadenoma E. Phyllodes tumour

ANSWER IS D Fibroadenoma’s are composed of fibrous and glandular tissue. They are highly mobile with well-defined edges. They are often termed ‘breast mouse’ due to their mobility. All breast lumps should undergo triple assessment. In this age group fibroadenoma is by far the most likely diagnosis with breast cysts a close second.

1201. A 50-year old female consults you regarding breast screening as she is unsure whether she should take this up. Which of the following is true?

Prepared by Dr: Mohammed Musa Brema Idress – My best wishes Page 526

A. Breast screening will be able to tell her if she does or does not have breast cancer B. A true positive is a positive test result in the presence of the disease C. A true negative is a positive test result in the absence of disease D. A false positive is a positive result in the presence of the disease E. A false negative is a negative result in the absence of the disease.

ANSWER IS B Breast screening has recently been extended to include patients between the ages of 47 and 73 years. It involves 3 yearly mammograms which are taken in two views and are also double read. The main aim of screening is to detect early stage breast cancers which are still localized to the breast and therefore amenable to treatment.

1202. A 34-year old woman undergoes ultrasound examination of a breast cyst. Which of the following feature on ultrasound is consistent with this diagnosis? Select ONE answer only. A. Irregular wall B. Smooth anterior border C. Black hypoechoic centers without internal echoes D. Blood vessels running through them E. Pulsatile.

ANSWER IS C Cysts have typical features of hypoechoic centers, smooth walls and sharp edges. Ultrasound has a sensitivity of 75% and specificity of 97% in the diagnosis of breast cancer and is used as an adjunct to mammography in patients over 35. In patients under 35 it is often the primary radiological modality as the breasts are too dense for accurate mammographic interpretation.

1203. A 42-year-old male is referred to his GP because of problems in his workplace linked to his mood swings. On taking a detailed history he reports episodes of light-headedness, disorientation with an associated aggressive temper. These resolve following the consumption of food. He has previously been investigated and was found to have a blood sugar of 1.8 mm/l during an attack. What is the most likely diagnosis? Select one answer only. A. Gastrinoma B. Glucagonoma C. Insulinoma D. Phaeochromocytoma E. Zollinger-Ellison syndrome.

ANSWER IS C Insulinomas are not common and occur in around 1 in a million of the population. They arise from ß islet cells and are mostly benign. They often cause symptoms relating to hypoglycaemia e.g. lightheadedness, mood swings and in some cases unconsciousness. The diagnosis should be suspected when Whipple’s triad is present

Prepared by Dr: Mohammed Musa Brema Idress – My best wishes Page 527

- attacks brought on by fasting, hypoglycaemia (<2.0mmol/l) during an episode and symptoms then improving with glucose administration.

1204. You are assisting in a mastectomy and a level two axillary clearance. Which of the following statements best describes the anatomical level to which clearance should be gained? Select ONE answer only. A. Up to the medial border of pectoralis minor B. Above pectoralis minor C. Below pectoralis minor D. To the outer border of the first rib E. Up to the medial border of pectoralis major.

ANSWER IS A Axillary clearance is divided into 3 levels: Level I: Lateral to pectoralis minor Level II: Up to medial border of pectoralis minor Level III: Beyond medial border of pectoralis minor.

1205. A 54-year-old female presents with a painless fixed lump in the upper outer quadrant of the right breast which has been present for the last four weeks. Mammogram has not revealed any abnormality but ultrasound has shown a 21mm irregular mass. What is the likely diagnosis? Single best answer - select one answer only. A. Benign Phyllodes B. Fibroadenoma C. Intraductal Carcinoma D. Lobular Carcinoma E. Radial Scar.

ANSWER IS D Lobular carcinoma is the second most common type of breast cancer but can be mammographically occult. Both a fixed lump on clinical examination and an irregular mass on imaging suggests malignancy rather than a benign lesion. A radial scar is most typically diagnosed on mammographic screening and usually impalpable.

1206. A 28-year-old female presents with a two week history of discomfort and a lump in the 6 o’clock region adjacent to the nipple-areolar complex. She is a BRCA 2 carrier, has mild asthma and smokes twelve cigarettes a day. What is the likely diagnosis? Single best answer - select one answer only. A. Fat necrosis B. Fibroadenoma C. Harmatoma D. Intraductal papilloma E. Periductal mastitis

ANSWER IS E

Prepared by Dr: Mohammed Musa Brema Idress – My best wishes Page 528

The most like cause of a painful lump in a young smoker is periductal mastitis. Smoking is the likely aetiology here. Her BRCA 2 history is a red-herring. She may develop an abscess, and mammary duct fistula to the skin can occur. Antibiotics can help settle the inflammation but smoking cessation is most important to allow resolution and prevent further complications.

1207. A 52-year-old female undergoes a thyroidectomy for a follicular carcinoma of the thyroid gland. Later you are called urgently to the ward as the patient has become acutely short of breath and dropped her oxygen saturations. On examination a tense swelling is seen beneath the dressings. What is the most likely diagnosis? Select one answer only. A. Anaphylaxis B. Infection C. Tension haematoma D. Thyroid storm E. Tracheal injury.

ANSWER IS C A tension haematoma can occur following a thyroidectomy and represents a surgical emergency as the airway could be compromised as evidenced by the shortness of breath and drop in oxygen saturations in this case. Immediate measures to control this include removing any clips or using a scalpel to release blood and thereby relieve the pressure. There is no precipitant in the history to induce anaphylaxis. The history is too acute to make an infection likely. Thyroid storm is very unlikely to manifest as a compromised airway and more likely restlessness, confusion, tachycardia and pyrexia. If tracheal injury had occurred intra-operatively, it is likely to present prior to the patient returning to the ward.

1208. You are assisting in a free flap reconstruction of the breast. Which of the following recipient vessels is commonly used in the micro-anastomosis? Select ONE answer only. A. Subclavian artery B. Internal mammary artery C. Axillary artery D. Thoracodorsal artery E. Serratus branch of thoracodorsal artery.

ANSWER IS B The internal mammary vessels are found underneath the medial 2-4th ribs, which are resected during the procedure. It is possible to use other recipient vessels, such as those based on the thoracodorsal axis, however the dissection is harder.

1209. A 19-year old female student presents to clinic having undergone genetic testing for breast cancer given her strong family history. She is noted to be positive for the BRCA1 gene. How should you advise her? Select ONE answer only. A. BRCA1 gene is responsible for apoptosis of normal breast cells B. Highest carrier rates of BRCA1 mutations occur in the black population

Prepared by Dr: Mohammed Musa Brema Idress – My best wishes Page 529

C. She has a 50% chance of developing breast cancer by age 50 D. She has a 100% chance of developing breast cancer by age 50 E. She is protected against developing ovarian cancer.

ANSWER IS C BRCA1 and BRCA2 are tumour suppressor genes responsible for DNA repair. The BRCA gene is found on chromosome 17 (long arm). Mutations are linked to development of breast and ovarian cancer. The risk of developing breast cancer is approximately 50% by 50 and 80% by 90. With BRCA 1 the lifetime risk of ovarian cancer is 55% and for BRCA 2 is 25%. They account for the majority of inherited breast cancers but are not the only genes linked with breast cancer. Options for BRCA positive patients range from surveillance, prophylactic surgery, modifying risk behavior and tamoxifen use.

1210. A 33-year old female is seen in your clinic. She has already undergone mastectomy for breast cancer and wishes to have a pedicled myocutaneous flap reconstruction. Which of the following is correct? Select ONE answer only. A. It is associated with a higher rate of failure as compared to free flap reconstruction B. The latissimus dorsi flap is based on the thoracodorsal vessels C. The latissimus dorsi flap is based on the circumflex scapular artery D. She should expect a small vertical scar on her back E. A ‘B’ cup breast is the largest that can be feasibly produced by this type of reconstruction.

ANSWER IS B The thoracodorsal vessels supply the latissimus dorsi which is a pedicled myocutaneous flap. It can be combined with an implant to recreate large breast volumes. An extended lat dorsi, which includes the surface fat, can be taken to increase the volume to approximately a C cup without using an implant. A skin paddle is taken with the muscle, and as such a long scar is not unusual. Its length and orientation varies according to patient and surgeon preferences.

1211. A 37-year-old with a history of benign breast disease enquires about her risk of developing malignant breast disease. Which of the following is associated with the highest risk of developing malignant breast disease? Select ONE answer only. A. Sclerosing adenosis B. Duct ectasia C. Fibroadenoma D. Apocrine metaplasia E. Lobular carcinoma in situ

ANSWER IS E LCIS carries a 10 times increased risk of developing malignant breast cancer. Close clinical clinical follow up and mammography are indicated. If asked directly whether the other conditions area associated with an increased risk of breast cancer the

Prepared by Dr: Mohammed Musa Brema Idress – My best wishes Page 530 answer is probably that any benign breast disease is associated with an increased risk but that it is very, very small.

1212. A 27-year-old woman 2 days post-thyroidectomy has numbness around the mouth and the sensation of ‘pins and needles’ in her fingers. Please select the most appropriate diagnosis from the list. A. Air embolism B. Haemorrhage C. Hypocalcaemia D. Hypothyroidism E. None of the above

ANSWER IS C Hypocalcaemia after thyroidectomy may occur for two reasons: metabolic and anatomical. Metabolic causes are not fully understood but may be secondary to the release of calcitonin during manipulation or a reduction of renal tubular resorption of calcium without a change in parathormone or calcitonin levels. Anatomical causes are the result of excision of all parathyroid tissue during total thyroidectomy.

If the calcium is > 2.0 mmol/liter, symptoms usually resolve within 2 days without treatment. If symptoms persist or worsen treatment with calcium supplementation and synthetic vitamin D will be necessary.

1213. A 50-year-old woman becomes acutely confused 12 h postthyroidectomy. She has been complaining of severe abdominal pain, palpitations and diarrhoea. An electrocardiogram demonstrates atrial fibrillation. Please select the most appropriate diagnosis from the list. A. Air embolism B. Thyroid crisis C. Hypocalcaemia D. Hypothyroidism E. None of the above

ANSWER IS B This is a rare but life-threatening condition precipitated by thyroid surgery, infection, stress, or radioactive iodine therapy in an unprepared patient. It is caused by the sudden release of massive amounts of thyroid hormone into the systemic circulation resulting in hyperpyrexia, tachycardia, extreme restlessness, diarrhoea and vomiting. It may mimic the acute abdomen but needs emergency treatment. Assessment and appropriate management of the airways, breathing and circulation should be performed. Fluid resuscitation with normal saline is required with propanolol together with potassium iodide, carbimazole and dexamethasone.

1214. A 45-year-old woman returns to outpatients 3 months after thyroidectomy. She complains that her vocal range appears diminished when she participates in her local amateur dramatics productions. Please select the most appropriate diagnosis from the list.

Prepared by Dr: Mohammed Musa Brema Idress – My best wishes Page 531

A. Superior laryngeal nerve paralysis B. Bilateral complete recurrent laryngeal nerve paralysis C. Bilateral incomplete recurrent laryngeal nerve paralysis D. Unilateral complete recurrent laryngeal nerve paralysis E. Unilateral incomplete recurrent laryngeal nerve paralysis

ANSWER IS A Damage to the superior laryngeal nerve can cause voice weakness or fatigue, mild hoarseness and loss of vocal range. The upper half octave in range is lost, which can be particularly troublesome for singers, and is unlikely to be recovered.

1215. A 64-year-old man develops severe difficulty breathing shortly after a total thyroidectomy. Examination reveals respiratory rate 34 breaths/min and auscultation demonstrates inspiratory stridor. Please select the most appropriate diagnosis from the list. A. Superior laryngeal nerve paralysis B. Bilateral complete recurrent laryngeal nerve paralysis C. Bilateral incomplete recurrent laryngeal nerve paralysis D. Unilateral complete recurrent laryngeal nerve paralysis E. Unilateral incomplete recurrent laryngeal nerve paralysis

ANSWER IS C Recurrent laryngeal nerve damage may be uni- or bilateral secondary to bruising, stretching, division devascularization or ligation. In bilateral incomplete recurrent laryngeal nerve paralysis both vocal cords lie in the midline, unable to abduct, and there is severe dyspnoea with stridor soon after operation. Tracheal collapse (secondary to tracheomalacia) and haemorrhage into the pre-tracheal space may both present similarly; however, the former is rare and the latter usually evident on examination. In bilateral complete recurrent laryngeal nerve paralysis the two vocal cords occupy the cadaveric position, midway between the normal resting position and the midline. No abduction or adduction is possible; the voice is lost, but the dyspnoea is not severe. In unilateral complete recurrent laryngeal nerve paralysis all the laryngeal muscles on the affected side are paralyzed except the cricothyroid and part of the arytenoideus. The affected cord lies in the cadaveric position; the opposite cord can be adducted, but the differences in vocal cord tension produce a hoarse voice. Unilateral incomplete recurrent laryngeal nerve paralysis produces slight dyspnoea on exertion and little or no alteration in the voice.

1216. Select the most appropriate treatment of thyrotoxicosis for a pregnant woman. A. Propranolol B. Carbimazole C. Propylthiouracil D. Subtotal thyroidectomy E. Total thyroidectomy.

ANSWER IS C

Prepared by Dr: Mohammed Musa Brema Idress – My best wishes Page 532

1217. Select the most appropriate treatment of thyrotoxicosis for a teenager. A. Propranolol B. Carbimazole C. Propylthiouracil D. Subtotal thyroidectomy E. Total thyroidectomy.

ANSWER IS B

1218. Select the most appropriate treatment of thyrotoxicosis for a 65-year-old individual with cerebrovascular accident (CVA)/atrial fibrillation (AF). A. Propranolol B. Carbimazole C. Propylthiouracil D. Subtotal thyroidectomy E. Total thyroidectomy.

ANSWER IS A

1219. Select the most appropriate treatment of thyrotoxicosis for failed medical treatment. A. Propranolol B. Carbimazole C. Propylthiouracil D. Subtotal thyroidectomy E. Total thyroidectomy.

ANSWER IS E Thyrotoxicosis is managed conservatively in the first instance. Propranolol, a blocker, is good for reducing anxiety and treatment of cardiac arrhythmias such as AF. Care must be taken when prescribing carbimazole because blood dyscrasias occur in 2% of cases. A pregnant woman should be prescribed propylthiouracil. Ideally if a woman is attempting conception, then she should be changed from carbimazole to propylthiouracil. In the instance of failed medical therapy, radio- iodine (i.e. not during pregnancy) or a total thyroidectomy may be considered. The increasing use of total thyroidectomy (i.e. versus subtotal thyroidectomy) as a treatment for thyrotoxicosis is due to its lower risk of recurrent hyperthyroidism; if the patient is euthyroid at the time of surgery, thyroxine is started immediately postoperatively.

1220. A 33-year-old woman is seen with a 1-day history of painful enlargement of her right breast. Of note, she is currently breastfeeding and is a smoker. On examination she has generalized enlargement of her right breast. The overlying skin is erythematous, hot to touch and tender. There are no masses palpable. Breast ultrasound is normal. Please select the most appropriate diagnosis from the list below. A. Aberrations of normal development and involution (ANDI)

Prepared by Dr: Mohammed Musa Brema Idress – My best wishes Page 533

B. Breast abscess C. Cyclical mastalgia D. Tietze’s syndrome E. Acute mastitis

ANSWER IS E The commonest form of mastitis is that occurring in relation to pregnancy and breast feeding, the so called ‘lactational mastitis’. The condition is more common among smokers. Most cases are caused by Staphylococcus aureus when an ascending infection from the nipple, along the lactiferous ducts, initiates the mastitis. In the early phase the spreading cellulitis produces the classical signs of inflammation. After a few days an abscess develops, which may give rise to a palpable lump. The time- scale and the absence of a lump, either clinically or radiologically, differentiate acute mastitis from a breast abscess. Treatment is with antibiotics alone.

1221. A 27-year-old woman presents with a 6-month history of intermittent pain along the inner aspect of her left breast. The pain occurs for a few days before resolving spontaneously. The frequency of ‘attacks’ is increasing. She is an amateur athlete and complains that the pain prevents her from taking part in any physical activity. Breast examination is unremarkable apart from some tenderness medially. Please select the most appropriate diagnosis from the list below. A. Aberrations of normal development and involution (ANDI) B. Breast abscess C. Cyclical mastalgia D. Tietze’s syndrome E. Acute mastitis

ANSWER IS D Tietze’s syndrome is an inflammatory condition of the costochondral cartilages. It is characterized by pain, tenderness and swelling of one or more costal cartilages, typically the first four. The syndrome usually affects older children and young adults and there are more female cases than male cases. The pain is usually exacerbated by physical activity, deep inspiration and coughing. Examination reveals the presence of exquisite tenderness and swelling over the affected joints. Treatment consists of local heat, analgesics, anti-inflammatory drugs, or local steroid injections. Most often the pain subsides after a few weeks or months but swelling may persist for longer.

1222. A 58 year-old woman presents with a 2-week history of a mild ache and prickling sensation in her right breast. She is worried as she thinks she can now feel a lump in the area of concern. She has had a previous hysterectomy and bilateral salpingo-oophorectomy at the age of 45 and has been on hormone replacement therapy since. There is no family history of breast disease. On examination a nontender, hard lump is palpable in the upper outer quadrant of the right breast. Please select the most appropriate diagnosis from the list below. A. Aberrations of normal development and involution (ANDI) B. Breast abscess C. Cyclical mastalgia

Prepared by Dr: Mohammed Musa Brema Idress – My best wishes Page 534

D. Tietze’s syndrome E. Breast carcinoma

ANSWER IS E Pain is an uncommon feature of breast carcinoma; however, when it does occurs it is typically described as an ache or prickling sensation in the breast. This tends to draw the patient’s attention to the more common complaint of a lump in the breast.

1223. A vicar who missed breakfast and swore during his sermon felt faint and got better after a late breakfast. Select the most likely cause for the presentation from the list of options given below. A. Zollinger-Ellison syndrome B. Insulinoma C. Conn's syndrome D. Glucagonoma E. VIPoma (VIP is vasoactive polypeptide).

ANSWER IS B

1224. A 52-year-old man with hypercalcaemia and recurrent stomach ulcers. Select the most likely cause for the presentation from the list of options given below. A. Zollinger-Ellison syndrome B. Insulinoma C. Conn's syndrome D. Glucagonoma E. VIPoma (VIP is vasoactive polypeptide).

ANSWER IS A Hypoglycaemia (i.e. in this case secondary to an insulinoma) can lead to alteration of neurological and cerebral functioning. The Zollinger-Ellison syndrome is associated with markedly elevated levels of gastrin and gastric acid hypersecretion. This gives rise to severe ulceration, not only in the stomach and duodenum but also in the jejunum.

1225. Man with serum Ca2+ of 2.6 mmol / liter, with renal stones and urine Ca2+ of 10 mmol / liter per day. Select the most likely diagnosis for this case. A. Primary hyperparathyroidism B. Secondary hyperparathyroidism C. Parathyroid carcinoma D. Hyperalciuric normocalcaemia E. Paget's disease.

ANSWER IS D

1226. A 50-year-old man with a corrected Ca2+ of 2.15 mmol/liter, a urinary Ca2+ of 10 mmol/day and an increased parathyroid hormone (PTH) level. Select the most likely diagnosis for this case.

Prepared by Dr: Mohammed Musa Brema Idress – My best wishes Page 535

A. Primary hyperparathyroidism B. Secondary hyperparathyroidism C. Parathyroid carcinoma D. Hyperalciuric normocalcaemia E. Paget's disease.

ANSWER IS B

1227. A 21 year old man is involved in a road traffic accident. After a transient period of concussion he is found to have a GCS of 15 by the paramedics. On arrival at hospital he is monitored in a side room of the emergency department. When he is next observed he is noted to have a GCS of 3 and a blown right pupil. Which of the processes below best accounts for this deterioration? A. Hydrocephalus B. Intraventricular bleed C. Subdural bleed D. Transtentorial herniation E. Subarachnoid haemorrhage

ANSWER IS D The presence of a blown right pupil is a sign of a third cranial nerve compression. The most likely cause is an extradural bleed. However, since this option is not listed the process of transtentorial herniation would be the most applicable answer. Intraventricular bleeds are typically more common in premature neonates, deterioration due to hydrocephalus is more chronic.

1228. A 53 year old teacher is admitted to the vascular ward for a carotid endarterectomy. Your FY1 does a preoperative assessment and notes that there is a right homonymous hemianopia. There is no other neurology. What is the most likely cause? A. Lateral medullary syndrome B. Middle cerebral artery infarct C. Anterior cerebral artery infarct D. Cerebellar infarct E. Posterior cerebral artery infarct

ANSWER E This patient has had a left occipital infarct, as there is only a homonymous hemianopia. If this patient had a temporal or parietal lobe infarct, there would be associated hemiparesis and higher cortical dysfunction. This is important to differentiate, as the carotid endarterectomy is inappropriate in this patient as the lesion is in the posterior cerebral artery.

1229. A 52 year man is admitted to the vascular ward for an amputation. The patient complains of unsteadiness. On further examination you detect right facial numbness and right sided nystagmus. There is sensory loss of the left side of the body. What is the most likely lesion?

Prepared by Dr: Mohammed Musa Brema Idress – My best wishes Page 536

A. Lateral medullary syndrome B. Posterior cerebral artery infarct C. Pontine infarct D. Lacunar infarct E. Anterior cerebral artery infarct

ANSWER IS A A combination of ipsilateral ataxia, nystagmus, dysphagia, facial numbness, cranial nerve palsy with contralateral hemisensory loss indicates this diagnosis.

1230. A 33 year old lady develops a thunderclap headache and collapses. A CT scan shows that she has developed a subarachnoid haemorrhage. She currently has no evidence of raised intracranial pressure. Which of the following drugs should be administered? A. None B. Atenotol C. Labetolol D. Nimodipine E. Mannitol

ANSWER IS D Nimodipine is a calcium channel blocker. It reduces cerebral vasospasm and improves outcomes. It is administered to most cases of sub arachnoid haemorrhage.

1231. What is the most likely explanation for a 48 year old type 2 diabetic who complains of numbness in his left arm and leg. Otherwise there is no other neurological signs. A. Lacunar infarct B. Lateral medullary syndrome C. Pontine infarct D. Middle cerebral artery infarct E. Pituitary mass

ANSWER IS A Isolated hemisensory loss is a feature of a lacunar infarct.

1232. A 65 year old male with known nasopharyngeal carcinoma presents with double vision over a few weeks. On examination he is found to have left eye proptosis and it is down and out. He reports pain on attempting to move the eye. There is an absent corneal reflex. What is the most likely diagnosis? A. Posterior communicating artery aneurysm B. Cavernous sinus syndrome C. Optic nerve tumour D. Migraine E. Cerebral metastases

ANSWER IS B

Prepared by Dr: Mohammed Musa Brema Idress – My best wishes Page 537

Cavernous sinus syndrome is most commonly caused by cavernous sinus tumours. In this case, the nasopharyngeal malignancy has locally invaded the left cavernous sinus. Diagnosis is based on signs of pain, opthalmoplegia, proptosis, trigeminal nerve lesion (opthalmic branch) and Horner's syndrome.

1233. A patient is referred due to the development of a third nerve palsy associated with a headache. On examination, meningism is present. Which one of the following diagnoses needs to be urgently excluded? A. Weber's syndrome B. Internal carotid artery aneurysm C. Multiple sclerosis D. Posterior communicating artery aneurysm E. Anterior communicating artery aneurysm

ANSWER IS D Painful third nerve palsy = posterior communicating artery aneurysm Given the combination of a headache and third nerve palsy it is important to exclude a posterior communicating artery aneurysm

1234. A 23 year old man was driving a car at high speed whilst intoxicated, he was wearing a seat belt. The car collides with a brick wall at around 140km/h. When he arrives in the emergency department he is comatose. His CT scan appears to be normal. He remains in a persistent vegetative state. What is the most likely underlying cause? A. Extradural haemorrhage B. Subdural haemorrhage C. Subarachnoid haemorrhage D. Intracerebral haemorrhage E. Diffuse axonal injury

ANSWER IS E Diffuse axonal injury occurs when the head is rapidly accelerated or decelerated. There are 2 components: 1. Multiple haemorrhages 2. Diffuse axonal damage in the white matter Up to 2/3 occur at the junction of grey/white matter due to the different densities of the tissue. The changes are mainly histological and axonal damage is secondary to biochemical cascades. Often there are no signs of a fracture or contusion.

1235. A middle aged lady is brought to the clinic by her husband who has noted a change in her appearance. She finds removal of rings difficult, her shoe size has changed and photographs show a marked change in her appearance. Which of the following is most likely to be identified on neurological examination? A. Binasal hemianopia B. Bitemporal hemianopia C. Inferior quadrantanopia D. Homonymous hemianopia

Prepared by Dr: Mohammed Musa Brema Idress – My best wishes Page 538

E. Unilateral loss of vision

ANSWER IS B The patient is most likely to have developed acromegaly. Since a pituitary lesion is likely to be present; compression of the optic chiasm may occur.

1236. A 25-year-old female with a history of bilateral vitreous haemorrhage is referred with bilateral lesions in the cerebellar region. What is the likely diagnosis? A. Neurofibromatosis type I B. Neurofibromatosis type II C. Tuberous sclerosis D. Von Hippel-Lindau syndrome E. Sarcoidosis

ANSWER IS D Retinal and cerebellar haemangiomas are key features of Von Hippel-Lindau syndrome. Retinal haemangiomas are bilateral in 25% of patients and may lead to vitreous haemorrhage

1237. A 28 year old man falls and hits his head against a wall. There is a brief loss of consciousness. When assessed in accident and emergency he is alert and orientated with a GCS of 15, imaging shows no fracture of the skull. What is his risk of having an intracranial haematoma requiring removal? A. 1 in 6000 B. 1 in 40 C. 1 in 4 D. 1 in 50,000 E. 1 in 120

ANSWER IS A

1238. The term signature fracture is synonymous with which of the following injuries? A. Depressed skull fracture B. Le Fort II fracture C. Orbital blow out D. Oblique fracture of the tibia E. Supracondylar fracture

ANSWER IS A Signature fractures are synonymous with depressed skull fractures, they are usually low velocity injuries where the fracture impression resembles the injurious source.

1239. Which of the following is not a form of primary brain injury? A. Subdural haemorrhage after being hit in the head with a hammer B. Meningitis resulting from infected CSF rhinorrhoea after a basal skull fracture

Prepared by Dr: Mohammed Musa Brema Idress – My best wishes Page 539

C. A truck driver is involved in a road traffic accident and suffers an axonal stretch injury D. A man is hit with a baseball bat and suffers a cerebral contusion E. A man suffers an intraparenchymal haemorrhage after being hit in head during a car crash

ANSWER IS B Primary brain damage occurs at the point of injury. It includes contusions and diffuse axonal injury. Non reversible. Secondary brain damage occurs after the injury. Complications include: 1. Haemorrhage 2. Meningitis 3. Herniation 4. Hypoxia 5. Oedema 6. Arterial damage: internal carotid, vertebral artery common

1240. With which of the conditions listed below is Boas' sign classically associated? A. Perforation of the thoracic oesophagus B. Acute cholecystitis C. Hepatocellular carcinoma D. Closed loop small bowel obstruction E. Acute diverticulitis

ANSWER IS B Boas' sign refers to hyperaesthesia of the tip of the right scapula and is seen classically in association with acute cholecystitis.

1241. What type of stoma is most likely to be encountered in a 56 year old man undergoing a low anterior resection for carcinoma of the rectum with a colorectal anastomosis? A. Loop colostomy B. End colostomy C. End ileostomy D. Loop ileostomy E. Caecostomy

ANSWER IS D Colonic resections with an anastomosis below the peritoneal reflection may have an anastomotic leak rate (both clinical and radiological) of up to 15%. Therefore most surgeons will defunction such an anastomosis to reduce the clinical severity of an anastomotic leak. A loop ileostomy will achieve this end point and is relatively easy to reverse. Loop colostomy is less popular in this setting as reversal can compromise the blood supply to the anastomosis.

Prepared by Dr: Mohammed Musa Brema Idress – My best wishes Page 540

1242. A 23 year old lady is persistently vomiting following a laparoscopic appendicectomy for a perforated gangrenous appendicitis. Imaging shows some dilated small bowel loops. What is the most appropriate course of action? A. Insertion of wide bore nasogastric tube B. Insertion of narrow bore nasogastric tube C. Administration of intravenous cyclizine D. Administration of metoclopramide E. Arrange a laparotomy

ANSWER IS A This patient is likely to have a paralytic ileus and the administration of anti emetic drugs in this situation achieves very little. It's important to decompress the stomach and this can be achieved with a wide bore nasogastric tube.

1243. A 78 year old lady is admitted with small bowel obstruction. On examination, she has a distended abdomen and the leg is held semi flexed. She has some groin pain radiating to the ipsilateral knee. What is the most likely diagnosis A. Inguinal hernia B. Obturator hernia C. Lumbar hernia D. Spigelian hernia E. Incisional hernia

ANSWER IS B The groin swelling in obturator hernia is subtle and hard to elicit clinically. There may be pain in the region of sensory distribution of the obturator nerve. The defect is usually repaired from within the abdomen.

1244. A 53 year old man undergoes a reversal of a loop colostomy. He recovers well and is discharged home. He is readmitted 10 days later with symptoms of vomiting and colicky abdominal pain. On examination he has a swelling of the loop colostomy site and it is tender. What is the most likely underlying diagnosis? A. Haematoma B. Intra abdominal adhesions C. Anastomotic leak D. Anastomotic stricture E. Obstructed incisional hernia

ANSWER IS E In this scenario the most likely diagnosis would be obstructed incisional hernia. The tender swelling coupled with symptoms of obstruction point to this diagnosis. Prompt surgical exploration is warranted. Loop colostomy reversals are at high risk of this complication as the operative site is at increased risk of the development of post operative wound infections.

1245. A 25 year-old lady presents to her GP complaining of a two day history of right upper quadrant pain, fever and a white vaginal discharge. She has seen the

Prepared by Dr: Mohammed Musa Brema Idress – My best wishes Page 541

GP twice in 12 weeks complaining of pelvic pain and dyspareunia. What is the most likely cause? A. Appendicitis B. Adnexial torsion C. Endometriosis D. Pelvic inflammatory disease E. Ruptured ectopic pregnancy

ANSWER IS D The most likely diagnosis is pelvic inflammatory disease. Right upper quadrant pain occurs as part of the Fitz Hugh Curtis syndrome in which perihepatic inflammation occurs.

1246. A 34 year old man undergoes a sub total colectomy to treat fulminant ulcerative colitis. What type of stoma is most likely to be fashioned? A. End colostomy B. Loop colostomy C. End ileostomy D. Loop ileostomy E. End jejunostomy

ANSWER IS C A subtotal colectomy involves the removal of the entire right, transverse, left and part of the sigmoid colon. The rectal stump is closed and an end ileostomy fashioned in the right iliac fossa.

1247. A 73 year old lady presents with peritonitis and tenderness of the left groin. At operation, she has a left femoral hernia with perforation of the anti mesenteric border of ileum associated with the hernia. What type of hernia is this? A. Richter’s hernia B. Littre’s hernia C. Morgagni hernia D. Spigelian hernia E. Bochdalek hernia

ANSWER IS A When part of the bowel wall is trapped in a hernia such as this it is termed a Richter’s hernia and may complicate any hernia although femoral and obturator hernias are most typically implicated.

1248. In which of the conditions described below is Rovsing's sign most likely to be absent? A. Locally advanced caecal cancer B. Paraileal appendicitis C. Right sided colonic diverticulitis D. Retrocaecal appendicitis E. Severe terminal ileal Crohns disease

Prepared by Dr: Mohammed Musa Brema Idress – My best wishes Page 542

ANSWER IS D Any advanced right iliac fossa pathology can result in a positive Rovsings sign. However, in retrocaecal appendicitis, it may be absent and this fact can contribute to a delayed diagnosis if undue weight is placed on the presence of the sign in making the diagnosis.

1249. An 8 year old boy presents with a 4 hour history of right iliac fossa pain with nausea and vomiting. He has been back at school for two days after being kept home with a flu like illness. On examination, he is tender in the right iliac fossa, although his abdomen is soft. Temperature is 39.3oC. Blood tests show a CRP of 40 and a WCC of 8.1. What is the most appropriate course of action? A. Abdominal MRI scan B. Abdominal CT scan C. Diagnostic laparoscopy D. Active observation E. Colonoscopy

ANSWER IS D The key point in the history is the preceding flu like illness and absence of abdominal signs. These make mesenteric adenitis the most likely diagnosis. The patient should have a period of active observation, if this were to represent early appendicitis, then the clinical picture may change and this would be detected with serial examination.

1250. A 6 day old child is suspected of having a malrotation and requires urgent abdominal exploration. What is the most appropriate surgical approach? A. Midline abdominal incision B. Paramedian incision C. Transverse supra umbilical abdominal incision D. Transverse infra umbilical abdominal incision E. Battle incision

ANSWER IS C In young children, laparotomy is performed via transverse supra umbilical incision. Access via midline incisions is very poor and they should not be used.

1251. A 14 year old female is admitted with sudden onset right iliac fossa pain. She is otherwise well and on examination has some right iliac fossa tenderness but no guarding. She is a febrile. Urinary dipstick is normal. Her previous menstrual period two weeks ago was normal and pregnancy test is negative. What is the most likely underlying diagnosis? A. Mittelschmerz B. Endometriosis C. Appendicitis D. Crohns disease E. Pelvic abscess

ANSWER IS A

Prepared by Dr: Mohammed Musa Brema Idress – My best wishes Page 543

The timing of the pain and the fact that it is mid cycle makes Mittelschmerz the most likely cause. When follicular cysts rupture, there is sometimes associated bleeding of small volume. The cyst contents and blood if present can be very irritant and pain can sometimes mimic appendicitis or pelvic inflammatory disease. However, normal inflammatory markers makes the former less likely and unless the 14 year old was sexually active, PID is also relatively rare in this group. Crohns disease and pelvic abscesses would typically present with a more protracted history and raised inflammatory markers and fever.

1252. A 1 day old infant is born with severe respiratory compromise. On examination, he has a scaphoid abdomen and an absent apex beat. Which of the following anomalies is most likely? A. Situs inversus B. Morgagni hernia C. Necrotizing enterocolitis D. Bochdalek hernia E. Cystic fibrosis

ANSWER IS D A hernia is the most likely diagnosis given the abdominal findings. The large hernia may displace the heart although true dextrocardia is not present. The associated pulmonary hypoplasia will compromise lung development.

1253. Which abdominal sign is described as being present when a patient with cholecystitis experiences pain on palpation of the right upper quadrant most marked on inspiration? A. Murphy's sign B. Boas' sign C. Rovsing's sign D. Cullen’s sign E. Grey Turners sign

ANSWER IS A

1254. Of the surgical incisions listed below, which is most suitable for a 45 year old female undergoing a first time renal transplant? A. Abdominal midline B. Paramedian C. Battle D. Rutherford Morrison E. Pfannenstiel

ANSWER IS D A Rutherford Morrison incision is the traditional approach for a renal transplant and provides extra peritoneal access to the iliac vessels.

Prepared by Dr: Mohammed Musa Brema Idress – My best wishes Page 544

1255. A 16 year old female presents to the emergency department with a 12 hour history of pelvic discomfort. She is otherwise well and her last normal menstrual period was 2 weeks ago. On examination, she has a soft abdomen with some mild supra pubic discomfort. What is the most likely underlying cause? A. Ruptured ectopic pregnancy B. Degenerating fibroid C. Pelvic inflammatory disease D. Appendicitis E. Mittelschmerz

ANSWER IS E Mid cycle pain is very common and is due to the small amount of fluid released during ovulation. Inflammatory markers are usually normal and the pain typically subsides over the next 24-48 hours.

1256. An 8 year old boy is examined by his doctor as part of a routine clinical examination. The doctor notices a smooth swelling in the right iliac fossa. It is mobile and he is otherwise well. What is the most likely underlying cause? A. Meckels diverticulum B. Spigelian hernia C. Mesenteric cyst D. Appendix mass E. Liposarcoma

ANSWER IS C Mesenteric cysts are often smooth. Imaging with ultrasound and CT is usually sufficient. Although rare, they most often occur in young children (up to 30% present before the age of 15). Many are asymptomatic and discovered incidentally. Acute presentations are recognized and may occur following cyst torsion, infarction or rupture. Most cysts will be surgically resected. Spigelian hernias are very rare in children, liposarcomas are not smooth swellings. An appendix mass will usually produce systemic illness.

1257. An 84 year old lady presents with a tender painful lump in the right groin and signs of small bowel obstruction. What is the most appropriate surgical incision to address this problem? A. Lothessien B. McEvedy C. Midline abdominal D. Paramedian E. Kocher’s

ANSWER IS B A McEvedy incision is traditionally used to approach incarcerated femoral hernias. The disadvantage of the Lothessien approach is that it weakens the inguinal canal and predisposes to inguinal hernia formation. The other incisions would not usually

Prepared by Dr: Mohammed Musa Brema Idress – My best wishes Page 545 address femoral hernias. Given the features of bowel obstruction, a low approach would be inappropriate.

1258. A 19 year old lady is admitted with lower abdominal pain. On examination, she is diffusely tender. A laparoscopy is performed and at operation multiple fine adhesions are noted between the liver and abdominal wall. Her appendix is normal. What is the most likely diagnosis? A. Mesenteric infarct B. Fitz Hugh Curtis Syndrome C. Perforated peptic ulcer D. Appendicitis E. Pancreatitis

ANSWER IS B This is Fitz Hugh Curtis syndrome in which pelvic inflammatory disease (usually Chlamydia) causes the formation of fine perihepatic adhesions.

1259. A 21 year old women is admitted with a 48 hour history of worsening right iliac fossa pain. She has been nauseated and vomited twice. On examination, she is markedly tender in the right iliac fossa with localized guarding. Vaginal examination is unremarkable. Urine dipstick (including beta HCG) is negative. Blood tests show a WCC of 13.5 and CRP 70. What is the most appropriate course of action? A. Open appendicectomy B. Laparotomy C. Abdominal ultrasound D. Laparoscopic appendicectomy E. Abdominal CT scan

ANSWER IS D The most likely diagnosis is appendicitis. The negative vaginal examination (and therefore by definition the absence of cervical excitation) makes pelvic inflammatory disease unlikely. Given the raised inflammatory markers, the correct course of action is to proceed with surgery. In females, there are considerable advantages of undertaking this laparoscopically as it allows evaluation of the pelvic viscera. Imaging with USS is unlikely to alter management as it has a false negative rate and given the context of the clinical findings, surgery is likely to occur in any case. Whilst a CT scan would allow for an accurate pre-operative diagnosis, it carries a significant radiation dose, and again, is unlikely to alter management.

1260. Which of the following signs is seen in patients who have a significant retroperitoneal haemorrhage? A. Boas' sign B. Pembertons sign C. Grey Turners sign D. Cullen’s sign E. Rovsing's sign

Prepared by Dr: Mohammed Musa Brema Idress – My best wishes Page 546

ANSWER IS C Bruising of the flank is described as Grey Turners sign

1261. What type of stoma should be considered in a patient undergoing emergency operative intervention for large bowel obstruction as a result of a carcinoma 5cm from the anal verge? A. End colostomy B. Loop colostomy C. End ileostomy D. Loop jejunostomy E. Loop ileostomy

ANSWER IS B Large bowel obstruction resulting from carcinoma should be resected, stented or defunctioned. The first two options typically apply to tumours above the peritoneal reflection. Lower tumours should be defunctioned with a loop colostomy and then formal staging undertaken prior to definitive surgery. An emergency attempted rectal resection carries a high risk of involvement of the circumferential resection margin and is not recommended.

1262. A 22 year old man is operated on for a left inguinal hernia, at operation the sac is opened to reveal a large Meckels diverticulum. What type of hernia is this? A. Richter’s hernia B. Morgagni hernia C. Littre’s hernia D. Spigelian hernia E. Bochdalek hernia

ANSWER IS C Hernia containing Meckels diverticulum is termed a Littre’s hernia.

1263. Which of the following agents increases the rate of emptying of the vagotomised stomach? A. Metoclopramide B. Ondansetron C. Cyclizine D. Erythromycin E. Ciprofloxacin

ANSWER IS D Erythromycin increases the rate of gastric emptying and increases GI transit times in general. This accounts for its side effect profile (diarrhea). It can be a useful agent in diabetic gastropathy.

1264. A 78 year old lady presents with colicky abdominal pain and a tender mass in her groin. On examination; there is a small firm mass below and lateral to the pubic tubercle. Which of the following is the most likely underlying diagnosis?

Prepared by Dr: Mohammed Musa Brema Idress – My best wishes Page 547

A. Incarcerated inguinal hernia B. Thrombophlebitis of a saphena varix C. Incarcerated femoral hernia D. Incarcerated obturator hernia E. Deep vein thrombosis

ANSWER IS C Femoral hernia = High risk of strangulation (repair urgently) Femoral hernia account for <10% of all groin hernias. In the scenario the combination of symptoms of intestinal compromise with a mass in the region of the femoral canal points to femoral hernia as the most likely cause.

1265. Which of the following is not a typical feature of acute appendicitis? A. Neutrophilia B. Profuse vomiting C. Anorexia D. Low grade pyrexia E. Small amounts of protein on urine analysis

ANSWER IS B Profuse vomiting and diarrhoea are rare in early appendicitis Whilst patients may vomit once or twice, profuse vomiting is unusual, and would fit more with gastroenteritis or an ileus. A trace of protein is not an uncommon occurrence in acute appendicitis. A free lying pelvic appendix may result in localized bladder irritation, with inflammation occurring as a secondary phenomena. This latter feature may result in patients being incorrectly diagnosed as having a urinary tract infection. A urine dipstick test is useful in differentiating between the two conditions.

1266. A 75 year old man is admitted with sudden onset severe generalized abdominal pain, vomiting and a single episode of bloody diarrhoea. On examination, he looks unwell and is in uncontrolled atrial fibrillation. Although diffusely tender his abdomen is soft. What is the most likely diagnosis? A. Pancreatitis B. Infective diarrhoea C. Ischaemic colitis D. Crohns disease E. Mesenteric infarction

ANSWER IS E Pain out of proportion to physical signs, AF and generalized abdominal pain suggest widespread infarction.

1267. An 28 year old man presents with a direct inguinal hernia. A decision is made to perform an open inguinal hernia repair. Which of the following is the best option for abdominal wall reconstruction in this case? A. Suture plication of the transversalis fascia using PDS only

Prepared by Dr: Mohammed Musa Brema Idress – My best wishes Page 548

B. Suture plication of the hernial defect with nylon and placement of prolene mesh anterior to external oblique C. Suture plication of the hernia defect using nylon and re-enforcing with a sutured repair of the abdominal wall D. Sutured repair of the hernial defect with prolene and placement of prolene mesh over the cord structures in the inguinal canal E. Sutured repair of the hernial defect using nylon and placement of a prolene mesh posterior to the cord structures

ANSWER IS E - Laparoscopic repair- bilateral and recurrent cases During an inguinal hernia repair in males the cord structures will always lie anterior to the mesh. In the conventional open repairs the cord structures are mobilized and the mesh placed behind them, with a slit made to allow passage of the cord structures through the deep inguinal ring. Placement of the mesh over the cord structures results in chronic pain and usually a higher risk of recurrence. - Laparoscopic inguinal hernia repair is the procedure of choice for bilateral inguinal hernias.

Types of surgery include: 1. Onlay mesh repair (Lichtenstein style) 2. Inguinal herniorrhaphy 3. Shouldice repair 4. Darn repair 5. Laparoscopic mesh repair Open mesh repair and laparoscopic repair are the two main procedures in mainstream use. The Shouldice repair is a useful procedure in cases where a mesh repair would be associated with increased risk of infection, e.g. repair of case with strangulated bowel, as it avoids the use of mesh. It is, however, far more technically challenging to perform.

1268. A 17 year old male is admitted with lower abdominal discomfort. He has been suffering from intermittent right iliac fossa pain for the past few months. His past medical history includes a negative colonoscopy and gastroscopy for iron deficiency anaemia. The pain is worse after meals. Inflammatory markers are normal. What is the most likely cause? A. Appendicitis B. Crohns disease C. Peptic ulcer disease D. Meckels diverticulum E. Irritable bowel syndrome

ANSWER IS D This scenario should raise suspicion for Meckels as these may contain ectopic gastric mucosa which may secrete acid with subsequent bleeding and ulceration. The iron deficiency anaemia makes a Meckels more likely than IBD.

Prepared by Dr: Mohammed Musa Brema Idress – My best wishes Page 549

1269. Which of the following is not a typical feature of irritable bowel syndrome? A. A change in the consistency of stools B. Abdominal pain relieved with defecation C. A change in frequency of defecation D. Abdominal bloating E. Pain at a single fixed site

ANSWER IS E The pain or discomfort of IBS is typically migratory and variable in intensity. Pain at a fixed site is suggestive of malignancy. Abdominal bloating is an extremely common feature.

1270. A 2 month old infant is troubled by recurrent colicky abdominal pain and intermittent intestinal obstruction. On imaging, the transverse colon is herniated into the thoracic cavity, through a mid line defect. What is the most likely defect? A. Bochdalek hernia B. Morgagni hernia C. Littre’s hernia D. Paraoesophageal hernia E. Hiatus hernia

ANSWER IS B Morgagni hernia may contain the transverse colon. Unless there is substantial herniation, pulmonary hypoplasia is uncommon. As a result, major respiratory compromise is often absent. Para oesophageal and hiatus hernias are very rare in children.

1271. A 56 year old lady is admitted with colicky abdominal pain. A plain x-ray is performed. Which of the following should not show fluid levels on a plain abdominal film? A. Stomach B. Jejunum C. Ileum D. Caecum E. Descending colon

ANSWER IS E Fluid levels in the distal colon are nearly always pathological. In general contents of the left colon transit quickly and are seldom held in situ for long periods, the content is also more solid.

1272. A 21 year old women presents with right iliac fossa pain. She reports some bloodstained vaginal discharge. On examination, she is a febrile and has a pulse rate of 97 bpm, normal blood pressure. She has diffuse lower abdominal tenderness. What is the most appropriate course of action? A. Laparotomy B. Laparoscopy

Prepared by Dr: Mohammed Musa Brema Idress – My best wishes Page 550

C. Abdominal and pelvic MRI scan D. Abdominal and pelvic CT scan E. Abdominal and pelvic USS

ANSWER IS E The history of blood stained discharge and tenderness makes an ectopic pregnancy a strong possibility, a USS should be performed and a pregnancy test undertaken. If the beta HCG is high then an intra uterine pregnancy should be found. If it is not, then an ectopic pregnancy is likely and surgery should be considered.

1273. A 56 year old lady presents with a large bowel obstruction and abdominal distension. Which of the following confirmatory tests should be performed prior to surgery? A. Abdominal ultrasound scan B. Barium enema C. Rectal MRI Scan D. Endoanal ultrasound scan E. Gastrograffin enema

ANSWER IS E Patients with suspected large bowel obstruction due to tumour should have this confirmed with gastrograffin enema, sigmoidoscopy or CT scanning prior to surgery. Patients with clinical evidence of large bowel obstruction, should have the presence or absence of an obstructing lesion confirmed prior to surgery. This is because colonic pseudo-obstruction may produce a similar radiological picture. A gastrograffin enema is the traditional test, as barium is too toxic if it spills into the abdominal cavity. An MRI scan will not provide the relevant information, unless the lesion is rectal and below the peritoneal reflection. A CT scan would be an acceptable alternative.

1274. A 21 year old male is admitted with a 3 month history of intermittent right iliac fossa pain. He suffers from episodic diarrhoea and has lost 2 kilos in weight. On examination, he has some right iliac fossa tenderness and is febrile. What is the most likely cause? A. Appendicitis B. Irritable bowel syndrome C. Inflammatory bowel disease D. Infective gastroenteritis E. Meckels diverticulum

ANSWER IS C The history of weight loss and intermittent diarrhea makes inflammatory bowel disease the most likely diagnosis. Conditions such as appendicitis and infections have a much shorter history. Although Meckels can bleed and cause inflammation, they seldom cause marked weight loss.

Prepared by Dr: Mohammed Musa Brema Idress – My best wishes Page 551

1275. A 78 year old man is walking to the bus stop when he suddenly develops severe back pain and collapses. On examination he has a blood pressure of 90/40 and pulse rate of 110. His abdomen is distended and he is obese. Though tender his abdomen itself is soft. What is the most likely diagnosis? A. Ruptured abdominal aortic aneurysm B. Perforated peptic ulcer C. Appendicitis D. Mesenteric infarction E. Perforated diverticulitis

ANSWER IS A This will be a retroperitoneal rupture (anterior ones generally don't survive to hospital). The debate regarding CT varies, it is the authors opinion that a systolic BP of <100mmHg at presentation mandates immediate laparotomy.

1276. Which of these factors does not increase the risk of abdominal wound dehiscence following laparotomy? A. Jaundice B. Abdominal compartment syndrome C. Poorly controlled diabetes mellitus D. Administration of intravenous steroids E. Use of Ketamine as an anaesthetic agent

ANSWER IS E Ketamine does not affect healing. All the other situations in the list carry a strong association with poor healing and risk of dehiscence.

1277. A 6 year old child presents with colicky abdominal pain, vomiting and the passage of red currant jelly stool per rectum. On examination the child has a tender abdomen and a palpable mass in the right upper quadrant. Imaging shows an intussusception. Which of the conditions below is least recognized as a precipitant? A. Inflammation of Peyers patches B. Cystic fibrosis C. Meckels diverticulum D. Mesenteric cyst E. Mucosal polyps

ANSWER IS D Mesenteric cysts may be associated with intra abdominal catastrophes where these occur they are typically either intestinal volvulus or intestinal infarction. They seldom cause intussusception. Cystic fibrosis may lead to the formation of meconium ileus equivalent and plugs may occasionally serve as the lead points for an intussusception.

1278. Which one of the following is least likely to cause malabsorption? A. Primary biliary cirrhosis

Prepared by Dr: Mohammed Musa Brema Idress – My best wishes Page 552

B. Ileo-colic bypass C. Chronic pancreatitis D. Whipples disease E. Hartmans procedure

ANSWER IS E In a Hartmans procedure the sigmoid colon is removed and an end colostomy is fashioned. The bowel remains in continuity and no absorptive ability is lost. An ileo- colic bypass leaves a redundant loop of small bowel in continuity, where the contents will stagnate and bacterial overgrowth will occur. Therefore this is recognized cause of malabsorption.

1279. Which of the following anti emetic drugs targets the chemoreceptor trigger zone of the area postrema? A. Metoclopramide B. Ondansetron C. Cisapride D. Cyclizine E. Domperidone

ANSWER IS B - 5 HT3 blockers are most effective for many types of nausea for this reason.

1280. A 10 year old girl has right iliac fossa pain and an appendicectomy is to be performed. What is the best incision for this procedure? A. Paramedian B. Midline abdominal C. Kockers D. Lanz E. Gridiron

ANSWER IS D Both Lanz and Gridiron incisions can be used to perform an open appendicectomy. However, a Lanz incision provides for a superior cosmetic scar and is more easily extended than a Gridiron incision.

1281. A 21 year old man is admitted with a tender mass in the right groin, fevers and sweats. He is on multiple medical therapy for HIV infection. On examination, he has a swelling in his right groin, hip extension exacerbates the pain. What is the most likely cause? A. Septic arthritis B. Psoas abscess C. Infected lymph node D. HIV related lymphadenopathy E. Femoral hernia

Prepared by Dr: Mohammed Musa Brema Idress – My best wishes Page 553

ANSWER IS B Psoas abscesses may be either primary or secondary. Primary cases often occur in the immunosuppressed and may occur as a result of haematogenous spread. Secondary cases may complicated intra abdominal diseases such as Crohns. Patients usually present with low back pain and if the abscess is extensive a mass that may be localized to the inguinal region or femoral triangle . Smaller collections may be percutaneously drained. If the collection is larger, or the percutaneous route fails, then surgery (via a retroperitoneal approach) should be performed.

1282. Which of the following interventions is most likely to reduce the incidence of intra abdominal adhesions? A. Peritoneal lavage with cetrimide following elective right hemicolectomy B. Use of a laparoscopic approach over open surgery C. Use of talc to coat surgical gloves D. Performing a Nobles plication of the small bowel E. Using stapled rather than a hand sewn anastamosis

ANSWER IS B Laparoscopy results in fewer adhesions. When talc was used to coat surgical gloves it was a major cause of adhesion formation and withdrawn for that reason. A Nobles plication is an old fashioned operation which has no place in the prevention of adhesion formation. Use of an anastamotic stapling device will not influence the development of adhesions per se although clearly an anastamotic leak will result in more adhesion formation.

1283. In which of the following scenarios is a mucous fistula most likely to be encountered? A. Following an elective right hemicolectomy and ileo-colic anastomosis B. Following an Ivor Lewis oesophagectomy C. Following an emergency subtotal colectomy for severe colitis D. Following an abdomino perineal excision of the colon and rectum for rectal cancer E. Following a small bowel resection and primary anastomosis for incarcerated femoral hernia

ANSWER IS C A mucous fistula is a conduit between the skin and a redundant segment of bowel. They are typically constructed because the section of bowel that is exteriorized is at very high risk of breakdown. They are not the same as an end stoma, by definition they are usually seen in patients who have an end stoma. They are typically seen following a sub total colectomy where the distal sigmoid colon is deemed too friable to close and it then brought onto the skin as a mucous fistula.

1284. Which of the following statements relating to a burst abdomen is false? A. Is seen in 1-2% of modern laparotomies B. Is more common in faecal peritonitis C. Is less common when a 'mass closure' technique is used

Prepared by Dr: Mohammed Musa Brema Idress – My best wishes Page 554

D. When it does occur is most common at 15 days E. Is similar in incidence regardless of whether 1/0 polydiaxone or 1/0 polypropylene are used

ANSWER IS D When it does occur, a burst abdomen is most common at 6 days and is usually the result of technical error when Jenkins rule is not followed and sutures are placed in the zone of collagenolysis. The choice of materials given above does not influence dehiscence rates.

1285. A 67 year old man is due to undergo Whipples procedure to resect a pancreatic adenocarcinoma. What is the most appropriate surgical incision to address this? A. Rooftop B. Kockers C. Paramedian D. Battle E. Thoracoabdominal

ANSWER IS A A rooftop incision is typically used to access the pancreas for resectional surgery.

1286. A 35 year old women who is a heavy smoker has long standing stool frequency and crampy abdominal pain. A colonoscopy is performed (which is macroscopically normal) and pan colonic biopsies are taken. Histological analysis of the biopsies demonstrates a thickened sub apical collagen layer together with an increase in lymphocytes in the lamina propria. What is the most likely diagnosis? A. Microscopic colitis B. Crohns disease C. Ulcerative colitis D. Pseudomembranous colitis E. Irritable bowel syndrome

ANSWER IS A Microscopic colitis is a common condition characterized by normal endoscopic appearances, microscopic features of colonic inflammation and thickening of the sub epithelial collagen layer. Features such as granulomas are absent. It is the normal endoscopic appearance that makes the other options less likely. IBS is a diagnosis of exclusion and the features of inflammation activity would favor a different diagnosis. Severe cases can be treated with steroids. Other agents such as cholestyramine may be beneficial. It is important to exclude infection with a stool culture if this has not been done, not least because the precipitant of many cases of microscopic colitis is an episode of infective diarrhea.

1287. A 45 year old man has recurrent colicky abdominal pain. As part of a series of investigations he undergoes a CT scan and this demonstrates a hernia lateral to the rectus muscle at the level of the arcuate line. What type of hernia is this?

Prepared by Dr: Mohammed Musa Brema Idress – My best wishes Page 555

A. Littre’s B. Richter’s C. Spigelian D. Morgagni E. Incisional

ANSWER IS C This is the site for a spigelian hernia.

1288. A 40 year old man presents with a long standing inguinal hernia. On examination he has a small, direct inguinal hernia. He inquires as to the risk of strangulation over the next twelve months should he decide not to undergo surgery. Which of the following most closely matches the likely risk of strangulation over the next 12 months? A. 50% B. 40% C. 25% D. 15% E. <5%

ANSWER IS E The annual probability of strangulation is up to 3% and is more common in indirect hernias. Elective repair poses few risks. However, emergency repair is associated with increased mortality, particularly in the elderly.

1289. An 18 year-old female presents to the Emergency Department with sudden onset sharp, tearing pelvic pain associated with a small amount of vaginal bleeding. She also complains of shoulder tip pain. On examination, she is hypotensive, tachycardic and has marked cervical excitation. What is the most likely explanation? A. Degenerating fibroid B. Adnexial torsion C. Ruptured ectopic pregnancy D. Ruptured appendicitis E. Endometriosis

ANSWER IS C The history of tearing pain and haemodynamic compromise in a women of child bearing years should prompt a diagnosis of ectopic pregnancy.

1290. A 60 year old women has fully recovered from an attack of pancreatitis. Over the following 12 months she develops episodic epigastric discomfort. Un upper GI endoscopy shows gastric varices only. An abdominal CT scan demonstrates a splenic vein thrombosis. What is the treatment of choice? A. Splenectomy B. Insertion of transjugular porto-systemic shunt C. Surgical bypass of the splenic vein

Prepared by Dr: Mohammed Musa Brema Idress – My best wishes Page 556

D. Gastrectomy E. Stapling of the gastro-oesophgeal junction

ANSWER IS A

1291. Which of the following is commonest cause of acute abdominal pain in acute unselected surgical 'take'? A. Non specific abdominal pain B. Biliary colic C. Acute appendicitis D. Ureteric colic E. Pancreatitis

ANSWER IS A Non specific abdominal pain is a commonly recorded diagnosis for patients presenting with acute abdominal pain. Following careful diagnostic work up, a proportion of patients may be identified with disorders such as coeliac disease and the diagnosis of non specific abdominal pain should not be used lightly.

1292. In which of the conditions listed below is Cullen’s sign most likely to be seen? A. Ruptured ectopic pregnancy B. Appendicitis C. Intestinal malrotation D. Perforated peptic ulcer E. Incarcerared femoral hernia

ANSWER IS A Cullen’s sign is seen with significant intra peritoneal haemorrhage.

1293. A 72 year old obese man undergoes an emergency repair of a ruptured abdominal aortic aneurysm. The wound is closed with an onlay prolene mesh to augment the closure. Post operatively he is taken to the intensive care unit. Over the following twenty four hours his nasogastric aspirates increase, his urine output falls and he has a metabolic acidosis. What is the most likely underlying cause? A. Colonic ischaemia B. Abdominal compartment syndrome C. Peritonitis D. Reactionary haemorrhage E. Aorto-duodenal fistula

ANSWER IS B Obese patients with ileus following major abdominal surgery are at increased risk of intra abdominal compartment syndrome. The risk is increased by the use of prosthetic meshes, which some surgeons favor following a major vascular case as they may reduce the incidence of incisional hernia. They prevent abdominal distension and may increase the risk of intra abdominal hypertension in the short

Prepared by Dr: Mohammed Musa Brema Idress – My best wishes Page 557 term. Although colonic ischaemia may occur following major aortic surgery it would not typically present in this way.

1294. What is the commonest site in the abdomen for fluid to collect following a perforated appendix? A. Pelvis B. Hepatorenal pouch C. Between small bowel loops D. Right iliac fossa E. Lesser sac

ANSWER IS A Following perforated appendicitis fluid is most likely to accumulate in the pelvis. Fluid may accumulate in the hepatorenal pouch although this is less common. Gravity favors the pelvis as the site of most collections. The incidence of these is higher with laparoscopic rather than open surgery.

1295. A 56 year old man undergoes a difficult splenectomy and is left with a pancreatic fistula. There are ongoing problems with very high fistula output. Which of the following agents may be administered to reduce the fistula output? A. Metoclopramide B. Erthyromycin C. Octreotide D. Loperamide E. Omeprazole

ANSWER IS C Octreotide is a useful agent in reducing the output from pancreatic fistulae. Prokinetic agents will increase fistula output and should be avoided. 1296. A 52 year old obese lady reports a painless mass in the groin area. A mass is noted on coughing. It is below and lateral to the pubic tubercle. What is the most likely cause? A. Direct inguinal hernia B. Indirect inguinal hernia C. Femoral hernia D. Obturator hernia E. Lipoma

ANSWER IS C A mass below and lateral to the pubic tubercle is indicative of a femoral hernia.

1297. What is the earliest complication that can occur following construction of an ileostomy? A. Prolapse B. Retraction C. Necrosis D. Parastomal hernia

Prepared by Dr: Mohammed Musa Brema Idress – My best wishes Page 558

E. Dermatitis

ANSWER IS C Construction of a stoma may be complicated by several factors. Necrosis may occur because of technical errors in mesenteric division, excessive tension or failure to construct a fascial defect of adequate size to permit safe passage of the mesentery and the bowel.

1298. A 56 year old man presents with episodes of pruritus ani and bright red rectal bleeding. On examination there is a mass in the anorectal region and biopsies confirm squamous cell cancer. What is the most likely cause? A. Anal cancer B. Rectal cancer C. Soft tissue sarcoma D. Retro-rectal cyst E. Pilonidal sinus disease

ANSWER IS A These are features of anal cancer. Anal cancers arise from the cutaneous epithelium and are therefore typically squamous cell. They are usually sensitive to chemoradiotherapy.

1299. A 25 year old man complains of passing painless bright red blood rectally. It has been occurring over the past week and tends to occur post defecation. He also suffers from pruritus ani. The underlying cause is likely to be amenable by treatment from which of the following modalities? A. Topical GTN B. Topical diltiazem C. Rubber band ligation D. Injection sclerotherapy E. Lateral internal sphincterotomy

ANSWER IS C The history of one of the haemorrhoidal bleeding. The recent HUBLE trial showed equivalence of banding vs HALO for haemorrhoids. Rubber band ligation has a 30% failure rate but is generally easy and well tolerated. Painful PR bleeding is more suggestive of a fissure which is treated with nitrates or surgery.

1300. A 74 year old gentleman presents with an obstructing carcinoma of the splenic flexure. Attempts at placement of a colonic stent have failed. Which of the operative options listed below offers the best solution to this problem? A. Subtotal colectomy B. Extended right hemicolectomy C. Standard right hemicolectomy D. Standard left hemicolectomy E. Transverse colectomy

Prepared by Dr: Mohammed Musa Brema Idress – My best wishes Page 559

ANSWER IS B - Standard right hemicolectomy involves colonic division to the right of the middle colic vessels - Extended right hemicolectomy involves division of the middle colic vessels and usually resection of the splenic flexure as well. The question always causes confusion and to understand it the information needs to be carefully read. Firstly, the tumour is definitely at the splenic flexure and the second point is that the operation is definitely an extended right hemicolectomy. A left hemicolectomy or even the older operation of a transverse colectomy could be considered if the patient was not obstructed. However, when obstruction is present, an extended right hemicolectomy (which involves an ileocolic anastomosis) is relatively safe even in the obstructed setting.

1301. A 45 year old female is diagnosed as having a carcinoma of the caecum. She undergoes a CT scan which shows a tumour invading the muscularis propria with some regional lymphadenopathy. What is the most appropriate initial treatment? A. Right hemicolectomy B. External beam radiotherapy C. Chemotherapy D. Combined long course chemo radiotherapy E. Referral for palliative care

ANSWER IS A Right sided colonic cancers should proceed straight to surgery. Radiotherapy to this area is poorly tolerated and almost never offered as first line treatment. The decision as to whether or not chemotherapy is given is dependent upon the final histology.

1302. A 65 year old lady is admitted with large bowel obstruction. On investigation with CT, she is found to have a tumour of the mid rectum with no evidence of metastatic disease. What is the most appropriate course of action? A. Formation of a loop colostomy B. Laparotomy and Hartmann’s procedure C. Pan proctocolectomy and end ileostomy D. Low anterior resection and covering loop ileostomy E. Low anterior resection and end colostomy

ANSWER IS A Avoid emergency resections in large bowel obstruction due to rectal cancer This patient has presented with large bowel obstruction. However, in the case of rectal cancer, she is incompletely staged as ability to completely resect the lesion can only be determined with MRI scanning and this information is not provided. Even if the lesion were resectable, in the emergency setting, it is often safer to undertake a simple procedure such as a loop colostomy and then complete surgery at a later date. A low anterior resection and loop ileostomy in this situation would almost certainly leak (and for the reasons outlined above, may be incomplete).

Prepared by Dr: Mohammed Musa Brema Idress – My best wishes Page 560

1303. A 78 year old lady from a nursing home is admitted with a 24 hour history of absolute constipation and abdominal pain. On examination, she has a distended abdomen with a soft mass in her left iliac fossa. An x-ray is performed which shows a large dilated loop of bowel in the left iliac fossa which contains a fluid level. What is the most likely diagnosis? A. Caecal volvulus B. Sigmoid volvulus C. Incarcerated femoral hernia D. Diverticular stricture E. Malignant colonic stricture

ANSWER IS B Sigmoid volvulus may present with an asymmetrical mass in an elderly patient. It may contain a fluid level, visible on plain films. It's very rare for femoral hernia to cause large bowel obstruction.

1304. A 56 year old lady is investigated with a colonoscopy for a change in bowel habit. However, due to adhesions from a previous hysterectomy, she experiences pain and requests the procedure be terminated. The endoscopist feels that he reached the splenic flexure. What is the best course of action? A. Discharge the patient B. Arrange an abdominal CT scan C. Arrange a CT colonoscopy D. Arrange a barium enema E. Arrange a gastrograffin enema

ANSWER IS C Failed colonoscopy should be managed with a CT colonoscopy in the first instance (more accurate than barium studies). If the procedure is needed for therapeutic intervention then GA colonoscopy may be needed.

1305. A 73 year old lady presents with large bowel obstruction. On examination, she has a rectal cancer 6cm from the anal verge which has occluded the colonic lumen. An abdominal x-ray shows a caecal diameter of 7cm. Which of the management strategies outlined below is the most appropriate? A. Construction of a loop ileostomy B. Construction of a loop colostomy C. Construction of a venting caecostomy D. Abdomino-perineal resection of the colon and rectum E. Low anterior resection and loop ileostomy

ANSWER IS B This patient should be defunctioned, definitive surgery should wait until staging is completed. A loop ileostomy will not satisfactorily decompress an acutely obstructed colon. Low rectal cancers that are obstructed should not usually be primarily resected. The obstructed colon that would be used for anastomosis would carry a high risk of anastomotic dehiscence. In addition, as this is an emergency

Prepared by Dr: Mohammed Musa Brema Idress – My best wishes Page 561 presentation, staging may not be completed, an attempted resection may therefore compromise the circumferential resection margin, with an associated risk of local recurrence.

1306. A 21 year old man presents with a 5 week history of painful bright red bleeding that typically occurs post defecation and is noted on the toilet paper. External inspection of the anal canal shows a small skin tag at the six o'clock position. The patient declines internal palpation. What is the most likely underlying diagnosis? A. Fissure in ano B. Fistula in ano C. Haemorrhoidal disease D. Solitary rectal ulcer E. Internal rectal prolapse

ANSWER IS A Painful bright red rectal bleeding is usually due to a fissure The presence of pain and the sentinel tag suggests a posterior fissure in ano.

1307. A 66 year old man is admitted as an emergency with torrential rectal bleeding. Following resuscitation, an upper GI endoscopy is undertaken and it is normal. However, he continues to bleed. What is the most appropriate course of action? A. Arrange a CT angiogram B. Arrange a laparotomy and sub total colectomy C. Undertake a colonoscopy D. Perform a laparoscopy and on table colonoscopy E. Arrange a

ANSWER IS A Heavy lower GI bleeding should be investigated with an angiogram. It is best to avoid emergency surgery as the bleeding site is very difficult to find. Unlike upper GI endoscopy, colonoscopy in patients who are acutely bleeding is often difficult and seldom helpful. If it is to be done, a therapeutic scope with twin working channels is useful.

1308. A 62 year old man has previously undergone a left hemicolectomy for carcinoma of the descending colon. On follow up imaging he is found to have two deposits of metastatic disease located in the right lobe of his liver. What is the best treatment strategy? A. Chemotherapy alone B. Chemotherapy followed by surgical resection C. Radiofrequency ablation D. Chemoradiotherapy E. Palliation

Prepared by Dr: Mohammed Musa Brema Idress – My best wishes Page 562

ANSWER IS B Liver metastasis from colorectal cancer are still potentially curable. Without resection, survival at 5 years is around 5%. With resection, this figure rises to around 20%. The best outcomes are seen where chemotherapy is given, followed by resection. Radiofrequency ablation is an option for those patients who lack the physiological reserve for surgery. However, there is longer term recurrence rates will all the non resectional strategies. There is no role for radiotherapy.

1309. A 55 year old man presents with tenesmus and rectal bleeding. On examination he has a large bulky rectal cancer at 5cm with tethering to the prostate gland. Imaging shows no distant disease. What is the most appropriate initial treatment modality? A. Abdomino-perineral resection of the colon and rectum B. Pelvic exenteration C. Abdomino-perineal excision of the colon and rectum with prostatectomy D. Long course chemoradiotherapy E. Short course radiotherapy

ANSWER IS D Rectal cancers with threatened resection margins are managed with radiotherapy and chemotherapy initially. This is not the case with colonic cancers which are usually primarily resected. T4 rectal cancers are managed with long course chemoradiotherapy. A dramatic response is not uncommon. To embark on attempted resection at this stage is to court failure.

1310. A 21 year old female presents with a 24 hour history of increasingly severe ano-rectal pain. On examination, she is febrile and the skin surrounding the anus looks normal. She did not tolerate an attempted digital rectal examination. What is the most likely diagnosis? A. Fissure in ano B. Haemorrhoidal disease C. Proctalgia fugax D. Solitary rectal ulcer E. Intersphincteric abscess

ANSWER IS E The presence of fever and severe pain makes an abscess more likely than a fissure. Although fissures may be painful they do not, in themselves, cause fever. The usual management for this condition is examination of the ano-rectum under general anaesthesia and drainage of the sepsis.

1311. A 22 year old man presents with a 6 day history of passage of bloody diarrhoea with passage of mucous and slime. He is passing an average of 8 to 9 bowel movements per day. On digital rectal examination there is no discrete abnormality to feel, but there is some blood stained mucous on the glove. What is the most likely diagnosis? A. Solitary rectal ulcer syndrome

Prepared by Dr: Mohammed Musa Brema Idress – My best wishes Page 563

B. Ulcerative colitis C. Irritable bowel syndrome D. Rectal cancer E. Diverticulitis

ANSWER IS B The passage of bloody diarrhoea together with mucus and a short history makes this a likely first presentation of inflammatory bowel disease. A rectal malignancy in a 22 year old would be a very unlikely event. The history is too short to be consistent with solitary rectal ulcer.

1312. A 70 year old female is admitted with a history of passing brown coloured urine and abdominal distension. Clinically she has features of large bowel obstruction with central abdominal tenderness. She is maximally tender in the left iliac fossa. There is no evidence of haemodynamic instability. What is the most appropriate investigation? A. Cystogram B. Abdominal X-ray of the kidney, ureters and bladder C. Computerized tomogram of the abdomen and pelvis D. Flexible sigmoidoscopy E. Barium enema

ANSWER IS C This lady is most likely to have a colovesical fistula complicating diverticular disease of the sigmoid colon. In addition she may also have developed a diverticular stricture resulting in large bowel obstruction. A locally advanced tumour of the sigmoid colon may produce a similar clinical picture. The best investigation of this acute surgical patient is an abdominal CT scan, this will demonstrate the site of the disease and also supply regional information such as organ involvement and other local complications such as a pericolic abscess. A barium enema would require formal bowel preparation and this is contra indicated where large bowel obstruction is suspected. A flexible sigmoidoscopy is unlikely to be helpful and the air insufflated at the time of endoscopy may make the colonic distension worse. A cystogram would provide only very limited information.

1313. A 24 year old man is identified as having a 5cm carcinoid tumour of the appendix. Imaging and diagnostic work up does not demonstrate any distant disease. What is the best course of action? A. Appendicectomy B. Right hemicolectomy C. Resection of the caecal pole D. External beam radiotherapy E. Observation with imaging

ANSWER B Large carcinoid tumours should be formally resected. In many cases, they will be identified as an incidental finding.

Prepared by Dr: Mohammed Musa Brema Idress – My best wishes Page 564

In such cases, it can be difficult to distinguish between carcinoid tumours and other appendiceal neoplasms.

1314. Which of the following statements in relation to fistula in ano is untrue? A. High fistulae are safest treated with a seton insertion B. Low fistulae may be laid open C. They are typically probed with Lockhart Mummary probes D. When discovered during incision and drainage of perianal abscess; should always be probed to locate the internal opening E. When complicating Crohns disease, may respond to infliximab

ANSWER IS D Probing fistulae during acute sepsis is associated with a high complication rate and should not be undertaken routinely.

1315. A 78 year old lady is admitted with a 3 hour history of passage of dark red blood per rectum. Prior to this event, she was otherwise well with no major medical co-morbidities. On examination, she has a mild tachycardia but other vital signs are normal, abdomen is soft and non tender. Digital rectal exam reveals dark blood but not other findings. What is the most likely underlying cause? A. Diverticular disease B. Meckels diverticulum C. Jejunal diverticulosis D. Angiodysplasia of the colon E. Colonic cancer

ANSWER IS A Diverticular disease is the commonest cause of lower GI bleeding in adults.

1316. A 75 year old man is admitted with large bowel obstruction and on investigation is found to have a significant sigmoid diverticular stricture as the underlying cause. What is the most appropriate treatment? A. Colonoscopy and pneumatic dilatation of the stricture B. Laparotomy and Hartmann’s procedure C. Colonoscopy and insertion of self expanding metallic stent D. Loop ileostomy E. Laparotomy, sigmoid colectomy and colorectal anastomosis

ANSWER IS B Diverticular strictures have a high complication rate with stent insertion. Where patients present with large bowel obstruction, the best option is to resect the affected area. Given the fact that there is underlying colonic obstruction, a primary anastomosis would be unwise. Diverticular strictures should not be dilated.

1317. A 19 year old female presents with colicky abdominal pain, bloating and alternating constipation/diarrhoea. Her grandmother died from colon cancer at the

Prepared by Dr: Mohammed Musa Brema Idress – My best wishes Page 565 age of 87 years. A digital rectal examination and general physical examination are normal. What is the best course of action? A. Measurement of faecal calprotectin B. Arrange a barium enema C. Undertake a colonoscopy D. Undertake a proctoscopy E. Undertake a rigid sigmoidoscopy

ANSWER IS A This patient fulfills the Rome criteria for irritable bowel syndrome. Examination is normal, therefore it's likely that this patient will have IBS. However, it’s prudent to exclude IBD and since endoscopy is poorly tolerated in patients with IBS, measurement of faecal calprotectin is a reasonable alternative.

1318. A 19 year old male presents with bright red rectal bleeding that occurs post defecation onto the paper and into the pan. Apart from constipation his bowel habit is normal. Digital rectal examination is normal. What is the most likely cause? A. Haemorrhoidal disease B. Fissure in ano C. Solitary rectal ulcer D. Rectal cancer E. Crohns disease

ANSWER IS A Uncomplicated grade 1 or 2 haemorrhoids are usually impalpable This is likely to be haemorrhoidal disease. A sigmoidoscopy should always be performed to exclude more sinister pathology.

1319. What is the most likely explanation for a 63 year old male to complain of a painless blood stained mucous rectal discharge 6 months following a Hartmann's procedure? A. Pelvic abscess B. Crohns disease C. Dysplasia of the rectal stump D. Diversion proctitis E. Fissure in ano

ANSWER IS D Once the bowel has been disconnected, a degree of inflammation is commonly seen in the quiescent bowel. This is typically referred to as diversion colitis. Dysplasia is not usually seen in this context as a Hartmann’s procedure is not usually a treatment modality used for IBD (which is the main risk factor for dysplasia).

1320. A 48 year old lady has previously undergone a sigmoid colectomy for carcinoma. On follow up imaging she is found to have a 3 cm foci of metastatic disease in segment IV of the liver. What is the most appropriate course of action? A. Palliative chemotherapy

Prepared by Dr: Mohammed Musa Brema Idress – My best wishes Page 566

B. External beam radiotherapy C. Brachytherapy D. Surgical resection alone E. Chemotherapy followed by surgical resection

ANSWER IS E The treatment of colorectal liver metastasis is usually with chemotherapy followed by surgical resection. Where surgery is performed for liver metastasis with curative intent, the 5 year survival is 20%. Palliation would generally only be considered if the patient were frail or widespread disease found on imaging. Radiotherapy is not part of the treatment of liver metastasis.

1321. A 19 year old man presents with diarrhea and rectal bleeding that has been present for the past two weeks. In addition, he has noticed that he has had incontinence at night. What is the most likely cause? A. Viral gastroenteritis B. Inflammatory bowel disease C. Intersphincteric abscess D. Irritable bowel syndrome and haemorrhoids E. Irritable bowel syndrome and a fissure in ano

ANSWER IS B Nocturnal diarrhea and incontinence is a key feature in the history and is strongly suggestive of a diagnosis of IBD. More benign IBS presentations seldom have nocturnal events or a short history.

1322. A 63 year old man presents with episodic rectal bleeding the blood tends to be dark in colour and may be mixed with stool. His bowel habit has been erratic since an abdominal aortic aneurysm repair 6 weeks previously. What is the most likely cause? A. Ischaemic colitis B. Diverticulitis C. Angiodysplasia D. Cancer E. Ulcerative colitis

ANSWER IS A The inferior mesenteric artery may have been ligated and being an arteriopathy collateral flow through the marginal may be imperfect.

1323. What is the commonest type of fistula in ano? A. Trans-sphincteric B. Supra levator C. Complex supra levator D. Intersphincteric E. Suprasphincteric

Prepared by Dr: Mohammed Musa Brema Idress – My best wishes Page 567

ANSWER IS D Intersphincteric fistulas are the commonest type and the external opening may be internal or external. These are the classical type of fistula and will have an internal opening near the anal verge and obey Goodsalls rule. Primary fistulotomy in this situation usually poses little risk to continence.

1324. A 33 year old lady is admitted with recurrent discharging fistula in ano. She is also known to have ano rectal Crohns disease. On examination, she is found to have a low anal fistula with involvement of a very small amount of the external anal sphincter muscle. What is the most appropriate course of action? A. Insertion of a loose seton B. Fistulotomy C. Core fistulectomy D. Core fistulectomy and advancement flap E. Insertion of a cutting seton

ANSWER IS A Fistula in ano in patients with Crohns disease should be managed with insertion of seton. In patients with IBD, management of fistula should be minimalistic and complex procedures best avoided. Laying open fistulas in this situation is likely to result in a chronic and non healing wound.

Anal fistula Fistula in ano is the most common form of ano rectal sepsis. Fistulae will have both an internal opening and external opening, these will be connected by tract(s). Complexity arises because of the potential for multiple entry and exit sites, together with multiple tracts. Fistulae are classified into four main groups according to anatomical location and the degree of sphincter involvement. Simple uncomplicated fistulae are low and do not involve more than 30% of the external sphincter. Complex fistulae involve the sphincter, have multiple branches or are non cryptoglandular in origin.

Assessment Examination of the perineum for signs of trauma, external openings or the stigmata of IBD is important. Digital rectal examination may reveal the cord linking the internal and external openings. At the same time the integrity of the sphincter mechanism can be assessed. Low, uncomplicated fistulas may not require any further assessment, other groups will usually require more detailed investigation. For the fistula, the use of endo-anal USS with instillation of hydrogen peroxide into the fistula tract may be helpful. Ano-rectal MRI scanning is also a useful tool, it is sensitive and specific for the identification of fistula anatomy, branching tracts and identifying occult sphincter involvement.

Identification of the internal opening Fistulas with an external opening less than 3cm from the anal verge will typically obey Goodsalls rule.

Prepared by Dr: Mohammed Musa Brema Idress – My best wishes Page 568

Therapies:- A) Seton suture A seton is a piece of material that is passed through the fistula between the internal and external openings that allows the drainage of sepsis. This is important as undrained septic foci may drain along the path of least resistance, which may result in the development of accessory tracts and openings. Their main use is in treating complex fistula.

Two types of seton are recognized, simple and cutting. Simple setons lie within the fistula tract and encourage both drainage and fibrosis. A cutting seton is inserted and the skin incised. The suture is tightened and re-tightened at regular intervals. This may convert a high fistula to a low fistula. Since the tissue will scar surrounding the fistula it is hoped that this technique will minimize incontinence. Unfortunately, a large retrospective review of the literature related to the use of cutting setons has found that they are associated with a 12% long term incontinence rate.

D) Fistulotomy Low fistulas that are simple should be treated by fistulotomy once the acute sepsis has been controlled. Fistulotomy (where safe) provides the highest healing rates. Because fistulotomy is regarded as having a high cure rate, there are some who prefer to use this technique with more extensive sphincter involvement. In these patients the fistulotomy is performed as for a low fistula. However, the muscle that is encountered is then divided and reconstructed with an overlapping sphincter repair.

A price is paid in terms of incontinence with this technique and up to 12.5% of patients who were continent pre-operatively will have issues relating to continence post procedure. The same group also randomized between fistulotomy and sphincter reconstruction and ano-rectal advancement flaps for the treatment of complex cryptoglandular fistulas and reported similar outcomes in terms of recurrence (>90%) and disturbances to continence (20%).

Other authors have found adverse outcomes following fistulotomy in patients who have undergone previous surgery, are of female gender or who have high internal openings, in these patients careful assessment of pre-operative sphincter function should be considered mandatory prior to fistulotomy.

B) Anal fistula plugs and fibrin glue The desire to avoid injury to the sphincter complex has led to surgeons using both fibrin glue and plugs to try and improve fistula healing. Meticulous preparation of the tract and prior use of a draining seton is likely to improve chances of success. The use of anal fistula plugs in high transphincteric fistula of cryptoglandular origin is to be discouraged because of the high incidence of non response in patients treated with such devices. In most patients septic complications are the reasons for failure.

Fibrin glue is a popular option for the treatment of fistula. There is variability of reported healing rates In some cases initial success rates of up to 50% healing at six

Prepared by Dr: Mohammed Musa Brema Idress – My best wishes Page 569 months are reported (in patients with complex cryptogenic fistula). Of these successes 25% suffer a long term recurrence of fistula. There are, however, no obvious cases of damage to the sphincter complex and the use of the devices does not appear to adversely impact on subsequent surgical options.

C) Ano-rectal advancement flaps This procedure is primarily directed at high fistulae, and is considered attractive as a sphincter saving operation. The procedure is performed either with the patient in the prone jack knife position or in lithotomy (depending upon the site of the fistula). The dissection is commenced in the sub mucosal plane (which may be infiltrated with dilute adrenaline solution to ease dissection). The dissection is continued into healthy proximal tissue. This is brought down and sutured over the defect.

Follow up of patients with cryptoglandular fistulas treated with advancement flaps shows a success in up to 80% patients. With most recurrences occurring in the first 6 months following surgery. Continence was affected in some patients, with up to 10% describing major continence issues post operatively.

D) Ligation of the intersphincteric tract procedure In this procedure an incision is made in the intersphincteric groove and the fistula tract dissected out in this plane and divided. A greater than 90% cure rate within 4 weeks was initially reported. Others have subsequently performed similar studies on larger numbers of patients with similar success rates.

Fistulotomy at the time of abscess drainage? A Cochrane review conducted in 2010 suggests that primary fistulotomy for low, uncomplicated fistula in ano may be safe and associated with better outcomes in relation to long term chronic sepsis. However, there is a danger that such surgery performed by non specialists may result in a higher complication rate and therefore the traditional teaching is that primary treatment of acute sepsis is incision and drainage only. All agree that high/ complex fistulae should never be subject to primary fistulotomy in the acute setting.

1325. A 75 year old lady is admitted with large bowel obstruction. She is previously well. She is investigated with an abdominal CT scan and this shows an obstructing carcinoma of the ascending colon. What is the best course of action? A. Laparotomy, right hemicolectomy and ileo-colic anastomosis B. Laparotomy and ileo-colic bypass C. Laparotomy and loop ileostomy alone D. Insertion of self expanding metallic stent E. Laparotomy and sub total colectomy

ANSWER IS A Obstructing right sided cancers can be safely resected by right hemicolectomy. An abdominal CT scan will provide sufficient information to allow operative planning. Even if distant disease were present, immediate treatment is still warranted unless the patient is in the terminal phase of illness. In many cases, a primary anastomosis

Prepared by Dr: Mohammed Musa Brema Idress – My best wishes Page 570 can be undertaken. Insertion of colonic stents for right sided obstructing lesions is not generally undertaken.

1326. A 25 year old male presents with altered bowel habit. He is known to have familial polyposis coli. A colonoscopy shows widespread polyps, with high grade dysplasia in a polyp removed from the rectum. What is the best course of action? A. Undertake a pan proctocolectomy B. Undertake an abdomino perineal resection of the rectum and sigmoid colon C. Undertake a sub total colectomy D. Undertake a left hemicolectomy E. Perform sequential colonoscopic polypectomies until all polyps are resected

ANSWER IS A Since high grade dysplasia has been found in 1 polyp, the correct course of action is to remove the entire colon, rectum and anus. An ileo-anal pouch could be offered should the patient wish. None of the other procedures listed would be acceptable or safe under any circumstances.

1327. What is the most likely diagnosis in a 17 year old man who presents with painful bright red rectal bleeding that has been noticed to occur in past 2 weeks? A. Fistula in ano B. Fissure in ano C. External haemorrhoids D. Internal haemorrhoids E. Ulcerative colitis

ANSWER IS B Painful rectal bleeding is typically seen with fissure in ano (most will be posterior). The initial history is often short (as in this case). A fistula is more likely to present with discharge than just blood. Haemorrhoidal disease bleeding is usually painless. Although thrombosed haemorrhoids may be painful, they typically occur in patients with a longer history.

1328. A 53 year old man is due to undergo a right hemicolectomy for a caecal carcinoma. Which of the following would be usual practice prior to surgery? A. Oral carbohydrate loading drink 2 hours pre operatively B. Mechanical bowel preparation with oral sodium picosulphate C. Mechanical bowel preparation with oral mannitol D. Total gut cleansing with oral antibiotics 3 days pre-operatively E. Iodine rectal washout pre-operatively

ANSWER IS A Of the options presented here, only the oral carbohydrate drink would be standard practice prior to a right sided colonic resection. Whilst some surgeons may administer phosphate enemas before surgery, total gut clearance confers no benefit for right sided resections and delays recovery. In contrast, the carbohydrate loading drink is part of enhanced recovery protocols.

Prepared by Dr: Mohammed Musa Brema Idress – My best wishes Page 571

1329. You embark on a laparoscopic appendicectomy and find an appendix mass. There is no free fluid and the patient has no evidence of peritonitis. Which is the best option? A. Convert to a midline laparotomy and perform a limited right hemicolectomy and end ileostomy B. Convert to midline laparotomy and perform and appendicectomy after taking down the adhesions C. Place a drain laparoscopically and administer parenteral antibiotics D. Send the patient for CT guided drainage E. Wrap omentum around the area and avoid drainage

ANSWER IS C Attempt conservative management for appendix mass without peritonitis. Dissection of appendix masses can be associated with a considerable degree of morbidity, the gains of formally dissecting them over simple drainage and antibiotics are minimal.

This was initially described as the Ochsner-Sherren regime and was based on the teachings of Albert Ochsner of Chicago and James Sherren of the London hospital. The key facts of both methods (which essentially consisted of non surgical management and careful observation) were combined and published by Hamilton Bailey in 1930 (Bailey H. The Oschner- Sherren treatment of acute appendicitis. BMJ 1930 Jan 25; 1(3603): 140143.)

1330. A 23 year old male presents with bright red rectal bleeding that occurs post defecation onto the toilet paper. He has been suffering from severe pain associated with this. On external anal examination there is a skin tag located at the 6 O'clock position. Which of the treatments listed below is most likely to be helpful? A. Topical GTN B. Rubber band ligation C. Injections of oily phenol D. Milligan Morgan haemorrhoidectomy E. Lords anal dilatation

ANSWER IS A Since the most likely diagnosis is a fissure, the correct treatment is topical nitrates. Haemorrhoidal treatments are not going to be helpful. Whilst a Lords anal dilation was the traditional treatment, there are few surgeons (and even fewer patients!) that would advocate a significant anal stretch these days as there are significant long term continence risks.

1331. A 28 year old male presents with painful, bright red, rectal bleeding. On examination he is found to have a posteriorly sited, midline, fissure in ano. What is the most appropriate treatment? A. Topical GTN paste B. Sub lingual GTN paste

Prepared by Dr: Mohammed Musa Brema Idress – My best wishes Page 572

C. Anal stretch D. Advancement flap E. Tailored division of the external anal sphincter

ANSWER IS A Topical vasodilator therapy is the most commonly utilized treatment for fissure in ano. Surgical division of the internal anal sphincter is a reasonable treatment option in a young male. Division of the external sphincter will almost certainly result in incontinence and is not performed. Anal stretches were associated with a high rate of external sphincter injuries and have been discontinued for this reason.

1332. During a colonoscopy, a patient is found to have a colonic cancer in the caecum and a 1cm polyp (which looks adenomatous) in the sigmoid colon. What is the correct management of the sigmoid polyp? A. Undertake a snare polypectomy B. Leave in situ until the cancer has been resected C. Perform a hot biopsy D. Perform a cold biopsy E. Resect the sigmoid at the same time as the cancer resection

ANSWER IS B When a cancer has been identified during endoscopy, it is safest to avoid undertaking polyp interventions as there is a risk of seeding.

1333. A 56 year old man is admitted with passage of a large volume of blood per rectum. On examination, he is tachycardic, his abdomen is soft, although he has marked dilated veins on his abdominal wall. Proctoscopy reveals large dilated veins with stigmata of recent haemorrhage. What is the most appropriate treatment? A. IV terlipressin B. Excisional haemorrhoidectomy C. Injection sclerotherapy D. Proctectomy E. Rectal pack insertion

ANSWER IS A Rectal varices are a recognized complication of portal hypertension. In the first instance they can be managed with medical therapy to lower pressure in the portal venous system. TIPSS may be considered. Whilst band ligation is an option, attempting to inject these in same way as haemorroids would carry a high risk of precipitating further haemorrhage.

1334. A 78 year old lady is admitted with a lower GI bleed and on investigation with a CT angiogram is found to have bleeding sigmoid diverticular disease. She is otherwise well and apart from tachycardia, she is stable. What is the most appropriate course of action? A. Laparotomy and Hartmann’s procedure

Prepared by Dr: Mohammed Musa Brema Idress – My best wishes Page 573

B. Laparotomy, sigmoid colectomy and colorectal anastomosis C. Colonoscopy and application of endoscopic clips to the area D. Conservative management with close observation E. Laparotomy and sub total colectomy

ANSWER IS D Most lower GI bleeds occur secondary to diverticular disease and will settle with conservative management. Attempts at endoscopic haemostasis are usually unsuccessful.

1335. A 73 year old lady presents with constipation and no organic disease is identified on investigation. Which of the following types of laxatives works by direct bowel stimulation? A. Magnesium sulphate B. Lactulose C. Potassium sodium titrate D. Methylcellulose E. Senna

ANSWER IS E Senna contains glycosides. It passes unchanged into the colon where bacteria hydrolyze the glycosidic bond, releasing the anthracene derivatives. These stimulate the myenteric plexus.

1336. A 32 year old man is diagnosed as having a carcinoma of the caecum. On questioning, his mother developed uterine cancer at the age of 39 and his maternal uncle died from colonic cancer aged 38. His older brother developed a colonic cancer with micro satellite instability aged 37. What is the most appropriate operative treatment? A. Limited ileocaecal resection B. Right hemicolectomy C. Extended right hemicolectomy D. Panproctocolectomy E. Subtotal colectomy

ANSWER IS D The likely diagnosis is one of a familial cancer syndrome and now that he has developed a colonic cancer the safest operative strategy is a total colectomy and end ileostomy.

1337. A 53 year old man has a 1.5cm polyp identified and completely removed during a colonoscopy. Histology confirms a low grade adenoma. What is the correct follow up? A. Discharge. B. Repeat endoscopy in 5 years. C. Repeat endoscopy in 3 years. D. Segmental resection of the affected area.

Prepared by Dr: Mohammed Musa Brema Idress – My best wishes Page 574

E. Barium enema at 5 years.

ANSWER IS C It would be unsafe to discharge. Follow up with barium enemas for polyps is counter intuitive. In the UK NICE guidance (2011) this patient would only be classified as high risk if other adenomas were present, or the removal incomplete, in which case a repeat endoscopy at 1 year would be required. Otherwise the patient is at intermediate risk and repeat endoscopy at 3 years is warranted.

1338. A 62 year old man is identified as having a rectal cancer. Following diagnostic work up no metastatic disease is identified. The tumour is 2cm from the anal verge. On MRI, lesion is T2, N0. Which of the following represents the correct course of action? A. Proceed to abdomino-perineal excision of the colon and rectum (ELAPE) B. Undertake a low anterior resection and loop ileostomy C. Undertake a Hartmann’s procedure D. Offer radical external beam radiotherapy followed by abdomino-perineal excision of the colon and rectum (ELAPE) E. Offer radical radiotherapy followed by low anterior resection and loop ileostomy

ANSWER IS A The tumour is too low for restorative surgery to be considered with an acceptable functional outcome. The tumour will therefore require an ELAPE style abdomino perineal resection. Since the lesion is T2 there is no prognostic benefit from adding radiotherapy which will confer additional morbidity.

1339. A 67 year old man has had multiple episodes with fever and left iliac fossa pain. These have usually resolved with courses of intravenous antibiotics. He is admitted with a history of increasing constipation and abdominal distension. A contrast x-ray is performed which shows flow of contrast to the sigmoid colon, here the contrast flows through a long narrow segment of colon into dilated proximal bowel. What is the most likely cause? A. Diverticular stricture B. Malignant stricture C. Ischaemic stricture D. Volvulus E. Crohns stricture

ANSWER IS A The long history of left iliac fossa pain and development of bowel obstruction suggests a diverticular stricture. These may contain a malignancy and most will require resection. Whilst colonic Crohns strictures can occur, they would be quite rare in this age group, with this history as an isolated finding.

1340. A 24 year old woman presents with a long history of obstructed defecation and chronic constipation. She often strains to open her bowels for long periods and

Prepared by Dr: Mohammed Musa Brema Idress – My best wishes Page 575 occasionally notices that she has passed a small amount of blood. On examination, she has an indurated area located anteriorly approximately 3cm proximal to the anal verge. What is the most likely diagnosis? A. Haemorrhoids B. Rectal cancer C. Ulcerative colitis D. Solitary rectal ulcer syndrome E. Fissure in ano

ANSWER IS D Solitary rectal ulcers are associated with chronic constipation and straining. It will need to be biopsied to exclude malignancy (the histological appearances are characteristic). Diagnostic work up should include endoscopy and probably defecating proctogram and ano-rectal manometry studies.

1341. A 34 year old man presents with symptoms attributable to a fistula in ano. He is examined in the lithotomy position and the external opening of the fistula is identified in the 7 o'clock position. At which of the following locations is the internal opening most likely to be identified? A. 7 o'clock B. 12 o'clock C. 9 o'clock D. 3 o'clock E. 6 o'clock

ANSWER IS E Goodsalls rule: Anterior fistulae will tend to have an internal opening opposite the external opening. Posterior fistulae will tend to have a curved track that passes towards the midline. According to Goodsalls rule the track of a posteriorly sited fistula will track to the posterior midline (i.e. 6 o'clock)

1342. A 19 year old man presents with painful rectal bleeding and is found to have an anal fissure. Which of the following is least associated with this condition? A. Leukaemia B. Syphilis C. Tuberculosis D. Sickle cell disease E. Crohn's disease

ANSWER IS D Anal fissures are associated with: 1. Sexually transmitted diseases (syphilis, HIV) 2. Inflammatory bowel disease (Crohn's up to 50%) 3. Leukaemia (25% of patients) 4. Tuberculosis 5. Previous anal surgery

Prepared by Dr: Mohammed Musa Brema Idress – My best wishes Page 576

1343. A 53 year old man presents with a full thickness external rectal prolapse. Which of the following procedures would be the most suitable surgical option? A. Rectopexy B. Delormes C. Altmeirs D. Thirsch tape E. Abdomino-perineal excision of the rectum

ANSWER IS A As this man is young and has full thickness prolapse a rectopexy is the most appropriate procedure. It will give the lowest recurrence rates.

1344. A patient has an appendicectomy and a 1.2cm carcinoid tumour is identified in the tip of the appendix. What is the most appropriate management? A. Watchful waiting B. Discharge C. Right hemicolectomy D. Limited ileocaecal resection E. Radioisotope scan

ANSWER IS B Individuals with small carcinoids can be discharged (<2cm and limited to the appendix). Larger tumours should have a radioisotope scan. Where the resection margin is positive or where the isotope scan suggests lymphatic metastasis a right hemicolectomy should be performed.

1345. A 31 year old male presents with recurrent episodes of knife like pain within his rectum. On examination, there is no abnormality to find on either proctoscopy or palpation. What is the most likely diagnosis? A. Proctalgia fugax B. Fissure in ano C. Fistula in ano D. Anal cancer E. Intersphincteric abscess

ANSWER IS A Proctalgia fugax is a functional anorectal disorder characterized by severe, intermittent episodes of rectal pain that are self-limited. The diagnosis of proctalgia fugax requires exclusion of other causes of rectal or anal pain. Diagnostic work up should also include imaging (for example with MRI) to exclude occult pathology.

1346. A 73 year old lady is admitted with a brisk rectal bleed. She is otherwise well and the bleed settles. On examination, her abdomen is soft and non tender. Elective colonoscopy shows a small erythematous lesion in the right colon, but no other abnormality. What is the likely cause? A. Diverticular bleed B. Angiodysplasia

Prepared by Dr: Mohammed Musa Brema Idress – My best wishes Page 577

C. Colonic cancer D. Ischaemia E. Infective colitis

ANSWER IS B Angiodysplasia can be difficult to identify and treat. The colonoscopic stigmata are easily missed by poor bowel preparation.

1347. A 55 year old man develops an acute colonic pseudo-obstruction following a laminectomy. Despite correction of his electrolytes and ongoing supportive care he fails to settle. Which of the drugs listed below may improve the situation? A. Buscopan B. Neostigmine C. Metoclopramide D. Mebevrine E. Sodium picosulphate

ANSWER IS B Neostigmine affects the degradation of acetylcholine and will therefore stimulate both nicotinic and muscarinic receptors. It may produce symptomatic bradycardia and should therefore only be administered in a monitored environment. In colonic pseudo-obstruction it produces generalized colonic contractions and its onset is usually rapid.

1348. An 83 year old man is admitted on the acute surgical take. His presenting symptom is of painless, profuse rectal bleeding of dark blood. His medical history comprises a previous TIA for which he takes clopidogrel and a statin. What is the most likely underlying cause? A. Meckels diverticulum B. Colonic cancer C. Diverticular bleed D. Ischaemic colitis E. Diverticulitis

ANSWER IS C The majority of patients with colonic bleeding will be found to have bleeding secondary to diverticular disease. Of note, inflammation (i.e. diverticulitis) is not seen in such cases. Around 70% will stop bleeding spontaneously. Anti platelet and anti coagulants are sometimes complicating factors and may make bleeding less likely to cease spontaneously. Ischaemic colitis often has more dominant colitis symptoms.

1349. A 55 year old man is found to have a carcinoma of the sigmoid colon on screening colonoscopy. How should this be staged? A. MRI of the abdomen and CT of the chest B. Liver MRI and Chest CT C. CT scanning of the chest, abdomen and pelvis alone

Prepared by Dr: Mohammed Musa Brema Idress – My best wishes Page 578

D. MRI of the rectum and CT of the abdomen and chest E. Endoluminal USS and CT scanning of the abdomen

ANSWER IS C Colonic cancers are staged with CT scanning of the chest, abdomen and pelvis. Rectal cancer is staged with MRI rectum (and sometimes endolumenal USS for low T1 lesions) together with CT scanning of the chest, abdomen and pelvis. Historically, colonic cancer was staged with liver USS and CXR. However, modern imaging has made this practice obsolete.

1350. Which of the agents listed below is most likely to help a 22 year old lady with severe perianal Crohns disease and multiple anal fistulae. The acute sepsis has been drained and setons are in place. She is already receiving standard non biological therapy. A. Trastuzumab B. Bevacizumab C. Imatinib D. Cetuximab E. Infliximab

ANSWER IS E Infliximab is a popular choice in managing complex perianal Crohns. It is absolutely vital that all sepsis is drained prior to starting therapy.

1351. A 78 year old man has undergone a hemi-arthroplasty for a intracapsular hip fracture. Post operatively he develops electrolyte derangement and receives intravenous fluids. Over the previous 24 hours he develops marked abdominal distension. On examination, he has a tense, tympanic abdomen which is not painful. A contrast enema shows flow of contrast through to the caecum and through the ileocaecal valve. What is the most likely cause? A. Ogilvie’s syndrome B. Diverticular stricture C. Malignant stricture D. Volvulus E. Adhesive obstruction

ANSWER IS A Patients with electrolyte disturbance and previous surgery may develop colonic pseudo-obstruction (Ogilvie’s syndrome). The diagnosis is made using a contrast enema and treatment is usually directed at the underlying cause with colonic decompression if indicated.

1352. A 68 year old man with ulcerative colitis is admitted with an exacerbation. You are called to see him because he is having brisk dark PR bleeding. He has been on intravenous hydrocortisone for 5 days. The gastroenterologists have done an OGD to exclude a duodenal ulcer, this was normal. What is the best course of action?

Prepared by Dr: Mohammed Musa Brema Idress – My best wishes Page 579

A. Subtotal colectomy and end ileostomy B. Pan proctocolectomy and end ileostomy C. Colonoscopy D. CT angiogram E. Flexible sigmoidoscopy

ANSWER IS A This man requires surgery to remove the bleeding segment of bowel. Medical management has failed here. Note that a pan proctocolectomy is not a suitable option in the emergency setting because there is increased morbidity from the pelvic dissection. In the unlikely event that a subtotal colectomy did not address the bleeding then consideration may have to be given to removal of the rectum but this would not usually be the case. Note that in this case, there is not really any benefit to be derived from imaging, endoscopy would be very dangerous and risk perforation as the bowel would be very friable.

1353. A 21 year old lady presents with a 6 month history of an offensive discharge from the anus. She is otherwise well, but is increasingly annoyed at the need to wear pads. On examination, she has a small epithelial defect in the 5 o'clock position, approximately 3 cm from the anal verge. What is the most likely cause? A. Fissure in ano B. Fistula in ano C. External haemorrhoid D. Squamous cell carcinoma of the anus E. Proctalgia fugax

ANSWER IS B Fistulas usually occur following previous ano-rectal sepsis. The discharge may be foul smelling and troublesome. Patients should be listed for examination under anaesthesia. Fistulas which are low and have little or no sphincter involvement are usually laid open.

1354. A 77 year old man is admitted with large bowel obstruction and on investigation with an abdominal CT scan is found to have an obstructing cancer of the sigmoid colon. What is the most appropriate course of action? A. Laparotomy, sigmoid colectomy and formation of end colostomy B. Laparotomy and loop colostomy C. Laparotomy and loop ileostomy D. Laparotomy, high anterior resection and colo-rectal anastomosis E. Palliation

ANSWER IS A Obstructing sigmoid cancers can be resected or stented. If stented, then the patient may need definitive surgery later. If resected, then a resection and end colostomy (Hartmann's ) procedure is usually undertaken because of the risks of anastomotic leak in the setting of anastomosing obstructed colon to rectum.

Prepared by Dr: Mohammed Musa Brema Idress – My best wishes Page 580

1355. A 56 year old man presents with his first attack of diverticulitis. Which of these complications is least likely to ensue? A. Formation of colonic strictures B. Malignant transformation C. Development of colovesical fistula D. Formation of a pericolic abscess E. Formation of a phlegmon

ANSWER IS B Diverticulitis may result in a number of complications. However, whilst malignant disease may coincide with diverticulitis it is not, in itself, a risk factor for colonic cancer.

1356. A 60 year old lady is investigated for abdominal pain. A polyp is identified at the proximal descending colon, three small polyps are also noted in the sigmoid colon. The largest lesion is removed by snare polypectomy and the pathology report states that this polyp is a low grade dysplastic adenoma measuring 3cm in diameter. The remaining lesions are ablated using diathermy. What is the correct management? A. Repeat endoscopy in 1 year B. No further endoscopic surveillance C. Repeat endoscopy in 5 years D. Segmental colonic resection E. Repeat endoscopy in 3 years ANSWER IS A She is at high risk of malignancy and should be closely followed up. Fulguration of polyps without histology is unhelpful.

1357. A 34 year old lady presents to her general practitioner with perianal discomfort. The general practitioner diagnoses pruritus ani, which of the following is least associated with the condition? A. Hyperbilirubinaemia B. Anal fissure C. Leukaemia D. Syphilis E. Tuberculosis

ANSWER IS E Causes: 1. Systemic (DM, Hyperbilirubinaemia, aplastic anaemia) 2. Mechanical (diarrhoea, constipation, anal fissure) 3. Infections (STDs) 4. Dermatological 5. Drugs (quinidine, colchicine) 6. Topical agents

Prepared by Dr: Mohammed Musa Brema Idress – My best wishes Page 581

1358. A previously well 21 year old man is admitted with 2 week history of diarrhoea and passage of blood and mucous rectally. He has previously undergone an ileocaecal resection in the past for an inflammatory bowel disorder and takes mesalazine. What is the most likely underlying cause? A. Ulcerative colitis B. Diversion proctitis C. Crohns proctitis D. Diverticular associated bleed E. Irritable bowel syndrome

ANSWER IS C The history of a right sided resection is the patients young age are all strongly suggestive of an existing diagnosis of Crohns disease (segmental resections are not undertaken for UC). Since the bowel has remained in continuity, a diversion colitis is not possible.

1359. A 28 year old male presents with a discharging sinus in his natal cleft. He is found to have a pilonidal sinus. Which statement is false? A. Can occur in webs of fingers and the axilla B. After drainage pilonidal abscesses should not be primarily closed C. A rare complication is squamous cell carcinoma D. In a patient with an acute abscess the Bascoms procedure is the treatment of choice. E. Treatment involves excising or laying open the sinus tract.

ANSWER IS D When performing incision and drainage for pilonidal abscess try to avoid making the incision in the midline of the natal cleft. Acute pilonidal abscesses should receive simple incision and drainage. Definitive treatments such as a Bascoms procedure should not be undertaken when acute sepsis is present.

1360. A 23 year old lady presents with a posteriorly sited fissure in ano. Treatment with stool softeners and topical GTN has failed to improve matters. Which of the following would be the most appropriate next management step? A. Lords procedure B. Injection of botulinum toxin C. Lateral internal sphincterotomy D. Endoanal advancement flap E. Surgical division of the external anal sphincter

ANSWER IS B The next most appropriate management option when GTN or other topical nitrates has failed is to consider botulinum toxin injection. In males a lateral internal sphincterotomy would be an acceptable alternative. In a female who has yet to conceive this may predispose to delayed increased risk of sphincter dysfunction.

Prepared by Dr: Mohammed Musa Brema Idress – My best wishes Page 582

Division of the external sphincter will result in faecal incontinence and is not a justified treatment for fissure.

1361. A 17-year-old male is being investigated for tremor and dysarthria. His GP has performed blood tests which show very deranged LFT’s. On examination, he is noted to have a tremor, impaired memory and slit-lamp examination reveals Kayser-Fleischer rings. What is the most likely diagnosis? Select one answer only. A. Alcoholic cirrhosis B. Alpha-1 antitrypsin deficiency C. Autoimmune hepatitis D. Hepatitis B E. Wilson’s disease

ANSWER IS E Wilson’s disease is an autosomal recessive disorder that results in the toxic accumulation of copper in the liver and CNS (in particular the basal ganglia). Children often present with liver disease, and adults with CNS features. Kayser-Fleischer rings are copper deposits in the iris (Descemet’s membrane) that are pathognomic but not invariable and may require a slit-lamp to be seen.

1362. A 22-year-old female presents to her GP with a history of vague abdominal pain and bloating. On further questioning she reports weight loss and passing offensive stools which are difficult to flush away. As part of her investigations a jejunal biopsy is performed. This shows crypt hyperplasia and subtotal villous atrophy. What is the most likely diagnosis? Select one answer only. A. Bacterial overgrowth B. Coeliac disease C. Crohns disease D. Ulcerative colitis E. Tropical malabsorption.

ANSWER IS B Coeliac disease is a T-cell mediated autoimmune disease affecting the small bowel caused by gluten intolerance. It causes a variety of symptoms including steatorrhoea (due to an inability to absorb fats as evidenced in this case by offensive stools that are difficult to flush away), abdominal pain, bloating. It is diagnosed by a jejunal biopsy which classically shows subtotal villous atrophy and crypt hyperplasia. The management involves a gluten-free diet which can relieve symptoms.

1363. A 27-year-old male who works in the city presents with crampy abdominal pain and a 6-week history of bloody diarrhoea with mucus. He is referred for investigation where a sigmoidoscopy reveals an inflamed mucosa and a rectal biopsy shows an inflammatory infiltrate with goblet cell depletion and crypt abscesses. What is the most likely diagnosis? Select one answer only. A. Coeliac disease B. Crohn’s disease C. Irritable bowel syndrome

Prepared by Dr: Mohammed Musa Brema Idress – My best wishes Page 583

D. Ulcerative colitis E. Whipple’s disease.

ANSWER IS D Ulcerative colitis commonly presents between the ages of 15 and 30 years. It is characterized by diarrhoea often with mucus and blood. There may be a crampy abdominal pain, and the frequency of motions may be related to the severity of an attack. It may affect only the rectum (proctitis in approximately 50%), or spread proximally to involve the rest of the colon. Sigmoidoscopy shows an inflamed and friable mucosa, and histologically there may be an inflammatory infiltrate, goblet cell depletion, glandular distortion, mucosal ulcers and crypt abscesses.

1364. A 17-year-old A-level student presents with diarrhoea, abdominal pain and weight loss. On examination, she is tender in the right iliac fossa (RIF) and on further questioning she had a perianal abscess drained 1 year ago. A Barium enema shows cobble stoning in the RIF. What is the most likely diagnosis? Select one answer only. A. Coeliac disease B. Crohn’s disease C. Irritable bowel syndrome D. Tropical sprue E. Ulcerative colitis.

ANSWER IS B Crohn’s disease is a form of inflammatory bowel disease that has several features in common with Ulcerative colitis. However, it is characterized by transmural granulomatous inflammation and can affect any part of the gut. It is most commonly found in the terminal ileum (and therefore may produce discomfort in the RIF). Other features seen include diarrhoea and weight loss. Perianal disease is also common and may manifest as abscesses and fistulae. Barium enemas will classically show cobble stoning and ‘rose-thorn’ ulcers. MRI can also be used to assess the extent of pelvic disease.

1365. A 35-year-old male with known Ulcerative colitis (UC) reports a generalized itching, fatigue and abdominal pain. On examination, he is found to be jaundiced and his blood tests show a markedly raised bilirubin and alkaline phosphatase. An MRCP shows multiple strictures in the biliary tree. What is the most likely diagnosis? Select one answer only. A. Cholangitis B. Cholecystitis C. Primary biliary cirrhosis D. Primary sclerosing cholangitis E. Wilson’s disease

ANSWER IS D Primary sclerosing cholangitis (PSC) is a condition in which inflammation, fibrosis and strictures of the intra- and extra-hepatic bile ducts occur. MRCP shows multiple

Prepared by Dr: Mohammed Musa Brema Idress – My best wishes Page 584 strictures in the biliary tree and a characteristic ‘beaded’ appearance. Around 80% of patients with PSC will have a diagnosis of UC.

1366. A 62-year-old male presents to his GP as his family have noticed he has ‘gone yellow.’ On further questioning he admits to generalized pruritus and around 2 stone of weight loss. He denies any abdominal or chest pain. On examination you note he has icteric sclera and widespread scratch marks. What is the most likely diagnosis? Select one answer only. A. Biliary colic B. Cholangitis C. Cholecystitis D. Gallstone ileus E. Pancreatic Cancer

ANSWER IS E This patient has several features of jaundice- icteric sclera and generalized pruritus, other questions which would be useful in the history would be regarding any colour change in his urine or stools.

Obstructive jaundice classically produces dark urine and paler stools. The absence of pain goes against the first four options and the significant recent weight loss suggests a malignant process may be present. In any patient with a history of painless, obstructive jaundice, the diagnosis of Pancreatic Ca must be considered.

1367. A 56-year-old man presents with episodes of severe epigastric pain radiating through to his back for several weeks. He also reports he has noticed his stools tend to float more recently and are very difficult to flush away. On examination erythema abigne is noted over the epigastrium. What is the most likely diagnosis? Select one answer only. A. Acute pancreatitis B. Biliary colic C. Coeliac disease D. Chronic pancreatitis E. Pancreatic Cancer.

ANSWER IS D There are a number of features of this case to suggest it is chronic pancreatitis rather than acute- its prolonged history and the patient has symptoms suggestive of steatorrhoea due to an inability to absorb fats due to reduced pancreatic enzyme production. Erythema abigne is caused by using a hot water bottle to relieve pain and is often seen in the epigastrium or on the back.

1368. A 57-year-old businessman from Taiwan presents to his GP with symptoms of malaise, weight loss and right upper quadrant pain. On examination he is noted to have hepatomegaly which is irregular. A bruit is also heard over the liver. His blood tests show a very raised alpha-fetoprotein. What is the most likely diagnosis? Select one answer only.

Prepared by Dr: Mohammed Musa Brema Idress – My best wishes Page 585

A. Cholangiocarcinoma B. Fibroarcoma of the liver C. Gallstones D. Hepatocellular carcinoma (HCC) E. Liver haemangioma.

ANSWER IS D Hepatocellular carcinoma (HCC) is the commonest malignant tumour of hepatocytes accounting for 90% of primary liver cancers. It is more common in China and Taiwan and is caused by chronic Hepatitis B or C infection, or cirrhosis. On examination they may be palpable as an irregular hepatomegaly and sometimes a bruit may be heard over them. The alpa-fetoprotein levels are raised in 80% of patients.

1369. An 18-year-old male presents to A and E after vomiting blood following a night out for his birthday. On further questioning he admits to consuming a large amount of alcohol and then vomiting heavily. After vomiting around 12 times he noticed streaks of blood in his vomit. What is the most likely diagnosis? Select one answer only. A. Aortic-enteric fistula B. Gastric Cancer C. Mallory-Weiss tear D. Oesophageal varices E. Peptic ulcer disease

ANSWER IS C A Mallory-Weiss tear is a tear in the mucosal lining of the oesophagus which classically occurs after heavily vomiting (often following an alcohol binge). Most cases of bleeding from this are self-limiting and usually minor.

1370. An 18-year-old A-level student is referred from his GP with RIF pain and nausea. On examination, she has guarding and tenderness in the RIF. On deep palpation in the LIF she complains of pain in the RIF. What is this sign called? Select one answer only. A. Aaron’s sign B. Dunphy’s sign C. Murphy’s sign D. Rigler’s sign E. Rovsing’s sign

ANSWER IS E Rovsing’s sign describes pain in the RIF more than the LIF when the LIF is pressed, and is pathognomic of appendicitis. Aaron’s sign is pain referred to the epigstrium upon continuous firm pressure over McBurney’s point and is indicative of appendicitis. Dunphy’s sign is a medical sign in which increased abdominal pain occurs with coughing and is often present in appendicitis. Murphy’s sign occurs

Prepared by Dr: Mohammed Musa Brema Idress – My best wishes Page 586 when pressing over the RUQ with two fingers and asking the patient to breathe in, causes pain and arrest of inspiration as the inflamed gall bladder impinges on the examiner’s fingers. It is only positive if repeating the test on the left does not result in a similar pain. Rigler’s sign is a radiologic sign in which air is seen on an abdominal X-ray on both sides of the intestine.

1371. A previously well 83-year-old woman is brought into A and E with abdominal pain and vomiting. On examination, she has a distended abdomen, and denies any previous surgery. On assessment of her groins, a tender lump is palpable in the left groin. It is below and lateral to the pubic tubercle. What is the most likely cause of her symptoms? Select one answer only. A. Epigastric hernia B. Femoral hernia C. Direct inguinal hernia D. Indirect inguinal hernia E. Obturator hernia.

ANSWER IS B Femoral hernias classically emerge below and lateral to the pubic tubercle, and are more common in women due to the wider pelvis.

Inguinal hernias emerge from above and medial to the pubic tubercle and come in two forms - direct (which pass through Hasselbach’s triangle) and indirect (which enter via the deep inguinal ring). As the neck for femoral hernias is relatively narrow and stiff, they are more likely to obstruct and strangulate.

1372. An 82-year-old male is recovering in ICU 4 days after an emergency repair of a difficult juxta-renal leaking abdominal aortic aneurysm. The nurse bleeps you because he has become tachycardic and his blood pressure has dropped to 80/50. On assessment he is very pale and the nurse reports he has just had a large PR bleed. What is the most likely cause for this? Select one answer only. A. Angiodysplasia B. Aortic-enteric fistula C. Diverticular disease D. Gastric Cancer E. Peptic ulcer disease.

ANSWER IS B Aortic-enteric fistulae occur mostly following AAA repair in which a communication has developed between the aorta and the small bowel. They result in very heavy blood loss into the small bowel and are often fatal.

1373. A 50-year-old man presents to the emergency department with dizziness and melaena. Select the most likely single investigation from the list of options below for this case. A. Upper gastrointestinal endoscopy B. Colonoscopy

Prepared by Dr: Mohammed Musa Brema Idress – My best wishes Page 587

C. Selective angiography D. Double contrast barium enema E. Barium meal and follow through.

ANSWER IS A Peptic ulceration is the commonest cause for such a presentation and therefore upper gastrointestinal endoscopy is the investigation of choice in this patient. The procedure may also allow treatment e.g. adrenaline injection.

1374. A 65-year-old woman presents with dark red rectal bleeding, hypotension (blood pressure 95/60 mmHg), and tachycardia (pulse 100 beats/min). Full blood count reveals anaemia (haemoglobin 8.6 g/dl). Upper GI endoscopy shows no abnormality. Colonoscopy reveals dark red blood in the colon and no other abnormality. Select the most likely single investigation from the list of options below for this case. A. Upper gastrointestinal endoscopy B. Colonoscopy C. Selective angiography D. Double contrast barium enema E. Barium meal and follow through.

ANSWER IS C Selective angiography is a superior investigation in this instance. Although of interest, the likely diagnosis in this case does not significantly affect the choice of investigation.

Mesenteric angiography is the logical next step in the management of a patient in whom colonoscopy has failed to detect the source of bleeding. This is of particular importance in an elderly patient who has evidence of significant ongoing bleeding, but who is cardiovascularly stable.

Angiography relies on active bleeding for diagnosis, following which therapeutic embolization of the offending vessel may be performed. Should the investigation fail to demonstrate the cause, and the patient continues to bleed, then laparotomy and colectomy may be necessary as a life-saving procedure. If the bleeding is clearly of an upper GI source (i.e. dark, altered blood or even melaena), upper GI endoscopy would be the initial investigation of choice.

1375. An anxious 20-year-old man, who has a strong family history of colorectal cancer, presents with a positive faecal occult blood test. Select the most likely single investigation from the list of options below for this case. A. Upper gastrointestinal endoscopy B. Colonoscopy C. Selective angiography D. Double contrast barium enema E. Barium meal and follow through.

Prepared by Dr: Mohammed Musa Brema Idress – My best wishes Page 588

ANSWER IS B Colonoscopy is a more accurate screening test than double contrast barium enema. However, the latter is cheaper and associated with lower morbidity and mortality rates. In view of the family history and positive FOB test, colorectal polyps should be excluded. Polyps smaller than 1cm in size can be easily missed by barium enema.

1376. An 80-year-old lady with atrial fibrillation has a 4-day history of abdominal pain and some rectal bleeding. Select the most likely diagnosis from the list. A. Mesenteric ischaemia B. Appendix mass C. Caecal carcinoma D. Intussusception E. Meckel’s diverticulum

ANSWER IS A The finding of atrial fibrillation in a patient with sudden-onset abdominal pain must raise the possibility of an embolic event. Embolism accounts for 25–30% of patients with mesenteric ischaemia. It is notoriously difficult to diagnose and the passage of blood per rectum is relatively rare.

1377. A 16-year-old boy presents with a 6-month history of diarrhoea, vomiting and vague right iliac fossa mass. He has a microcytic anaemia. Select the most likely diagnosis from the list. A. Mesenteric ischaemia B. Appendix mass C. Crohn’s disease D. Intussusception E. Meckel’s diverticulum

ANSWER IS C A longer history of diarrhoea and vomiting over several months, together with a vague right iliac fossa mass and established microcytic anaemia, is more suggestive of Crohn’s disease because of iron loss from haemorrhage. A macrocytic anaemia may also be seen as a result of vitamin B12 malabsorption due to terminal ileitis.

1378. An 18-year-old girl presents with a 4-day history of pain in the right iliac fossa, a temperature of 38 °C and diarrhoea. Select the most likely diagnosis from the list. A. Mesenteric ischaemia B. Appendix mass C. Crohn’s disease D. Intussusception E. Meckel’s diverticulum

ANSWER IS B An appendix mass forms when an inflamed appendix perforates locally and pus is prevented from spreading throughout the peritoneal cavity by adherent omentum

Prepared by Dr: Mohammed Musa Brema Idress – My best wishes Page 589 and small bowel. The patient has a history suggesting appendicitis, and occasionally a tender mass may be felt in the right iliac fossa.

1379. Dual sacs straddling the inferior epigastric vessels - select the most appropriate hernia from the list. A. Littre’s hernia B. Maydl’s hernia C. Pantaloon hernia D. Richter’s hernia E. Sliding hernia.

ANSWER IS C

1380. Two separate loops of bowel - select the most appropriate hernia from the list. A. Littre’s hernia B. Maydl’s hernia C. Pantaloon hernia D. Richter’s hernia E. Sliding hernia.

ANSWER IS B

1381. The posterior wall of the hernial sac is formed by a herniating viscus - select the most appropriate hernia from the list. A. Littre’s hernia B. Maydl’s hernia C. Pantaloon hernia D. Richter’s hernia E. Sliding hernia.

ANSWER IS E

1382. Portion of circumference of the bowel - select the most appropriate hernia from the list. A. Littre’s hernia B. Maydl’s hernia C. Pantaloon hernia D. Richter’s hernia E. Sliding hernia.

ANSWER IS D

1383. Meckel’s diverticulum - select the most appropriate hernia from the list. A. Littre’s hernia B. Maydl’s hernia C. Pantaloon hernia

Prepared by Dr: Mohammed Musa Brema Idress – My best wishes Page 590

D. Richter’s hernia E. Sliding hernia.

ANSWER IS A In a sliding hernia the posterior wall of the hernial sac is formed by a herniating viscus. This is usually sigmoid colon or bladder. During repair, particular care is required not to injure the herniated viscus, which can be seen when examining inside the opened sac.

1384. A 45-year-old drug addict was admitted to the general medical ward with septic shock. His right hip was flexed at 30° and there was reduced sensation at the medial part of his thigh. In addition, the patient experienced weakness in flexing his thigh at the hip and weakness in knee extension. In view of this clinical scenario, what nerve is most likely involved? A. Femoral nerve B. Saphenous nerve C. Common fibular nerve D. Obturator nerve E. Sciatic nerve.

ANSWER IS A The primary focus of this patient’s sepsis is likely to be an abscess of the psoas major. The femoral nerve may be damaged in the abdomen as a result. The femoral emerges from the lateral border of the psoas major in the iliac fossa and passes into the anterior thigh posterior to the inguinal ligament and lateral to the femoral artery. The femoral nerve innervates muscles in the anterior compartment of the thigh and the skin of the anterior and medial thigh.

1385. A 23-year-old man was tackled while playing rugby earlier on that day and the force was directed at the outer and lower part of his left knee. He noticed pain and paraesthesia in the dorsum of his left foot and found it difficult to dorsiflex his left ankle. Which nerve has been injured? A. Femoral nerve B. Saphenous nerve C. Common fibular nerve D. Obturator nerve E. Sciatic nerve.

ANSWER IS C The common fibular nerve is frequently injured as it passes around the neck of the fibula. Classically, such patients experience foot drop, which results from a loss of dorsiflexion at the ankle and a loss of eversion. Patients have pain and paraesthesia in the lateral leg and dorsum of the foot.

1386. A 31-year-old woman underwent primary varicose vein surgery of her left lower limb 2 weeks ago. She came back to the follow up clinic complaining of pain

Prepared by Dr: Mohammed Musa Brema Idress – My best wishes Page 591 and paraesthesia in the skin of the medial aspect of her left leg and foot. Which nerve has been injured during the procedure? A. Femoral nerve B. Saphenous nerve C. Common fibular nerve D. Obturator nerve E. Sciatic nerve.

ANSWER IS B The saphenous nerve enters the adductor canal but leaves the canal without passing through the adductor hiatus. The saphenous nerve courses with the great saphenous vein and innervates skin of the medial side of the leg and foot. The nerve may be injured during surgical removal of the great saphenous vein, or it may be lacerated as it pierces the wall of the adductor canal.

1387. A nursing student was given a lecture regarding intramuscular injection. She wonders why injection has to be given at the upper outer quadrant of the gluteus maximus. What nerve is likely to be injured if not done so? A. Femoral nerve B. Saphenous nerve C. Common fibular nerve D. Obturator nerve E. Sciatic nerve

ANSWER IS E The sciatic nerve is susceptible to damage from an intramuscular injection in the lower medial quadrant of the gluteus maximus muscle. Such injury may affect both the common fibular and tibial nerves.

1388. A 79-year-old lady with a poor cardiac history and mild renal impairment is admitted with haematemesis and melaena. A duodenal ulcer is found at endoscopy and is found to be actively bleeding. The ulcer was injected with adrenaline; however 12 hours later she was noted to have rebled with further passage of fresh melaena. She remains haemodynamically stable. Select the most appropriate option from the list below for the treatment of this patient. A. Total gastrectomy B. Partial gastrectomy C. Oesophago-gastrectomy D. Excision of ulcer E. Repeat endoscopy

ANSWER IS E New NICE guidelines (June 2012) suggest that in the case of post-endoscopic rebleeding, patients should be offered repeat endoscopy. If this fails to achieve haemostasis then surgery should be considered. If the re-bleeding patient is hameodynamically unstable then interventional radiological procedures should be

Prepared by Dr: Mohammed Musa Brema Idress – My best wishes Page 592 considered (if available). In this case, the patient has significant co morbidities and so the minimum safe procedure (in this case under running) should be performed.

1389. A fit 59-year-old man with a known benign gastric ulcer on the greater curvature of the stomach presents with melaena. He has completed an 8-week course of omeprazole. He has required a transfusion of 6 units of blood and repeat endoscopy shows signs of an active bleed. Select the most appropriate option from the list below for the treatment of this patient. A. Total gastrectomy B. Partial gastrectomy C. Oesophago-gastrectomy D. Excision of ulcer E. Repeat endoscopy

ANSWER IS B This patient’s ulcer is known to be benign. Despite best medical therapy his ulcer has not healed and so it should be excised. The available evidence is not conclusive, but suggests that partial gastrectomy may have lower morbidity and mortality rates than ulcer excision.

1390. An 85-year-old man is found at laparotomy to have a perforated gastric ulcer high on the lesser curve. His past medical history includes COAD (chronic obstructive airways disease) and angina. Select the most appropriate option from the list below for the treatment of this patient. A. Total gastrectomy B. Partial gastrectomy C. Oesophago-gastrectomy D. Excision of ulcer E. Repeat endoscopy

ANSWER IS D This patient’s age and significant co-morbidities suggest conservative surgery (shorter anaesthetic, procedure time, lower burden of surgical trauma) is indicated. Also the perforation is high on the lesser curve, excision of the ulcer could be performed safely without injuring the gastric vasculature.

1391. A 13-year-old boy presents to The Emergency Department with a severe headache and vomiting; for the past 2 to 3 days he has been off school for presumed influenza. On examination he is flushed and has a temperature of 39°C. There are no specific findings; however, on further questioning his parents state that he had an emergency splenectomy following a road traffic accident 1 year ago. Please select the most appropriate diagnosis from the list. A. Overwhelming post-splenectomy sepsis B. Pancreatic fistula C. Portal vein thrombosis D. Splenosis E. Subphrenic abscess

Prepared by Dr: Mohammed Musa Brema Idress – My best wishes Page 593

ANSWER IS A After splenectomy the ability of the spleen to destroy encapsulated organisms is lost and infection can lead to the distinct clinical syndrome of overwhelming post- splenectomy infection (OPSI). In children splenectomised for trauma the incidence is about 1–2%, while for adults it is about 0.5%. Long-term penicillins are of proven value in children, as the maximal incidence of OPSI occurs within 2 years of surgery. Prophylaxis with polyvalent vaccines is also recommended (usually to pneumococcus, meningococcus and Haemophilus influenzae).

1392. A 42-year-old woman complains of left-sided chest and abdominal pain, worse on inspiration, 7 days post splenectomy. Examination reveals a temperature of 38.6°C, decreased air entry at the left lung base, dullness to percussion and left upper quadrant tenderness. . Please select the most appropriate diagnosis from the list. A. Overwhelming post-splenectomy sepsis B. Pancreatic fistula C. Portal vein thrombosis D. Splenosis E. Subphrenic abscess

ANSWER IS E Post-operative haematomas are common post-splenectomy. They may collect in the subphrenic space, and infection can readily lead to a subphrenic abscess. They may cause an associated pleural effusion as in this case. Characteristically they present with swinging pyrexia, sweats, rigors and left-sided pain on deep inspiration. Treatment is usually by percutaneous drainage. Some operations have a good list of specific complications that are beloved of examiners. Favoured are probably thyroidectomy, splenectomy, laparoscopic cholecystectomy and inguinal hernia repair.

1393. A 46-year-old man attends colorectal clinic presenting with a 4 month history of a change in bowel habit with blood mixed with stool. He has no relevant past medical history. What is the most appropriate investigation? Single best answer - select one answer only. A. Barium enema B. Capsule endoscopy C. Colonoscopy D. CT scan E. Oesphagoduodenoscopy.

ANSWER IS C This 46-year-old male requires a lower GI endoscopy for his red flag symptoms of change in bowel habit associated with PR bleeding. This test is the best investigation as it allows inspection of the bowel mucosa, biopsy if required and is potentially even therapeutic, for example, if a polyp is seen this can be removed by snare and sent for histology. It is an invasive test, requiring bowel preparation prior to the

Prepared by Dr: Mohammed Musa Brema Idress – My best wishes Page 594 procedure but he is otherwise well and there are no contradindications in undertaking colonoscopy.

A barium enema, though it could be useful, is not the best test for this patient. It may give an impression of a lesion in the bowel wall but does not give such accurate information as colonoscopy in this case. Furthermore he would still require an endoscopic biopsy if a lesion was seen. CT scans maybe useful for detecting gross bowel pathology and staging for cancers but would not be as useful as colonoscopy here as commented above.

Oesphagoduodenoscopy (OGD) is useful if an upper GI source is suspected but this patient has lower GI symptoms. Capsule endoscopy is an expensive test and not readily available. It is useful for looking for a source of GI bleeding that occurs in the small bowel beyond the scope of the OGD and proximal to the caecum/Terminal ileum which can be accessed by colonoscopy.

1394. A 27-year-old female presents with severe pain during defecation. She also reports a small amount of blood on the toilet paper following defecation, and her only past medical history is constipation which has become worse because of the pain. What is the most likely diagnosis? Select one answer only. A. Anal fissure B. Anal fistula C. Haemorrhoids D. Perianal abscess E. Pilonidal sinus

ANSWER IS A The history in the case is quite classical for an anal fissure where pain is the dominant symptom. In haemorrhoids the patient often does not complain of any pain (unless strangulation of a prolapsed haemorrhoid has occurred).

1395. A 17-year-old male presents with right sided abdominal pain, which is worse on movement, and anorexia. He is listed for an open appendicectomy which reveals a normal appendix. Further exploration intra-operatively reveals a normal caecum, and a terminal ileum with no obvious inflammation. However, an out pouching is noted on the terminal ileum around 60cm from the ileocaecal valve. What is the most likely diagnosis? Select one answer only. A. Abdominal TB B. Appendicitis C. Caecal carcinoma D. Crohn’s disease E. Meckel’s diverticulum

ANSWER IS E The normal appendix goes against a diagnosis of acute appendicitis. The normal caecum makes a caecal tumour causes the symptoms very unlikely and the lack of inflammation in the terminal ileum makes Crohn’s unlikely. The description of an out

Prepared by Dr: Mohammed Musa Brema Idress – My best wishes Page 595 pouching in this location is consistent with a Meckel’s diverticulum. These are embryological remnants of the vitello-intestinal duct and are a free diverticulum of the terminal ileum.

They occur in 2% of the population, are commonly 2 feet (60cm) from the ileocaecal valve, often 2 inches (5cm) in length and twice as common in males. Acute inflammation of them may mimic appendicitis and it is important when performing an appendicectomy to search for a Meckel’s diverticulum if the appendix is normal.

1396. An 18-year-old female presents with intermittent abdominal pain and discomfort which is cramping in nature and a feeling of bloating. The discomfort is often relieved through opening her bowels, and she also reports intermittent episodes of diarrhoea, she denies any weight loss, rectal bleeding or mucous. A colonoscopy is unremarkable. What is the most likely cause of her symptoms? Select one answer only. A. Coeliac disease B. Crohn’s disease C. Irritable bowel syndrome D. Hiatus hernia E. Ulcerative colitis.

ANSWER IS C Irritable bowel syndrome (IBS) is one of the commonest causes of abdominal pain and is a functional disorder causing a mixture of pain, discomfort, bloating and altered bowel habit. It does not cause PR bleeding or mucus, and these symptoms are suggestive of organic pathology and should be investigated appropriately and not attributed to IBS. Investigations for IBS will show a normal rectum and colon, and treatment focuses on symptomatic relief through dietary measures, antispasmodics and psychological approaches.

1397. A 78-year-old nursing home resident is admitted with severe diarrhoea and left iliac fossa pain. At flexible sigmoidoscopy diffuse pseudomembranes are seen. Biopsy is neutrophil rich. What is the most likely diagnosis? Single best answer - select one answer only. A. Crohn’s Disease B. Melanosis Coli C. Malignancy D. Pseudomembranous Colitis E. Ulcerative Colitis.

ANSWER IS D The appearance of pseudomembranes are typical for Pseudomembranous Colitis caused by the Clostridum Difficle. This pathogen is more common in nursing home or hospitalized patients.

1398. An 84-year-old male with multiple co morbidities presents with severe abdominal pain looking very unwell. Blood gas reveals a severe metabolic acidosis

Prepared by Dr: Mohammed Musa Brema Idress – My best wishes Page 596 and a CT scan confirms a diverticular perforation, but the extent of abdominal contamination is unclear. A diagnostic laparoscopy is performed to assess if the patient can avoid a laparotomy at present. This shows a purulent peritonitis, but no faecal contamination. How would this be classified? Select one answer only. A. Hinchey I B. Hinchey II C. Hinchey III D. Hinchey IV E. Hinchey V.

ANSWER IS C The Hinchey classification is used for colonic perforation secondary to diverticular disease. It is divided into four descriptions. Hinchey I describes localized (paracolic) abscesses, Hinchey II describes pelvic abscesses, Hinchey III describes the presence of pus in the abdominal cavity (purulent peritonitis) as described in this case. Hinchey IV describes faeculent peritonitis. It is sometimes possible for cases with purulent peritonitis to be managed laparoscopically and through inserting a drain. It may then be possible to perform the appropriate resection later as an elective procedure and stoma formation may not be necessary.

1399. A 32-year-old woman with a history of chronic diarrhoea undergoes a flexible sigmoidoscopy. There is brownish discolouration on the wall of the sigmoid in a moire pattern. Biopsy reveals characteristic pigment-laden macrophages within the mucosa on PAS staining. She works as a swimwear model and is very concerned with her appearance. What is the most likely diagnosis? Single best answer - select one answer only. A. Crohn’s Disease B. Campylobacter infection C. Lymphoma D. Melanosis Coli E. Ulcerative Colitis.

ANSWER IS D These features, both on endoscopy and histology, are typical of melanosis coli and typical of overuse of laxatives. The appearance does not have any negative sequalae. She should be advised to stop abusing laxatives. Note her occupation as a model suggesting she may feel pressurized to maintain her weight. Offer psychological support if required.

1400. An 88-year-old female who has a past medical history of dementia and is bed bound in a nursing home, presents with a history of constipation for the last 7 days. Her careers have noted her abdomen has become grossly distended and is tender to touch. An abdominal X-ray demonstrates the coffee bean sign. What would be an appropriate option to manage this patient initially? Select one answer only. A. Antibiotics B. CT scan

Prepared by Dr: Mohammed Musa Brema Idress – My best wishes Page 597

C. Exploratory laparotomy D. Insertion of flatus tube E. Sigmoid colectomy.

ANSWER IS D The coffee bean sign and clinical history is highly suggestive of sigmoid volvulus which commonly occurs in elderly patients who have a chronic history of constipation. Given the patient’s pre-morbid status of being a bed bound nursing home patient, the risks of any major surgery e.g. laparotomy or sigmoid colectomy would be great. A CT scan would confirm the diagnosis, but offers no therapeutic benefit to the patient. Insertion of a flatus tube represents a relatively low risk procedure that could potentially resolve the patient’s symptoms and therefore would be a good first line option.

1401. An 89-year-old woman is seen on the post-take ward round following CT scanning. She presented with tenderness in the left iliac fossa for 7 days. Bloods show raised inflammatory markers with a WCC of 18 and CRP 160. She has localized tenderness but is surprisingly well and feels better than on admission yesterday. Past medical history reveals CVA 5 years ago which has left her bed bound and COPD requiring multiple admissions. The CT scan shows diverticulitis with a localized perforation. What is the best initial management? Single best answer - select one answer only. A. Discharge B. Intravenous antibiotics and close monitoring C. Laparotomy and primary anastomosis D. Laparoscopic resection E. Laxatives.

ANSWER IS B This elderly patient has localized diverticular perforation with absence of peritonitis and is showing signs of improvement. Together with the fact that she has significant past medical history that would make her a high risk candidate for surgery, therefore conservative management with IV antibiotics and close monitoring for signs of deterioration is the best management path. If surgical intervention were required, laparoscopy may be difficult due to her COPD.

A primary anastomosis would not be advisable in an acute case with potential contamination with a higher chance of anastomotic leak. An anastomotic leak could be disastrous in this patient and so a colostomy would be best surgical practice. Laxatives are not the primary concern in this patient's acute management, and discharge is obviously not appropriate at this point.

1402. An 82-year-old male presents with intermittent abdominal pain and vomiting. On further questioning it is noted he has not opened his bowels for 5 days and not passed wind for 2 days. On examination he looks uncomfortable and his abdomen is grossly distended. His only past medical history of note is an open

Prepared by Dr: Mohammed Musa Brema Idress – My best wishes Page 598 repair of an abdominal aortic aneurysm 4 months ago from which he recovered well. What is the most likely cause of his symptoms? Select one answer only. A. Adhesional small bowel obstruction B. Aorto-enteric fistula C. Incisional hernia D. Diverticular disease E. Sigmoid tumour.

ANSWER IS A This patient has a number of characteristic features of small bowel obstruction - vomiting, constipation and abdominal distension. Adhesions are the most common cause of small bowel obstruction in the developed world and can occur following any form of abdominal surgery. An aorto-enteric fistula would classically cause PR bleeding, there is nothing in the history of the case to suggest the patient has an incisional hernia which could be causing obstruction, and there are no features that point to diverticular disease or a sigmoid tumour causing the symptoms.

1403. A frail 87-year-old female is brought in by her concerned family as she has started vomiting and not opened her bowels or passed wind for 3 days. On examination she looks poorly with a distended abdomen, but no previous incisions, and on further questioning she reports the pain radiates along the right upper medial thigh. Vaginal examination reveals a swelling in the right side wall of the vagina. What is the most likely diagnosis? Select one answer only. A. Adhesional small bowel obstruction B. Epigastric hernia C. Para-umbical hernia D. Obturator hernia E. Sigmoid volvulus.

ANSWER IS D Although the most common cause of small bowel obstruction is adhesions in the developed world, the lack of previous surgery makes this very unlikely. The pain radiates to the thigh is suggestive of pressure on the obturator nerve which supplies sensation to the medial thigh. This can be present in around 50% of patients with an obturator hernia. Obturator hernias occur most commonly in frail old women and often there are no signs. If pressure is present over the obturator nerve patients often hold the leg flexed to reduce pain. Rectal and more often vaginal examination can reveal a swelling in the region of the obturator foramen.

1404. An 85-year-old male is recovering 2 days after a hemiarthroplasty for a fractured neck of femur. The nurses call you as he has started vomiting profusely and his abdomen is distended. He has not opened his bowels for 3 days and on examination he has a distended, tender abdomen. He has never undergone any abdominal surgery and has no palpable herniae. His rectum is empty. His blood tests show a potassium of 2.6, creatinine of 144. What is the most likely diagnosis? Select one answer only. A. Adhesional small bowel obstruction

Prepared by Dr: Mohammed Musa Brema Idress – My best wishes Page 599

B. Constipation secondary to opioids C. Incisional hernia D. Obstruction hernia E. Pseudo-obstruction

ANSWER IS E The history of vomiting, constipation and a distended abdomen are suggestive of an obstruction. However, the most common cause for small bowel obstruction namely adhesions and herniae are unlikely in the case given the patient has never had abdominal surgery and has no palpable herniae respectively. His empty rectum points away from constipation. In this case this gentleman has developed pseudoobstruction which often resembles bowel obstruction and can occur in any post-op patients and can also be triggered by deranged electrolytes e.g. hypokalaemia. It is best managed conservatively through correcting the underlying causes.

1405. A 35-year-old male presents after being referred from the GP with a painful swelling which is discharging around his anus. He has had a similar problem twice in the past and on examination a very tender, erythematous swelling is noted in the natal cleft, but separate from the anal verge. What is the most likely diagnosis? Select one answer only. A. Ischiorectal abscess B. Hidradenitis suppurativa C. Perianal abscess D. Pilonidal abscess E. Supralevator abscess.

ANSWER IS D The commonest site for pilonidal abscesses is the natal cleft and there may be a history of recurrent episodes over months or years. Pilonidal means ‘hair’s nest’ in Latin and theories regarding its aetiology involve shed hairs getting trapped in pits and becoming a nidus for local infection. Clinically there may be openings in the midline or either side of it sometimes with tufts of hair. There may be inflammation of the adjacent skin and pus may be expressed via the openings. Management involves incision and drainage of the abscess and then other procedures can be done electively to try and prevent recurrence.

1406. A 56-year-old man undergoes an anterior resection. The resection specimen shows invasion beyond the muscularis propria but no nodal involvement. What is his 5 year survival according to Dukes’ stage? Single best answer - select one answer only. A. 6% B. 47% C. 77% D. 93% E. 100%.

Prepared by Dr: Mohammed Musa Brema Idress – My best wishes Page 600

ANSWER IS C This patient has Dukes’ stage B colon cancer defined by invasion into the muscularis propria but not beyond to involve lymph nodes on the histology specimen. This equates to a 5 year survival of 77%. Dukes' staging (A-D) is still in common usage as an adjunct to TNM staging in determining the management of colorectal cancer. Stage Description 5 year survival % A Confined to bowel wall 93.2% B Through bowel wall - Lymph node not involved 77% C Lymph nodes involved, no other metastases 47.7% D Distant metastases 6.6%

1407. A 65-year-old male undergoes an emergency laparotomy for large bowel obstruction. This reveals an obstructing sigmoid tumour. This is resected and a Hartmann’s’ procedure is completed. Histology for the specimen reveals the tumour has extended through all the muscle layers, but no lymph node involvement is noted. What Dukes’ stage would this tumour be classified as? Select one answer only. A. Stage A B. Stage B C. Stage C D. Stage D E. Stage E.

ANSWER IS B The Dukes’ system is one of the most well-known staging methods for colonic carcinoma. Stage A- Neoplastic cells are restricted to the mucosa and the 5 year survival is 90%. Stage B- The tumour has extended through the muscle layers and possibly reached the serosa. Lymph node involvement is not present though and the 5 year survival is 75%. Stage C- This occurs where lymph node involvement is present and can be subdivided into C1: (local lymph node involvement only) and C2: (more proximal lymph node involvement). The overall survival is around 50%. Stage D- This wasn’t part of the original description, but is often used to describe widespread malignant disease. The 5 year survival is 6%. There is no such stage as E.

1408. A 65-year-old woman presents with a change in bowel habit. She has had fresh bloody diarrhoea for the last 2 months. Flexible sigmoidoscopy revealed inflamed and ulcerated mucosa from the rectum to as high as the scope could extend in the splenic flexure. Biopsy shows superficial ulceration and diffuse mucosal inflammation. What is the likely diagnosis? Single best answer - select one answer only. A. Crohn’s Disease B. Ischaemic Colitis C. Lymphoma

Prepared by Dr: Mohammed Musa Brema Idress – My best wishes Page 601

D. Salmonella Infection E. Ulcerative Colitis

ANSWER IS E This patient has ulcerative colitis as evident in history, endoscopy findings with confluent inflammation, and histology results. This inflammatory disorder differs from Crohn’s Disease as granulomas are absent, fissures are rare and inflammation is superficial involving the mucosal layer.

1409. A 20-year-old lady presents with a 3-week history of bright red rectal bleeding associated with pain on defaecation. Her symptoms started post-partum. Select the most likely answer from the list. A. Anal carcinoma B. Anal fissure C. Colonic polyp D. Crohn’s disease E. Haemorrhoids

ANSWER IS B Pain on defaecation can be due to an anal fissure, anal carcinoma or strangulated haemorrhoids. Haemorrhoids per se are not painful. Anal fissures are common in young adults and have an increased incidence following pregnancy.

1410. A 32-year-old man presents with a 1-week history of colicky lower abdominal pain. This is associated with bloody diarrhoea, increased stool frequency and weight loss. A mass is palpable in the right iliac fossa. He is anaemic and has a CRP of 200. Select the most likely answer from the list. A. Anal carcinoma B. Anal fissure C. Colonic polyp D. Crohn’s disease E. Haemorrhoids

ANSWER IS D This history is typical of inflammatory bowel disease. The most likely diagnosis here would be Crohn’s disease in view of the weight loss and a palpable mass. These latter features are uncommon in ulcerative colitis.

1411. A 37-year-old homosexual presents with a 3-month history of episodes of bright red rectal bleeding associated with pain and itching. On examination he has an area of ulceration at the anal verge with an everted irregular edge. Select the most likely answer from the list. A. Anal carcinoma B. Anal fissure C. Colonic polyp D. Crohn’s disease E. Haemorrhoids

Prepared by Dr: Mohammed Musa Brema Idress – My best wishes Page 602

ANSWER IS A Anal carcinoma has a strong association with human papilloma virus (types 16, 18, 33) infection. The everted edge is characteristic of a neoplastic process and a carcinoma should be suspected. Bleeding is another common presentation of anal carcinoma.

1412. A 53-year-old woman presents with a 2-year history of increasing difficulty passing stool. She currently opens her bowels daily or on alternate days. However, she has to strain excessively and often has to press on her perineum to achieve evacuation. She also reports a ‘bulge’ in the vagina when she gets constipated. She has attended clinic for the results of her recent investigations. Blood investigations and colonoscopy were normal. Select the most likely diagnosis. A. Outlet obstruction B. Colorectal carcinoma C. Constipation-predominant irritable bowel syndrome D. Functional faecal retention E. Idiopathic megabowel

ANSWER IS A This patient has ‘functional constipation’, because investigations have excluded an organic cause. Constipation may refer to the infrequent and/or difficult passage of stools. A predominance of symptoms of difficult evacuation, which is often referred to as obstructed defaecation (e.g. excessive straining, a sensation of incomplete evacuation, digitation etc) is suggestive of outlet obstruction, rather than slow transit constipation, although physiological confirmation is required as symptoms do not accurately predict underlying pathophysiology. The history of perineal massage and a ‘bulge’ in the vagina (posterior wall) is suggestive of the presence of a rectocoele, which may lead to outlet obstruction, as a result of redistribution of evacuatory forces during defaecation.

1413. A 74-year-old man presents to the surgical clinic with a 2-month history of constipation. Previously, he opened his bowels daily, passing stool of ‘normal’ consistency, but his bowels have become irregular, and he has experienced episodes of ‘diarrhoea’ during the last few weeks. On direct questioning, he reported episodic fresh bleeding per rectum, which he attributed to his ‘piles’. Select the most likely diagnosis. A. Outlet obstruction B. Colorectal carcinoma C. Constipation-predominant irritable bowel syndrome D. Functional faecal retention E. Idiopathic megabowel

ANSWER IS B This diagnosis must always be excluded in patients presenting with altered bowel habit. It is now clear that most patients with colorectal cancer who present with altered bowel habit report loose stools or diarrhoea. This represents ‘overflow’ of proximal bowel content because of narrowing of the lumen in the affected segment

Prepared by Dr: Mohammed Musa Brema Idress – My best wishes Page 603 of bowel. Careful history taking often reveals a period of constipation preceding the change in stool consistency. It is unwise to attribute potentially sinister symptoms (e.g. bleeding per rectum) to ‘benign’ pathology (e.g. haemorrhoids) until proximal pathology has been excluded.

1414. A 22-year-old man with a history of constipation since early childhood attends The Emergency Department having not opened his bowels for the previous 3 weeks. He was admitted with similar symptoms several months ago, when a rectal biopsy was performed. This demonstrated normal ganglion cells in the myenteric plexus, and no other abnormality. On examination he appears well. Abdominal examination reveals a large mass arising in the pelvis and extending to the umbilicus. Select the most likely diagnosis. A. Outlet obstruction B. Colorectal carcinoma C. Constipation-predominant irritable bowel syndrome D. Functional faecal retention E. Idiopathic megabowel

ANSWER IS E Persistent dilatation of the bowel is known as megabowel. This may occur secondary to an absence of ganglion cells in the myenteric plexus (Hirschsprung disease), where failure of relaxation of the affected segment leads to gross proximal dilatation. Alternatively, no obvious cause may be identifiable, when it is termed idiopathic megabowel. This condition is characterized by severe infrequency of defaecation, with several weeks between bowel movements. There is usually associated passive leakage of stool as a result of ‘overflow’ around impacted stool in the rectum. The diagnosis is confirmed on barium enema, which reveals dilatation of the rectum, and sometimes colon.

Management involves behavioural, medical and, rarely, surgical treatment. Constipation is the second most common gastrointestinal symptom in the developed world. In most patients, low fluid intake, low dietary fiber, and lack of exercise or mobility may contribute to ‘simple’ constipation. However, constipation may be caused by ‘organic’ pathology when it occurs secondary to structural or systemic abnormalities. Organic causes may affect the gastrointestinal tract itself, e.g. mechanical obstruction secondary to carcinoma/stricture, and persistent dilatation of the bowel (megabowel) occurring without obvious cause (idiopathic) or secondary to Hirschsprung disease.

Extragastrointestinal pathology may also cause constipation. Examples include endocrine/metabolic, neurological and psychological disorders. Constipation may also occurrs secondary to certain medication (opiates, antidepressants, anticholinergics, anticonvulsants). In the absence of an organic cause for constipation, the term ‘functional’ constipation is adopted to indicate disordered function of the hindgut. On the basis of physiological investigations, such patients may be divided into those with a delay in transit through all or part of the colon (slow transit constipation), and/or those with abnormalities of rectal evacuation

Prepared by Dr: Mohammed Musa Brema Idress – My best wishes Page 604

(outlet obstruction) or those with no abnormality (constipation-predominant irritable bowel syndrome). The causes in the list provided may be classified using this system to provide a comprehensive differential diagnosis for constipation.

1415. Family history of colonic cancer: brother (aged 35 years), sister (aged 32 years) and father (aged 60 years). Select the most likely answer from the list. A. Barium enema B. Colonoscopy C. Evacuation proctogram enema D. Flexible sigmoidoscopy E. Single contrast gastrografin.

ANSWER IS B Screening of colonic cancer in patients with a positive family history should be performed with colonoscopy as the whole colon must be visualized. The use of computed tomography (CT) colonography and magnetic resonance (MR) colonography for screening and primary detection of colorectal cancers is the subject of ongoing research.

1416. For A 25-year-old man with bright red rectal bleeding select the most likely answer from the above list. A. Barium enema B. Colonoscopy C. Evacuation proctogram enema D. Flexible sigmoidoscopy E. Single contrast gastrografin.

ANSWER IS E Bright red rectal bleeding in a young patient with no change in bowel habit can be suitably investigated with flexible sigmoidoscopy, as it is most likely that the source is located in the left colon. - But I think D is more acquired than E

1417. Which of the agents listed below is a phosphodiesterase inhibitor? A. Milrinone B. Metaraminol C. Dopamine D. Dobutamine E. Adrenaline

ANSWER IS A Milrinone:- Phosphodiesterase inhibitor - Elevation of cAMP levels improves muscular contractility, short half life and acts as vasodilator

1418. A 28 year old lady with known von Willebrands disease is bleeding following the excision of a sebaceous cyst. Administration of which of the following agents is most likely to be beneficial?

Prepared by Dr: Mohammed Musa Brema Idress – My best wishes Page 605

A. Desmopressin B. Factor IX concentrate C. Factor VII concentrate D. Factor X concentrate E. Vasopressin

ANSWER IS A Desmopressin is useful in managing the bleeding from the commonest type of vWD. Vasopressin is less likely to be disease specific and at most would have a vasoconstrictor approach which is likely to be of brief duration.

1419. A 44 year old man undergoes a distal gastrectomy for cancer. He is slightly anaemic and therefore receives a transfusion of 4 units of packed red cells to cover both the existing anaemia and associated perioperative blood loss. He is noted to develop ECG changes that are not consistent with ischaemia. What is the most likely cause? A. Hyponatraemia B. Hyperkalaemia C. Hypercalcaemia D. Metabolic alkalosis E. Hypernatraemia

ANSWER IS B The transfusion of packed red cells has been shown to increase serum potassium levels. The risk is higher with large volume transfusions and with old blood.

1420. Which of the following muscle relaxants will tend to incite neuromuscular excitability following administration? A. Atracurium B. Suxamethonium C. Vecuronium D. Pancuronium E. None of the above

ANSWER IS B Suxamethonium may induce generalized muscular contractions following administration. This may raise serum potassium levels.

1421. A 23 year old man is recovering from an appendicectomy. The operation was complicated by the presence of perforation. He is now recovering on the ward. However, his urine output is falling and he has been vomiting. Which of the following intravenous fluids should be initially administered, pending analysis of his urea and electrolyte levels? A. Hartmans solution B. Dextran 70 C. Pentastarch D. Gelofusin

Prepared by Dr: Mohammed Musa Brema Idress – My best wishes Page 606

E. 5% Dextrose

ANSWER IS A He will have sequestration of electrolyte rich fluids in the abdomen and gut lumen. These are best replaced by use of Hartmans solution in the first instance.

1422. A 73 year old man develops disseminated intravascular coagulation following an abdominal aortic aneurysm repair. He receives an infusion of cryoprecipitate. What is the major constituent of this infusion? A. Factor VIII B. Factor IX C. Protein C D. Protein S E. Factor V

ANSWER IS A Cryoprecipitate: - Blood product made from plasma - Usually transfused as 6 unit pool - Indications include massive haemorrhage and uncontrolled bleeding due to haemophilia

1423. A 32 year old man presents to the acute surgical unit with acute pancreatitis. Over the next few days he becomes dyspnoeic and his saturations are 89% on air. A CXR shows bilateral pulmonary infiltrates. His CVP pressure is 16mmHg. What is the most likely diagnosis? A. Cardiac failure B. Pneumococcal pneumonia C. Staphylococcal pneumonia D. Pneumocystis carinii E. Adult respiratory distress syndrome

ANSWER IS E Acute pancreatitis is known to precipitate ARDS. ARDS is characterized by bilateral pulmonary infiltrates and hypoxaemia. Note that pulmonary oedema is excluded by the CVP reading < 18mmHg.

1424. Which of the anaesthetic agents below is most likely to induce adrenal suppression? A. Sodium thiopentone B. Midazolam C. Propofol D. Etomidate E. Ketamine

ANSWER IS D

Prepared by Dr: Mohammed Musa Brema Idress – My best wishes Page 607

Etomidate is a recognized cause of adrenal suppression, this has been associated with increased mortality when used as a sedation agent in the critically ill.

1425. A 63 year old man is commenced on an infusion of packed red cells following a total hip replacement. Which of the following adverse events is most likely? A. ABO mismatching B. Immune mediated intolerance of rhesus incompatible blood C. Pyrexia D. Jaundice E. Graft versus host disease

ANSWER IS C Mild pyrexia during blood transfusion is the most common event and commonly occurs during transfusion.

1426. A 32 year male with leukaemia attends the day unit for a blood transfusion. Five days after the transfusion he attends the Emergency Department with a temperature of 38.5, erythroderma and desquamation. What is the most likely explanation? A. Graft versus host disease B. Acute haemolytic transfusion reaction C. Delayed haemolytic transfusion reaction D. Transfusion associated lung injury E. Neutrophilic febrile reaction

ANSWER IS A This is associated with transfusion of unirradiated blood in immunosuppressed patients. Transfusion associated GVHD can occur 4-30 days after a transfusion and follows a sub acute pathway. Patients may also have diarrhoea and abnormal liver function tests. Management involves steroid therapy.

1427. A patient with tachycardia and hypotension is to receive vasopressors. Which of the following conditions are most likely to be treated with vasopressors? A. Hypovolaemic shock B. Septic shock C. Neurogenic shock D. Cardiogenic shock E. None of the above

ANSWER IS B The term septic shock has a precise meaning and refers to refractory systemic arterial hypotension in spite of fluid resuscitation. Patients will therefore usually require inotropes. Individuals suffering from neurogenic shock will usually receive intravenous fluids to achieve a mean arterial pressure of 90mmHg. If this target cannot be achieved then these patients will receive inotropes. Hypovolaemic shock

Prepared by Dr: Mohammed Musa Brema Idress – My best wishes Page 608 requires fluids and the management of cardiogenic shock is multifactorial and includes inotropes, vasodilators and intra-aortic balloon pumps.

1428. Infusion with which of the following blood products is most likely to result in an urticarial reaction? A. Packed red cells B. Fresh frozen plasma C. Platelets D. Cryoprecipitate E. Factor VIII concentrate

ANSWER IS B - Pyrexia is the most common adverse event in transfusing packed red cells - Urticaria is the most common adverse event following infusion of FFP

1429. A 54-year-old man is admitted for an elective hip replacement. Three days post operatively you suspect he has had a pulmonary embolism. He has no past medical history of note. Blood pressure is 120/80 mmHg with a pulse of 90/min. The chest x-ray is normal. Following treatment with low-molecular weight heparin, what is the most appropriate initial lung imaging investigation to perform? A. Pulmonary angiography B. Echocardiogram C. MRI thorax D. Ventilation-perfusion scan E. Computed tomographic pulmonary angiography

ANSWER IS E CTPA is the first line investigation for PE according to current BTS guidelines This is a difficult question to answer as both computed tomographic pulmonary angiography (CTPA) and ventilation-perfusion scanning are commonly used in UK clinical practice. The 2003 British Thoracic Society (BTS) guidelines, however, recommended that CTPA is now used as the initial lung imaging modality of choice. Pulmonary angiography is of course the 'gold standard' but this is not what the question asks for

1430. Which of the inotropes listed below can be administered via the peripheral route in the non cardiac arrest setting? A. Milrinone B. Noradrenaline C. Adrenaline D. Metaraminol E. Dobutamine

ANSWER IS D As a general rule, inotropes can only be administered via a central vein. Metaraminol is an exception to this as it can be administered via a peripheral line.

Prepared by Dr: Mohammed Musa Brema Idress – My best wishes Page 609

1431. Which of the following agents is most likely to be helpful in controlling refractory hypotension in a 23 year old female with severe pyelonephritis? A. Noradrenaline B. Adrenaline C. Dobutamine D. Dopamine E. Milrinone

ANSWER IS A Since the main issue here is vasodilation, a vasoconstrictor is most likely to be helpful.

1432. A 17 year old man undergoes an elective right hemicolectomy. Post operatively he receives a total of 6 liters of 0.9% sodium chloride solution, over 24 hours. Which of the following complications may ensue? A. Hyperchloraemic acidosis B. Hypochloraemic alkalosis C. Hyperchloraemic alkalosis D. Acute renal failure E. None of the above

ANSWER IS A Excessive infusions of any intravenous fluid carry the risk of development of tissue oedema and potentially cardiac failure. Excessive administration of sodium chloride is a recognized cause of hyperchloraemic acidosis and therefore Hartmans solution may be preferred where large volumes of fluid are to be administered.

1433. A 28 year old man with Crohn's disease has undergone a number of resections. His BMI is currently 18 and his albumin is 18. He feels well but does have a small localized perforation of his small bowel. The gastroenterologists are giving azathioprine. What is the most appropriate advice regarding feeding? A. Nil by mouth B. Nil by mouth and continuous intra venous fluids until surgery C. Enteral feeding D. Parenteral feeding E. Nutritional supplements

ANSWER IS D This man is malnourished, although he is likely to require surgery it is best for him to be nutritionally optimized first. As he may have reduced surface area for absorption and has a localized perforation TPN is likely to be the best feeding modality.

1434. A 51 year old man is shot in the abdomen and sustains a significant intra abdominal injury. A laparotomy, bowel resection and end colostomy are performed. An associated vascular injury necessitates a 6 unit blood transfusion. He has a prolonged recovery and is paralyzed and ventilated for 2 weeks on intensive care. He receives total parenteral nutrition and is eventually weaned

Prepared by Dr: Mohammed Musa Brema Idress – My best wishes Page 610 from the ventilator and transferred to the ward. On reviewing his routine blood tests the following results are noted: Full blood count Hb 11.3 g/dl Platelets 267 x 109/l WBC 10.1 x109/l Urea and electrolytes Na+ 131 mmol/l K+ 4.6 mmol/l Urea 2.3 mmol/l Creatinine 78 µmol/l Liver function tests Bilirubin 25 µmol/l ALP 445 u/l ALT 89 u/l γGT 103 u/l What is the most likely underlying cause for the abnormalities noted? A. Delayed type blood transfusion reaction B. Bile leak C. Anastomotic leak D. Total parenteral nutrition E. Gallstones

ANSWER IS D TPN is known to result in derangement of liver function tests. Although, cholestasis may result from TPN, it would be very unusual for gallstones to form and result in the picture above. Blood transfusion reactions typically present earlier and with changes in the haemoglobin and although they may cause hepatitis this is rare nowadays.

1435. Which statement regarding post operative cognitive impairment is true? A. Use of Benzodiazepines preoperatively reduces long-term post operative cognitive dysfunction B. Pain does not cause delirium C. Delirium has no impact on length of hospital stay D. A regional anaesthetic rather than a general anaesthetic is more likely to contribute to post operative cognitive impairment E. Visual hallucinations are not a feature of delirium

ANSWER IS A Anaesthetic technique and Post operative cognitive impairment (POCD): - Use of benzodiazepines preoperatively reduces long-term POCD (9.9% vs. 5%) - Do not stop drugs for cognitive function - Regional techniques reduce POCD in first week, but no difference at 3 months

Prepared by Dr: Mohammed Musa Brema Idress – My best wishes Page 611

1436. Which statement is false about pethidine? A. It is thirty times more lipid soluble than morphine B. Structurally similar to morphine C. Pethidine has a toxic metabolite (norpethidine) which is cleared by the kidney D. Pethidine is metabolized by the liver E. Can be given intramuscularly

ANSWER IS B It has a different structure. It is much more lipid soluble than morphine. It produces less biliary tract spasm than morphine.

1437. Which of the following anaesthetic agents is least likely to be associated with depression of myocardial contractility? A. Propofol B. Etomidate C. Sodium thiopentone D. Ether E. None of the above

ANSWER IS B Of the agents mentioned, etomidate has the most favorable cardiac safety profile.

1438. Which of the following is a not a diagnostic criteria for brain death? A. No response to sound B. No corneal reflex C. Absent oculo-vestibular reflexes D. No response to supraorbital pressure E. No cough reflex to bronchial stimulation

ANSWER IS A Brain death - Criteria for brain stem death testing 1. Deep coma of known aetiology. 2. Reversible causes excluded 3. No sedation 4. Normal electrolytes

Testing for brain death 1. Fixed pupils which do not respond to sharp changes in the intensity of incident light 2. No corneal reflex 3. Absent oculo-vestibular reflexes - no eye movements following the slow injection of at least 50ml of ice-cold water into each ear in turn (the caloric test) 4. No response to supraorbital pressure 5. No cough reflex to bronchial stimulation or gagging response to pharyngeal stimulation

Prepared by Dr: Mohammed Musa Brema Idress – My best wishes Page 612

6. No observed respiratory effort in response to disconnection of the ventilator for long enough (typically 5 minutes) to ensure elevation of the arterial partial pressure of carbon dioxide to at least 6.0 kPa (6.5 kPa in patients with chronic carbon dioxide retention). Adequate oxygenation is ensured by pre- oxygenation and diffusion oxygenation during the disconnection (so the brain stem respiratory centre is not challenged by the ultimate, anoxic, drive stimulus)

The test should be undertaken by two appropriately experienced doctors on two separate occasions. Both should be experienced in performing brain stem death testing and have at least 5 years post graduate experience. One of them must be a consultant. Neither can be a member of the transplant team (if organ donation contemplated).

1439. You are the cardiothoracic surgical registrar reviewing a patient referred for an aortic valve replacement. The 40-year-old man is being investigated for progressive breathlessness in a previous respiratory clinic. The notes show he has smoked for the past 25 years. Pulmonary function tests reveal the following: FEV1 1.4 L FVC 1.7 L FEV1/FVC 82% What is the most likely explanation? A. Asthma B. Bronchiectasis C. Kyphoscoliosis D. Chronic obstructive pulmonary disease E. Laryngeal malignancy

ANSWER IS C These results show a restrictive picture, which may result from a number of conditions including kyphoscoliosis. The other answers cause an obstructive picture.

We note that most people have chosen COPD as the answer. In COPD the FEV1/FVC would show an obstructive picture with the FEV1/FVC value being low (approximately less than 70%). In restrictive conditions the FEV1/FVC is normal or increased (greater than 70%). With the FEV1/FVC being over 70% the most likely answer is kyphoscoliosis.

1440. A 78 year old man presents with a ruptured aortic aneurysm. This is repaired but the operation is difficult as it has a juxtarenal location. A supra renal cross clamp is applied. Post operatively he is found to be oliguric and acute renal failure is suspected. Which of the following statements relating to acute post-operative renal failure are untrue? A. Intravenous dopamine does not prevent acute renal failure. B. It is more common after emergency surgery. C. Use of excessive amounts of intravenous fluids may lead to falsely normal serum creatinine measurements.

Prepared by Dr: Mohammed Musa Brema Idress – My best wishes Page 613

D. Vasopressor drugs have a strong renoprotective effect E. It is minimized by normalization of haemodynamic status.

ANSWER IS D Key points : Renal injury and acute renal failure: RIFLE Classification. 1. R=Risk (Serum Creatinine x1.5) 2. I=Injury (Serum Creatinine x 2) 3. F=Failure (Serum Creatinine x3) 4. L=Loss (Loss of renal function >4weeks) 5. E=End stage kidney disease Vasopressor use is linked to renal failure as they are a marker of haemodynamic compromise.

1441. Which of the following is most suggestive of malnutrition? A. Hypoalbuminaemia B. BMI of 22 kg/m2 and unintentional weight loss of > 5% over 3-6 months C. BMI of 18.5 kg/m2 D. Reduced skin turgor E. Unintentional weight loss of > 10% over 3- 6 months in a 60 Kg female who is 1.6m tall

ANSWER IS E An unintentional weight loss of >10% in a three to six month period is highly suggestive of malnutrition. This is particularly true of people with a normal/ low BMI. Hypoalbuminaemia is not, in itself, a reliable marker of nutrition.

1442. A 52 year old man is recovering following an elective right hemicolectomy for carcinoma of the caecum. His surgery is uncomplicated, when should oral intake resume? A. Only once bowels have been opened to stool B. Only once the patient has passed flatus C. Between 24 and 48 hours of surgery D. More than 48 hours after surgery E. Within 24 hours of surgery

ANSWER IS E As part of the enhanced recovery principles oral intake in this setting should resume soon after surgery. Administration of liquid and even light diet does not increase the risk of anastomotic leak.

1443. With which of the following blood products is iatrogenic septicaemia with a gram positive organism most likely? A. Cryoprecipitate B. Platelets C. Packed red cells D. Factor VIII concentrate E. Factor IX concentrate

Prepared by Dr: Mohammed Musa Brema Idress – My best wishes Page 614

ANSWER IS B Platelets are stored at room temperature and must be used soon after collection. This places them at increased risk of culturing gram positive organisms. Iatrogenic infection with gram negative organisms is more likely with packed red cells as these are stored at 4 degrees. Infections with blood products of this nature are both rare.

1444. Which of the following is a recognized feature of ketamine when used as an anaesthetic agent? A. Malignant hyperpyrexia B. Adrenal suppression C. Myocardial depression D. Dissociative anaesthesia E. Marked respiratory depression

ANSWER IS D Unlike most anaesthetic agents ketamine does not cause myocardial or marked respiratory depression. It is not associated with the adrenal suppression that may occur with etomidate. It is however, associated with a state of dissociative anaesthesia which patients may find distressing.

1445. A 73 year old man undergoes a right below knee amputation for end stage peripheral vascular disease. He is reviewed in the clinic 8 weeks post operatively and complains of a persistent, burning discomfort over his amputation site stump. On examination his wound has healed and proximal pulses have a biphasic signal on Doppler ultrasound. What is the post appropriate management? A. Commence amitryptyline B. Commence fentanyl patch C. Arrange duplex scan D. Arrange MRI scan of the stump E. Commence carbamazepine

ANSWER IS A This patient has neuropathic pain. Amitryptyline is the treatment of choice. Carbamazepine is mainly used for trigeminal neuralgia.

1446. A homeless 42 year old male had an emergency inguinal hernia repair 24 hours previously. He has a BMI of 15. His electrolytes are normal. What is the best initial feeding regime? A. Give 10 kcal/kg/day initially, oral thiamine 200-300mg/day, vitamin B co strong1 tds and supplements. B. Give 35 kcal/kg/day initially, oral thiamine 200-300mg/day, vitamin B co strong 1 tds and supplements. C. No change to diet needed D. Oral thiamine 200-300mg/day, vitamin B co strong1 tds and supplements. E. Give 35 kcal/kg/day initially

ANSWER IS A: This patient is at high risk of refeeding syndrome.

Prepared by Dr: Mohammed Musa Brema Idress – My best wishes Page 615

1447. A 48 year old man is recovering on the high dependency unit following a long and complex laparotomy. His preoperative medication includes an ACE inhibitor for blood pressure control. For the past two hours he has been oliguric with a urine output of 10ml/hr-1. What the most appropriate immediate course of action? A. Stop the ACE inhibitor B. Administer a fluid challenge C. Start an infusion of nor adrenaline D. Administer intravenous frusemide E. Insert a Swann-Ganz Catheter

ANSWER IS B Hypovolaemia is the most likely cause for oliguria and a fluid challenge is the most appropriate action. Blind administration of inotropes to hypovolaemic patients is unwise, with the possible exception of cardiac patients.

1448. A 45 year old man is involved in a polytrauma and requires a massive transfusion of packed red cells and fresh frozen plasma. Three hours later he develops marked hypoxia and his CVP is noted to be 10mm Hg. A chest x-rays shows bilateral diffuse pulmonary infiltrates. What is the most likely diagnosis? A. Pulmonary embolus B. Myocardial stunning C. Myocardial infarct D. Fluid overload E. Transfusion associated lung injury

ANSWER IS E The risk of transfusion associated lung injury is greatest with plasma components. Transfusion lung injury may occur after infusion of plasma components. Microvascular damage occurs in the lungs leading to diffuse infiltrates on imaging. Mortality is high.

1449. A 52 year old man undergoes a laparotomy for perforated bowel after a colonoscopy. 2 days after surgery the nursing staff report there is pink, serous fluid discharging from the wound. What is the next most appropriate management step? A. IV antibiotics for wound infection B. No further management C. Examine the wound for separation of the rectus fascia D. Insert a drain into the wound E. CT abdomen

ANSWER IS C The seepage of pink serosanguineous fluid through a closed abdominal wound is an early sign of abdominal wound dehiscence with possible evisceration. If this occurs, you should remove one or two sutures in the skin and explore the wound manually, using a sterile glove. If there is separation of the rectus fascia, the patient should be taken to the operating room for primary closure.

Prepared by Dr: Mohammed Musa Brema Idress – My best wishes Page 616

1450. A 63 year old man undergoes a subtotal colectomy and iatrogenic injury to both ureters is sustained. He develops renal failure and his serum potassium is found to be elevated at 6.9 mmol/L. An ECG is performed, what is the most likely finding? A. Increased PR interval B. Prominent U waves C. Narrow QRS complexes D. Peaked T waves E. Low ST segments

ANSWER IS D Peaked T waves are the first and most common finding in hyperkalaemia.

1451. A 56 year old man with chronic schizophrenia undergoes a cholecystectomy. He receives metoclopramide for post operative nausea. Twenty minutes later he becomes agitated and develops marked oculogyric crises and oromandibular dystonia. Which of the following drugs may best alleviate his symptoms? A. Procyclidine B. Lorazepam C. Chlorpromazine D. Haloperidol E. Sulpiride

ANSWER IS A This man has developed an acute dystonic reaction. Administration of further anti dopaminergic drugs will worsen the situation. Procyclidine will help to reverse the event. This is most likely to have occurred because the patient is on long term anti psychotics and has then received metoclopramide.

1452. Which of the following anaesthetic agents has the strongest analgesic effect? Sodium thiopentone A. Ketamine B. Midazolam C. Etomidate D. None of the above

ANSWER IS B Ketamine has a moderate to strong analgesic effect. It may be used for emergency procedures outside the hospital environment to induce anaesthesia for procedures such as emergency amputation.

1453. Which statement is true on enteral feeding? A. A PEG can only be used 12 hours after insertion B. A motility agent is avoided for ITU patients with an Nasogastric tube C. A regime of 24 hours continuous feeding is recommended for ITU patients D. It is associated with more hepatic synthetic disruption than use of TPN E. Enteral feeding is not possible in upper GI dysfunction

Prepared by Dr: Mohammed Musa Brema Idress – My best wishes Page 617

ANSWER IS C Enteral Feeding - Identify patients as malnourished or at risk (see below) - Identify unsafe or inadequate oral intake with functional GI tract Consider for enteral feeding - Gastric feeding unless upper GI dysfunction (then for duodenal or jejunal tube) - Check NG placement using aspiration and pH (check post pyloric tubes with AXR) - Gastric feeding > 4 weeks consider long-term gastrostomy - Consider bolus or continuous feeding into the stomach - ITU patients should have continuous feeding for 16-24h (24h if on insulin) - Consider motility agent in ITU or acute patients for delayed gastric emptying. If this doesn't work then try post pyloric feeding or parenteral feeding. - PEG can be used 4 hours after insertion, but should not be removed until >2 weeks after insertion. Surgical patients due to have major abdominal surgery: if malnourished, unsafe swallow/inadequate oral intake and functional GI tract then consider pre operative enteral feeding.

Patients identified as being malnourished A. BMI < 18.5 kg/m2 B. Unintentional weight loss of > 10% over 3-6/12 C. BMI < 20 kg/m2 and unintentional weight loss of > 5% over 3-6/12

At risk of malnutrition 1. Eaten nothing or little > 5 days, who are likely to eat little for a further 5 days 2. Poor absorptive capacity 3. High nutrient losses 4. High metabolism

1454. Which of the following does not need monitoring during home parenteral nutritional support? A. Folate levels B. Zinc levels C. Vitamin D D. Thyroid function E. Bone densitometry

ANSWER IS D Nutrition Monitoring-NICE guidelines 1. Weight: daily if fluid balance concerns, otherwise weekly reducing to monthly 2. BMI: at start of feeding and then monthly 3. If weight cannot be obtained: monthly mid arm circumference or triceps skin fold thickness 4. Daily electrolytes until levels stable. Then once or twice a week. 5. Weekly glucose, phosphate, magnesium, LFTs, Ca, albumin, FBC, MCV

Prepared by Dr: Mohammed Musa Brema Idress – My best wishes Page 618

6. levels if stable 7. 2-4 weekly Zn, Folate, B12 and Cu levels if stable 8. 3-6 monthly iron and ferritin levels, manganese (if on home parenteral regime) 9. 6 monthly vitamin D 10. Bone densitometry initially on starting home parenteral nutrition then every 2 years

1455. Which of the following is not typically included in total parenteral nutritional solutions? A. Fiber B. Lipid C. Potassium D. Glucose E. Magnesium

ANSWER IS A There is no indication for inclusion of fiber in solutions of TPN, nor would it be safe to do so.

1456. A 28 year old man with poorly controlled Crohns disease is nutritionally compromised. The decision is made to start TPN, via which of the following routes should it be best administered? A. Internal jugular vein via a central venous catheter B. Internal carotid artery C. Cephalic vein via peripheral cannula D. Basilic vein via peripheral cannula E. Common femoral vein via a central venous catheter

ANSWER IS A Since TPN solutions are irritant to veins they are best administered via a central line. The femoral route has a higher incidence of line associated sepsis and is thus best avoided in this setting.

1457. Which of the following statements relating to the use of human albumin solution is false? A. When administered in the perioperative period it does not increase the length of stay in hospital compared with crystalloid solutions B. Concentrated solutions may produce diuresis in patients with liver failure C. It may restore plasma volume in cases of sodium and water overload D. It may be associated with risk of acquiring new variant Creutz field-Jacob disease E. Hepatitis C remains a concern when large volumes are infused

ANSWER IS E Human albumin solution went out of vogue following the Cochrane review in 2004 that showed it increased mortality. This view has been challenged and subsequent

Prepared by Dr: Mohammed Musa Brema Idress – My best wishes Page 619 studies have confirmed it to be safe for use. Viruses are inactivated during the preparation process. However, theoretical risks regarding new variant CJD still exist. Outcomes in the perioperative setting are similar whether colloid, crystalloid or albumin are used.

1458. Which of the anaesthetic agents listed below is associated with hepatotoxicity? A. Halothane B. Sevoflurane C. Propofol D. Ketamine E. Desflurane

ANSWER IS A Halothane is largely of historical interest and that is because of its hepatotoxicity.

1459. A 22 year old fit and well male undergoes an emergency appendicectomy. He is given suxamethonium. An inflamed appendix is removed and the patient is returned to recovery. On arrival in the recovery area; the patient develops a tachycardia of 120 bpm and a temperature of 40 ºC. He has generalized muscular rigidity. What is the most likely diagnosis? A. Acute dystonic reaction B. Malignant hyperthermia C. Pelvic abscess D. Epilepsy E. Serotonin syndrome

ANSWER IS B Anaesthetic agents, such as suxamethonium, can cause malignant hyperthermia in patients with a genetic defect. Acute dystonic reaction normally is associated with antipsychotics (haloperidol) and metoclopramide. These lead to marked extrapyramidal effects.

Serotonin syndrome is associated with the antidepressants selective serotonin reuptake inhibitors (SSRIs) and selective serotonin/norepinephrine reuptake inhibitors (SSNRIs). This causes a syndrome of agitation, tachycardia, hallucinations and hyper-reflexia.

1460. A 45 year old man develops acute respiratory distress syndrome during an attack of severe acute pancreatitis. Which of the following is not a feature of adult respiratory distress syndrome? A. It usually consists of type I respiratory failure. B. Patients typically require high ventillatory pressures. C. A Swann Ganz Catheter would typically have a reading in excess of 18mmHg. D. It may complicate acute pancreatitis. E. It may heal with fibrosis.

Prepared by Dr: Mohammed Musa Brema Idress – My best wishes Page 620

ANSWER IS C Right heart pressure should be normal.

1461. An 87-year-old arteriopathy complains of heavy chest pain post-operatively and he is suspected of having an MI. His blood examination reveals raised urea and creatinine. What type of renal failure is this termed? Select one answer only. A. Iatrogenic B. Intrinsic renal failure C. Pre-renal renal failure D. Post-renal renal failure E. This is unlikely to be renal failure.

ANSWER IS C Renal failure is defined as failure of the kidneys to maintain the correct composition and volume of the body's internal environment. Patients undergoing major surgery are at risk of developing renal failure, particularly in the postoperative period where inadequate fluid rehydration is not uncommon. Cardiogenic shock and hypoperfusion are the likely reasons for his renal failure in this example.

1462. You are asked to see a 69-year old man who has undergone elective knee replacement as his early warning score has risen from 2 to 6. Which of the following is a SIRS criteria? Select one answer only. A. Heart rate >120 bpm B. Respiratory rate > 30 breaths per minute C. Temperature > 380C or < 360C D. Urine output < 30mls/hour E. White cell count > 9,000 or < 2,000 per mm3

ANSWER IS C The systemic inflammatory response is defined as: 1) Temperature > 380C or < 360C 2) Heart rate > 90 bpm 3) RR > 20 breaths per minute 4) WCC > 12,000 or < 4,000 per mm3.

1463. A 26-year old man with known insulin dependent diabetes presents with sepsis, raised blood sugar and ketones. Which of the following acid base disturbances would you expect? Select one answer only. A. pH 7.2 PaCO2 51 HCO3 normal B. pH 7.6 PaCO2 29 HCO3 normal C. pH 7.2 PaCO2 normal HCO3 18 D. pH 7.6 PaCO2 normal HCO3 31 E. pH 7.6 PaCO2 high HCO3 31.

ANSWER IS C He would have a compensated metabolic acidosis, as indicated by C. 1) A = respiratory acidosis

Prepared by Dr: Mohammed Musa Brema Idress – My best wishes Page 621

2) B = respiratory alkalosis 3) D = metabolic alkalosis.

1464. A 68-year-old man who has undergone an AP resection for bowel cancer develops respiratory distress. He is found to have bilateral pulmonary infiltrates on chest x-ray. A diagnosis of ARDS is made. Of the following, which is used as a supportive measure? A. Address nutritional status B. Fluid boluses C. Inhaled nitric oxide D. Ventilatory support with high tidal volumes E. Ward based management.

ANSWER IS A Patients with ARDS should be managed in an ITU setting with appropriate ventilator support with low tidal volumes so as to reduce further lung injury. Nitric oxide has no proven benefit in large studies. Addressing his nutritional status would be of undoubted benefit.

1465. A 77-year-old woman develops a low-grade pyrexia 12 hours after dynamic hip screw fixation for a fractured neck of femur. Examination is unremarkable and her vital signs are otherwise normal. What is the most likely cause? Select one answer only. A. Basal lung atelectasis B. DVT C. PE D. Urine infection E. Wound infection.

ANSWER IS A The most common cause of early low-grade pyrexia after major surgery is basal atelectasis from inadequate ventilation of the lung bases that is often a consequence of insufficient analgesia (i.e. which reduces respiratory excursion and coughing). This often responds to chest physiotherapy (not antibiotics).

Sepsis typically occurs after 24-48 hours, as do DVT/PE, although these must be properly excluded when assessing any postoperative pyrexia.

1466. A 31-year-old IVDU is admitted with suspected necrotizing fasciitis. After surgical debridement, where should he have his continued care? Select one answer only. A. CCU B. ITU C. Medical HDU D. Plastic surgery ward E. Surgical HDU.

Prepared by Dr: Mohammed Musa Brema Idress – My best wishes Page 622

ANSWER IS B Patients with a combination of necrotizing fasciitis and the co morbidies associated with long-term IVDU are likely to require prolonged noradrenaline support and should be cared for in ITU. This question is testing your ability to recognize a potentially very sick patient. Abbreviations: - IVDU - Intravenous drug user - CCU - Coronary Care Unit - ITU - Intensive Therapy Unit - HDU - High Dependency Unit

1467. 58-year-old man has just undergone an AP resection for bowel cancer. You are asked to prescribe post-operative fluids. What one indicator best assesses adequacy of fluid resuscitation? Select one answer only. A. Blood pressure B. Heart rate C. Respiratory rate D. Temperature E. Urine output

ANSWER IS E Urine output is the single best indicator of end organ perfusion. 0.5mls/kg/hour in adults and 1ml/kg/hour in children should be the minimum target. Fluid challenges in the order of 200-250ml bolus can be given if output drifts below this.

1468. A 65-year-old man sustains severe lower limb crush injury with neurovascular damage that required below knee amputation. When performing a below-knee amputation, what is the most important factor in postoperative prosthetic function? Single best answer - select one answer only. A. Preservation of the fibula B. The tibia should be divided approximately 15cm below the knee joint C. The anterior border of the tibia should be beveled D. Nerves should be cut cleanly under tension E. A local anaesthetic infusion into the tibial nerve to aid postoperative pain relief.

ANSWER IS B The mortality rate of 10% in below-knee amputation and 20–40% in above-knee amputation is a reflection of the high incidence of severe cardiovascular disease in these patients.

At least 8 cm of tibia is required to fit below-knee prosthesis, but 15cm is desirable, this is very important to ensure adequate lever arm for prosthetic fitting and the anterior border should be beveled to remove the sharp edge of the tibial crest.

Prepared by Dr: Mohammed Musa Brema Idress – My best wishes Page 623

Nerves are best cut cleanly under tension with a blade to reduce the incidence of neuroma formation. Some surgeons insert one end of an epidural catheter into the sheath of the tibial nerve and bring out the other end through the skin. A local anaesthetic infusion can be run through this catheter to aid postoperative analgesia, and is said to help in the diminution of phantom pain.

1469. A 54-year-old man with COPD undergoes bladder resection for carcinoma. Post-operatively he develops breathing difficulties. Blood gas analysis reveal type 1 respiratory failure. What is the possible cause? Select one answer only. A. Alveolar hypoventilation leading to collapse B. Chest trauma C. Excess morphine D. Myasthenia gravis E. Raised intracranial pressure.

ANSWER IS A The other options are causes of ventilatory failure, whereas type 1 respiratory failure occurs due to failure to oxygenate.

1470. A one-year old child is admitted for cleft palate repair. Shortly after the procedure you are called to see her urgently as there is some bleeding from her surgical site. What should you do? Select one answer only. A. Call for help and manage the child using ABC first principles B. Commence fluid resuscitation with colloids C. Cross match and transfuse 2 units D. Immediately take the child back to theatre E. This is normal. It is not necessary to do anything immediately.

ANSWER IS A All such cases require management according to first principles. It is likely that a suture has come undone. Children can become haemodynamically unstable very quickly and as such the child would most likely need to be returned to theatre as a CEPOD 1 in order to stop the bleeding but this decision should be made by your seniors following assessment

1471. A 54-year-old unkempt man with insulin dependent diabetes is admitted with bilateral infected deep burns to his feet. How should his diabetes be managed? Select one answer only. A. Add in oral hypoglycaemics B. Allow him to self-medicate C. Await diabetic review before deciding D. Insulin as per normal routine E. Sliding scale until his infection improves

ANSWER IS E In the presence of an infected burn, his blood glucose levels will be deranged and he is more prone to systemic infections. In order to stop the cycle from perpetuating, it

Prepared by Dr: Mohammed Musa Brema Idress – My best wishes Page 624 is important to bring his BM’s into control quickly. This is best affected using a sliding scale which can react to changes in blood glucose in a rapid fashion.

1472. You are asked to see a 68-year-old man who has undergone a laparotomy for a perforated appendix 3 days previously. He has deteriorated acutely and has a pulse rate of 140/min, a systolic BP of 60mmHg unresponsive to fluid resuscitation, temperature of 39.5o C and urine output of 5ml/hour. You promptly commence empirical antibiotic therapy for sepsis secondary to a presumed postoperative collection. How would you define his state of sepsis? Select one answer only. A. Bacteraemia B. Hypovolaemic shock C. Sepsis D. Septic shock E. Severe sepsis.

ANSWER IS D Septic shock is defined as sepsis with hypotension despite adequate fluid resuscitation. It is associated with a high mortality rate. He is exhibiting signs of impending end organ failure (cardiovascular and renal).

1473. An 88-year old woman undergoes excision of a skin cancer with local flap to the back. She is anticoagulated and post-operatively experience bleeding to the surgical site. How should she be managed? Select one answer only. A. Apply direct pressure for 30 minutes to the bleeding point B. Apply more dressing gauze to the area C. Nurse on back D. Return to theatre for haemostasis E. Any of the above, they are all equally effective.

ANSWER IS A Whilst all of the answers are acceptable, the quickest way to gain haemostatic control is to apply direct pressure which will contain the majority of venous bleeding points.

1474. A 21-year-old patient undergoes an appendicectomy and is prescribed a PCA for post-operative analgesia. What is the advantage of this? Select one answer only. A. Dose cannot be changed therefore toxicity is not an issue B. Patient determines own analgesic requirements C. Patients have less nausea D. Patients mobilize quicker with a PCA E. There is no need to prescribe multiple analgesics.

ANSWER IS B PCA allow the patient to control how much analgesia is given. Usually started at 1mg morphine with a 5 minute lock out time.

Prepared by Dr: Mohammed Musa Brema Idress – My best wishes Page 625

Background simple analgesics should also be given to reduce the need for breakthrough morphine. As the patient often has a smoother analgesic profile they may well mobilize quicker also but this depends on the patient’s general health. Also being attached to a PCA machine may paradoxically limit mobilization.

1475. A 64-year-old man arrives in Accident & Emergency collapsed with suspected ruptured abdominal aortic aneurysm (AAA). Blood gases show a pH of 7.05, p(CO2)of 3.5 kPa, p(O2) of 12 kPa and bicarbonate of 7mmol/l. What is the diagnosis? A. Metabolic acidosis B. Respiratory acidosis C. Metabolic alkalosis D. Respiratory alkalosis E. Compensated respiratory alkalosis.

ANSWER IS A The pH is low so this is an acidosis, the p (CO2) is reduced not elevated so the cause is not respiratory. The bicarbonate is low, suggesting a metabolic acidosis (ruptured abdominal aortic aneurysm.

1476. A 72-year-old man becomes confused on the ward 6 hours after major abdominal surgery. Blood gases show a pH of 7.24, p(CO2) of 8 kPa, p(O2) of 8 kPa, and bicarbonate of 25mmol/l. What is the diagnosis? A. Metabolic acidosis B. Respiratory acidosis C. Metabolic alkalosis D. Respiratory alkalosis E. Compensated respiratory alkalosis.

ANSWER IS B The pH is low so this is an acidosis. The p(CO2) is raised so this is respiratory. The bicarbonate is normal so there has been no renal compensation (narcosis due to excess administration of opiates.

1477. A 30-year-old man with a three-week history of vomiting has blood gases with a pH of 7.56, p(CO2) of 5.2kPa, p(O2) of 13kPa and bicarbonate of 45 mmol/l. What is the diagnosis? A. Metabolic acidosis B. Respiratory acidosis C. Metabolic alkalosis D. Respiratory alkalosis E. Compensated respiratory alkalosis.

ANSWER IS C The pH is high so this is an alkalosis. The bicarbonate is very high so this is metabolic in origin. (i.e. due to gastrointestinal loss of hydrogen ions from prolonged vomiting).

Prepared by Dr: Mohammed Musa Brema Idress – My best wishes Page 626

1478. A patient developed acute renal failure 15 d post quadruple artery bypass operation with a complicated post-operative course. A central line has been inserted. The patient has been hypotensive for the last 2 h; this is improving with fluid resuscitation. He develops distended jugular venous pressure, a sudden drop in blood pressure, and right-sided chest pain. Choose the most appropriate diagnosis for this case. A. Cardiogenic Shock B. Septic Shock C. Hypovolaemic Shock D. Anaphylactic Shock E. Obstructive Shock

ANSWER IS E Distended neck veins usually indicate obstructive shock; in this case, it is likely to be due to an iatrogenic pneumothorax when the central line was placed. This patient has also been hypovolemic, but the most pressing problem is the pneumothorax.

1479. A 28-year-old motor cycle dare devil attempted to jump over 21 cars. He hit a wall sideways and sustained multiple long bone fractures and a deceleration injury. He is clinically shocked with angiographic evidence of a ruptured aorta. Choose the most appropriate diagnosis for this case. A. Cardiogenic Shock B. Septic Shock C. Hypovolemic Shock D. Anaphylactic Shock E. Obstructive Shock

ANSWER IS C This is a clear case of hypotension; the patient has multiple fractures and a ruptured aorta; the injury could also lead to obstructive shock; but from the details given, hypovolemic shock is by far the most likely diagnosis.

1480. A 55-year-old man has been in a significant crush injury involving a forklift truck. He has fractured his sternum as well as multiple ribs. He has quiet heart sounds and is peripherally poorly perfused with a raised jugular venous pressure. Choose the most appropriate diagnosis for this case. A. Cardiogenic Shock B. Septic Shock C. Hypovolemic Shock D. Anaphylactic Shock E. Obstructive Shock

ANSWER IS E Quiet heart sounds and a raised jugular venous pressure indicate cardiac tamponade. This is an obstructive shock, and the blood needs to be evacuated urgently, either by pericardiocentesis or surgically.

Prepared by Dr: Mohammed Musa Brema Idress – My best wishes Page 627

1481. A 68-year-old woman commences a blood transfusion as she has lost a lot of blood post-operatively following neck dissection for a base-of-tongue carcinoma. She has been adequately resuscitated, but her blood pressure falls suddenly. She becomes tachycardic and short of breath, and feels peripherally warm. Choose the most appropriate diagnosis for this case. A. Cardiogenic Shock B. Septic Shock C. Hypovolemic Shock D. Anaphylactic Shock E. Obstructive Shock

ANSWER IS D There is a correlation between the blood transfusion and fall in blood pressure. She has had an anaphylactic reaction to the blood, the commonest cause being incorrectly labeled blood.

1482. A 24-year-old teacher with Crohns disease has unfortunately developed an enterocutaneous fistula post bowel resection 2 weeks ago. She has been stable on the ward and managing the fistula well. Her blood results are all normal but show an upward trend of urea and creatinine. Choose the most appropriate diagnosis for this case. A. Cardiogenic Shock B. Septic Shock C. Hypovolemic Shock D. Anaphylactic Shock E. Obstructive Shock

ANSWER IS C Hypovolaemic shock can also be secondary to high-output fistulas. It is important for candidates to have a strong basic understanding of the physiological principles of shock. For a fuller discussion, The Oxford Textbook of Medicine (5th ed, 2010, Oxford University Press) is useful and defines circulatory shock as a decreased effectiveness of circulatory blood flow to meet the metabolic demands of the body, describing four basic functional aetiologies: Hypovoaemic shock (haemorrhage, dehydration), Cardiogenic shock (myocardial infarction), Obstructive shock - mechanical obstruction of blood flow (pulmonary embolism) or ventricular filling (cardiac tamponade), and Distributive shock.

1483. A 15-year-old schizophrenic is brought into casualty having jumped from a height. He has a large occipital haematoma and has a Glasgow Coma Score of 14. You are unable to exclude a cervical spine injury. What is the initial management of the airway? A. Jaw thrust B. Chin lift C. Surgical airway D. Guedel airway E. Nasopharyngeal tube

Prepared by Dr: Mohammed Musa Brema Idress – My best wishes Page 628

ANSWER IS A The airway must be opened, and the jaw thrust is a manoeuvre which allows immobilization of the cervical spine, hence avoiding spinal cord damage in a possible fracture.

1484. A cyclist has a base of skull fracture following a road accident. Which of the above is an absolute contraindication? A. Jaw thrust B. Chin lift C. Surgical airway D. Guedel airway E. Nasopharyngeal tube

ANSWER IS E Nasopharyngeal airways are contraindicated in skull fractures, as they could potentially be introduced into the brain. They should be avoided if there is a suspicion of skull fractures.

1485. A 55-year-old man is estimated to have 60% burns, following a chip-pan fire. He has soot around his mouth and has a partial pressure of oxygen of 80 MmHg. A consultant anaesthetist has been unable to intubate him. What should be done next? A. Jaw thrust B. Chin lift C. Surgical airway D. Guedel airway E. Nasopharyngeal tube

ANSWER IS C This patient clearly needs a surgical airway. He is likely to develop Adult Respiratory Distress Syndrome with hypoxia refractory to oxygen therapy, and it is vital to secure a patent airway at an early stage.

1486. In recovery, a patient with Down syndrome and renal failure has been extubated following insertion of a peritoneal dialysis catheter. He is unable to tolerate a guedel airway, but the anaesthetist is still concerned about his airway. What should be done to keep patent airway? A. Jaw thrust B. Chin lift C. Surgical airway D. Guedel airway E. Nasopharyngeal tube

ANSWER IS E The nasopharyngeal airway is a good alternative to the oropharyngeal airway in the occasional cases where the patient does not tolerate it.

Prepared by Dr: Mohammed Musa Brema Idress – My best wishes Page 629

1487. A surgical patient is septic and needs an urgent drainage of an abdominal collection. As she is unstable, she is accompanied by the SHO to the radiology department. When she has been moved on to the couch, she has a respiratory arrest. What should be done next? A. Jaw thrust B. Chin lift C. Surgical airway D. Guedel airway E. Nasopharyngeal tube

ANSWER IS B The chin lift is the immediate step in this case; this is perfectly safe, if cervical spine injury is not suspected (i.e. in which case, jaw thrust is recommended).

1488. Six hours after major surgery, a patient on the ward becomes restless, tachycardic and confused, with blood gases showing a p(O2) of 8 kPa, p(CO2) of 8 kPa and bicarbonate of 28 mmol/l. What is the most likely cause for postoperative respiratory disturbance in this case? A. Segmental collapse B. Fluid overload C. Central depression D. Pulmonary embolus E. Acute respiratory distress syndrome.

ANSWER IS C Carbon dioxide retention in the face of hypoxia, which should stimulate respiration and blow off CO2. This is likely to be due to central depression, usually the combined effect of anaesthesia and opiates.

1489. Six hours after major surgery, a patient on the ward becomes breathless and tachycardic with a raised blood pressure and jugulovenous pressure. The trachea is central. Gases show a p(CO2) of 4 kPa, p(O2) of 9 kPa and normal bicarbonate. What is the most likely cause for postoperative respiratory disturbance in this case? A. Segmental collapse B. Fluid overload C. Central depression D. Pulmonary embolus E. Acute respiratory distress syndrome.

ANSWER IS B A breathless patient with a raised jugulovenous pressure and raised blood pressure with gases showing a mild reduction in gas exchange is likely to be due to fluid overload.

1490. 72 hours after major surgery a patient becomes breathless and tachycardic, with a low blood pressure and high central venous pressure. Gases show a p(CO2)

Prepared by Dr: Mohammed Musa Brema Idress – My best wishes Page 630 of 4 kPa and a p(O2) of 8 kPA with normal bicarbonate. What is the most likely cause for postoperative respiratory disturbance in this case? A. Segmental collapse B. Fluid overload C. Central depression D. Pulmonary embolus E. Acute respiratory distress syndrome.

ANSWER IS D An acutely breathless, tachycardic patient with a low blood pressure but raised jugulovenous pressure has probably had a pulmonary embolus.

1491. A 65-year-old gentleman who is day one post-left hemicolectomy for cancer of the bowel. On examination, he is lying flat in bed, does not have productive sputum and is complaining of abdominal pain which is preventing him from breathing in deeply. He develops apyrexia postoperatively. What is the most likely cause of it in this case? A. Atelectasis B. Chest infection C. Anastomotic leak D. Urinary tract infection E. Pulmonary embolism.

ANSWER IS A Recognized complication following abodominal surgery due to diaphragmatic irritation.

1492. A 70-year-old gentleman who develops a productive, green cough 4 days post-left hemicolectomy for bowel cancer. He is a previous heavy smoker. Examination shows deceased air entry at the right lung base, with a few scattered crepitations. He develops apyrexia postoperatively. What is the most likely cause of it in this case? A. Atelectasis B. Chest infection C. Anastomotic leak D. Urinary tract infection E. Pulmonary embolism

ANSWER IS B Classical symptoms of chest infection.

1493. A 63-year-old gentleman 4 days post-left hemicolectomy for bowel cancer. He complains of vague abdominal pain in the suprapubic region, has passed very little flatus and no stool yet. Bowel sounds are absent. He develops apyrexia postoperatively. What is the most likely cause of it in this case? A. Atelectasis B. Chest infection

Prepared by Dr: Mohammed Musa Brema Idress – My best wishes Page 631

C. Anastomotic leak D. Urinary tract infection E. Pulmonary embolism.

ANSWER IS C Suspect anastomosis leak in any patient post colectomy who is complaining of abdominal pain.

1494. A 75-year-old smoker develops shortness of breath 10 days post-left hemicolectomy for bowel cancer. On examination he has a respiratory rate of 25/min. Postoperative mobility has been slow because of osteoarthritis of his left hip. He develops apyrexia postoperatively. What is the most likely cause of it in this case? A. Atelectasis B. Chest infection C. Anastomotic leak D. Urinary tract infection E. Pulmonary embolism.

ANSWER IS E - Postoperative pyrexia is common. The timing of the pyrexia often gives a clue to its cause: A. 24–72 h: atelectasis, chest infection B. 3–7 days: chest infection, wound infection, urinary tract infection, anastomatic leak C. 7–10 days: deep vein thrombosis, pulmonary embolism.

1495. A 5 year old boy develops a persistent fever following an open appendicectomy for gangrenous appendicitis. On examination, he has erythema of the wound and some abdominal distension. What is the most appropriate course of action? A. Diagnostic laparoscopy B. Open the wound C. Arrange an abdominal ultrasound scan D. Arrange an abdominal CT scan E. Arrange an abdominal X-ray

ANSWER IS C This patient has risk factors for a wound infection which is not in itself an indication for scanning. However, he also had abdominal distension and this, together with the history of distension would generally attract a recommendation for imaging. A USS will show an abdominal wall collection and more importantly any phrenic or pelvic collections. Unlike adult practice, CT scanning is rarely performed in children.

1496. Which of the following substances is commonly used to sterilize endoscopic equipment? A. Aqueous phenol

Prepared by Dr: Mohammed Musa Brema Idress – My best wishes Page 632

B. 80% phenol C. Absolute alcohol D. Glutaraldehyde E. Gamma irradiation

ANSWER IS D Most endoscopic processors use glutaraldehyde which is an effective antimicrobial agent. It is widely recognized as being a cause of hypersensitivity reactions and is thus used in closed systems.

1497. An orthopaedic surgeon makes a modification to an operative approach for total knee arthroplasty. After he has completed 25 cases, he stops and reviews his patient outcomes. He publishes the data. What level of evidence is supplied by this type of data? A. II B. IV C. III D. V E. I

ANSWER IS B Case series that are non randomized and lack concurrent controls at best supply level IV evidence only. To qualify for level I and II evidence a prospective randomized controlled trial with appropriate blinding, control matching and power calculations is needed.

1498. Which of the following is not utilized as a descriptive statistic? A. Mean B. Median C. Mode D. Z score E. Standard deviation

ANSWER IS D The z score is determined using the normal distribution and is not a descriptive statistic.

1499. A 73 year old man undergoes a laparotomy for mesenteric infarction. An extensive small bowel resection is undertaken. A planned re-look laparotomy is scheduled to occur in 24 hours time. What is the most appropriate closure strategy in this situation? A. Mass closure of the abdomen obeying Jenkins rule using 2 PDS B. Application of a Bogota bag C. Mass closure of the abdomen obeying Jenkins rule using 2/0 PDS D. Mass closure of the abdomen obeying Jenkins rule using 2 nylon E. Mass closure of the abdomen obeying Jenkins rule using 2/0 nylon

Prepared by Dr: Mohammed Musa Brema Idress – My best wishes Page 633

ANSWER IS B Where there are definite plans for early re-look surgery and it is desirable to visualize the viscera (as it would be in this case), application of a Bogota bag will provide the best closure option.

1500. Which of the following visceral anastomoses has the lowest risk of anastomotic leak? You may assume that all are constructed in ideal circumstances. A. Stapled ileocolic anastomosis B. Hand sewn anastomosis of the proximal ileum C. Stapled colorectal anastomosis defunctioned with loop ileostomy D. Stapled colorectal anastomosis defunctioned with loop colostomy E. Hand sewn oesophagojejunal anastomosis

ANSWER IS B Rectal and oesophageal surgery have some of the highest rates of anastomotic leakage. Following anterior resection leaks are quoted to occur in up to 10% of cases. Small bowel anastomoses are the most technically forgiving. Factors increasing the risk of anastamotic leakage include previous irradiation, sepsis, malnutrition, poor blood supply and poor technique.

The defunctioning of rectal anastomoses may reduce the clinical impact of anastomotic leak and make it amenable to percutaneous drainage, but does not necessarily reduce the incidence of leaks themselves.

Prepared by Dr: Mohammed Musa Brema Idress – My best wishes Page 634